You are on page 1of 456

e COMPLETE STUDY e COMPLETE PRACTICE e COMPLETE ASSESSMENT

Topic wise T heory in Solved Topic wise & Chapter wise Questions, Tbpic Exercise, Chapter Exercise,
Examples, Figures, Graphs, NCERT & NCERT Exemplar Questions, Challengers &
Try Yourself etc. Exams' Questions etc. 10 Sample Question Papers.

The First Choice ofStudents & Teachers since its First Edition ...
Fu I ly Revised
& Updated
Edition

COMPLETE STUDY I COM PLETE PRACTICE


one
I COMPLETE ASSESSMENT

MATHEMATICS
CBSE Class X

TERM 11

Author
Er. PREM KUMAR
Editor
Nitika Singh Bisla

1�! ARIHANT PRAKASHAN, MEERUT


twtj
>::: arihant
ARIHANT PRAKASHAN, MEERUT

ALL RIGHTS RESERVED


�©PUBLISHER
No part of this publication may be re-produced, stored in a retrieval system or by any means, electronic mechanical,
photocopying, recording, scanning, web or othenvise without the written permission of the publisher.
Arihanl has obtained all the information in this book from the sources believed lo be reliable
and true. However, Arihant or its editors or authors or illustrators don't take any responsibility
for the absolute accuracy of any information published, and the
damages or loss suffered thereupon.

All disputes subject lo Meerul (UP) jurisdiction only.

�ADMINISTRATIVE & PRODUCTION O FFICES


REGD. OFFICE
'Ramchhaya' 4577/1.5, Aganval Road, Darya Ganj, New Delhi -110002
Tele: 011- 47630600, 43518.550; Fax: 011- 23280:�16

HEAD OFFICE
-
Kalindi, TP Nagar, Meerut (UP) 250002
Tel: 0121-240147!>, 2.512970, 4004199; Fax: 0121-2401648

!:h SALES & SUPPORT Off ICES


Agra, Ahmedabad, Bengaluru, Bhubaneswar, Barcilly, Chennai, Delhi, Guwahati, Haldwani,
Hyderabad,Jaipur,Jhansi, Kolkata, Kola, Lucknow, Meerul, Nagpur & Pune

� ISBN: 978-93-8<ll79-4S-6

!:h PRICE:� 295.00

(I) Ltd. (Press Unit)


Typeset by Arihanl DTP Unit at Meerut
Printed & Bound by Arihant Publications

PRODUCTION TEAM
Publishing Managers Keshav Mohan & Amit Verma Inner Designer Ravi Negi
Project Coordinator Apeksha Tyagi Page Layouting Mudit, Harish, Shravan
Cover Designer Shami Mansoori Proof Readers Sona! Aggarwal

Jog on to www.arihantbooks.com or email to info@arihantbooks.com


For further information about the products from Arihanl,
A WORD
WITH THE READERS
Allin-one Mathematics Class I 0th Term I I
has been written keeping in mind the needs of
students studying in Class I 0th CBSE. This book has been made in such a way that students
will be fully guided to prepare for the exam in the most eflcctive manner, securing higher
grades.
The purpose of this book is to equip any CBSE student with a sound knowledge of
Mathematics at Class I 0th Level. It covers
the whole syllabus or class I 0th Mathematics divided
into chapters as per the NCERT Textbook. This book will give you support during the course
as well as guide you on Revision and Preparation for the exam i tse l f. The material is presented
in a Clear & Concise manner and there are questions for you to prac tic e .

KE¥ FEATURES

To make the students understand the chapter completely, each chapter has been divided into
Individual Topics and each such topic has been treated as a separate chapter. Each topic has
detailed theory, supported by solved examples (with The Plan), Black Board Examples, Note,
Table, Flow chart, etc., followed by NCERT Folder having detailed solutions of all the Exercises of
NCERT Textbook.

• Summative Assessment Section of each chapter contains questions in that format in which these
are asked in the examinations. Questions have been divided into Very Short Answer Type
Questions (1 Mark), Short A11s\\•er Type Questions (2 & 3 Marks) and Long Answer Type
Questions (4 Marks). All the questions given here are fully solved. Step Marking has been shown
with individual solutions.

Formative Assessment Sectio� of each chapter contains diflerent types of activities like
Mathematical Activities, Mathematical Quiz, Crossword Puzzles, Projects, etc.


At the end of each chapter, new additional feature Challengers is added, it is a set of some special
questions based on the content of the chapter but can give you a good mind exercise, it will be an
added advantage for the students participating in School Competitions or Olympiads.

• At the end of the book, there are I 0 Sample Questimi Papers to make the students practice for the
examination as weJJ. In addition, they will be abJc Lo learn the Time Management during the
examination.

All-in-One Mathematics for CBSE Class I 0th has all the material r equired for L ear n i n g ,

Understanding, Practice & Assessment and will surely guide the students on the Path to S uccess.

I am highly thankful to ARI HANT PRAKASHAN, MEER UT for giving us such an excellent
opportunity to write this book. The role of Arihant DTP Unit and Proofreading team is
praiseworthy in the making of this book. Suggestions for further improvement of the book will
be welcomed & incorporated in further editions.

In the end, I would like to wish Best of Luck to our readers!

Author
Er. PREM KUMAR
Allinone Mathematics
Class 10
PREVIEW
CHAPTER1

QUADRATIC EQUATIONS
TOPICAL DIVISION
In du.,h.rtn. •r •JI uiMJr qy.a.J1•lk rq·.ui...., vu111U:1•n11J1i�dw11 l1•,.,
�Topic Checklll v.·••..11at..J-..u-11on••rr'a-M•'"•'".,,....,,.• t'QW'....... t..,.Jnnd.ulrWC"ww111J11U1
To make the student understand the chapter
TOPIC 0 Quadratic Equation and Its Solution
C..-t.ro1f.,..i,.. .... ,
•• ��A.rrrl
completely, each chapter has been divided
Quoldratlc Equation
o.t..cl�,..
into Individual Topics and each such topk
.
""""''.. '''"'°''.,.
(>.. ...... ftJ�ftr>
has been treated as a separate chapter. Each "a..1.-.·�·�­
l'J.d'X'f-,l•�•·r.
":dUrir.JaU...l'.t£
111.&Dni� c..,.....1111 rn .,.,..,blir •.•fw"'•·• MLie 1n tnl numhrn mJ • -o
t I !1 · • _. - lhlJ •O_ • 1 · •I• VJJ1 • n. \, •.fl• •.,. •U, �.a - !\ .. n He
topic has detailed theory, supported by
lrJ....a'll"tCrrr"Pl'fl'QI'•
�.d•:w'IJ•,,,._,.ar"fl-itv1

�Ufhnl11.Af•on•.
illustrations, tables, flow charts etc.
�Jll'<.lr.l'J"T'l.-nJ
r."'q..a•

�"' Bloch 80G1d &ampl•

the r IS ol tho qu1dr1tic equ1t1on


'
-!t• • 6•0by f•ctor1s.ation method. U•••• P• Q• - •.ow.ir-J •C.
'ind
1
,, .. -�•'ftJQ .. J
11'-91(J.am)#W:�lf'l'1�b"'
�J W't4tr'eC'Crf\!.w10(.Jl..-'•Ol•c•D&l(P•UJ#'(J
BLACK BOARD EXAMPLE - �I • (I • Q •...,I
S'131n2h ...... �ab�C
SJlpl

('.....,, QI� It< tQA II •'


'.t,1 • I\ - e •I '] JJ• o 6
f'1W1f«JCJ••

i"I �!M'ICl.8J ........,., I �' • (1


-I •I .�
O"llc•�no,....,,... ,bf,r. .. a•w1 11·1• 1111 - J;
• 11•-:'t · l•
•·•IJ .. '1111..,c-r.
SJ.p .. f�uef'IUdt"l'lomorci'ftjEhov.... r"Ja
) • 0 ) I• J
Herc we have given a Question with detailed solution,
2 l.'...LAfl"1 • .,.,lcrdl(Jll «.ftoP'W"" 'wt:ir1 l1Yn l&.tl I·;"• 0 .. I •:!Of I -

2..:ilmah''lllM'f'llllC<bd•Q'Y"'i
clearly indicating all the steps involved in the solulion
lndf60�PWV'IQIUC1'lnal/1•Q·b
�,,... .... 11'1.n
l')AT31CGQu.'J'O"t
along with their execution. This feature will help students
�. 1-0� (I
to understand the problem solving In an 'air.ttnolOaro:'.:wdl

organised manner.

NCERT FOLDER 1.2


�:r-=-;;-���.=-�..2.™Lr:'iiiliiJIW -
1 rand lhr ronh, ol ftu- lo1Juw1n11 ,, adr1111l1t 111) G1Yf"n"f\MOon11 �1
1 •
• ,, •• o
.!..- .... - .h -b• fl

(IJ •1 -J• -10• (Ill •I I: 4 •I-II• -1: ,nJ • - J •II


rqu11Uont by lac1rtnwl1u
l\1 - li .. O

(Ill) ,/j 1' •711 • ,,.7 •O 2'' ·I •


I
..
� 1 h •.?J-.'(· ·�)·0
B ,.. • .ZH!I' _,,.o
MI w' -201 • t•
llwl

• •: •D ot

TOPIC 0 Solution of a Quadratic Equation by


SoL (r) (;In uon n •'-.h -1
1•-' •O
•O j

Completing the Square and Nature of Roots J·: -'\. c:i. -rn anJ _.,.: - -'I Hmn-, rtw n-.i1sal1hr
� -'•-'> • :c, _.,,.o z..· • • -b •O -! ..I;.
In dv We 1 lnm1 t.n wd ql.!.ldr.ar" 1.111NU1 Method of Completing the Square fin) G1 ....n
f:
• _, • • • • �-o 11111
111 1 ..'J•' • '"• •' .. : .. o

rxroriunon mnb11 J.. Th" mnbud (i "'"• qu.adt.11 .. . - J


1nl"1T 1hc ution of qu.admt<: cqu.nWn of dtt' form
.
.. _, Of h!•IO
rqu.adon U 1Jui1cJ1 IOI� .di qu.adr.11ic ftlu.&I. n. Naw. th•t•0.11 .. 0 complctin 1M-qu.att.•T W HmU", dw o(lhr wuoa
- .!•nJ'\.
L--' '•.: • 7 J
To
a1
"'"I' •ill karn rwo mott mahudt 'I '° ir&J qua.ImIL
••h!•O

inlH ·
,_,1.-•\x •:

s-1-.b-IO•Oarr
ftlu.&lion. lhar lft cumr�in1 1tw aquirr rmtbud 1nJ
::·i�":.!i;t/.::7." in
s r..'!1 •')•.., !I.., �.. • \)•0

-""f I fum1 fi( n


'

qwdrirk (..,mui.. Roth ol' thr,. olC'I .W "" h< u 10


ooh� &II qwdrat' rqu.it' n. Also, "" •ill 11uJy &bt;u1 tlot
" rJ

aiumce u wdl u NhlH' o( n. i.e. a'+ +t • O." 11 0


To find rhr tolu1 ncf a q-.lmic r11u.ition
th< tqu.itt, ... uw thr l'ol'-ing nwthnd.
cur rktina 1 "f II Mak oh< Iii imr oh' unity by
. .-h.nk n1w.1iun b1 1ti c aldTw:Urn cl•'
Jmdii.en1. tht

lil'itiono<uniry)
NCERT FOLDER
To make the students fully familiar with the
NCERT Textbook (the most important
books for CBSE examinations), solutions of

�I Very Short Answer Type Questions [1 M.r• uch ]


all the Exercises ofNCERT Textbook have
been provided with each chapter.
I. II b • 0, < < 0, ii II
3. °""9 th• WhOH
11 + b1 • < • 0 am
true that lhe tools ol

ent1 aro rallo1141 but both ol 111 ruot1


ul•t • quadroUr equaUon
numoncally equal and
roelll

NCERT l!lnnplar
oppoaolo In srgnf Jwllly i·our on1wor. aro lrrallonall Ju111ly 111ur on.wet
NCERTEumplu

2. II tho disrrlmlnant ol tho pqualion


611 -bll +2 .0111,thrnllndthevalueolb 4. whothrr
CllSHZOIZ Uur or fal
C:hock tho lollow1ng 1t•l•m•nt 11
. Ju\Uly your an1wH.

ruin 1."
"Ev•ry quadraur equaUon haa aU1w111 one

� I Short Answer Type I Questions U M.ubuch]


SUMMATIVE ASSESSMENT
J 3. Which ul t � lnlluwlnlJ 11 n t a qu•tlrallr
14. 11 -c./j • Q1 ,/j •O
rquallon I

NCEJn' l!umplar CBSBZOIS
Sulvclorx:
Summative Assessment have questions in that format in

,/6.-;-7 - C21 - 7) •O
10 hlr • �jl' • J11 - 51
15.
!IO !r1 •hi' •1' •l •h1
which these are asked in the examinations, i.e .. Multiple
CCl!ZOll Choice Questions ( 1 Mark), Fill in the Blanks (I Mark), True
Solvclor1:

or False ( 1 Mark), Very Short Answer Type Questions


� I Short Answer Type II Questions [JMubHch]
( l Mark), Short Answer Type Questions (2 & 3 Marks),
29. the root• ol the lnU ..1RIJ 32. 1hr 15 Long Answer Type Questions (4 Mark ) & Application
Oriented Type Questions. All the questions are fully sol ed.
Find Find ol the loUo..ing

11 -3./S I 10. 0 I -
quadrollr equollon cquollon.

. . .°3; 0, 2
-2
I •
CB51!2011

I I
30.
The explanations given here teach the students, how to write
· 4'1Jr2 51 - 2,J • 0 33. the answer in the examination to get full m ark s. Students ,an
Solve lho loUo..ing qu1drot1r "'1U•t1on
+

CCE2013, 12
·: - I ::, -I
lonr

1 0,
use these questions for practice •llld assess their
Solwlor

• ; I •

31.
:

numben, 1um ol who1r squall's II IJO. understanding & re,all of the 'haptcr.
find two ron!l'Cllllv• odd nolurol

CCE2013
for CBSE Class 10th Term II Examination is a complete book
which can giveyou all Study, Practice & Assessment. It is hoped long Answer Type Questions
that this book will reinforce and extendyour ideas about 48. Sol... • •

• I. l. l
h -IJ(w . I I• ll h JI J
• .1
the subject andfinally will placeyou in the ranks of toppers. · -
CCEJOI•
47,. lnr a
� .. . .
'

•• - - l. - l.. - 4
_.. •I
CCEJOI•
48. lnr1
' 4 • a 6 I �
7 l
• . • • 1
-
• - � •

SUMMARY
rtt'9i< , � t..:w" r1 ••' • C• • c • 0 __....,. . • • .-.:11,, o .-.Jc .,u ,_. �b9f·t
erw:�•·' • b• •' ""O • •Ot.f

• c.• •O � t o1n"·'•l'he """9"' �''< '•'orCG.tCtolt .,.,._ '"''°'•


fktor'\t11:111 • O•
...ch f..cld' IO,_DIDIJl'lpl'l-W.
b9 titJhl'Wd CCm'T'll'H�� ,..... tiQL.M• �
Ult t!' t"
sr... C(o,
£111 • tr'f • C • 0. • • 0 (.911\
.oura.t..or•cto�r,..lJ...-1t<.•ut�-U.'
wx... -:nr��.�- en••' . rn• . ". o •

b I \bt

CHAPTER EXERCISE
2•
\\
r!
At the end of the chapter, these unsolved i
CHAPTER EXERCISE
:

\'\ """'- ------- ---- -


questions are for practice and

t_j
-

N ;
---
assessment of students. By practicing
lll Vay Short Answer Type Questions (1 M11• uch)

Jj
these questions, students can assess their
preparation level of the chapter. t. r.-:••"-••.alurnlr.1 C"'4'...' f'A1 II• • I ... on•rnfn1 "IV•rr•
%. r�lhc...-.•lkrqu.111 .. -JI' • I •O,flndtltc•ulw
-
-11 I •

-"Ji
!

3. ,. ....... dllCI l.n.&nl 1tf qu..adr•Uc t'�u.11 •' - ... . I .. 0 CCB:lllll:ll


4 • .,._., • b • t •O h.ltrq!l.ll nul\. Ihm h I W.llllC' ult CCEZDl3

lll Short Answer Type I Questions (lM11k1.,ch)

C'.Cl!:ZOl3
6. Dt1 rr�l nr 1ht uln· of r f, • "hl�h 1
7. Wlwll>

FORMATIVE ASSESSMENT M. H1hrd1 I uh of.J


a:llZDll
J11 Ji, J • o by ut.ln• •hr h.11k f,1mula:
itnf1hrrii.1Dt1of t
- -
•' -'7• -• •O.

Activity .
TopkC-.1 °"°*� .. ...... ,,,.. Ml..... ._,_.,,._
Olljldl"9 Tolnd_.......... ..., ... -dllw liq'�.... ....., ......
�..-flllWll -_,•e>-O·Cl­
•.b >0

51111�,,,...� .,... dw'll II. .,.


nm.�11-
l'N-llrMMlldge r • ze • • , •• Ill' ..., ,f'� 1... ...,. . , --...-
...., ..._ ,._ _.,_,,--r-
__ - �_:.;,
·rr ·�•--·
-, ·I
·
-"'�""-

�� �;!,�;.��:���.��rtr:.���
....w..� ...- - -·
,.. Kt ... nauUt.
Method
1. rm ....... __ ; ... -20.0
·6 .•
2.- .... -20-0 . .... 20
• 2un
.... _byodd'"'l
of school examination. To make the students
HenaP.IN roc:t ef r' •l•- 20 • 0,i 2
),a.!UJC

u .. -...r... -- Conduslon �
�I
ready for Formative Section, this section of the
n...,.,... root,11htdl"""''<•t.twflinU'lflitdl"I 1;
. ,.,, ·20 •!•>' book contains different types of Mathematical

[-.- r; ·J'. G ·BJ'.•']


......
"'""'"'..

.,,_,.,, Oral Quntlons


Activities, Mathematical Quiz, Crossword
'
,,, •>' • lO . )6 ·6' l.Wl>4t .. ltlt q ...
, ....,
·

Puzzles, Projects etc.
4. Dr••ltl ohido6cm. 2.1u'-S1 '
J.- .
.

... -p.
"-

4. !>O.i/louldbe_.,..,l:AJ!li<i.dWI
.,,. �,_,.., � , ,.. • l• • 11acomp.i.
OWsqi.urr'
s. Wlrydo• 'o'<""""°"wl'°"
lole "ngq..011.:
•bl•C•Ot-1mrlhodal�-,

CHALLENGERS
I. F,,,,.,.__....,,""'' �....,
· · ·• 0- � -
.
2. Any lr"" • """ IO l>eOlangor< IDh� II "" P<><'
1�CU"Oe Ol <r
..,._...."'
.,, .

3. 9- • I • 0I • 211tdl < 1
(I Mark)
Know your Examination Pattern
Very Short Answer Type Questions 79-80
Al ne Short Answer Type Questions (2 Marks)
r

Face to Face with ii-iii 80-82


Short Answer Type Questions (3 Marks) 82-85
(4 Marks)
Secrets of a Student Success iv
Long Answer Type Questions 85-88
Effective Ways to Study v-vi FORMATIVE Assessment 89-90
Summary 91
How to Attempt the Examination Paper? vii
Chapter Exercise 92-93

1. Quadratic Equations 1-46 Challengers 94

Quadratic Equation and Its Solution


1. 1
1-3

3. Circles 95-127
NCERT Folder 3-4
Solution of a Quadratic Equation by Factorisation
NCERT Folder 1.2
5-6
7-8 Tangent to a Circle 96-98

Solution of a Quadratic Equation by Completing NCERT Folder 3.1 98-99

the Square and Nature of Roots


NCERT Folder 1 .3
9-12 Number of Tangents from a Point on a
Circle 101-103
13-15
NCERT Folder 1.4 16 NCERT Folder .L� 103-107

Word Problems and Its Solution SUMMATIVE Assessment 109-121


17-21
NCERT Folder 1.1 Very Short Answer Type Questions (I Mark) 113-114

Short Answer Type Questions (2 Marks)


22
114-116
NCERT Folder 1.2
(3 Marks)
22-24
Short Answer Type Questions 117-119

Long Answer Type Questions (4 Marks)


NCERT Folder 1.3 24-26
119-12 0
NCERT Folder 1.4 26-27
FORMATIVE Assessment 122-123
SUMMATIVE Assessment 28-40
Very Short Answer Type Questions ( 1 Mark)
124
Summary
31-32
Short Answer Type Questions (2 Marks)
Chapter Exercise 124-126
32-33
Short Answer Type Questions (3 Marks)
127
Challengers
Long Answer Type Questions (4 Marks)
34-36

37-40
FORMATIVE Assessment
Summary
41-42 4. Constructions 128-159
43
Chapter Exercise Division of a Line Segment and
43-45 128-134
Construction of a Triangle
NCERT Folder -t. l
Challengers 46 134-137

Construction of Tangents to a Cirlce 139-142


2. Arithmetic Progressions (AP) 47-94 NCERT Folder -i.2 143-145

SUMMATIVE Assessment 14 7-156


Very Short Answer Type Questions ( 1 Mark)
Introduction to Arithmetic Progression 48-49
NCERT Folder 2.1 50-53
150

nth term of an AP 54-56 Short Answer Type Questions (2 Marks) 150-152

NCERT Folder 2.2 56-61 Short Answer Type Questions (3 Marks) 152-153

Sum of First n Terms of an AP 62-65 Long Answer Type Questions (4 Marks) 154 -156
NCERT Folder 2.3 65-71 FORMATIVE Assessment 157
NCERT Folder 2.4 72-74 Chapter Exercise 158-159
SUMMATIVE Assessment 76-88
5. Some Applications of 160-197 Summary 282
Trigonometry Chapter Exercise 282-284
285
NCERT Folder s . 1
Challengers
165-170

SUMMATIVE Assessment 171-191

Very Short Answer Type Questions ( 1 Mark) 176-179 8. Areas Related to Circles 286-��30
Short Answer Type Questions (2 Marks) 179-180 of Arca and Perimeter of a Circle 286-289
Short Answer Type Questions (3 Marks)
Review of Concepts
180-181 NCERT Folder 8.1 289-291
Long Answer Type Questions (4 Marks) 187-191 Areas of Sector and Seg ment of a Circle 282-284
FORMATIVE Assessment 192-193 NCERT Folder 8.2 284-298
Summary 193 Areas of Combination of Plane Figures 299-301
Chapter Exercise NCERT Folder 8.3 301-307
194 -196
Challengers SUMMATIVE Assessment 309-323
Very Short Answer Type Questions (I Mark)
197
314-315
Short Answer Type Que stion s (2 Marks) 315-317
6. Probability 198-235 Short Answer Type Questions (3 Marks)
Long Answer Type Questions (4 Marks)
317-321
NCERT Folder 6.1
NCERT Folder 6.2 (opt'on al
207-212
213-214
321-323
FORMATIVE Assessment
SUMMATIVE Assessment 215-227
324-325
Very Short Answer Type Questions ( 1 Mark)
Summary
326
220-221
Short Answer Type Questions (2 Marks)
Chapter Exercise
221-222 327-329
Short Answer Type Questions (3 Marks)
Challengers
223-225 330
Long Answer Type Questions (4 Marks) 225-227
FORMATIVE Assessment 228- 229 9. Surface Areas and Volu1nes 331-379
Summary 230
Surface Area and Volume of Combination of Solids
9.1
333-336
Chapter Exercise 231-234 NCERT Folder
NCERT Folder 9.2
337-339
Challengers 235 339-342
Conversion of Solid from One Shape to Another 343-345
NCERT Folder 9 . 3 345-348
7. Coordinate Geometry 236-285 Frustum of a Cone
236-238 Folder 9.4
349-351
NCERT
239 Folder 9.5 (Optional)*
Review of Concepts of Coordinate Geometry 352-35 3
Folder NCERT
353-356
24 0-2 43 SUMMA TIVE A s sessment
24 - 5
2
Distance Formula
Very Short Answer Type Questions (I Mark)
351-362
NCERT Folder 7.1 3 4

246-250 (2 Marks)
362-36 3

251-254
Section Formula and Area of a Triangle Short Answer Type Questions
NCERT Folder 7.2 (3 Marks)
363-36 6
255-258
Short Answer Type Questions
(4 Marks)
366-370
258-259
Area of a Triangle Long Answer Type Questions
NCERT Folder 7.3 370-37i
260-264
FORMATIVE Assessment
NCERT Folder 7.4 373-374

265-279
Summary
375
269-271 6
37 -378
SUMMATIVE Assessment Chapter Exercise
(1 Mark)
272-274
Very Short Answer Type Questions
(2 Marks) 379
Challengers
Short Answer Type Questions
274-277
Solution to Challengers 383-399
Short Answer Type Questions (3 Marks)
Long Answer Type Questions (4 Marks) 277-279
FORMATIVE Assessment 280-281 Sample Question Papers ( J-10) 403-444
KNOW YOUR
EXAMINATION PATTERN
CONTINUOUS AND COMPREHENSIVE
EVALUATION (CCE)
Continuous and Comprehensive Evaluation (CCE) refers to a system of school based
evaluation or students that covers all aspects of a student development. It is a
developmental process of a child which emphasises on two fold objectives . These objectives
are continuity in evaluation on the one hand and assessment of broad based learning and
behaviourial outcomes on the other.

The first term Continuous means regularity of assessment, diagnosis of learning gaps, use
of corrective measures and leedback of evidence to teachers and students for their self
evaluation.

Second term Comprehensive means that the scheme attempts to cover both the scholastic
and the co-scholastic aspects of students' growth and development.

SPLIT OF SYLLABI
rfo facilitate smooth implementation of C CE, CBSE has provided split of syllabi for all the
subjects termwise. This curriculum has further sub-divided the syllabi reflecting the name
and number of units/ lessons covered for FA I , FA 2, SA I , FA 3, FA 4 and SA 2.

Though the weightage for each of the formative assessments is I 0%, the number of
units/lessons may vary for each of these depending on the time available in the academic
calendar.

Evaluation of Academic Subjects in Class 10th


Six assessments are proposed by CBSE. These are valid for most ofthe schools.
- - ---- - - - ----- -

Assessment % of weightage Morith Term

w
- -·

Formative Assessment I (FA I) 10% April-May


Formative Assessment 2 (FA2) 10% July-August FA 1 + FA 2 = 20%

Summative Assessment l (SA I) 30% September SA 1 = 30%


- ---- - ---

[UJ
Formative Assessment 3 (FA3) 10% O ctober-November
Formative Assessment 4 (FA4) 10% January-February FA 3 + FA 4 = 20%
Summative Assessment 2 (SA2) 30% March SA 2 = 30%

Total Formative Assessment = FA I + FA 2 + FA 3 + FA 4 = 40%


Total Summative Assessment = SA I + SA 2 = 60%
FACE TO FACE
WITH AllHUJne
,,.

All-in-One for Mathematics Class READING AND BLACKBOARD


1 Oth Term II has been written by an
LEARNING EXAMPLE
experienced examiner specially for
Each chapter starts with the Here we have given a question
students studying in Class 10th with
introduction of the chapter, with detailed solution clearly
CBSE Curriculum. It provides all the
stating what is to be studied in indicating all the steps involved
explanations and guidance they
the chapter and major in the solution along with their
need to study efficiently and succeed
highlights of the chapter. execution. Through this
in the exam. This book strictly
Further, to make it easy for feature students will be able to
follows the CCE Pattern of CBSE.
the students, each chapter has understand the problem
The purpose of this book is to aid
been divided into topics and solving in an organised
any CBSE student to achieve the best
each such topic has been dealt manner.
possible grade in the exam. This
like a separate chapter.
book will give the student support Theory has been followed by
Each topic has been discussed
during the course as well as advise the practice section. It has
in a very detailed manner
on revision and preparation for the Solved NCERT Exercises
covering all its terms, their
exam itself concerning the topic. Solutions
types, concerned formulae
All-in-One Mathematics is the only given to the NCERT questions
along with Solved Exampies
book which covers all the purposes of are very detailed, so that
having very detailed solutions,
a student required for the school students can understand
ensuring the complete
curriculum, i.e. Reading, Learning, completely how to apply their
absorption of the concept to
Practice and Assessment. knowledge for solving the
the students. NCERT book problem. Wherever required,
being the bible for CBSE
curriculum , has been given a
additional questions have also
been given along with NCERT
very special treatment. All of questions to make the practice
its key points, highlighted complete.
matter, etc. , have been
covered comprehensively. All Note Here, we have given
the theorems of the chapter some special matter related to a

have been given with their particular topic. This

proofs. Some special elements information will be highly

have been added in the useful in understanding the

theory. concept and in problem


solving.
SUMMATIVE ASSESSMENT
Summative Assessment is carried out at the end of a course oflcarning. It measures or 'sums up'
how much a st udent has learned from the course. I t carries 90 Marks.
After the NCERT Section, there is Summativc Assessment having questions in that format in
which these arc asked in the examinations, i.e. Very Short Answer Type Questions (I Mark), Short
Answer Type Questio ns (2 & 3 Marks) & Lo ng Answer Type Qu e stion s (4 Marks). All the questions
are fully solved with clear indication of Step Marking. The explanations given teach the students
how to write the expla natio n in the examinations to get full marks.

FORMATIVE ASSESSMENT
Formative Assessment is a tool used by the teacher to continuousl y monitor student progress in a
no n threatening suppo rtive environment.
- ,

1··-· --

1 DESCRIPTION OF FORMATIVE ASSESSMENT TASKS


Formative Assessment includes at least one ofthe following in each formative assessment duration
./ Oral Assessment ./ Mathematical Games
./Quiz ./ Math Magazine
./ Discussion ./ Self Composed MathPoem
./ Presentation by Students
./ Making ofCharts/Material
./ RolePlay

To make tlie students ready for Formative Assessment, this section of All-in-One for Mathematics contains different types of
Mathematical Activities, Mathematical Quiz, CrosswordPuzzles, Projects, etc.

ASSESSMENT
All-in-One Mathematics provides two way assessment for the students; one
is chapterwise assessment and other assessment is focused towards full
length examination.

Chapterwise Assessment Along with learning of the subject, All-in-One


Books ensure the Assessment of the students in the real sense. Along with
the fully solved exerc�ses there are sufficient number of Practice Questions
(Unsolved) under the head 'Self Assessment Sheet'. By attempting these From the above description, it can
questions, students can assess their knowledge/ understanding of be concluded that All-in-One
individual chapters. If they find themselves in difficulty while solving a
Mathematics for CBSE Class 10th
particular question, they can refer to the theory portion/ solved exercise in
the book. has all the material required for
Real Practice for the Examination For the major time of the year, students both Summative Assessments &
study each subject in Chapterwise manner, but examination papers assess Formative Assessments and will
the students on the whole syllabus in a complete manner. So along with the
surely guide the students on the
Chapterwise study, practice as per the Examination Pattern is also required.
path to Success.
To serve the above purpose,

All-in-One books have 10 Sample Question Papers, strictly prepared as


CBSE Blue Print & Examination Pattern.
SUCCESS
SECRETS FOR A STUDENT

Discover your Hidden Potential Each of us has the Improve your Memory There is nothing in the world which
capacity to become someone great which results in us doing we call good memory or bad memory. The difference in
great things. You are your best judge and critic too. The more retention power is between a trained and untrained memory.
you know about yourself, the better your performance will be. Better memory is because of better observation power, evaluation
By discovering your hidden potential, you will develop and training. Memory cannot be improved by medicines. Use
interest in your studies. your memory regularly and you can depend on it. Always
remember, the more information present in your brain, the
Set your Goals Unless we know where to go, we cannot
better the chances of good memory.
reach there. The moment we fix a goal, we start thinking
about it. In this process our brain releases a success Learn Time Management We all make the mistake of taking
mechanism which helps us in achieving the goal. time lightly and then repenting later. Once you understand the
importance of time and how to use it successfully, you will
Always Be Motivated Motivation is a strong driving force.
become an expert in time management. The basic funda of time
To motivate means to provide one with a motive to achieve
management is to devote maximum time to the important tasks,
something. Success is the greatest motivator; each success
as they may turn into urgent tasks tomorrow if not dealt in time.
takes us a step further to yet another success.
Be a Voracious Reader There is no better way of acquiring
Improve Concentration Concentration means to devote
knowledge than through books, if you don't want to be left
total attention to a subject. Just think of the subject which you
behind in life, you must read, read and read. The best thinkers
want to concentrate on and banish all other things out of
and idea generators are voracious readers. When you read, you
your mind. When you pick up the· next subject, detach
are feeding your brain, just like the food to enjoy good health.
yourself from the previous subject.
Reading good literature will help you share thoughts with some
Getting Started There is always an initial resistance which great people. Reading good books is essential for an overall
makes us want to give up and we find a hundred reasons for knowledge base.
not starting. Students who do this get stuck in grooves. So get Be Proficient in Writing The best way to attain expertise is to
started with your preparations as early as possible; don't wait
be proficient in writing. The best way of studying is by reading
for examination time.
and then writing notes. Usually all of us have to face written
A vo id Distraction Learn to control your thoughts. They examinations, so it is also a reason to be good in writing. Writing
are the biggest source of distraction. A mind full of thoughts creates a reference, you can always go back and refer to it if you
is like an ocean with high waves. No impact wilJ be created have put down all that you have learnt.
whatever you throw into it. However, a stone thrown in a Eat Smart Food There are certain foods which make you smart,
small pond will make ripples. A quiet & calm place is
so we can call them smart foods. These foods are basically
conducive to study.
nutritious foods which help you think better. Include all fruits
Be Self-disciplined Success of a person depends upon and vegetables in your diet which can be eaten raw. Have fruits
discipline. Self discipline begins with each of us on an empty stomach. Consume the nut family-almonds,
understanding ourselves well. We all must be aware of our cashews, pistachios, walnuts, etc. Eat protein rich food like
strengths & weaknesses. Then only we can lay greater germinated seeds, as these give enormous nutrition and help you
emphasis on our strengths and can take steps to transform stay alert. Drink at least! 0-12 glasses of water daily. If you don't
our weaknesses into strengths. drink enough water, you will feel lazy and sleepy.
EFFECTIVE WAYS TO
ST
'foday al111nsl 1'1'1'1)'0111' should enable you to recall the e ntir e
stud it's thoroughly Im I Adopt a Study suqject. Good notes are simple, well

s111 jJrisi11gly ft'W artually organised and easy to understand. On

study t'jfativt'/y a11rl


Schedule returning home from school, the ideal

111axi111i.\'f' thf'ir rl1r111rt's r�f' situation will be to revise what has been
Every student studies many sul�jects in the
s11t u1ss . Studying 1l'ithout r1 taught from the textbook and create notes
school but nobody leaches how lo study
jJ/rm is /illr1 rli111bi11g a incorporating the matter taught in the
them cflcctivcly. For cflCctivc learn ing, we
a
11101111/ain ·without /mil class.

11w jJ. } iJll 111ay be ·ut' ,�,.


should know how lo achieve more in less
time and with lesser effort. It can he done
i111t'/lig1111I or r111 m1t1mg1' by adopting a study schedule. When we
st11rlt111 t, hut al whall'1't'1' make a schedule, we mentally prepare Mind Mapping
slagt? you 11rr>, you 1111NI to ourselves to meet iL. It removes the
study rjji:rtivt'/y. Study ing
Mind mapping involves visual images and
uncertairuy as to what we need to or not to
is a rontinuous jJroass
other graphic details tomake a deeper
do. Study schedule should be flexible to
' " ro IIgit 0 II t t" l' I f'1" Ill jJaiod.
impression. It is based onthe fact that the
absorb some variations. It should be
A II t lw arti·uiI ies, It'sso m,
brain often recalls information easily in the
designed in such a way that it leaves
form of pictures, symbols, sounds, shapes
and fee lings. Mind mapping helps to
hmneworll, j1rartiw l ;.oorli sometime in, between for relaxation for the
anrl asst'ssmenls a rt' major student. Personal preferences must be kept
prepare notes from the textbooks. It is a
j{LCtors in detnmining your in mind while designing the study
whole brain approach through which an
final examination gmrfr. schedule. You should also resolve to follow
emir� subject can be displayed on one
II is our lwjJf tlwl this
the schedule religiously; only then
page.
hr1/jJ
maximum benefit can be derived from it.
artirfr will to guirlf
you on your way to rfrvelojJ Study schedule should include provisions

your own STUDY PLAN. for homework, revision and sometime for
How to Illake Mind
making notes. You should also have
separate study schedules for school days Maps
and holidays.
Make mind maps on your own. Write the
main topic in the middle and enclose it
Make'Notes within a particular shape you like.
branch extending out from the centre for
Add a

Notes making is the most important skill each important point, write a keyword on
leading to greater success in exams, but it is each branch and build it up by adding
not enou gh to j ust make notes. Notes must details. You can also use abbreYiations. but
be clear and understandable, th ey should be sure to be familiar with them so tha t vou

be written in s uc h a way that a glance are able to identi fy them instantl\'.


. .
Look, Melllorise, Write and Check Tech nique
Study a topic for sometime and then close the book and w rite dow n w h at you re m e m b e r. N o w c h eck you r work w i t h t h e
book. Repeat this practice until you have most of the work correct a n d t h e n m m·e on t h e a n o t h e r t o p i c .

Repeat this technique o n the same topic, a t least. once b u t pre fe ra b ly tw ice s o o n a fterward s , t h a t i s e i t h e r l a t e r o n t h e
same day or the next day. Sooner or later, th is tec h n ique will g r e a t ly i n crease you r lun g t
t e r m re e n t i on ul' the top ic.

Practice for Perfection


Practice is the key to achieve perfection in any field and school studies also re qui re constant p ractice. Practice re fe rs t o
achieving excellence in a particular field. Prop e r planning and practice pro m o t e s bet ter p e r f o r m a n ce a n d re p e a t e d
practice lessens one's deficiencies.

Most of the students take school practice tests lightly; t h ey t h i n k that these a rc j ust class tests w h ich do n o t even co u n t i n
the report cards. This is a wrong perception , as prac tic e tests make a di ffe re n ce T h ey h e l p you assess yo u r ab i l i t ies a n d
_
.

the hardwork put in by you.

Revision Techniqu es
Well in advance of the examination, prepare a revision time table for all your subjects. I deally you o u g h t to go t h ro u gh
the complete course twice. Keep a checklist of the chapters studied as it is encouragi n g to observe t h e record of you r
progress. Do not exceed you r revision time and break u p the available time into study sessio n s a n d b rea k s .

Revision, like all learning, must b e active. D o not believe that just l oo ki n g a t a b o o k i s a n e ffective way of l ear ni n g .

Reward and Punishment System


H u mans work o n t h e p a i n pleasure form ula. A human b eing seeks pleasure and avoids pain. U sually it h a p p en s t h a t
most o f t h e plans o f time aJJocation run i n to trouble. T< > c i rc u m ve nt this pro bl e m use a Reward and Punis h ment Syst e 111 •
,

The method of implementing this system is to p u t p l easu ra b l e tasks at the e n d w h i l e you c om p lete painful tasks like
s tudying. S uppose you want to enj oy TV for half an h o u r, then allow you rself to watch TV o n l y after your study for two
h o u rs.
How to Attempt the
E XA MI NAT I O N PAP E R?
11w t'xami11alion queslion J)(l/Jerconlaiwi diff'i rt'1ll t)jU?s ofquestions like Ve1y Short Answer ry/Je
Questions, Short A 11swa 'J_)•/Jl! Q1wsl ions, Long A nswer 1)'/Je Questions, etc., but broadly there are
two t)jJes ofquestions wilwd in //w 1�.w1111inatio11s, one is Short Answer ryjJt! & th e other is Long Answer 1jjJe.

Both tyjJes ofquestions are dealt within different ways which are discussed below

• Structu re your answer properly.


Short Answer Type Questions • Use capital letters, bullets, and u nderlining and
Attempt all the questions, as there are no penalties for incorrect high ligh tin g as tools to attract attention to i mporta n t
answers. If a calculation is involved , be careful to work logically. points .

Always keep in mind that Very Short Answer Type Questions and I fy o u have used any page for rou gh work, it should b e
Short Answer Type Questions do not involve very lengthy crossed and clearly marked ' Rough Work'.
calculations. They generally have a fact hidden behind the m . If
• We are s u re that using the above tips and asvices, you will
you fi nd yourselfinvolved in length y calcu latio n , then for sure
surely be able to improve your grades i n the examination .
you are going wrong. Always read these L ypes ofquestions very
carefully and u nderstand the demand of the question and answe r
accordingly. Be Perfect & Precise
I fyou cannot answer a questio n , or remain u ncertain as to what is
lbo many candidates write at length without a nswerin g
the correct answer, leave it and return to it when you have
t h e question p roperly. Never forget that marks a r e n o t
completed the other questions.
awarded for j u s t the correct answer but for correct answer
Read the question carefully, remembering that at least one ofthe
that answers the question most appropriately. Do not
incorrect answers, called 'distractors' , will seem to be correct. Do
rewrite the information given in the question and expect
not rush-think. to gai n marks.

Long Answer Type Questions Time Management During Exam


Once you reach the Long Answer Questions section , go through
• As you read through the question paper, plan your
the section and choose a question you feel confident about. You
strategy. If you have any choice between the question s ,
need not start with the first question. Read the question twice,
t h e n choose carefully, because i fy o u do one option a n d
look at the mark allocation for each part and then decide exactly
later decide t o attempt t h e other o n e , t h e n y o u w o u l d b e
what is required to secure full marks.
wasting your time.

• Do not rush, as this is a major cause of m istakes,


How to Write Examiner Friendly particularly ofmisreading the question . The time
allocated for the examination is adequate for t h e s t u d e n ts
Answers to complete the paper. Leave the h ardest questions to b e
Follow the instructions in the question , carefully responding to tackled t o t h e end.
words and phrases such as describe and give a reason for.
• I fyou finish early, take the opport u n ity to check t h ro u gh
• Try to attempt the questions exactly in the same order as your answers. Ask yourself, h ave I a nswered the q uestion
written in the question paper, as it helps the examiner identify and have I made sufficien t points to be awarded fu l l
the answer immediately and he recognises the set pattern. marks ? .
• Never attempt a question consisting of two parts at widely
different places in the answer book .
Take reasonable care t h a t your writing is legible-it should
• Tie your supplementary sheets in the right order.
be easily read and be m arked . Satisfy the examiner and
• There should be minimum cutting on the paper and it should
you can score good marks.
look neat.
COURS E STRUCTU RE
Class X (Second Term )
Class 1
Term Durat ion Marks
---------- ---- -

X II 3 Hours 90

Units Descript ion �larks


--- ------ - --- - .

II ALGEB RA (Contd.) 23

III GEOMETRY (Contd . ) 17

IV TRIGONOMETRY (Contd.) 08

v PROBABILITY 08

VI COORDINATE G EO M ETRY 11

VII M ENSURATION 23

TOTAL 90

Unit II . Al gebra (Contd . )


3. Quadratic Equations ( 1 5 Pe 1·iods)
Standard form o f a quadratic equation ax'.! + bx + c = 0, (a 1 0). Solution o f t h e
quadratic equations (only real roots) by factorisatio n s , b y co m ple ti n g t h e sqt:1are a n d
by using quadratic form ula. Relation s h i p between d iscri m i n an t a n d natu re o f roots.
Problems related to day to day activ ities to be i n corporated .

4. Arithmetic Progressions (08 Per i od s )


Motivation for studying Arith m etic Pro g ress i on , Deri vation of stand ard resu lts o f
finding the nth term and s u m of first n terms a nd thei r app lication i n sol v i n g d aily
life problems.

U ni t II I. Geometr (C
y ontd . )
2 . Circles ( 0 8 Pe riod s)
Tangents to a circle motivated by chords d rawn from poi n ts co m i n g closer a n d clo ser
to the point.
(i) (Prove) The tangent at any poin t o f a circle i s perpend i cu l a r to the rad i u s through the
point of contact.

(ii) (Prove) The lengths of ta ngen ts drawn from an e xternal point to ci rcle are equal .
3 . Constructions (08 Period s)
(i) Division of a line segmen t in a given ratio ( i n tern a l l y ) .

(ii) Ta n ge n t to a ci rcle from a poi n t ou tside it .


(iii) Construction of a trian gle sim i lar to a given tri a n gl e .
Un i t IV. Trigonometry
Heigh ts and Distances (08 Perio ds)
Simple and believable p rob lems on heights a nd distances . Problems sho u ld no t
involve more t han two right triangles. Angles of elevation I depression s h o u l d be o nly
30°, 4 5 ° , 60°.

U n i t V. Statistics and Probab i lity


Probability ( I 0 Pe riods )
Classical definition o f probability. Con nection with p robabil ity a s given in Class I X .
Sim ple problems on single events , not using set notation.

U n i t VI . Coordinate Geometry
Lines ( I n two-di m e n sions) ( 1 4 Periods)
Review the concepts of coord i nate geometry done earlier including grap h s of l inear
equations . Awareness of geometrical representation of quadratic polynomials. Distance
between two points and section formula (internal ) . Area of a triangle.

Un i t VII . M.e nsuration


1. Areas Related to Circles ( 1 2 Periods )
Motivate the area o f a circle; area o f sectors a n d segme nts o f a circle . Problems based
on areas and perimeter /circumference of the above said p lane figures. (In calculating
area of segment of a circle , p roblems should be restricted to central angle of 60°, 90°
and 1 2 0° only. Plane figu res i nvolving triangles, simple quadrilaterals and circle
should be take n . )

2 . Su rface Areas and Volumes ( 1 2 Periods)


(i) Problems on finding s urface areas and volu mes of combinations of any two of the
following: cubes , cuboids, spheres , hemispheres and right circular cyl i n de rs/cones .
Frustum of a cone.

(ii) Problems i nvolving converting one type of metallic solid into another and other
mixed problems. ( Problems with combination of not more than two different solids be
taken.)
C H A PT E R 1

QUAD RATI C EQUATI O N S

In this chapter, we will study quadratic equations and various ways of finding their roo ts.
We will also discuss some applications ofquadratic equations, based on daily life situations.

Q uadratic Equation and


Its Solution
TO P I C 0 Quad ratic Eq uation and I ts Solution
, Quadratic Equation
0 Solutions or Roots of a Quad ratic Eq uatio n
Quadratic Equation
An equation o f the form
Solution of a Quadratic
Equation by Factorisation [
+bx +c =0 j2
------

Solution of a Quadratic
Equation by Completing the x,
is called quadratic equation in variable where a, b and c are real numbers and a -::!:. 0.
Square and Nature of Roots 2x2 +x - 100 =0, -x2 +1 +300x =0, -3x2 +4x + =0, 4x2 -25 =0
e.g. 7 are
" Method of Completing quadratic equations.
the Square
0 Solution of a Quadratic The form ax
2 + bx + = 0c is called the standard form of a quadratic equation. To express
a quadratic equation in its standard form, write an equation in the decreasing order of
2
Equation by Quadratic

0
Formula
Nature of Roots
3x 2 + x + 2 = 0, x -2x + = 0,
power of variable. e.g. 6 are in standard form whereas,

Word Problems and Its


x2 +4x =0, x +x2 +8 =0
-3 are not in standard form.

S olution M ethod to C heck


Whether a Given Eq uation is Quad ratic or Not
To check whether a given equation is quadratic or not, first write the given equation in its
simplest form i.e. if equation has two terms containing variable x in product/division form
x
(or term containing variable has square or cube) and then compare the equation with the
standard form of a quadratic equation, i.e. ax 2 c a -::;:.
+bx + = 0, 0.
If the given equation follows the form of quadratic equation ( ax 2 +bx + 0), c = then it is a
quadratic equation otherwise it is not a quadratic equation.

Example 1. Check whether the following are quadratic equa tions or n o t.


(i) (x - 1) (x + 2) = (x - 3) (x + 1) (ii) (x + 2) 2 = 4 (x + 3)
Sol . (1) Given,
(x -I)(x + 2 ) ::: (x -3)(x + 1 ) . . . (i)
Here, in LHS (x - 1) and (x + 2) are in produce and in RHS, (x - 3) and (x + 1) are also in
pro duct. So, first we simplify their product.
LHS = (x - 1) (x + 2) = x2 + 2x - x - 2 ::: x .! + x - 2
RHS = (x - 3 ) (x + l) = x2 + x - 3x - 3 = x1 - 2x 3
-
Alll'nOne MATHEMATICS Class 1 0th Term 1 1

On substituting these values in Eq. (i) , we get Exa m p le 2. In th e followin g eq u a tion, determine
- 2 = i - 2x - 3
x2 + x x whether the given values are solu tion of the given
=> xi -x2 + x + 2x + = => + = -2 0 1 0 equation or not 6x2 - x - 2 = 0 x = _!_ , x = �

..
3 3x ; _

ax 2 0, 0.
2
It is not of the form + bx + = a '* Since, herec
p(x) = 0, where
3

a = 0 and it is an equation of degree I . Sol. Given equation is i n che form


Hence, the given equation does not represent a quadratic p(x) = 6x.! - x - 2 . (i)
- 1 .m E q. ( 1. ) , we gee
equation.
0 n pumng x =
.
-

� 2

=> p( ) =
In the above example, the given equation appears co be a
quadratic equation. but it is not a quadratic equation. So,
simplify the given equation before deciding whether it is
quadratic or not.
(it} Given, (x + 2)2 = 4(x + 3) . . . (ii) � + _!_ - 2 = 6 + 2 - 8 = 8 - 8 -1
0
2
=
Here, term (x + 2) has power 2. So, first expand it. 4 2 4 4
LHS = (x + 2)2 x2 + 4 + 4x
=
x = .=..!.2 is a
[·: (a + b)2 = a2 + b2 + 2ab] So, soluc ion of chc given cq uacion .

() () ()
On substituting chis value in Eq. (ii), we get Now, on putting x = .!_ in Eq. (i), we gee
12
·

x2 + 4 + 4x = 4x + 3
=>x2+4x -4x + 4 = O - 12
=> x2 8 =0or x2 + Ox 8 0
p .!_ = 6 .!_ .! - .!_ - 2 = 6 x .!_ - .!_ _ 2
- = -
3 3 3 9 J

m�o
It is of the form'ax2 +bx+ a '* 0. � .!_ 2 - 3 - 1 8
c = 0,
= - - = 6 = -15
9 3 9 9

. te e eu . .
Hence, given equation represents a quadratic equation.

Solutions or Roots of a Quad ratic � p


1 .
Equation
S o, x = - 1s not a so I unon o f h g .i v n q a i
All the values of variable which satisfy the quadratic
t on
3
equation, are called roots or solution ofquadratic equation.
M ethod to Dete rm in e
quadratic equation ax 2
+bx a =0, 0, root
In other words, a real number
+c is said to be a
a * if
of a
An U n known Constant I nvolved in a Qua d ratic
aa 2 +ba +c =0. Equation when its Solutio n or Root is G iven
If ax 2 +bx + c = 0, saya that,fix =a solution
then we also is a Sometimes, given quadratic equation involves an unknown

re n
of the quadratic equation or s atis e s the q uadr ati c constant and its solution or roo t is given. Then, to find the
equation. Any quadratic equation can have atmost two value of unknown constant, we put the value of root or
roots. solution in given quadratic equation and simplify to get the
!'1 ot_�_ ·The zeroes of the quadratic polynomial ax 2 + bx + c and the q u i red unknown c o nsta t .
roocs of che quadratic equation ax 2 + bx + c = o are Sometimes, quadratic equation involves two unknown
che same.
constants and its both roo ts are given. The n, to find
Met hod to Check unknowns we put both roots one-by-one in quadratic
equation and get two linear equations in two unkn owns. On
Wh eth er the Gi.ven Values are Solution s of simplifying these equations, we get the required values of
th e G iven Equati.on or Not unknown constant.
Let p(x) = 0 be the given quadratic equation and be x =a Examp le 3. In each of the following equa tions,
value of To x.
check whether is a x =a
.
the gi ven solution of
th e gi ven eq uatio n or not, use the following steps
determine the value of k for which the given valu e is a

� Write the given eq ua tio n in the fo rm, p(x) = O


solution of the equ a tion .
( zj kx2 +2x 3 - = 0, x = 2 ( izj x2 +2ax -k x =
0, =-a

[1r1) I!J
(if not given in this fo rm)
Now, put x = a in LHS, i .e. i n p (x). If p (a) = 0 , Sol. (t) We have, kx2 + 2x - 3 = 0,

then x = a is the solution of given equ atio n, here k is unknown. Since, x = is a solution of given2
equation, so it will satisfy the equation.
otherwise not.
Quadratic Equations I 3 I
On puccing x = 2 in given equation, we get 0
. . a = 1 5 . Eq. ( .. ) we gee.
n su bst1rutmg - m 11 ,

k( 2 )2 + 2(2) - 3 = 0 2
=> 4k + 4 - 3 = 0 => 4 k + I = 0 2 x !2 - b = 6
-1 2
=> k =-
4 => 1 5 - b = 6 => b = l 5 - 6 = 9 => b = 9
(i1) We have, x2 + 2ax - k = 0, Hence, the required values of a and b are 1 5/2 and 9,
here k is unknown. Since, x = a is a solution of -
respectively.
given equation, so it will satisfy the equation.
On puccing x = - a in given equation, we get TRY YOURSELF
(-a )2 + 2a(-a ) - k = 0
=> a2 - 2a2 - k = 0 => -a 2 -k =0 Q. 1 Check whether the following are quadratic equations
or not.
=> k = - a2
(1) x - x1
- = 8
Exam ple 4. If x 2 and x =
equation 3x2
a and b.
2 ax + 2b =- 0,= 3 are roots of the
then find the value
(ii) ( x + 2 ) (x - 2 ) = (x + 3) (x + 4)
(iii) X2 + (x + 2 )2 = (x (1) Yes (ii) No (iit) Yes]
- 2)2 [Ans.
Sol. Given, 3x2 - 2ax + 2b = 0 . . . (i) Q. 2 In each of the following equations, determine whether
the given values is a solution of the given equations
Here, a and bare unknown constants. Since, x = 2 and x = 3 or not.
are che solution of given equation, so, it will satisfy the given (1) x2 - x + 1 = O; x = 1, x = -1
equation . .
On purc.ing x = 2 and x = 3 in Eq. (i) one-by-one, we get (ii) x2 + .J2x - 4 = X = .J2 X =
0; 2 .J2 I -

3(2)2 - 2a x ( 2 ) + 2b = 0 . (iii) 2 x2 - x + 9 ::;: x2 + 4x + 3 ; x = 2, x = 3


=> [Ans. (1) Both 1 and - 1 arc not the solution of
3 x 4 - 4a + 2b = 0
=> (it) Boch .J2 and -2.fi. are the solution
the given equation
1 2 - 4a + 2b = 0
=> - b - 6) = 0
-2( 2a
(iit) Both 2 and 3 are the solution of the
of the given equation
=> 2a - b = 6 [·: - 2 -:1; O] ... (ii) given equation.]
and 3(3 ) 2 2a x 3 + 2b = 0
Q. 3 In each of the following equations, find the value of
-

=> 27 - 6a + 2b = 0
=>
unknown constant(s) for which the given value (s) is
6a - 2b = 27 ... (iii) (are) solution of the equation .
On multiplying Eq. (ii) by 2 and then subtract it from
Eq. (iii), we gee (1) 3x2 + 2ax - 3 = ; x = 0 �
1

6a - 2b - 4a + 2b = 27 - 1 2
2a = 1 5 (ii) kx2 + .J2x - 4 = = 0 ; x .J2
(iii) ax2 + bx + = ; x = x = 2
-3]
15
a =-
1 0 1 ,

2 [Ans. (1) a = 4 (it) -9 k = l (ii1) a = ! , b =


2 2

FO L 1 .1
1 Check whether the following are quadratic Sol. (t) Given equation is (x + 1 )2 = 2 (x 3) -

=>
equations : x2 + 2x + 1 = 2x - 6
(i) (x + 1)2 = 2(x - 3) [·: (a + b )2 = a 2 + 2ab + b 2]
(ii) x2 - 2x = (-2) (3 - x)
=> x2 + 1 + 6 =O Ci
+ 1) = (x - 1) (x + 3) LI-
=>
(iii) (x - 2) (x
x2 + 7 = 0
(iv) (x - 3) (2x + 1) = x (x + 5)
(v) (2x - 1) (x - 3) = (x + 5) (x - 1)
which is of the form ax 2 + bx + = 0 a * 0 c ,

(vi) x2 + 3x + 1 = (x - 2)2 : . It is a quadratic equation.


(vii) (x + 2)3 = 2x (x2 - 1) (ii) Given equation is x2 - 2x = ( 2 ) (3 - x ) -

(viii ) x3 - 4x2 x + 1 (x - 2)3


- =
=> x2 - 2x = - 6 + 2x
AlllnOne MATH EMATI CS Class 1 0th Te rm 1 1

=> x2 - 2x -2x + 6 = 0 => x·' - 4x.:? x I x·'


- + = -6x.? + - l 2x 8
=> x2 -4x + 6 =O => x·' - 4x.? - x I x3 x .? - 1 2x 8 0
+ - +6 + =
which is of the form ax2 + bx + = c 0, a :1: 0. = 2x.? - I 3x + 9 0
:. It is a quadratic equation. which is of the form + =
ax .? + bx c 0, 11 � 0
(iii) Given equation is
(x - 2) (x + I ) = (x -1) (x + 3) :. le is a quadratic equation.
=> x2 - 2x + x - 2 = x2 -x + 3x -3 N ote I n pare (vi i i ) . the given equa t i o n a p p e a rs co be a cubic
equation and not a quadra t i c equat i o n but it c u m s out co
=> x2 - 2x + x -2 -x2 + x -3x + 3 = 0
be a quadra t i c equat ion . So. we have c o s i rn p l i ry t h e
=> -3x + 1 =O given equa t i o n befo re deciding whet h e r it is quad ra t i c
which is not of the form ax2 + bx + = since here
c 0, equation o r not .
See Q . 2 of N C E RT Folder at page 2 6 .
a:. =0.It is not a quadratic equation.
Additional Question s
(iv) Given equation is (x -3) (2x + 1) = x (x + 5)
=>
=> 2x2 -6x + 2x2x -3-6x-x2+ x-3 = x2 + 5x
- 5x = 0
2 Check whether 2 is a root of t h e equation
2x 2 - 7x + 6 = 0. CCE 20 1 3
=> x2 -lOx-3 =0 Given equation is 2x.:? - 7x + 6 = 0
0,
Sol.
which is of che form ax2 + bx + = c a :;: 0.
Lee p(x) = 2x.:? - 7x + 6
:. It is quadracic equation.
a

(v) Given equacion is (2x -1) (x -3) = (x + 5) (x -1) On puccing x = 2 in p(x}, we gee
=> 2x2 -x -6x + 3 = x2 + 5x -x - 5 p(2) = 2(2) .? - 7(2 ) + 6
=> 2x2 -x -6x + 3 -x2 -5x + x + 5 = 0 p(2) =8 -14 + 6
x2 -1lx + 8 =O p(2) = 0
0,


which is of the form ax2 + bx + = a c :1: 0. Hence, 2 is che root of the given equation.
:. It is a quadratic equation. 3 Find the value of p, when
(vt) Given equation is
x2 +3x+1 = - 2)2 (x px 2 + (.,/3 - .Ji) x - 1 = 0 and x = is one root
x2 + 3x + = x2 + 4 -4x
I
of this equation. CCE 20 1 3
[·: (a -b)2 = a 2 + b2 -2ab ] Sol. Given quadratic equation is
x2 + 3x + -x2 -4 + 4x =
I 0
px 2 + ( .f3 - .f2.) x - =
7x -3 = O I 0

� in given equation, we get


which is not of the form ax2 + bx + = since here
c 0, I
and x = .j3 a
.
1s. root o f th"1s equauon.
a =O.

( � J + ('1j - .fiJ �
:. It is not a quadratic equation. On purring x =
(viz) Given equation is (x + 2)3 = 2x (x2 -1)
=> x3 + 8 + 3x2 (2) 3x (2)2 = 2x3 - 2x
+
P -I =0
[ ·: (a b)3 = a3 + b 3 + 3a2b + 3ab2]
+
x3 +8+ 6x2 +12x = 2x3 - 2x
x3 + 8 + 6x2 + 1 2x - 2x3 + 2x =
=> 0
=> -p3 + J3 - Fi
.f3
- .f3 =0
=> -x3 + 6x2 + I 4x + 8 = 0
=> !!_ _ .J2 = O
h h
w ic is not of the form ax2 + bx + c = 0, 3 .f3
p = Ji. 3 = 3 .f2. .f3
3
because it has cubic term, i.e. x • x x
:. Ir is
not a quadratic equation. J3 .f3 .f3
(viii) Given equation is x3 - 4x2 -x + 1 = (x - 2)3 x
= 3 .Ji.3 .J3 = -J2.
3 3
x .J3 .J6
=
=> x -4x2 - x + 1 = x -3 (x2) 2 + 3 x (2) 2 - (2)3

[·: (a - b) 3 = a 3 -3 a2b +3ab 2 -b3] Hence, the value of p is J6.


TO P I C EXERCISE
- ...
:-- . - . · .:-::.··_: . - :::.:. .._ :
. - .
...... - --- -
.. ·-
.... -- -=-:... ...:. ------ :--. :_-:-:.::....
:. ::
·- ·- .. - - - ·-- - -- . --- -·
; :::-_-; a:--=..- .::- ..:.. :-:-...:· ·.- :.....:- . . --:.: - -·=· -
-
. .
·- .
- · --

Very Short Answer Type Questions 7 I n equation 7x + kx


2
-3 = 0, find the value of k for
1 Is, .J2x2 + 7x + s.J2 = 0 quadratic equation? If yes which x = � is a solution of the g iven equation?
3
then give reason.
2 Is x = - 2 a solution of the equation x 2 - 2x + 8 = O? Short Answer Type II Questions
8 Which of the following are the solutions of
- 3 is a solution of equation -
CBSE 2008
3 Show that x = 2x2 S x 3 = 0? -

x2 + 6x + 9 = 0. (i) x = 3 (ii) x = 4
"' -1
(Ill ) X = -
Short Answer Type I Questions 2
9 Which of the following are the roots of
4 Check whether the following equations are
3x 2 + 2x - 1 = O?

-
quadratic or not.
(ii) 2x 2 - S x = x2 - 2 x + 3
1
( II )
'' 1
3
(I ) X + = x-') ( Ill
. . .) X = - -
2
' (i) x = - 1 X=-
X 3
... 2 1 0 I f (x + a) is a factor of 2x 2 + 2a x + S x + 10 = 0, then
(iv) x2 - 3x - .JX + 4 = 0
1
( Ill ) x - - = 5
x2 find the value of a.
-
CBSE 2008
5 Which of the following are quadratic equations
1 1 Find the values of a and b for which x = � and x = 2
(i) x(2x + 3) = x + 2
-
(ii) (x - 2)2 + 1 = 2x - 3 4
(iii) y(By + 5) = y 2 + 3 are the roots of the equation ax 2 + bx 6 = 0.
(iv) y ( 2y + 15) = 2(y2 + y + 8)
6 Simplify the equation (2x - 1) 2 - 4x 2 + 5 = 0 and
1 2 If x = - 2 and x = -l are solutions of the quadratic
5
equation Sx2 + px + q = 0, then find the values of p
show that it is not a quadratic equation .
and q.

Answers
1 . Yes, because its degree is 2. 2. No 4. (1.) No (ii) Yes (iii) No (iu) No
5. (i), (ii), (iii) -1 -1
2
7. k = - 8 . X= 3 , X = -
1
6
9. X = - 1, X = - 1 0. a= 2 11. a = 4, b = 5 1 2. p = 11, q = 2
3

TO P I C 8 Solution of a Quad ratic


Eq uation by Factorisation
Mainly, quadratic equation can be solved using three M ethod for Findin g Solutio n
methods; by factorisation, by completing the square method o f a Qua dratic Eq ua tion
and by quadratic formula. We will discuss all three methods
To find the solution of a quadratic equation by factorisation
in full detail. Here, in this topic, we will discuss the solution
method, we use the following steps:

( Step ) First, write the given equation in standard form


of quadratic equation by factorisation.
To find the solution of a quadratic equation by factorisation I
method, we first write the given quadratic polynomial as (if not given in standard form)
product of two linear factors by splitting the middle term 1.e. ax 2 + bx + c = 0, a *- 0
and then equate each factor to zero to get possible roots of and then find the values of a, b and c .
given quadratic equation.
Alllnone MATH EMATICS Class 10th Term 11 I

(S ep II ) Multiply the coefficient of x 2 and constant term


t . . a = 1. b = 5 and c = -6

ac = 1 x ( -6 ) = -6
i.e. multiply a and c. Then, split the product ac Now, ;
I

into factors and from these factors take Here, c has -vc sign. so we have co find chose faccors o f 6 whose � ·

difference is 5 . Such factors arc 6 and 1 .


combination of factors p and q such that
=> x2 + (6 - l )x - 6 = 0 =:> x 1 + 6x - x - 6 = 0
p +q = (if sign of c is +ve )
b => x(x + 6 ) - 1 (x + 6 ) = 0 =:> (x - l ) ( x + 6 ) = 0

�lS-tt_'P_Il_I)
or p - q = b (if sign of c is -ve ) · => x - l = 0 or x + 6 = 0 => x = I or x = - 6

-- --
Write the middle term constant b of Hence, roots o f cquacion x 1 + 5 x - 6 = 0 arc 1 an<l - 6.
ax 2 + bx + c = O as (p + q) or (p - q) depending
Exam ple 2. Using factorisa tion, s olve th e followin g

-- -- -
upon the sign of c, and factorise the equation by . . x -1 x -3 1
quadratic equat10n + =3- CBSE 2005
grouping method. Finally, we get factors in the x -2 x -4 3
form of (x + m) and (x + n) or (rx + m) and (sx + n).
(step 1v)
x -1 x -3 1
Now, equate each factor to zero and find the Sol. Given, + =3
x-2 x-4 3
values of x, i.e. put x + m = O and x + n = 0 or (x - l ) ( x - 4 ) + (x - 3 ) ( x - 2 )
=>
10
rx + m = 0 and sx + n = 0 separately and solve (x - 2 ) (x - 4 ) 3


them for x. These values of x gives the required x2 - 4x - x + 4 + x1 - 2x - 3x + 6
=>
10
roots of given quadratic equation.
x2 - 4x - 2x + 8 3

Black Board Example =>


2x2 - 1 Ox + I O 10
=- =>
2 [ x .! - 5x + 5 ] 10
x2 - 6x + B 3 x.! - 6x + 8 3
Find the roots of the quadratic equation => 3(x2 - 5 x + 5 ) = 5 ( x .! - 6x + 8 )
x2 - Sx + 6 = 0 by factorisation. method. [dividing both sides by 2 and cross-multiplying)
Step 1 Write the given quadratic equation in standard form => 3x2 - 1 5x + 1 5 = 5 x .! - 30x + 40
and find the values of a, b and c. => 5x2 - 30x + 40 - 3x 2 + l 5x - 1 5 = 0

-
Given, quadratic equation is + x2 - 5x 6
0, already =
=> 2x 2 - 1 5x + 25 = 0
in standard form.
=>
On comparing it with ax2 + bx + c = 0, we get
2 x 2 - 1 Ox - 5x + 25 = 0 [ by faccorisation]
1
a= , b and c
= -5 = 6 => 2x(x - 5 ) - 5(x - 5 ) = 0 =:> (x - 5 ) ( 2x 5) = 0
=>
5
Step 2 Multiply a and c and split ac into prime factors. Then, x - 5 = 0 or 2 x - 5 = 0 =:> x = 5 or x = -
find two numbers p and q such that p + q = b 2
(because c has + ve sign) . Hence, the roocs of the given quadratic equation arc 5 and 2-.
ac = 1x6 6 =
2
Factors of 6 are 2 and 3.
+
p q = - 5 and pq 6 =
Here ,
TRY YOURSELF

- 6 x2
.. p = -2 and q = -3
Step 3 : Write the constant b of ax2 + bx + c O as (p + q) and =
Directions (Q. Nos. 1 -3) Find the roots of the each of the

[ Ans. 23 ' 3-1]


following quadratic equation by factorisation method.

Q. 2 1 0x - - 3
then factorise it.
+ 6 = x2 + (-2 3)x + =
x2 - 5x Q. 1

Now, put x - 2 = 0=> x = 2 or x - 3 = 0=> x = 3


= x(x - 3(x (x (x
2) -
+ (- 2 - 3)x + 6
- 2) = - 2 ) - 3)
6x2 + 1 1x + 3 = 0 -

[ Ans. 21 ' s- 1 ]
Step 4 Equate each factor to zero and find the values of x. 1 =

[Ans.2,i]
Thus, 2 and 3 are the required roots of a given
Q. 3 2 -
2 5
-+2 =

-2
quadratic equation. 0
CCE 2 0 I 3

[Ans.�,-�]
! - - -- - - - - - - - --- · - - -
x x

Example 1. Solve the following quadratic equation


by factorisation method. Q. 4 Solve for x : 6x2 - J2.x = o
x 2 + 5x - 6 = 0

x+ 3J
2(� ]
Sol. The given q uadratic eq uation is x2 + 5x - 6 = 0. Q. 5 Using factoris atio n , solve the q u adratic e q u atio n
On comparing wi th standard form of q uadratic equation i.e.
- 3 (�
2x - 1
)=5 [ Ans. =.!.
s ·
_
lo
11x2 + bx +c = 0,
OLDE 1 .2
1 Find the roots of the following quadratic [·: (-4) x (- 4) = 16 and -4 - 4 = - 8)
equations by factorisation: 4x ( 4x - 1 ) - 1 ( 4x - 1 ) = 0
(i) x2 - 3x 10 = 0
- (ii) 2x2 + x - 6 = O ( 4x - 1 ) ( 4x - 1 ) = 0
(iii) .fi. x2 + 7x + s.fi. 0 (iv) 2x2 x + .!.8 O Now, 4x - 1 = 0 => x = 1 . Clearly, chis rooc is
-
= - =
4
(v) 10ox2 - 20x + 1 o = repeated twice, one for each repeated factor 4x - I .
Sol. (1) Given equation is x 2 - 3x - 10 = 0
Hence, the roots of the equation
=> x 2 - 5x + 2x - 10 = 0 2x 2 - x + 81 = Oare -and
-
I -.1
4 4
[·: -5 x (2) = -10 and -5 + 2 = -3] (v) Given equation is 100x 2 - 20x + I = 0
=> x (x - 5) + 2 (x - 5) = 0 => I 00x 2 - 1 Ox - 1 Ox + 1 = 0
=> (x - 5) (x + 2) = 0
=>
[·: (-1 O) x (-10) = 100 and -10 - 1 0 = - 20]
x - 5 = 0 or x + 2 = 0 => 1 Ox (1 Ox - 1 ) - 1 (1 Ox - 1 ) = 0
=> x = 5 or x=-2 => (1 Ox - 1) (1 Ox - 1) = 0
Hence, the roots of the equation Now, l Ox - 1 = 0
xi -3x - 1 0 = 0 are - 2 and 5.
=> x = _!__, Clearly, this root is repeated twice, one for
(iz) Given equation is 2x 2 + x - 6 = 0 10
=> 2x 2 + 4x - 3x - 6 = 0 each repeated factor 1 Ox - 1.
[·: 4 x (-3) = -12 and 4 - 3 = I] Hence, che roots of the equation
=> 2x (x + 2) -3 (x + 2) = 0 100x 2 - 20x + I = 0 are - 1 and -. 1
=> (x + 2) (2x -3) = 0 10 IO
x + 2 = 0 or 2x -3 = 0 2 Solve the following quadratic equations :
=> x = -2 or x = -3 (i) x2 - 45x + 324 = 0 (ii) x2 - 55x + 7 50 = 0
2 Sol. (1) Given equation is x 2 - 45x + 324 = 0
Hence, the roots of the equation => x 2 -36x -9x + 324 = 0

]
2xi + x - 6 = 0 are - 2 and �.

[','
2 [·:(-36) x (-9) = 324 and - 36 - 9 = -45 ]
=> x (x - 36) - 9 (x - 36) = 0
(iii) Given equation is Ji.xi + 7x + 5 ..fi. = O => (x - 36) (x - 9) = 0
=> .fi.xi + 5x + 2x + 5 ..fi. = 0
5 x 2 =10 => x -36 = 0 or x -9 = 0
and 5 + 2 = 7 => x = 36 or x = 9
=> x c .fi.x + 5) + .J2 ( .fi.x + 5) = 0 Hence, the roots of the equation x l 45x + 324 = 0
-

=>
are 9 and 36.
c .fi.x + 5) (x + .fi. ) = 0 {it) Given equation is x 2 - 55x + 750 = 0
=> .fi.x + 5 = 0 or x + ..fi. = 0 => x 2 - 30x - 25x + 750 = 0
=> x=- -5 or x = - v2 r;: [·:(-30) x (-25) = 750 and -30 - 25 = - 55)
J2 => x (x - 30) - 25 (x - 30) = 0
=> (x - 30) (x - 25) = 0
Hence, the roots of the equation
.fi.x 2 + 7x + 5 .fi. = 0 are Ji and - .fi.. => x - 30 = 0
or x - 25 = 0
. .
(iv) G 1ven equanon 1s 2 x 2 - x + - = 0
. 1 => x = 30 or x = 2 5
8 Hence, the roots of che equacion x � - 5 5 x + 750 = 0
On multiplying both sides by 8, we gee are 25 and 30.
16xi - Bx + 1 =0
N ote See Q. Nos. 3 to 6 of NCERT Fold e r at page 26 and 2 7 .
=> l 6x2 - 4x - 4x + 1 = 0
a \ Allin one MATHEMATICS Clas s 10th Te rm I I ;

Additional Questions 4 Solve the quadratic equation --


x +3
-- =-
1-x 17
4

-- --
x
1 by
x - 2
3 Fin d the roots of 5 1x + 11 + 3 +
5 12 - x i = 5
by factorisation method .

------
factorisation method.
. . . x +3 1 - .'\.· 17
Sol Given equation is, 5<.r + 0 + 5<2 - .r> = 53 + l Sol. Given equaeton a s - =-
x-2 x 4
r == 1 26

5 y + - = 1 26
=> 5.r . 5' + 52 · 5 -.r = 1 2s + 1 => s··-. s + x(x + 3 ) - (1 - x ) (x - 2 ) 17
5.r =-
x(x - 2 ) 4
Let 5.r = y, then given equation becomes
4 [x 2 + 3x - (x - 2 - x i + 2x)] = 1 7( x 2 - 2 x )
52 �
5y2 + 25 = 1 26y
y 4x2 + 1 2x - 1 2x + 8 + 4x .'? = 1 7x ! - 34x

=> 5/ - 1 26y + 25 = O => 5/ - 1 25y - y + 25 = 0 Bx2 - 1 7x 2 + 34x + 8 = 0


[ ·:(-125) x (- 1) = 1 25 and - 1 25 - 1 = - 1 26] -9x2 + 34x + 8 = 0 => 9x .'? - 34x - 8 = 0
=> 5y(y - 25) - l (y - 25) = 0 => (y - 25 ) (5y - 1) = 0 [dividing boch sides by - 1)
=> y - 25 = 0 or 5y -1 = 0 9 x 2 - 36x + 2x - 8 =0
=> y = 25 or
1
y=-
x
[·: ( -36 ) 2 = - 72 and ( -36 ) + 2 = - 34 ]
5 9x(x - 4 ) + 2 ( x - 4 ) = 0
=> 5.r = 25 or 5 .r = !.5 [ put y = 5.r )
x-4=
(x - 4 ) (9x + 2 ) = 0
0
or 9x + 2 = 0
-2
=> 5.r = 5 2 or 5.r = 5-1 x = 4 or x = -
9
On comparing the powers, we get
x 2 or x = - 1 Hence, the re i e
qu r d roots of the given equation arc
Hence, required=roots ofthe given equation are -1and2.
the
-2
4 and -.
9

TO P I C EXERCISE
Short Answer Type I Questions 12 1 1 _ = !; x * 3 . - 5
x+S 6
__ _ _

Directions (Q. Nos. 1 -6) Solve each of the following


x-3
equations by factorisation. 1 3 21

-
29
- - 10 = 0; x * 0
- 10 = O 2 x2 - 10x + 21 = 0
x2 x
1 t 2 + 3t

3 8x2 22x - 21 = 0 4 15 y 2 = 41 y - 14

5 6x2 + 40 = 31x 6 21x2 - 2x + �=0 Long Answer Type Quest ions


21
Using factorisation method, solve the following
Solve the following (Q. Nos. 7 to 10) quadratic eq u ations (Q. Nos. 15 to 17) quadratic equations.
7 3x2 + 2 .JSx - 5 = 0 8
15 �+ :;a!: 4,
2x - 5 25
- ( J2 + 1) x + J2 = 0
= ;x 3
x2 x-4 x-3 3

( ) 9( 3) =
9 ./3x2 + 11x + 6 ./3 = 0 1 0 4J5x2 + 3x - 2 .JS = O 16 x+ 3 = 3x - 7
x+2 2x - 3
Short Answer Type II Questions 17 2 x+ 2 2x -
- 3
Directions (Q. Nos. 11 to 14) Solve the following quadratic 2x - 3 x+ 2

1 8 Solve for x __ = 2 ; x � p , q
e q uation by method of factorisation.
: _E_ + q

1 1 Sx + _! = 6 ; x�0
x-q x-p
x
Quadratic Equations / 9

Ans wers
1 . -5, 2 2. 3, 7 3 -
7 -3
. 2' 4
- 4 . '!._ �
3' 5
5 � �
. 3' 2
6 __!_ __!_
. 21' 21
.J5 3J3 - 2J3 .J5 - 2.J5
7. - .J5 . 9. 1 1 . L _!
,
8 . .J2, 1 -

4
1 0. '
1 2. -9, 7
3 3 5 5

1 3. - 7 �
-1
1 5. 6' 4
0 11 5
1 8. p + q, p+ q
5' 8
1 4. 5 . - 1 6. 5, - 1 1 7.
2 ' 5 5 13 2

TOPIC 0 Solution of a Quad ratic Eq uation by


Completi ng the Sq uare and Natu re of Roots
(Step V ) Write LHS in the form of perfect square and
( x + ..!!._ ) 2 = b 2 - �ac
In the last topic, we learnt to solve quadratic equation by
factorisation method. This method for solving quadratic simplify RHS to get
equation is not suited to solve all quadratic equation. Now,
we will learn two more methods of solving quadratic
equation, these are completing the square method and by
2a 4a-
quadratic formula. Both of these methods can be used to [ ·: A2 +2AB + B2 = (A + B)2]
solve all quadratic equation. Also, we will study about the
Take square root on both sides, we get
existence as well as nature of roots.
To find the solution of a quadratic equation by completing
the square, we use the following method.
x +-2ab -- -+ �b'4a2-4ac -- + .Jb 22a-4ac
[while taking square root we consider both +ve

(:a ) to RHS.
M ethod of Completing the Sq u a re
(Step J
and -ve value]
VII
To solve the solution of quadratic equation of the form
ax 2 +bx + c = 0, a -:t 0
by completing the square, we use the x
Find the values of by shifting the constant
following steps: term i.e.
(Step I ] Write the given equation in standard form (if not
given in standard form) ,
t.e. ax2 +bx+c = O, a -:t 0 i.e. x= ± �b22a-.. 4 ac - -2ab
(Step ) Make the coefficient of x 2 unity by dividing the
II
Black Board Example
whole equation by the coefficient of x 2 t.e. a,
(if it is not unity) Solve the quad ratic eq uation 2x 2 + 10x + 3 = 0 by
1.e. x-.., +-x
b +-c =0
a a Step
completi ng the sq uare method.
1
(Stl/J III) Shift the constant term !._a on RHS, we get
Write the given eq uation in standard form .
The given equation is already in standard form , so we
can write it as it i s .
2 x� + 1 Ox + 3=O
x2 u n ity by divid i n g the whole
b =--c
x 2 +-x Step 2 Make the coefficient of
a a equation by the coefficient of x2 i . e . 2 .

(Stt'fJ V) Add square of half the coefficient of x i.e.


I
. ( b J2 O n d ividing the whole equation by 2 , w e g et
2 10
x + -x+ - = 0
3
2a 2 2
on both sides to obtain 2
x + 5x + � = 0
( _!!..__ J 2 ( _!!..__ J 2 c 2
Now , coefficient of x2 is u n ity .
a + 2a = 2a a
x2 +!!_x
All{none MATHEMATICS Class 10th Term 1 1

Shifting the constant term on RHS. (x - = )


"3 + 1 ' -4.J3 + ( .fj + I )�

)
Step 3
2 4
� on RHS, we get + // = ( 11 /J ) � )
2
On shifting the constant term i.e.
( • : II � - 211b

(
-

+ 5X= -3 "3 + 1 ' -4J3 h/3 ) � + I + 2 .J3


+
2 =>
-
x2
x- =
2

(x -
4
Step 4 Add square of the half of the coefficient of x on both [·: (a + bf· = a � + b� + 2ab I

)
sides.

On adding rnJ both sides, we get


=>
FJ + I
'
(.Jj )� + I - 2.fj
4

( r( r
2
x2 + 5x + (25 )2 = -23 + (25 )2
=>
"3 + 1 "3 - 1
=

)
x-
Step 5 Write LHS in the form of perfect square and simplify 2 2

( ( )
the RHS. On caking square root both sides, we gee
Writing the LHS in the form of perfect square "3 + 1 = ± ../3 - l
=> x = ../3 1 ± ../3 1
x- + -
(x+ �2 )2= -�2 + 245 [·: a2 + 2ab + b2 = (a+ b)2) 2 2 2 2
On caking '+ ve' sign, we gee
(x+ �)2 = -6+ 25 = 1 9
2 4 4
- 2
x = .J3 + + .J3 I = .J3 = .J3
I
2 2 2
or on caking '- ve' sign, we gee
Step 6 Take the square root on both sides.
On taking square root both sides , we get x = .J3 + 1 __ .J3 - 1 = J3 + t - .J3 + 1 = � = I
2 2 2 2
(x+ �)= ±ft (x+ �) = ±� �
Hence, .J3 and I are che roots of the given quadratic equation . .
Se 7
tp Find the values of x by shifting the constant term on N o te l nscead of making coefficienc of x 2 u n i cy , we can m a k e c h e
RHS. first t e r m as perfect square (i f i c is not) , by multiplying ea c h
term by 'a ' . i . e . write t h e first t e r m a s (a 2 x 2 ) o r (aX) 2 a n d
On shifting � to RHS, we get
2 then complete t h e square .
.J19 5
X= ±---
2 2
� X=. ---
±./19 - 5
2
Example 2. Find the roots of the equation
-
Sx 2 - 6x - 2 = 0 by � ompleting the square method .
Now, X= --
./19
2
5 [taking +ve sign)
Sol. Given quadratic equation is
5x2 - 6x - 2 = 0

(�
or ---
-
X= 2 - 5
./19 [ taking -ve sign) . . . (i)
On multiplying Eq. (i) by 5, we gee

)+
So, roots of the equation 2 x2 + 1 Ox+ 3 = O are 25x2 - 30x - 1 0 = 0 => Sx )2 - 2 x Sx x 3 = I o
(
-5
an �+ 5 ) =:) (5x) 2 - 2 x 5x x 3 + 32 = 32 + 1 0
[adding 3 2 on bo ch si des]
(5x - 3)2 = 9 + 1 0 [·: a2 - 2ab + b2 = (a - b)2 ]
Example 1. Solve the equation is
=> (5x - 3)2 = 1 9
x2 - (../3 + 1) x + .J3 = O by completing the square On taking square root both sides, we get
method. 5x - 3 = ± Jl9 =:) 5x = 3 ± Jl9
Sol. Given quadratic equation is On taking '+ ve' sign, we get
3 + Jl9

�-
x 2 - ( .../3 + 1) x + .../3 = 0 [here,coefficient of x2 is unity] 5x = 3 + .J19 =::) x = -�-

( -)
5
:::::> x 2 - (.../3 + 1) x = - .../3 or on talcing '- ve' sign, we get
3 - .f19

(� J (�
.../3 + 1 2
On adding -2 5x = 3 - .{19 =::) x = -
. .

- ,J3
both sides, we get
5
3+ 3
9 are rhe roots of the give n

J
+1 - ,/3 + +1 Hence, __ _J}9
_i_
9 and _ _
-_J}9
_I_
x' ( + J) x + = 5 5
quadratic equation.
Quadratic Equations f 11 i
Solution of a Quad ratic Eq uation by E x a mp le 1 Using the quadratic form ula, solve
the following equations:
Quad ratic Form u la
.J3x2 + 1 lx + 6.J3
.J3 2
The method of completing the square to solve quadratic equations is
a long way to solve a quadratic equation. But if we use this method
to solve general quadratic equation ax 2 + bx + c = 0 then we get a
Sol. The given equation is x + 1 lx + 6.J3
= 0.
On comparing with ax 2 + bx + c = 0, we get
formula as the result that can be used to solve all quadratic equation, a = .J3, b = 1 1 and 6.J3
c=
the formula so obtained is called quadratic fo rmula.
On substituting the values of a, b and c m the
quadracic formula,
Method to Solve
A Quadratic Equation by Using Qua dratic Formula
-b ± .Jb2 - 4ac -
x = ----
2a
To solve a quadratic equation by quadratic formula, we use the
following steps: x = - 1 1 ± �(1 1)2 - 4(.J3)(6.J3)
[ s;,p_I_] Write the given quadratic equation in standard form i.e.
2
2(..fj)
ax + bx + c = 0 (if not given in standard form) a '¢ 0.
- - 1 1 ± �12 1 - 72
- - -- · - - - - .

�t�p 1q Find the values of a, b and c by comparing the given 2.Jj


-:__�-==- equation with 11x 2 + bx + c = 0.
(Ste.e__!! � Put the values of a, b and c in the quadratic formula. i.e.
· - -1 1 ± ./49
2./3
-b ± .Jb 2
4 -- ac- -1 1 ± 7
x= -- 2a 2./3
[�tei!SJ Simplify the expression obtained in Step III and get the
required roots or solutions.
=> x = -1 1 + 7 =
2./3 2.J3
-4 = -2
.J3
or x = -1 1 - 7 = -18
2./3 2./3
Solve 12x2 + S x - 3 = 0 by quad ratic formula. - -9
Step 1 Write the given quadratic equation in standard form.
Jj
Hence,
-2 and .J3
J3 -9
are the required solutions of the
The given quadratic equation is already in standard form, so it
can be written as it is.
1 2 x2 + 5x - 3 = 0 given equation.
Step 2 Find the value of a, b and c by comparing the given equation
with ax2 + bx + c = 0.
On comparing the given equation with ax2 + bx + c = 0, we get
TRY YOURSELF
a = 1 2 , b = 5 and c = -3
Find the roots of the following (Q. Nos. 1 to 3) q u adratic
·.-b ± �b2 - 4ac ---
Put the values of a, b and c 1n formula 1 .e. x = --
. equations by method of completing the square.
Step 3 2a
Q. 1 x2 - 4J2.x + 6 = 0 [Ans . .J2. 3 .J2]
o n p utt ing a = 1 2, b = 5 and c = -3, we get
2 3x2 + 1 1x + 1 0 = 0

[ 2$5]
X=
-5 ± �(5)2 - 4 x 1 2 x (-3) Q.
2 x12
Q. 3 � x2 - Bx + 3 = 0
.Jf69
Step 4 Simpli fy the expression obtained in Step 3.
Ans.
4 16 ±
-5 ± -5 ± ---
X=
$+144 =
=�
-5 ± 1 3 3
24 24
to 5) quad ratic

[Ans.� J
Find the roots of the following (Q. Nos. 4
.. . x = � [taking +ve sign] or x =
-5 - 1 3 equations, if they exist, by using quadratic formula.
[taking -ve sign)
24
Q. 4
24
8 1 18 3 3x2 + 2.JSx - 5 =O
x = - = - or x = - = - -
- . - JS
24 3 24 4

Hence. the ro ots of th e given equation are 3 an d - 4.


3
Q. 5 [Ans. Roots does not exist]
. . 1 x2 + x + 2 = O

....__________ __ __ _ ________________
_
_,
I 12 Allinone MATHEMATICS Class 1 0th Term 1 1

Nature of Roots (1) First , determine the discriminant by using formu l a


D = b 2 - 4ac .
By quadratic formula, the roots of the quadratic equation

I x = -b ± �:: -4ac I or I x = -b �:5 I


(ii) Further, check t h e conditions .
ax 2 +bx +c =0, a -:;: 0
are given by (a) If D > 0, then it has two real and d istinct roots.
(b) If D = 0, then it has two real and equal roots .
(c) If D < 0, then it has no real roots.
(iii) If D = O or D > 0, then d etermine the roots by using

-b ± JD
D
formula, x = .
where, = b2 -4ac is called discriminant. The nature of
roots depends upon the value of the discriminant Since, D. D
2a
Sol. (1) Given quadratic equation is 4x! + I 2 x + 9 = 0
can be zero, positive or negative, so three cases arises. On comparing with tlX! + bx + c = 0, we gee
(Cme I ) When =0, i.e. b2 -4ac =0.
D a = 4, b = 1 2 and c = 9
Now, D = b 2 - 4ac
If D =b 2 -4ac =0, then x = --
-b ± O = ( 1 2 )2 - 4( 4 ) (9) = 1 44 - 1 44 = 0

=> x=--, b b 2a Since, D = 0, so given quadratic equation has cwo equal


2a --2a
So, the quadratic equation has two equal real roots
and real roocs which are given by
-b ± .fi5
x = ---
2a
or repeated roots or coincident roots.
-12 ± 0 -1 2 ± 0
[ Crse II] When D > 0, i.e. b2 -4ac > 0. =
2(4)
=
8
x = - 1 2 + 0 or x =
If D =b2 -4ac >0, then -1 2 -0
8 8
x= -b +2a.JD and ----
-b - .JD
2a 3 3
x = - - or x = - -
2 2
So, the quadratic equation has two distinct real -3 3
Hence, the roots are - and --.
roots. 2 2
�ase I I I ) When D< 0, i.e. b2 -4ac< 0. (ii) Given quadratic equation is
3x2 + 5 x - 7 = 0
If D =b2 -4ac< 0, then .JD can not be evaluated
On comparing with llX2 + bx + c = 0, we get
as there is no real number whose square is b2 -4ac. a = 3, b = 5 and c = - 7
So, the quadratic equation has no real roots or we NQw, D = b2 - 4ac = ( 5 ) 2 - 4(3) (-7)
can say that roots of quadratic equation do not = 2 5 + 84 = 1 09
exist. Since, D > 0, so given quadrat ic equatio n has two di s t i nct
This can be explained through following flow chart. real roocs which are given by
.fi5 -5 ± .JW9
x = ----
-b ±
Quadratic equation
2a 2(3)
ax2+bx + c = O, a -:1: O
-5 + .JW9
x=
6
-5 - .Jl09
or x =
6
D=O D>O
D<O
� Roots are
-5 + [109 and -5 - Jl09
=> Roots are real => Roots are Hence, the roots are ·

and equal imaginary 6 6


5 =0
real and distinct
or not exist
(iit) Given quadratic equation is 7/ - 4y +
Example 6. Find the nature of roots of the following On comp a rin g w ith a/ + by + c = 0, we gee
quadratic equations. If the real roots exist, then also
a = 7, b = - 4 and c = 5
find them .
(5)
4x 2 + 1 2x + 9 = 0
Now, D = b2 - 4ac = (-4)2 - 4(7)
(i)
(ii) 3x 2 + 5x - 7 = 0 = 1 6 - 1 40 = - 1 24 <0

(iii) 7y 2 - 4 y + 5 = 0 Since, D < 0, so given q uadratic equation has no rea l


roots.
Quadratic Equations / 13
Method to Determ ine . Now, D = b1 - 4ac = (3k)2 - 4(9) (4) = 9k 2 - 1 44
The Value of U n known when Nature of
Since, roots of given equation are real.
.. D �O
Roots is Given => 9k2 - 144 � o => 9(k2 - 1 6) 2! 0
k2 - 1 6 � o :#
If nature of roots of a quadratic equation is given and [·.·9 O]
quadratic equation involves an unknown, then to find the => k2 -(4)2 � o
=>
value of unknown, first we find the value of discriminant
in terms of unknown. After that use the given condition
(k - 4) (k + 4) � 0 [·: a2 - b2 = (a - b ) (a + b )]
i.e. D > 0 or D = 0 or D < 0 and simplify it. k �-4 or k � 4

Example 7. Find the values of k for which the No te • x2 - a 2 > O => X < - a or x > a
following equation has equal roots. x2 - a 2 � o => x s - a or x '?!:. a
(k - 1 2) x2 + 2(k - 1 2) x + 2 = 0 • x2 - a 2 < o => -a < x < a
Sol. Given quadratic equation is x 2 - a 2 S O => - a s x s a
(k -12) x2 + 2(k - 12) x + 2 = 0 • (x - a) (x - b) > O and a < b => x< a or x > b
(x - a) (x - b) � O and a < b => x s a or x '?!:. b
On comparing with ax2 + bx + c = 0 , we get
• (x - a) (x - b) < O and a < b => a < x< b
=
a = k -12, b 2(k -12) and c = 2
• (x - a) (x - b) s O and a < b => a s x s b
Now, D = b1 - 4ac
= [2(k -12) ]1 - 4(k -12) (2) = 4(k -12)2 -B(k - 12)
= ( k - 12) [ 4( k -12) - 8 ] = ( k -12) ( 4 k - 48 -8) TRY YOURSELF
Find the nature of roots of the following (Q. Nos. 1 to 3)
= (k -12) ( 4k -56)
Since, roots of given equation are equal. ·

[Ans.
quadratic equation. If roots are real, then find them.
.. D =O

Km
=:> (k -12) (4k - 56) =0 =:> k -12 =0 or 4k -56 =0 Q. 1 3x2 - 2 .J6x + 2 = 0 CBSE 2009
=:> k = 12 or k = 564 => k = 1 2 or k = 1 4 RW and •q�
But k = 12 does not satisfy the given equation because if
=
k 12, then coefficients of x2 and x become zero. Q. 2 � x2 - � x + 1 = 0
Hence, required value of is k 14. 5 3 [Ans. No real roots]

Q. 3 (x - 2a)(x - 2b) = Real and discincc : 0, 2(a + b)]


Ex ample 8. Find the values of k for which the
4ab [Ans.
equation 9x2 + 3kx + 4 = 0 has real roots. Q. 4 Find least positive value of k for which the equation
9x1 x2 + kx + 4 = O has real roots. k = 41 [Ans.
Sol. Given quadratic equation is + 3kx + = O 4
On comparing with ax2 + + c = 0, we get Q. 5 Find the value of m for which
[Ans. iJ
bx the equation
a =9, b =3k and c = 4 mx.2 - Sx + 50 = O has equal roots.
m=

FO D E R 1 . 3
(� x J. i.e. ( � x % J = ::
0:::
1 Find the roots of the following quadratic u.J
equations, if they exist, by the method of On adding coefficient of D
completing the square: __!
both sides, we get
(i) 2 x- cu) 2 x-

( �J ��
x+ 3 =o
- 1 + x 4 =o
- 2
7 49 = - -3 + -49 0
(iii) 4 r + 4.J3x + 3 = 0 (iv) 2 r + x + 4 = 0
x - -x +- u....
2 16 2 16
2x 2

I�
(1) Given equation is
=)
Sol. 7x + 3 =0
-

(x - H = m'
x- = [·: a 2 - 2ab + b' = (a - b) 2 ]
On dividing both sides by 2, we get
l
7 3 2 7 3
x - -x + - =0 => x - -x = --
2 2 2 2 =)
14 I All{none MATHEMATICS Class 10th Term 11)
I

On taking square root both sides, we get


x - -7 = ± -54
4
=> x = .Z.4 ± 2_4 = 7 ± 5 On taking square root both sides, we gee
4
7
x = -- + 5 or 7 -5 -- J3 = ± 0
=>
4 4 x+-
2
=>
12
x = - or -
4
2
4 => x+- J3 = -0
J3 = O or x + -
1
2 2
=> x = 3 or -
2 =>
J3 or x = - -
x =. - - J3
Hence, the roots of equation 2x 2 - 7x + 3 = 0 are 2 2
+ 4 ./3x + 3 =
-1 an d 3.
Hence, the roots of equation 4 x 2 0 are
2
+ x - 4 =0
(it) Given equation is 2x 2 - J32 and - ../3 .
2
On dividing both sides by 2, we get Given equation is, 2x2 + x + 4 = 0

G J. ( � H = 1�
(iv)
x2 + -1 x - 2 = 0 => x 2 + -x 1 On dividing both sides by 2, we get
2 2 =2
x 2 + -1 x + 2 = 0
On adding coefficient of x i.e. x
2

G )'. ( � H = 1�
=>
1 =-2
x 2 + -x
both sides, we get 2
1 + -1 1
x2 + -x x
2 16 = 2 + -

( x + _41 )2 = - 1 6
On adding coefficient of x i.e.
16

( ±J (�J
=> x 2 + -1 x + -1 = -
33 both sides, we get
2 16 16 �
x2 + .!.x + _!_ = - 2 + _!_ =>
x+ = 2 16 16
[·: a 2 + 2ab + b2 = (a + b)2]
[·: a2 + 2ab + b2 = ( a + b )2] which is not possible as the square of a real number
cannot be negative. Therefore, roots of the equation
On taking square root both sides, we gee 2x2 + x + 4 = 0 does not exist.
x + .!. = ± J33 2 Find the roots of the quadratic equation s
4 4 given in Q. 1 above by applyin g t h e quadratic
f33 = - 1 ± .Jii
x = - -1 -+ -- ----
formula.
4 4 4 Sol. (t) Given equation is
x J33 - 1 or -Jii - 1
---
2x2 - 7x + 3 = 0
4 4 On comparing with ax 2 + bx + c = 0, we get
Hen ce, the roots of equation 2x2 + x - 4 = 0 are
a = ·2, b = - 7 and c = 3
- .J33 - 1 .J33 - 1 . Now, discriminant, D = b2 - 4ac
and
4 4 = ( - 7) 2 - 4 x 2 x 3 = 49 24 = 25 > 0 -

Given equation is 4x 2 + 4J3x + 3 = 0 As D > 0 , so roots exist and

x = - b ± .JD = - (-7) ± .fi5 = 7 ± 5


On dividing both sides by 4, we get
--

(� J" (� ..J3) !
x 2 + ../3x + � = 0 => x2 + .J3x = 'i -
2a 2 2 x 4
4 4 7 + 5 or x = -- 7 -5
=> x = --
' =
4 4
On adding coefficient of x i.e.
12 2
x = - or x = - => x = 3 or -I
borh sides, we ger 4 4 2
3 3 3 Hence, the roots are 3 and -.
I
x2 + J3x + 4 = - 4 + 4 2
Quadratic Equations 15

(ii) Given equation is 2xl + x - 4 = 0


On comparing with axl + bx + c = 0, we get
Now, x - b ± Jbl - 4ac [by quadratic formula]
2a
a = 2, b = 1 and c = - 4
Now, discriminant, D = bl - 4ac = -(-3) ± J(-3)l
2 (I )
- 4 (1) (- I )
= (1 )2 - 4 (2) (-4) = 1 + 32 = 33 > 0
As D > 0, so roots exist and therefore = 3 ± .j9+4 = 3 ± Jl3
2 2
x - b ± JD - 1 ± ./33 = - l ± J'ii x - 3 + Jl3 or ---
3 - Jl3
2a 2x 2 4 2 2
.J33 - 1 or fii - 1
x = --- -
1 3 + .Jl3
4 4 Hence, the roots of equation x - - = 3 are ---
x 2
Hence, the roots are
-/ii - 1 .J33 -1 .
and --- 3 - .J13
4 4 an d .
2
(iit) Given equation is {it) Given equation is
4xl + 4..J3x + 3 = 0 -- 1 1 =- 1 1 , x :t:. - 4, 7
On comparing with axl + bx + c = 0, we get x + 4 - -- - 7 30
x

a = 4, b = 4./3 and c = 3 (x - 7) - (x + 4) = -
11
(x + 4) ( - 7) 30
=>
x
Now, discriminant, D = bl - 4ac
x - 7 -x - 4 - - 11
= ( 4./3 ) 2 - 4 x 4 x 3 = 48 - 48 = 0 =>
x2 - 7x + 4x - 28 30
As D = 0, so roots exist and therefore

x - - b ± JD = -- -b ± O =>
-11 =-
x 2 -3x - 28 30
1 1 => -1 = -1
x 2 - 3x - 28 30
2a 2a
-4-/3 ± 0 -.J3 ± 0 --
-:J3 or --
--13
=> - 30 = x 2 -3x - 28
--- = => xl - 3x + 2 = O
2x 4 2 2 2
On comparing with ax2 + bx + c = 0, we get
--13 and --.
Hence, the roots are -- --13
2 2 a = l, b = - 3 and c = 2
(iv) Given equation is 2xl + x + 4 = 0 Now, x =
- b -+ �b2 - 4ac [by quadratic formula]
On comparing with axl + bx + c = 0, we get 2a �-----

a = 2, b = 1 and c = 4 �( - 4 (1) (2)


= - (-3) ± 2-3)2
Now, discriminant, D = bl - 4ac xl
= (1)2 - 4 x 2 x 4 = 1 - 32 = - 3 1 < 0 3 ± F8
As D < 0, so roots does not exist. 2
3 . Find the roots of the following equations
3 ± .Jl = 3 ± 1
= --
( ')
X - -1 = 3, X *- 0 ( ") 1 1
, X *- - 4, 7
2 2
l 11
11
-- - -- =-
x + 4 x - 7 30
x
3 + 1
x = -- or x = -- 3 -1
2
(t)
2
Sol. Given equation is 4 2
x = -2 or x = -2 => x = 2 or x = 1
x - -1 = 3, x ;1:. 0
x . 1 1 11
Hence, the roots of equanon -- - -- = - are
xl
-- - 1 = 3 => xl -3x - 1 0
= x + 4 x - 7 30
x 2 and 1. •' _.:::...

On comparing with axl + bx + c = 0, we get


Note See Q. Nos. 4 to t t of NCERT Folder ac page 2 8 . 29 and
3 0 and 3 t .
a = l, b = - 3 and c = -1
NCERT FO L D E R 1 . 4
]
1 Find the nature of the roots of the following Now, discriminant,
D = b2 - 4ac = (-6) 2 - 4 (2) (3)
0
quadratic equations . If the real roots exist, then

0 0 = 36 - 24 = 1 2 >
find them.
3x + 5 (ii) 3x2 hlJx + 4 = cwo

-
(i) 2x2 =- - Hence, che equation 2x2 - 6x + 3 = 0 has
(iii) 2x2 6x + 3 = 0
3x
distinct real roots, which are given by
- b ± .JD 6 ± ./12
ax2 c 0,
Sol. (1) Given equation is 2x 2 + 5 =0 -
x= = ---

2a 4
a - c
On comparing wich + bx + = we get
= 2, b = 3 and = 5 6 ± 2./3 3 ± ./3 = 3 + ./3 or 3 - .J3
--- --- ---

-4ac =(-3)2 -4
4 2 2 2
.
Now, discriminant,
D = b2
-40 3 -0 0
(2) (5) 3 + ./3 3 - .J3
Hence, the real roots are and
=9 =- 1 < 2 2
Hence, the equation 2x 2 3x + 5 = has no real 2 Find the values of k for each of the following
roocs. quadratic equations, so that they h ave two
(ii) Given equation is equal roots .

ax2 0, (i) 2x2 + kx + 3


3x2 - 4 .J3x + 4 =O
-
. . . (i) =0
(ii) kx (x 2) + 6
a 3, c 4
On comparing with + bx + c = we gee =0

-4ac
c 3.c
= b = - 4.J3 and = Sol. (t) The given equation is 2x1 + kx + 3 = 0

(-4./3)2 -4 (3)(4) 48 -48


Now, discriminant, D = b2 On comparing with ax1 + bx + = 0 ,we get
a = 2, b = k and =
3x2 -4..J3x 4 0
= = =O
Hence, the equation + = has two Now, discriminant, D = b1 - 4ac
0 0
equal and real roots, which are given by = (k )1 - 4 x 2 x 3 = k2 - 24
For equal roots , D = => k1 - 24 =
x = -b ± .JD
a
---
=> k1 = 24 => k = ± .J24 =:> k = ± 2../6

x = -(-4.J3)±0 = ---4..J3 ±0
2
-----
{it) The given equation is
0,
kx (x - 2) + 6 = which can be written as

4.J3 + 0 or x = 4..J3 -0
2x 3 6
c 0,
kx2 - 2kx + 6 = O

c
x=
6
---

6 On comparing with ax1 + bx + = we get


a = k, b = - 2k and = 6
3 3
2 .J3 2FJ
::::::) x= or
-- -- Now, discriminant, D = b2 - 4ac
2 2
= (- 2k)2 - 4 x k x 6 = 4k2 - 24k = 4k (k - 6)
::::::) J3 or -
x=- J3 For equal roots, D = 0
� 4k (k - 6) = 0
2 2 �
Hence, che equal roots are J3 and J3" k = 0 or k = 6
But k = 0 does not satisfy the given equation, because
(iii) - 6x + 3 = 0 if k = 0, chen coefficients of x 1 and x become zero.
ax2 + bx + c = 0, we gee
Given equation is 2x2

2, b = 6 and c = 3
On comparing wich Hence, required value of k is 6.
a = - Note See Q . Nos. 3 to 5 of NCERT Folder ac page 3 t a n d 3 2 .
TO P I C EXERCISE
Very Short Answer Type Questions Short Answer Type I I Questions
1 Write the discriminant of .J3x2 + 2.J2x - 2.J3 = 0. 8 Find the roots of x2 - 4ax + 4a2 - b 2 = 0 , if they exist
2 Determine whether the roots of the following by method of completing the square.
quadratic equation are real or not, 9 Using method of completing the square, find the
.J3x2 + 2.J2x - 2.J3 = 0 roots of quadratic equation .J3x 2 + 10x + 7.J3 = 0

3 Find the nature of roots of quadratic equation 10 Determine the roots of the following quadratic
3x2 - 2.J15x + 5 = 0 equation, if they exist, by the method of completing
square
4 Determine the discriminant of the following
quadratic equation x2 - (J2 + 1)x + .J2 = 0
13x
1 2x 2 + 3
1= Long Answer Type Questions

Short Answer Type I Questions


Find the roots of the following (11 to 13 ) quadratic

5 Find the value of K for which following equation


equations by applying the quadratic formula :

has real and equal roots 11 abx2 + (b2 - ac)x - be = 0, a. b :;t: 0

x2 - 2x( 1 + 3 K ) + 7(3 + 2K) = 0 12 3a2 x2 + Babx + 4b2 = 0, a :;t: 0


6 Determine the value(s) of m for which the equation
x2 + m ( 4x + m - 1) + 2 = 0 has real roots. 13 If roots to the equation
7 For what value(s) of n, the equation (c2 - ab)x2 - 2(a2 - bc)x + b2 - ac = 0 are equal.
-
(n + 3)x2 - (5 n)x + 1 = 0 has coincident roots? then prove that either a = 0 or a3 + b3 + c3 = 3abc.

Answers
1 . 32 2 . Yes 3. Real and equal 4. 25 5. 2 or -
-10

2
9
6. m < - - 1 or m � - 7. 1 or 13
3
8. 2a - b , 2a + b 9. --vr;;
" or -7
.J3 1 0 . .J2 or 1
b 1 2 -2b -2b
11. �. - . .
b a a 3a

TO PI C 8 Word P roblems and Its Solutio n


Quadratic equation comes up, when we deal with many real (Step II J According to the given condition, write the
life situations. The solution of such quadratic equation gives equation in the quantities which is obtained from
us the answer(s) to the problem. In this topic, we will discuss Step I.
some simple applications of quadratic equation.
(Step I I I J Simplify the equation obtained i n Step II to get
Method to Solve Word Problems quadratic equation.
To solve such problems, working steps are given below Now, simplify the quadratic equation by any o ne
[Step I ) First, consider the first quantity as a variable of the methods i.e. factorisation, completing the
(say x) and then find the other quantities in terms of square and quadratic formula and get the value(s)
x by using the given condition. of variable x.
All{nOne MATHEMATICS Class 1 0th Term 1 1

(Step V) Further check whether the value(s) of variable x,


+ 12
1 Ox + - 36 = 1 0 x
12
- + x
satisfies the given condition or not.
x x

(Step VI) Find the values of the quantities. Also, find the ::) Ox + - + 36
1
12
."(
=
1 20
-

x
+ x

values of other information, if asked in the


+---+ 36 = 0 => 9x -
12 1 20 1 08
question. ::) 1 Ox - x
x x
-
x
+ 36 = 0

Different Types of Word Problems => +


9x2 - 1 08 36x = 0 [multiplyi ng by -")
Here, various types of problems occurs, which can be => + xi 4x - 1 2 = 0 [dividing boch sides by 9]
understood with the help of the following examples. which is the required quadratic equacion.

[Type I) Word Problems Based on Numbers


By factorisation, we get
xi + 6x - 2x - 1 2 = 0
Example 1 . The sum of the squares of three =>
=>
++
x(x 6) - 2(x 6) = 0
(x 6)(x - 2) = 0
+
consecutive natural numbers is 1 1 0. Determine .the
numbers.
=>
::) +
x 6 = 0 or x - 2 = 0
x = - 6 or x = 2
Sol. Let three consecutive natural numbers be x, x + 1 and x + 2. But a digit can never be negative. So, x = 2.
According to the question, Hence, the required two-digit number
x2 + + + + 2)2
(x 1)2 (x = 1 10
= 10 x 2 + � = 20 + 6 = 26
=> x2 + x2+ 2x + 1+2 + x + 4x 4 = 1 10 2
[·: (a + b)2 a 2 + 2ab + b2]
= Example 3. "John and Janvi togeth er have
=> 3x + +
2 6x ::) 3x2 +
5 =1 10 6x - 1 05 = 0 45 marbles. Both of them lost 5 marbles each a n d the
::) 3(x + - 35)
2 2x =O product of the n um ber of marbles they n o w h a ve, is
::) x2 + 2x - 35 = 0 [·: 3 * O] 1 24. Find out how many marbles they had to start

I
which is the required quadratic equation. with ?"
Now, by factorisation, we get Here, we have to find the number of marbles, so we assume
+
x2 7x - 5x - 35 = 0 that one of them (John or Janvi) has x marbles and then

++ + ::) +
apply all the conditions to get required quadratic equati o n .
::) x(x 7) "'."" 5(x 7 ) = 0 ::) (x 7)(x - 5) = 0
=> x 7 = 0 or x - 5 = 0 x = - 7 or x = 5 Sol. Given, John and Janvi together have 45 marbles.
But x * - 7 because x is natural number. Let John has x marbles.
.. x =5 Then, number of marbles Janvi had = 45 - ·x
Hence, the three consecutive natural numbers are 5, 5 + 1 and · : Both of them lost 5 marbles each.
5 + 2 i. e. 5, 6 and 7. :. The number of marbles John had = x 5 -

, and the number of marbles Janvi had = 45 - x - 5 = 40 - x

Example 2. 'A two-digit n umber is such tha t the Now, product of the number of marbles = 1 24
product of the digits is 12 . When 36 is added to the .. (x - 5) ( 40 - x) = 1 24
n umber the digits interchange their places. Fin d. the => 40x - x2 - 200 5x = 1 24 +
=> 2+
I
two- digit n umber. -x 45x - 200 - 1 24 = 0
Here, we have to find out a two-digit number, so we assume => -x 2+ 45x - 324 = 0
ten 's digit of the number as x and then apply all conditions to
=> 2 +
x - 45x 324 = 0 [ multiplying by ( l ....,_

get required quadratic equation.


which is the required quadratic equation.
)]
Sol. Let the cen,s digit of the number be x. Now, by factorisation method, we get
According ro the question, x2 - 36x - 9x + 324 = 0
Product of the digits = 1 2
=> x(x - 36) - 9(x � 36) = 0
i.e. Ten,s digit x Un i es digi t = 1 2
=> (x - 36)(x - 9 ) = 0
=> Unit s d igu
, " . =- 12 .
. [·: ten ,s d"1g1t = x ] � x - 36 = 0 or x 9 = 0 -
x
x = 36 or x = 9
+ 12

Two-digir number = 1 0x when John has 36 marbles, then Janvi has
x
= 45 - 36 = 9 marbles.
Also, it is given that if 36 is added ro the number, the digits get
when John has 9 marbles, then Janvi has
in terchange.
= 45 - 9 = 36 marbles.
Quadratic Equations

[Type II] Word Problems Based on Age [Type III) Word Problems Based on Time,
Distance and Speed
Example 4. The sum of the reciprocals of Anjali's
Exam p le 6. A passenger train takes 3 h less for
·
age 3 yr ago and 5 yr from now is .!.. Find the present
3 a journey of 360 km if its speed is increased by
,

age of Anjali. 10 km/h from its usual speed. Fin d its usual speed.
Sol. Lee present age of Anjali be x yr. Here, we have to find out the usual speed of the train , so we

--
:. Anjali's age 3 yr ago = (x - 3) yr assume it as x km/h and then translate the word problem into
and Anjali's age 5 yr from now = (x + 5) yr symbolic form leading to a quadratic equation by using the
According co che question, formula : Time = Distance
1 1 1 Speed
+ -- = -
x -3 x + 5 3 Sol. Let the usual speed of the train = x km/h.
x + S + x -3 1 . Distance
=> =- . the formula, Time
Then, by usmg = • we get
(x - 3) (x + 5) 3 Speed
2x + 2 360
=
3
=>

(
x2 - 3x + 5x - 1 5 Time taken by the train = h [given, distance = 360 km]
x

J
=> 3(2x + 2) = x2 + 2x - 1 5 If speed is increased by 1 0 km/h, chen the new speed of the
6x + 6 = x2 + 2x - 1 5 train = (x + I 0) km/h.
360
=> x2 + 2x - 1 5 - 6x - 6 = 0 :. Time caken by the train = h
x + IO
=> x 2 - 4x - 2 1 = 0,
According to the question,
which is the required quadratic equation. 360 360
Now, by factorisation method, we gee = +3
x x + lO
x2 - 7x + 3x - 2 1 = 0
360 360
=> x(x - 7) + 3(x - 7) = 0 => - =3
x x + lO
=> (x - 7)(x + 3) = 0 360 (x + 1 0 - x)
x - 7 = 0 or x + 3 = 0 =3
x(x + 1 0)
=> x = ?or x = -3
360 x 1 0
Bue x = -3 is not possible because age cannot be negative. => = x(x + I O) => 1 200 = x 2 + ! Ox
:. x = 7 3
Hence, Anjali's present age is 7 yr. => x2 + l Ox - 1 200 = 0
[by factorisation]
Example 5. One year ago, a man was 8 times as old
=> x 2 + 40x - 30x - 1 200 = 0

as his son. Now, his age is equal to the square of his => x(x + 40) - 30(x + 40) = 0
=> (x + 40)(x - 30) = 0
son's age. Find their present ages.
=> x + 40 == 0 or x - 30 = 0
Sol. Let present age of his son = x yr => x = - 40 or x = 30
One year ago, his son's age = (x - 1) yr. Bue speed cannot be negative.
One year ago, man's age = B(x - 1) yr = (Bx - B) yr .. x = 30
Present age of man = (Bx - B + 1) yr = (Bx - 7) yr Hence the usual speed of passenger train is 30 km/h.
According to the question,
Bx - 7 = x2 => x2 - Bx + 7 = 0 Example 7. The speed of a boat in still water is
15 km/h. It can go 30 km upstream and return
which is the required quadratic equation.
downstream to the original point in 4 h and 30 min
Now, x2 - 7x - x + 7 = 0 [by factorisation]
Find the speed of stream .
x(x - 7) - l(x - 7) = 0

-
=>
=> (x - 7) (x - 1) = 0 Here, we find out the speed of the stream . So, w e assume it
as x km/h and after that we use the condition that, if speed of

=�
x - 7 = 0 or x - 1 = 0
boat in still water is a km/h (say) , then speed of boat
=>
:::::::> x = 7 or x = 1
upstream = (a x) km/h and speed of boat downstream
But x = I is not possible because if x = 1, then present age of the
= (a + x) km/h.
son and father are same. So, x = 7.
Now, we use Time
.

D stan e
and apply the condition given in
Hence, present age of his son = 7 yr and present age of man pee
= B x 7 - 7 = 49 yr. the question to reach the quadratic equation .
\ 20 \ Allinone MATHEMATICS Class 1 0th Term I I

[ ]
,

[ ]
Sol. Let speed of che scream = x km/h 40 1 + 1 = I
Given, speed of boat in still water = 1 5 km/h Ll -; x + 3
. ·. Speed of boat upstream = (1 5 - x) km/h
and speed of boat downstream = (1 5 + x) km/h
=> 4 o x + 3 + ·"" = 1 3
x(x + 3)

[·:
=>
According to che question,
40( 2x + 3) = l 3x(x + 3 )
� + � = 4 .!_ => S Ox + 120 = 1 3x2 + 39x
15 -x 1 5 + x 2

( 4 + :�) ( 4 + �) 4H
13x2 + 39x - SOx - 1 20 = 0
di·seance = 30 km and
distance =>
and also, 4 h 30 mm
.
speed => 1 3x2 - 4 lx - 1 20 = 0
=> 13x2 - 65x + 24x - 1 20 = 0
= h= h=
l 3x (x - 5 ) + 24 (x - 5 ) = 0
30(1 5 + x) + 30(1 5 - x) 9 => (x - 5)(1 3x + 24) = 0
=> =>
(1 5 -x)(l 5 + x) 2 x - 5 = 0 or 1 3x + 24 = 0
x = 5 or x = --24

[·;
450 + 30x + 450 -30x = -9
=>
=>
(1 5)2 -x2 2 13
·: Time cannot be negative.
( A -B)(A + B) = A2 - B2] .. x =5
=>
900 = -9
--- Hence, faster pipe takes 5 min to fill the cistern while slower
225 -x2 2 pipe takes (5 + 3) = 8 min to fill the cistern.
=>
900 x 2 = 225 _xz
9 (Type V) Word Problems Based o n Geometry
=> 200 = 225 - x2 Example 9. The hypoten use of righ t angled tria n gle
x2 - 225 + 200 = 0 => x2 - 25 = 0 is 6 m more than twice the shortest side. If the third
x2 = 25 => x = ± 5 side is 2 m less than the hypoten use, then find all
[on taking square root both sides] sides of triangle. CBSE 20 1 2, 1 1
But speed cannot be negative. x = 5
Hence, speed of stream is 5 km/h.
Sol. Lee length of the shortest side = x m . Then, hypote n use
=(2x + 6)m and third side = (2x + 6 - 2)m = (2x + 4 ) m
(Type IV) Word Problems Based on By Pythagoras theorem,
Work and Time
(2x + 6)2 = x2 + (2x + 4)2 [·: H 2 == P2 + 8 2 ]
=> 4x2 + 24x + 36 = x2 + 4x2 + 16x + 16
Example 8. 1\.vo pipes running together can fill a [ ( a + b )2 = a 2 + 2ab + b2]
cistern in 3 _!_ min. If one pipe takes 3 min more than => x2 + 4x2 + 16x + 1 6 - 4x2 - 24x - 36 == O
13
=> x2 - Bx - 20 == O
the other to fill it, then find the time in which each
pipe would fill the cistern. By quadratic formula,

�:: - 4ac ;
Sol. x = (-8) ± �(
- -8)2 - 4 x 1 x - 20

[·:
x min to fill the cistern.

"'-�\)]
Let faster pipe takes
2xl
Then, slower pipe will rake (x + 3) min to fill che cistern.
Since, portion of the cistern filled by the faster pipe in x
=
-b ± here a = I, b = - B a nd c
. 1
J mm = -

8 ± J64 + 80 => x = --
x
8 ± .Ji44 -
. . Portion of the cistern filled by rhe faster pipe in 3 _!__ m in => x=
13 2 2
8 + 1 2
=>
I I 40
=3- x - = -
8 ± 1 2
x = -- => x =
8 -12
or x = --
--

13 x 1 3x 2 2 2
_!__ min 20 -4
Si m i larly, ponion of the cistern filled by slower pipe in 3 x = - or x = -
13 2 2
40 I 40
= - x -- = --- => x = IO or x= -2
13 (x + 3) 1 3(x + 3) But length of side cannot be negative.
Acco rding to the q uestion,
40
.. x = 10
+
40 =I Hence, shortest side is I 0 m, hypo ten use is 2 x I 0 + 6 ==
2 6 Ill
+ 4 = 24 m
-

1 3x 1 3(x + 3) and third side = IO 2x


Quadratic Equations 21

[Type VI) Word Problems Based on Sol. Lee che cosc price (CP) of che coy = � x
Then, gain % =cosc price [given]
Mensuration
Gain o/o = x
Example 10. A rectangular park is to be designed Garn. = ,,,,' Gain 1%00 x CP
whose breadth is 3 m less than its length . Its area is to
be 4 sq m more than the area of a park that has Gain = � x100
x x
already been made in the shape of an isosceles
,

triangle with its base as the breadth of the rectangular


= � -=-=-
park and of altitude 12 m. Find its length and breadth 1 00
of the rectangular park. CCE 201 6 Now, SP = CP + Gain [·: SP = � 24)
x2
Sol. Lee breadch of reccangular park = x m 24 = x + -
Then, lengch of reccangular park = (x + 3) m 1 00
Now, area of reccangular park = x(x + 3) = (xi + 3x) m 2 => 24 = 100+ x 2
100x
[·: area =lengch x breadch] => 2400 = I OOx + x 2
Given, base of criangular park
= Breadth of the reccangular park => 2
x + 100x - 2400 =0
:. Base of criangular park = x m => x 2 + 2 x 50x + 2500 - 2500 - 2400 =0
and also ic is given chac altitude of criangular park = 12 m [by complering rhe square merhod]
. . Area of criangular park = .!._ x x x 12 = 6x m 2 => (x + 50 ) - 4900 =0
2 [·: a 2 + 2 a b + b2 = ( a + b)2 ]
2 => (x + 50) 2 = 4900
[·: area of criangle = .!._ x base x alcirude] => x + 50 = ± 70
2
According co che quescion, [on caking square roor boch sides]
Area of reccangular park = 4 +Area of criangular park => x + 50 = 70
=> xi + 3x = 4 + 6x or x + 50 = 70 -

=> x = 70 - 50
=> xi + 3x - 6x - 4 = 0 => x 2 - 3x - 4 = 0
or x = - 70 - 50
On comparing wich ax 2 + bx + c = 0, we gee => x = 20
a = 1, b = -3 and c = - 4 or x = - 120
Now, discriminanc, D = b i - 4ac = (-3) 2 - 4(1) (-4 ) Bue cosr price cannor be negacivc.
=9 + 16 = 25 > 0 .. x = 20
So, roocs exisc. Hence, che cosr price of che coy is � 20.
Now, on puccing the values of a, b and D in quadracic formula,
-b ± .fD , we gee x = -(-3) ± fi5 = 3 ± 5 .

x=
2a 2(1)
--

2 TRY YOURSELF
3+5 3 -5
x = -- or x =
=>
2
--

2 Q. 1 The sum of a number and its reci procal is 2 �. find the

[ 2-J
30
8 2
=> x = - or x = - -2 => x = 4 or x = -1 � and
number.
2 5 6
Ans.

Since, breadch cannoc be negacive, so neglecc x = - 1.


Q. 2 The p roduct of Ram ' s age 5 yr back and 9 yr hence ( i n yr)
[Ans. 6 yr I
.. x=4
Hence, breadth of che reccangular park = 4 m is 1 5. Find the present age of Ram.

and lengch of che reccangular park = x + 3 = 4 + 3 = 7 m Q. 3 The perimeter of a right angled triangle is 70 u n its and its

( Type VII) M iscellaneous Word Problems


hypotenuse is 29 u n its we would l i ke to f i n d the length of
the other sides. lAns. 20 unirs and 2 1 un irs]

Ex am p le 11 . A dealer sells a toy for � 24 and gains Q. 4 A person on tour has � 360 for his ex p enses. If h e exte n d s
his tour for 4 days , then he h a s to cut d ow n h i s dai ly
as much per cent as the cost price of the toy. Find the
expenses by � 3. F i nd the original d u ration of the t o u r.
cost price of the toy. [Ans. 20 days l

Here , we have to find out the cost price (CP) of the toy. S o, Q. 5 In a f l ig ht of 600 km , an ai rcraft was s lowed down d u e to
bad weather. Its average speed for the trip was red uced by
we assume it � x and use the formulae,
200 km/h and the time of fl ight i ncreased by 30 m i n . F i n d
G ain % x CP
an d S P = CP + G a1n ! Ans. l h 1·
. .
G a1n = the duration o f t h e flight.
1 00
NCERT FO LD E R 1 .1 . .. ,
I

2 Represent the following situations in the After three years,


form of quadratic equations. Rohan's age = (x + 3) yr
(i) The area of a rectangular plot is 528 m2 • The Rohan's mother's age
length of the plot (in metres) is one more than = [(x + 26) + 3 ] yr = (x + 29 ) yr
twice its breadth. We need to find the length According to the questi o n ,
and breadth of the plot. (x + 3 ) (x + 29) = 360
(ii) The product of two consecutive positive => 2
x + 29x + 3x + 87 = 360
=>
integers is 306. We need to find the integers.
x 2 + 32x - 273 = 0,
(iii) Rohan's mother is 26 yr older than him. The
product of their ages (in yr) 3 yr from now will which is the required quadratic equation.

[ ]
be 360. We would like to find Rohan's present (iv) Lee che speed of the train be x km/h.
age. Distance travelled by the train = 480 km
(iv) A train travels a distance of 480 km at a Therefore, time taken for travelling 480 km
uniform speed. If the speed had been 8 km/h 480 dista nce
= h ·: time =
less, then it would have taken 3 h more to x spee d
cover the same distance. We need to find the
speed of the train. If the speed had been 8 km/h less, i.e. (x - 8 ) k m / h ,
(t) Let the breadth of the plot be x m.
then
Sol. 480
Then, the length of the plot be ( 2x + 1 ) m. Time taken for travelling 480 km =
x -B
h
[by given condition]
According co the question,
·: Area of the ·rectangular plot = Length x Breadth
480 480 => 480 480
.. 528 = ( 2x + I ) x [given] =3 + - =3
=>
x -8 x x - 8 x
528 = 2x 2 + x
=> 2 480 x - 480 (x - 8 )
2x + x - 528 = 0, =3
x (x - 8)
which is che required quadratic equation.
(it) Let the two consecutive positive integers be x and
480x - 480x + 3840 = 3x (x - 8 )
x + 1. Then, according to the question, 3840 = 3x 2 - 24x
x (x + I ) = 306 3x 2 - 24x - 3840 = 0
=> x 2 + x = 306 => x 2 + x - 306 = 0, 3 (x 2 - Bx - 1 280 ) = 0
which is the required quadratic equation. => x 2 - Bx - 1 280 = O
(iii) Let the present age of Rohan be x yr. [o n divid ing by 3 both sides]
Then, his mother's present age = (x + 26) yr which is the required quadratic equation .
[by given co nditi on ]

NCERT FO L D E R 1 .2 I
-j

3 Find two numbers whose sum is 27 and => 27x - x 2 = 1 82 => x2 - 2 7x + 1 8 2 = 0


=>
product is 1 82 . 2 4
x - 1 x - 1 3x + 1 82 = 0
Sol. Ler o n e num ber be x. - 2 7]
[ ·: ( - 1 4 ) x ( - 1 3 ) = 1 82 and - 1 4 - 1 3 =
Then , anorher num ber be 27 - x => x ( x - 1 4 ) - 1 3 (x - 1 4 ) = 0 => ( x - 1 4 ) ( x - 1 3 ) ::: 0

[·: sum of rwo numbers = 27] => x - 1 4 = 0 or x -13 = 0


According ro rhe quesrion, => x = 1 4 or x =13
Producr of rhese rwo num bers = 1 82 If x = 1 4, then 27 - x = 27 - 1 4 = 1 3
x(27 - x) = 1 82 and if x = 1 3, then 27 - x = 27 - 1 3 = 1 4
Hence, in both cases, the numbers are 1 3 and 1 4 .
Quadratic Equations 23

4 . Find two consecutive positive integers, sum of 6 A cottage industry produces a certain number
whose squares is 365. of pottery articles in a day. It was observed on a
Sol. Lee che two consecutive positive integers be x and x + 1. particular day that the cost of pro duction o f
According co che question,
each article (in � was 3 more than twice the
number of articles produced on that day. If the
x 2 + (x + I ) .? = 365 total cost of production on that day was � 90,
=> x 2 + x 2 + 2x + 1 = 365 then find the number of articles produced and
[·: (a b + )2 = a l + 2ab + bl ] the cost of each article.
=>
l
2x + 2x - 364 = 0 Sol. Let th e number of porrcry articles produced on a particular
day be x.
=> x 2 + x - 1 82 = 0 [dividing by 2]
Then, cost of production of each ar t icl e = ( � 2x + 3 )
=>
2
x + 1 4x - 1 3x - 1 82 = 0
(by given condition]
[ ·: 1 4 x ( - 1 3 ) = - 1 82 and 1 4 - 1 3 = 1 ) So, rhe total cost of production = Number of porrery articles
=:) x (x + 1 4) - 1 3 (x + l 4) = 0 x Cost of production of each article
=:) (x + 1 4) (x - 1 3 ) = 0 = � x ( 2x + 3 )
=> x + 1 4 = 0 or x - 1 3 = 0 According co the question,
=:) x = - 1 4 or x = 1 3 x (2x + 3 ) = 9 0
Since, x is a positive integer, so neglect x = - 1 4. => 2x 2 + 3x = 90 => 2x 2 + 3x - 90 = 0
=:)
Therefore, we gee x = 1 3
2x l + l 5x - 1 2x - 90 = 0
Hence, che two consecutive positive integers are 1 3 and 1 4.
[·: 1 5 x ( - 1 2) = - 1 80 and 1 5 - 1 2 = 3 ]
5 The altitude of a right angled triangle is 7 cm =:) x ( 2x + 1 5 ) - 6 ( 2x + 1 5 ) = 0
less than its base. If the hypotenuse is 13 cm =:) ( 2x + l 5 ) (x - 6) == 0
long, then find the other two sides. =:) 2x + l 5 = 0 or x - 6 = 0
=>
Sol. Given, hypotenuse of right angled triangle = 1 3 cm 15
x = - - or x = 6
Lee t he base of the right angled triangle = x 2
Then, altitude of che triangle = (x - 7) cm Bur x cannot be nega t ive , as number of porrery articles
should be positive.
A
x =6
Hence, the number of articles produced = 6
and the cost of each article = 2 x 6 + 3 = � 1 5
(x - 7)cm
Additional Question .
7 In the centre of a rectangular lawn of
x dimensions 50m x 40m, a rectangular pond has
In !!:.A.BC, by Pythagoras theorem, to be constructed, so that the area of the grass
AC i = BC 2 + AB i surrounding the pond would be 1 1 84 m 2
=> (1 3 ) i = x i + (x - 7) l [see figure] . Find the length and breadth of the
NCERT Exemplar

!_-_-_- - -1
pond.
=> 1 69 = x i + x i - 14x + 49
[·: (a -b)2 a 2 . t. .

]
b2]
.
= - 2ab + : ·. ·

. . .. . .
. · .

- .-
·. · . .
.
:- _-- - .·-_-
=>
.

2x 2 - 1 4x - 1 20 = 0 ·
-. .
.
- _ _ _ _ _ _ _ _ _ _ -
_- _
_-
_
- .
..
. .. . ·. _-_-_-_-_- _- _ - _ - __
- - .· -

� = -: = =-- -:-�.:- - �
.

.-
=>
· - -� -
. . .. ... · --=== ==�====�==========� : =-�·. : == �- - � �= =
- - - --- -- - -_ - -_ ..
x 2 - 7x - 60 = 0 [dividing by boch sides 2] .
·_
·

=> x 2 - 1 2x + J;:i66�o
1
5 = -60

i
and - 1 2 + 5 = - 7
=> x (x - 1 2) +. 5 (x - 1 2) = 0
=>
t
Sol. Given, a x
'"=
(x - 1 2) (x + 5 ) = 0

j
=> x - q = 0 or x + 5 = 0 rectangular pond is

l
40 m
x =-5 constructed in rhe
=> x = 1 2 or !
x x
·-
centre of a - - ··
Since, base of the triangle cannot be negative, so neglect
rectangular lawn o f x
x = - 5.
dimensions SO m
50 m
Therefore, we get x = 1 2. x 40m . So, the '�
Hence, base of the triangle = 1 2 cm distance between
and altitude of the triangle = 1 2 - 7 = 5 cm pond and lawn wo uld ht' same around the pond. say x m .
24 AllinOne MATHEMATICS Class 1 0th Ter m I I

Clearly, length of rectangular lawn (� ) = 50 m :::::> 4x 2 - 1 80x + 1 1 84 = 0


and breadth of rectangular lawn (b1 ) = 40 m ::::) x 2 - 4Sx + 296 = 0 [dividing both sides by 4)
: . Length of rectangular pond (/2) = SO - ( .'\: + x) = 50 - 2x 2
and breadth of rectangular pond (b2 ) = 40 - (x + x )
::::) x - 37x - Bx + 296 = 0 [by facciorisacion)
:::::> x (x - 37 ) - 8 (x - 37 ) = 0
= 40 - 2x
:::::> ( x - 37 ) (x - 8 ) = 0 :::::> x - 37 = 0
2
Also, it is given that, the area of grass surrounding the pond
:::::> x = 37 or x - 8 = 0 :::::> x = 8
= l 1 84 m
Since, at x = 37, length and bread ch o f pond arc -24 and
: . Area of rectangular lawn - Area of rectangular pond -34, respectively but length and bread ch cannot be negative.
= 1 1 84 m 2 So, x = 37 cannot be possible.
[·: area of grass surrounding che pond = Area of rectangular :. Now, length of pond = SO - 2x = 50 - 2 (8 )
lawn - area of rectangular pond) = S0 - 1 6
=> l, x b, - [2 x b2· = 1 1 84 = 34 m
[·: area of rectangle = length x breadth) and bread ch of pond = 40 - 2x = 40 - 2 ( 8 )
=> 50 x 40 - (50 - 2x) (40 - 2x) = 1 1 84 = 40 - 1 6 = 24 m
=> 2000 - ( 2000 - 80x -1 00x + 4x ) = 1 1 84
2 Hence, required length and breadth of pond are 34 m and
:::::> 80x + 1 00x - 4x 2 = 1 1 84
24 m, respectively.

NCERT FO L D E R 1 . 3
4 The sum of the reciprocals of Rehman's ages, When x = 1 2 marks in Mathematics, then marks in English
{in years) 3 yr ago and 5 yr from now is .!. . Find
= 30 - 1 2 = 1 8.
3 When x = 13 marks in Mathematics, then marks in Englis h
his present age. = 30 - 1 3 = 1 7.
Sol. Do same as Example. 4 at page 23. [Ans. 7 yr) 6 The diagonal of a rectangular field is 60 m m o re
than the shorter side. If the longer side is 3 0 m
5. In a class test, the sum of S hefali's marks in


more than the shorter side, then find the sides
Mathematics and English is 30. Had she got of the field.
2 marks more in Mathematics and 3 marks less
Sol. Lee PQRS be the rectangular S R
in English, the product of their marks would
have been 2 1 0 . Find her marks in the field, which has shorter side
QR = x m.
two subjects.
:.Diagonal of the rectangle, PR
Sol. Lee Shefali's marks in Mathematics be x. = 60 m more than the shorter Po
Given, che sum of Shefali's marks in Mathematics and side = (x + 60) m
English = 30 and longer side of the rectangle PQ
:. Shefali's marks in English = 30 - x = 30 m more than the shorter side = (x + 30 ) m
According co che quescionl In right angled llPQR, by Pythagoras theorem,
(Marks m Machemacics + 2) x (Marks m English -3) PR 2 = PQ 2 + QR 2
[·: in rectangle, every adjacent sides m akes at\
= 210
:::::> (x + 2) x [(30 - x) - 3 ] = 2 1 0
angle 90° co each oth er]
=> ( x + 2 ) (27 - x) = 2 1 0
--!l 2 + 54 - 2x = 2 1 0
=> (x + 60)2 = (x + 30)2 + x 2
=> 27x
=> x + 1 20x + 3600 = x 2 + 60x + 900 + x 2
2
x 2 - 25x + 1 56 = 0
x 2 - 1 3x - 1 2x + 1 56 = 0 [ ·: ( a + b)2 = a2 + 2ab + b 2 ]

[by faccorisacion] => (2x2 - x 2 ) + (60x - 1 20x) + (900 - 3600) = 0


=> x (x - 1 3) - 1 2 (x - 1 3) = 0 => x 2 - 60x - 2700 = O
=> (x - 1 3) (x - 1 2) = 0 => X2 - 90x + 30x - 2700 = 0 [ by factorisa tion]
=> x - I 3 = 0 or x - I 2 = 0 => x (x - 90) + 30 (x - 90) = O
=> x =13 => (x - 90) (x + 30) = O
Of x =12 => x - 90 = 0 o r x + 30 = 0
Quad ratic Eq uations 25

x = 90 or x = - 30 x 2 + 5x - 1 800 = 0
Negative value of x is not possible, because side cannot 2
x + 45x - 40x - 1 800 = 0 [by factorsacion]
be negative.
x (x + 45) - 40 (x + 45) = 0
x = 90
(x + 4 5 ) (x - 40) = 0
Hence, breadth of the rectangle = 90 m and length of the x + 4 5 = 0 or x - 40 = 0
rectangle = 90 + 30 = 1 20 m .
x = - 4 5 or x = 40
7 The difference of squares of two numbers Since, speed of train cannoc be negative.
is 1 80. The square of the smaller number is So, we neglect x = - 45.
8 times the larger number. Find the two x = 40
numbers. CBSE 20 12 Hence, che original speed of the crain is 40 km/h.
Sol. Lee che required numbers be x and y, where x > y. 9 Two water taps together can fill a tank in 9 �h
Given, difference of squares of cwo numbers = 1 BO
8
The tap of larger diameter takes 1 0 h less than
We have, x 2 - y 2 = 1 BO . . . (i)
the smaller one to fill the tank separately. Find
and also ic is given chat che square of smaller number the time in which each tap can separately fill
= B x Larger number the tank.
We have, y 2 = Bx . . . (ii)
Sol. Let che time taken by the larger tap co fill the tank
From Eqs. (i) and (ii), we gee =xh
x 2 - 1 BO = Bx Then, time taken by the smaller tap to fill the tank
=:> x 2 - Bx - 1 BO = 0 = (x + 1 0) h . [by given condition]
=:> x 2 - 1 Bx + 1 0x - 1 BO = 0 [by factorisation] . . Portion of the tank filled by the larger tap i n 1 h = ..!_
=:> x (x - 1 B) + I O (x - 1 B) = O x
=:> (x - 1 B ) (x + 1 0 ) = 0 and portion of che rank filled by che smaller tap i n I h
=:> x - 1 8 = 0 or x + 1 0 = 0 1

-- ----
=:> x = 1 B or x = - 1 0 x + IO
Now, if x = 1 8, chen square o f · smaller number
Portion of che tank filled by both the taps in 1 h
= B x I B = 1 44 [from Eq. (ii)]
1 I x + lO + x
=:> Smaller number = ± 1 2 =-+ =
x x + 1 0 x (x + 1 0 )
=:> Smaller number = 1 2 or - 1 2
and if x = - 1 0, chen square of smaller number 2x + 1 0 2(x + 5 )
= . . . (i)
= [B. x ( - I O)] = - BO, which is not possible as square of a x (x + 1 0 ) x (x + 1 0 )
number cannot be negative. According co che question, cwo caps together can fill che
Hence, che required numbers are I B and 1 2 or
tank in 9 1._ =
75
lB and - 1 2. h.
B B
8 A train travels 360 km at a uniform speed.

( �r
So, portion of the tank filled by both taps in I h
If the speed had been 5 km/h more, then it I 8
= . . . (ii)
would have taken t h less for the same journey. 75
Find the speed of the train.
Sol. Let che uniform speed of che train be x km/h. From Eqs. (i) and (ii) , we get

[ ]
360 2 (x + 5 ) 8 x+5 4
Then, time taken co cover 360 km = h and rime taken --=----
- - = - =:> 2 =
x x (x + 1 0 ) 75 x + l Ox 75
to cover 360 km, when the speed is increased by 5 km/h =:> 2
75x + 375 = 4x + 40x
360
·: speed
distance
= -- h 4x 2 + 40x - 75x - 375 = 0

-
=:>
=:>
x+5 time
Now, according to che question, 4x 2 - 35x - 375 = 0
360 360 On comparing with ax 2 + bx + c = 0, we get
= -1
x+5 x a = 4, b = - 35 and c = 375
360(x + 5 ) - 360x 1
= 1 =:>
360 360
_ - b ± �b2 - 4ac
=:> = .. x= [by quadratic formula]
x x+5 x(x + 5) 2a
�--------

=:> 360 (x + 5) - 360 x = x (x + 5) -( -35 ) �(-35 ) 2 - 4 x 4 x ( -37 5 )


=:> 360x + 1 800 - 360 x = x 2 + 5x 2x 4
26 I AllinOne MATH EMATICS Class 10th Term 1 1

= 35 ± �25 x 49 + 16 25 x 1 5
x x2 + 1 lx - 1452 = O
8 => x2 + 44x - 33x - 1452 = 0 [by factorisation]

--- --
= 35 ± �25 x 289 = 35 ± 5 x l7 = 35 ± 85 => x (x + 44) - 33 (x + 44) = 0
(x + 44) (x - 33) = 0

( )
8 8 8 =>
=> 35 + 85 35 -85 => x + 44 = 0 or x - 33 = 0
x= or x = => x = - 44 or x = 33
8 8
Since, speed can never be negative. So, x � - 44.
=> 20
x = l or x = -5o = -25 => x = 1 5 or x = - 6 .!.
8 8 4 4 Hence, the average speed of the passenger train
= 33 km/h
But negative value is not possible because time cannot and the average speed of the express train
be negative. = 33 + 1 1 = 44 km/h
So, x =15
11 Sum of the areas of two squares is 468 m 2 •
Hence, time taken by each tap to fill the tank separately are
1 5 h and 1 5 + 10 = 25 h. If the difference of their perimeters is 24 m,
1 0 An express train takes
then fin d the sides o f the two squares .
1 h less than a passenger
Sol. Let the length of each side of a square be Then, its x.
train to travel 132 km between Mysore and
Bengaluru (without taking into consideration
perimeter is 4x. [·: perimeter of a square = 4a]
the time, they stop at intermediate stations) .
It is given that the difference of the perimeters of the cwo
squares is 24 m.
If the average speed of the express train is :. Perimeter of second square = (24 + 4x)
1 1 km/h more than that of the passenger train, => Length of each side of second square
then find the average speed of the two trains.
Let the average speed of the passenger train be x km/h. = 24 + 4x = (6 + x) m
Sol. 4

]
It is given that the sum of the areas of two squares is 468 m

[
Then, the average speed of the express train 2•

= (x + 1 1) km/h .. x2 + (6 + x)2 = 468


Time taken by passenger train to cover 132 km x2 + (36 + 12x + x2) = 468
·: speed = dis�ance
=>
= 132 h => 2x2 + 12x - 432 = 0
x time
132
=> x2 + 6x - 2 1 6 = 0 [dividing both side by 2]
Time taken by express train to cover 132 km = h which is the required quadratic equation in x.
(x + 1 1) => x2 + 1 8x - l 2x - 2 1 6 = 0 [by factoris ation]
. to the quesnon,
According . -- 132 = -132 - 1 => x (x + 18} - 1 2 (x + 1 8} = 0 => (x + l S) (x - 1 2) = 0
·

x + 11 x => x + 1 8 = 0 or x - 12 = 0
=> 132 - � = l => 132(x + l l) -132x = I => x = -1 8 or x = 1 2
x x + 11 x(x + l l) But, negative value is not possible because side can never be
x
132x + 132 1 1 -132x = 1 negative.
=> So, x = 12
x2 + 1 1x Hence, side of the first square = x = 1 2 m

NCERT FO L D E R 1 .4
1452 = x2 + 1 1x and side of the second square = y = 1 2 + 6 = 1 8 m

- -- , - · - ..
- · ··--- ..,. _ -·-- · · ·- • _ ...
. ___ __ J '.r :.. �. .:... . I�.! :u�;..L: :.!..�!.:�1:�1.�_:;.,._�c._��ii:i.;;:i.!.l_,_\_...._ Ud � �L � --�
· .- ··· - · -- . · - · ..,,_ · - -· · - - --
.
... · · r�
- -- · - -- ... - - .- .- - · - · ·· - - - - -

·.
. .

--- · · · . . . ··.-JI
.--.

· ·-

3 Is it possible to design a rectangular mango => 2x2 = 800


grove whose length is twice its breadth and the => x2 = 400 => x = ± 20
area is 800 m 2 ? If so, then find its length But, x = -20 is not possible because breadth can never be
and breadth. negative. So, x = 20.
Sol. t
Le breadth of a rectangular mango grove = x m Thus, length = 2x = 4 0 m and breadth = 20 m .
Then, Jengrh of a rectangular mango grove = 2x m 4 Is the following situation possible? If s o , then
According to the question, determine their present a ge s .
Area of rectangular mango grove = 800 m 2
The sum o f the ages o f two friends i s 2 0 yr. Four
� 2x (x) = BOO years ago, the p roduct of their ages (in years)
[·: area = lengrh x breadth] was 4 8 " .
Quadratic Equations f 21
Sol. Let the age of one of two friends be x yr . Sol. Let the breadth of che park be x m.
Then, age of other friend = ( 20 - x) yr Given, perimeter of a rectangular park = 80 m
[·: the sum of the ages of two friends is 20 yr] => 2 (Length + Breadth) = 80 m
4 yr ago, age of one of two frie nds = (x - 4) yr => Length + Breadth = 40 m
and age of the other friend = ( 20 - x - 4) yr => Length = ( 40 - x) m
= (1 6 - x) yr Now, area of a rectangular park = Length x B readth
According co che question, = (40 - x)x m 2
(x - 4) (1 6 - x ) = 48 But according to the question, area of the rectangular park is
=> l 6x - x 2 - 64 + 4x = 48 400 m 2 •
x 2 - 20x + 1 1 2 = 0 ( 40 - x) x = 400
x 2 - 40x + 400 = 0
On comparing with ax2 + bx + c = 0, we gee
=> x 2 - 2x- x 20 + ( 20) 2 = 0
a = l, b = - 20 and c = 1 1 2
[by completing the square method]
Now, discriminant, D = b 2 � 4ac = (- 20) 2 - 4 x 1 x 1 1 2 2
=> (x - 20 ) = 0 => x = 20
= 400 - 448 = - 48 < 0 [·: a 2 - 2ab + b 2 = (a - b ) 2]
which implies that the real roots are not possible because this
condition represents imaginary roots. So, the solution does Thus, breadth of the park = 20 m
not exist and hence given situation is not possible. and length of the park = 40 - 20 = 20 m
Hence, it is possible to design the rectangular park having
5 Is it possible to design a rectangular park of permeter 80 m and area 400 m equal length and breadth,
perimeter 80 m and area 400 m 2 ? If so, then i.e. 20 m each.

TO P I C EXERCISE
find its length and breadth.

Short Answer Type I Questions 7 It the product of two consecutive odd numbers is
1 If two numbers differ by 2 and their product is 143, then find the numbers.
360, then find the numbers. [Hint Two consecutive odd numbers a re (2x - 1)
2 If the product of two consecutive natural
and ( 2x - 3 }]
numbers is 210, then determine the numbers.
Long Answer Type Questions
3 The sum of a numbers and its reciprocal is
17
. 8 A rectangular field is 20 m long and 14 m w ide.
4
There is a path of equal width all a round it.
Find the numbers.
having an area of 111 sq m. Find the width of the
Short Answer Type I I Questions path. CCE 20 13, 1 2
4 Two numbers differ by 3 and their product is 504. [Hint Area of path = Area of the field including
Find the numbers . the path - Area of the field excluding the path]
5 The product of two consecutive integers is 20.
9 The area of a right angled triangle is 480 cm 2 • If
Using quadratic equation, find the integers.
the base of the triangle is 8 cm more t ha n twice
6 Divide 29 into two parts, so that the sum of the
the height (altitude} of the triangle. then find t he
squares of the parts is 425.
sides of the triangle . CCE 20 1 3

Answers
1
1 . 18, 20 or -20, - 18 2. 1 4 an d 1 5 3. 4 or - 4. 2 1 , 24 or -24, -2 1
4
5. -5, - 4 or 4, 5 6. 1 6, 13 7. 1 1 , 1 3 8. 1 .5 m 9. 20 cm, 48 cm and 52 cm
SUMMATIVE ASS ESS M E N T
�I Very Short Answer Type Questi ons [ 1 M a rk ea c h ]

1. Ifb = 0, c < 0 , i s it true that the roots of 7. Find the roots of quadratic equation
x2 + bx + c = 0 are n{imerically equal and 2x2 - 6x = 0.
opposite in sign? Justify your answer.
NCERT Exemplar 8. What is the nature of roots of the quadratic
equations 5x2 - 2x - = ? 3
CCE 2 0 1 50
2. If the discriminant of the equation
6x2 - bx + 2 = O is 1 , then find the value of b. 9. What is the nature o f roots o f the quadratic
CBSE 201 2
equation 2x2 - ..JSx + 1 = 01 ·

3. Does there exist a quadratic equation whose NCERT Exemplar


coefficients are rational but both of its roots
are irrational? Justify your answer. 1 0. Which constantmust be added and
NCERT Exemplar subtracted to solve the quadratic e quation

9x2 + �x .J2 = 0
4. Check whether the following statement is 4
� by the method of

true or false. Justify your answer. completing the s quare ? NCERT Exemplar
"Every quadratic equation has . atleast one
real root. " 1 1 . lHOTSI A quadratic equation with integral
coefficients has integral roots . Justify your
5. I f the product of two consecutive integers is
answer. NCERT Exemplar
306, then write the quadratic representation
12 . lHOTSI Does there exist a quadratic e quation
of this situation.

6. If a numbe� is added to twice its square, then whose coefficients are all distinct
the resultant is 2 1 . Write the quadratic irrationals but both the roots are rationals?

�I Short Answer Type I Questi ons


representation of this situation. CCE 2014, 15 Why? NCERT Exemplar

[2 M arks eac h ]

13. Which o f the following i s not a quadratic 1 7. Find the roots o f the equation
equation? NCERT Exemplar ax2 + a = a2 x + x C BSE 20 1 2
(i) (..f2x + .J3Y. = 3x2 - sx
(ii) (x2 + 2xf = x4 + 3 + 4x2 1 8. Show that (x2 + 1)2 - x2 = 0 h a s n o rea. 1
roots . NCERT Exemp la.-
14. Solve for x : x 2 - (.Ji + 1) x + .J3 = 0
CBSE 201 5 1 9. Find the roots of the quadratic equation
cl b2 x2 + b2 x - a2 x - 1 = 0
1 5. : + 7
CBSE 201 2, I I
Solve for x .J6x - (2x - 7) =0
CCE 2016
20. = 13
= 4 x
Solve for x : �2 x + 9 +
1 6. Find the roots of the equation x + .Jx - 2 CCE 2016
Quadratic Equations 29

2 1. Find the numerical difference of the 25. Find the value of p, so that the quadratic
roots of equation x2 - 7x - 18 = 0 equation px (x - 3) + 9 = 0 has equal roots .
CCE 201 5 CBSE 20 1 4

22 . 26. Find the value o f k for which the quadratic


equation 2x2 - kx + k = 0 has equal roots .
Find the least positive value o f k for
which x2 + kx + 16 = 0 has real roots.
CBSE 2010 NCERT Exemplar

27. In a cricket match. Harbhajan took three wickets


2 3. Determine the value of k for which the less than twice the number of wickets taken by
quadratic equation 2x2 + 3x + k = 0 Zaheer. The product of the numbers of wickets
has real roots . taken by these two is 20. Represent the above
situation in the form of a quadratic equation .
24. I f .!. is a root of the equation CCE 201 5
2
-� =
2 8. [fil>TSJ Find the quadratic equation , i f

�5
x2 + kx 0, then find the value
4 x = �5 + . . . oo
+ �5 + and x is a natural

I Short A nswer Tvpe I I Questions


of k. NCERT Exemplar; CBSE 201 1
number.

[3 Marks e a c h ]

29. Find the roots of the following 36. The sum of two numbers i s 9 and the s u m of their
quadratic equation reciprocals is .!. . Find the numbers .
x2 - 3.JS x + 10 = 0 CBSE 201 1 2 CBSE 2009

30. Solve the following quadratic equation 37. At t min past 2 pm, the time needed by the
forx : 4.J3x2 + 5x - 2.JJ = 0 minute hand of a clock to show 3 pm was found
2
CCE 201 3, 1 2 to be 3 min less than !___ min. Find t.
NCERT Exemplar
4
31. Find two consecutive odd natural
numbers, sum of whose squares is 130.
CCE 2013 38. Seven years ago, Varun 's age was five times the
square of Swati ' s age. Three years hence ,
32. Find the roots of the following Swati ' s age will be two -fifth of Varun ' s age. Find
equation. their present ages .
1
- - -- =
1 3 ;x "# 0, 2 39. Find the roots of the equation
x x -2 a2x2 - 3 a bx + 2b2 = 0 by the method of
completing the square .
33. Solve for
16 - 1 15 ; x '¢ 0, - 1 40. The sum of a number and its positive square root
x :- = -- is � . Find the number.
x x+1 25
34. The difference of two numbers is 4. If 41. The numerator of a fraction is 3 less than its
the difference of their reciprocals is _i_ , denominator. If 1 is added to the denominator,
21 the fraction is decreased by _!_ , Find the fraction.
then find the two numbers. CBSE 2008 15

35; The sum of two number is 1 1 and the 42. Three consecutive natural numbers are such that
. .
sum of their rec1proca I s is -.
l l F'm d the
. the square of the middle number exceeds the
28 difference of the squares of other two by 60. Find
numbers . the numbers . CCE 20 1 6
CCE 201 3
30 l Allin.One MATHEMATICS Class 1 0th Term �
I

43. Find the roots of the following quadratic 45. fHQf_� If the roots of the eq uation
equation by factorisation. x2 + 2cx + ab = 0 are real and

x2 - 2 ax + a2 - b2 = 0 unequal, then prove that the equation

1 x2 - 2(o + b)x + o2 + b2 + 2 c2 = 0
44. !HOTS! Solve for x: = .!. + .!. + .!., has no real roots .
a+b+x a b x
where a, b, x -:;; 0 and a + b + x * 0

Long Answer Type Questions [ 4 M arks each]

46. Solve for x : 5 4. " 6500 were divided equally among a


1 1 2 certain number of persons . I f there had
----- + = - ;x *" 1 , 2, 3 been 15 more persons, each would have
(x - 1) (x - 2) (x - 2) (x - 3) 3 got " 30 less. Find the o riginal number
CCE 201 6
of persons . C C E 20 1 3, 12
47. Solve for x:
1 2 4
*"
55. A shopkeeper buys a number o f books
fo r t 1 200. I f he had bought 10 more
-- + = --, x -- - 1 , - 2, - 4
x +l x +2 x + 4
CCE 2016 books for the same amount, each b ook
48. Solve for x:
would have cost him � 2 0 les s . H ow
many books did he buy? C B S E 2012
x -4 x 6 10 -

-- + -- = - , x * 5, 7
x -5 x -7 3 CCE 201 4 5 6. I f the equation (1 + m2 )x2 + (2 mc)x
+
(c2 - o2 ) = O has e qual roots , then
49. If - 5 is a root of the quadratic equation
prove that c2 = o2 (1 + m2 ).
2x2 + px - 15 = O and the quadratic equation
p(x2 + x) + k = 0 has equal roots, then find the 5 7. A train takes 2 h less for a journey of
value of k. CCE 2016 3 0 0 km , if its speed is incre a s e d by
5 km/h from its usual speed. Fin d the
50. If x = - 2 is a root of the equation usual speed of the train. C B SE 2 0 1 2
3x2 + 7x + p = 0. Find the values of k, so that
the roots the ofequation 58. A motor boat whose speed i s 2 4 km/h in
x2 + k (4x + k - 1) + p = 0 are equal. still water takes 1 h m ore to g o 32 km
CCE 201 5 upstream than to return downstream to
the same spot. Fin d the speed of the
5 1. Solve the quad ra tic equation stream. CCE 20 16

9x2 - 1 5x + 6 = O by method of completing the 59. Two pipes running together can fill a
s quare . CBSE 201 2
cistern in 1 1 .!. min . If one pipe t a kes
9
52. The perimeter o f a right angled triangle is 5 min more than the other to fill it . Fi nd
60 cm. Its h yp oten us e is 25 cm. Find the area of the time in which each pipe would fill
the t rian gle . the cistern. CCE 20 1 6

53. A p i ec e of cloth costs � 2 00. If the piece was 5 m 60 . A factory kept increasing its output by
longer and each metre of cloth costs � 2 less, the
the same percentage every year. Find
cost of the piece would have remained
the percentage, if it is known that then
unchange d . How lon g is the piece and what is
output is doubles in the last two years .
the original rate per metre? CCE 201 5 C CE 20 1 5, 13
Quadratic Equations

1
61. Solve x = ------- ; x * 2.
1
2 -
1
2 -
2 x -

62 . If the roots of the equation (a - b)x 2 + (b - c)x + (c - a) = 0 are equal, then prove that 2 a = b + c.
CCE 20 1 5

63 . O ne-fourth o f a herd of camels was seen in the forest. Twice the square root o f the herd had gone to
m oun tai n s and the remaining 15 camels were seen on the bank of a river. Find the number of camels.

64 . �t:i9f$� There is a square field whose side is 44 m . A square flower bed is prepared in its centre leaving a
gravel path all round the flower bed . The total cost of laying the flower bed and gravelling the path at
� 2 .75 and � 1 .50 per m 2 respectively, is � 4904. Find the width of gravel path.

Va l ue Based Question (VBQ) . (4 Ma rks]

65. Due to some technical problems, an aeroplane started late by one hour from its starting point. The
pilot decided to increase the speed of the aeroplane by 1 00 km/h from its usual speed to cover a
journey of 1 200 km in time.
(i) Find the usual speed of the aeroplane.
(ii) What values (qualities) of the pilot are represented in the question?

Sol u tio ns
1 . True, since the discriminant of quadratic equation S. Lee the two consecutive integers be x and (x + 1).
x2 + bx + c = 0 is b 2 - 4 (1 ) c > 0 Produce of two consecutive integers = 306 [given]
[·: b 2 - 4c = (0) 2 - 4c = -4c] => x(x + 1) = 306 [by given condition]
So, roots will exist and these are given by => x 2 + x = 306 => x 2 + x - 306 = 0
-b ± JD -0 ± .JD ±.JD Hence, che quadratic representation co this situation is
x = --- = ----

2 2 2 x 2 + x - 306 = 0 [1 ]
le is clear that, the roots of x 2 + bx + c = 0 are 6. Let the number be x.
numerically equal and opposite in sign. (1 ] According to the question,
2. Givcn, 6x - bx + 2 = 0
2 2x 2 + x = 2 1
We know that, => 2x 2 + x - 2 1 = 0, [1 ]
D = b 2 - 4ac [1 /2] which is the required quadratic representation.
1 = (-b ) 2 - 4(6)(2) [·: given, D = l] 7. Given quadratic equation is
1 = b2 - 48 => b 2 = 49 => b = ± 7 [1 /2] 2x 2 - 6x = 0
Hence, the value of b is 7 or -7. => 2x (x - 3) = 0
3. Yes, consider che quadratic equation => 2x = 0 or x - 3 = 0
2x 2 + x 4 = 0, which have rational coefficients.
-
=> x = 0 or x = 3
The roots of the given quadratic equation are Hence, roots of the given quadratic equation are 0
and 3. [1 ]
J33 .
-

8. Given quadratic equation is


1 + Iii
- - 1 - . .
---- and , which are mauonal. [1 ]
5x 2 - 2x - 3 = 0
4 4

On comparing with ax 2 + bx + c = 0, we get


4. False, (1 ]
e.g. The quadratic equation, x 2 +
a = 5, b = - 2 and c = - 3
4 = 0, has no real
roots. [1 /2]
32 ' Allin one MATHEMATICS Class 1 0th Term '

Now, discriminant, = b2 - 4ac


D (x - J3) - 1) = 0 (x
= (-2)2 4(5) (-3)= 4 + 60 = 0
- 64 > => x - .J3 = 0 - 1 = 0 or x
Hence, roots of the given quadratic equation are real => x = .J3 x = or 1
x J3 1. [1 J
and distinct. [1 /2] Hence, required values of are
9. Given equation is
and
2x2 - .JSx + 1 = 0 1 5. Given .J + 7 - (2x - 7) = 0
bx
6x
On comparing with ax 2 + + c = 0, we get
.J + 7 7
a = 2, b = .JS c = 1
- and [1 12]
6x = 2x -

D = b2 - 4ac On squaring both sides, we get


J + 7 = (2x - 7)2
Now, discriminant,
)2
= (- J5)2 - 4 (2) (1) = 5 - 8 = - 0
x x 3 <
=>
( 6x

+ 7 = 4x2 - 28x + 49
6x
Since, discriminant is negative, therefore quadratic

9x2 + �4 x - = 0
equation 2x2 - .J5x + 1 = 0
has no real roots, i.e. it => 4x2 - 2Sx + 49 - - 7 = 0 6x
imaginary roots. [ 1 /2] => 4x2 + 42 = 0 - 34x
1 0. Given equation is ./2. => 2 ( 2x2 - 1 7x + 21) = 0
� G �J= (�J
:::::> (3x)2 + 4(1 3x) - v2 = 0 r::
=> 2x2 - 7 = 0 [·: 2 O] (1)
1 x + 21 �

=> 2x2 - 14x - + 21 = 0 3x [by factorisation]


Now, coefficienr of(3x) = · [1/2] =>
=> (--x7 7)-= 07) (2x2 --- 3)7)3 === 000
2x (x - 3 (x

So, (.!.8 J2 = 64 _!_ must be added and subtracted to solve


=>
=>
x
x = 7 = -32
or x
or x
the given quadratic equation by completing the square
method. [1 /2] B ut x=2 -
3
does
. not sans
.
" fy the given equanon. .

1 1 . A quadratic equation with integral coefficients may :. Required value ofx is 7. [1 ]


not have integral roots. Consider the quadratic
1 6. Given equation can be written as
equation 4x2 -Bx + = 0, 3 which have integral
coefficients. The roots of the given quadratic equation
� = 4 - x
12 32 .
are - an d -, wh"1ch are not mtegers. . [1 ]
On squaring both sides, we get
(�)2 = )2 (4 - x

1 2. Yes, consider the quadratic equation =>


x2 - 9x + - 182
x
= 1
0 + x2 - Bx
= 6
.J3x2 - 7.J3x + 12.J3 = 0. [1 ]
which have distinct =>
irrationals coefficients. The roots of this quadratic
4,
equation are 3 and which are rational. [1 ]
=> x2 - - 3(x3x + 18 == 00
6x
=> x(x - 6)
-

- 6)
[by facto risa ti o n]

1 3 . (1) Given that, (


.fi. x + .J3)2 = 3x2 - 5x
·
=>
But x == 3(x - 3) (x - 6 ) =0 x =3
=>
6 does not satisfy the equat io n.
or 6

2 => + /6 =
· x2 3 + 2 · x 3x 2 - 5x :. x
(1 ]
1 7. Given, ax2 + a = a2x +
=> 2 + 2.J6 = 0,
x - (5 )x - 3
which represents a quadratic equation because it is => 2ax +a - a2x - = 0 x x
of the form ax 2+ bx + c = 0, a 0.� [1] => ax 2 a2x- + - =x0
ax - a)- -a)I. (ax - a)1) == 00a

(it) Given that,(x2 + 2x)2 = + 3 + 4x2 => (x (x -

x 4
=> (x
=> + 4x2 4x3 = + + 4x2
x4 + x4 3 => x - a = ax - I 0I 0 or =
=> 4x3 = 0 - 3 => = x a or x = -
a (1 ]
which is not of the form ax 2+ bx + c,is aacubic0. �
,
Hence, the required roots of the given eq u ation are tZ
Thus, the equation is not quadratic. This
equation. [1]
and-;;I · [1 /2]
1 4. Given , x 2 - (.J3 +l)x + .J3 =0 2 + 2
1 8. Given equation is (x - 1) x2 = O
=> x 2 - .J3 + .J3
x -x =0 => +I + - x1 = 0
x4 2x 2
=> x(x ./3) -- )3) = 1 (x - 0 [1] [·: (a + b = a1 + b1 + 2ab] )
2
Quadratic Equations f 33
x4
+ x2 + 1 = 0 [1 /2] Now, discriminant, D = b2 - 4ac
Lee x2 = y = k2 - 4 1 x x 16
(x2 )2 + x2 + 1 = 0 D = k2 - 64 ... (i) [1 /2]

y2 + y + l = O Since, equation has real roots.


D
[1 /2]
�O
On comparing with ay2 + by + c = 0, we get
=> k2 - 64 � 0 [from Eq. (i)] [1 /2]
a = l, b = land c = 1
Discriminant, D = b2 - 4ac = (1)2 - 4 (1) (1)
=> (k - B)(k + B) � 0
=> k :s; - B or k��
=1 -4 =-3 Hence, least positive value of k is B. (1 ]
Since, D < 0
3x + k = 0
23. Given quadratic equation is 2 x2 +
:. / + y + 1 = 0
On comparing with ax2 + bx + c = 0, we gee
2

-
i.e. x 4 + x2 + 1 = 0 or (x + 1)2 - x2 = 0 has no a = 2, b = 3 and c = k
real roocs. Hence proved. [1 ] Now, discriminant,
1 9. Given quadratic equation is D = b2 - 4ac = (3 )2
-4 x 2 x k
a2b2 x2 + b2x - a2x 1 = 0 = 9 - Bk [1 /2]
b2x (a2x + 1) - 1 (a2x + 1) = 0 (1 /2] Since, the roots are real.
.. D
�0
(a2x + 1) (b2x - 1) = 0
a2x + 1 = 0 or b2x - 1 = 0 => 9 - Bk � 0 => 9 � Bk => (1 ]

=>
1
x = - - or x =
a2
1
b2
(1 ] - Hence, the value of k is less than or equal to 2..
8
[1/2]

Hence, the roots of the given equation are - and 2·


-1 1
24. Given, .!... is a root of the quadratic equation

-
a2 b 2
[1 /2]
x2 + kx - � = 0 .. . (i)
20. Do same as Q. 16. [Ans. B] 4

w + k (i) - % = 0
2
21 . Given, x 7x - 1 8 = 0. Now, putting x = .!... in Eq. (i), we gee

-
On comparing with ax2 + bx + c = 0, we gee 2
a = 1, b = 7 and c = - 1 B [1 1

-4 =0
Now, by using quadratic formula, we get
- (- 7) ± �(- 7)2 -4 x 1 x (-l B) => .!.. + � - � = 0
x= 4 2 4
x 1
1 + 2k - 5
2
7 ± 49 + 72 => =0 => 2k
=> x = � [1 ] 4
2 => 2k = 4 => k = 2
7 ± .JUi =>
7 ± 11 Hence, the value of k is 2. [1 ]
25. Do same as Q. 2 (it) of NCERT Folder
=> x= x = ---

--- --
2 2 1 .4
+ 11 11
x= 7 x =
7 - [Ans. p = 4]

-
7
=> or

=--
2 26. Do same as Q. 2(t) of NCERT Folder 1 .4 .
lB 4
=> x =- or x [Ans. k = 0, 8]
2 2
27. Let the number of wickets taken by Zaheer in a cricket
=> =9 =

:
x or x 2 (1 /2] match are x.
Now, required numerical difference of the roots
fj
Then, according to the question,
= 9 ( -2) = 1 1 - [1 /2] Number of wickets taken by Harbhajan �!
[1 ] q
By numerical value of a number, we me� n the number.
Note
= 2x - 3

-- -
regardless o f its sign . Here, we can also find th �
- \�
Since, product of number of wickets = 2 0, therefore we
d ifference of roots as 2 9 t t. But numerica l
=
have x(2x - 3) = 20 => 2 x 2 - 3x 2 0 = 0 l\1
d i fference will be t t . which is the required quadratic equation. [1 ]
22. Given, equation is x2+ kx + 1 6 = 0.
On comparing with ax2 + bx + c = 0, we get
28. Given, x = �5 + �S + �S + .. . oo

a = 1, b = k and c = 1 6 Then, x =� [1 ]
AllinOne MATHE MATICS Class 1 0th Termj
On squaring both sides, we get x2= (�)2 On comparing with ax2 + bx + c = 0, we get
=> x2 = 5 + x => x2 - x 5 = 0 - a = 3, b = - 6 and c = 2
Hence, the required quadratic equation is - b ± V/b2 4 ac [by quadratic formuI ]
-
a
x=
x2 - x - 5 = 0. [1 ] 2a
29. Given, x 2 - 3.J5x + 10 = 0 . . (i) .
± Jc- x x
6 )2 - 4 3 2 = 6 ± �36 - 24 l1J
On comparing Eq. (i) with ax2 + bx + c = 0, we get
=6
x2 3 6
2 ./3 3 ± fj
= ± .JU 6 ±
a = l , b = - 3.Js and c = 10 [1 /2] 6
=
Now, discriminant, D = b2 - 4 c a 6 6 3
= (- 3.J5)2 - 4 1 1 0 x x Hence, che roots are
3 + ./3 and 3 - !3 . [1)
= 4 5 - 40 = 5 > 0 3 3
:. Roots ofEq. (i) exist. [1 ] 3 3 • G iven, � - 1 = __!2_
x x + 1
Byquadratic formula, x =
- b ± ./D , we get
2a 16 � = l
_
=> [1 /2]
(- 3 . .JS> ± .J5 3.JS ± .J5 x x+ 1
x=- => x = ---- (1 /2]
1 6 (x + 1) - 1 5 x = 1
2 x1 2 => ( 1 /2]
3.Js + .J5 or x = ---- 3.Js - .JS x (x + I )
=> x = => 1 6x + 1 6 - 1 5x = x (x + I )
2 2
4.Js 2.Js => x + 1 6 = x2 + x
=> x2 + x - x - 1 6 = 0 => x 2 1 6 = 0 [1 ]
=> x = -- or x = --
2 2 -

=> x = 2 .J5 or x = .JS => x2 = 1 6 ==> x = ± 4

[Ans. � Js]
Hence, the required roots of the given equation are 2.JS [on talcin g squa re roots both sides]
and .JS. [1 ] :. The required values ofx are 4 and - 4 . [ ] 1
34. Let first number be x.
30. Do same as Q. 29. and - Then, second number = x + 4
[·: difference of two num b ers = 4] [1 ]
3 1 . Let the two consecutive odd natural numbers be According to the question,
(2x + I) and (2x + 3). 1
(1 /2]
-I - -- 4
=- [1 /2]
According to the question, x x + 4 21
(2x + 1)2 + (2x + 3)2 = 1 30
=> 4x2 + 4x + 1 + 4x2 + 1 2x + 9 = 1 30
[1 /2] Do same as Q. 32. [Ans. - 7, - 3 or 3 , 7]
3 5 . Let one number be x.
=> Bx2 + 1 6x - 1 20 = 0 => x2 + 2 x - 1 5 = 0 Then, another number = (1 1 - x)
[dividing both sides by BJ [1 ] [·: sum of two n umbers = 1 1, g iven ] [1 /2]
=> x2 + 5x - 3x - 1 5 = 0 According to the question,
1 11
[by factorisation] -1 + =-
=> -
x (x + 5) 3 (x + 5) = 0 x (1 1 - x) 28 [ 1 /2]

=> (x - 3) (x + 5 ) = 0 1 1 - x + x 1 1 => --
---- = -
1 1 -- - 1 1
- --
=> x - 3 = 0 or x + 5 = 0 x (1 1 - x ) 28 l lx - x i 28
1 1 ==> - ::::::
28
=> x = 3 ru x = - 5 => ---- = l lx - x 2
But for x = - 5, ( 2 x + I) and (2x + 3) becomes l
1 x-x 2 28
negative. x 2 - I Ix + 28 = 0
=>
x 2 - 7x - 4x + 28 = 0 [by facto risa [1 1
x =3
=>
. Hence, the two consecutive odd natural numbers are ti on ]
2 x 3 + I and 2 x 3 + 3, i.e. 7 and 9. [1 ] => x(x - 7) - 4 (x - 7) = 0
(x - 4 ) (x 7) = 0
-
32. Given equation is -
. . 1 1 =>
x = 4 or x = 7
- -- = 3
x x -2 =>

x - 2 -x When x = 4, then 1 1 - x = 1 1 - 4 = 7
----
x (x - 2)
=3 When x = 7, then 1 1 - x = 1 1 - 7 = 4
- 2 = 3x (x - 2) Hence, the numbers are 4 and 7. [1 1
=> 3x2 - 6x + 2 =0 [1 J 36. Do same as Q. 35. [Ans. 6 a n d 3]
Quadratic Equations
I 3s

1 = 60 2 3
3 7. We know chat, che time between
= pm to pm.
=> -3ba x = - -
xi -
2b2
a2
t 2 3
h min [1 /2]
Given that, at min past pm, the time needed by the
0 n addmg
. b ot h si"d es ( 2 x )2 • coefficien t of
2
t (': - 3) = 60
minute hand of a clock to show pm was found to be 1.e.
3
( 23ba )i = 4a2
:.__
9bi ,
2 ( 2a3b } ( 2a3b )2 = 2bia2 ( 2a3b )2
4
min less than min.
we get
+
[1 ]
+

4t t 2 - 12 = 240 + xi _ _
+ [1]

(x - 32ba )2 - 2b2ai 4a2


=>
t2 4t - 252 = +
0 [1 /2) 9b2 +

1 Bt - 14t - 252 = 0
ti + [by factorisation]
=> =

a2 - 2ab bi = (a - b)2]
=>
=> t - 14 (t == 0
(t + 1 8 )
+
18) 0
+
[·: +
1 8) (t - 1 4 ) 2
t 1 = 0 t = -14t 14 (x - 32ba ) = 4a2
=> (t
=> + 8 or t =0 =>
- 1 8 or =
x - 3b2a 2ab
=>
:;e - 1 8] ±-
t =14.14
[since, time cannot be negative, so t => -= [on taking square root]

x= -3b2a ± -2ab
x=-
Hence, the required value oft i s [1 ] => [1 ]
38. Seven years ago, let Swati's age be x yr, then Varun's age
.
. 3b b 2h
+
= 5 xi yr. [1 /2) Taki ng, + ve sign, we get
2a -2a = -a +
:. Swati's present age = (x 7) yr
+ . . x = -2a3b - -= b -b
and Varun's present age = (5 x 2 7) yr Taking -ve sign, we get

2b � 2a a
Swati's age = + 7(x 3) (x
Three years hence, we have
+
=
+
1 0) yr Hence, the required roots are
a a and . [1 /2]
Varun's age = (5xi 3) =
+
7
+
(5x2 + I O) yr [1 ]
40. Let the number be x, then its positive square root be
According to the question, x + 1 0 = 3. 5
(5x 2
+
10) �.
x 10 4 + Then, according to the given condition.
x 25 25x 25.f; = 6 25]
= 2x i +

+
+ .[; =
2xi - 6 0
=>
- x

. . (i)
=>
=> = [1 /2]

2x 2x2 (2x-2)4x 3)3 (x3x(x --- 2)62)= 00


+ [multiplying both sides by
=>
(x -
+
[by factorisation]
25x 25.f; - 6 +
=0 [1 1
x
=>
+
.[; =
=> =0
=>
=> x- 2 = 0 2x 3 0- 3 or ·
+
=
=
Put y => = y2
25 y2 25 6 = 0 y -
.

=> x = 2 or x= -2 Then, above equation becomes


25 y2 30y - 5y - 6 = 0
+
+
[by factorisation]

-
x 23 0
=>

:;e
-
> 6)
+
- 1 6) = 0
But as x [since, age cannot be negative] => 5 y(S y
(5 - 1) 1 6) 0 6 (Sy +
+

-�
=2 => y

. 5
( 5y =

(2)2 (2 = 9
:. x [1 /2]
+ => y = - or y = -
Hence, Swati's present age = 7) = yr and
+ 5
.[; = �
. Varun's presem age = [(S ) x ( 7)] 27 yr [1 /2]
=> or .f; = [fro m Eq. (i)] [1 ]
First, make the coefficient of x2 unity and then shift
.[; - 6
39. 5 5
.[;
5
the constant term to RHS. Add the square of half of
B ut = is not possible, because is always
the coefficient of x on both sides to express LHS as

x = -1
the perfect square of a suitable binomial positive.
expression. Simplify the RH S and take t�e square
root on both sides to get the required roots.
=>
25
We have, a2xi - 3abx 2b2 0 +
= Hence, the required number is _!__,

x2 3ba 2b2a2 0
[1 ]
25
=> - -x + - = 41 . Let the n umerator of the fraction be x. Then , the

[dividing both sides by a2] [1 /2)


denominator of the fraction be
+
x 3.
Allinone MATHEMATICS Class 10th Term�!
x- Since,
-2a
Thus, required fraction = _ [1 /2)
x+3 -(a - b) + {-(a + b)} = -a + b - b= a -

According to the question, Let p = - (a - b) and q = - (a + b) [1]


. w rmc n as
.,
b .,· = 0 (1 I2)
Then, given equation can be
x x 1 ·
---- = -- -- [1/2) x 2 - (a - b) x - (a + b) x + a -

(x + 3) + 1 x+3 15
=> x 2 (a - b )x - (a + b) x + (a -
x - -- x 1
- b) ( a + b) = 0
=> -- =-- => x { x (a - b)} - (a + b) {x - (a - b)} = 0
-

x+4 x+3 15
=> {x - (a - b)} {x (a + b)} = 0 -

=>
x [(x + 3 ) - (x + 4)] = - 1 => x - (a - b) = 0 or x - (a + b) = 0
-
(x + 4) (x + 3) 15 => x = a - b or x = a + b [1 ]
x[x + 3 - x - 4) I
=> =-- Hence, the roots of given quadratic equation arc
x2 + 7x + 12 15 (a - b) and (a + b). (1 /2)
=> 44. We have, + -1
- (x2 + 7x + 1 2 ) = - 1 5x 1 1
= - +
I
-

=> x2 + 7x - 15x + 1 2 = 0 a+b+x a b x


x2 - Bx + 1 2 = 0 1 1
=> [1 ] => -+-= (1 /2)
=> x2 6x - 2x + 1 2 = 0 [by factorisation]
-
a b a+b+x x
=> x (x - 6) - 2 (x - 6) = 0 a + b = x - (a + b + x)
--
=> (x - 2) (x - 6) = 0 => x = 2 or 6 ab (a + b + x)x
When x = 2, then firact1on = -- = -- = -
x . 2 2 a +b =x-a-b -x
=> --

x+3 2+3 5 ab (a + b + x)x


When x = 6, then fraction =
6- = §. = �, which
- a + b = - (a + b)
6+3 9 3 --

ab (a + b + x)x
is not possible, as here numerator is not 3 less than che
denominator. => -1 = -1
[1)
. is. - .
. d firacnon 2 ab (a + b + x)x
Hence, requue [1 1 .
5 => x2 + ax + bx + ab = 0 [1 /2]
42 Let three consecutive natural numbers be => x(x + a) + b(x + a) = 0
x, x + l, x + 2. [1 /2] => (x + a) (x + b) = 0
According co the question, => x + a = 0 or x + b = 0
(x + 1)2 = 60 + [(x + 2)2 - x 2 ] x = - a or x = - b
Hence, x = -a or -b. [ 1 ]
=> x2 + 2x + 1 = 60 + x2 + 4x + 4 - x2
45. Given equations are
[·: (a + b)2
a2 + 2ab + b2]
=
x2 + 2cx + ab = 0
=> x2 + 2x + I = 64 + 4x . . . (i)
and x 2 - 2 (a + b) x + a2 + b2 + 2c 2 0 . . (i i) ==
=> x2 + 2x - 4x + I - 64 = 0 Let Di and D be the discrimi nants of Eqs. (i) a nd (ii
.

=> x2 - 2x - 63 = 0 [1] 2
respectively . Then,
),

=> x2 9x + 7x - 63 = 0 [by factorisation]


- D1 = ( 2c)2 - 4 x 1 x ab [·: D = B 4 A C ]2
-

=> x(x - 9) + 7 (x - 9) = 0 = 4c2 4ab = 4 (c2 - ab)


_

=> (x - 9) (x + 7) = 0 and D2 = {- 2 (a + b )} 2 - 4 x I
=> x - 9 = 0 or x + 7 = 0 x (a2 + b2
+ ic 2
x = 9 or x = - 7 [1 ] )
But x is natural number. = 4(a + b)2 - 4(a2 + b2 + 2c 2 )
x=9 = 4(a2 + b2 + 2ab - a2 - b 2 - 2c 2 )
Hence, the required numbers are 9, 1 0, 1 1 . [1 /2]
= 4(2ab - 2c2) = - 8 (c2 - ab)
43. Given, x2 - 2ax + a2 - b2 = 0 (1 J
Since, the roots of Eq. (i) are real and unequal.
On comparing with Ax2 + Bx + C = 0, we get
[1 /2]
.. Di > 0
A = l, B = - 2a and C = a 2 - b 2 => 4(c2 - ab) > 0 => c2 - ab > O
Lee p + q = - 2a and pq =
a 2 - b2 => -B (c2 - ab) < 0 [·: A > 0 => - A < OJ
<
[1 ]
Now, possible pair of faccors of a2 - b 2 are -(a - b), => D2 0
-(a + b) and (a - b)(a + b). Hence, roots ofEq. (i i) are not real. Hence proved. [ 1 /2]
Quadratic Equations ' 37
I + I = -2 - 1 5x + 6 = 0
(x- I) (x - 2) (x - 2) (x - 3) 3 15 + -6 = 0
9x 2

x 2 - -x
46. G iven,

x - 3 + x - 1 = -2 =>

(x - l)(x - 2)(x - 3) 3
9 9
=>

5 2
(1]

2x - 4 2 3 + -3 = O 2
x 2 - -x
[dividing both sides by 9] [1 /2]

=>
(x - 1) (x - 2) (x - 3) 3 = - =>

2 (x - 2) 2 5
(x - 1) (x - 2) (x - 3) = -3 => x - -x 3 = 3
x' - 2 (fr + (H = (H �
2
=>
- - [1 /2]

I - -I
[ (�r
=>
(x - I) (x - 3) 3 [1 ]

]
(1 ]

=> 3 = (x - I) (x - 3)
=> 3 = x2 - 3x - x + 3 adcling on both sides

x 2 - 4x = 0
(x - H = �� - �
=>

x(x - 4)x == 00 x - 4 = 0
[1 ]
=> =>
=>
x = 0 or x = 4 25 - 24
( x - H 36
or

Thus, the required values fx are 0


=>
=> = [1 ]

same as Q. 4 6 2 ± .fj 5 1
and 4.
1
o [1]

( x - H = -36 x - -6 = ± -6
47. Do . [Ans. 2 ]

same as Example. 2 ofTopic 2.


=>

x = -65 ± -6
48. Do
1

G iven, - S is a root of 2x2 + px - l S = 0


[taking square root]

+ 5 1 +1 6
=>

2(- S)2 p x(- S) - lS = 0


49.
+
x = -6 + -6 = -- 5 = -6 = 1
Taking ve sign, we get

2x 2S - Sp -- lSlS == 0
[1 /2]
=>
=> SO - Sp - Sp = oO 6
=>
Spp == 73S 35 x = -6 - -61 = --
Taking - ve sign, we get
5- 1 = -4 = -2
6 3
s

..
=>

1 32
·6
p (x2 + x) + k = 0
[1 ]
Hence, the roots are and -. [1 ]

7 (x2 x) + k = 0
Now,
=>
52. Let ABC
7x2 + 7x k = 0
+ be the given right
=> +
ax2 + bx + c = 0,
... (i) [1 /2]
BCAC= x = 25
angled triangle such that
base, cm and

a = 7, b = 7 c = k = 60
On comparing Eq. (i) with we get hypotenuse, cm

discriminant, D = b 2 - 4ac :. AB + BC + CA = 60
and Since, perimeter cm

= (7)2 - 4 x 7 x k = 49 - 28k AB + x + 25 60
Now,
=

Since, p(x2 x) k = 0 equal


=>
=> AB = 35 - x
MBC, [1 /2]

7x2 + 7x + k = 0 equal AB2 + BC2 = AC2


+ + has roots B +--- x cm--.. ..::;
In by Pythagoras
1.e.

28kD == 00 (35 - x)2 + x2 = (25 )2


has roots. theorem, we get

4 - 2Bk
. .

1225 + x2 - 70x + x2 025


[1 ] =>
=> 9
= 4949 2x2 - 70x +-600 = 0
k = 28 k = -74
=>
=> =-

=> =>
x2 - 35x + 300 = 0
=>
=>

Hence, required value of k = ?.... . 2]


x2 - 20x - 1 Sx + 300 = 0
[1 ] [dividing both sides by [1 ]
4
same as Q. 4 x(x - 20) - 15 (x - 20) = 0
=> [by factorisation]

(x - x20=) 2(x - 15) 0


50. Do 9 . :=>
=
0 x = 15
=>
5 1 . Given quadratic equation is => or [1 /2]
38 ' A llinOne MATH EMATICS class 1 0th Tent

[ J
If x = 20, then AB = 35 - x = 35 - 20 = 1 5 and
=>
6500 6500 = 3
- 0
BC = x = 20 [1 /2] x x + 15

[ ]
: . Required area of triangle = .!.. x AB x BC 1
2 6500 .!_ - -- = 30
x x+ 15
= !.
x 1 5 x 20 = 1 50 cm 2 [1 /2]
2 x + 15 - x
65oo = 30
If x = 1 5 , then AB = 35 - x = 35 - 1 5 = 20 and x(x + 1 5 )
BC =x = 1 5 [1/2] 6500 x 1 5
= 30
!.
=>
: . Required area of triangle = x BC x AB x2 + 1 5x
2
=> x 2 + 1 5x = 650 x 5
.!.. x +
[1
15 x
=>
= 20 = 1 50 cm2 [1 /2] 2
2 x l 5x - 3250 = 0
53. Let the length of piece be x m. => x2 + 65x - 50x - 3250 = 0 [ by facrorisatioo

-
Then, rate = � - per m
200 => x(x + 65) - 50 (x + 65) = 0
(1 /2]
% => (x - 50) (x + 65) = 0
Now, new length = (x + 5) m => x - 50 = 0 or x + 65 = 0
Since, the cost remains same. => x = 50 or x = 65
:. Ncw rate ='
200
--
per m Since, che number of persons cannoc be ncgarivc.
[f
[1/2)
x + 5 Therefore, x = 50
According to the question, Hence, the original number of persons is 50. [1 ,1
200
=
200
- 2 (1 /2] 55. Lee number of books bought by shopkeepe r = x
x + 5 x
--
1 200

- _1-)5
Then, cost ofeach book = [1/f
200 - 200 x
=> = 2
x x + 5 If1 0 more books, he bought for same amount. Then
1 .!.. number of books = x + I 0 and cost o f each b ool
---
20
"l x x +
= 2
=
1200
x + 10
[1 f2l
x + 5 -x 2 5 1
=- => =-
5.)
=> According co che question,
x(x + 200 x2 + 5x 1 00 1 200 1 200
- = 20 [1 /21
=> x2 + 5x = 500 x x + 10
=> x2 + 5x - 500 = 0 (1 ] 1 200 [ x + I 0 - x] = 20
=> x2 + (25 - 20)x - 500 = 0 [by factorisation] x (x + I O)
=> x2 + 25x - 20x - 500 = 0 => --
1 200 x 1 0 -- = x 2
+ l Ox
=> x(x + 25) - 20(x + 25) = 0
20
=> (x + 25) (x - 20) = 0 600 = x 2 + l O x
=> x + 25 = 0 or x - 20 = 0 x2 + I Ox - 600 = 0 (1 ]
=> x = - 25 or x = 20 => x2 + 30x - 20x - 600 = 0
But x cannot be negative, so x = 20 m. => x(x + 30) - 20 (x + 30) = 0
:. Rate = - = - = f I O per m
200 200
[1 ] => (x + 30) (x - 20) = 0
. x 20 => x + 30 = 0 or x - 20 = 0
HenC4:� the original length of the piece is 20 m and its => x = - 30 or x = 20
original rate per metre is ' I 0. [1 /2) B ut number of books cannot be negative.
54. Lee rhe original number of persons be x. :. . x = 20

I
[1]
6500
--

-
Then, share ofeach person =' Hence, number of books bought by sho pkeeper are 2
[1 /2) 0.
. x
[1 /2]
56. Given equation is
If the number of persons is incr�d by 1 5.
· · ·6
The, new share ofeach person = f x +)§
500
. [1 /2]
2 + (2mc)x
(I + m )x
2
+ (c
2
a2 ) = o
On comparing with Ax
. 2 +
According to the question,
Bx + C = O, we gee
A = (I + m ), B = 2mc and C = (c 2 - a 2 ) [ 1 /
2
- 30
650 0 6500 2]
=
Si n ce, the given equation has equal roots.
[1 /2)
x + 15 x
Quadratic Equations f 39
= 0 B1 - 4AC = 6 1 . We have,
( 2mc)1 - 4(1 m 1) (c1 - a1) =
. . Discriminant, D => O
=> x = -- 1 --- = ----
4m 2c 2 - 4 (c 2 - a1 m2c1 - m2a1) =
+ O [1 ]
=> 2 - 2 - -1- 2 - -- 2(2 - x) - 1 ---
m 2c 2 - (c 2 - a 2 mlc l - mla l ) 0
+ O
=> + =
1 2 -x 1 2 -x [1 /2]

m2c 2 - c 2 a l - mlcl [dividin mla l g= by0 x = -- - x --


2 - 2x -_1 2 32_--_2xx_
2
_ _
4] [1 ]
+
[ 1 /2]
+
=>
=>

-c l al mla l = 0
_ _ _
_

=> + +

-c1 a2(1 m1) = 0


4

2(3 - 2x) - ( 2 - x) --
[1 /2]
=> x = -- 1

- c 2 = - al (l ml )
+ +
+
=>

c l = a2(1 ml) - 2x
3
=> +
x = -- 3 - 2x
=>
2(3 - 2x) - ( 2 - x)--
D o same as Example. 6 o f Topic 4.
Hence proved. [ 1 ]

x = - 4x3 --2x2 + x x = --- 3 - 2x [1 )


57.
[Ans. Usual speed of che train = 25
=> =>
- 3x
x( - 3x) = 3 - 2x
km/h] 6 4
4
4x - 3x1 = - 2x
Do same as Q. 1 2 of NCERT Folder =>

[Ans. x = 1 .3
58.
3 (1 ]

Do same as Example of Topic 4.


8 km/h] =>

=> 4x -3x2 -3 + 2x = 0
3x1 + = 0
59. 8
[Ans. 20 min; 25 min] => - · 6x 3
Lee be che initial production (2 yr x1 - 2x-+1)11 == 00 [dividing both by 3]
= p 100Px = p (l I 00x J
increase in production every year be xo/o. (x
60. P and ago) che => sides

[·: (a - b)1 = a1 - 2ab b2]


Then, =>
production ac che end of first year. +
x= 1 l,

= ( � P [(i �JJ
+ + [1 ] [1 ]
62. Given equation is
Production ac che end second year
x
of
(a - b)x1 + (b - c)x + (c - a) = 0
with Ax2 + Bx + C = 0, we
= ( � J
P 1 +
J
100 100 100 + +

= (a - b ), B = (b - c = (c - a)
On comparing get

(1 � = P(l � 1 A )and C

100 J 100 J 100 Since, it has equal roots.


D = B2 - 4AC = 0

( �
P l + + + [1/2]
:. Discriminant, [1 )

1 . = 2P
Since, production doubles in che lase cwo years.
=> (b - c)1 - - b) (c - a) = 0
4(a

100 J
P l + (1 ] => b2 + c2 - 2bc - - a1 - be + ab) = 0 1-
b2 + c2 - 2bc - 4ac + 4a1 + 4bc - 4ab 0 2
4(ac

1
( 1 -=--J = 2 (100 x)1 = 2 (100)1 4a2 + b2 + c2 - 4ab + 2bc - = 0
=> = [1 ]
=> + => + x LL
=> 4ac

10000 x1 200x = 20000 (2a)2 + (-b)2 + (-c)2 + 2(2a) ( ) 2


1 00
=>
=> +
b
+ 2(-b) ( -c) + 2(-c) 2a = 0
-

x2 200x - 10000 = 0 v
+

(2a - b - c )2 = 0 v
(1 ]

On comparing with ax1 bx c = 0, we gee


+ (1 /2]
=> LL

a = b = 200 and c = - 10000


+ +
[·: (a + b + c = a1 b1 +2ca]c2 v
)2
+ 2ab + 2bc +
+
v
x = - 200 �(200)2 40000
I,

2a - b -c 0 2a = b c
[by quardratic formul�, x = - b f: - 4ac ]
± + = => +
<l [1 ]
=>
Hence proved . u.
...
±
63. Let x be the total number of camels. - --

= =.
!;;
- 100 100.fi. = 100( - 1 .fi.)
Then, number of camels in the forest
± ±
4

= 100(- .fi.) [·: cannot be nega iv


=
Number of camels on mountain = 2J; =: [ 1 /2]

= 100 (- 1 1.4 14) = 100 (0.4 14) = 41.4


x t e]
=
1 +
and number of camels on the bank of the river = 1 5

Hence, che required percentage is 41.4%.


-
+ 2.[;
4
+
[1 ] :. Total number ofcamds = =. + 15 - [1 /2]
"
40 ' Allinone MATHEMATICS Cla ss 1 0th Te

= (Area of gravel pa th )

4 2../x 4
According to the question,
m,
x (Rate of gravelling path per s q u a re
=. + sJ;
4x-) 100 6(44x x-) ''
.
, 150 .,
+ 15 = x => x + + 60 = x
.
= (1 76x - x -= - [1� 1

0
[multiplying both sides by 4]
=> x + s ../x + 60 - 4x =
0 4904.
cost of laying the
It is given that the total
gravelling the path is f
bed u flower
I
=>
=>
-3x + s..fx
- sJ;
+ 60 =
0 .. 11(22 - x) 2 6( 44x - x 2 ) = 4 904 +

(484 x 2 ) ( 264x - 6x 2 ) = 4904 1


3x - 60 =
[ by o t given c nd i iof :

[multiplying both sides by - 1 ] (1/2] -

5324 - 484x lx2 264x - 6x 2 = 49o4


=> 1 1 44x + +
Let .f; = y => x = y 2
-8 0
... (i) (1 /2]

5 xi - 220x 5324 4904


=> + 1 +
j - 60 =
2

2 - 18 + 10 0
Now, we get 3 y

5xi - 220x 420 = 0


+ =
=>

0
3y y y - 60 = [by factorisation]

xi - 44x 84 0
=> +
=> 3y(y - 6) + l O(y - 6) =
=> (y - 6)(3y +
0 = 10)10 00 => + =

xi - 42x - 2x 0
[dividi ng b h or sides by �
=> y - 6 = or 3y + =

3
IO => + 84 =
=> -
facto risation] (1�
y = 6 or y = - (1 ]

42) - 2(x - 42) 0


[by
r
r
vx vx
(x - 42)(x
-

- 2)x 042
-10 => x(x =
=> = 6 or = -
· -
3
2 =
=>
r
=

4 42,
vx *
-10 ·

But - , as square root of a number is always => x = or


positive. 3 But x * as flower bed
the side of square i
( 4 - 2x) m.
Hence, the number of camels is 36. [1 ]
x = 2m
2
x 1200
Hence, roots are real and equal. of
Hence, width the gravel path is m. (1/1
(z)
2
64. Let the width of the gravel path be x m. Then, each side of
cover 1200
65. Let the usual speed the aeroplane be km/h.
of the square flower bed is ( 44 - x ) m .
Now, area of square field = 44 x 44 = m2 1936 Then, the time taken to km =
x
[1/2l

100)
Now, if plane started late by one hou r, then iO

1200
1200 x 100
Gravel path speed = (x + km/h and time taken to cove!
km = [1/21
+

1200 - 1200 1
1200[_!_ 100 J 1
According to given condition, we have

100 = [1/21

1200[ 100100)- XJ
x x +
Square field
I

(44 - 2x)2
---- 44 m ���+ - =
-
>
- Area of flower bed = m2
x x +
l
(1 1
n
x + =
. . Area of gravel path
,..,
120000 1 00..\"
x(x +

2
= Area of the field - Area of the flower bed
LJ
0
= x2 +
,.,
= 1 936 - (44 - x) 2
1936
=>
- 120000
i 400x - - 120000
300x
=> x 2 + I OOx =
f) (1 936 - 1 76x + 4x 2 )
-
= O
4 )
= [1/21

400) - 300(x 400)


=> x +
:( = (1 76x - x 2 m 2
= O
(x 400)(x - 300)
=>
[1 /2] x(x + +
� => = O
x -400
Cost of laying the. flower bed

300
+
:.
=> x =
= (Area of the flower bed)
• = or

27 5
x (Rate of the flower bed per square metre)
[1 /2)

(44 - 2 ) 100

.. x 300
·: Speed cannot be negative.
c x 2 x
=
300 k h
= [ 1 /2)

'


I =
1: (44 - 2x)2 11(22 - )2= x
Hence, usual speed of the aeroplane is
(1 /2)
m/ .


(iz) The values (qualities) of the pilot represente d in
(1 )
Cose of gravelling path chis question are leadership and punctuality.
• •

Act1v1ty
Topic Covered Quadratic equation 6. Paste this coloured square in one corner of bigger
Objective To find experimentally, the positive root of the � quare, as shown below.
quadratic equation of type x2 + ax - b = 0, where ---- 6 cm ----

a I b > 0. A s R B
Ski l l Developed Analysis, problem solving creativity.
Time Requ i red 15 min
Pre-knowledge o 2 + 2ab + b 2 = (a + b) 2 and
method of completing the square. P - 4 cm -- Q
Material Req uired Drawing sheet, coloured paper, D '------' c
scale, pencil, scissor and glue stick.
Observation let us find AS.
Method
1. Take quadratic equation x2 + Bx - 20 = 0
AS = AR - SR

r
= 6 - 4
2. Write x2 + Bx - 20 = 0 as x2 + Bx = 20
= 2 cm
3. let us complete the square by adding
G
Hence, the positive root of x2 + Bx - 20 = O is 2.

r G r
of coeffi cient of x on both sides. Conclusion

[ ·.- G ]
The positive square root is the difference between the sides
.. x2 + Bx + (4) 2 = 20 + (4) 2 of squares.
of coefficient of x = x a = 42
Oral Questions
=> (x + 4) 2 = 20 + 16 = 36 = 6 2 1. What is the quadratic equation?
4. Draw a square of side 6 cm. 2. Is x2 = 5 a quadratic equation?
A ------- B 3. What is the coefficient of x in quadratic equation
Sx2 - p = O?
4. Which number should be added and subtracted in
the quadratic polynomial x2 + 3x + 1 to complete
the square?
5. Why do we take 'a ' common, while solving quadratic
equation ax2 + bx + c = 0 by method of completing
0 - a cm - C the square?
5 . Cut-out a square of side 4 cm from the coloured 6. If a = 0, will the equation ax2 + bx + c = 0 remain
sheet of paper. quadratic? Give reason. u

Sr------ R 7 . How will you differentiate between a zero and a root?


8. What is the standard form of a quadratic equation.
9 . What type of equation Sx2 + Bx + 4 = 2x2 + 4x - 6
is?
1 0. If D � 0 for a quadratic equation, then what will be
P '------'Q the nature of the roots?
4 cm
42 ' Allinone MATHEMATICS Class 1 0th Term 11

True/False Across
1. A quadratic equation has atleast two roots. 1. An equation of the form ax2 + bx + c = 0, where
2 . A quadratic equation may have no real root. a, b, c are real n u mbers and a * 0, is called ... .
3. The degree of a quadratic polynomial is atmost 2. 4. For a ny quadratic polynomial P(x), a rea l n u m ber a
such that P(a ) = 0, is cal led ...
4. If 2 is a zero of the quadratic polynomial P(x), then 2 is a
5 . Maximum n umber of real roots a quadratic
·

root of the quadratic equation P (x) = 0.


5. The equation (x + 2) 2 = 0 has real roots.
equation can have, is ... .
6. The value of b 2 - 4ac for any quadratic eq u ation
ax2 + bx + c = 0, is cal led ... .
6. If the value of discriminant is equal to zero, then the

7. The roots of a quadratic equation whose


equation has real and distinct roots.
7. If the product ac in the quadratic equation
ax2 + bx + c = 0 is negative, then the equation
discriminant is g reater than or equal to zero, a re ...


cannot have non-real roots.
8. If the value of discrimina nt of a quadratic e q uation
8. x = is a root of 2x2 + 3x - 1 = 0. is greater than zero, then its roots are rea l a n d ... .

9. (2x - 1) (x + 1) = (x - 1) (x + 1) represents a quadratic Down


equation.
2. For any quadratic equation ax 2 + bx + c = 0, a
1 0. If the discriminant of a quadratic equation is greater
real number which satisfies it, is called ... .
3 . The roots of a quadratic equation whose
than zero, then the equation has real and distinct roots.

Crossword Puzzle discriminant is zero, are real a nd ... .


9 . Number of terms i n the quadratic equation
2 3
ax2 + bx + c = 0, is ...

P roject /S u rvey W o r k
Write a short article or short note (lecture) o n the
following topic.
Contribution of Bhaskaracharya towards g rowth of
Mathematics.

Answers
Ora l Questions

(3)2 2, 1.e. 4
1 . A polynomial equation of degree two is known as quadratic equation.
9
2. Yes 3. 0 4. •

5. In order to solve ax2 + bx + c = 0 by method of completing the square, we take a common to make coefficient of
x2 unity.
6. No, because in this case, there will be no term containing x2• Hence, it will be a linear equation when a = 0.

ax2
7. The word 'zero' is used for polynomial, while 'root' is used for an equation.
8. + bx + c = 0, a -:1: 0 9. Quadratic equation. 1 0. Roots will be real.

True/Fa lse
1 . False 2. True 3. False 4. True 5. True
6. False 7. True 8. False 9. True 1 0. True

crosswo rd Puzzle
1. Quadratic 2. Root 3. Coincident 4. Zero 5. Two
6. Discriminant 7. Real 8. Distinct 9. Three
SU M M ARY
• The sta ndard form of quadratic equation is ax2 + bx + c = 0 where, a * 0 and a, b and c a re real n um bers .
• A real n umber (say) a is said to b e a root o r solution of a quadratic equation, ax2 + bx + c = 0 , a * 0 , i f a n d only if
aa 2 + ba. + c = 0
• If we can factorise ax2 + bx + c, a * 0, then we first write the given quadratic polynomial as product of two lin ea r factors
by spl itting the middle term and then equate each factor to zero to get possible roots of given quadratic eq uation .
• The roots of q uadratic equation, ax2 + bx + c = 0, a * 0 can b e obtained b y completing the squ a re method.
• The roots of the quadratic equation ax2 + bx + c = 0, a * 0 can also be obtained by quadratic formula,

-b ± �b2 - 4ac
X = ---'----
28
• T he natu re o f roots c a n b e identify as
(1) If D = b2 - 4ac = 0, then quadratic equation has two equal real roots or repeated roots.
(i1) If D = b 2 - 4ac > 0, then quadratic equation has two distinct real roots.

(iii) If D = b2 - 4ac < 0, then quadratic equation has no real roots.

A x s
�I Very Short Answer Tvpe Questions [1 Mark each ]

2
1. For what value of p, the equation px - I B x + l = O is a perfect square?

2 . For the quadratic equation x 2 - 2x + l = 0, find the value of x + .!...


x

3. Find the discriminant of quadratic equation x 2 - 4x + I = 0 . CCE 20 13

+ bx + c = O has equal roots, then find the value o f c .


2 CCE 20 1 2
4. I f ax

�I Short Answer Type I Questions [2 M a rks ea c h ]

5. If the equation px
2
+ 4x - 3 = O has real roots, then find the value of p. CCE 20 1 3

6. Determine the value of p for which the quadratic equation 2px 2 + 6 x + 5 = 0 has equal roots.

7. What will be the value of p for which the equation x 2 - 4 x + p = O has equal roots?
8. If the discriminant of 3x
2
+ 2x + a = 0 is double the discriminant of x 2 - 4 x + 2 = O , then find the value of a .
CCE 201 1

9. Find the roots of the quadratic equation 2x


2
- .JSx - 2 = O by using the quadratic formula.

1 0. Find the difference of the roots of equation x


2
- 7x - 9 = 0.

1 1. Find the roots of 6 x


2
- .fi.x - 2 = 0 by the factorisation of the corresponding quadratic equation.
Allinone MATHEMATICS Class 1 0th TernrJI

�I Short Answer Type II Questions [ 3 M a r k s e a ch ]

12. If the product of two consecut ive odd positive 1 6. C heck whether the equation 6 x 2 - 7 x + 2 = 0 has
integers is 3 2 3, then find the integers . CCE 20 13 real roots and if it has, then find them by t he method
of completing the squares.
13. If -4 is a root of the quadratic equation
x 2 + px - 4 = 0 and the equat ion 2x 2 + px + k = 0 1 7. A train travels a t a certain average s peed for a
has equal roots, then find the value of k. CCE 201 1 distance of 63 km and then travels a d ist ance of
72 km at an average speed of 6 km/h more than its
14. Find the roots of the quadratic equations b y using the original speed. If it takes 3 h to complete the total
quadratic formula in each of the following journey, then what is its origina l average s peed?
CCE 20 1 1
( i ) 2x 2 - 3 x - 5 = O ( ii) 5x 2 + Bx + 8 = O 1 8. [[OTSJ Had Ajita scored 1 0 more marks in her
Mathematics test out of 30 marks, 9 times these marks
15. If one root of the quadratic equation 3x 2 - px - 2 = O would have been the square of her actual marks. How
is 2, then find the value of p. Also, find the other root. many marks did she get in the test?
CCE 2013

�I Long Answer Type Questions [ 4 Marks each]

1 9. Find the roots o f the following quadratic equations by 2 7 . Find the condition tha t one root o f px 2 + qx + r = 0
the factorisation method may be doubled of the other.
( i ) 2x 2 + ?. x - 2 = O 2 8. If -4 is a root of the equation x 2 + px - 4 = o and the
3
equation x 2 + px + q = O has equal roo t s , t hen find
( ll ) - x2 - x - - = 0
" 2 3
the values of p and q.
5 5 NCERT Exemplar
x 2 - bx k -l
20. State whether following quadratic equations have 2 9. For what value of k will the equation =
two distinct real roots. Justify your answer. ax - c k + l
NCERT Exemplar have roots reciprocal to each other?

( i ) x 2 - 3x + 4 = O 30. For which condition, the quadratic equa tion


-
( ii) 2x 2 + x - l = 0 (a 2 + b2 )x 2 - 2(ac + bd)x + (c 2 + d 2 ) = O h a s equa l
roots? CCE 201 3
21. Find the roots of the following equation
31. Sum of the areas of two squares is 400 cm 2 • I f the·
; x * - 3' 6'
l l 9
- = difference of their perimeters is 1 6 cm, then find the
(x + 3) (x - 6) 20
sides of two squares. CBSE 20 1 3
2 2 . The denominator o f a fraction is one more than twice
the numerator. If the sum of the fraction and its 3 2 . A shopkeeper buys some books for f 80. I f he had
bought 4 more books for the same amount, each book
reciprocal is 2 �. then find the fraction. would have cost f l less. Find the number of books h e
21
bought. CBSE 2005
2 3 . The sum o f the squares o f two consecutive odd
numbers is 394. Find the numbers. 3 3. A person on tour has f 360 for his daily expense s. I f he

2 4 . A na tural number, when increased by 1 2, equals 1 60


exceeds his tour programme by 4 d ays, he must cut
down his daily expenses by f 3 per day. Then, find the
times its reciprocal. Find the number.
number of days of his tour programme.
NCERT Exemplar

2 5 . Find whether the following equations have real roots . 3 4. A train travelling at a uniform speed for 3 60 km ,
If real root s exist, then find them. would have taken 48 min less to travel the same
( i ) B x 2 + 2x - 3 = 0 ( ii ) - 2x 2 + 3x + 2 = 0 dis tance, if its speed was 5 km/h more. Find the
original speed of the train. NCERT Exemplar
NCERT Exemplar

2 f.i . I f the equa tions 2 x2 - 7 x + 3 = O and 3 5 . A motorboat whose speed in still wa ter is 1 8 km/h,
2
4 x + ax - 3 = O have a common root, then what is takes I h more to go 24 km upstream than to return
the value of a? CCE 20 13 downstream to the same spot. Find the speed of the
stream. CBSE 20 14
Quadratic Equations
I 45 I
3 6. I n a flig ht of 1 600 km, an aircraft was slowed down due to bad weather. Its average speed for the trip was reduced by
400 km/h and the time of flight increased by 40 min. Then, find the actual time of flig h t .

3 7. Out of a group of swans, ?_ times the square root o f t h e total number a r e playing o n t h e shore o f a tank. T h e two
2
remaining ones are playing in the water. Find the total number of swans. CCE 201 2

3 8. [ffo__f$JIn a group of children, each child gives a gift to every other child . If the number of gifts is 1 32, then find the
number of children.

3 9. ��f� Find the solu tion of the equation �x 2 - 1 6 - (x - 4) = �x 2 - 5x + 4.

Answers
b2
1 . 81 2. 2 3. 12 4. -

4a

5. p � --
4
6. � 7. 4 8. - 1
3 10
JS + .J2i .J5 - J2i .J2 ../2
9. an d 10 . ../85 11 • - - and - 1 2. 1 7, 1 9
4 4 3 2

1 3. �
8
1 4. (i) �' - l (ii) - 1 , - � 1 5. p = 5, other root = -3
1
1 6.
2 1
3 '2
-3 2 1
1 7. 42 km/h 1 8. 15 1 9. (i) - and - (ii) - - and 3
2 3 2

20. (i) no real roots (ii) distinct real roots 21 . 1 and 2

22. � 23. 13, 15 24. 8


7

26. - 1 1 or 4 27. 2q2 = 9pr 28. p = 3 and q = -4


+ 1
30. ad = be
c
29. 31 . 12, 16 32. 16
c- 1

33. 20 days 34. 45 km/h 35. 6 km/h 36. � h


3

37. 1 6 38. 12 39. 4


CHALLE N G E RS*
1 . Find the extraneous root of equation

.Jx + 10 -
6
= 5.
.Jx + 10
2. Any line is said to be a tangent to the cuNe, if it intersects the cuNe at one point.

If the line y = kx - 3 is a tangent to the cuNe y = 2x2 + 7, then find the possible values of k .

3 . Show that, for linear factors o f the equation x 2 + kx + 1 = 0, k � 2 and k � - 2.


4. The coefficient of x in the quadratic equation x2 + px + q = O was taken as 1 7 in the place of 1 3 and its roots were fou n d to be
-2 and -1 5. Find the roots of the original equation .
5 . A graph of quadratic polynomial is given below

y
3 y =x2 - 3x + 2
2
1
X' x

-1
-3 -2 -1 3
-2
-3
Y'
(1) If y =O, then find the roots of the equation .
(it) If we rotate the axes a t a n angle o f 90° in anti-clockwise direction , t h e fig ure remains a t t h e s a m e position . F i n d t h e
equation o f the graph.

6. If sin a and cos a are the roots of the equation ax2 + bx + c = 0, then prove that a2 + 2ac = b2 .
7. Draw the graph of y = x2 + x - 1 2. If y = O, then find the roots of the cu Ne. Also, find the area of the triangle formed by
joining the intersection point of cu Ne .

8. Find the roots of the equations x2 - 4x + 3 = 0 and 2y2 - 7y + 3 = 0.

Also, plot their roots on the axes and find the area of the smallest triangle formed by joining these points and orig i n .

9 . Out o f a certain number of saras birds, one-fourth the number are moving about lotus plants , 2 th are coupled with _! t h a s well
9 4
as 7 times the square root of the number move on a hill, 56 birds remain in vakula tree. What is the total n u m ber of bird s ?

* These q uestions m ay be or may not be asked in the examination, have been given just for additional practice.
Note Solutions to these questions are available at page number 383-384.
C H A PT E R 2

ARITH M ETI C
PROG RESS I O (A P)

(?J Topic Checklist_ In chis chapter, we will study che sequence (list of numbers) in which succeeding terms
(consecutive terms) are obtained by adding a fixed number to the p receding terms.
Introduction to Arithmetic We will find the formulae co find the value of any term of this type o f sequence and the
Progression sum of terms of chis type of sequence.
nth Term of an AP
·· Types of Problems based on
nth Term of an AP
Seq uence (List of N u m bers)
, nth Term from the End of A list of numbers, arranged in a definite order according to some definite rule, is called
an AP a sequence.
S um of First n Terms of an AP e.g. (i) l , 1 0, 1 2, 1 6, 1 00, . . . In chis sequence, numbers are arranged in ascending order
.

, Different Types of Problems except first number.


based on Sum of First n (ii) 1 , 2,3,4, 5, 6, . . . . I n chis sequence each number is 1 more than the number
Terms of an AP
preceding it.
(iit) 2, 4, 8, 1 6, . . . I n chis sequence, each number is obtained on multiplying the
• Arithmetic Mean
.

preceding number by 2, except first number.


The various numbers occurring in a sequence are called i ts terms.
Consider, a sequence a1 , a 2 , a3 , a4 . . .
Here, a 1 i s first term, a 2 is second term and s o o n . I ts nth term can be written a s a ,, ,
which is known as the general term of sequence. A sequence is said to be finite or infinite
accordingly ic has a finite or infinite number of terms.

Progression
'Those sequences whose terms always follow certain patterns are called progressio ns . '
e.g. (i) The sequence 5, 7, 9 .
3, . . i s a progression, a s each term c a n be found b y adding 2
to the term preceding it.
(ii) The sequence 1 0, 1 8 ,
3, 21, 35 . . . . is not a progression as its terms are not
following any certain pattern.
N ote All progressions are sequences buc all sequences need nor be progressi o n s .

In this chapter, we will study abouc one o f the progression, arithmetic progressio n .
TOPIC 0 Introduction to Arithmetic P rogression
An Arithmetic Progression (AP) i s a list of Example 2. Examine that the list of n umbers obtained from
n umbers i n which each term is obtained by adding following situation, will be in the form of an AP. ''Am o u n t left
a fixed number to the preceding term except the with Sandeep (in f) out of the total amount of f 1 2000 which he
first term. This fixed number is called the common had in the beginning, when he spends f 5 00 in the beginning
difference (d) of the AP. It can be positive, negative of every month . "
or zero . Sol. Given, total amount Sandeep had = � 1 2000
In o ther words, a list of numbers a 1 , a 2 , a3 , , a,, • • •
In the beginning of every month, he spend = ' 500
is called an arithmetic progression (AP) , if there
So, in the beginning of 1 st month, he had amount, t1 = � 1 2000
exists a constant number d (called common In the beginning of 2nd month, he had amount,
difference) such that ti = 1 2000 - 500 = ' 1 1 500
a2 = a 1 +d In the beginning of 3 rd month, he had amount,
a3 = a 2 +d t3 = 1 1 500 - 500 = ' 1 1 000
a 4 = a3 +d In the beginning of 4th month, he had amount,
t4 = 1 1 000 - 500 = ' 1 0500 and so o n .
Thus, the list of amount i s 1 2000, 1 1 500, 1 1 000, 1 0500, ...
a,, = a,, _ 1 + d and so on.
Here, ti - t1 = t3 - ti = t4 - t3 = - 500
Each of the numbers in th i s list is called a term. So, the above list of numbers forms an AP.
In general, a, a + d, a + 2d, a + 3 d , . . . represent
an arithmetic progression, where a is the first term M ethod to Find Common
and d is the common difference. This is called D ifference of an AP
general form of an AP. Let a n AP a 1 , a 2 , a3 , a 4 , b e given. Then , common difference of
• • •

I f number of terms in an AP is finite, then it is this AP will be the difference between any two consecutive terms, i.e.
called a fin i te AP , otherwise it is called an infinite common difference (d) = a 2 - a 1 or a3 - a 2 o r a 4 - a 3 and so on.
AP (i.e. number of terms is infinite) .
Note
-- I f the com mon difference of a n AP i s zero, i . e . d = O, then each term
of the AP will be same as the fi rst term of the AP.
M ethod to Check an AP
When a List of Numbers is G iven Example 3. Find the common difference of the following APs :
Someti mes, a list of numbers or sequence is given (i) 3 , - 2, - 7 , - 1 2 , . . . (ii) 1 1 , 1 1 , 1 1 , 1 1 , . . .
and we have co check that chis sequence is an AP or Sol. (t) Given, AP is 3, - 2, - 7, - 1 2, .. .
not. For this, we have to find the difference of any
Here, a1 = 3, a2 = -2, "3 = -7, a4 = - 1 2 and so on.
rwo consecutive terms. If this difference is same,
:. Common difference (d) = a 2 - a1 = -2 - 3 = -5
rhen given list of numbers or sequence is an AP,
{it) Given, AP is 1 1 , l l , 1 1 , . . .
o therwise n ot. Here, a 1 = 1 1, a 2 = 1 1, a3 = 1 1 a n d so on.
N ote For fi n d i ng r h e di fference between two
:.Common difference (d) = a2 - a1 = 1 1 - 1 1 = 0
consecutive terms. i . e . d. we should subtract the
p reced i n g term from the succeeding term even , if
c h e succeed i ng cerm is smaller.
TRY YOURSELF
Example 1. Examine that the sequence 1 3 , Q. l Examine that the sequence 6, 8 . . . . . . i s an AP. If, so it i s AP. then
2 , 4,
1 0 , 7 , 4 , . . . is a n A P. find common difference of given sequence. [Ans. Yes, 2)

Sol. Give n , seq uence i s I 3, I 0, 7, 4 , . . . . Q. 2 Find the common difference of an AP,


H ere,
Now,
a1 1 3, a2 = 1 0, t1.J = 7, a4 = 4
=

a2 - a1 = 1 0 - 1 3 ::: -3,
1 -21 , 2 -21 , 3 -21 , 4 2-1 . [Ans. I ]

a3 - a2 = 7 - 1 0 = -3, Q. 3 The given sequence -1 , -2, -3, -4, -5, . . form a n A P. I f s o , then find
.
tt1i - a3 = 4 - 7 = -3 and so on the common difference. [Ans. Yes, - 1 ]

S i n ce, difference berween rwo consecutive terms in Q. 4 Examine that the sequence s !2 , g !2 , 1 3 ..!,2 1 1 ..!2 . . i s a n AP. Also , find
.
each case is same, so rhe given sequence is an AP.
the common difference of it. [Ans. Yes, 4)
Arithmetic Progression

M ethod to Write an AP On subtracting che value of a,, from an+ J • we get


( 2n 2 +. 4 n + 3) - ( 2 n 2 + I )
When First Term and
Com mon Difference a re G iven = 2 n 2 + 4 n + 3 - 2 n 2 - 1 = 4 n + 2,

To write an AP, the minimum information required is to


which is not independent of n.
an = 2n 2 + l
know the first term a and the common difference d of the Hence, given lisc of numbers defined by is not
arithmetic progression. Then, put the values of a and d in an AP.
a, a + d, a + 2d, a + . . . to get required AP.
3d, Exam p le 6. Check that the list of n umbers defined by
Example 4 . Write an AP having 4 as the first term an = 3n + 2 is an AP or not. If it is an AP, then fin d the
and -3 as the common difference. common difference of it.
Sol. Given, first ccrm ( a ) = 4 and common difference (d) = -3 Sol. Given, a,, = 3n + 2 . . . (i)
On puccing the values of a and d in general form On replacing n by (n + I ) in Eq. (i), we gee
a, t1 + d, ti + 2d, 11 + 3d, ... , we gee a,, + 1 = 3(n + I ) + 2
4, 4 - 3. 4 + 2(-3), 4 + 3(-3), . . . = 3n + 3 + 2
4, I , 4 - 6, 4 - 9, . .. = 3n + 5 . . . (ii)
or 4, 1, - 2, - 5, ... On subtracting Eq. (i) from Eq. (ii), we gee
which is the required AP. a,, + 1 - a,, = 3n + 5 - 3n - 2
a,, + 1 - a,, = 3
Method to C h eck an AP which is independent of n.
When Algebraic Formula for Hence, the ·given list of numbers defined by a,, = 3n + 2 is
General Term is Given
an AP and common difference, d = a,, + 1 - a,, = 3
Sometimes, an algebraic formula for the general term of the
sequence is given and we have to check that this sequence is TRY YOURSELF
an AP or not.
For this, we use the following steps: Q. 1 Write an AP, whose first term and common difference are
[Ans. - 1.5, - 2, - 2.5, - 3, . . . J
Step
- 1.5 and - 0.5, respectively.
I Consider the algebraic formula as a,, (if not given)
Q. 2 Check that the list of numbers defined by t n 4n 3 + 3 is
Step
=

I I Replace n by (n + I) i n a,, co get a,, + I ·


[Ans. No, bccausc t n + 1 - tn
an AP or not. Also, give reason .
Step I I I Find a,, + 1 - a,, .
is noc independenc of n]
Step IV If a,, + 1 - a,, is independent of n, then the given Q.3 Examine that the sequence defined by an = 7n - .! is an
list of numbers (or sequence) is an AP, otherwise
3
AP or not. If it is an AP. then find the common d ifference
not. of it . [Ans. Yes, 7)

Example 5. Check that the list of n umbers defined by Q. 4 Check that the sequence obtained from the following
= 2n + l is an AP or not.
a0
2 situation, will be in the form of an AP Number of students
left in the school auditorium from the total strength of 1 000
Sol. Given, 1111 = 2 11 1 + I ·
[Ans. 1 000, 975, 950, 925, . . . ]
students when they leave the auditorium in batches of 25.
On replacing n by (n + 1 ) in 1111 , we gee
ll11 + l = 2 (n + 1) 1 + 1 Q. 5 Examine that the sequence obtained from the followin g
situation, will b e in the form o f a n A P or not .
= 2 (n 1 + 2 n + l ) + l[ ·: (a + b) 1 = a 1 + 2ab + b 1 ] 'The amount of money in the account every year when
= 2 11 2 + 4 n + 2 + 1 = 2 n 1 + 4 n + 3 � 1 00 are deposited annually to accumulate at compoun d
interest at 4 % per annum. ' [Ans. No]
NCERT FO L D E R 2 .1
1 In which of the following situations ,
(iv) We know that, Amount, A = P ( 1 + �)
100
/'

P = 10000, = 8%
does the list of numbers involved make an

= 10000 ( 1 + 100-.£ )1
R
=1, 2, 3, .
arithmetic progression and why?
Here, � per a n n u m and
(i) The taxi fare after each kilometre when the fare T . .
is f 15 for the first kilometre and f 8 for each

= 10000 x 100108
additional kilometre. Now, amount in 1 st year

10000 ( 1 + J!._)2
(ii) The amount of air pres�nt in a cylinder when a
vacuum pump removes 1/4 of the air remaining
in the cylinder at a time . = ' 10800
U50
(iii) The cost of digging a well after every metre of

0
Amount in 2nd year =
100
= 10000100x x108100x 108 = ;,:' 11 664
digging, when it costs for the first metre
and rises by f 5 for each subsequent metre .
an d so o n .

0000, 0800, 1 1664, ...


(iv) The amount of money in the account every
year, when H OOOO is deposited at compound
interest at 8% per annum.
(z) According to the question, the fare for journey of
Here, 10800 -10000
Thus, the list of numbers is I

1 1664 -10800 = 864 = 800


I

1 km, 2 km, 3 km, 4 km,


Sol. and
'15, , (15 +8), ' 0539,+ 2 x 8), ' (15 + 3 x 8)
15, 23,31,
... are respectively
and
i.e. the common difference is not same.

8
Hence, it does not form an AP.
so on, i.e. ... .
2
Here, each. term is obtained by adding

(iz) Let the amount of air present in the cylinder be


to the
preceding term except first term. So, it forms an AP. term a
Write first four terms of the
and the common difference
AP, when the first
d a re given

y as follows
a = -2,d =O
= =- a= =�
units. (i) a = l 0, d = 1 0 (ii)
According to the question,
= -0. 2 5
(iii) a 4, d (iv)· -1, d

y 4 = 3Y4
3
Remaining air in the cylinder after using vacuum
a=- 1 . 25 , d
a = l 0 d = 10
(v)
pump first time = !_ (z)
+ d, a + 2d, a + 3d,
_

Sol. Given, and


Remaining air in the cylinder after using vacuum We know that, a, a . .. arc
in AP

= -3y4 - -41 X -3y4 = 12y16-3y = -916y aa++2dda === 101010 ++ 20x=1020= 30


pump second time
Here,
I

a+3d =10 + 3 x l0 =40 AP 10, 20, 30


y, 34Y ., 916Y , ...
and so on. and
Thus, the list of n umbers is
and40.
Thus, the first four terms of the are

Here, -3y4 -y =- -y4 (iz)Given, a = - 2 a, a +d d,=0a .+ 2d, 3d,


and

9y 3y = 9y -12y =- 3y a=-=-22 + 0 =- 2 a + ...


We know that, a re i n AP.

163y 4 9y 16 3 16
_ Here,
a+d 2d == -2- 2 ++ 2(0)
aa++3d 3(0) == -- 22 -2 ,
and

Since,
4 -y 16 - 4Y
-:¢: and
Thus, the first four terms of the AP a re - 2, - 2

a = 4 d = -3
i.e. difference between two consecutive terms is not and -2.
same. Hence, it does not form an AP.
a, a + d, a + 2d, a + 3d,
(iii) Given, and

a =4
(iir) According to the question, the cost of digging for the We know that, . . . are in AP.
' 150, 50 + 50), 50 + 2 x 50),
fi rst metre, second metre, third metre, . . . are

a + 2d == 44 ++ (-3)
a+d = 4 -3 = l
Here,
respectively f (I f (I ...
and so on.
150, 200, 250, . 2( -3) = =4 4-6-9= =- - 5
50
...
a + 3d = 4 + 3( -3)
Th us, the list of n umbers is 2
Here, each term is obtained by adding f to the and
preceding term except first term. So, it forms an AP.
4, l, -2 -5.
Thus, the first four terms of the AP are
and
Arithmetic Progression

(iv) a = -1 and d = _!_2


Given, 4 Which of the following are APs? I f they form

We know char, a, a + d, a + 2d, a + 3d, ... are in an AP, then find the common difference d and
write three more terms.

Herc,
AP.
fl = -1 (i) 2, 4, 8, 16, . . .

fl+d =-1 + -2 =- -21


a+ 2d = -I + 2 m =0
(ii) 2, � , 3 , z. ' . . .

- .
l 2 2

and a+3d =-I + 3 m = �


(iii) 1.2, - 3.2, - 5.2, - 7.2, . .

+ + + ..
(iv) - .1 0, - 6, - 2, 2 , . . .
(v) 3 , 3 J2, 3 2 J2, 3 3 J2, .
(vi) 0.2, 0.22, 0.222, 0 .2222, . . .

Thus, chc firsc1 four cerms1 of che are AP


(vii) 0, - 4, - 8, - 1 2 , . . .

-1, - -2 , 0 and -.2 (viii) - .!.


1 1 1
-
2 · -2· 2· -2· ···

(v) Gi ven, a = - 1. 2 5 and d = -0. 2 5 ..


(ix) 1, 3, 9, 27, . . .

We know char, a, a + d, fl + 2d, + 3d, . . are in


a
(x) a, 2 a, 3 a, 4 a, .
(xi) a, er , ci3 , a4 , • • •
Here, a + da = -1.25 = -I.SO
AP.
J2, .JS, $8, ./32, . . .
a+ 2d == -1.-1.2255 +-0.2(-0.
25 25)
(xii)
(xiii) FJ. ./6, ../9, J12, . . .

and a +3d === --1. 50 =25)-1.75


- 1.2255 +-0.3(-0. (xiv) 12 3 2 5 2 7 2 r " •
I

(xv) 12 5 2 7 2 73,
I r

25 -of0.7che5 = - are2.00
Thus, che firsc four 1.ccrms Here, a1 = 2, al = 4, a3 = and = 16
I r I • • •

Sol. (t) a4

Now, a2 -a1 = 4 - 2 = 2
8
AP
-1. 2 5, - 1.50, - 1.75 and - 2.00. and
3 For the following APs, write the first term and Since, al --ala1 = a3 -4al= 4
� 8-

Hence, the given list5of numbers does not 7form an


-=F
AP.

Here, a1 = 2, a2 = z' a3 =3 and = 2


the common difference
(i) 3, 1 , 1. -3 , . . .
-
a4

1
(iz)

Now, al -a1 = -25 - 2 = 5 -4


(ii) - 5, - 1 , 3, 7, . . .
(iii) .!. . � . � .
13
2 1
--= -
2
5 6 -5
, ...

a3 -al =3 - -2 =-- =-2


3 3 3 3

First term =3
(iv) 0.6, 1 . 7, 2.8, 3 .9, . . .
2
Sol. (t)
Common difference (d) =2nd term -1st term
(a)
and a4 - a3 27 -3 = 7 2-6 =-21 =- --

=1 -3 Since, thethediffgievrence
Therefore, en list ofof numbers
successivforms
e termsan is andsame.its
=-2 AP
(iz) FiCommon
rst term (a)diffe=rence
-5 (d) =2nd term-1st term common difference, (d) is ..!..2
=-1 -(-5) Now, next three terms7 of chis1 AP are,
=-1 + 5 =4· = a4 + d = -2 + -2
a5

Firsc term (a) = .!.3 . a5 =a+ 4d =a+ 3d + d = a4 + d]


7 + 1
= 2 = �2 = 4
·'
(iit) """'

Common difference (d) =2nd5 4


[·:

=-3 --3 =-3 a6 = a5 + d = 4 + -21


term - 1 st term
1

(iv) rsccerm (a)diffe=0.rence6 (d) =2nd


FiCommon 8 + 1 =-9 and
=--
1.7
term - l st term
2 2 9 1 9 + 1 10
= - 0.6
= 1.1 = a6 + d = -2 + -2 = --2 = -2 =
a7 5
/

52 AllinOne MATH EMATICS C l a s s 1 0t h Te r m 11

(iii) Here, a 1 = - 1 .2, a2 = 3.2, � = - 5.2 and - Now, next r h ree renns o f r. h is A P a rc,
a = tt4 + d = - 1 2 + ( - 4 ) = -
-
fl4 = - 7.2 IG
Now, a 2 a1 = - 3.2 - ( - 1.2) = - 3.2 + 1.2 = - 2
- aG = 115 + d = - 1 6 + ( 4 ) - = 20
� - a2 = - 5.2 - ( -3.2) = - 5.2 + 3.2 = - 2 a7 = a6 + d = - 20 + ( - 4 ) = - 24

H �) � �
and

( viiz) Here, tt1 = - -, a 2 = - - , fl3 = - - ::i n d n


and
- 2
l l l l
a4 - � = - 7.2 - ( - 5 . 2 ) = · - 7.2 + 5. 2 =
2 2 2 2

- - � -( -�) � �
Since, the djfference of successive terms is same.
Therefore, the given list of numbers fo rms an AP and Now, a2 - a 1 =- - =- + =0
its common difference, d is - 2.

-(- ) - _ _-

Now, next three terms of this AP are,
a2 = = - + = 0
a5 = a4 + d = - 7.2 + (- 2) = - 9.2
a6 = a5 + d = - 9.2 + ( - 2) = - 1 1 .2
and a7 = a6 + d = - 1 1.2 + (-2) = - 1 3.2 and tl4 - fl3 = - _!_ _!_ = ! + _!_ = 0
.. 2 2 2 2
(iv) Here, a 1 = - 1 0, a 2 = - 6, � = - 2 and a 4 = 2
Si nce, rhe d i fference o f success i ve terms is s a 11 1 e .
- Therefore, the given list o f n u m be rs forms :. r n AP a n d
Now, a 2 a1 = -6 - (- 1 0 ) = - 6 + 1 0 = 4
-

4J a2 = - 2 - (- 6) = - 2 + 6 = 4
i ts com m o n d i fference, ( cl ) is 0 .
and a4 - � = 2 - (- 2) = 2 + 2 = 4
Now, n e x t three terms o f c h i s A P a re ,
Si nce, rhe difference of successive terms is same.
a5 = a4 + d = - - + 0 = - -
l I
Therefore, r.he given l ist of numbers forms an .AP and
its common difference, (d) is 4.
2 2
a6) = a5 + d = - - + 0 = - -
l I
Now, next th ree terms of this AP are,
2

-l
2
a5 = a4 + d = 2 + 4 = 6
= a6 + d = - l + 0 =
l l
a6 = a5 + d = 6 + 4 = 1 0 and a7

(ix) Here, a1 = I, a2 = 3, a3 = 9 and


and a7 = a6 + d = 1 0 + 4 = 1 4
a1 =
( v) Here, a1 = 3, a 2 = 3 + .J2., a3 = 3 + 2.f2 and
27
Now, a2 - a1 = 3 - I = 2
a,1 =3 + 3.fi. and � - a 2 = 9 - 3 = 6
Now, a2 - a1 =3 + .f2 -3 = 12 Si nce, a2 - a1 :F- a3 - a2
Hence, the given l is t of n u mbers d oes not: fo r m an A l�
a3 - a2 = 3 + 2.fi. - (3 + 12) = .fi.
(x) Here, a 1 = a, a2 = 2 a, r13 = 3 a and a4 = 4a
and a4 - a3 = 3 + 3.fi. - (3 + 2.f2 ) = .Ji Now, a 2 - a1 = 2a - a a =

Si nce, the difference of successive terms is same. � - a 2 = 3a - 2a = tl


Therefore, the given l ist of nu mbers forms an AP and and a4 - � = 4a - 3a = ti
its common difference, ( d) is .fi.. Si nce, rhe d i fference of s uccessive terms is sa me.
Now, next th ree terms of this AP are, Therefo re, the given list of n u m bers forms an A P and
i ts common d i fference, ( d ) is a .
a5 = a4 + d = 3 + 3.fi. + .J2 = 3 + 4.fi.
Now, next three terms o f chis A P a re ,
aG = a5 + d = 3 + 4.f2 + .Ji = 3 + 5 .fi_ a5 =
a4 + d = 4 a + a = S a
and a? = aG + d = 3 + 5 .f2 + .J2 = 3 + 612 a6 = a5 + d = 5 a + a = 6 a
a n d a 7 = a6 + d = Ga + a = 7 a
(vi) Here, a1 = 0.2, = 0.22 , � = 0.222 4
(x11
a2 and 2
� H ere, a 1 = a, a 2 = a , a3 = ti 3 an d a 4 = a

-
a4 = 0. 2222
2
-a a2- a1=
Now, a2 - a1 = 0.22 0.2 = 0 .02 Now, a 2 - a1 = a - a = a ( a - 1 )
and � 0 . 2 2 2 - 0.22 0.002 =
3 2 2
and a3 - a 2 = a - a = a ( a - 1 )
S i n ce, a2 - a1 :F- � - a 2
S i n ce, 2a3 - a 2 -:t=

Hence, rhe given list of n u m bers does n o t form a n AE


-,
Hence, the given l ist of numbers does not form an AP.

( vii) Here , a 1
- 4 -
0,
(xii) Here, .J2., a 2· = .JS = 2-Ji.,
= a2 =

a1

- =- - =-4
8 and
=

c13 = a4 = 12
Now, 4 tl3 ::: FiB = 3 12
- - - (- 4) = + = -
a2 a1 0
-8 4
- =-
a,3 a2 8 4
and J32 = 4-Ji.

I
=
a4 =
a4 a3 12 -
=-12 + = (- 8) 8 4
Now, a2 - a 1 = 2 .J2 - J2 = Ji
and -

Si nce, the d i fference of successive terms is same.


Therefore, the given l ist of n u mbers forms an A P and - a 2 = 3 .J2 - 2 -Ji. = Ji
- 4.
a3

i rs com mon d i fferen ce, is


(d) and a4 - a3 = 4 12 - 3 -Ji. = .J2
Arithmetic Progression 53
Since, the difference of successive terms is same. On substrac ting Eq (i) from Eq. (ii), we gcr
.

Therefore, the given list of numbers forms an AP and t,, + 1 - t,, = - 1 1n 2 - 22 n - 2 - 9 + I 1 11 2


its common difference, (d) is Ji. = - 2211 -11
Now, next three terms of this AP are, ::::> tll + 1 -
t,, =- I 1 ( 2n + I )
as = a4 + d = 4 .fi + .J2 = 5 J2 which is nor independent of n.
+ d = 5 .fi. + .J2 = 6.fi.
a6 = as Hence, given lisr of numbers defined by t11 = 9 - 1 1 11 2 is nor
and a7 = a6 + d = 6.fi. + .fi. = 7.fi. an AP.

(xiii) Here, a1 = .J3,


a = .J6, a3 = .J9 and a4 = JU. 6 Show that the sequence, defined by its nth
1 3 + n, forms an AP. Also, find the common
Now, a - a1 =
1
.J6 - .J3 term
4
= � - .J3 = .J3 (Ji - 1) difference of i t .

and 113 - a2 = ..{9 .J6 - Sol. Let nth term of the sequence = a,, , i.e.
= 3 - � = � c.J3 -Ji > 3 +- n
all = -
4
Since, a2 - a1 * a3 - 112
. . . (i)
Hence, the given list of numbers does not form an AP.
On replacing n by n + 1 in Eq. (i), we get
(xiv) Here, a1 = 1 1 = l , a = 31 = 9, 3+n+1
1 a,, + I =
113 = 5 1 = 25 and a4 = 71 = 49
4
4 +n
Now, a2 - a1 = 9 - 1 = 8 ::::> a,, + 1 = -
-
4
and a3 - a2 = 25 ....: 9 = 1 6
Since, a 2 - a 1 * a3 - a2 . . . (ii)
Hence, the given list of numbers does not form an AP. On subtracting Eq. (i) from Eq. (ii), we ger
2 4 + n 3 +-11
(xv) Here, a1 = 1 = 1, a,, + 1 - 11,, = -- - -
4 4
2
a2 = 5 = 25, 4 + 11 - 3 - n
a3 = 71 = 49 and a4 = 73 4

a2 - a1 = 25 1
- 1 = 24 ::::> a,, + ' - a,, = 4
Now,
a3 - a2 = 49 - 25 = 24
and a4 - a3 = 73 - 49 = 24 which is independent of 11.
' 3 + II
Since, the difference of successive terms is same. Hence, the given sequence defined by a11 = -4- for
ms
Therefore, the given list of numbers forms an AP and
its common difference, (d) is 24. 1
an AP and common difference, (d) = all + 1 - a,, = 4
Now, next three terms of this AP are,
as = a4 + d = 73 + 24 = 97
a6 = as + d = 97 + 24 = 1 2 1 7 For what value of k : 2k, k + 1 0 a nd 3 k + 2 are
and a7 = a6 + d = 121 + 24 = 1 45 in AP?

Sol. Given, numbers are in AP.


a 1 = 2 k, a 2 = k + l O and aj = 3 k + 2 arc
Additional Questions Let in AP.
5 Check that the list of numbers defined by a1 - a 1 = a3 - a2
t n = 9 1 1n2 is an AP or not.
-
::::> k + 1 0 - 2k = 3 k + 2 ( k + 1 0 ) -

::::> - k + 1 0 = 3k + 2 - k - 1 0
Sol. Given, tn = 9 - l ln1 .•. (i) ::::> - k + l 0 = 2k - 8
On replacing n by ( n + 1) in Eq. (i), we get ::::> 2k + k = 1 0 +8
2
t,, + 1 = 9 - 1 l (n 1 ) + ::::> 3k = 1 8
= 9 - l l ( n2 + 2n + 1) ::::> �
k=
3
=9 - l ln1 - 22n - 1 1
::::> k =6
= - l ln2 - 2211 - 2 . . . (ii) which is the required value of k.
TO P I C EXERCISE
Very Short Answer Type Questions Short Answer Type II Questions
1 Write first three terms of the sequence which 6 Find the common difference and the next two terms o f the
is defined by an = Sn + 3. AP : 6, 12. 18, ... .
2 Write the first term and common difference 1 Show that the sequence defined by an = 3n2 - 5 i s not
of an AP: 5. 8, 11, 14 . ... an AP.
3 Write the common difference of an AP:
8 Examine that the sequence 7. 13, 19. 25, . . . is a n A P. Also.
J3. 2J3. 3../3. ...
find the common difference.
Short Answer Type I Questions
Long Answer Type Questions
4 Which of the following form of an AP?
9 Write the first four terms in each of the list of numbers
Justify your answer. NCERT Exemplar
(i) 1. 1. 2. 2. 3, 3, ...
defined by
(i) an = (-1) n-1 n4
(ii) .J3. m. m. ../48. ...
(ii) an = 2n + 3
5 Write next three terms of the given AP:
1 0 Find the first four terms of an AP, whose first term i s -2
(a + b), (a + 1) + b, (a + 1) + (b + 1) . ...
NCERT Exemplar and the common difference is -2.

- - - - - - - - - - - - - - - - - - - -·- - - · - - · - · - - - · - · - · - · - · - · - · - · - · - · - - · - · - - · - · - · - · - · - - · - - - - - - · - - - - · - - - - - - - - - - - - - - - - -

Answers
1 . 8, 13 and 18 2. 5, 3 3. J3 4. (i) No (ii) Yes
5. (a + 2) + (b + 1), (a + 2) + (b + 2), (a + 3) + (b + 2) 6. d = 6 , The next two terms are 24 and 30.
8. Yes, 6 9 . (t) 1, - 1 6 ' 8 1 , - 2 56 (it) 5 , 7, 9, 1 1
1 0. - 2 , - 4. - 6 and - 8

TOP IC 8 nt h Term of an AP
' i: r • ·P fi rst term of an AP is 'a' and i ts common Sol. Given, sequence is 7, 3, -1, - 5, . .. .
: ac n ce is 'd', then its nth term is given by the
Here, 3 -7 = - 4
I 11 : i.
-1 -3 = - 4
,:, = 11 + ( 1J l)d -5 + 1 = - 4

, f · a n A J> i s also cal k :<..J i t s general term. So, given sequence is an AP, in which a = 7 an d d = - 4.

1 . v· · A P , rhen nrh rerm is known


Since, n th term, an = a + (n - 1 )d
J1 .. He 11 c ._ . ih : � an

m of \ I ) .i ;1 d /. wh ich is given by the


On putting n = 20, we get
a� i .1 - . ;: w

form u l . :
a20 = a + ( 20 - 1 )d
= 7 + 1 9( -4 ) [·: a = 7, d = -4]
l = tt + (n - l ) d I
= 7 -1 9 x 4
where, a is fi rst term a n d d i s co m n 1 1 , , . J i l-frrt' llle. = 7 - 76

Example 1. Fin d th e 2 0 111 ! P rm n f th� c-.;�quencP = -69


r . • "'"\ rp: 20th term " ' given sequence is -69.
7, 3, - 1 , -5 . . .
Arithmetic Progression 55 I
Sol. Let a be the first term and d be the com mon difference of the
Types of P ro b lems Based
AP, whose 7th term is - 1 and 1 6th term is 1 7.
on nth Term of a n A P Thus, a7 = - 1 and a 1 6 = 1 7
[ TyPe I ) Problems Based on Finding when nth n
Now, a,, = a + (n - I )d
Term or Last Term and AP, are Given => a + ( 7 - l )d = - 1
In this type of problems, an AP containing n terms is given
=> a + 6d = - 1 . . . (i)
(or nth term of an AP is given) or AP with last term is given
and a + (1 6 - I )d = 1 7
=> a + 1 5d = 1 7 ... (ii)
and we have to find the value of n.
O n subtracting Eq. (i) from Eq. (ii), we get
Exa m p le 2. Which term of the AP: 2 1, 1 8, 15, . . . a + 1 5d - a -6d = 1 7 + I
is -8 1 ? => 9d = 1 8 � d = 2
.
Sol. Given, AP is 2 1, 1 8 , 1 5 , . . .
On substituting d = 2 in Eq. (i), we get
a + 6 x 2 = -1
Here, a = 2 1 and d = 1 8 - 2 1 = -3
a + 12 = -1 � a = -13
Let nth term of given AP be - 8 1.
Hence, general term, a,, = a + ( n - I )d
Then, an = -8 1
= - 1 3 + (n - 1 )2 [·: a = - 1 3 and d = 2]
=> a + (n -l )d = -8 1 = - 13 + 2n - 2 = 2n - 1 5
On putting the values of a and d, we get
(Type III) Word Problems
In this type of problems, first we write the AP or list of
2 1 + (n - 1)(-3) = -8 1
=> 2 1 -3n + 3 = -8 1
=> 24 -3n = -8 1 numbers (sequence) with the help of given information and
=> -3n = -8 1 - 24 = - 1 05
then find the required value.
- 1 05 Exa m p le 5. How many n umbers of two digits a re
=> n= = 35
-3 divisible by 7?
Hence, 35th term of given AP is 8 1. - Sol. Two-digits numbers are 1 0, 1 1, 1 2, 1 3, 1 4, 1 5, ... , 97, 98, 99 in
Example 3. How many terms are there in the which only 1 4, 2 1 ,28, ... , 98 are divisible by 7.
seq u en ce 3 , 6, 9, 1 2 , . . . , 1 1 1 ? Here, 21 - 14 = 28 - 21 = 7
Sol. Given, sequence is 3, 6, 9, 1 2, ... , 1 1 1 .
So, this list of numbers forms an AP, whose first term ( a ) = 1 4,
common difference (d) = 7 and nth term = 98.
Here, 6 - 3 = 9 - 6 = 1 2 - 9 = 3 . · a + (n - l)d = 98 [·: a,, = a + (n - 1 )a']
So, i t is an AP with first term, a = 3 and common difference,
d = 3. Let there be n terms in the given sequence.
=> 1 4 + (n - 1)7 = 98 � 1 4 + 7n - 7 = 98
Then, nth term = 1 1 1 91
=> 7n = 9 1 � n = - = 1 3
=> a + (n - l)d = 1 1 1 7
=> 3 + (n - 1) x 3 = 1 1 1 Hence, 1 3 numbers of two digits are divisible by 7.
=> 3(1 + n - 1) = 1 1 1
Example 6. A sum of � 2 000 is invested at 7% simple
111
=> n = - => n = 37 interest per year. Calculate the interest at the end of
3 each year. Do these interest form an AP? If so, then
Hence, the given sequence contains 37 terms. find the interest at the end of 20 yr making use of this
[ Type II] Problems Based on Finding the AP or nth fact.
Sol. Given, initial money P = � 2000
Term or Both, when its Two Tenns are Given
Rate of interest, R = 7% per year; Time, T = 1 , 2, 3, 4, . . .
In this type of problems, two terms (or relation between two
terms of an AP) is given and we have to find the AP and nth
We know that, simple interest i s given by the following formul a
PR T
term of AP. SI =
1 00
Example 4. Deter1!1ine the general term of an AP 2000 x 7 x 1
whose 7th term is - 1 and 16th term is 17. :.SI at the end of 1 st year = = � 1 40
1 00
Here, with the help o f given two terms o f AP, we will form two
equations in a and d and then solve them to find the values SI at the end of 2nd year = ----
2000 x 7 x 2

of a and d. By using these values of a and d, write general


. 1 00
term of AP.
= � 280
56 I Allinone MATHEMATICS Class 1 0th Te rm 11

SI at the end of 3rd year =


2000 x ? x 3
= � 420 Example 7. Determine the 1 0th term from the
1 00 end of the AP : 4 , 9 , 1 4 , . . . , 254 .
Sol. Given, AP is 4, 9 , 1 4 , ... , 254.
Thus required list of numbers is 1 40, 280, 420, . . . .
Herc, 280 - 1 40 = 420 - 280 = 1 40 Here, I = last term = 254
So, above list of numbers forms an AP, whose first term d = common difference = 9 - 4 = 5
( a ) = 1 40 and common difference (d) = 1 40.
:. 1 0th term from che end = / - (1 0 - l ) d = I - 9d
Now, SI at the end of 20 yr will be equal co 20th term of the
= 254 - 9 x 5
above AP.
a 0 = a + ( 20 - 1 )d = 1 40 + 1 9 x 1 40
= 254 - 45
2 = 209
= 1 40 + 2660 = 2800
Hence, the interest at the end of 20 yr will be � 2800. Alternate Method
On reversing che given AP, new AP is 2 5 4 , . . . , 1 4, 9, 4 .
TRY YOURSELF Here, first term ( a ) = 254
and common difference (d) = -5
Q. l Find the 1 8th term of the sequence 1 , 4, 7, 1 0, . . . Now, 1 Och term of new AP = a 10
[Ans. 52]
= 254 + (1 0 - 1 ) ( - 5 )
Q. 2 In an AP. , if a = 3.5, d = 0, n = 1 01, then find the value of an . = 254 - 9 x 5 = 209
[Ans. 3.5]
Hence, l Och term from che end of given AP is 209.
Q.3 The 1 1 th term of an AP is 80 and the 1 6 th term is 1 1 0. Find
the 31st term . [Ans. 200]
TRY YOURSELF
Q. 4 A man starts repaying a loan as first instalment of � 1 00. If
he increases the instalment by � 5 every month, then Q. 1 Find the 8th term from the end of the AP
what amount he will pay in the 30th instalment. [Ans. 24 5] 7, 1 0, 1 3 , . . . , 1 84. [Ans. 1 63)

nth Term from the End of an AP Q. 2 Find the 49th term from the end of the AP
'a'
Let be the fi rst term, 'd' be the common difference and 2, 4, 6, 8, . . . , 1 00. [Ans. 4]

't be the last term o f an AP, then nth term from the end can Q.3 In an AP. if a = 1 0, d = 5 and n = 1 00, then find the
be fo und by the formula value of a100 and also find the 50th term from the end.
I nth term from the end = l - (n - l)d I [Ans. 505, 260]

NCERT FO L D E R 2 .2
1 Fill in the blanks in the following table, given Sol. (t) Here, a = 7, d = 3 and n = 8
that a is the first term, d the common difference The nth term of an AP is a n = a + (n - 1 ) d
and a0 the nth term of the AP. .. a8 = 7 + (8 - 1 ) 3 = 7 + 7 x 3
= 7 + 2 1 = 28
a d n an
{it) Here, a = - 1 8, n = 10 and an = 0
--- -- - - - - ----

(1) 7 3 8
We know chat, the nth term of an AP is
an = a + (n - I ) d
(it) - 18 - - - - · - - - - - ---- ----
10 0 On puccing che given values, we gee
(iii)
-

-3 18 -5 0 = - 1 8 + (1 0 - 1 ) d

(iv)
=> 18 = 9 d
- 1 8.9 2.5 3.6
(v)
18
3.5 0 1 05 => d=- =2
- · - - - - - - --- - - - --- · 9
Arithmetic Progression

(iir) Herc, 11 = 1 8, d = -3 and an = - 5


We know char, che nch cerm of an AP is
3 In the following APs, find the missing terms
of the boxes.
CCE 20 1 5
a ,, = a + (n - 1 ) d
On puccing chc given values, we gee (i) 2, 0, 2 6
- 5 = a + (1 8 - 1 ) (- 3) (ii) o 1 3, o 3
.
.

=:> - 5 = a + l 7 ( - 3} =:. - 5 = a - 5 1 (iii) 5, o o 9 .!.


2
. .

=:. It = - 5 + 5 1 = 46
(iv) - 4 . 0. o. o. o. 6
(iv) Herc, a = - 1 8.9, d = 2.5 and an = 3.6
(v) O. 3 8, O. O. O. - 22
We know char, chc mh cerm of an AP is
a ,, = a + (11 - I ) d Sol. (1) We know char, a, a + d, a + 2 d, . . . is rhe general form
of an AP.
On puccing chc given values, we gee
We have, a = 2 and a + 2d = 26
3.6 = - 1 8.9 + (11 - 1 ) 2.5
=:> 2 + 2d = 26 [ ·: a = 2 ]
=:. 3.6 + 1 8.9 = (11 - I ) 2.5
=:.
=:>
2 d = 26 - 2 = 24
22.5 = (11 - 1 ) 2.5
=:.
24
=:.
22.5 d = - = 12
II - 1 =
2.5 2
=:> 11 - 1 = 9 Hence, the m issing term = a + d = 2 + 1 2 = 1 4
=:. (iz) We know char, a, a + d, a + 2d, a + 3 d, is the
general fo rm of an AP.
11 = 9 + 1 = 1 0
(v) Herc, a = 3.5, d = 0 and n = I 05
:. We have, a + d = 13 . . . (i)
We know char, che nch cerm of an AP is
and a + 3d = 3 . . . (ii)
a ,, = a + (n - 1 ) d
On puccing chc given values, we gee On subtraccing Eq. (i) from Eq. (ii), we gee
a,, = 3.5 + (I 05 - 1 ) x 0 = 3.5 + 0 = 3.5 2d = - 1 0
=:. d =-5
2 Choose the correct choice in the following On puccing d = - 5 in Eq. ( ii) , we get
and justify. a + 3 (- 5 ) = 3
(i) 3 0 th term of the AP : 1 0, 7, 4, . . . is a -15 =3
(a) 97 (b) 77 a =3 + 15 = 18
(c) - 7 7 (d) - 87 Hence, the m issing terms are a = 1 8 and
(ii) 1 1 th term of the AP : - 3, - .!. , 2, . . . is a + 2d = 1 8 + 2 ( - 5) = 1 8 - 1 0 = 8.
2 (iit) We know chat, a, a + d, a + 2d, a + 3 d, . . . is che
(a) 28 (b) 22 general form of an AP.
(c) - 38 (d) - 48 ! :. We have, a =5 . . . (i)
2 1 19
SoL (� ( � HB� fi�c rerm (a) = I O and a + 3d = 9 - = - . . . ( i i)
2 2
Common difference (d) = 7 - 1 0 = -3
On subcraccing Eq. (i) from Eq. (ii), we gee
and number of terms (n) = 30
19 1 9 - 1 0 .2.
We know chat, the nch term of an AP is 3d = -5 = =
2 2 2
a,, = a + (n - 1 ) d
9
1130
= I O + (30 - 1 ) ( - 3) =:. 3d = -
2
= I O + 29 (- 3) = 10 - 87 = - 77
1 9 3
(iz) (b) Here, fi rsc cerm (a) = - 3 =:> d=-X - =-
3 2 2
Common difference (d) = - .!_ - (- 3) Hence, rhe missing terms are
2 3 r � j
- 1 + 6 2_ a + d=5 + -
.!..
=- +3 = = 2
2 2 2 10 + 3 13
= = = 6 .!_
2 2 2
and number of terms (n) = 1 1
Now, nth term of an AP is an = a + (n - 1 ) d and a + 2d = 5 + 2 x �=5 +3 =8
2
11 1 1 = - 3 + (1 1 - 1 } - (iv) We know chat, a, a + d, a + 2d, a + 3d, a + 4 d.
5
2 a + 5d,. .. is the general form of an AP.
= -3 + 10 x
2- :. We have, a =-4 . . . (i )
2 and a + 5d = 6 . . . (ii)
- - '.1 _L 'l i:; - TJ
58 \ All{nOne MATHEMATICS Class 10th Term 11

On putting a=-4 in Eq. (ii), we get (ii) Suppose, there are 11 terms in the given AP.
-4+5d=6 So, we have an= - 47
10 Also, first term (a ) =18 and
=> Sd=6+4 =10 . => d= =2
s common difference (d)=15 _!_ -18 = � - 18
Hence, the missing terms are, a+d = - 4+ 2 =- 2 2 2
a+2d=-4+2x2=-4+4=0 31 -36 -5
:::: ---
a+3d=-4+3x2=-4+6=2 2 2
and a+4d=-4+4x2=-4+8=4
(v) We know that, a, a+d, a+ 2d, a+3d, a+4d,
18+ ( 11 �I) ( ;)
-
= - 47 [·:a+ (11 - 1) d =a.I

( ;)
a+Sd, ... is the general form of an AP.
-
:. We have, a+d =38 ... (i) => (n -1)= -47 -18 = - 65
and a+Sd=- 22 . .. (ii)
On subtracting Eq. (i) from Eq. (ii), we get 2
=> (n -1)= -65x - = -13 x (- 2) = 26
60 5
4d =- 60 => d=- =-15
4 => n = 26+1= 27
On putting d = -15 in Eq. (i), we get Hence, the given AP has 27 terms.
a+(-15)=38
6 Check whether -150 is a term of the AP:
=> a -15 =38 =>a =38+15 =53
Hence, the missing terms are a =53 11, 8, 5, 2, ... :
a+ 2d=53+ 2x(-15)= 5 3 - 30= 23 Sol. Here, a =11
a+3d=53+3x(-15)=53 -45 =8 and d =8 -11= -3
and a+4d =53+4x( -15)= 53 -60=- 7
Assume that, -150 be the nth term of the given AP.
4 Which term of the AP: 3, 8, 13, 18, ... is 78? We know chat, the nth term of an AP is

Sol. Given, 3, 8, 13, 18, ... are in AP. a,,=a+ (n -1) d


=> -150 =11+ (n -1) ( - 3 )
Here, first term (a) =3
=> - 3 (n-1)=-150 -11 = -161
and common difference (d)=8 -3 = 5
Let nth term, an= 78 => n-1= �
3
.. a+(n-1) d= 78
164
=> n= �+1=
=> 3+(n-1) 5 = 78 3 3
=>
(n-1) 5= 78 - 3 But nshould be a positive integer. So, - 150 is not a term of
=> 5 (n -1)= 75 the given AP.
=> n-1=15
7 Find the 31st term of an AP, whose 11th term
=> n=15+1
is 38 and the 16th term is 73.
=> n=16
Sol. Let a be the first term and d be the common difference of
Hence, 16th term is 78. an AP.
5 Find the number of terms in each of the We know chat, the nth term of an AP is
an=a+ (n-1) d
following APs.
Then, 11th term, a11 =a+10 d = 38 . . . (i)
(i) 1, 13, 19, . .. , 20s (ii) 1a,1s.!.,13, ... ,
-
47
[·:a11 38, given]
2 =

(1) Suppose, there are n terms in the given AP. and 16th term, a16=a+15 d = 73 . .. (ii)
Sol.
:� Here, nth term (an)= 205. [·: a16 = 73, given]
On subtracting Eq. (i) from Eq. (ii), we get
First term (a)= 7 and
5d =35
common difference (d) =13 - 7 =6
35
a + (n-1) d =an => d =-=7
7+ (n-1) 6 = 205 5
6 (n -1)= 205 - 7 On putting the value of din Eq. (i), we get
6 (n -1)=198 a+10x7=38
198 => a = 38 - 70 = -32
=> n-1=-=33
6 Now, 3 lsc term, �1 = a+30 d
=> n= 33+ 1= 34 =-32+30x7
Hence, the given AP has 34 terms. =-32+ 210=178
Arithmetic Progression

8 An AP consists of 50 terms of which 3rd term is


·'
(a + 1 6d) - (a + 9d) = 7
1 2 and the last term is 106. Find the 29th term. 7d = 7 => d = l
Hence, the common difference of this AP is 1 .
Sol. Let t1 be rhc first term and d be the common difference
of an AP. 11 Which term of the AP: 3 , 1 5 , 2 7 , 3 9 , . . . will be
Now, rhe mh term of an AP is t1,, = a + (n - 1) d 132 more than its 5 4 th term? Given AP is
• • 11.\ = ti + 2d = 1 2 [·: a3 = 1 2, given] . . . (i) 3, 15, 27, 39, . . . .
and n5 0 = ti + 49 d = 106 . . . (ii) Sol. Here, first term (a) = 3
[ ·: a50 = 106, given ] and common difference ( d) = 1 5 - 3 = 1 2
On subtracting Eq. (i) from Eq. (ii) , we get Then, 54ch term, a54 = a + 53 d = 3 + 53 x 1 2
. 94 [·: a,, = a + (n - I ) d]
47 d = 94 ::::) d = - = 2 = 3 + 636 = 639
47
Let a,, be 1 32 more than its 54th term.
On purring the value of d in Eq. (i) , we get Then, a,, = a54 + 1 32 [ by given condition]
a + 2 x 2 = 12 =:> a,, = 639 + 1 32
::::) t1 = 1 2 - 4 = 8 =:> a,, = 77 1
Now, 29th term, a29 = 11 + ( 29 - l) d =:> a + (n - I) d = 77 1 [·: a,, = a + (n - I ) d]
= 8 + 28 x 2 = 8 + 56 = 64 = 3 + (n - 1) 1 2 = 77 1
=:> 1 2 (n - 1 ) = 77 1 - 3
9 If the 3 rd and 9th terms of an AP are 4 =:> 1 2 (n - 1) = 768
and - 8 respectively, then which term of this 768
AP is zero? =:> n - 1 = - = 64
12
Sol. Let a be the first term and d be the common difference of n = 65
an AP. Hence, 65th term is 1 32 more than the 54th term of this AP.
·: The nch term of an AP is a,, = a + (n - 1 ) d
1 2 Two APs have the same common difference.
.. a3 = a + 2d = 4 [·: a3 = 4 , given ] . . . (i) The difference between their 1 00th term s is 1 00 .
and a9 = a + B d = - 8 [·: a9 = -8, given] . . . (ii) What i s · the difference between their 1 OOOth
On subtracting Eq. (i) from Eq. (ii), we gee terms?
-12 Sol. Lee the two APs be ap a2, a3 , ... , a,, and q , b2 , � , ... , b,,.
6d = - 1 2 ::::) d = - = - 2
6 Also, lee d be the same common difference of two APs. Then,
On puccing the value of d in Eq. (i), we get the nth term of first AP is
a + 2 x (- 2) = 4 a,, = a 1 + (n - I) d
=:> a-4=4 and the nth term of second AP is
a =4 + 4 =8 b,, = q + (n - 1 ) d
Lee the nth term of chis AP be zero, i.e. Now, a,, - b,, = [a1 + (n - I) d] - [q + (n - I ) d]
a,, = 0 =:> a,, - b,, = a1 - q , 'V n e N
a + (n - 1 ) d =0 · · a1 00 - b1 00 = a1 - q = 1 00 [given]
8 + ( n - 1) ( - 2 ) = 0 [·: a = 8, d = -2 ] So, a1 000 - biooo = a1 - bi
=:> a1 0 00 - b1000 = 100 [·: a 1 - q = 1 00]
(n - 1) (- 2) = - 8
Hence, the difference between their 1 OOOch terms is also 1 00
n - 1 = -B = 4 for all n e N.
-2 .,"'"),.
:F
�-:I=-
13

How many three-digit numbers are divisible
n =4 + 1 =5
·�
by 7? CBSE 201 3
Hence, 5th term of chis AP is zero. ���
Sol. We know char, 105 i s the first a n d 994 i s t h e lase three-digit
1 0 The 1 7th term of an AP exceeds its 10th term numbers divisible by 7. Thus, we have to determine the .,'· '-�.
number of terms in the list 1 05, 1 1 2, 1 1 9, ... , 994. '--
:.
by 7 . Find the common difference.
',_,,..h-_,
Sol. Lee a be the first term and d be the common di fference of Clearly, the successive difference of the terms is same. So,
above list of numbers forms an AP, with common difference �·l, i�i
an AP.
(d) = 1 1 2 - 1 05 = 7 f� •I

Now, according co the question, a17 = a1 0 + 7


Lee there be n terms in the AP.

1/
I•' '!
��
= �? - � o = 7 i� ;��'.
r�
.. "'
*=-- ·�-
Then, nth term = 994
=:>
tl
[a + (1 7 - l)d] - [a + (1 0 - l)d] = 7
=:> 105 + (n - 1 ) 7 = 994 [· : a11 = + (n - 1 ) d] ' ::;.u_ ,· · . .
··•
[·: a,, = a + (n - l )d] ; ...::..:: .-��·
60 \ AllinOne MATH EMATICS Class 1 0th Term 11

=> 7 (n - 1 ) = 994 -105 => 2d = 1 2


=> 7 (n - 1 ) = 889 => d =6
889 = 1 27 O n putting d = 6 in Eq. (i), we gee
=> . n -1 =- a + 2 x 6 =16
7
=> n = 1 27 + 1 = 128 => a = 16 - 12 = 4
Hence, there are 1 28 numbers of three digits which are We know that, general form of an AP is
divisible by 7. a, a + d, a + 2d, a + 3d, . . . .
1 4 How many multiples of 4 lie between 1 0
Then, the required AP is
and 250?
4, 4 + 6, 4 + 2 x 6, 4 + 3 x 6, ... , i.e. 4, I 0, 1 6, 22, ... .
Sol. We know that 1 2 is the first integer between 10 and 250, 1 7 Find the 2 0th term from the last term of
which is a multiple of 4. Also, when we divide 250 by 4, the the AP: 3, 8 , 1 3 , . . . , 2 53 .
remainder is 2. Therefore, 250 - 2 = 248 is the greatest Sol. Given, last cerm (/) = 253
and common difference ( d) = 8 - 3 = 5
integer divisible by 4 and lying between I 0 and 250.
Thus, we have to find the number of terms in an AP,
whose first term (a) = 1 2 , last term (/) = 248 and :.20th cerm from the end = I - ( 20 - 1) d ( ·: II = 20)
common difference (d) = 4. = 253 - 1 9 x 5
Let the nth term of this AP is an � 248 = 253 - 95 = 1 58
Then, 1 2 + (n - 1) 4 = 248 [·: an = a + (n - 1) d] 1 8 The sum of the 4th and 8th terms o f a n AP is
=> 4 (n - 1) = 248 - 1 2 24 and the sum of the 6th and 1 0th terms is
=> 4 (n - 1) = 236 4 4 . Find the first three terms of the AP.
236 Lee a be the first term and d be the common difference of
=> n -1 = = 59 Sol.
4 an AP.
=> n = 59 + 1 = 60 Given, a4 + a8 = 24
Hence, there are 60 multiples of 4 lying between 1 0 and . . (a + 3d) + (a + 7 d) = 24 [·: a11 = a + (11 - 1) d ]
250. => 2a + l Od = 24
1 5 For what value of n, the nth terms of the APs => a + 5d = l 2
63 , 65, 67 , . . . and 3 , 10, 1 7 , . . . are equal? [dividing both sides by 2] . . . (i)
Given, APs are 63, 65, 67, ... and 3, 1 0, 17, ... .
and a6 + a10 = 44
Sol. (a + 5d) + (a + 9d) = 44
Here, first term of first AP (a1 ) = 63
=>
2a + l 4 d = 44
Common difference of first AP (d1 ) = 65 - 63 = 2
=>
=> a + 7d = 22
First term of second AP (b1 ) = 3 [dividing both sides by 2] . . . (ii)
and common difference of second AP (d2 ) = 1 0 - 3 = 7 On subtracting Eq. (i) from Eq. (ii), we gee
According to the question, nth term of both APs are equal. 2d = 1 0 => d = 5
. . 63 + (n - 1) 2 = 3 + (n - 1) 7 [ ·: a,, = a + (n - 1) d] On purring d = 5 in Eq. (i), we get
=> 7 (n - 1) - 2 (n - 1) = 63 - 3 a + 25 = 1 2
=> (n - 1) (7 - 2) = 60 => a = - 13
=> 5 (n - 1) = 60 Hence, the first three terms are
(n - 1) = 60 a, (a + d) and (a + 2d)
5 = 12
=>
1.e. - 13, (- 13 + 5) and (- 1 3 + 2 x 5)
=> n = 12 + 1 = 13 1.e. - 13, - Sand - 3.
Hence, the 13th term of the two given APs are equal. 1 9 Subba Rao started work in 1 995 at a n annual
1 6 Determine the AP whose 3rd term is 1 6 and the salary of � 5000 and received an increm ent o f
7th term exceeds the 5th term by 1 2 . � 200 each year. In which year, did his income
Sol. Lee a be the first term and d be the common difference of reach � 7000?
an AP. Sol. The annual salary received by Subba Rao in the years
Given that, the third term of the AP or 1995, 1996, 1997, .. . are is � 5000, � 5200, � 5400, ... ,
� =16 respectively.
=> a + 2d = 16 [·: an = a + (n -l)a'] . . . (i) Here, the list of numbers 5000, 5200, 5400, .. . , forms
Also, it is given char an AP.
7th term of an AP = 1 2 + 5th term of an AP, i.e. a2 - a1 = � - a2 = 200
a7 = 12 + a5 Lee nth term of this AP, an = 7000
=> a7 - a5 = 1 2 => 7000 = a + (n - 1 ) d
=> (a + 6d) - (a + 4d) = 1 2 [·: a,, = a + (n - I ) d]
Arithmetic Progression 61

7000 = 5000 + (n - 1 ) (200) (n - l) x 1.75 = 20.75 - 5 = 1 5.75


200 (n - 1 ) = 7000 - 5000 = 2000 1 5.75
n 1 = _ =9
n - 1 = -- = 1 0
2000 1.75
200 n =9 + 1 = 10
::::::) n =lO + l =l l
Thus, in 1 1 th year of his service or in 2005, Subba Rao received Additional Question
an annual salary f 7000. 2 1 Find p, if the given value o f x is the pth term o f th e
20 Ramkali saves �5 in the first week of a year and following A P :
then increased her weekly savings by ' 1.7 5. If in 25, 50, 75 . 1 00, ... ; x = 1 000
the nth week, her weekly savings becomes � 20.7 5, Sol. Given AP is 25, SO, 75. 1 00, ... , where first ccrm, a = 25,
then find the value of n. common difference, d = 50 - 25 = 25 and x = pth te rm o f
Sol. Ramkali 's savings in the subsequent weeks are respectively this A P.
f 5. f 5 + f 1.75. f 5 + 2 x f 1.75, ' 5 + 3 x ' 1.75, ... .. a + (p - l)d = 1 000 [·: nth term = a + (n - l )d]
i.e. f 5, f 6.75, f 8.50, � 1 0.25, ... ::::::) 25 + (p - I ) x 25 = 1 000
Clearly, Ramkali's savings forms an AP with first term (a) = 5 => 25(1 + p - 1) = 1 000
and common difference (d) = I.75. 1 000
Thus, in nth week, her savings will be � S + (n - 1) x 1.75 ::::::) p = � ::::::) p =40
::::::) s + (n - n x 1.75 = 20.75
Hence, the required value of p is 40.
[·: in the nth week saving amount = f 20.75)

TO P I C EXERCISE
Very Short Answer Type Questions 6 Which term of AP : 3 , 8. 13. 18 . . . will be 130
. more

1 If d = - 4, n = 7 and an = 4, then find the value of a. than its 31sc term?


7 Find the 6th term from the end of the AP: 17. 14. 1 1 .
. .. (- 40).
NCERT Exemplar

2 Find 11th term of AP : - 5, - 5 I 2, 0, 5/2, ...


NCERT Exemplar Long Answer Type Questions
3 Check 0 is a term of the AP : 31, 28, 25, ... . 8 A sum of t 5000 is invested at 8% s i m p l e i n t e rest pe r
annum. Calculate the interest at t h e e n d of each
Short Answer Type I Question year. Do these interests form a n A P ? F i n d t h e
4 The first term of an AP is 5, common difference is 3
interest at the end of 30 yr.
and the last term is 80. Find the number of terms. 9 The taxi fare after each kilometre. when t he fa re i s �
15 for the first kilometre and t 8 for each .:i cl d i t io n a l
Short Answer Type II Questions kilometre, does not form an A P a s the t o t a l f.J rc ( i n � )
5 In a flower bed, there are 43 rose plants in the first after each kilometre is 15. 8. 8 8 . . . . . I s t h e s t d t c rn c n t
.

row, 41 in the second, 39 in the third and so on. true? Give reasons. N C F RT Exl.'rnplar

There are 11 rose plants in the last row. How many 10 How many numbers lie between 10 a nd 300. w h ic h
rows are there in the flower bed? divided by 4 leave a remai nder 3 ?

Answers
1. 28 2. 20 3. No 4. 26 5. 17

6. 57ch 7. - 25 8. Yes, f 12000 9. No 1 0. 73


TOPIC 9 Su m of Fi rst n Terms of an AP

I � I
If first term of an AP be 'a' and its common difference is 'd', ll
:::::> S = - [a + a ]
II I/
then the sum of its first n terms Sn , is given by the formula 2
s. = [2a + (n - 1) d ] where, a = nth term = a + (11
,1 - I) d

[ � J
Ex a m p le 1 . Find the s u m of th e first 2 2 terms of t h e
This formula can also be written as AP : 8 3 - 2, . . .
I I

Sol. G iven, AP is 8, 3, - 2, . . .
s. = [ a + a., ]
Here, first term, ( a ) = 8
where an = nth term of the AP Common difference, ( d ) = 3 - 8 = - S and 11 = 22
II
If last term of the AP is l , then sum of its n terms is given by ·: Sum of first 11 terms, ( S,, ) = - [ 211 + ( n I ) ti ]
-

I � I
2
this formula
:.Sum of firsc 22 terms, ( S,--, ) = - [ 2 x 8 + ( 22 - 1 ) x ( - 5 ) ]
22
s. = [ a + /J 2
= l I [I 6 + 2 I x <- " >I
Proof Consider, an AP a 1 , a 1 , a3 , • • • , an - t , a11 , whose first = I I [ I 6 - I 05 ]
= I I ( - 89) = 979 -

Hence, sum of first 22 terms of an AP is - 979.


term is a and common difference is d. Then,
a 1 = a, a1 = a + d, a3 = a + 2d, . . . ,
Exam p le 2. Find the sum of first 20 terms of an A P i n
which a = 1 a n d 20th term = 58.
an- I = a + (n - 2) d and an = a + (n - l) d
Let S denotes the sum of the first n terms of this AP.
Sol. G iven, a = 1, l = 58 and 11 = 20
Then, S = a 1 + a l + a3 + . . .+ an - 1 + an
� S = a + (a + d) + (a + 2d) +. . . Now, sum of 20 ccrms, S-,11 = l2O (I + 58) [·: S ,,
= !!_ ( 11 + /)]
+ {a + (n - 2) d } + {a + (n - l) d} . . . (i)
2
= I O (59) = 590
On writing the above series in reverse order, we get
S = {a + (n - l) d } + {a + (n - 2) d} + . . . + (a + d) + a . . . (ii) Example 3. Find the sum of first 24 terms of an AP,
On adding Eqs. (i) and (ii) , we get whose nth term is given by
an = 3 + 2 n
2S = {a + a + (n - l) d} + {a + d + a + (n - 2) d }
Sol. Given, nch cerm o f an AP, a ,, = 3 + 2n
Clearly, sum of firsc 24 terms, ( 524 )
+. . . + {a + (n - 2) d + a + d} + {a + (n - l) d + a}
n times 24
:::::> 2S = {2a + (n - l) d } + {2a + (n - l) d } = l [ a + a24 ]
+ . . . + {2a + (n - l) d } = 1 2[5 + 5 1 ]
[·: a, = 3 + 2 = 5 and a:?A = 3 + 2 x 24
n times
= 3 + 48 = s 1]
= 1 2 x 56 = 672
2S = n{2a + (n - l) d}

S = � {2a + (n - l) d }
2 Different Types of P ro b le m s B ased o n
where, a = first term, d = common difference the S u m of n Terms of a n AP

I � I
and n = number of terms
S um of fi rst n terms is also denoted by Sil . !Type-I]
-- ---
Problems Based on Fi n d i ng the S u m of
S., = [2a + (n - l) d ] First m Terms, When Sum of Fi rst p Terms
and q Terms are Given
Also, it can be written as In this type of problems, we form two equations in a and d
n with the help of given information and solve them to get a
Sil = [ a + a + (n - 1) d ]
-

2 and d. Then, find the sum of required n umber of terms.


Arith metic Progression 63
Example 4 . If the s u m of first 10 terms of an AP is 140 => 600 3 = 1 22n - 2n 2 => 1 800 + 2n 2 - 1 22n = 0
x

and the sum of first 1 6 terms is 320, then find the sum => 2 [n2 -61n + 900 ] = 0 => n 2 - 61n + 900 = 0
of first m term.'l . => n2 -36n - 25n + 900 = 0 => n (n - 36) - 25(n - 36) = 0
Sol. I .e r r h c fi rsr r n m o f r h i s A P b e 11 and common difference be d. => (n - 36)(n - 25) = 0 => n = 36 or 25

=
Since, a i s positive a n d d i s negative, s o both values o f n are
T h e n , s u m o f 11 terms, (S )
" = !!_2 [2a + (n - I id ] possible.
C i vc n , su m of fi rst I 0 terms, ( 510 ) = 140
Hence, sum of 25 cerms of given AP
1o 140 = Sum of 36 cerms of given AP 300.
I 2 t1 ( I 0 - I )ti/] = I 40
(iY_Qe IIIj
::::> + => 2a + 9d --
1
5

-
Problem Based on Finding the nth Term ,
=> 211 + 9d = 28 . . . (i) When the Su m of First n Terms is G iven
Also, g i ve n sum o f fi rst 1 6 terms , ( 51 6 ) =320
320 S,,
If is the sum of fi rst n terms of an AP, then i ts nth term a ,,
- ! 2a ( I 6 I )d] = 320 => 2a + 15d = -
16
=> + is given by
I l
2 8
211 1 5 d = 40
+ ... (ii) a11 = S,, - S11 _ 1

+ 9(2) = 28
fro m Eq. (ii) , we gee
Example 6 . If the sum of first n terms of an AP is
On s u b r rac r i n g Eq . (i)
6d = 12 => d = 2
given by Sn = 6n + 7 n2 , then fin d the nth term of the
On p u r r i n g rl = 2 in Eq . (i), we gee
AP. Also, find 1 0 th term of the AP.

=
= 6n + 7n 2
2tt
= 28 - 1 8 =>
10 = 5
a=- Sol. We have, S,,
=> S,, _ 1 = 6(n - 1) + 7 (n - 1) 2 = 6n - 6 + 7(n 2 + 1 - 2n)
2tt
2
11 = 5 a n d d =2
Th us.
6n - 6 + 7n 2 + 7 - 1 4n
fi rst m terms, (S"' ) = � [2a + (m - l)d]
= 7n 2 - Bn + 1

(5+11 -1)
H e nce, s u m of
2
= - [ 2( 5 )
Ill
+ ( m - 1)2] = m[5 + (m - 1)] Now, a,, = S,, - S,, _ 1 = 6n + 7n 2 - 7n 2 + Sn - 1
2 = 14n - 1
=m = m (m + 4)=m 2 + 4m :. 1 0th term of the AP a1 0 = 1 4 1 0 - 1x

'. Type II ! = 140 - 1 = 1 39


(frQ? IV 1
Problems Based on Finding the Number of
, Terms, When Sum of First n Terms and AP Word Problems
are G iven
To solve word problems, fi rst we form the l ist of numbers
In this type of p roblems, we first find and d and then a with the help of given i n formation and check that this l ist is
assume that n u m ber of terms is n. After that, we use the an AP or not. If i t is an AP, then use the fo rm ula o f sum of
formula of s u m of fi rst n terms to calculate n. If a is positive first n terms and calculate the m issing value.
and d is negative, then sometime we get two values of n
because in this case, sum of its some terms becomes zero. Exam ple 7. A man repays a loan of � 3 2 5 0 by � 2 0
in the first mon th and then increases the paym en t by
1 2 � 15 every month. How long will it take him to clear
Example 5. How many terms of the AP 20, 19 - , 18 -,
3 3 the loan?
. . . must be tal\en, so that their sum is 300? Sol. Given, coral amount of loan = � 3250
Sol. Given AP i s 20, 19 .!.. , 1 s � .... . Amount paid in first month = � 20

-
3 3 and amount increase every month = � 1 5
Here, a = 20 Clearly, che amounts of repayment form an AP wich first
and d = 1 9 .!._ 20 =
58 - 20 = 58 -60 = - -2 cerm, (a) = 20 and common difference, (d) = 1 5

=
3 3 3 3 Let the loan be cleared i n n months.
Let terms of given AP be required co gee sum 300. S,, = 3250

{ �2 )] -[2 (20)+ =3250


n Then,

[
1l
S = !!_ [ 2a + (n - l)d] => -[2a + (n - l)d] 32 50

+
We know thac,
2 II
2
=> 300 = � 2( 20) + ( 11 - 1

=
ll

600 = 11 [40 -jn + j]


=> (11 - 1) 1 5]
[·:a = 20 and d = - 2 / 3] 2

+ Sn
=> n [ 40 1 5 - 1 5 ] = 3250 2
600 = � [120n - 2n 2 + 2n]
12 x

=> =>
=> 2511 + 1 511 2 6')00
=> 3ri 2 - 1 300 = 0 ldividing both sides by 5 ]
Allinone MATH EMATICS Class 1 0th Te rm r

=> 3n2 + 65n ..;.. 60n -1300 =0 E xa m ple 9. Kanika was given h er pocket money on
=> n(3n + 65) - 20 (3n + 65) =0 Jan 1st, 2008. She puts � 1 on day 1, � 2 on day 2, � 3
=> (n -20)(3n-65+ 65) = 0 on day 3 and continued doing so till the end of the
=> n =20or n =-3 month, from this m oney into h er piggy bank. She also
'
spent 204 of h er pocket m oney and found tha t at the
. n should be an pos1nve -65
so neglect n = -. end of the month she still had � 1 00 with h er. How
. . integer,
3
S mce, .
m uch was h er pocket m oney for the m on th ?
n = 20 NCERT Exemplar
Hence, the loan is cleared in 20 months . Let her pocket money be ,x.
l, 2, 3
Sol.
Now, she takes �1 on day n on day �3 on day and so OD
Example 8. The interior angles of a polygon are in till che end of che monch, from this money.
AP.
5°.
The smallest angle is 12 0° and the common

1. 31,
Clearly, che amounts that she cakes every day of chc month,
form an AP, in which number of terms is first term (a) =I
difference is
the polygon.
Find the number of sides of

31 31
and common difference ( d) = 2 -1 =
n Now, sum of first terms, (S31 )

[·: sum of n terms, (S.) = % [2a + -1) J]


Sol Let be the number of sides of the polygon.
n = 2 [2 x 1 + (31 -1) x I]
= (2n -4) x 90°
We know that, sum of interior angles of a polygon of sides
. .. (i)
Given, interior angles of polygon are in AP. (11
(a) = 120°

So, smallest angle = first term,
and common difference, (d) =
= { % 2(120) + (n -1} S]} = 312 (2 + 30)
[·: s. = % { 2a + (n -l)d]J
0
Then. sum ofinterior angles, ( S.) [
31 x 32
2
= { %<240 +Sn -s>}°
=31 x 16= 496
So, Kanika takes '°496 rill the end of the month from her
Also, she spent '204 of her pocket money and found that at the
pocket money.
end of the month, she still has '100 wich her.
={ %<Sn + 23Sf ... (ii)
(x - 496) - 204 = 100
According to che above condition, we have
From Eqs. (i) and (ii), we get => x - 700 =='BOO
100
=>
{ %(5n + 23Sf = (2n -4) x 90° x

Hence, '800 was her pocket money for the month.


=> 5n2 + 235n = (2n -4) x 90° x 2 Arithmetic Mean (AM)
=> 5n2 + 235n = 360n -720 If a, b and c are in AP , then b is known as arithmetic mean
5n2 + 235n -360n + 720 = 0
Sn2 -l25n + 720 =O of a and c.
· ·
a, b and c are in AP.
n2 -25n + l = 0 [dividing both sides by 5]
44 b -a =c -b
n2 -9n -l6n + 144 =O 2b = a + c

n(n- 9) -l6(n-9) =0
I
a +c
b=
(n -9)(n -16)n =0=9 orl6 2
=>
Ifn = l 6, then the greatest angle, given by AM of a and c is T
a16 =19
=120°+
5° (16 -1)5° E x ample 10. Find the arithmetic m ean of 2 and 1 2.
which is not possible, as no interior angle of a polygon can be Sol. 2 12 be+b.12 14
Let the arithmetic mean of and
more chan I 80°.
n =9 Then, b= 2 = 2 2

Hence, number of sides of rhe polygon 9. is => b=7


Arithmetic Progression

TRY YOURSELF
Q . 1 Find the sum of the sequence 99, 95, 91 , 87 , . . . upto Q. 3 Find the sum of first 35 terms of an AP, if second term is
20 terms. [Ans. 1 220) 2 and the 7th term is 22 . [Ans. 23 I OJ

Q . 2 If the sum of first n terms of an AP is given by Q.4 How many terms of the AP : 3, 5, 7, 9, . . . must be added
S n = n( 4n + 1), then find the nth term of the AP. Also, find to get the sum 1 20? [Ans. I OJ
the AP. [Ans. 811 - 3: 5 . 1 3 , 1 2 .. . ] NCERT Exemplar

CERT FO L D E R 2 . 3
1 Find the sum of the following APs : = s o (1.2 + 1 08.9 )
(i) 2, 7 , 1 2 , . . . , to 1 0 terms = 50 x 1 1 0.l
(ii) - 37, - 33, - 29, . . ., to 12 terms = 5 50 5
(iii) 0.6, 1 . 7 , 2 . 8, . . . , to 1 00 terms (iv) Given AP is _!__ , _!__ , _!__ , ... co 1 1 terms.
( 1v )
. 1 1 1 15 12 10
- , - , - , . . . , to 1 1 terms
15 12 10 1 1 1 5 -4 1
Here, a = - , d = - - - = -- = - and n = 1 1
Sol. (i) Given AP is 2, 7, 1 2, . . . , co 10 terms. 15 12 1 5 60 60

Here, a = 2, d = 7 - 2 = 5 and n = 1 0
·: Sum of first n terms of an AP,
n
Sum of first n terms o fa n AP, S,, = - [ 2a + (n - l ) d]

�]
ll
2
,, 0 n puccang a = - , d = - an d
S = - [ 2a + (n - 1 ) d] . 1 1
2 n = 1 1, we gee
2,
2 (�
15

J (� .!. )
On puccing a = d = S and n = 1 0, we get 60
IO S1 1 = .!...!_
[2 _!__ + ( I 1 - 1 )
= - [ 2 x 2 + (I 0 - 1 ) 5 ]
s,o x
60
2 2 15
= 5 ( 4 + 9 x 5) = 5 ( 4 + 4 5) = 5 x 49 = 24 5
= !.!.+ 1 0 x J_ = �
(ii) Given AP is - 37, - 33, 29 . . . co 1 2 terms.
+
- , 15 60 2 15 6
= - x -- = - X - = -
Here, a = - 37 , 11 4 +5 11 9 33
d = - 33 - (- 37 ) = - 33 + 37 = 4 and n = 1 2 2 30 2 30 20
·: Sum of first n terms of an AP,
n
2 Find the sums given below
S = - [ 2a + (n - 1) d]
" 2 (i) 7 + 10 .!. + 14 + . . . + 84
2
On purring a = -37, d = 4 and n = 1 2, we get
(ii) 34 + 32 + 30 + . . . + 1 0
(iii) - 5 + (- 8) + (- 1 1) + . . . + (- 2 3 0)
12
s, 2 = - 12 x (- 37) + (1 2 - 1) 4 1
2
= 6 (- 74 + 1 1 x 4) Here, an AP with first and last terms is given, so assume
= 6 (- 74 + 44) that last term is an , if there are only n terms and find the
value of n. Then, find sum of n terms by using the formula,
S n = 2 (a + /).
= 6 x (- 30) n
= - 1 80
(iii) Given AP is 0.6, 1 .7, 2.8, . . . co 1 00 terms. Sol. (i) Clearly, the given numbers 7, 1 0 ! , 1 4, . . . 84
Here, a = 0.6, d = 1.7 - 0.6 = 1 . 1 and n = 1 00 2
Sum of first n terms of an AP,
n
·: 1 0 ! - 7 = 1 4 - 1 0 ! = ... = �
2 2 2
:.The given numbers forms an AP.
= - [ 2a + (n - 1 ) d]
S
" 2
On puccing a = 0.6, d = 1 . 1 and n = 10 0 , we gee Here, first term, a = 7
' -�:-·_

Common difference, d = 1 0 .!.. - 7 = 3 � = ?_


1 00
S OO = - [2 x 0.6 + (1 00 - 1 ) 1 . 1 ]
I 2 2 2 2
= 50 (1.2 + 99 x 1 . 1 ) and last term, I = a,, = 84
Allf.nOne MATH EMATICS Class 1 0th Term I

[ -.%]
an = a + (n - 1) d 3 In an AP,
(i)
84 = 7 + (n - 1) -
7
-: a = 7 and d given a = 5, d = 3, a n = 5 0, f i n d n a n d s n .
(ii)
.. 2 given a = 7, a 13 = 35, find d a n d S i :i . CCE 20 U
7 (iii) given a12 = 37, d = 3, find a a n d S i i .
=>
given cz.i = 1 5, S10 = 1 25, find d a n d a 10 •
- (n - 1 ) = 84 - 7
2 (iv)
=>
7
- (n - 1 ) = 77 (v) given d = 5, S9 = 75, find a a n d a 9 •
2 (vi) given a = 2, d = 8, s n = 9 0, find n and a n .
(vii)
2
=> n - 1 = 77 x - given a = 8, a n = 62, S n = 2 1 0, fi n d n a nd d.
(viii)
7
=>
given a n = 4, d = 2, S n = - 1 4, f i n d n a n d a.
n - 1 = 22
=> n = 23 (ix) given a = 3, n = 8, S = 1 92, fin d d.
(x)
·: Sum of n terms of an AP, given 1 = 2 8, S = 1 4 4 and there a re total 9 terms.
Find a.
Sn = !!.. (a + / ) (z) Here, a = S, d =3 and a,, = SO
2 Sol.
23 an = a + (n - 1 ) d
= - (7 + 84) = - x 9 1
23
:. Sum of 23 terms, (S23)
: . 5 + (n - 1 ) 3 = 50
2 2
3 (n - 1) = SO - 5
= 1 046 .!_
2093 =>
= 45
2 2 => (n - 1 ) =
AP 34 + 32 + 30 + ... + I O.
=15
(it) Given is 3
Here, first term, a = 34 => n =l5 + 1 = 16
On putting n = 1 6, a = 5 and I = a,, = 5 0 in
Common difference, d = 32 - 34 = - 2 n
and last term, I = an = 1 0 S,, = - (a + /) , we gcr
2
an = a + (n - 1). d 16
1 0 = 34 + (n -. 1) (- 2)
S16 = - <s + 50) = 8 x s s = 440
2
[·: a = 34 and d:::: -2] Hence,n = 1 6 and Sn = 440.
=> ( - 2) (n - 1) = 1 0 - 34
(it) a = 7 and au = 35
Here,
=> (- 2) (n - 1) = - 24
a1 3 = 3S
=> n - 1 = 12
a + 1 2d = 3S [·: a,, = a + (11 - 1 ) d]
=> n = 12 + 1 = 13
=> 7 + 1 2d = 35 [·: a = 7]
·: Sum of n terms of an. AP, Sn = !!. (a + /) l 2d = 35 - 7 = 2 8
28 ?...
2
s. 3 =
13 13 => d= =
(34 + 1 0) = x 44 12 3
2 2
= 1 3 x 22 = 286
On putting n = 1 3, a = 7 and / = a13 = 35 i n
AP - S + (- 8) + ( - 1 1) + ... + ( - 230 ).
Ciit) n
Given is S,, = (a + /) , we ge e
-

· a =-S
Here, first term, 2
s. 3 = < 7 + 35 > =
Common difference, d = - 8 - (- 5 ) 13 13
x 42
=-B + S =-3 2 2
and last term, I = an = - 230 = 1 3 x 2 1 = 273
an = a + (n - 1 ) d Hence, d = -7 and 51 3 = 273
- 230 = - 5 + (n - 1) ( - 3) 3
[·: a = - S and d = -3] (iit) Here, a12 = 37 and d = 3
=> (- 3) (n - 1 ) = - 230 + 5 Then, a1 2 = 37
=> (- 3 ) (n - 1) = - 225 => a + l l d = 37 [·: a,, = a + (n - 1 ) d]
-22S => a + 1 1 (3) = 37 [·; d ::: 3 )
=> n _1 =
-3 => a = 37 -33 = 4
=> n - 1 = 7S
=> n = 7S + 1 = 76 On puccing n = 1 2, a = 4 and / = a1 2 = 37 in
·: Sum of n terms of an AP, S n
= !!. (a + /) Sn = - (a + /), we gee
n
2
2
s76 = - < - 5 - 230 > = 3s x < - 235 >
76 12
s. 2 = - (4 + 37 ) = 6 x 4 1 = 24 6
2 2
= - 8930 Hence, a = 4 and 51 2 = 246.
Arithmetic Progression
I 61
(iv) Here, a_\ =fl13S=1and = 125
s
S10
-(2n 4 + Bn -8) = 90
:. (I + 2d = 15 [
. . . (i)
-1) dJ
·: an = a + (11
n2 -4) =90
-(8n
= 125
4n2 n-2n(4n -90-2) =O=90
Also, S10
-102 [2a + (10 -1) d =125 J
4n2-20n+18n-90 =0
s.
[·: =% {2a + (n -l)df] 4n (n (n-S)-5)+ 18( 4n(n+-S)18) == 00
=>
=>
5 (2n2a+9d)=125
+ 9d = 25 [dividing by 5] . . . (ii)
n-5 =O or4n+l8 =0-18
On multiplying Eq. (i) by 2 and then subtracting => n=S orn=-4
2Eq.(a(ii)+ 2d)from-(2a
=>
it, we get
4d+-5d
-9d9d) =5=30
= 2 x-2515 -25 n=5 or --95
=> Since, n cannot be negative.
=> d=-�=-1 n =5= + (5 -1) 8
Now, 1110 =15 = 9d7 (-1)5 + 2d) + 7[from
a+
+
= (a d Eq. (i)] Now, as 2
(n
[·: an = a + - I)d, 2 and d 8]
a= =

Hence, d ==15- I and


-7=8a10 8. Hence, n = 5 and = 34.
=2 + 32 =34

2n + an) = 210 [·: n 2 + ]


= an
(v) Here, d = 5 and = 75 S., (viz) Here, 8, =62 and
a= an 210210
Sn =
s., =75 Sn =
a)
2 2a + (9 -1) 5] = 75
2. [
!!.. ( a S = !!.. (a

[·: = % {2a + (n -1) ]


s. di => -(8
2 +62) =210 ==8 and =62 ] an
[·: a

=>
2.
2 (2a9a ++ 18040) ==7575 2 x 70 = 210
!!..

= 75 -180 n =210x �=3 x 2 =6


9a9a =-105 70
=> =>
=>

=>
-1059 = -
a =-- -35 Thus,
=>
=62 a + 5d =62
an =62
=> a6
3
Now, a., = a +Bd => 8 + S5dd ==6262 -8 =54 (n·.·-1)= d]
[·: an = a +
[ a 8]

= -353 + 8 x 5 =>

d= 545
-35 3+120 = -3 85 =a + ( n
[·: an-1) d] =>

545 and n =6.


--- Hence, d = -
Hence, a =--353 and a., =-.853 (viii) Here, an =4, d =42 and = -14
= S,,

(n -1) 2n==44 -2 (n -1)


a
(n -1). . . d]
� =-14(--· % ]
:. a + [·: an = a +
(vz) Here, a = 2, d =8 and =90 Sn => a (i)
Also, =-14
n- [2 x 2 + -1) 8] ==9090
S

[·: =� (n -1) ]
Sn
(a + a.) s. = (a + a. )
2 (11

n + 4) = - 28 4]
s.
[·: an =
di
=> (a
{2a +
=> n [4- 2 (n -1) + 4] =-28 [ from Eq. (i)]
68 \ Allinone MATH EMATICS Class 1 0th Term II

n ( 4 -n 2n(-2n+ 2++10)4) == -- 2828 !!.

n (-n + S)[dividing
=-14 both sides by 2]
=>
2 (18n + 8n -8) =636
=> - (Sn + 10) = 636
2
-n2 + 5n =-14 => n (411 + 5) =636
n2 -Sn -14 =O => 4n2 + Sn -636 =0
(n - 7) (n + 2) == 00 [byor nfactorisation
2 = 0 method] => 4n2 + S3n -48n -636 = 0 [by factorisation]
n-7n =7 orn +=-2 => n (4n+53) -12 (411+S3) =0
=>

411 +( 4nS3+=S3)0 or(n n-12) =0


=>
Since, n cannot be negative. -12 = 0
. n =7 n = - -S34 or n =12
a = 4n - 7 (7 - 1) = 4 -2 x 6 = 4 -12 = -8
On puc cing
2
n = 7 in Eq. (i), we get
Since, n cannot be negative.
Hence, = n =12
and a == - B.
Hence, the sum of 12 terms is 636.
(ix) Here, a = 3, n = 8 and Sn = 192
Sn =!!.2 [2a + (n -1) d] S The first term of an AP is 5, the last term is
45 and the sum is 400. Find the n u mber of
" 1 92 = �2 [2 x 3 + (8 -1) d] Sol.
terms and the common difference.
n
Let be the number of terms in the given AP and d be che
=> 192 = 4 (6 + 7d) common difference.
[dividing both sides by 4]
=>
=>
48 =6 + 7d
7d = 48 -6 = 42
n (SJ = 400. (a) = S, n (/) 45
Given, first term
terms
last term = and sum of

=> d=-427 =6 S =-2 (a + /) n

Hence, d =6 => 400 = !!.2 + 4 (s s)


(x) Here, I= 28, Sn = 144 and n = 9 => 400 xx 22 =SOn
n- ( a +S/)n =144
=144 => n= 400 =8 x 2 =16
so
2 /=45
[·: s. = % (first term last term)]
Now,
+ => a + (n d = 4S [·: I = a,, = a + (11 -l)d]
- 1)

9 28) =144 => 5 + (16 d = 4S - 1)


=> - (a +
2 => l 5d = 4S - = 40 s [·: a = 5, n = 16)
=>
40
d = 5 =�3
a+ 28 =144 x �9 1

a + 28 =32 . 8
:.The number of terms is 16 and the common difference
IS -.

Hence, aa ==324 -28 =4 3


6 The first and the last terms of a n AP are 1 7 a n d
4 How many terms of the AP: 9, 1 7 , 25, . . . must be
350, respectively. If the common differen ce is 9 ,
take n to give a sum of 636?
then how many terms are there a n d what is
Sol. Here, first term, a =9
and common difference, their sum?
d = 17 -9 = 8. n = 17, I = 350
[2a + (n d] =636 [·: S = {2a + (n -l)d}]
Let terms are required to give the sum Sol. Given, first term, a last term,
636. d=9
Then, Sn =636 and common difference,
Lee there are terms in the given AP. Then,
n

=> !: - 1) !!. I= an =3SO


2 2 n

=> a + (n -1) d = 350 [ ·: an = a + (n d] -1)


=> !!.2 . [2 x 9 + (n -1)8 ] =636 [·: a =9 and d=8] => l? + (n -1)9 =350 [·: a =l7and d =9 ]
9 (n -1) =350 -17 =333
=>
Arithmetic Progression I 69
II -1 =
333
= 37
9 If the sum of first 7 terms of an AP is 49 and that
9 of 17 terms is 2 89, then find the sum of first
=:) II = 37 + 1 = 38 n terms.
On pucring '' = 1 7, I = 350 and 11 = 38 in sum of first Sol. Given, sum of 7 terms, S7 = 49

[·: s. = % {2a + (n -1) d)J


n terms, S,, = - (a + /), we gee
11
l -2 [2a -1) d] = 49
7
+ (7
38
s \!! = - (1 7 + 350)
. •
2
= 1 9 (367 ) = 6973
Hence, chcrc arc 38 ccrms in chc AP having their sum 6973. => -2 (2a + 6d) =
7
49

7 Find the sum of first 22 terms of an AP, in which => a + 3d =7 . . . (i)


d = 7 and 22nd term is 1 49 . and sum of terms, s.7 = 289
17
Sol. Given , d = 7 and a !! = 149
a,, = a + (11 - 1 ) d
=:> [2a (17 -l)d] = 289
17
2
+

=>
{/ + ( 22 - 1 ) d = 1 49

a + 2 1 x 7 = 1 49
=> -2 (2a + = 289
17
1 6d)

=> a = 1 49 - 1 47 = 2 => a + Bd =17


On subtracting Eq. (i) from Eq. (ii), we gee
. . . (ii)
On puccing a = 2, 11 = 22 and d = 7 in
n
S = - [ 2tz + (11 - 1 ) d], we gee
=10 5d
2 "
=:) d=2
22 On putting = in Eq. (i), we get
d 2
Sn = - ( 2 x 2 + ( 22 - I ) 7]
••
2 a+ =7 3(2)
= 1 1 (4 + 2 1 x 7 ) a +6 =7
= 1 1 ( 4 + 1 47) => a =1
= l l x 151 Now, sum of first n terms, Sn = [ 2a + (n d] !!.. - 1)
= 1 66 1 2
Hence, chc sum o f first 2 2 terms is 1 66 1 . = 2 [2 1 + (n 2] !!.. x - 1)
8 Find the sum o f first 5 1 terms of an AP whose
second and third terms are 14 and 18,
[·: a =land d = 2]
respectively. = !!.. (2 2n - 2)2
+
Sol. Given, second term, a2 = 1 4 and third term, a3 = 1 8
a,, = a + (n - 1) d
= !!.2 . 2n =n2 x

1 0 S how that al I Cl.z an I form an AP, where an


We have, a + d = 14 ... (i)
I • • • I • • •

and a + 2d = 1 8 . . . (ii)
On subtracting Eq. (i) from Eq. {ii), we gee is defined as below
d=4 (i) an = 3 + 4n (ii) an = 9 - Sn
On putting d = 4 in Eq. (i), we gee Also, find the sum of the first 15 terms in
a + 4 = 14 each case.
=:) a = l4 - 4 = 10
Sol. (t) Given, an = 4n 3+ 0:
·: Sum of firsc n cerms, S = !!.. [ 2a + (n - 1) d] On putting n 2, ... , we get the list of
= 1, 3, 4, L..L
" 2 numbers c
51
:.Sum of first 5 1 terms, 55 1 = - [ 2 x 1 0 + (5 1 - 1) x 4] l, 4
3 + 4 x 3 + x 2, 3 + 4 x 3, 3 + 4 x 4, . . . , (3 + 4n)
2
[·: a l O and i.e. 4n) which is an
7, 1 1, 1 5, 1 9, . . . , (3 + with AP c
= d = 4] common difference 4. U,
= � [ 20 + 50 x 4]
Here, first term common difference (d) = 4
( a ) = 7,
2 and number of terms (n) = 1 5.

= � ( 20 + 200) = � x 220 Sum of n terms, S = -n [2a + (n - 1) d]


2 n 2
2
15
= 5 1 x l l 0 = 56 1 0
. . S,5
2
- 1) 4
= - [ 2 x 7 + (1 5 ]
70 AllinOne MATHEMATICS Class 1 0th Te rm a

= -
n ch te r m 5,, S,, _ 1
= !22 (14 + 14 4)= !22 (14 + 56) x :. :. +
= - (
411 II - - II 611 - 5)
= + - ,/ + 5 =5 - 211
= !22 x 70 = l S x 35 = S2S 411 - 11:. 6 11

1 2 Find the sum of the first 4 0


(ir.} Given, a,, = 9 -S n positive integers
divisible by 6 .
On putting n = 1, 2, 3, 4, . . . , we gee the list of numbers
9i.e.-54, -1,1,9--56, -1 2,1, ...9, -S(9 Sn)3, 9which
x x x
-S is4,an... , AP(9 -wich
xSn) Sol. The first positive integers d i visible b y 6 a rc 6 ,
40 12, 18, ·-

:--
240 . Clearly, these numbers forms an AP w i c h first term.
a = 6, 6 a n d lase c c rm I = 240 . ,
common difference -S. common d i fference, ti =
II
Here, first term, a = 4, common difference, d = -S ·: Sum of 11 terms , S = -2 [a + /]
and number o f terms, n = 15 n

· : Sum o fn terms, :. Sum of first 40 terms,


S = !!..2 [2a + (n -1) d] S.IU = -402 [6 + 24 0 1 = 20 x 24(l = 4 9 2 0
n

SI = -152 [ 2 4 + (15 -1) ( - s ) ]


s x
1 3 Find the sum of the first 1 5
S o l. 15
The first multiples o f 8 a rc
m u lt i p l e s of 8.

15 [·: alS= 4 and d = -5] 8 x l,8 x 2, 8 x 3, 5


..., 8 x I
=-2 [8 + 14 (-5)] = -2 (8 -70) i.e. 8,16, 24, .. . , 120
wh ich arc in A I�
Here, first term
=(a)15.=
8, lase term (/) = 120 and
=-1 5 (-62) = lS (-31)= -46S
2 x x
number of terms (n)
Sum of n terms, 5 = -2 + /) II
( 11
Hence proved. n

15 + 120) = -1 5
11 If the sum of the first n terms of an AP is (8 x 1 28 . . Sum of 1 S terms, S15 = -
2
4n - n2 ' then what is the first term (i.e. sl )?
What is the sum of first two terms? What is the
=152 x 64 =960
second term? Similarly, find the 3rd, 1 0th and 1 4 Find the sum of the odd numbers b e tween
the nth terms . 0 and 50.
Sol. Given, che sum of first n terms, Sol. 0 50 1, 3, 5 , 49 which
The odd numbers between and are • . . .

5,, = 4n - n 2 . . . (i) form an AP.


i n Eq. (i) , we gee
n =1 (a) = 1
Here, first term and lase term (/) = 49
and common difference (d) 3
On putting
= 4 1 -12 = 4 -1 = 3
s. x
= -1 = 2.
Lee there are n numbers in che AP.
Thus, first term = 3 Then, nch term (n
(a,,) =1 a + -1) = I
d
On puccing n = 2 in Eq. (i), we get => + (n -1) (2) = 49
= 4 x 2 - 22 = - 4 = 4
s2 8 => 2 (n -1) = 48
Thus, sum of first cwo terms = 4 => n -1 = 24
·: The n th term of an AP, a,, = 5,, - 5,, _ 1 => n = 25
: . Second term = - S1 = 4 -3 = 1
!!.. + /)
52
Now, sum of n terms, S , = (a
On putting n = 3 in Eq. (i), we get 2
25 25
S3 =4 x 3 -32 =12 -9 =3 :. Sum of25 terms, S 5 = - (1 + 49) = - x 50
:. Third term = 53 -S = 3 -4 = - 1 2 = 252 x 25 =625 2
2
On putting n = 9 in Eq. we get (i),
S9 4 9 -92 = 36 - = - 4S
x 1 5 A contract on construction j ob s p ecifies a
penalty for delay of completion b eyond a
= 81
Again, putting n = 10 n Eq. we get i (i), certain date as follows
S1 0 4 x -102 = 40 -100 = -60
= IO
:. 1 term = S1 0 - 59
Och
� 200 for the first day, � 250 for the s e c o n d day,
� 300 for the third day etc . The p en alty for each
=-60 -(-45)=-60 + 4S = - 15 [given] succeeding day being � 50 m ore than for the
n
Now, on replacing by n - 1 in Eq. (i), we get preceding day. How much m o n ey the
5,, _ 1 = 4 (n - I) - (n -1)2 contractor has to pay as penalty, if he has
= 4n - 4 -n2 + 2n -1 = -n2 + 6n - 5 delayed the work by 30 days?
II 71
Arithmetic Progression

Sol. Given , rhc p cna lry fo r each succeeding day is f 5 0 more a section of class I I will plant 2 trees and so o n
chan the preced i n g day, t h e re fore the penalties for the first till class XII . There are three sections o f each
day, the secon d day, che third day etc. , will form an AP. class. How many trees will be planted by the
He rc , 111 = ( 200, 11 2 = ( 250, 11_, = ' 300, students? CCE 201 4
=
:. 11 = 200, d 2 50 - 200 = 50 and 11 = 30. Sol. Accordi ng co the question, there are three sections o f each
Clearly, c hc mo ney re q u i re d by che contractor to pay as class, so the number of trees thac are planted by students o f
penalty, if he delayed t h e work by 30 days, will be SJtJ. I,
class class II,
class .. . and class XI I , are respectively,
III,
We know char, x x x
1 3, 2 3, 3 3, ... and 1 2 3 . x
II
s = I 211 + (11 - 1 ) d ] Thus, we get the following list of numbers. 3, 6, 9, . . . , 36
2
-
II

Clearly, i c forms an AP.


30
s 10 =
2
x
1 2 200 + (30 - 1 ) 50 ] Here, the firsr term, a = 3
Common difference, d = 6 - 3 = 3
= 1 5 ( 4 00 + 29 x SO) and the last term, I = 36
= 1 5 ( 400 + 1 4 5 0) = 1 5 x 1 850 = 27750 Let the lase term of this AP
be i cs nth term.
Hence, che co nt racc o r has co pay ' 27750, if he delayed Then, an = a + (n - 1 ) d = I
the work.
=> 3 + (n - 1 ) (3 ) = 36
1 6 A sum of f 700 is to be used to give seven cash => (n - 1 ) 3 = 33
prizes to students of a school for their overall

[· /)]
::::) n - 1 = 1 1 ::::) n = 1 2
academic performance. If each prize is f 20 less Hence, rhe number of the rrees planted by the students
than its preceding prize, then find the value of = Sum of 1 2 terms of above AP

. . s = !!_ ( ll +
each of the prizes . 12
= (3 + 36)
Sol. Herc, ( 7 0 0 i s c o b e used t o give prizes co 7 scudencs. 2 2 n

So, coral a m o u nt received by 7 scudcncs = 700 =6 x 39 = 234


Let prize given co first scudencs be 11. Then, prize given co
1 8 A spiral is made up of successive semi-circles
2nd scudcnr is (11 - 20 ), prize given to 3rd student is
[(a - 20) - 20 J a nd so on. with centres alternately at A and B, starting
Thus, we get an AP whose first term is 11 and common with centre at A, of radii 0.5 cm, 1 . 0 cm, 1 . 5 cm,
difference is - 20. 2 . 0 cm, ... as shown in below figure . What is the

[·: �
total length of such a spiral made up of thirteen

]
Since, 57 = 700
consecutive semi-circles? [take, 7t = 2 2 /7)
z [ 211 + (7 - 1 ) ( -20)] = 700
2
s. = { 2a + (n - l)d)}

2
=> 211 - 20 x 6 = 700 x -
7
211 - 1 20 = 200
211 = 200 + 1 20
211 = 320 [Hint Lengths of successive semi-circles are /1 , /2 , /3 , /4 , •
with centres at A, B, , respectively. ]
• •

320 . . .

a= = 1 60
2 Sol. Length of spiral made up of thi rteen consecutive
Hence, value of 1 sc p rize = ' 1 60 semi-circles
Value of 2nd prize = ' 1 60 - ' 20 1 40 =' =( 7t x o.s 7t x
+ 1.0 + 7t x
1.5 + x 2.0 + . . . +7t 6.5 ) 7t x
Value of 3rd prize = ' 1 40 - ' 20 = f 1 20 [·: circumference of semi-circle = rrr ,
Value of 4 c h prize = ' 1 20 - ' 20 = ' 1 00 where r is radius of circle]
Value of 5 th prize = ( 1 00 - f 20 = ( 80 = 0.5 7t [l + 2 + 3 + 4 + ... + 1 3 ]
Value of 6th prize = ( 80 - ( 20 = ' 60 13
and value of 7ch prize = f 60 - f 20 = f 40 = 7t x 0.5 x2
[ 2 x I + (1 3 - 1 ) x 1 ]
-

1 7 In a school, students thought of planting trees [·: the numbers 1 , 2, 3 . . . 1 3, forms an AP with a = 1 ,
in and around the school to reduce air d = 2 - 1 = 1. Also, here n = 1 3 and use
1l
pollution. It was decided that the number of S,, = - { 2a + (n - l )d \ ]
trees , that each section of each class will plant, 2
13
will be the same as the class, in which they are 22 5
-x- x 1 4 = 1 43 cm
2
=-x
studying, e . g . section of class I will plant 1 tree, 7 10
72 AllinOne MATH EMATI CS C lass 1 0t h Te rm 11

1 9 2 0 0 logs are stacked in the following manner Hence, th nu mber of r w i 1 6 and numb r o f lo
in the top row i 5.
vi ng r r ivi
20 logs in the bottom row, 19 in the next row, 1 8
(ii) The pattern of log pa � •,

in the row next to it and so on (see below figure) .


reasoning and balanci ng.
{i) In how many rows , are the 200 logs placed and
how many logs are in the top row? 20 I n a potato race, a b u cket i p l a d at th
{ii) Which value is depicted in the pattern of log? starting point, which is 5 m fr m th fi r t
p otato and the other potatoes a re p l a c e d 3 m
apart i n a s traig ht l i n e . Th er a r t n p tato
in lines (see below fig u re ) .

S o l. (t) Number of logs stacked in each row form a sequence Sm 3m 3m


20, 1 9, 1 8, 17, ... . , which is an AP with fi rst term, A competitor s ta rts from t h e b u c ke t , p i c ks u p
a = 20 and common difference, d = 1 9 - 20 = - 1. the nearest potato , r u n s b a c k w i t h i t , d ro p s i t i n

[·: % J
Suppose number of rows is n, then Sn = 200 t h e bucket, runs back to p i c k u p t h n ext
sh

n- [2 x 20 + (n - 1 ) (- 1 )] = 200
p otato, ru ns to the b u cket to d ro p i t i n a nd
c on tin ues i n the same way u n til a l l th
2
s. = { 2a + (n - l ) d} potatoes are i n the buck t . Wh a t is t h e total
distance the competitor has to r u n ?

[Hint To p ick u p t h e fi rst potato and t h e s e c o n d potato,
400 = n ( 40 - n + 1 )
� 400 = 40n -n2 + n the total di stance (in metres) run by a c o m pet i t o r is
� n2 - 4 ln + 400 = 0 2 5 + 2 x (5 + 3).)
x

� n2 - 25n - l 6n + 400 = 0 [by factorisation] Sol. The distance run by the com petiror t picks u p the first

potato, second potato, th ird potato, founh potato , . . . . ar
respectively 2 x 5, 2 x (5 + 3 ), 2 x (5 + 3 + 3 ),
n (n - 25) - 1 6 (n - 25) = 0


(n - 25) (n - 1 6) = 0 2 x (5 + 3 + 3 + 3 ) i . e. 1 0, 1 6, 22, 28, ... .
Clearly, i t is an AP with first term, a == I 0 and com mon
n - l 6 = 0 or n - 25 = 0
� n = 1 6 or n = 25 difference, d = 1 6 - 1 0 = 6.
Hence, the number of rows is either 25 or 1 6. n
When n = l 6, ·: Sum of fi rst n terms, S = - [ 2a + (n - 1 ) d ]
a,, = a + (n - 1) d 20 + (1 6 - 1) ( - 1 )
II 2
:. Sum of first 1 0 terms, 510 = - ( 2 x 1 0 + (1 0 - 1 ) x 6]
==
10
= 20 - 1 5 5 ==
2
When n = 25,
n =
1 0, given]
an = a + (n - 1) d = 20 + (25 - 1) (- 1 )
[ ·:
= 5 ( 20 + 54) = 5 x 74 = 370
= 20 - 24 = - 4 Hence, the total d istance the com peti tor has to ru n , is
[·: n umber of logs cannot be negative] 370 m .

NCERT FO L D E 2 .4
1 Which term of the AP : 1 2 1 , 1 1 7 , 1 1 3 , . . . is its first For fi rst negative term, consider a,, < 0
ne gat ive term? � 1 25 - 4n < 0
� 1 25 < 4n => 4n > 1 25
[Hint Find n for which an < 0.)
n>-
1 25
S o l. Given AP is 1 2 1 , 1 1 7, 1 1 3 , ... .
Here, first term, a = 1 2 1
4
and com mon difference, d = 1 1 7 - 1 2 1 =-4 � n > 3 1 -1
Now, nth term of this AP,
4
an = a + (n - 1 ) d = 1 2 1 + (n - 1) x ( - 4)
Least i ntegral val ue of n = 3 2
= 1 2 1 - 4n + 4 = 1 25 - 4n
Hence, 32nd term of the given AP is the fi rst n egative t rm.
Arithmetic Progression

= 8 [2(5) + 15 ( -rn
2 The sum o f the third and the seventh terms of =8 (2a + 15d)
=8 (10 - 1;) =8 ( %) = 20
an AP is 6 and their product is 8. Find the sum
of first sixteen terms of the AP.
Lee ehcd, respecei
firse ecrmvcland AP
Sol.
and y. ehc common difference of ehe be a
According co ehc qucscion, Hence, s�m of first sixteen terms, S16 = 20 or 76.
::)Third ccrm + [aSeven ch ecrm = 6
+ (3 -l)d] + [a + (7 -1) d] =6 d] 3 A ladder has rungs 25 cm apart
[·: a =a + -1)
(a + 2d) + (a + 6d) n 6
(n (see below figure) . The rungs decrease
::) = uniformly in length from 45 cm at the bottom
::) 2a + Bd =6 ... (i)
to 25 cm at the top. If the top and the bottom
::) a + 4d =3 rungs are 2 .!. m apart, then what is the length
and ehird ecrm x Scvcnch ecrm = 8 2
=> (a + 2d) (a + 6d) =8 of the wood required for the rungs?
=> - =8
{(a + 4d) (32d} {(a + 4d) + 2d} using Eq. (i)]

M----r ----
=> - 2d) (3 + 2d) = 8 [
=> 9 -4d2 =8 [·: (a -b)(a +b) =a2 - b2] : 3 �
=> 4d2 =9 -8 I
I
I
I
I
I
I I
I I
2 1. m

"-------'JI
d = ± -21
When d .!.,2 ehcn from Eq. (i), we gee
=
45 cm
a + 4 m =3 [Hint Number of rungs
a + a2 =3=3 -2 =
Distance between top and bottom rungs
+
11

a =1 Distance between two rungs


Accordinofg torungsthe question,
:. Sum of first sixceen terms of the AP, Sol.
Number
16
s,6 =- [2a + (16 -1) d] Distance between top�and����..--.:
������--=- bottom rungs� +1
2
[·: s. = � {2a + -1) di] 12-m Distance between two rungs
=8 [2a +15d]=8 [2(1) +15(�)]
(n

=- 252- cm + 1
[-: a =I, d = �]
� x l OO cm

=8 ( 2 + 1; ) =8 (1:) =76
= 2· 25cm + 1 [·: I m = 100 ] cm

= 25025 cmcm + 1=10+1=1 1


When d = - .!..2 , ehen from Eq. (i), we get Hence,lengththereofarethe11wood
The rungs.required for the rungs
=Sum of 1 1 rungs
[·: = % (a + I)J
a + 4 H ) =3 = !..!.2 (25 + 45)
a - 2 =3 a=5
� s.
AP,

[·: s. i {2a + di]


:. Sum of first sixteen terms16of this
s,6 =
-

2 [ 2a + (16 -1) d] - = 121 70 = x 35 = 38 5


x 11 cm

= (n - 1 ) Hence,cm. the length of the wood required for the rungs is


385
74 ' Allinone MATH EMATICS Class 1 0th Term II

4 The houses of a row are numbered 5 A small terrace at a football ground c o mprises
consecutively from 1 to 49. Show that there is a of 1 5 steps each of which is 50 m long and built
value of x such that the sum of the numbers of of solid concrete. Each step has a ris e of .!. m
the houses preceding the house numbered x is 4
equal to the sum of the numbers of the houses and a tread of .!. m (see b elow figu re ) . Calculate
2
following it. Find the value of x.
the total volume of concrete required to build
[Hint Sz_ 1 =

2, 3, . . , 49 .
S49 - Sz] the terrace.
Sol. The numbers on the houses of a row are l, .
Clearly, this list of n�bers forms an AP.
and d

...�
Here, a =1 = 2 -1 = 1 �m
.!
= 2 [2a + -1) d]

[ Hint
S n
(n

sz-1 = -x -1- [2 x 1 + (x -1 -1) x 1)


2
= --
x -1 (2 + x -2)
2
Volume of concrete required lo build t h e first step

= � � x x 50 m3 J
= (x -1)2 x = --
x2 -x

( 2 x �) x � x 50, (3 x �) x � x 50
2 Sol. Clearly, volume of concrete required to build the I seep.
I I step, III step, .. are respectively, _!_ _!_
Now, S %
.=_
= 2 (2 x 1 + (x -1) x 1] 4x2 5 0 x ,

x + l) = --
=-(x x2 + x • ... •

249 2 50 2 50 , 3 x -, 50 .
and = 2 [2 x 1 + ( 49 -1) x 1] i.e. -, x - . .
S49
8 8 8
= 492 (2 + 48] Now, total volume of concrete,
= 50 + 2 x 508 + 3 50 + ...
v
= -429 x 50 = 49 x 25
x
8 8

=1225 = 508 [l + 2 + 3 + ... ]


According to the question, Note chat the numbers in the bracket forms an AP with first
Sz-1 = term (a) = 1, common difference = 2 -1 1 (d) =
and number of terms = 15 (n )
S49 - S"
2
x -x =1225 - -- x2 + x
[-.- s. = % {2a + (n - ) J]
=> -- 2 2 2 v=-508 152 (2 x 1 + {15 -1) 1]
x -
=--::...=._
x

=>
2 + x + x = 1225
22 22 l d
x -x + x + x =1225
=>

=>
2
x2 =1225 = 508 x !22 x (2 + 14)
=> x = ± 35 = 25 x 15 16 = 750 m 3x
Since, x is a counting number. 8

3•
Hence, the total volume of concrete required to build the
J
:. Taking positive sign, we get terrace is 750 m
x =35
TO P I C EXERCISE
. _ _ ·-- � _..
_._ -:�--�:='. G2i_-�-:: :-_:�.:::T
::r �EL:��L�,.:...!�-:���=�·N���� -!�·� -�!
"- . ,- : . .·.-- -: ;. - _ _ - :;._-rr;-; �.:r.r.J:r�.. .. �....;K:.r..; ::-� ����-=-==:-=-
=- -�� -�L;����=�m:.::.:-�
: c;"'-

Very Short Answer Type Questions 9 Find the number of terms of the AP 64,60,56, . . . , so that their

1 Find the su m of 10 terms of an AP :


sum is 544. Explain the double answer.
50. 46, 42, ... . 1 0 If sum of first 6 terms of an AP is 36 a nd that of the first 16

2 Find the su m of 100 terms of an AP :


terms is 256, then find the sum of first 10 terms.
NCERT Exemplar

1 1 Find the sum of first 17 terms of an AP, where 4th and 9th
2. 4, 6, .... 200.
3 The su m of the fi rst n terms of an AP is given terms are - 15 and - 30, respectively.
by S ,1 = 2n2 + Sn. Find the nth term of the AP.
Long Answer Type Questions
Short Answer Type I Questions
1 2 The sum of first six terms of an arithmetic progression i s 42.
4 Find the sum of first 25 terms of an AP The ratio of its 10th term to its 30th term is 1 : 3. Calculate
whose nth term is 1 - 4n. the first and the 13th term of the AP.
5 228 logs are to be stacked in a store in the 1 3 A man arranges to pay off a debt of ' 3600 by 40 annual
following manner: 30 logs in the bottom, 28 instalments which are in AP. When 30 of the instalments
in the next row. then 26 and so on, in how are paid, he dies leaving one-third of the debt u npaid. Find
many rows can these 228 logs be stacked? the value of the 8th instalment.
How many logs are there in the last row? 1 4 The sum of the first five terms and the sum of the f irst
seven terms of an AP is 167. If the sum of the first ten terms
Short Answer Type 1 1 Questions of this AP is 235, then find the sum of its first twenty terms .
NCERT Exemplar
6 Find the sum of first 21 terms of an AP
whose 2nd term is 8 and 4th term is 4. 1 5 Yasmeen saves ' 32 during the first month, ' 36 in the
second month and ' 40 in the third month. If she continues
7 If the sum of first p terms of an AP is q and to save in this manner, in how many months will she save
the sum of first q terms is p, then find the ' 2000? NCERT Exemplar
sum of first (p + q) terms.
1 6 The ages of the students in a class are in AP, whose common
8 Prove that the sum of later half of 2n terms difference is 4 months. If the youngest student is 8 yr old
of an AP is equal to one-third of the sum of and the sum of the ages of all the students is 168 yr, then
the first 3n terms. find the number of students in the class.

Answers
1 . 320 2. 10100 3. 4n + 3 4. - 1275 5. 1 2 a nd 8
6. - 2 10 7. - (p + q) 9. 16, 17 1 0. 100 1 1 . - 5 10
1 2. Cli = 2, Cli3 = 26 1 3. " 65 1 4. 970 1 5. 25 months
1 6. Number of student in the class = 16
� I Very Short Answer Tvpe Questi ons [ 1 M a r k e a ch

1. Find the next term of the AP : 3, 1, - L - 3, . . . 1 0. What is the sum of all natural numbers from
CCE 201 2 1 to 1 00?

1 1. Find the 1 5 th term of the AP :


2. Find the common difference of the AP : .!:.,
p y - 7, y - 2, y + 3, . . . .
1 - p 1 - 2p
-- , , ... CCE 20 1 3
p p 12. Find the 7th term of the sequence whose nth
term is given by an = (-1) - l · n 3 •
n

3. For what value of k will k + 9, 2k - 1 and CCE 2013

2k + 7 are the consecutive terms of an AP.


CCE 2016
1 3. Find the 25th term of the AP :
5 5
- 5, - -, 0, -, . . .
4. If k - 1, k + 3 and 3k - t are in AP, then 2 2 CCE 2015

find the value of k. CCE 201 1


1 4. Is 68 a term of the AP : 7 , 1 0 , 1 3 , . . ?
.

{
CCE 2013
5. Find the 1 9th term of the following seuence.
t n2 , where n in even 15. Which term of the AP : 2 1 , 4 2 , 6 3 , 8 4 , . . . is
n =
n2 l , where n is odd 2 1 O? NCERT Exemplar
-
CCE 201 5

6. Find the missing term of the AP : 1 6. Find the 4th term from the end of an AP :
3, o , 33, 48, . . . . - 1 L - 8, - 5, . . . , 49. NCERT Exemplar

7. I n an AP, if a = - 7 .2, d = 3.6 and an = 7 .2, 1 7. Find the 7th term from the end of the AP :
then find the value of n. NCERT Exemplar
7 , 10, 1 3 , . . . . , 1 84 . CCE 201 5

8. For an AP, if a18 - a14 = 32, then find the 18 . If the nth term of an AP is (2n + 1), then find
common difference d. the sum of its first three terms.
CCE 2009
NCERT Exemplar ; CCE 2012

9 • If 17th term of an AP exceeds its 1 0th term 1 9� If the common difference of an AP is 3 , then

�I
by 14, then find the common difference. find � o - a15 • CCE 201 1

Short Answer Tvpe I Question s [ 2 M a rks ea ch ]

2 0. 22. If the numbers 2 n - L 3 n + 2 and 6n - l are


1 + 3 + 5 + . n
If .. = 9, then
in AP, then find n and hence find the
2 + 5 + 8 + ... 8
find the value of n. numbers . CCE 20 1 0

21. 2 3. If 2 1 , a, b and - 3 are in AP, then find the


If the 2nd term of an AP is 13 and 5th term is
25, then what is its 7th term?
NCERT Exemplar value of (a + b).
Arithmetic Progression In
2 4 . Find the number of natural numbers 32. The nth term of an AP is an = 2 n + L find its
between 1 0 1 and 999 which are divisible by sum. CCE 201 3
both 2 and 5 . CCE 2014
3 3 . The 1 6th term o f an AP is 1 more than twice
25. The 4th term of an AP is zero. Prove that the
25th term of the AP is three times its 1 1 th its 8th term. If the 1 2th term of an AP is 4 7 ,
term . CCE 2016 then find its nth term.

2 6 . The fourth term of an AP is 1 1 . The sum of 34 . Determine k, so that k 2 + 4k + 8, 2k 2 + 3k + 6


the fifth and seventh terms of the AP is 24. and 3k 2 + 4k + 4 are threeconsecutive
Find its common difference. CCE 2015 terms of an AP. NCERT Exemplar

27. Which term of the AP : 1 20, 1 1 6, 1 1 2 , . . . is 35. Find the sum of first 24 terms of the AP
first negative term? CCE 2012
a1 , ai , a3 • • • , if it is known that
28. Find how many two-digit numbers are
divisible by 6 . CCE 201 1 01 + Os + a10 + 01s + Clio + £li4 = 2 2 5

29. Find the sum of all three-digit natural 36 . Find the sum of a ll multiple of 7 lying
numbers, which are multiples of 1 1 . between 500 and 900.
CCE 2009
3 7. How many terms of the AP : 9 , 1 7 , 2 5 , . . .
3 0. Find the sum of first 3 1 terms of an AP must be taken to get a sum o f 450?
whose nth term is given by 3 + � n.
3 3 8. Find the sum of all two-digit numbers
3 1 . Find the sum of the first 25 terms of an AP, greater than 50 which when divided by 7
leaves remainder 4 .
whose nth term is given by an =7 -3n.

�J
CCE 2009

Short A nswer Tvpe II Questions [3 M a r k s e a c h ]

3 9. Split 2 07 into three parts such that these are 44 . Sum of the first n terms of an AP is 5 n 2 - 3n.
in AP and the product of the two smaller Find the AP and also find its 1 6th term.
parts is 462 3 . NCERT Exemplar CCE 20 10

4 0. The sum of the first three terms of an AP is 4 5. The sum of first, third and seventeenth
33. If the product of the first and the third terms of an AP is 2 1 6. Find the sum of the
term exceeds the second term by 29, then first 13 terms of the AP.
find the AP. NCERT Exemplar
46 . The sum of 5th and 9th terms of an AP is 7 2
4 1 . Which term of the AP: 4, 1 2 , 20, 28, . . will be
.
and sum o f 7th and 1 2th terms i s 9 7 . Find the
120 more than its 2 1 st term? AP. NCERT Exemplar, CBSE 2009

42. Find the sum of all two-digit odd positive 4 7. If four numbers are in AP such that their
numbers. CCE 2009 sum is 50 and the greatest number is
4 times the least, then find the numbers .
43 . If 12th term of an AP is 2 1 3 and the sum of
its four terms is 24, then what is the sum of
48 . If there are (2n + 1) terms in an AP, then
its first 1 0 terms? CCE 2015 prove that the ratio of the sum of odd terms
and the sum of even terms is (n + 1) : n.
MATHEMATICS C la s s 1 0th Term l :
\
78
AllinOne

49 . Find the number of terms in the sequence 52 . Find the common di ffe re n c e o f a n A P whoSi
first term is 5 and th e s u m of i t s f i rs t 4 ter�
20, 19 1 8 � , . . . , of which the sum is 300.
_!_ ,
3 3 is half the sum of the next 4 t e r m s .
Explain the double answer.
5 3 � !HO� The sum of first n, 2 n a n d 3 n t e r ms
5 0. I f the nth terms of the two APs 9 , 7 , 5 , . . . an AP are S1 , Si and S:i , res p e ct i v e l y.

Prove that � = 3(Si. - S1 ).


and 2 4 , 2 1 , 1 8 , . . . are the same, then fin d
the value o f n . Also, that term .
NCERT Exemplar
5 4. l!!_oTSJ The sum of frist n t e r m s o f t h ree APs aJ'i
5 1. The sum of the first term and the fifth term of S1 , Si and � . The first t e r m o f e a c h AP L
an ascending AP is 26 and the product o f the unity and their c o m m o n cl i f f e n' nces a re 1 , Z
second term by the fourth term is 1 60 . Find and 3 , respectively.

�I
the sum of the first seven terms of this AP. Prove that S1 + S:i = 2 S2 . CCE 2016

long Answer Type Questi ons [ 4 M a rks e a ch]

55. Each year, a tree grows 5 cm less than it did 6 1. The sum of the first p, q, r te r m s of a n AP
the preceding year. If it grew by 1 m in the are a, b and c, respectively. S h ow t hat
first year, then in how many years will it a
- (q - r) + - (r - p) + - (p - q)
b
=
c
CCE 201 5 0.
r
have ceased growing?
p q
56. Jaipal Singh repays his total loan of
� 1 1 8000 by paying every month starting 62. The sum of four consecutive n u m b e rs in an
with the first instalment of � 1 000. If he AP is 3 2 and the ratio of t h e p ro d u c t of thr
increases the instalment by � 1 0 0 every first and the last terms to t h e p ro d u ct of the
month, then what amount will be paid by two middle terms is 7 : 1 5 . Fi nd thr
him in the 30th instalment? What amount of numbers . N C ERT Exemplar
loan does he still have to pay after 30th
instalment? NCERT Exemplar; CCE 201 2, 1 1
63. The ratio of the sums of firs t n t e r m s of h\·l
57. The sum of first 5 and 9 terms of an AP is 72 APs is (4n + 2) : (3n + 4 7). Fin d t h e ratio o
and the sum of first 7 and 1 2 terms is 97 . their 9th terms .
Find the AP. CCE 2009
64. I f a1 , CJi , . . . , an _ , an _ 1, a11 a re i n A P, then
58. 2
Solve the equation
- 4 + (-1) + 2 + . . . + x = 437. prove that

(_..!__ _!_J
-- +
NCERT Exemplar 1 1 1 1
� . an - a3 . a n -
+ + . . . + --
59. If the sum of first 7 terms of an AP is 1 1 9 and al . an 1 2 an . 01
that of the first 1 7 terms is 7 1 4 , then find the
= _!_ + ... +
2

sum of its first n terms. +
al + an at an
60. If the mth term of an AP is _!_ and nth term is
n 65. !H OTS] The ratio of the sums o f first m and 11
_!_, terms of an AP is m 2 : n 2 . S h ow t h a t tht'
�1
�1 ;1
;· � m
then show that the sum of mn terms is
ratio of the mth and nth terms i�
<.JOI Ii
- ( m n + 1).
1
� ;·� (2m - 1) : (2 n - 1).

f! 2 CCE 201 2
f
Arithmetic Progression 79

�I Va l ue Based Questions (VBQ s) [ 4 M ar ks each ]

69. How many three-digit numbers a re there


66. Ram asks the labour to dig a well upto a between 97 to 2 0 1 , which a re divisible b y 3 ?
depth of 1 0 m . Labour charges � 1 50 for first Reena calculated it by using AP, while Ritu
metre and � 50 for each subsequent metres .
As labour was uneducated, he claims � 550
calculated it directly. Which valu e is
depicted by Reena?
for the whole work.
(i) What should be the actual amount to be 70. Mamta has two options to buy a house
paid to the labour? (i) She can pay a lumpsum amount o f
(ii) What value of Ram is depicted in the � 2200000 .
question, if he pays � 600 to the labour? (ii) She can pay � 4 00000 cash and balance
67. Nidhi saves � 2 on first day of the month, in 1 8 annual instalments of � 1 00 0 0 0
� 4 on second day, � 6 on third day, and so plus 1 0% interest on the unpaid amount.
on. She prefers the option (i) and d on ates 5 0 %
(i) What will be her savings in the month of of the difference of t h e costs in t h e above
February 20 1 2 ? two options to National Relief Fun d .
(ii) What value is depicted by Nidhi? ( a ) What amount was donated to National
Relief Fund?
68. H ow many two -digit numbers are there (b) By choosing to pay a lumpsum amount
between 6 and 1 02 , which are divisible by and donating 5 0% of the difference to
6? Ram calculated it by using AP, while National Relief Fund, which value is
S hyam calculated it directly. Which value is depicted by Mamta?
depicted by Ram?

So l u ti o n s
3 , l , - l, - 3, . . . are in AP Here, a1 = a = 3
[·: a11 = a + (11 1) d]
1 . Given,
Herc, fi rst term, a = 3 and common difference,
= a + 2d = 33
and a.3 -

:::::> 3 + 2d = 33
d = 1 - 3 = - 2. :::::> 2d = 30
:. Next term of given AP = a5 = a + 4d :::::> d = 15
[·:a,, = a + (11 - l)d] a1 = a + d
= 3 + 4 x ( - 2) = 3 - 8 = - 5 [1 ] = 3 + 15 = 18 [1 ]
I I - p 1 - 2p .
2. , . . . are AP. 7. Do same as Q. 1 (iv) ofNcerc Folder 2.2. [Ans. n = 5 ]
.
Given, -, --, -- m

p p p 8. Given, a 18 - a 14 = 32
. ·. Common difference = Second term - First term :. [a + {18 - l)d] - [a + {14 - l)d] = 32
d=l - p _ }_ [·: a11 = a + (n - l )d]
p p :::::> a + 17 d - a - 13d = 32
:::::>
=1 -p -l=- -p =-1
:::::>
4d = 32
d=8 \�.-

p p

= { 112,
[1 ] ..� . ·

3. Do same as Q. 7 on page 76.


which is che required com mon difference. [1 ]
[Ans. k= 18]
[Ans. k = 4 ]
9. Do same as Q. 10 of Ncerc Folder 2.2 [Ans. d = 2]
i'
4. Do same as Q. 7 o n page 76. - ·- · -

1 0. We know chat, nacual numbers fo rms an AP with fi rst


n
t,,
where in even
5. G iven, terms (a) =
I and common differe nce d=1
n 2 - 1, where 11 is odd 100
19 ch term, i.e. = 19 which is odd, we cake 51 00 = - [2 (1) + (1 00 - 1 ) . l]
For 11 2
2 2
t = 11 - 1 = (19) - 1
"
= 1 00 [ 2 + 100 - 1] = 1 00 (I 00 + 1)
= 361 - 1 = 360 [1 ] 2 2
6. Lee a and d be che first term and common difference, = 1 00 (101) = 5 0 -5 0
respectively of given AP. 2 [1 ]
80 i AllfnOn e MATH EMATICS Class 1 0th Te rm 1 1
t

11. Here, a = - 7and d = - 2) - - 7) = 5


y (y (y !!_ (2n)
Now, a15 = a 14d [·: an = a (n - )d]
+ + l => �2-
4 (25) = 9
7 +
=== -- 63 14(5)
y
y
y +
7 + 7 0 (1 ]
=>
..
n 2 = 900
n = 30 [taking positive square root][1 ]
1 2. Given, nth term(an) = 7 ( - l )n- I · n3 21 . Given, a2 = 13anda5 = 25
. . 7th term (a7 ) = ( - 1) - 1 = (-1)6 • ( 7)3 • 73 . . a (2 -a 1) d == 1313 [·: an = a - 1...) (i)d]
+ + (n
= 1 343 = 343 x => + d
(1 ]
1 3. Given, - 5, - 2-,2 0, 2-,2 ... are in AP.
and a + (5 a- 1 )4dd == 2525
+ ... (i i )
=>
Here, first term, a = - 5 On subtracti=ng25Eq.- (i)13 from = 12 Eq. (ii), we get
and common difference, d = - 2_2 5 = 2_2 3d
=>
+ d=4

[-: a. = a (n - l)d, a = - 5, and d = %]


:.25th term of given AP =- 5 (25 - 1) 2_2 + x
From Eq. (i), we get a = 13 - 4 = 9 (1 /2]
d
(1 /2]
Now, a7 = a (7 - 1 )
+
+ = 9 6 4 = 33
+ x [1 /2]
Hence, the 7th term is 33.
= - 5 24 2-2 = - 5 60 = 55
+ x + 22. Given, 2n - 3n 2 and 6n - 1 are in
l, + AP.
(1 /2]
as
1 4. Do same Q. 6 ofN cert Folder 2 .2. [Ans. No] :.Second term -First term =Third term -Second term (1 /2]
as
Do same Q. 4 ofNcert Folder 2.2. [Ans. 10th term]
33nn 22 -- (2n - 1) = 6n - 1 - (3n 2)
1 5. => + + (1 /2]
as
1 6. Do same example on page 56. 7 [Ans. 40] => + 2n 1 = 6n - 1 - 3n - 2
+
1 7. 7
Do same as example on page 56. [Ans. 166] => n 3 = 3n - 3
+
. 1 8. Given, an = 2n 1 + => 3n - n = 3 3 +

On putting, n = 1, 2, 3, we get => 2n = 6


n = -6

[ = �2 [3 7]
a1 = 2(1) 1 = 3 =First term (a)
+ =>
2
a2 = 2(2) 1 = 5 +
� 7
=> n = 3, which is the required value of n. (1 /2]

·: = �(a a = 3, I = 7 and n = 3J
= 2(3) 1 = + (1 /2]
Now,
53 + i.e. 5, 11,numbers
17. are 2 3 - 3 3 2, 6 3 - 1
x l, x + x

(1 /2]
23. AP.

= �2 10 =. 3 5 = 15
Given, 21, a, b and - 3 are in
a
s. + /), So, aa - a21 -= b -21
=
-;;;·;- => 53
=>
=>
+ b
x x
(1 /2]
and 2a - b = 21 ... (i)
Alternate Method
-a3--bb-=bb=-3 a
53
=>
= 3 5 7 = 15 + +
=> ·a -(i)2bby=23and subtracting Eq.... (ii(ii))
1 9. Given, common difference, d = 3 On multi p lyi n g Eq.
Consider, a20 - a1 5 = (a 19d)-(a 14d) + + from Eq. (i) , we get
[ ·: an = a (n - l)d] + 3aa == 1339
= 5d (1]
5 x 3 = 15 (1 ] On putting a = 13 in Eq. (ii), we get
. r
�,
20.
.
Given, 1 3 5 . . . upto n terms = 9
+
+
+ +
2 5 8 . . . upto 8 term => -132b- =2b3 =-313
- 2b = - 10
+
=>
+

=>
n- [2(1) (n - 1)2]
+ =5
b (1 /2]

[-: = �{2a (n - )d)]


2 =9 Now, a b = 13 5 = 18
+ + [1 /2]
24.
8 2(2) (8 - 1)3]
-[
2
+
(1 ]
Natural numbers between 101 and 999 divisible by
both 2 and 5, i.e. divisible by 10, are 110, 120, 130, ... ,
:.1 s. + l 990
Here, 120 - 110 = 130 - 120 = ... = 10.
Arith metic Progress i on 81
So, ir is an AP wirh firsr rerm, 11 = 1 1 0, com mon Lee I = 1111 = a + (n - I )d
difference, ti = 1 2 0 - 1 1 0 = I 0 and lasr rerm,
I = 99 0 .
990 = 1 1 0 + (n - I ) x 1 1

I = 11,, = II + (11 - I )d
=> 990 - 1 1 0 = l l (n - I )
Let [1 1
990 = 1 1 0 + (11 - 1 ) x 1 0
=> 880 = 1 I (n - I )
Then, n - I = 80
990 - 1 1 0 = 1 0( 11 - I )
=>
=>
=> n = 81 [ 1 /2]
=> 880 = 1 0(11 - 1)
=> 11 - I = 88 SI I = -n [a + /]
2
=> II = 88 + 1 81
=> II = 89 58 1 = - (1 1 0 + 990]
Hence, 89 namral numbers arc there.
2
[1 ] 81
25. Lee ti and ti be the fi rse term and common difference of = - x 1 1 00
2
the given AP, respeceivcly.
= 8 1 x 5 50
t14 = 0

J
Given
1111 = II + (11 - 1 )dj
=> 58 1 = 445 50
(·;
[Ans. !�I
ti + 3d = 0
=>
=> II = - 3d .
. . (i) [1/2)
which is rhe required sum. [ 1 /2]

To !'rotJt' 11 :!.5 = 3 11 , , 30. Do same as Example 3 ar page 62.


Proof fly., = II + ( 25 - J )d
3 1 . Do same as Example 3 at page 62. [Ans. 8 00]
= II + 24d

= - 3d + 24d [from Eq. (i)] 32. Do same as Example 3 at page 62. [Ans. n 2 + 2n]
=> 11:!.5 = 2 1 d . . . (ii) [ 1 /2) 33. Given, a1 2 = 47
and 11 1 1 = 11 + (1 1 - l )d => a + I I d = 47 [·: aII = a + (n (i)
- l )d] . . .
= II + 1 0d [ 1 /2]

I)
= - 3d + 1 0d and a16 = I + 2a8 [by given condition]
=> 11 1 1 = 7d => [a + ( 1 6 - I) d] = I + 2 [ a + (8 - d ]
:=:) 3tl1 1 = 2 1 d . . . (iii) [1 /2 ] => a + I 5 d = I + 2 ( a + 7d)
[on mulciplying boch sides by 3] => a + l 5 d = l + 2a + I 4 d
From Eq . (ii) and Eq . (iii), we gee => a - d = -1 [1 /2] ... (ii)
ti 15 = 3a1 1 Hence proved. [1 /2)
On subtracting Eq. ( i i) from (i), we get
Eq.
1 2d = 48
26. Lcr a and d be che fi rst term and common difference of => d =4
the given AP, rcspcccivcly. i) we
On purring the value of d in Eq. ( i , ger
G iven , a4 = 1 1 a - 4 = -1
=> II + 3 d = 1 1 . . . (i) a = -1 + 4
[·: 11,, = II + (11 - l )d] => a =3 [1 /2]
and 115 + 117 = 34 Now, aII = 3 + (n - 1 ) 4 = 4 n - 1
=> a + 4 d + a + 6d = 34 [1 ] lln = II + (n - 1 ) d]
[using,
=> 211 + 1 Od = 34 Hence, the nch term is ( 4n - 1) . [1 /2]
=> 2[a + 5d] = 34 34. Let, k 2 + 4k + 8, 2 k 2 + 3k + 6 and
AP.
=> a + 5d = 1 7 . . . (ii) [1/2)
3k 2 + 4k + 4 are three co sec t v
n u i e terms o an f
[on dividing both sides by 2]
[ 1 /2]
On subtracting Eq. (i) from Eq. (ii), we get
:. 2 k 2 + 3k + 6 - ( k 2 + 4k + 8 )
II + 5 d - II - 3d = 1 7 - 1 1 => 2d = 6
= 3k
1 + 4 k + 4 - ( 2 k2 + 3 k + 6 )
[1 ]
=> d =3 [1/2]
So, the requi red common difference of the given AP => k2 - k - 2 = k2 + k - 2
is 3. => - k = k => 2k = 0 => k = 0
27. Do same as Q. 1 o fNcerc Folder 2.4 [Ans. 32nd cerm] which is the requi red value of k. [ 1 /2]
28. Do same as Example 5 at page 55. [Ans. 1 5]
35. Lee 11 be rhe fi rst term and d be the common of given
29. All three-digic natural numbers, mulciple of 1 1 are AP.
1 1 0, 1 2 1 , 1 32, . . . , 990. ·
Then, a1 + 115 + a10 + a1 '\ + a 20 + a 24 = 2 2 5
Herc, common difference => a + (a + 4d) + ( a + 9d) + ( a + 1 4d)
1 2 1 - 1 1 0 = 1 32 - 1 2 1 = . . . = 1 l. + (tz + l 9d) + ( a + 23d) = 2 2 5
So, ic is an AP with first cerm, a = 1 1 0, common
difference, d = 1 1 and lase term, I = 990.
=> 6a + 69d = 225
[1 ] => 2a + 33d = 75 . . . ( i) [ 1 ]
AllinOne MATHEMATICS Class 1 0th Term I

Now, S24 = -242 [2a + (24 - l)d] n + d = -- 207


3
x
== 1212 [2a75+ 23d][usingEq. (i)) and produce of smaller partsa +=d4623
=> = 69 ... (i) ((1)

= 900 [1 ] => a(a + d) == 4623 4623


The of 7 lying between 500 and 900 are 504, => a (69)a = 67 [from Eq. (i)]
36.
511,multi
518, p. ..les, 896. =>
On putting a 67 in Eq. (i), we gcr
=
(1 /2)
Clearly, it forms an AP. 67 d = 69
+
Here, a = 504 and d = 511 -504 = 7 [ 1 /2]
=> d=2
Let there are n terms, i.e. an = 896 ( 1 /2]
First part , a =67
(1/2)
=>
504a ++ (n(n -- 1)1)d7 == 896 896 [·: an = a + (n - l)d]
:.Secondpart,a +2dd== 6767 ++ 24 = 6971
=>
=> (n(n -- 1)71) 7 == 392
896 - 504 Third part, a + =
=>
Hence, three pans of 207 are 67, 69 and 71 . [1)
=> n = 56 + 1 40. Let the three terms in AP are
=> n = 57
Then, we have a-d, a + d a,
(1 /2] ( 1 /2)
l)
Now, S57
n
= -2 (a + = -572 (504 + 896) (a -d) + a + (a + d) =33
s. [·: = % (a + /)J =>
=>
3aa =1=331 (1 /2)
x
= 57 21400 =39900 Also, it is giexceeds
thihaverd term ven thatthethesecondproduct termofbythe29, first and thewe
therefore
Hence,
and 900theis 39900.sum of all multiples of7 lying between 500 (1 /2]
(a-d)(a + d) =a + 29 (1 1

Do same as Q. 4 of Ncert Folder 2.3. [Ans. 10) => a2 -d2 =a + 29


37.
=> 121-d2 =11 + 29
38. Alldividtwo-di
ed byg7itlenumbers
aves remaigreater are 50 which when
nder 4, than =>

=>
121-40 =d2
d2 =81±
53, 60, 67, . .. , 95.
whi
commonch formsdiffanerence,AP width=first 60 -term53 a= =7 53, Hence, there are two APs d=and they
=> 9 are
and l
term be=the95. term of given AP. 2, 11, 20, . . . and 20, 1 1, 2, ..... .
Let llasastt term
(1 /2] (1 ]
nth
41 . Do same as Q.11 ofNcert Folder 2.2. [Ans. 36th term]
Then, l = a + (n - l)d
· .

=>
95 =
7(n7(n - 1) = 9553 +- 53(n - 1)7 ( 1 /2] 42. Two-digit odd positive numbers are
11, 13,an AP.15, 17, ... , 99. [1 /2]
=>
n --1)n1 === 6427
ClHere,early, it forms
=>
ana == 1991, d = 13 - 1 1 = 2
= % (53 + 95) [·: = % <a + /)]
=> (1 /2] and (1 /2]
Now, required sum = 53 + 60 + 67 + ... + 95 a11++ (n(n -- 1)1)d2 == an99
s. => (n - 1)2n2 == 8890 n2n=-452 = 88
=>
=> => [1 ]
= ?_2 x 148 = 518 (1 /2] Now, S45 =. - 45 [2 x 1 1 ( 45 - 1) 2]
+
2
39. Let a be the first terms and d be the common difference
ofAlso,
the given AP. [-: = % I 2a + (n I) d)]
s. -

let first part = lst term of= an AP a,


· '

!.1
1.;
Second term = 2nd term of an AP = a + d
and third term= 3rd term ofan AP= a + 2d = 425 ( 22 + 44 2) = 425 ( 22 + 8 8)
x

,;
ii According to the question, we get =45sum(IIofal+ l 44)two-di= g45it odd
(55)=2475
!.i1 1 a + a + d + a + 2d = 207 Hence, the positive numbers is
u
!1
:::::> 3a + 3d =207 2475. [1 ]
Arithmet i c Progressio n 83

a + 4d +aa + Bdd == = a + (n - I )d]


4 3 . Lee n a n d ti b e r h c fi rst term and common difference of
Now a5 + a9 = 72
given AI� rcspccrivcly. => [a + - l)d] + [a + d] =
(5 (9 - 1 ) 72
·: [ 1111
According co rhc given condition , we have
fl1 1 = 2 1 3 => 72
=> fl + - d =
(1 2 I) 213 => 2 + 12 72 ... (i) [1 /2]
l id = 213 a+ = a1 2
fl + [a + - l)d] + [a7 + - l d =
=> . . . (i) [1/2] and 97
and 54 = 24
=> (7 (1 2 97 ) ]

=> _! [ + d] = => a + Gd + a + I I d = 97
2
2fl (4 - 1) 24 => a+ d=
2 17 97 . . . {ii) [ 1 /2 ]
=> [ + 3d] =
2 2fl 24
On subtracting Eq. (i) from Eq. (ii), we gee
5d = d=
+ 3d = 25 => 5 [1 /2]

-
2fl 12
=>
[dividing boch sides by 2] . . . (ii) [1 /2]
On putting d= a+ =
5 in Eq. (i), we get 2 60 72
=> a=
2 a=
1 2 => 6 [1 /2]

= [ ( - ) + (I O ]
On solvi ng Eqs. (i) and (ii), we get
Hence, the required A P is given by
fl = d= a, a++ a+ + x a ++ x a + + 5 x
507 414
and d, 2 d, 3 d, 4 d, . . .
19 19 [1 1 i.e. ( 6, 6 5, 6 5 2, 6 5 3 , 6 4, ...
Now, 51 0
10
2 x
507 - 1) x
414 i. e. 6,11, 1 6, 2 1 , 26, . . . [1 /2]
2 19 19
47. Let a d
and > 0 be che fi rst term and com mo n

[·: S., = %Ila + - l)d}] (n difference of given AP, respectively.


Let four numbers in AP are
= 2- [- + = 1014 3726]
27 1 2 x 5 a, a + d, a + 2 d and a + d
3 . . . (i)
19 19 Here, smallest number is
a+ d a and greatest number is

= 7 1 3.68
3 . [1 ]
= 1 3560
19 [1 ]
According to the given condition,

[Ans. (a) + (a + d) + (a + 2d) + (a + 3d) = 50


44. Do same as Q. 1 1 ac page 69. 2, 1 2, 22 and 1 52) => 4a + Gd = => a + =
50 2 3d 25 . . . ( i i)
a d
45. Lee be che fi rsc term and be che common difference. [dividing both sides by 2 ]
(a + 3d) = (a)
Then, third term a3 = a + 2d and 4
and sevenceenth term 1 =
a 7 (a[·; +an = a + - l)d]
1 6d) => 3a - 3d = 0
(n [1 ] => a=d . . . (iii)
According co the question, we have On puttinga=d i n Eq. (i) , we get
a + +a =a3 2d + 3d =
a + a + 2d1 + a + 16d17 = 216
216 25

3a + d = 216
=> 5d == d = d = 5
25 => [1 ]
=>
a + 6d =
18 a 5 [using Eq. (ii)] [1 /2]

--.
=> 72 . . . (i) [1 ] H ence the required numbers are
3]
[dividing both sides by 5, 5 + +5, 5 2 x 5 and 5 + 3 x 5 [1 /2]
10,
[-: = % a + (n - l)d}]
Now, sum of fi rst 1 3 terms of the AP, i.e. 5, 1 5 and 20

S1 3
= !i [2a + (13 - l)d] 48. Let and be the first term and com mo n difference o f
a d ak '

2 the given AP. Let denotes the kth term o f the given �:
ak = a + - I )d
. = � [ (a + 6d)]
12

: = ; 1 a, a ) ;,: ·: ·; /)]
s.
AP. Then, (k
let
�' ==:l : :� + a
51 Sum of odd terms

r i�:
Now,

=
�[ a + 2 1 2d] 2 n
2 2 s,
�[ = 2 x 72] [from Eq. (i)]

= 13
2
72 = 936
x
=> S1 = -- + I [a + a +
n
2
+I- ( 2n I ) d]

S2 = t12 + a4 + a(J + .. +
13
Hence, the sum of first terms of the AP is 936. (1 ] _
[·: a11 = a + - (n l ) d]
=> S1 l)(a
+ + nd)
= � -·

S.-, = -2 - + -
(n [1 ]
a d
46. Let be che fi rst term and be the common difference and 52 = Sum of even terms
=> a 2"
of given AP.
a5 + a9 =
It is given chat, 72
1l
(n, n , 11 )
·

and a7 + a1 2 = 97 [1 /2] =>


84 \ Allinone MATHEMATICS Class 1 0th Term 11

=> S2 = -n2 [(a + d) + {a + (211 - I)}d] Thus, n th term of firsc= AP,


T,; 1 1 - =
1 I (16)
211 - 2
[·:a,, = a + (n - I)d] = 11 - = 32 - 21

S = (a + n d) 1l
and mh term of second AP,
r,;' = 27 - 3 11 = 27
:. Required ratio2 ==S1(n: S2+ l)(a + n d) : n( a + n d)
=> [1 ] - 3 (1 6 )
= 27 - 48 = - 21
Hence, che val u e ofn i s 16 and mh ccrm is 1 - 2 . (1 12)
= (n + I) : n Hence proved. (1 /2] 51. Let erence >
a and d 0 be the first term commonon, and
49 . The given sequence is an AP with first term, a = 20 diff
we have of an AP. According co the given condici
and the common difference, d = 19 .!_3 - 20. a1 + a 5 = 26
=> a + a + (5 - l)d = 26
=-
58 - 20 = - 2 - => 2a + 4d = 26
=>
3 3 [1 /2] + 2d = 13
Let the sum of first n terms be 300. [diaviding both sides by . (i)
114
2) . . [ 1 /2]

� [2 20 + (n - 1 { - rn = 300
Then, S,, = 300 (1 /2] Also, we have d) (aa2 + 3 d) == x 1 60 [1/2)
=> !!.. =>
(13 - 2d(a ++ d) (13 - 2d + 3 d) = [ from Eq. (i))
1 60
2 [2a (n - I)d] = 300
+

x

1 60
� x
=> d)
(I 3 - (I 3 + d) = 1 60
� n(l20 -2 2n + 2) = 1800 => (13)2 - (d)2 = 160 [1/2)
=> - 2n + 122n = lSOo => d2 = 169 - 1 60
=> n2 - 6 ln + 900 = 0 � d2 = 9

� n2 - (25 36)n [di900vidi=ng0both sides by-2]
+ + Bue
d=±3
d 0 >
� (n - 25)(n - 36) =0 d=3 [1/2)
=> On putting d = 3+in 2Eq. (i), we get
Hence, sum of 25 terms =Sum of 3625orterms36 = 300
n = (1 ]

tl x
3 = 13
Here, gotheoncommon diffngeandrence31stis term
negatibecomes
ve, therefore a = 13 - 6
terms dimini s hi zero. � a=7 (1 /2]
Allnegatitheveterms after
termsv(from 31st term
32nd are
term negati v e. When
toterm36thto term) these
when Now, the sum of first seven terms,
7
added to positi e terms (from 26th 30th term), - x
S7 = 2 [2 a + ( 7 - 1 )d]
they cancel out each other and thus sum remains same. (1 1 = ?_2 [2 x 7 + 63] x

50. Let the first term and common difference of the AP : 9,


-
7, 5, ... are a1 and d1 respectively. Then, first term
(a1 ) = 9 and common difference (d1 ) = 7 - 9 = - 2.
= ?_2 [14 + 18] = 2 32 1 12 ? x
==

[ 1 /2)
>
----
Now, its nth term, T,; a1 + (n l)d1 = -
11 52. Given, first term, a = 5
:::::> T,; = 9 + (n - (-2) 1)
n According to the question,1
. � r,; 9 - 2n + 2 =
117 , 118)
=>
115 ,
I
-
=> T,; = 1 1 - 2n ...of(i) (1 ]
S4 = 2 {Sum of a6 ,
Let the first term and common respecti difference the
a2 and d2
"'"" AP : 24 , 2 1 , 1 8 , . . . . a re
1
first term (a2 ) 24 and common differencevely. Then,
..., _
=- (S8 - S4 )
2
=
�-S: 11 => 23 S4 = Sa - S4
;- · f;1 (d2) = 21 - 24 = - 3 ... (i) [1]
Now its mh term, T,;' = a2 (n - l)d2

[·: s,, ]
S4 = Sa
.. \\ => T,;' = 24 ( 3) +
+ (n - l) - Clearly, S4 = _i2 [2a + (4 - J) d]
=> T,;' = 24 - 3n 3 +
=> T,;' 27 - 3 n = ... (ii) [1 ] = % 1 2a + (n - l)dl
-; Since, nch terms of the both APs are same, i.e. T'II = T"
II
,_.,

.:? � ·,
2n = 27 - 3n [from Eqs. (i) and (ii)]
11 -
= 2 [2 (5) + 3 x d]
�· => n = = 20 + 6d
I�
....
16 [1 /2)
=> S4
85

S8 =-[2
Arithmetic Progression

8 5 + (8 - I)d]x
55. Given nchat, g year.tree grow 5 cm formed. or 0.05 m less than
On putcing3 the values of S4 and S8 in Eq. (i), we get
and 2 precedi
= �2 (10 + 7d) = 40 + 28d :.The folI, l(Iowi-n0.g 0sequence 2
can be0.05), ... ,0
I = an
[1 ] 5), (I
-

i.e. I, 0.95, 0.90, .. . ,0 which is an AP. x

[1 ]
(20 + Gd) = 40 + 28d Here, a = I, d = 0.95 - 1 = - 0.05 and I = 0
GO + 18d20 == lOd 40 + 28d 0 I
=

+a + I)(n(-- 0.I)d0 5)
-
d=2 (n - 1) 0.05 = I 1 = (n [1 ]

Hence, common difference is 2. [1 ] n - 1 = 0.-05

S1 = S" =!!.2 [2a + (n - l)d]


53.
ofLetgiavenbe AP.the first term and d be the common difference n=- 100 + 1= 20 + 1 = 21
5
S2 = S2,, = -2112 [2a (2n - l)d] ... (ii)
According to the question, [1]
.. . (i) Hence, in 21 yr, tree wil have ceased growing. [1 ]

and S3 = S3n = -[2a


56. Clearly, second
month, the amount month, (inthird') ofmonth
instalment
... are, in the vfirst
respecti el y,
+
1000,term,1100, 1200, 1300, .. which forms an AP, with
Now, S2 - S1 = -[2a
.
311 + (3n - l)d] . . . (ii ) first
2n2 [1 1
Now,a =amount 1000 andpaicommon
d i n 30th diff
i n erence, d = 100
stalment,
2 + (2n - l)d] a30 == 1000
1000 ++(3029 -1)100
x 100 [ = - )d]
·: a,, a + (n I

- !!.2 [2a + (n - l)d] [1 ] = 1000 + 2900= '3900


!!.
=
2 [2{2a + (2n - l)d} - {2a + (n - l)d}] Amount paid30in 30 instalments,
3(S. 2 - S1) = -[2a [·: S" = !!.2 . { 2a + - I)d}]
= !!.2 [2a + (3n l)d] S30 = -[2 2 1000 + (30 - 1)100)
3( 52 - S1 ) S3
-
x

[1 1

S3 = 3(S2 - S1)
3n + (3n - l)d]
2 (n

S11 and= Sumcommon


[using Eq. (ii )]
== 15[4900]=,
15(2000 + 2973500100) = 15[2000 2900]
= x +

Hence proved. [1 ] [ �] 1

:. S1 = !!.2 [2 1 + (n - 1) 1]
54. of first n terms of an AP whose first term is Hence, remaining amount of loan chat he has co pay
= ' 118000 - ' 73500 = ' 44500
S5 + S9 = 72
differences 1
= 2 (n + 1) [·: Sn = !!.2 . { 2a + (n -l)d}]
[1 )
x x

ofGivtheen,a berequitherfirst
57. Lee
ed term and d be the common difference
AP. • I �---- :;:

!!_

S2 = !!..2 [2 1 + 1) 2] = [-: = % I) d}J


2_ (2a + 4d) + 2. (2a + Bd) = 72 :·_":_�-�
[1 /2]
S2 = Sum of first n terms is andof ancommon
AP whose first term =>
2 2

is 1 andS3 =common
1 difference is 2
x
-
x (n - n2 s. (2a + (n

S3 = !!.2 [2 1 + (n - 1)3]
[1/2)
and SumoffirscncermsofanAPwhosefirsccerm => 5a + IOd +14a9a ++ 46d 36d = 72
=

difference is 3 =>
=> 7a[dividing
+ 23d both 72
= 36sides by .. (i) .
x
2) [1 J

Now, S1 + S3 = !!.2 . (n + 1) + !!..2 (3n 1)


:�
= !!..2 (3n - 1) and 7 S7 + S1212= 97 . �·---

=>
[1 ] 1 '; _:- -_
-

2 (2a + Gd) + 2 l ld) = 97- (2a +

SI + S3 = 2S2
-
...: :....-.._:

= !!.2 . (4n) = 2n2 => 7 a + 2 ld + 1 2a 66d + 97 =

On multiplying Eq. (i) by 119a


=> +
87d = 97
9 and Eq. (ii) by 7, we
.. . (ii) [1 ]
gee
I I
' 1
-...:....::..,
.

[1 ] 133a 437d = 684


+ .. . (iii)
Hence proved. and 133a + 609d = 6 79 . . . ( iv)
86 � AllinOne MATH EMATICS Class 1 0th Term 11

O n subtracting Eq. (iv) from Eq. (iii), we get a


60. Lee be chc fi rst ccrm and d be t he com mo n d i fference
-172d = 5- 5 of given AP.

7a + 23 (�)
=> =- 172 d
[1 1
G iven ,

On putting the value of d in Eq. (i) , we get


a+ (m - I) d =- I
11
... (i)
172 = 36 a,, = -
=> 7a = 36 + - 115 7a = --=> =>
901
6307 a :::: - and
m

172 172 172 a+ (n - I) d = -m I


. . . (ii) [1]
Thus, the required AP is a, a + a + 2d, ... d,
901 ' 901 - 5 , 901 - 10
- - - - -

= -- :::::::> = -
On subtracti ng Eq. (ii) from Eq. (i), we get
1.e.
172 172 172 172 172 (m - 11) d = -I - -
I
1.e.
901 , 896 , 891
- - -
m- 11 111

172 172 172 [1 1 (m - 11 ) d


""'
[1 1
11
d
I
1/lll
58. Given, equation is 1
d = - 1 :- ( - )
- 4, - + 2 + . . . + x = 437
l, . . . (i) On puccing d = -- in Eq. (i), we get
forms an AP with first term,
11111

a = - -4, 4, - 2, , x 4 = 3
Here, I, ...
common difference a + (m 1 = -1- I) -
1/lll 11

a,, = x a + -n1 - - 1 I
and last term, [1 1
mn = -n 11J11

-- --
· : nth term of an AP, [1 /2)
Now, sum of nm terms,

[·: S, = 2 2a + - l)dJ ] [1 /2)


= a + (n - 1) d
a,,
mn + (mn - 1)
=> x = - 4 + (n - 1) 3 = -[2a

:
Sm,, d]
2
= mn2 [� nzn J
=> x + 4 = n - 1 =>n = x + 7
�{
. . . (ii)
3 3
= x2 +x 7 [2 (-4 ) + ( x + 4 ) . 3 ]
[1/2] (11

· Sum of first terms,


n
n S,, = -2 [2a + (n - 1) d]
1
=> S"111
mn + (mn - - I)

-3-
sx+7
3 3 = -21 (mn + 1) [1]
= x2 +x 37 (- 8 + x + 4 ) = (x + 27)x(x3 - 4 ) Hence proved.

Now, from Eq. (i), we have S,, 437 =


61 . A
Let be the fi rst term and D be the common d i fference
of the given AP.
=> ( x + 7) (x - ) = 437
4
Given, a= Sum of first p rem�

[·: % (2n + (n - J
2 3 x [1_ 1
=> x2 + 7x - 4x - 28 = 874 3 x => a = f!...2 [2A + (p - l)D]

=> x2 3x - 2650 = 0
+ [1 /2] S., = l )d }

.. = -3
x
� ±
(3) 2 - 4 (- 2650) a 1 + (p - 1
2 -p = -[2A
2 )] . . . (i)
[by quadratic formula] [ 1 /2)
= Sum of first q terms
- 3 -J9 + 10600 = - 3 JUi609
=
± ±
b

� !!_ [2A + - 1) (q
-3 1032 100 - 106 2
±
b
21 D]

= 2 = 2 , = 50, - 53 i - = -2 [2A + - 1)
b
(q D] . . . (ii)
q
= 50x [ 1 /2]

]
Hence, [1 1
[·: x c

2
and

n
= Sum of r terms

5
cannot be negative as given

[
-
= [2A + - 1)
AP is an increasing AP] r
=> c (r D]

59. Do same as Q. 9 2. 3 . Ans. 2- n 21


(r -
of Ncerc Folder c
=> - = -2 [2A + 1)
r [ 1 /2]
D] . . . ( i i i)
1.
Arithmetic Progression I 87
On mulci plying Eqs. (i), (ii) and (iii) by
(r - p) p-q (q - r) , Now, consider che ratio of their 9 th terms, which is
and respectively and then adding,
= a'a ++ 8d'
8d 2 (a + 8 d)
we get - q
P
ti
( - + -b (r - p) + -r (p - q)
r)
q
c 2 (a' + 8d' )
[mulciplying Numerator and denomi nator by
2a + 6d 2a + (17 - I )d 2]
- I)
= -2 [2/1 + (p DJ (q - r) + {2A + (q - I )DJ
1 l

(r - p) + + (r - l ) D } (p - q)] [1 ] 2a' + 16d' 2a' + - I )d' (I 7 [1 ]


( 2A 4 17 + 2
x

= -21 [ 2A (q - r + r - p + p - q) + D (p - I ) 3 x 1 7 + 47
- I)
(q - r) + (q (r - p) + (r - l ) (p - q) [1 /2) n = 17
[subsrirucing, in Eq. (i)]

I 70 5
2 x 0 + D{pq - pr-r -+ qp ++ rrp+-rq
= -[2A qr - qp 78 7 [1 /2]
- p + q }] 64. Lee first term and common difference of given AP be a1
= -2 [O + D O]= 0
I
x Hence proved. and d, respectively.
a1 + a,
62. Lee the four consecutive nu mber in AP are
[1 1
Now, -a11 + -a1, = �-� a1 · a,,
a - 3 d, a -d, a+ d, a +3d [1 /2)
-1 + -- 1 = ----a2 + al - 1
Then, we have a, a2 · al - I - 1
( 11
(a -3d) + -d) +(a+ d) +(a+ 3d) =32
=> 4a =32 = a1 .+fl d +a,, a, - d - I
=8 ti [1 /2]
Also, it is given chac the ratio of the produce of first and
[ ·: a,, - a,,_ 1 = 2d=> a, _ = a,, - d ]

che lase terms to the product of the two middle terms is


7: 15, (11 -3d)(a +3d) 7
therefore we have .a2 · al - 1 [1 ]
=-15 1- + -- a 1 + a,
S 1m1. 1 ar1 y,
a_,, al _ 2 = --' a_,, -· a--"---
. 1
2 (a -d)(a+d) [1 1 ,_ 2

a 2 -9
=> 2 tl2 = !_ => 15a 2 -135d2 =7a2 - 7d2
a -d 215

: [ � : 7 :� . _ _ ]
=> 8a 2 =128d => a 2 =16d2 -a1, + -a11 = --'--
a1 - + a,,
=> 64 =16 d2 [·: a = 8) a1 · al--- [1 ]
On adding all t e a o , e
=> d=± 2 14
a1 a2 a3 +.. + a!, . [1 ]
[-1- 1
Hence the number are 2, 6, 10, 14 , 10, 6, 2. [1)[1]
or

63. Lee a, d
be the first term and common difference of
= (01 + al ) a1 al + a2 al
first AP, and a', d'
be the first term and common · • - 1
difference of second AP.
n [1 /2)
+ a_,, . Ial - 2 +...+ _ I_ ]
1 1 1
Then, sum of first terms of first AP
II a, . a1 [1 ]
= -[2a
2 + (n - l )d]
and sum of first 11 terms of second AP => -- + --- + + ...+ --
- (11 -
1l
=
2 [2a' + l )d' ]
Since, ratio o f sums o f first n terms of two APs is
[1 ]
2
H ence proved.
4n + 2 :n3n + 47, therefore we have
- [2a (n - I )d] 411 + 2
+
�2=---�--- =
a d
65. Lee be the first term and be the com m o n d i ffere nce
of rhe given AP. Then, rhe sums of first m and n terms
� [2a' +(n - I )d' ] 3n + 47 b y S/11 = -m [ 211 + -
2 2a + (n - )d 4n + 2 ·
are g1ven
2
(m l) d]
I . . . (i)
= [ + - !!_ 2a l ) d], rcs p cctivelv
2a' + (n - l)d' 3n + 47 (1 ]
and 511
2
(n
. [1 ]
88 \ .Allin.One MATHEMATICS Class 1 0th Term 11

S ince, =-229 (4 56) =-292 60 =870


+ x
[1 ]
m Hence,
be t' 870.her savings in the month of February will
2 [2a (m - 1) + d]
Saving values.
!!..
{it) (1 )
22a[2a (n- 1)- 1) + (m
+
d
d]
m
68. Do same as Q. 28 (3)
[Ans. 15]

2a (n - 1) = -n + d
Ram cal c ul a ted i
intelligency are depicted by Ram. t by using AP. So, time saving and
(1 )
(1 ]
=> [2a n = [2a - 1) Do same as Q. 13 of Ncert Folder page 59.
2a (n -(mm)- =1) d [(=n - 1) - (m 1) n]
+ d] + (n d] m 69.
=> m - [Ans. 32nd term]
2a (n - m) (n AP,
Byintelliusing Reena sames time. So,
= 2a d
d - m)
(1 ]
gency are depicted by Reena. time saving and (1 ]
am a Given, total costCashof thepayment house == '' 400000
2200000
-----
+ (m - l) d 70.
Now, required ratio = -an = a (n - 1) + d
+ (m
= aa (n -- 1)1) 2a2a = 2m2n -- 11 ---
Balance = " 2200000
In 1st instalment -' 400000
she pay= 100000= ' 10%
+
1 800000
of balance
+
or am : an =
(2m - 1) : (2n - 1) Hence proved. = 100000 �
+ x
100 1800000
(1 ] = 100000 180000 = '280000
( t)
+
66. Labour
150, 150charges50, 150
+
in � for2each50,subsequent
+ 150 3 metres
x 50, are + x
Balance after 1st instalment = 1800000 - 100000
i.e. 150, 200, 250, 300 ,.. . [1 ] In 2nd instalment she pay= 100000 = � 1700000 [1 )
10% of 1 700000
AP .
+
Thus, amount of labour charges form an 100000 1 70000
== '270000
+
Here, first term, (1 /2)
a = labour charge for first metre = 150 Balance after 2nd=1700000 instalment-100000
·subsequent
: Labour chargemetres.is increasing by 50 for each �

.. = 50 d [1 ] :. In 3rd instalment, ='1600000


she pay
Total depth =10 m == 100000
+ 10% of 1600000
:. Labour charge for 10 m depth 100000
+ 160000
=150 9 -1) 50 = a (n - 1)
n=instalments
" 260000 . . and so on. [1 /2]
+ (I O
=150 + x
50 [ · : an + d] :.Total amount= � i280000
=150 450 =600 + + '270000. .
Hence, � 600 should be paid to the labour. � + .
260000 terms
Here, a = 280000, = -10000 and upn =to 1818
[1 ] +
Ifhonesty
(ii)Ram paysand �sin600ceritoty.the labour, then it shows his d

67 . (t)
On first day, she saves=� 2
(1 ]
= �2 [2(280000) (18 - 1)(-10000)]
+

On second day, she saves = � 4 n !!..


[·: S = [2a + (n -
l )d ]
On third day, she saves = � 6 and so on. 2
AP,
Thus,
term, asavi= n2gs amount form an
whose first === 999 [560000
[560000 - 170000]
+ 17( -10000)]
[390000] =' 3510000
and common difference, = 4 - 2 = 2 d (1 ]
.·.Total=cost3510000of house 400000 = ' 3910000
We
are 29knowdaysthat, year 2012
in month is a leap year. So, there
of February. + [1 ]
So, we have to find her savings in 29 days. Difference of costs-of2200000
= 3910000 the house= in' 1710000
two options
Thus, we haven n = 29 (1 ]
(a) Amount donated to National Relief Fund
Now, Sn = -2 [2a (n + - l) d] = 50% oH. 1 710000
= 292 [2 2 (29 - 1) 2]
529 x + =� x
100 1 7 1 0000 =� 855000

= 292 (4 + 28 2) x
(b) National responsibility and help to weaker section
of the society. (1 ]
• •

Act1v1ty
Topic Covered Arithmetic progression. We have, n = 5

Objective To verify graphically that the sum of first n even Then, S = n (n + 1) = 5 (5 + 1) = 5 x 6 = 30


natural nu mbers is n(n + 1). Thus, formula and experiment g ive the sa m e
answer.
Skill Developed C reativity, analysis, knowledge of concept
of natural numbers, formula of areas of a square and a
rectangle. Oral Questio n s
Time Req u i red 15 min 1 . What i s seq uence?
2. Define arithmetic prog ression.
Material Req u i red A 10 x 10 grid paper, coloured pencil, 3.
scale. What is the genera l formula for nth term of an AP.

Method 4. Write the formula for sum of first n terms of an AP.

1. First, we choose n. 5. Common d ifference of a n AP is a lways consta n t or

let n
variable.
= 5
6 . For a n AP : 2, 4, 6, 8, ... , what is the first term a n d
Then, S = 2 + 4 + 6 + 8 + 10
common difference.
[sum of fi rst 5 even natural numbers]
2. (a) The first n u m ber is 2. Colour the first two squares True / False
6n + 5, then common d ifference is 6.
green as shown in fig ure.
1. If Tn =

2 . The number of terms of the A P : 3 , 6, 9, . . . , 111 i s 37.


(b) The next n u m ber is 4. Colour the next four squares

3 . 1 2 , 2 2 , 3 2 , 2 , a re in AP.
adjacent to the two green squares as blue.
4 •••
(c) The next n u m ber is 6. Colour the next six squares
adjacent to the fou r blue squares as red. 4. a30 - a20 of AP : a, a + d, a + 2 d, a + 3 d, ... i s 1 0d.
(d) The next n u m ber is 8. Colour the next eight squares 5. If nth term of the l ist of n u m bers is 2 n 2 - 5, t h e n it
adjacent to the six red squares as yellow. forms a n AP.
(e) The next n u m ber is 10. Colour the next ten squares 6 . The missi ng term of the AP : 2, D, 20, 29 is 1 1 .
adjacent to eight yel low squares as pink.
7 . The sum o f a l l natural n u m bers from 1 t o 1 00 i s
5050.
I I I I I-
5
I I I I Pink
4 I I
--·
Fill in the Bla n ks
I I
I I
:I Yellow
I
I
--
I
-- -

I I
3 1 . I n an AP, the common d ifference is a lways ... .
I I
,.. -- ---
2. The constant difference between the consecutive
Red
'"
--
I
I Blue
I
-- - -- - - ---
I
I -
1 terms of an AP is cal led ... .
3. If there a re p terms i n a n AP, then nth term from
Green
1 2 3 4 5 6
the end is ... th term.
Observation 4. If a = -10, n = 15 and on = 1 8, then d = . . .
The number of coloured squa res in the grid is 5. Q = . . ., d = -5, n = 20, On = -
15 .

i.e. (length x breadth) 5 x 6, i .e. 30. 6. Find the missing term of the AP : 1 , D, 7, 1 0.
Let us verify using the formula.
I-
Allinone MATHEMATICS Class 1 0th Term II

Match the Colu m ns


I. Match the following columns.

Column I Col u m n I I
1. Sum of first 1 O terms of the list of numbers (a) 1 1 70
.J2 .J8 .JIB, m. is
I I "•

2. The common difference of the AP given by an = 3n + 2 is (b) 80


3. The sum of AP : (-5) , (-8) , (-1 1), . . . , (-230) is (c) 55J2
4. Sum of last ten terms of the AP : 8, 1 0, 1 2, . . . , 1 26, is (d) 3
5. If a1 0 - a5 = 400, then common difference is (e ) -8930

IL Match the following columns.


Column I Column I I
1. In an AP, it is given that a5 = - 9 and a9 = 7. Then a14 =? (a) 8
2. In an AP, Sn = (n2 + 3n). Then, a1 6 = ? (b) 34
3. In an AP, a1 0 = 44 and a1 5 = 64. Then, its first term is (c) 30
4. How many 2-digit numbers are divisible by 3? (d) 27

Project / S u rvey Work


Write a short article on examples ofAPfrom daily life.
[Hint Simple interest rate EMI of loan]

Answers
O ra l Q uestions
1 . A list of numbers arranged in a definite order according to some definite rule is called a sequence.
2 . A n arithmetic progression is a list o f numbers, i n which each term i s obtained b y adding a fixed number t o p receding
term except the first term. The fixed number is called common difference (d) of AP.
3. an = a + (n - 1) d where, a = first term and d = common difference
n n
4. Sn = 2 [2a + (n - 1) d] = 2 (a + l)
where, a = first term, d = common difference, n = number of terms and l = last term
5. Always constant
6. Here, a = 2 and d = 4 - 2 = 2

Tru e/Fa l se
1 . True 2. True 3. False 4. True
5. False 6. True 7. True

F i l l i n t h e B l a n ks
1 . constant 2. common difference 3. a + (p - n)d 4. 2
5. 80 6. 4

M atch t h e Col u m ns
I. 1 . (c) 2. (d) 3. (e) 4. (a) 5. (b)
II. 1 . (d) 2. (b) 3. (a) 4. (c)
S U M M A RY
• Sequence is a list of numbers arranged in a definite order according to some definite rule.
• Progressions a re those sequences whose terms always follow a definite pattern.
• An A P is a sequence in which each term is obtained by adding a fixed number to the preceding term except the first
term. This fixed number is called the common difference ( d) of the AP.
• In genera l a, a + d, a + 2d, a + 3d, .. represent an arithmetic progression where, 'a' is the first term and 'd' is the
.

common difference.
• If number of terms in an AP is finite, then it is called a finite AP, otherwise its is called an infinite AP (i. e. n u m ber of
terms is infinite).
• The nth term an of the AP, having first term 'a' and common difference 'd' is g iven by an = a + (n - 1) d, an is a lso
called general term of the AP.
• If there are m terms in an AP, then mth term is known as last term of an AP a nd it's denoted by '/'.
I am = I = a + (m - 1) d I
where, a = first term and d = common difference.

I � I
• n t h term from the end of an AP = I - (n - 1) d where, d = common difference and I = la st term.
• Sum of first n terms of an AP

I � I
Sn = [2a + (n - 1) d ]

or Sn = [a + a0 ]

where, a = first term, d = common differences, an = nth term of an AP.


If there a re only n terms in an AP, then
n
Sn = 2 [ a + /] ,
where a = first term and I = last term.
. . . n( n + 1)
• S um o f f.irst n pos1t1ve integers, Sn = .
2
(1) Three numbers in AP can be selected as a - d, a, a + d.
(it) Four numbers in AP can be selected as a - 3d, a - d, a + d, a + 3 d.
(iii) Five numbers in AP can be selected as a - 2d, a - d, a, a + d, a + 2d.
CHAPTE R EXE RCIS E
� \ Very Short Answer Type Questions [ 1 M a r k ea ch ]

1. How many terms of AP 1 8, 1 6, 1 4, ... should be taken, 3. If the common di fference o f a n A P is 5 , t h en w h <i t is
so that their sum i s zero? CCE 201 3
NCERT Exemplar
2. For what value of P, are 2 P + I , 1 3 and S P - 3, three 4. If a,, = 5 - l 1 n, then fi nd t h e co mmon d i ffe re n ce.
consecutive terms of an AP? CCE 2009 C C E 2008

� I Short Answer Type I Questi ons [2 M a rks eac h]


5. The eighth term of a n AP is hal f its second term and 6. The angles of a triangle a rc i n A l� The g rea t e s t a n g l e is
the eleventh term exceeds one- third of its fou rth term twice rhc lea s t . Find all the a n gks o f the l ri a n g l c.
by I . Find the 1 5 th term . NCERT Exemplar NCERT Exemplar

� I Short Answer Type I I Questi ons [3 M a rks each ]


7 . I f a n A P h a s a = I, a ,, = 20 and S,, = 399, then find 11 even na t u ra l n u m bers is
(I !; }
1 2. Show tha t the s u m of fi rst
the value of n. equal to + imes the sum o f fi rs t " od d n a t u ra l
8. The sum of the first n terms of an AP whose first term
is 8 and the common difference is 20, is equal to the numbers .
sum of first 2n terms of another AP whose first term is 1 3. If S,, denotes the sum of firs t 11 term s o f a n A l� t h en
-30 and the common difference is 8. Find the value show that the common di fference d, o f the AP is given
of n. NCERT Exemplar
by
9. How many multiples of 9 lie between d = 511 - 2 �11 I + S,, 2
2
_ _ •
I O and 300?
14. In an AP, if s,, = n p and S,,, = m 2p, 111 * 1 1, t h e n p ro v e
1 0. 3
3n2 5n
tha t SP = p •
In an AP, the sum of first n terms i s + . Find its
2 2
CCE 20 1 0
1 5. If the ratio of the sum of fi rst n te r m s o f t wo A P' s is
2 5 th term .
1 1. Find the s u m o f those integers between I and 5 00, ( 7n + l ) : (4n + 2 7) , then fi nd t h e ra t i o o f t heir m t h
which are mul tiples of 2 as well as of 5 . terms.
NCERT Exemplar
1 6. The sum of the first three term s o f a n A P i s 3 3 . If t he
[Hint Take the LCM of 2 and 5 . ] product of the first and third term exceed s t he second
term by 29, then find the AP.

� I Long Answer Type Questi ons [4 M a rks eac h ]

1 7. Find the sum of the in tegers between I 00 and 200, 20. A farmer buys a used tractor for � 1 2 000. H e pays
tha t are not d ivis ible by 9. � 6000 cash and agrees to pay the ba lance in a n nual
instalments of � 500 wi th 1 2 % i n t eres t o n u npaid
I
1 8. The ra tio of the I th term to the 1 8th term of an AP is
2 : 3 . Find the ra tio of 5 th term to 2 1 st term and also
amount. What will the tractor cos t him?

the ratio of the sum of the first 5 terms to sum of the 21. Solve the equation :
fir s t 2 J term s . NCERT Exemplar I + 4 + 7 + 10 + .. . +x = 2 87

J 9 . An A P con s i s t s of 37 term s . The sum of the three 22. If the sum of first p terms of an A P i s s a m e a s t h e s u m
m i d d l e m o s t term s is 2 2 5 and the sum of the last o f i t s fi rst q terms (p * q), then show t h a t t h e s u m o f
th ree i s 42 9 . Find the AP. NCERT Exemplar its firs t (p + q) terms is zero . C C E 20 1 5
Arithemetic Progression 93
23. Find the sum o f all three-digit numbers which 2 9. Show that the sum of an A P whose firs t term is a, s econd
leaves remainder 2, when divided by 3 . term is b and the la s t term is c, is equal t o
(b + c - 2a) (a + c)

b) J·
24 . 36 0 bricks arc s tacked in the following manner,

[
30 bricks in t he bottom row, 29 bricks in the nex t 2 (b a) _
NCERT Exemplar

row, 28 bricks in the row nex t to it, and so on. In


30. If pth term of an AP is a and qth term is b. Prove t ha t t h e
(a -
how many rows, 360 bricks are placed and how
s u m of i t s (p + q)th term is P a
+ q
p - q
many bricks arc there in the top row? + b +
CCE 20 12 2

2 5. Find the sum o f the two middle most terms of 31. If a 1 , a 2 , ... , a ,, are in AP where a; > 0 for all i, then s h o w
the AP : - - , - I , - , . . . 4 -.
4 -2 I
tha t
3 3 3
c- '- + '- � r::-
l l
NCERT Exemplar ; CBSE 20 10 '- + . . . +
[ Hint S u m of the two middle most terms =
"lj U 1 + va2 va2 + Va3 V a ,, - I + " a ,,
a9 + a 1 0 ] n - 1

26. I f n times t h e n t h term o f a n A P i s equal t o m


times the m t h term, then prove that its
(m + n ) t h term is equal t ? zero. 3 2. IHOTSJ A person buys ca sh certifica tes of � 1 2 5 .
Thereafter, every year, h e buys these certifica tes o f the
_!_ and nth term is _!_, value exceeding previous year purchase by � 2 5 . F ind the
m
27. If the mth term of an AP is
n total value of certifica tes purchased by him in 20 yr.
then show that its mnth term is 1 .
33. !HOTS]
a, b and c
Interior angles of a polygon are in AP. I f the
2 8. I f pth, qth and rth terms of an AP are smallest angle is 1 07 . 5 ° and common d ifference i s 5°, t h en
respectively, then show that find the number of sides of the polygon.
(a - b) r + (b - c)p + (c - a)q = 0

� I Value Based Questions (VBQs) [4 Marks e ac h ]


3 4. Diya planted 2 3 rose plants in the first row of a flower 36. Anil repays the total loan o f � 1 1 80 0 0 b y paying every
bed, 2 1 plants in the second row, 1 9 in third row and month starting with the first instalment o f � l 0 0 0 . H e
so on. I f there were 5 rose plants in the last row, then increases the instalment by � 1 00 every month.
find the number of rows in the flower bed. What
( i ) What a m o u n t will he pay a s the l a s t i n s t a l m e n t o f
value is depicted from this action?
loan?
35. Rajeev dona ted ' 700000 to a school from his life long ( ii ) On 5 th o f every month the a mo u n t o f i n s t a l m e n t
savings for giving 7 cash prizes to the students for i s d irectly t ra n s ferred from h i s b a n k a cco u n t .
their academic performances. If the worth of each Therefore, Anil e n s u res s u fficient fun d s i n h i s
prize is � 20000 less than the worth of its preceding bank account b e fore 5 t h o f every m o n t h . W h a t
prize, then find the worth of each prize. What value is values a re depicted by Anil i n t h i s a c t ?
depicted from this action?

Answers
1 . 19 2. 4 3. 25 4. - 1 1
5. 3 6. 40°, 60° and 80° 7. 38 8. 1 1
9. 32 1 0. 76 1 1 . 12250 1 5. 1 4m - 6 : Sm + 23
1 6. 2, 1 1 , 13 or 20, 29, 38 1 7. 13 167 1 8. Gs : �1 = 1 : 3 ; � : 82 1 = 5 : 49
1 9. 3, 7, 1 1, 15 . . . 20. � 16680 21 . x = 40
23. 164850 24. 16 rows, 15 bricks 25. 3 32. � 7250
33. 6 34. 10, preservation of environment and making it pollution free.
35. � 160000, � 1 40000, � 120000, � 100000, � 80000, � 60000, � 40000; helping students of the country and promoting educational
standards.
36. (i) � 4900 (ii) Honesty and sincerity
CHALLE N G E RS*
-----�!£::'":'!': T : ;��- ·-�_:-�_- ....
....�. -

1 . If the nth term of a progression is a l i near expression i n n, then show that it i s an AP.

2. If a , b and c are in AP, then prove that b + c , c + a and a + b are also in AP.

3 . A line graph x =2y + 4 is shown in figure.


y -4
-3
- 2 B
-1
X' x

Y'
Find the sum of all the integer values of x lying on the line from A and B by using AP.

4. Find the sum of the series


2 2 2 2 2 2 2 2
45 - 43 + 44 - 42 + 43 - 4 1 + 42 - 40 + . . . upto 30 terms.

5. Take a point A(3. 4) on the graph and d raw two l ines from it, one i s parallel to X-axis and another parallel to Y-axis . Aga i n , take
four points on both lines on both sides of A , such that their x-coordinates and y-coordinates form an AP with c o m m o n
difference 2 . Now, draw a circle passing through these four points and find its area.

[Hint Area of circle = 1t1 2 , where r is a radius of circle.]


-1- -- 1 ,
6. Find the sum of n terms of sequence
1
-- , , ... .
1 x 2 ·2 x 3 3 x 4
7. A circle with area A 1 is contained in the interior of a larger circle with area A 1 + A 2 • If the radius of the larger c i rc l e i s 3 a n d
A 1 , A 2 and A 1 + A 2 are in A P, then find the radi u s o f the smaller circle.

8. Su ppose b, , � . . . . , b24 are in AP, such that b, + b5 + b, 0 + b, 5 + b20 + b24 = 300. Find the sum of first 24 terms o f the AP.

9. Along a road lines , an odd number of stones placed at i ntervals of 1 O m . These stones have to be assembled a ro u n d t h e
middle stone. A person have t o b e assembled around the middle stone. A person can carry only o n e stone b y carryi n g t h e m i n
succession. I n carrying , a l l the stones b e covered a d istance o f 3 k m . Find the number o f stones.

* Th e s e q uestions m ay be or m ay not be asked in the exa mination, have been given just for additional practice.
Note Solutions to these questions a re availa ble at page number 384-386.
C APT E R 3
'
·.·.\ '; .,,3//�:tllf•·t'�
• 4.. L ��
• .... �
I, .· .
•• ' • , ,; . �-.
� - ,. ,...
' l• . - &, •
, ., '
( ; '·•. .
lP a
,"@iif.'
i:J!.' :.�'JO
i'::t.
rrr

C I RC LES

In earlier class, we studied circle and various terms related to it like chord, segment,
sector, arc, etc. In this chapter, we will discuss different situations that can arise when a
circle and a line are given in a plane.
' Tangent to a Circle
• Tangent Circle
• Length of Tangent A circle is a collection of all points in a plane which are at a constant distance from a
• Properties of a Tangent fixed point.
Number of Tangents from a This fixed point is called centre of the circle and constan·t distance
Point on a Circle
is called radius of the circle.
• Length of Tangents from an 1-------t1---1 A
External Point In the adjoining figure, 0 is the centre of circle and OA is the B
radius of the circle. Also, AB is the diameter of the circle.
N ote Two or more circles having the same centre are called concentric
circles.

Some I m portant Terms Related to C ircle


Chord
A line segment joining any two points on a circle is called a chord of
the circle. The maximum length of the chord is equal to the length
of the diameter.

Arc
R
A piece of a circle between two points is called an arc.
In adjoining figure, there are two pieces, one longer and the other
smaller. The. longer one is called the major arc (QfiP) and the smaller
---
one is called the minor arc (PMQ).

Semi-clrcle M
A diameter of a circle divides it into two equal parts, which are two equal arcs. Each of
these two ar cs is called a semi-circle.

Circumference
The length of the complete circle is called the circumference of the circle.
AllinOne MATH EMATICS Class 1 0th Term,j
/

Segment Important Results Related to Circle


The region between a chord and either (I) The perpend icular from che cen c re of a c i rcle c o a
chord bi seccs the chord.
(ii) The l i ne d rawn t h rough the cen c re o f a c i rc l e co bisect
of its arcs is called a segment of the
circular regio n or simply a segment of a chord is perpe ndicular co c h e c h o rd .
the circle. (iii) Equal chords of a c i rcle ar <; equ i d i s c a n c from the
centre.
(iv) The angle subcended by an arc (or
The segment formed by minor arc along C
with chord, is called minor segment and Minor segment correspond i ng chord) at the cen c re
the segment formed by major arc, is of che circle is cwice che a ngle
called the major segment. subtended by the same a rc a t a n y
poi nc on che rema i n i ng pare o f c h e
Sector (v)
circle.
Equal chords of a c i rcle subcend
The region between an arc and the two equal angles at the cen cre.
(vi) The angle i n a semi-circle i s a righc a ngle .
(vii)
radii, joining the ends of the arc to the
Angles in che same segmenc of a c i rcle a re equal.
(viii)
centre, is called a sector. The sum of any p a i r o f oppos i te angles o f a cycl ic
quad ri lateral i s t 80 ° .
(ix)
The sector formed by minor arc, is called
If two circles i n cersecc i n cwo po i n cs . t h e n c he l i ne
minor sector and the sector formed by ch rough the cen c res i s c h e perpe n d i c u l a r b i sector of
major arc, is called major sector. che common chord .

TO PI C O Tangent to a Ci rcle
In this topic, we will discuss the relation between a circle and Secant
a line which are given in a plane. A line which intersects a circle in
Consider, a line AB and a circle. Now, there can be three two distinct points is called secant
·

possible cases arised according to the position of line AB of the circle.


with respect to circle, which are given below. In this case, the line AB is said to
be secant of the circle.
(Case I ) When there is no common point between a circle
'

and a line AB. [case I I I ) When there is only one


Q) common point between a
� A circle and a line AB
C>
.� p
� In this case, the lin� AB is said
f!?
Q)
to be tangent to the circle at
:£ point P.
C:::
0
z
Tangent
In this case, line AB is said to be
the The word 'Tangent' has been
non-intersecting line with respect to a circle. derived from the Latin word
[Case II] When there are two common points P and Q 'Tangere' which means 'To
between a circle and a line AB. touch' and it was introduced
by the Danish Mathematician
Thomas Fincke in 1 5 83.
Thus, a tangent to a circle is a p Tangent line
Point of contact
line that intersects or touch the
circle at only one point. The common point o f the tangent
and the circle is called the p oint ofcontact.
Also, we can say that the tangent to a circle is a special case of
the secant, when the two end points o f its corresponding
chord are coincide.
I

Circles ! 97
Nole • There is only one c a n gem ac a poinl of the circle. Note l cs converse is also t ru e . i . e . a l i n e d rawn through che e n d
• A i rc l e have m a x i m u m c wo parallel cangen cs which can
can
poi n t of a rad i u s and perp e n d i c u l a r c o it. is a tangent to t h e
d ra w n 10 t ile opposicc s i d e of che centre. c i rcle.

L e ng t h of Ta n g e nt Example 1. A tangent PQ at a point P of a circle of


radius 5 cm meets a line through the centre 0 at a
Th I n r h o f r h e scgmcnr of rhc rangent, between the given
point Q such that PO = 1 2 cm. Find the length of 00.
poi n r ( n the ta nge n t ) a n d the point of contact, is called the
I n c h f ra n gc nt fr o m the given point. Sol. We know that, tangent at a point of a circle is perpendicular to
the radius through the point of contact.

---.--- a
Segment of the tangent between p 1 2 cm Q
tile g iven point B and point of contact A
Tangent line
Therefore, OP PQ.
is perpendicular to
In the figure, AB is called the length of tangent. Given, radius (OP) = 5 PQ = 12 cm
cm and
In right angled D.OPQ,
Properties of a Ta ngent OQ2 = OP2 + PQ2 [by Pythagoras theorem]
Th eorem
=> OQ2 =52 + 122
The ta ngent at a ny point of a circle is
perpendicul a r to the radius through the point of
=> OQ2 = 25 + 144
contact. CCE 2013, 12, u
=> OQ2 =169
Given A circle with centre 0 and a tangent AB at a point P
=> OQ =13 cm [taking (+ve) square root
length cannot be negative]
as

of the circle.
Exam ple 2. Find the radius of a circle, if the length of
To prove OP .l A B tangen t from a point a t distance of 25 cm from the
Co11structio11 Take a ny point Q , other than P on centre of the circle, is 24 cm.
the tangent AB a n d join OQ. Sol. Let AB = 24 cm and OB = 25 cm
We know that, the cangenc at a point of a circle is perpendicular
to the radius thro ugh the point of contact.
:. In D. OAB, OA 1- AB
Now, in right angled .1.0AB,
OB2 = OA2 + AB2 [by Pythagoras theorem]

A p Q B
Proof Here, Q is a point on the tan ge n t AB, other than the
point of con tact P. So, Q l ies o u tside the circle [if Q lies
inside the circle, then AB beco mes a secan t and not a
tangent to the circle] .
A 24 cm B
Let OQ in tersects the circle at R .
OP = OR [radii of the same circle] => (25)2 = OA2 + (24)2
Now, OQ = OR + RQ :::::>- OQ > OR
=> 625 = OA2 + 576
=> OQ > OP or OP < OQ [·: OP = OR]
=> OA2 =625 -576 = 49
Thus, OP is shorter than any other segment joining O to any
=> OA = 7 cm
Hence, the radius of the circle is 7 cm.
poi nt of AB. Also, we know that the shortest distance
between a point and a line is perpendicular distance from Exam p le 3. If PQ is a tangent to a circle with cen tre
the point to the line. So, OP is perpendicular to AB. 0 and radius 6 cm such that L.PQO = 6 0 °, then find the
1 .e. OP .l AB Hence proved. length of a tangent PQ and a line OQ.
98 Allinone MATH EMATICS Class 1 0th Te"'}
'J
c
Sol. Given, PQ is a tangent, OP = 6 cm and LPQO = 60°,
We know that, tangent at a point of a circle is perpendicular
to the radius through the point of contact.

OB .L AB
=> LOBA = 90°
���....__..:::;_�����-'-�.;::,,,.. o

[·: ran 0 �]
p Given, LA OC = 1 20°
.. OP 1- PQ In ll.ABO, LAOC = L OAB + L OBA
Now, in right angled ll. OPQ, [·: external angle = sum of opposite internal �
OP => 1 20° = LOAR + 90°
tan 60° = =
PQ => LOAB = 1 20° - 90°
LOAB = 30°
=> .
=>
v�3=
:> __£ � PQ = .J3
6 x .J3 [on rationalising]
PQ .J3
=>
PQ = 2.J3 cm TRY YOURSELF
OP
60° =
Q. l Find the length o f tangent to a circle from a point at ,
and sin
OQ
=> .J3 2 x 6 x .J3 [on rationalising] distance of 5 cm from centre of the circle of racf�
= � => OQ = .J3 3 cm. [Ans. 4 ·

2 OQ .J3
=> Q.2 If the length of a tangent to a circle from a point ,..
0.5 cm and distance of a point from its centre is 1 .5
OQ = 4.J3 cm
Hence, length of a tangent PQ is 2.J3 cm and a line OQ then find its radius. (Ans. Ji a:i
is 4..J3 cm.
Q.3 If AB is a tangent drawn from a point B to a circle w
Exam p le 4 . lfAB is a tangent drawn from a point centre C and radius 1 . 5 cm such that LCBA = 30°, t�
A to a circle with centre 0 and BOC is a diameter of
find the length of a tangent AB and a line CB. .
the circle such that LAOC = 120°, then find LOAB.
Sol Given, AB and BOC are the tangent and diameter of the
[ Ans. 3 2 3
[
cm and 3 an
.

circle with centre 0 , respectively. We know that, tangent at Q. 4 PQ is a tangent drawn from a point P to a circle Vt'.
centre 0 and QOR is a diameter of the circle such th..�
[Ans. 4 ·�
a point on a circle is perpendicular to the radius through the
point of contact. LPOR = 1 35°, then find LOPB.

NCERT FO L D E R 3 .1
1 How many tangents can a circle have? Sol. (tj one
Sol (itj secant
J
From every point of a circle, we can draw a tangent.
Therefore, infinite tangents can be drawn. (iitj two
j point of contact
2
(iv)
J Fill in the blanks.
(i) A tangent to a circle intersects it in . . . point(s) . 3 A tangent PQ at a point P of a circle of radiu�
' 5 cm meets a line through the c entre 0 at
(ii) A line intersecting a circle in two points is called a·

a ... point Q, so that OQ = 1 2 cm . Length of PO is



- (iii) A circle can have . . . parallel tangents at the (a) 12 cm
(b) 13 cm

I
most.
(iv) The common point of a tangent to a circle and

(c) 8.5 cm
the circle is called . . . . (d) .Jug cm
Circles I 99
Sol. ( d) Herc, PQ is a r:rngcnr, which couches rhe circle ar We know char, the tangent ac a point of a circle is
po i n r /' and OP =5
cm is rh e radius of circle. Join perpendicular co che radius through the point of contact.
2
OQ =1 cm . We know rhat, rangenr ar a poinr of a .. OQ ..L PQ
circle is perpendicular ro rhc rad i us rh rough r he po i n r [·: PQ is a tangent to a circle with centre OJ

cosLPOQ = �� [-: cos0 !J


of conracr. Now, LPOQ == 180° -
LPOR
180° - 120° = 60° [linear pair]

Inright angled llPQO,


=

=> os 60°=
c
4
-

p Q PO
[·: LPOQ = 60° and OQ 4 cm] =
So, OP J_ PQ
Now, i n righ c angled !lOPQ, =:> -2 = PO4 PO =B ern
- =>

OQ 2 = OP 2 + PQ 2 [by Pychagoras ch eo rcm ]


PQ
=:> (12)2 = (5 ) 2 + PQ 2
and . LPOQ = OQ
tan

=:>
PQ
PQ 2 = 144 - 25 =119 => can 60° =
=:> = J1l9 cm
4
PQ
=>
.J3 = PQ =>
4 Draw a circle and two lines parallel to a given
4 PQ = 4.J3 cm
line such that one is a tangent and the other a Hence, che required values of OP and PQ are 8 cm and 4 .fj
secant to the circle . cm, respectively.
Sol. Firsc, draw a circle wich cen rre 0 and draw a l i ne /. Now, we
draw two l i nes paral lel co /, such char one l ine say m, i s a
6 If O is the centre of a circle, PQ is a chord and
cangenc co che circle and anochcr say 11, is a secan t to che
the tangent PR at P makes an angle o f 5 0 ° with
circle. PQ, then find LPO Q.
Sol. G ive n , PQ is a chord and PR is a tangent co a circle at P.
m Also, LRPQ = 50° . . (i).

Additional Questions

5 If PO is a tangent drawn from an external point We know that, tangent at a point of a circle is perpendicular
P to a circle with centre 0 and 00 R is a diameter co che radius through the point of contact.
where length of QOR is 8 cm such that .. OP ..L PR => LOPR = 90° . . . (ii)
LPOR = 120°,
then find OP and PQ.
=
LOPQ LOPR - LRPQ
Sol. Given, LPOR = 120°, QOR is a dia m ecer of circle with
LOPQ = 90° -50° [from Eqs. (i) and (ii)]
0::
cencre 0.
=40°
LL
R In llPOQ ,

OP = O Q [radii ofa circle] c


=:> LOQP = LOPQ -

= 40° c
[·: g es opposite to equal sides of triangle are equal]
an l Ll.
By a n gl e sum property of D.POQ, we gee
LOQP + LOPQ + L.POQ =180° �
8 40° 40° =180° Q:
= = -2 = 4
OR OQ cm + + L.POQ LL
LPOQ ::::180°-80°=100° �
TO P I C EXERCISE
---

Very Short Answer Type Questions


1 How many tangents you can draw to a circle which
a re parallel to a secant? CCE 201 3

2 I f a l i ne which is perpendicular t o the radius of the


circle through the point of contact. What will you
call that line?
T
3 W hat will you call a line, if it intersect a circle at two
7 In the given figu re. PA is a ta n g e n t f ro m .J n ex terna
point P to a circle with centre 0. I f L POB = 1 1 5 °. ther
distinct points?

Short Answer Type I Questions find LAPO.


0
4 In the given figure, find the value of x •

0 + y 0•
5 From the given figure. find the value of x Long Answer Type Questio n
p 8 In the given figure, FG is a ta ngen t t o t h e c i rcle with
centre A. If LDCB = 15 ° and CE = DE. t h e n find
LGCE and LBCE. CCE 2015
F G
R

Short Answer Type I I Questions


6 In the given figure, AT is a tangent to the circle with
D
centre 0 such that OT = 4 cm and LOTA = 30°. Find
[Hint LCED = 90°, as a ngle i n sem i - c i rc le.]
the length of AT.

Answers
1 . Two 2. Tangent 3. Secant 4. 1 20° 5. 270°
6. 2f3 cm 7. 25° 8. LGCE = 4 5° and LBCE = 30°
TO PI C 0 N u m ber of Tangents from a Point on a Circle
The number of tangents drawn from a point on a circle This result is proved in the form of followin g theorem.
depends upon the position of the point with respect co
the circle. So, clucc cases are possible. (Tu�o��-��I The lengths of two tangents drawn from
an external point to a circle are equal.
(Case I ] When point 'P' lies outside the circle.
Let AP and AQ are two tangents from a point A to a c i rcle w i th
If point P lies
centre 0.
outside the circle,
then two tangents
can be drawn co
the ci rcle from P
point P.
Hence, there are
exactly two tangents
(PT1 and PT2 ) To prove AP = A Q
from a poi nt lyi ng outside the circle.
Construction Join OP, O Q and OA.
(Case 11) When point 'P ' lies on the circle.
Proof We know that, a tangent at any point of a c i rcle i s
If poi nt P lies on perpendicular to the radi us through the p o i n t o f con tact.
the circle, then
Here, AP is a tangent and OP is the radi us of the c i rcle
there is one and
through P.
only one line
which touch the .. OP 1- AP
circle m one Similarly, OQ 1- A Q => L OPA = L OQA = 90 ° . . (i) .

point, all ocher


First Method
lines meet the circle in more than one point.
H ence, there is one and only one tangent to the In !lOPA and !lOQA, we have
circle passing through a point P lying on the OP = OQ [radii o f a c i rcle]
circle and this point P will be it point of LOPA = LOQA = 90 ° [ from Eq . (i)]
contact.
OA = OA [common sides]
(Case III) When point 'P , lies inside the circle. So, !lOPA � !l OQA [by RHS congruence rule]
If a point P l ies i nside AP = A Q [by CPCT]
the circle, then any line Second Method
passi ng through the No
In right angled !l OPA,
point P will i ntersect tangent
the circle at two points. OA 2 = 0P 2 + AP 2 [by Pythagoras t heorem]
2
Hence, there is no =:> AP 2 = OA 2 - OP 2 => AP 2 = OA 2 - OQ
tangent to a circle [·: OP = OQ = radii of a c ircle] . . . (ii)
passing through a point lying inside the circle.
Now, in right angled !l OQA,
Le n gth of Tangents Drawn from an OA 2 = 0Q 2 + AQ 2 => AQ 2 = 0A 2 - OQ 2 . . . ( ii i)
External Point From Eqs. (ii) and (iii) , we get
We know that, the length of a tangent is the length of
=:> AP 2 = AQ 2 => AP = AQ Hence proved.
the segment of the tangent between the external point
'P' (say) and the point of contact of tangent with the Note
L.OAP L.OA Q [ · : �OPA :: �OQA.]
From the above t heore m .
circle. =

� OA is the angle bisector o f L.PA Q.


If two tangents are drawn from an external point, then Thus. the cen tre of the circle l i e s on t h e b i sector of t h e a n g l e
they are equal in length. between the t w o tange n t s .
1 02 Al11�none MATHEMATICS Class 1 0th Term I

Important Results Related to Tangent to a Circle Exam p le 2. In the given figure, if A B = AC, th en
(i) lf two ci rcles touch i n ternally or externally . then p o i n t prove that BC = 2CE.
A
of contact l ies on t h e straight l i ne through the two
cen t res.

(ii) A pair of tangents drawn at two points of a c i rcle a re

Sol.
e i ther parallel or they i n tersect each other at a poi n t
outside t h e circle. Given A MBC in which A B == AC.
(iii) I f two tangents drawn to a ci rcle are parallel to each To prove BC = 2CE
other. then the line segment joining the i r poin t of
Proof We know char, cangcnrs from an external point co a
con tact is a diameter of the circle.
(iv) If two tangents are drawn to a ci rcle from a n external
circle are equal in lengch.
.. AD == AF [·: A is an external point] ... (i
[·: B is an external point] . . (ii
point. then
BD == BE .

and CE == CF [·: C is an external point] . . (iii.

Now, AB = A C [given
=> AB - AD = A C - AD [subcracci ng A D from boch sides)
=> AB - AD = A C - AF [using Eq. (i))
=> BD = CF => BE = CF [using Eq. (ii))
=> BE = CE [using Eq. ( iii)] . .. (h·
(a) they subte nd equal angles at the centre.
Now, BC = BE + EC = CE + CE [ fro m Eq. (iv))
(b) they a re equally incli ned to the segment, joi n i ng the BC = 2CE Hence proved.
centre to that poi n t .
(v) The opposite si des of a quad ri lateral circumscrib i ng a Examp le 3 . 0 is the centre of a circle of radi u s 5 cm.
circle subtend supplemen tary angles at the centre o f T is a point such that OT = 1 3 cm and OT in tersects the
0
circle at E. If AB is the tangen t to the circle at E, then
t h e circle.

find the length of AB. NCERT Exemplar


Sol. Given, radius of circle = 5 cm and O T = 1 3 cm .
We know chat, tangent at any point of a circle is perpendicular
to the radius through the point of contact.

Example 1. In the given figure, XP and XO are


tang en ts from X to the circle with cen tre 0. R is a poin t
on the circle. Prove tha t XA + AR = XB + BR .
Sol. Given, XP and XQ are
tangents of che circle wich So, OP _L_ PT => L. OPT = 90°
centre 0 and R is a point on
In right angled t10PT,
che circle.
We know char, lengths of x OT 2 = OP 2 + PT 2 [by Pythagoras theorem)
cangencs from an external 1 3 = 5 + PT 2 => PT 2 = 1 69 - 25 = 1 44
2 2
point co a ci rcle are equal. PT = 1 2 cm
.. XP = XQ AJso, we know chat length of tangents d rawn from a point co a
[·: Xis an external point] . . . (i) circle are equal. Therefore,
AP = AR [·: A is an external point] ... (ii) AP = AE = x [say)
and BQ = BR [·: B is an external point] ... (iii) Now, A T = PT - AP = ( 1 2 - x ) cm
Now, from Eq . (i) , Since, AB is the tangent to the circle at point E and OE i�
XP = XQ => XA + AP == XB + BQ radius.
X4 + AR = XB + BR [using Eqs. (ii) and (iii)] Therefore, OE _L_ AB => L.OEA = 90 ° => LAET = 90°
Hence proved. [·: OT is a straight line, so L. OEA + LAET = 1 80"!
Circles 103
MET,
Now, in righc angled We know char, tangent ar any point of a circle is
T2 A T
= AE2 + E 2
[by Pythagoras cheorem] perpendicular co rhe radius through rhe point of contact.
(12-x)2 = x2 5 f [·: ET =OT-OE= 13- 5]
:::::> + (1 3 -
So, .l AB
OP
144 - 24x x2 x 2 64 24x = -80
:::::> + = + =:> - Now, AB is a chord of circle C, and .l AB, so
OP OP
is rhe
bisector of rhe chord AB because chc perpendicular from che
:::::> x 10
= - cm centre bisect rhe chord.
A =B
P P
3
10 A E = - cm
So, Hence proved.

Similarly, BE IO
= - cm
TRY Y6URSELF
3
10 20 IO Q. 1 If the radius of a circle is 5 c m , then find the d istance
Hence, AB = AE + BE = - + - = - cm
3 3 3 between two parallel tangents . [Ans. I 0 cm]

E xample 4. Prove th at in two concentric circles,


Q. 2 If AB and BC are two tangents to a circle with centre 0
such that LAOC = 1 20°, then find LABC. [Ans. 60° )
the chord of the larger circle, which touches the
smaller circle, is bisected at the point of contact. Q. 3 In the given figure. 0 is the centre of two concentric
CBSE 2009 circles of radii 4 cm and 5 cm, respectively. PO i s t h e
chord of outer circle wh ich touches t h e i n n e r circle .
Sol. Lee C, and C be cwo given
concentric
2
circles
with common
Find the length of chord PO. [Ans. 6 cm]

centre 0 in which circle C1 is larger


and let AB be a chord of che larger
circle C1 touching che smaller circle
C2 atP.
To prove
AP = BP. Chord AB is bisected ac the point of contact, i.e.

Construction Join OP. Q. 4 In a right angled MBC, right angled at B, BC = 1 5 c m


Proof Since, chord AB couches che circle C2 at one point P, and AB = 8 c m . A circle i s inscribed in MBC. Find the
[Ans. 3 cm]
so it is a tangent co C2 and OP is radius. radius of a circle .

. FO D E R 3 .2
Directions (Q. Nos. I to 3) Choose the correct option and give justification.
1 From a point Q, the length of the tangent to a
circle is 2 4 cm and the distance of O from the
centre is 2 5 cm. The radius of the circle is
(a) 7 cm (b) 1 2 cm (c) 15 cm (d) 24.5 cm
Sol. (a) Given, rhe length of the tangent
QO = 25 cm . QP = 24 cm and
Join OP. :. In right angled !lOPQ,
We know char, radius OP is perpendicular co the
tangent PQ ar rhe point of contact, i.e.
OQ2 = OP2 PQ2
+ [by Pythagoras theorem]

OP..L PQ :::::> 252 OP2 242 OP2 625 - 576


= + =:> =

:::::>LOPQ =90° :::::> OP2 = 49 OP = 7


=:> cm
1 04 \ Allin.One MATHEMATICS Class 1 0th Temi'i

2 In the given figure , if TP and TQ are the two Join QA and OB,
tangents to a circle with centre 0, so that
Clearly, OA .l. PA and OB .l. PB
L POQ = 1 1 0°, then LPTQ is equal to [·: radius of a circle is perpendicular to d.r
rangenc ac che point o f conta"i
T
:. LOAP = 90° and L OBP = 9 0 °
Now, in quadrilaceral OAPB,
LOAP + LAPB + L OB/> + LA OB = 360 °
[·: sum of all i n cerior angles = 360'.
=> 90 °+ 80° + 90 °+ LA OB = 360 °
=> LAOJJ = I 0 0 °
Since, cangencs drawn c o a circle from an extcrnl
(a) 60 ° (b) 70 ° (c) 80 ° (d) 90 ° point subcend equal angles ar rhe centre.
Sol. (b) TP and TQ are cwo tangents to a circle and .. LPOA = LPOB
CQ
Given,
LPOQ = 1 1 0°. Join PQ. LAOB= LPOA + L POB . • •

T LAOB= LPOA + L J>OA [ fro m Eq. far


=> LPOA = _!_ LA OB = _!_ x I 00 ° = 5 0 °
2 2
4 Prove that the tangents d rawn at the e nds of t
diameter of a circle are parallel.
Sol. Let AB be a diameter of a given circle and lM and PQ br
the tangent lines drawn to the circle at poi nts A and A
In llOPQ, OP = OQ [radii of circle]
respectively.
=> LOPQ = LOQP To prove LM ll PQ
[angles corresponding to equal sides are equal]
Proof We know thac, che B
can gene at any point of a circle L ----=--�o::::--- ti
1 800 - l l 0 °
Then, LOPQ = LOQP = = 35 °
2
Since, OP .l. PT
is perpendicular to che radius
chrough che point of contacc.
[·: radius of a circle is perpendicular to the .. OA .l. PQ
tangent at the point of contact] and OB .l. LM
LOPT = 90°
=> AB .l. PQ
=>
p -_ --

AB .l. LM
LOPQ + LQPT = 90° ___;:__.,,
: A� -Q
and
=> 35 ° + LQPT = 90° => L QPT = 55°
=> LPAB = 90°
Sim ilarly, LPQT = 5 5 ° and L ABM = 90°
Now, in APTQ,
=> LPAB = LABM [alternate angles)
LQPT + LPQT + LPTQ = 1 80° => PQ l l LM
[·: sum of all angles in a triangle is 1 80°] Hence, che tangents drawn ac che ends of a diamecer of a
=> 55° + 55° + LPTQ = 1 80° circle are parallel. Hence proved.
=> LPTQ = 1 80° - (5 5 ° + 5 5 ° ) = 70° 5 Prove that the perpendicular at the p oint of
3 If tangents PA and PB from a point P to a circle contact to the tangent t<? a circle passes through
with centre 0 are inclined to each other at angle the centre.
of 80° , then L POA is equal to Sol. Let AB be the tangent drawn at a point C on the circle with
(a) 50° (b) 60° (c) 70° (d) 80° centre 0.
To prove Perpendicular at point C passes through tht
Sol. ( a ) Given, PA and PB are two tangents drawn from a centre 0.
poin t P, such that LAPB = 80°.
80° If possible, let the perpendicular passing through some other
p point, say O'.
Construction Join OC and O' C.
Proof Since, tangent a t any poin t of a circle as : ,

perpendicular co the radius through the point of contact.


.. OC .l. AB => LOCB = 9 0 °
Also, LO' CB = 90 ° [as it is supposed that CO' J_ AB ·
I'
Circles

LOCB = LO' CB On adding Eqs. (i) , (ii) , (iii) and (iv) , we gee
which is possible only when points 0 ( AP + BP) + (CR + DR) = ( AS + BQ) + ( CQ + DS)
and O' coincide. => AB + CD = ( AS + DS ) + ( BQ + CQ)
So, our supposition is wrong. => AB + CD = AD + BC Hence proved .

Hence, the perpendicular at the point 9 In the given figure , XY and X' Y' a re two
of contact co the tangent co a circle A C B
always passes ch rough the centre. Hence proved. parallel tangents to a circle with centre 0 a n d
another tangent A B with p oint o f contact C
6 The length of a tangent from a point A at a intersecting XY at A and X' Y' at B. Prove that
distance 5 cm from the centre of the circle is LAOB = 90°. CCE 20 13, 1 2, 1 1
4 cm. Find the radius of the circle.
x p A y
Sol. Do same as Q. 1 . [Ans. 3]
7 Two concentric circles are of radii 5 cm and
3 cm . Find the length of the chord of the larger c
circle which touches the smaller circle.
Sol. Let C1 and C2 be two ci rcles of radii, Y'
1j = 3 cm and r2 = S cm and having Q B
common centre 0. Sol. Given XY and X' Y' are two parallel tangents. Another
Now, Ice AB be the chord of circle C2 tangent AB couches the circle at C and intersect XY at A
such chat it touches the circle cl at and X' Y' at B.
To prove LAOB = 90 °
point D.
Clearly, AB be the tangent co the
circle C1 at point D. Proof We know chat, tangents drawn from an external
.. OD J_ AB [·: radius is perpendicular co the tangent point co a circle are equal in length.
at the point of contact] .. AP = A C [·: A i s a n external point] ... (i)
=> AD = BD [·: perpendicular from centre co Thus, in /!iAPO and M CO, AP = A C [from Eq. (i)]
the chord bisect the chord] AO = AO [common sides]
Now, in right angled l!! ODB, OP = DC [radii of circle]
OB 2 = OD 2 + DB 2 [by Pythagoras theorem] MPO =: M CO [by SSS congruence rule]
=>
=> 2 2
5 = 3 + DB 2 DB 2 = 2 5 - 9 = 1 6 Then, L OAP = L OA C [by CPcn . . . (ii)
LPA C = LOAP + L OA C
=> DB = 4 cm
:. Length of chord = AB = 2 AD = 2 x 4 = 8 cm LPA C = LOAP + L OA C
=> L PA C = 2 L CAO [from Eq. (ii)]
8 A quadrilateral ABCD is drawn to circumscribe [·: L OA C = L CA O]
a circle (see figure) . Prove that Similarly, we can prove chat L CBO = L OBQ
AB + CD = AD + BC. CCE 20 16 => L CBQ = 2 L CBO . . . (iv)
XY II X' Y'
R C Since, [ given]
D .. L PA C + L QBC = 1 80 °
[·: sum of interior angles on the same
side of transversal is 1 80°]
=> 2 L CA O + 2 L CBO = 1 80° [from Eqs. (iii) and (iv)]
=> LCAO + LCBO = 90 ° [dividing both sides by 2] . . . (v)
B Now, in MOB, L CAO + L CBO + L A OB = 1 80 °
A [·: sum o f all angles o f a triangle i s 1 80°]
Sol. Given A quadrilateral ABCD, circumscribing a circle. => LCAO + L CBO = 1 80 ° -
LAOB . . . (vi)
To prove AB + CD = AD + BC From Eqs. (v) and (vi) , we get
Proof We know chat, the lengths of tangents drawn from an 1 80 ° L AOB = 90°
-

external point co a circle are equal. => L AOB = 90 ° Hence proved.


.. AP = AS
[·: both are can genes to a circle from point A] . . . (i) 1 0 Prove that the angle between the two tangents
Similarly, drawn from an external point to a circle is
BP = BQ, . . . (ii) supplementary to the angle s ubtended by the
CR = CQ . . . (iii) line segment joining the points of contact at the
and DR = DS . . . (iv) centre .
1 1 0& 1· All/nOne MATHEMATICS Class 1 0th Term I : I

Sol Let PQ and PR be two tangents drawn from an external [·: opposice sides of a pa rallelogram arc equall ,

point P to a circle with centre 0 . Then, from Eq. (i), we ge e


O �- 2 AB = 2 B C => A B = BC . . . (iii
From Eqs. (ii) and (iii), we gee
AB = BC = CD = DA
=> ABCD is a rhombus.
Hence, che parallelogram circumscribing a c ircl e is a
R rhombus. Hence pro\-ed.
To prove L QOR = 1 80° - LQPR 1 2 A MBC is drawn to A
or L QOR + LQPR = 1 80° circumscribe a circle
Proof In ll.OQP and ll.ORP, PQ = PR of radius 4 cm such
[·: tangents drawn from an external that the segments BD
point are equal in length] and DC into which BC
OQ = OR [radii of circle] is divided by the point
OP = OP [common sides] of contact D are of

}
.. ll.OQP ::: ll.ORP [by SSS congruence rule] lengths 8 cm and
[by CPCT] 6 cm, respectively (see
6 cm
Then, L QPO = L RPO
and L POQ = L POR [by CPCT] figure) . Find the sides C
LQPR = 2 L OPQ AB and AC.
Sol.
=> . . . (i) Given, CD = 6 cm , BD = B cm a nd rad i us = 4 cm
and LQOR = 2 L POQ
Join OC, OA and OB.
Lee che circle couches che ocher sides AB an<l A C ac poincs E
Now, in right angled ll.OQP, L QPO + LQOP = 90°
=> L QOP = 90° - L QPO and F respectively.
=> 2 LQOP = 1 80° - 2 LQPO We know chat, cangencs drawn from an e x t e rn a l poinc co tilt
[multiplying both sid�s by 2] circle are equal in length.
=> L QOR = 1 80° - L QPR [from Eq. (i)] :. CD = CF = 6 cm [·: C is an ex te rn a l point)
=> LQOR + L QPR = 1 80° Hence proved.
BD = BE = 8 cm [·: B is an exccrr:ial point]
11 Prove that the parallelogram circumscribing a and AF = AE = x cm (s ay) [·: A is a n excernal point]
circle is a rhombus . CBSE 20 1 2 Area of AOCB,A1 = _!_ x Base x Heighr = _!__ x CB x OD
2
2
Sol Le t ABCD b e a parallelogram circumscribing a circle.
1 2
To prove ABCD is a rhombus. = 2 x l 4 x 4 = 28 cm [· :CB = CD + BD = 6 + 8 = 14]
i.e. to prove AB = BC = CD = DA A
D 0 C Area of �OCA,
A2 = -I x AC x OF
2
p N I
= - (6 + x) x 4
2
= (1 2 + 2x) cm 2
A M B and area of �OBA ,

Proof We know that, the tangents co a circle from an A 3 = 1 x AB x OE


-

external point are equal in length. 2


B
B ern
= - (8 + x) x 4
. . AM = AP, BM = BN, CO = CN and DO = DP I
O n adding all above equations, we get 2
= (1 6 + 2x) cm 2

[ ]
( AM + BM ) + (CO + DO) = AP + BN + CN + DP
==> AB + CD = (AP + PD) + (BN + NC ) Thus, area of llA.BC = A , + A 2 + A 3
= AD + BC . . . (i) = 2 8 + (1 2 + 2x) + {1 6 + 2x)

·: AB = AM + MB, BC = BN + NC, = 5 6 + 4x . . . (i)


CD = CO + OD, AD = AP + PD Now, semi-perimeter of MBC = _!_ (AB + BC + CA )
2
Given, ABCD is a parallelogram. 1
. .

AB = CD and BC = AD . . . (ii) => s = - (x + 8 + 1 4 + 6 + x ) => s = 14 + x
2
Circles f 101 f
�- -
Using Heron's formula, and L A OD + L BOC = 1 80 °
Arca of M BC = .Js (s-- ( s---b-)-
- a) - - c)
(s-- Construction Join OE, OF, OG c
and OH.
(1 4 + x ) ( 1 4 + x - 1 4 )(1 4 + x - x - 6)
=
Proof We know that, two
(1 4 + x - x - 8 ) eangents drawn from an external
point to a circle subeend equal
= �(1 4 + x) x x x 8 x 6 = �(1 4 + x) 48x . . . (ii) angles at the centre.
From Eqs. (i) and (ii), w e gee
�(1 4 + x ) 4 8x = 56 + 4x
On squaring boch si<les,

2x = 1 4
we gee
(1 4 + x ) 4 B x = 4 2 ( 1 4 + x)2
=


4 (1 4 + x )

3x = 1 4 + x
x=7
and

�� :�!1
L5 = L6
L7 = LB
. . . (i )

Also, we know that ehc sum of all angles subtended at a point is


Lcngeh of A C = 6 + x = 6 + 7 = 1 3 cm 360°.
and lengeh of AB = B + x = 8 + 7 = 1 5 cm :. LI + L2 + L3 + L 4 + LS + L6 + L 7 + LB = 360°
1 3 Prove that opposite sides of a quadrilateral � 2 ( L 2 + L3 + L6 + L 7) = 360 °
circumscribing a circle subtend supplementary � ( L 2 + L3 ) + ( L6 + L7) = 1 80 °
angles at the centre of the circle. � LA OB + L COD = 1 80°
Similarly, we have
Sol. Let ABCD is a quadrilaeeral circumscribing a circle with centre
0. Let circle touches the sides of a quadrilateral at points 2 ( Ll + LB + L 4 + LS ) = 360° [from Eq. (i)]
� ( LI + LB ) + ( L4 + LS ) = 1 80 °
Hence proved.
£, F, G and H.
To prove L A OB + L COD = l B0° � LAOD + LBOC = 1 80°

TO P I C EXE RCISE
Very Short Answer Type Question 3 In the given figure, PP' and QQ ' are the two common
1 In the given figure, PT and PM are two tangents to tangents of the two circles. Show t hat PP' = QQ '.
the circle with centre 0. If OT = 6 cm and OP = 10 p
cm, then find the length of PT and PM.
T

Short Answer Type II Questions


01..--,,.......
... �.. --..,
4 In the adjoining figure, a circle
c
Short Answer Type I Questions
touches all the four sides of a
2 PA and PB a re tangents from P to the circle with quadrilateral ABCD whose three
centre 0. At point M, a tangent is drawn cutting PA sides are AB = 6 cm, BC = 7 cm
at K a nd PB at N. Prove that KN = AK + BN. and CD = 4 cm. Find the length
A L-..__.:::=-....::;____. 8
A of AD.

5 In two concentric circles. a chord of length 24 c m of


p larger circle becomes a tangent to the s m aller circle
whose radius is 5 cm. Find the radius of the larger
circle.
\ 10s \ AllinOne MATH EMATICS Class 1 0th Term

6 In the given figure, PA is a tangent from an external R and S. respectively. If BC = 38 c m . CD = 25 cm a nd


point P to a circle with centre 0. If LPOB = 115°, then BP = 27 cm. then find the va l u e of r .
find LAPO. 9 From an external point P. t a n ge n t s PA ,rn cl PB a re
drawn to a circle with cen t re 0. I f CD i s t h e t a n ge nt
to the circle at a point E a n d PA = 14 cnL t he n f i n d
the perimeter of �PCD.
A
p
p
Long Answer Type Questions
7 I n the given figure, 0 is the centre of two concentric
circles of radii 4 cm and 6 cm, respectively. PA and
PB are tangents to the outer and inner circles,
respectively. I f PA = 10 cm, then find the length of 1 0 In the given figure.PA and PB a rc t �1 1 1gc n t s to t he
PB upto one place of decimal. given circle such that PA = 5 c m ct n cl LA PB 60 °. =

Find the length of chord AB .

p
p

8 ABCD is a quadrilateral such that LD = 90°. A circle


C (O. r) touches the sides AB. BC. CD and DA at P. Q,

Answers
1 . 8 cm, 8 cm 4. 3 cm 5. r = 1 3 cm 6. 25° 7. 10.9 cm 8. r = 1 4 cm 9. 28 cm 1 0. 5 cm
I Very Short Answer Type Questions [ 1 Mark each]

1. How many tangents can be drawn to a circle 12. In the given figure, AB, AC and PQ are
from a point P lies outside the circle? tangents . If AB = 5 cm, then find the
perimeter of MPQ. CCE 2009
2. At which point a tangent is perpendicular to A
the radius? CCE 201 2

3 . What term will you use for a line which


intersect a circle at two distinct points?
CCE 2012

4 . Write the name of the common point of the


tangent to a circle and the circle. CCE 2012 13. In the given figure, all three sides of
triangle touch the circle. Find the value of x.
5. What do you say about the line which is CCE 2009
perpendicular to the radius of the circle
A
through the point of contact? CCE 201 1

6. Write the number of tangents to a circle


which are parallel to a secant. CCE 2014, 1 1

7. Find the distance between two parallel


tangents of a circle of radius 3 cm.
CCE 201 1, 13
1 4. In the following figure, PQ is a tangent at a
8. If two tangents inclined at an angle of 60° point C to circle with centre O. If AB is a
are drawn to a circle of radius 3 cm, then diameter and LCAB = 3 0 °, then find LPCA.
find the length of each tangent. c
NCERT Exemplar; CCE 2012, 1 1

9. Two concentric circles of radii a and b (a > b)


are given. Find the length of the chord of the
larger circle which touches the smaller CBSE 2016
CBSE 2015 u
15.
circle.
Two concentric u
u
10 . circles with
centre 0 are of
Two concentric circles are of radii 1 0 cm and
6 cm. Find the length of the chord of the
<
radii 5 cm and p --==� -- --+---+ .....
-

longer circle which touches the smaller


3 cm. From an
circle.
external point LI
...
1 1. P, two tangents -
In the adjoining -
PA and PB are drawn to these circles,
figure, if PA and
respectively. If PA = 1 2 cm, then find length ...
PB are tangents to p c:
of PB. CCE 201 5, 1 0 -=
the circle with Ill:
centre 0 such that 1 6 . !HOTSJ Show that the length of tangent from -=
11111
LAPB = 50°, then B an external point P to a circle with centre 0 -

NCERT Exemplar m
find LOAB. is always less than OP.
' 110\ Allinone MATHEMATICS Class 1 0th Term1
� \ Short Answer Type I Questi ons [ 2 M a r k s e ach

1 7. I n the adjoining 23. The tangents drawn at the e n d points of two


figure, 0 is the perpendicular diameters o f a circle are
centre of the parallel to each other, which form of a
circle, PT is the a p square and whose length o f s id e is 2 cm.
tangent and PAB Find the radius of circle .
is the secant
pass�s through 24. PA is a tangent B

the centre 0. If PT = 8 cm and PA = 4 cm, to the circle with


then find the length of the radiu s . centre 0. If
BC = 3 cm,
A AC = 4 cm and
1 8. In the adjoining figure,
MCB - L\PAO,
an isosceles MBC with
then find
AB = AC, circumscribes
OP
a circle. Prove that point OA and .
AP CCE 2013
of contact P bisects the
CCE 2008, 15 B '--____:�....:_
::::;_ _l. c
In the given figure, common tangents AF
base BC.
p 25.
and CD to two circles inters ect at E. Prow
19. In two concentric circles ,
prove that a ll chords o f the . that AB = CD. NCERT Exempl ar, C BSE 2014
outer circle which touch
the inner circle are of equal
length. CCE 201 5

20. In the adjoining A


figure, AD = 8 cm,
AC = 6 cm and TB 26. In two concentric circles , a c hord of thr
is the tangent at B larger circle touches the smaller circle. Ii
to the circle with the length of this chord is 8 cm and the
centre 0. Find OT, diameter of the smaller circle is 6 cm, then
if BT is T find the diameter of the larger circle .
4 cm . CCE 20 13 CCE 2009

2 1. I f PA and PB are two tangents drawn from a 2 7. In the adjoining


point P to a circle with centre O touching it figure, from an
at A and B, prove that OP is perpendicular external point P,
bisector of AB. CCE 2008 two tangents PT
and PS are drawn to f
22. I n the given figure, two circles touch each a circle with centre
other externally at C, prove that common 0 and radius r. If
tangent at C bisects the other two common OP = 2r, then show
tangents . that LOTS = LOST
= 30° CBSE 20 1 6

28. In two concentric circles , a chord o f le ngth


24 cm of larger circle becomes a tangent t,,
the smaller circle whose radius is 5 cm. Find
G F H the radius of larger circle. C CE 201 5
Circles 111

29 . [tt_ois� If a number of circles touch a given line segment PO at a point A , then prove that their centres
_

lie on the perpendicular of PO but do not lie on the bisector or PO .

3 0 . [ttgf_� I f a, b, c are the sides of a right angled triangle, where c is hypotenuse, then prove that the
radius r o f the circle which touches the sides of the triangle is given by NCERT Exemplar; CCE 20 1 2

r = ----
a + b - c

�I
2

Short A ns wer Type II Questi ons [ 3 Marks ea c h ]

31. PC i s a tangent to the circle at C . AOB is the 35. In the given figure, 0 is the centre of the
diameter which when extended meets the circle. Determine LAOB and LAMB, if PA
tangent at P. Find LCBA, LAOC and LBCO, and PB are tangents and LAPB = 7 5 °.
if LPCA = 1 1 0 °. CCE 2013

p CCE 20 1 2

36. AC and AD are tangents


at C and D, respectively. If
32 . Two tangents PA and PB are drawn to a LBCD = 4 4 ° , then find
circle with centre 0, such that LAPB = 1 2 0 °. LCAD, LADC, LCBD
Prove that OP = 2 AP. NCERT Exemplar and LACD.

33. A circle touches the A B


side BC of a MBC at
P and AB and AC 37. Tangents AP and AO are drawn to circle
when produced at 0 with centre 0 from an external point A.
and R respectively as Prove that LPAO = 2 LOPQ.
shown in the figure. CCE 20 1 3, 1 2, 1 1 , 09
1
Show that AO = - 38. In the given figure, LADC = 9 0 °,
2
( Perimeter of MBC) BC =38 cm, CD = 2 8 cm and B P = 2 5 cm,
or show that AO = _! (BC + CA + AB) then find the radius of the circle. CCE 20 1 1
2 B
NCERT Exemplar; CCE 2012, 1 1
c
34. At one end A of a
diameter AB of a R
circle of radius
5 cm, tangent XAY x 0
is drawn to the s
circle. Find the
length of the chord 3 9 . The radii of two concentric circles are 1 3 cm
CD parallel to XY and 8 cm. AB is a diameter of the bigger
and at a distance circle. BD is a tangent to the smaller circle
touching it at D. Find the length o f AD.
8 cm from A .
y
NCERT Exemplar CCE 20 1 0
112 � Allfflone MATH EMATICS Class 1 0th Term n

40. A circle is inscribed in A 4 1. A chord PQ of a circle is p a ra llel to the


a MBC having sides tangent drawn at a point R o f the circle.
AB = B cm, BC = l O cm Prove that R bisects the arc P RQ.
and CA = 1 2 cm N CERT Exemplar
as shown in figure .
Find AD, BE and CF. 42. If a hexagon AB CDEF circ u m s c ribe a circle,
CCE 20 1 2 prove that AB + CD + EF = B C + DE + FA
c....__-�-;....._-� a
E N CERT Exemplar

�I long A nswer Type Questi ons [4 M a rks each]

43. In the adjoining figure, 4 7. PA and PB are


a right angled MBC, the tangents to a
circumscribes a circle circle which
of radius r. If AB and circumscribes o p
BC are of lengths 8 cm an equilateral
and 6 cm respectively, MBQ. If
then find the value LPAB = 60°, as
of r. CCE 201 1 shown in the figure, prove that OP bisects
AB at right angle. C C E 2015

44. I n the given figure, AD is a diameter of a 48. !HOTS} In the given figure , OP is e qual to
circle with centre 0 and AB is a tangent at A. diameter of the circle. Prove that MBP is an
C is a point on the circle such that DC equilateral triangle .
produced intersects the tangent at B and
LABD = 5 0 °. Find LCOA. CCE 20 1 5

CCE 20 1 2, 1 1 , 09, 08

49. !HOTS! In the given figure , from an external


45.
point P, a tangent PT a n d a line segment
PAB drawn to a circle with c e ntre 0. ON is
OAB C is a rhombus
whose three vertices perpendicular on the chord A B. Prove that
A, B and C lie on a circle
(i) PA · PB = PN2 - AN2
with centre 0. If the
(ii) PN2 - AN2 =op2 - OT2
radius of the circle is
1 0 cm, then find the (iii) PA PB PT2
· = NCERT Exemplar

area of the rhombus.

4 6. Tangents PQ and PR are


drawn to a circle such that LRPQ = 30°. A
chord RS is drawn parallel to the tangent
;-;; ::1
... ·, PO . Find LRQS. CCE 20 1 5
Circles 113

�I Va l ue B ased Questi ons (VBQ s ) [4 M arks e a c h ]


It w a s held by s trings o f lengt h 8 m each ,
50. Two roa d s s ta rti ng from P are touching a
which inclined at an angle o f 60° at t h e
circu l a r p a t h a t A a n d B.
p oint, where it w a s tied a s s h own i n t h e
A figure.
(i) What is t h e l ength o f AB?
(ii) If the perpendicular distance from the
centre of the circle to the chord AB is 3 m ,
then fi nd the radius of the circle.

S a rita r u n s fro m P to A, 20 km and A to 0,


1 5 km a n d R ita runs from P to 0 directly.
(i) Fi nd the d i stance covered by Rita .
(ii) Wh o wi l l win the race?
(iii) Which value is depicted by Rita?

p
5 1. Peo p l e of villa g e wa nt to cons truct a road
neare s t to a circular village Rampur. The (iii) Which m e thod s hould be apply to find
roa d ca n n o t p a s s through the village . But the radius of circle?
the people wa n t the road s hould be at the (iv) What do you think of s uch campaign ?
s h o rtest d i s ta n ce from the centre of the
vi l l a g e . 53. For a S cience Exhibition, Rah u l presented a
( i ) Which road will b e the nearest to the diagrammatic rep re sentation of rain water
cen tre of village? harve sting a s a p roject.
(ii) If the road start from point 0 which is AB and A C, the pipes of A
outside the circular village and touch the 12 m long are bringing
boundary of the circular village at point water from the terrace o f a
A such that OA = 20 cm. If the straight
d istance of the point 0 from the centre C
building (as s hown in the
figure) . The trian g u la r
of the village is 25 cm, then find the
space is d eveloped a s a
shortest distance of the road from the
garden.
cen tre of the village .
(i) What is the perimeter
(iii) Which method should be apply to find
of the triangular
the shortest distance?
garden?
(iv) Which value is depicted by the people of
(ii) If the radius of circle is 5 cm, then fin d
village?
the length o f OA .
(iii) What qualities do you think is
52 . As a part of a campaign , a huge balloon with
encouraged by such exhibitions?
message o f ''.AWARE N E S S OF CANCER"
was displayed from the terrace of a tall
buildin g .

Sol u tio n s
1 . Two tangents can be drawn from a point P lies outside 4. The common point of the tangent to a c i rcle a n d the
the circle. [1 ] circle is called the point of contact. [1 ]
2. Tangent is perpendicular to the radius at the point of 5. The line which is perpendicular to the radi us of the
contact. [1] ci rcle through the poi n t o f contact will be tange n t to
3. The term used for a line which intersect a circle at two the circle. [1 ]
distinct points is secant . [1 1 6. Number of tangents to a circle which a re parallel to a
2
secanr is . [1 ]
11 4 Allinone MATHEMATICS Class 1 0th Term 11

7 . G iven , radi us o f a circle = cm, 3 Now, peri meter of MPQ == APAP ++ RPPQ ++ QR + AQ
AQ
i .e. OP = OQ = 3 cm
N ow, distance between two parallel tangents, = 5IO+cm5 [ fro m (i)] Eq.
PQ = OP + OQ 3 + 3 =
= 6 cm [1 /2]
13 . The tangents drawn from an
=
external point to the
[1 /2]
circle
are equal in length.
. . CR = CP = 6 cm [given]
BQ = BP = 10 cm
and AR = AQ = AB - BQ == 18 - 10 = 8 cm
Now, x == AC = AR CR == 8 + 6
+ = 14 cm [1 ]

1 4. Given, LCAB = 30°


Hence, the distance between two parallel tangents of a
. circle is cm. 6 (1 /2]
Join OC. OA = QC [·: radii of a circle]
8. Let,
centre
and PA0. PB �
are two ta gents to the circle with
A
:::::> LOCA = LOAC [·: angles o pposite to equal
sides of a triangle arc equal]
:::::> LOCA = 30° [given] . . . (i) [1 ]
A tangent is perpendicular co the radius at the point
of contact.
.l

B
:::::>
:::::>
LOCA30°OC++ LACPPQ = 90°LOCP == 90°
:::::>

LPCA = 90°
We know that, radius is perpendicular to the tangent at [from Eq. (i) and LACP == LPCA]
the point of contact.
.. .lOA AP :::::> LPCA = 90° - 30° LPCA = 60°
:::::> [1]
Given, PA = 12
[1 /2]
cm
Also, LOPA =
LAPB = 600 = 300 1 5.

2 2 [·: OP bisects LAPB] We know that, radius is perpendicular to the tangent at


the point of contact therefore, AO .l
AP.
Now, in right angled llPAO,
Now, i n /lOAP,
AP = cot 30° AP = ..J3
:::::>
OP = �(PA)2 + (A0)2 [by Pythagoras theorem]
OA 3 = �(1 2)2 + (5)2
:::::> AP = 3..J3 cm = �144 + 25 [·: radius, AO = 5 cm, given]
Hence, the length of each tangent is 3 ..J3 cm.
· Do same Q. ofNcert folder 3. 2 . [Ans. 2�a2 - b2 ]
as
[1 /2]
= .Jl69 = 13 cm (1 /2]
Similarly, in right angled llPBO, PB = �OP2 - OB 2
9. 7
1 0. Do same Q. of Ncert folder 3. 2 .
as 7 [Ans. 16 cm]
11. Clearly, OA .l PA and OB PB .l
== �(13)2 - (3)2 = ..j1 69 - 9 = .Jl60 = 4.JlO cm
z :.LAOB == 360°-LOAP -LOBP - LAPB [·: radius, OB = 3 cm, given] [1 /2]
i,,-'' � 180° - 50°= 130° [1 /2] 1 6. Let is a tangent drawn from external point P. Join
PT
� Now, in !lOAB, OT.
.1 f) 9A == OB [radii of circle]
'� r, LOBA LOAB . . . (i)
�----bl [·: angles corresponding to equal sides are equal]
«fJ Also, LOAB + LOBA + LAOB = 1_8 0° p
sum of all angles of a triangle is 180°]
:::::>
:::::>
LOAB + [·:LOAB 2 +LOAB
130° = 180°
50°
=
[from Eq. (i)] ·So,: OTOPT isPTa right angled triangle.
.l

,�_ii!] I! [ 1 /2]
:::::> LOAB = 25°
: �- l) [1 /2) In right angled triangle, hypotenuse is always greater
=·=.-:l.!! I:
Let PQ touch the circle at the point R.
:�:�� ! !
1 2. than the other sides of the triangle.
We know that, tangents drawn from an external point :. OP PT> or PT < OP Hence proved. [1 /2)
. ·, - . .--�.:J
9J !I 1 7. x
t� :...
.... ..:. �
1:
.to. a circleAPare+equal
AB =in length .

BP = AC AQ += 5QCcm = 5 cm . . . (i)
Let the radius of circle be cm.
Then, we have OA = OT = cm x [1 /2]
:::::> [1 /2] Clearly, OT PT.l
1-�::..:; . AP + PR = AQ + QR = 5 cm
_;:_ _ ;) , . .
=> [·: radius is perpendicular to the tangent
'• (/.. :} : ; [·: BP = PR and QC = QR] at the point of contact]
Circles : 11 5 r

:. /).() 11' is a righc angled crianglc, having right angle Now, in l:lACP and
ac T. [1 /2) !J.BCP,
Now, in !J. 0 1 1', we have AP = BP (1 /2]
OP 2 = O T 2 + PT 2 [·: lengch of cangencs
drawn from an excernal P�-------+--­
(1/2)
=> (x + 4 ) 2 = x 2 + 8 2 ==> x 2 + 1 6 + Bx = x 2 + 64
poinc co a circle are
=> Bx = 4 8 ==> x = 6 equal]
Hence, radius of circle is 6 cm. (1 /2) => PC = PC
1 8. Given In �A BC, AB = AC and BC meec che circle [common sides]
ac point P. LAPO = LBPO [from Eq. (i)]
To prove P bisccrs chc base or base is bisecced by che /:lACP :: �BCP (1 /2)
poinc of concacr. [by SAS congruence rule]
Proof AB = AC [given] . . . (i) Then, A C = BC [by C PCT]
and LACP = LBCP [by CPCT]
and AR = AQ . . . (ii) (1 )
[·: rangcncs drawn from an cxcernal poinc
I
= - x 1 80 ° = 90° [·: AB is a scraighc line]
co che circle are equal in lengch] 2
On subcraccing Eq. (ii) from Eq. (i), we gee Hence, OP is perpendicular bisector of AB. (1 ]
AB - AR = A C - AQ => BR = CQ . . . (iii) Hence proved.
Also, BP = BR . . . (iv) 22. Given PQ and GH are the com mon tangents to che
and CP = CQ . . . (v) circles and common cangenc ac C meecs PQ ac E and
From Eqs. (iii), (iv) and (v) , we gee BP = CP GH ac F.
Hence, P biscccs chc base BC. Hence proved. (1 ] To prove EF biseccs PQ and GH.
1 9. Lee che chord AB and CD of oucer circle, couch che Proof We know chat, tangents drawn from an
inner circle ac M and N. external point to a circle are equal i n length.
.. EP = EC and EQ = EC
=>
To prove AB = CD
EP = EQ [1 ]
Co11structio11 Join OM and ON. => PQ is bisecced by EF at E.
Proof Since, OM and ON arc radii of Similarly, FG = FC and FH = FC
che inner circle chrough che points of => FG = FH
contacc M and N of che cangents AB => GH is bisected by EF at F.
and CD, rcspeccivcly. [1 ]
. . OM ..l AB and ON ..l CD.
Hence, the common cangenc EF drawn at point C
biseccs che other two common tangents. [1 ]
Also, OM = ON [radii of che inner circle] Hence proved.
Thus, AB and CD are cwo chords of che oucer circle 23. Lee AB and CD are two
which are equidiscanc from ics centre 0. perpendicular diameters of a p T1 T2 R
Hence, AB = CD Hence proved. (1] = --r+-''- T3
circle with centre 0 and 7i , T2 -"-'1-r------=-""--==-
20. Given, AD = 8 cm, A C = 6 cm and BT = 4 cm are tangents co circle at A , B
Clearly LCAD = 90° [angle in a semi-circle] and 13 and T4 are tan genes to A l-T-----+--"-1 a
=-=-"'
/ .,

So, in l:lACD, circle at C and D,


. '.' I
;_:;..�
. .:.J
respectively. [1 ]
CD 2 = A C 2 + AD 2 = 36 + 64 = 100
From figure, PQSR is a Q I �:·
[by Pychagoras rheorem] D ��
square. [given]
=> CD = I O cm PQ I I RS
1O · ·� :
Therefore, OC = OD = OB = cm = 5 cm [1 ] AP I I BR . . . (i)
2 �· ·· ·�
Also, LCPQ = LOA Q = 90° ·

[·: radius ofa circle] PR I I AB . . . (ii)


Since, LOB T = 90° [·: corresponding angles are equal] 'L .

[angle becween radius and cangenc] �-.


From Eqs. (i) and (ii), we get
So, in !J.OB T, OT 2 = OB 2 + BT 2 = 25 + 1 6 = 4 1 APRB is a parallelogram.
[by Pythagoras theorem] .. AB = 2 [·: opposite sides of
OT = ..J4i cm [1 ] parallelogram are equal]
21 . Given PA and PB are che two cangencs from a point P So, radius of circle = AB = � = 1 cm [1 ]
co che circle wich cencre 0. 2 2
Lee OP incersecc AB ac a poinc C. 24. Given, BC = 3 cm, A C = 4 cm
To prove OP is perpendicular biseccor of AB. In MCB, LBCA = 90° [angle in a semi-circle]
Proof Clearly, LAPO = LBPO . . . (i) .. 2 2
AB = A C + BC 2
[·: 0 lies on biseccor of LAPB] [by Pythagoras theorem]
116 AllinOne MATHEMATICS Class 1 0th Term I

=>
=>
AB 2 = 4 2 + 3 2 => AB 2 = 1 6 + 9
AB 2 = 25 => AB = .J2s
=> L TOP = cos 60°
cos ·: cos 60° = [ i]
=> L TOP = 60°
=> AB = S cm => OA = 52 cm [1 ] -
=> L TOS = 2 L TOP = 2 x 6 0 ° = 1 20 ° [1 n)
Also given, MCB Di.PAO ..
In D.OTS,
AB - OP S OP I.e. -OP = S [1 ] OT = OS [-.· ra<l ii o f a circle)
.". -- - -
AC = AP => 4 = - AP AP 4 L O TS = LOST
[·: angles opposite co equal sides a rc equ al] [ 1 /2)
25. Given AB and CD are two common tangents to the Now, as LOTS + LOST + L TOS = 1 80 °
circles C1 and C2 (say) .
[by angle sum property o f triangle]
To prove AB = CD
Proof As we know that, the tangents drawn from an
=> 2 LOTS + 1 20 ° = 1 80 °
=> 2 LOTS = 60° => LOTS = 30 °
=>
exterior point to a circle are equal in length.
:. We have
LOTS = LOST = 30 ° Hence p roved. [1 /2]
EA = EC 28. Do same as Q. 26. [A n s . 1 3 cm)
29. Lee the circle S1 , S2 , �' , S4 . . . . . . circles a rc couch a li ne
. . . (i)
and EB = ED . . . (ii) [ 1 ]
O n adding Eqs. (i) and (ii) , we get segment PQ at a poi nt A . Lee the ccnc rcs o f the circles
EA + EB = EC + ED
S1 , S2 , S3 , 54 , . . . be C. , C 2 , C3 , C4 , . . . respectively.
1.e. AE + EB = CE + ED
=> AB = CD Hence proved. [1 ]
26. Let the common centre be 0
and AB be the chord of the
larger circle.
Given, AB = 8 cm
and a diameter CD of the
To prove Centres of these circles lie on the

� [
smaller circle, i .e. CD = 6 cm

J
OD = CD perpendicular of PQ bur not on the bisector of PQ.
On joining each centre of the circles to the point A , gt(

Now,
2
dia eter the line segment C1 A , C2A, C_� A, C4 A . . . so on.
= (6) = 3 cm ·: radius = We know chat, if we draw a line from the centre of a
JoinOA. Since, D is the point of contact. circle to its tangent line, then the line is always
OD J_ AB perpendicular to the tangent line. Bue i t docs not bisect
the line segment PQ.
=> D is the mid-point of AB.
So, C1 A _l_ PQ [for S. J
[·: perpendicular from centre to the
C2 A _l_ PQ [for S,]
AB
chord bisect the chord]
C3 A J_ PQ [for s; 1
=>
8
AD = - = - = 4 c m [1 ] C4 A _l_ PQ [for S4 ]
( 1)
2 2 so on
Now, in right angled MDO, Since, each circle is passing thro ugh a point A.
i-�-'
A0 2 = AD 2 + OD 2 [using Pythagoras theorem] Therefore, all the line segments
L-�
= 4 2 + 3 2 = 1 6 + 9 = 25 = 5 2 => AO = 5 cm C1A , C2 A , C3A , C4 A, . . . , so o n are coi ncident.
'f Diameter = 2AO = 2(5 ) = 1 0 cm So, centre of each circle lies on the perpendicular line of
·1 J PQ but they do not lie on the bisector of PQ.
� Hence, the diameter of the larger circle is 1 0 cm. (1 ]
H ence p roved . ( 1 ]
27. Given, PT and PS are tangents drawn from an external
1' .:f' 30. Lee a, b and c be the sides of right angled Di.ABC, such
. .# J point P to a circle with centre 0 and radius r.
· r that BC = a, CA = b and AB = c .

dA 1
_.;'/ ,,: Also, OP = 2r
�:::-�- Let the circle touches the sides
·: A tangent is perpendicular to the radius ac the point of
contact. BC, CA , AB at D, E and F,
OT _l_ PT => LOTP = 90° [1/2] respectively.
Then, AE = AF
r.·-:. . .. .

::.��
[ - =�-�..:..:..:- Now, right angled {).OTP, /c F
: ,-·
and BD = BE
-=?.J-,;J
OT
.
I cos LTOP =
OP [·: tangents drawn from an //
external point are equal in length] 8 L---�-:-"""'--....i
cos L TOP = !_ [·:OT = r and OP = 2r] Also, CD = CE = r
2r In quadrilateral OECD each
cos L TOP = -I angle is 90°. So, OECD is a square therefo re , we have
2 OE = EC = CD = OD
Circles f 11 7
AF = b - r and BF = a - r [1 1 and CP = CR [tangents from CJ . . . (iii) [1 ]
::::> AF + BF = (b - r) + (a - r) Now, perimeter of MBC
::::> AB = b r + a - r
- = AB + BC + AC
::::> c = a + b - 2r [·: AB = CJ [sum of three sides of a triangle]
::::> 2r = a + b - c
= AB + BP + CP + AC
r = a +b-
c
---- Hence proved. [1 ] = (AB + BQ) + (CR + AC) [1 ]
2 [using Eqs. (ii) and (iii)]
31 . Given, LPCA = 1 1 0° = AQ + AR = AQ + AQ = 2AQ [from Eq. (ii)]
::::> .!_ (Perimeter of MBC) = AQ
2
Hence, A Q = .!_ (Perimeter of MBC)
2
or
1
AQ = -2 (BC + CA + AB) Hence proved. [1 )
Join CO. 34. Given, a circle of radius
0.
LPCA = 1 10° 5 cm and centre
tangent XY at point and
A
A
Then, LACT = 180°-1 10°= 70° [linear pair]
a chord CD which is
Bue LOCT = 90° parallel to XY at a distance
[radius is perpendicular co che cangenc] 8 cm fromA.
So, LOCA = 90° - 70°= 20° JoinOC.
Now, LOAC = LOCA = 20° [·: OA = OC] Since, OA AY .1
LCBA = 90°-LOAC = 90° - 20° = 70° . . LOAY = 90°
[·: LACB = 90° angle in semi-circle] [1 ] [·: radius is perpendicular to the tangent of the circle
Again, LAOC = 180°-LOCA - LOAC at the point of contact]
[angle sum propercy of a triangle] Now, as LOAY + LOED = 180°
::::> LAOC = 180°- 20°- 20°= 140° [1 ] [·: XY 11 CD, therefore sum of cointerior
Now, LBCO = LOBC = LCBA [·: OB = OC] angles of a transversal is 180°]
So, LBCO = 70° [1 1 LOED = 90° [1 ]
32. Given PA and PB are cwo Now, in right angled llOEC,
tangents and LAPB = 120°. OC 2 = OE 2 + EC 2 [by Pythagoras theorem]
To prove OP = 2AP
Construction Join OA ==> Ec 2 = oc 2 - 0E 2 = s 2 - 3 2
and OB. [·: OC 5 cm, OE
= radius =
Proof Clearly, OA ..l AP = AE - AO = 8 - 5 = 3 cm]
and OB ..l BP � EC 2 = 25 - 9 = 16 ::::> EC = 4 cm [1 ]
Hence, length of chord CD = 2 CE = 2 x 4 = 8 cm
u�.J
[·: tangent co a circle is perpendicular co the [·: perpendicular from centre to the
radius through the poinc of contact] chord bisects the chord] [1 ] .�

LOAP = 90° = LOBP �


35. In quadrilateral APBO, AO ..l PA - '"
:Y
In llOAP and llOBP, AP = BP :· _,.,.

BO ..l PB
[·: tangents drawn from an external point to the circle]
OP = OP [common sides] [ ·: radius is perpendicular to tangent
-� �
OA = OB [radii ofa circle] at the point of contact] ..,, ... ,·

. . llOAP = llOBP [by SSS congruence rule] [1 ] LPAO = LPBO = 90° [1 J �,;�;,·]
Then, LOPA = LOPB [by CPCT] LAPB + LPAO + LPBO + LAOB = 360 ° ·�

[by angle sum property of a quadrilateral] i,1 ;bJ.k;J


= _!_ LAPB = .!_ 120°= 60°
x [1 ]
75° + 90° + 90° + LAOB = 360 ° :1
; r · ;-�.,,
2 2 1 ! -;:. ._-.....
AP � LAOB = 1 05 ° :·l �.:L
In llOAP, cos 60° =
AP
-::::> -1 = - i i :1:-
'
, :.:.!
-
OP 2 OP =:)
I I
LAQB = - LAOB = - x 105 ° = 52 - 1° I ,-_;-�,-
! �
=:) OP = 2AP Hence proved. [1 ] 2 2 2 ! ;�'�.:.1
33. We know that, tangents from an exterior point to a [angle subtended by an arc at the centre is double of ! ! :�:i .
; · ·�i
the angle subtended on remaining part of the circle] [1 ] � - -
circle are equal i n length. -:: • :! - ·

.. AQ = AR
[tangents from ... (i) A] In cyclic quadrilateral AQBM, •.•.
:..:l:..,
· --=-·::..-.

BP = BQ
[tangents from B] ... (ii) LAQB + LAMB = 1 80°
118 ' AlJ.fnOne MATH EMATICS Class 1 0th Term I'

[sum of opposite angles of a cyclic In MPQ, LPA Q = 1 80 ° - ( LA l'Q + LA QP )


quadrilateral = 1 80°] [ angle s u m p ro perer of a trianglt�
= 1 80 ° - ( x + x ) = 1 80 ° - 2x . .. (i) (1)
52 .!..: + LAMB = 1 80° OP .l AP
2
1 05 0 2550 10 [·: radius is perpend icula r co chc cangcnr
a c c h c po i nc of cont.ta.
LAMB = 1 80 0 _ = = 1 27
2 2 2
lo lo L O/'A = 9 0 °
Hence, LAQB = 5 2 - and LAMB = 1 27 -
2 2
[1 ] => L OPQ + LA PQ = 90 °
=> L OPQ + x = 90 °
=> L OPQ = 9 0 ° - x
36. G iven, A C and AD are tangents at C and D and
=> 2 L OPQ = 1 80 ° - 2.\·
LBCD = 44°
[ m u l c i plyi ng boch sides by 2) ... (ii
Clearly, LOCA = 90 °
From Eqs. (i) and (ii), we h a ve
[angle between tangent and radius]
Now, L OCA = LOCD + LACD
=>
LPA Q = 2 L OPQ Hence proved. (1)
LA CD = LOCA - LOCD
=> LACD = 90° - 44° 38. Given, BC = 38 cm, CD = 2 8 c m
=> LA CD = 46° [1 ] and BP = 25 cm
As, A C = AD Clearly, BQ = BP = 2 5 cm
[tangents drawn from an external point [·: BP and BQ arc tangent d rawn from an extcnul
are equal in length] p o i nc B co the circk]
So, LADC = LA CD = 46° Then, CQ = BC - BQ [1'2)
[·: angles opposite co che equal sides are equal] [1 /2] = 38 - 2 5 = 1 3 c m
Also, L CAD + LADC + LACD = 1 80° CR = CQ = 1 3 cm
[angle sum property of a triangle] RD = CD - CR
=> LCAD + 46°+ 46° = 1 80° = 28 - 1 3 = I S c m (1 )
=> LCAD = 1 80°-92° = 88° [1 /2] Here, OR .l RD and OS .l DA .
Again, [·: rangen c is perpend icular co the radiw
LCOD = 1 80°-LCAD
through the point of contact]
=> LCOD = 1 80°- 88° = 92° Also, LADC = 90 °, then fourth a ngle i n q uadrilatcr.Al
[·: in quadrilateral ODA C, LC + LD = 90°
[112]
ORDS will be 90°. Thus, ORDS will he a rectangle.
and also we know sum of all interior angles of a
quadrilateral is 360°] ·: D is an external point of a circle.
Further, LOBD = LODB DR = DS
[OB = OD radii of circle] Also, opposites sides are equal.
2 L OBD = LCOD [exterior angle theorem] .. RD = OR = OS = SD
2 LCBD = 92° Hence, quadrilateral DROS is a square.
LCBD = _!._ x 92° = 46° . . Radius = OR = RD = I 5 c m [1]
2 39. Given, radii of two concen tric c i rcles a re 13 cm and
Thus, LCBD = 46°, L.ADC = 46°, LCAD = 88° 8 cm.
and LA CD = 46°. [1 ] Produced BD co meet the bigger circle at £. Join AE.
3 7 . Given A circle of centre 0. Two cangencs AP and A Q Then, L.AEB = 90° [·: angle i n semi-cirdeJ
-�···:= are drawn c o the circle from a n external point A .
p

Clearly, OD .l BE
0 [·: BE is tangent co che smaller circle at D
To prove LPA Q = 2 L OPQ and OD is its radius!
Construction J o i n OP and OQ.
:. BD = DE
[·: BE is a chord of the bigger circle and OD J_ BEi
Proof We know chat, tangents drawn from an external Now, in MEB, 0 and D are che m id-points of AB and
poi nt to a circle are equal in length. (1 ] BE, respectively.
AP = AQ Therefore, by mid-point theorem, we h ave [1)
=> LAPQ = L.A QP = x
OD = _!._ AE
[say]
[ · : angles opposi te co equal sides are equal] 2
Circles 11 9
A r: = 2 x 8 = 1 6 cm LPSR = L QSR = 90 ° [·: RS ..l PQ]
I·: OD = radius of smaller circle = 8 cm] RS = RS [com mon sides]
In right angled � O/J/J, PS = SQ
OR 2 = OD 2 + /JD 2 [by Pythagoras theorem] [·: perpendicular from cen tre co rhe chord
1.1 2 8 2 + BD 2 bisect rhe chord]
=:> =

b.PSR = b. QSR
BD 2
.. [by SAS congruence rule]
=:> = J (,9 - 64 = 1 05 Then, PR = QR [by CPCTJ [1 ]
=:> IJ/J = Ji 05 cm We know chat, if two chords are equal, then thei r
D /:' JWs cm corresponding arcs arc also equal.
Hence proved. [1 /2]
=:> = [·: BD = DE] [1 ]
Hence, R bisects rhc arc PRQ.
2
Now, in right angled MED,
AD 2 = A /:' + ED 2 [by Pyrhagoras rheorem] 42. Given A hexagon ABCDEF circumscribe a circle.
A
=:. 1W 2 = (1 6 ) 2 + (JI05)2 = 2 5 6 + 1 05
=:> A D 2 = 36 1 =:> AD = J361 F
AD = 1 9 cm u
Hence, rhe length of AD is 1 9 cm.
E
[1 1
40. We know that, tangents drawn from an exrerior point
0
co a circle arc equal in lcngrh.
.. AD = A F = x cm [say] [1 /2]
BD = BE = y cm [say] To prove AB + CD + EF = BC + DE + FA
CE = CF = z cm [say] Proof AB + CD + EF = ( A Q + QB)
Given , AB = 8 cm + ( CS + SD ) + ( EU + UF)
=:> AD + BD = 8 cm =:> x + y = 8 . . . (i) [1 ] = AP + BR + CR + D T + ET + FP
BC = I O cm =:> BE + CE = I O cm = (AP + FP) + ( BR + CR ) + ( D T + ET) [1 ]
=:> y + z = I0 ... (ii) AB + CD + EF = AF + BC + DE
and CA = 1 2 cm =:. CF + AF = 1 2 cm A Q = AP, QB = BR, CS = CR,
=:> z + x = 12 ... (iii) DS = D T and EU = ET

[ �]
On adding Eqs. (i), (ii) and (iii), we gee [tangents drawn from an external point to a circle arc
2(x + y + z ) = 30 equal] Hence proved. 1
=:> x + y + z = 15 . . . (iv) [1]
On subcracri ng Eq. (ii) from Eq. (iv), we gee 43. Do same as Q. 30 at page 1 I 1 . [Ans . 2]
x = 15 - 10 = 5 44. Since, AD is a diameter of a circle, so AD is
On subtracting Eq. (iii) from Eq. (iv), we gee perpendicular to the tangent AB.
y = 1 5 - 12 = 3 .. L DAB = 90 °
On subtracting Eq. (i) from Eq. (iv), we get In MBD,
z = IS - 8 = 7 L. DAB + L. ABD + L. ADB = 1 80 ° [1 ]
.. AD = xcm = S cm, BE = ycm = 3 cm [·: sum of all angles in a crianglc is 1 80°]
and CF = z cm = 7 cm => 90° + 50° + L A DB = 1 80 °
Hence, the length of AD , BE and CE are 5 cm, 3 cm => L ADB = 1 80 ° - 1 40 ° = 40 °
and 7 cm, respectively. [1 )
In b.ODC,
41 . Lee MN is a tangent drawn from a point R of the circle OD = OC [radii of same ci rcle]
having centre 0. Then, chord PQ is drawn parallel co =:> L OCD = L CDO = 40 ° [1 ]
[·: angles op p osi te to equal sides arc equal]
the tangent line MN.
M R N L.DOC + L. OCD + L CDO = 1 80 °
[·: s u m o f angles in a triangle i s 1 80°]
=:> L DOC + 4 0 ° + 40 ° = 1 80 °
=:> L DOC = 1 00 ° [1 ]
Since, AD is a straight line.
[1 /2] .. L DOC + L COA = 1 80 °
To prove R bisecrs rhe arc PRQ. => 1 00 ° + L COA = 1 80 °
Consh-uction A perpendicular line OR is drawn on MN => L COA = 8 0 ° [1 ]
and extended RO co intersects PQ at S which is also 45. Given, OABC is a rhombus.
perpendicular co PQ. [1 ] .. OA = AB = BC CO =

Proof In b.PSR and 6.QSR, Also. OB = OA = OC 1 0 c m [ rad i i of("irdcl [ 1 ]


=
120 \ Allin.One MATH EMATICS Class 1 0th Term I

Thus,OAwe=have OBt10BC
= ABare and OB =triOC =s and
BC 47. Clearly, LQAB = 60° and LQBA = 60°
=> !:::.O
A, B and equilateral a ngl e So, LPAQ = LPAB + LQAB [·:=MBQ60° +
is60°equil= a120°
tcnl)
OB
.·.Areais theof rhombus
diagonal ofOABC
rhombus
= 2 OABC.
Area of ll.OAB [1 ] Similarly, [·: PABLPBQ = as120°LPA =PB] . . .
x
[·: diagonals of a rhombus = LPBA,
triangledis vofidequal
e it inarea]
two Now, in ll.PAPAQ=andPB ll.P. BQ,[tangents from external point

[ area of an equilateral triangle = 1: (side)2 ]


= 2 .J34 10 2
x x [1 ] =>
=>
AQ = BQ [MBQ equi
[each 120°, shown aOO\"t, l a teral
·:
So, LPAQ 6.PAQ =
= LPBQMBQ [by SAS congruence ruk
=> LAPQ = LBPQ [ b y CPCT) ... (ii) [1}

= 2 .J34 100 =50.J3 cm 2


x x
Let
Now,QPinLAPM intersect
fl.PAM=AB andLBPMatll.PM.BM,
Hence, the area of the rhombus is 50./3 cm 2 • [1 1 => PA = PB => PM = PM [from Eq. (ii
So, fl.PAM =
AM = BM ll.PBM [by SAS congruence ruk
JoiLetn0OQand
be the centre of circle.
46.
OR. Then, => [1)
1-
OQ[·: tangent 1-
PQ andis perpendi
OR PRcular to the radius and LAMP =LBMP LBMP= 180° [by ... CPCT] (iii
But LAMP
=>
+

LOQP = 90°andthrough
__ o
LORPthe= point
90° of contact] => 2 LAMP
LAMP + LAMP = 180°
=>
=and180°(iv), we getLAMP = 90° ..AB. ''
(i\·
From Eqs. (iii) that QP bisects
right angles. [2)

s p 48.
.Let
.ClearlrOAbey, the= rradiandus of theOPcircle.= 2r [given}
L[·:OAP =90° [1)
tangentthrough
is perpendicular tocontaa]
radiw

sin LOPA = sin 30° [·: sin 30° ..!..


So, LROQ + [LRPQ the point of
·: sum of=quadri
all180°interilateralor angl es of
is 360°] In fl.OAP, sin LOPA =- QA
=
OP 2r 2 - r = -1
=> LROQ + 30° = 180° => =
=> LROQ = 150° 2
=>
But LRSQ = -21 LROQ Similarly, LOPA LOPB == 30°
30°
LAPB = 30° + 30° = 60° (1)
So, LRSQ = 21 150° = 75° ... (i) [1 ]
- x Since, [·: lengths ofPAtangents = PB
z ng QO =to 90°
Now, on extendiLQAS intersect RS at A, we get drawn from anareexternal
=>
point equal
LPAB = LPBA [1)
[·: PQ ll RSandLOQP = 90°] [·: angles opposite to equal sides are equal
Therefore, from ll.QSA, Now, inLAPB AA.PB, + LPAB + LPBA = 180°
LSQA = 180° - 9 - 75° = 15° [1 1
Also, LPQR + LPRQ +0° LQPR = 180° =>
[·.· sum 60° of all+angles
2 LPAB of a =triangle
180° is 1 soi
LPQR + LPQR + 30° = 180° =>
[·: LPQR= = LPRQ because PQ = PR]
2 LPQR 150 150°0 nce,2 alLPAB
SiHence, angles=isof120°
lAABP MBP =>
are LPAB
60 °. = 60°
an equilateral triangle. (1 )
LPQR = 2 = 75 0 [11 Hence proved.
point P
But LAQP = 90°[angle between tangent and radius] 49. Given
a linePTis perpendicular
andON
0.
is a tangent
segment PABdrawn fromto ana circle
external
the chord AB. centrt
isondrawn with
So, LAQR = LAQP =
- LPQR
90° - 75° = 15° To prove

=
PAPN 2 PBAN= PN
(t) · 2 - AN 2
So, LRQS 15° LSQA + LAQR
=
(iz) - 2 =2 OP 2 - O T 2
[1}
+ 15° = 30° [1 ] (iit)PA PB = PT
·
Circles J 1 21
Proof (1) PA · PB = ( PN - AN) (PN + BN) => A C 2 = OC 2 - OA 2 = ( 2 5 ) 2 - ( 20 ) 2
[· : A N = BN as ON perpendicular ro chord AB = 625 - 400 = 225
and so bisecc ic]
=> AC = .J225 => A C = 1 5 cm
= PN 2 - AN 2 [1 )
(i1) PN 2 - A N 2 = ( OP !. - ON 2 ) - AN 2 Hence, rhe shorresr discance of rhe road from rhe
cenrre of village is 1 5 cm. [1 )
[·: in NJNP, OP 2 = ON 2 + PN 2 (iii) Concept of cangcnc to a circle and Pythagoras
=> PN 2 = OP 2 - ON 2 ] theorem. [1 )
= OP !. - ON 2 - AN 2 (iv) Progress and development o f village. [1 ]
= OP 2 - ( ON 2 + AN 2 ) = OP 2 - OA 2 52. ( 1) Given, PA = PB = 8 m
Clearly, PA and PB are
[·: in !1 0NA, QA 2 = ON 2 + AN 2 ] cangenrs co the circle.
= 0P 2 - O T 2 Now, draw OP which
[·: OA = OT = radii of circle] [1 ] bisects LAPE and
(iii) From pans (i) and (ii) , we gee perpendicular to the
chord AB.
PA PB = OP 2 - OT 2
·
Thus, we have
=> PA · PB = PT 2 LAPC = LBPC = 30 °
[·: in !10TP, OP 2 = OT 2 + PT 2
=> PT 2 = OP - OT 2 ] [1 ]
2 and LA CP = LBCP = 90°
Hence proved. In MCP, LAPC + LA CP + LPA C = 1 80 °
SO. (1) Given, PA = 20 km and QA = 1 5 km [·: sum o f all angles o f triangle i s 1 80 °]
=> 30° + 90° + LPA C = 1 80 °
We know char, chc rangenr ac any poinc of a circle is
perpendicular co chc radius chrough che poinc of => LPA C = 1 80° - 1 20 ° = 60°
concacc. Similarly, LPBC = 60°
OA 1- PA [1 ] Thus, !1APB is an equilateral triangle.
I n righ c angled !10AP, .. AB = AP = BP = 8 m [1 ]
OP 2 = OA 2 + AP 2 [by Pychagoras cheorem] (iz) Given, OC = 3 m
= (1 5 ) + ( 20 ) = 225 + 400
2 2 We know that, if a perpendicular drawn from the
centre of the circle to the chord, then it bisects the
=> OP 2 = 625 chord.
=> OP = .J6i5 = 25 km AB 8
A C = BC = - = - = 4
Hence, Rica covers 25 km. [1 ] 2 2
2
In right angled M CO, OA = A C 2 + OC 2
(iz) Discance covered by Rica = 25 km
and distance covered by Sarita [by Pythagoras theo_rem]
2 2
OA = ( 4 ) + ( 3 ) 2
= 20 + 1 5 = 35 km =>
So, it is clear chat Rita covers less distance chan = 1 6 + 9 = 25
Sarita to reach at 0. So, Rita will win the race. [ 1 ] OA = 5 m
(iiz) Rica choose shortest pach to reach a t 0 . So, i t shows which is the radius of the circle. [1 ]
her inrelligcncy. [ 1] (iiz) Pythagoras theorem. [1 ]
5 1 . (z) The road which is cangenc co rhe circular village
..
.. _
(iv) Creating awareness and taking initiative. [1 ]
Rampur will be rhe nearesr to the centre of village. [ 1]
53. (z) Do same as Q. 1 4. [Ans. 24 m]
(ii) Given, OA = 20 cm and OC = 25 cm (i1) Clearly OB ..L AB
A [·: tangents is perpendiuclar and encouraging co
the radius through the point of contact greenery]
LABO = 90°
Now, in M OB, we have
OA 2 = OB 2 + A B 2 1 :

Here, CA ..L OA => OA 2 = 5 2 + 1 2 2 [·: OB = radius = 5 cm]


=> OA = 25 + 1 44
[·: radius is perpendicular to the tangent 2
at the point of contact] [1 ]
In right angled !10AC, => OA = .Jl69 = 1 3 cm
OC 2 = OA 2 + AC 2 [by Pychagoras cheorem] (iii) Creativity, saving water, team work. [1 ]
FORMATIVE ASS ESS M E N T
• •

Act1v1ty
Topic Covered Tangents drawn to a circle from a n Oral Questions
external point are of equal length. · 1 . What is meant by a tangent to a circle?
Objective To verify that the lengths of the two tangents 2. What is the difference between a chord a nd tangent?
to a circle from an external point are of equal length. 3 . How many tangents can be d rawn to a circle from an
S ki l l Developed Creativity, analysis. external point?
Ti me Required 15 min 4. How many tangents can be d rawn to a circle from a
point on the circle?
5 . What is a secant?
M aterials Requi red Tracing paper, geometry box, paint
brush, grey colour paint, red colour paint.
Method
1. On the tracing paper, d raw a circle of radius r with
True/False
centre O and take a point P outside the circle. 1. I n the given figure, PA and PB a re tan ge nts to a circle
from an external point P. Then, PA a n d PB may or
2. From an external point P, draw PA and PB tangents to may not be equal.
the circle, where A and B are their points of contact.
3. Join QA, OB and OP.

p
2. In the given figure, incircle of LiABC touches its sides
at D, E and F. If perimeter of LiABC = 24 cm, then
A F BD CE = 12 cm.
+ +
A
4. Paint the right angled LiOAP with grey colour.
5. Pai nt the right angled tiOBP with red colour.
6 . Fold the tracing paper along the line OP as shown i n
Fig. (ii).
O bservation

D
We observe in Fig. (ii) that l10AP and aOBP cover each

3 . In the· given figure, BA and BC a re tangents to a circle


other, i.e. OB comes on OA and BP on AP.

with centre 0. If LABC = 63 °, then LAOC = 117°.

�p0
Fig. (ii)
B
Thus, we find that BP = AP.
i.e. the two tangents drawn from P are of equal length.
Hence, the tangents drawn from an external point to a circle
a re of equal length.
\
�ircles J 123
4. In the given figure, PT is a tangent to the circle F i ll i n the B la n ks
with centre 0 such that OP is 4 cm and LOPT = 30°,
1 . A tangent to a circle intersect it i n ... point.
2 . A line intersecting a circle i n two points is called a . . . .
then length of ta ngent is

3 . A circle can have . . . para l lel tangents atmost.


4. The common point of tangent a n d the c i rcle is cal led

5 . The distance between two parallel ta ngents d ra w n to


p a circle is equal to ... .

5. A circle can have maxi m u m two ta ngents.

Crosswo rd Puzzle
4 Ac ross
,-----

1. A line which meets a circle at two d istinct points i s . . .


1
of the circle.
2. The lengths of two ta ngents from an ... point to a
2
I I 5 circle are equal.
3 . A circle can have ... para l lel tangents atmost.

Down
4. A line which touches t h e circle at o n e point is cal led
... to the circle.
3 6 5. The tangent to a circle is perpendicular to the . . .
through the point o f contact.
- -
6. A ta ngent to a circle i ntersects it in . . . poi nt(s) .
,___

Answers
O ra l Questio n s
1 . A line which touch the circle at only one point is called tangent to a circle.
2. A chord meets the circle at two points, whereas a tangent meets the circle at only one point. ,
;�.-.-. -
3. Two
4. One
5. A line which intersects a circle in two pofots, is called a secant.

True/Fa lse
1 . False 2. True 3. True 4. False 5. False

Fi l l in the Bla n ks
1 . one 2. secant 3. two 4. point of contact 5. diameter .;
"--·

Crossword Puzzle
1 . Secant 2. Externa l 3. Two 4. Tangent �:- ·•� -.

-
� - -":"",-·

j t'. -
5. Radius 6. One
S U M MARY
a A line segment which join a ny two points of a circle is called a chord of the circle.
• A piece of a circle between two points is called a n arc. There are two pieces: one longer and the other sma l l e r . The long
one is called the major arc and the smaller one is called the m inor arc.
• A diameter of a circle divides it i nto two equal parts, which are two equal arcs. Ea·ch of these two a rcs is called a
semi-circle.
• The length of the complete circle is cal led the circumference of the circle.
• The region between a chord and either of its a rcs is called a segment of the circular region. The segment formed by min"·
arc, is called minor segment and the segment formed by major arc, is called the major segment.
• The region between an arc and the two radii, i s called a sector. The sector formed by m i nor a rc is called m i n o r sector ano
the sector formed by major arc, is called major sector.
• A line which intersect the circle in two points is called secant of the circle.
• A tangent to a circle is a line that intersects or touch the circle at only one point. This point i . e . the common point of the
tangent and the circle is called the point of contact.
• The tangent at any point of a circle is perpendicular to the radius through the point of contact.
• Converse of theorem is also true .
• Number of tangent according to the position of a point P are below:
• When point P lies outside the circle, then exactly two tangents can be drawn .
• When point P lies on the circle, then one and only one tangent can be drawn .
• When point P lies inside the circle, then no tangent can be drawn.
• The lengths of two tangents drawn from an external point to a circle are equal.

"•J
. ·- . •·

CHAPTE R EXE RCIS E


� / Very Short Answer Type Questions [ 1 Mark each]

1. In the adjoining figure, PA 3. If radii of two concentric circles a re 4 cm a nd 5 cm,


a nd PB are tangents from P then find the length of each chord o f o n e ci rcle which
to a circle wi th centre 0. If p is tangen t lo the other circle. NCERT Exemplar
LA OB = I 3 0 °, then find
LAPE. 4. In the given figure, AB is a chord of a circle a n d AOC is
its diameter such that LACB = 5 0 °. If AT is t ht•

A �B
tangent to the circle at point A, then fin d LBAT .
the given figure, if LAOB 1 2 5 ° , then find LCOD.

B
2. In =
c

o C NCERT Exemp lar


A T NCERT Exemplar
circles

5. In the given fig u re, if PQR is the tangent to a circle at Q 6. In the given figure AB and CD a re com m on tangen t s t o
whose centre is 0, AB is a chord parallel to PR and two circles of unequal rad ii.
LBQR = 70 °, then find LAQB. Prove that AB = CD.
NCERT Exemplar; CCE 20 1 3, 1 2
A

p NCERT Exemplar

� I Short Answer Type I Questions (2 Marks e a c h ]

7. In the given fig u re, if LATO = 40 °, then find LAOB.


A

CCE 20 1 6
T
12.

Q
In the adjoining figure,
quadrilateral ABCD is
circumscribed, touching the circle
B at P, Q, R and S. If AP = 5 cm,
8. In the adjoining figure, PQ BC = 7 cm and CS = 3 cm, then
is the common tangent to
find the length of AB. A Q B
both the circles. SR and PT 1 3. TWo concentric circles of radii 1 3 cm and 1 2 cm, are
are tangents. If SR = 4 cm given. Find the length of chord of the larger circle
and PT = 7 cm, then find which touches smaller circle. CCE 20 1 2, 1 1
RP.
9. Out of the two concentric
-......c:;;..___..l..-_� p
0 R 14. !HOTSJ In figure, AB and CD a re common tangen t s to
circles, the radius of the outer circle is 5 cm and the two circles of equal radii. Prove that AB = CD.
chord AC of length 8 cm is a tangent to the inner Further prove that dOAB = dOCD.
circle, then find the radius of the inner circle. [Hint Join OB and OD, then OB = OD. ]

CJ C)
10. In the adjoining figure, PQ is A B
a chord of a circle with
centre 0 and PT is a tangent
at P such tha t LQPT = 60 °,
then find LPRQ.
C D
In the given figure, 0 is the
A T
1 1.
centre of a circle, BOA is its diameter and the tangent 15. !HOTSJ Prove that a diameter AB of a circle bisects a l l
at the point P meets BA extended at T. If LPBO = 3 0 °, those chord s which are parallel to the tangen t a t t h e
then find LPTA. point A.

� I Short Answer Type II Questions [3 Marks each ]

1 6. A tangent PT is drawn parallel to a chord AB as 1 7. In the given figure, the incircle of MBC touches t he
shown in figure. Prove that APB is an isosceles sides BC , CA and AB at P, Q and R, respect ively.
triangle. Prove that (AR + BP CQ) = (AQ + BR + CP)
+
T P
A
= _!_ ( Perimeter of MBC) .
2
All·inone MATH EMATICS Class 1 0th Te rm II

1 8. If AB is a chord of a circle with centre 0, AOC is a 2 1 . (fioiSJ Let s denotes the semi- peri meter o f a MBC in
d ia meter and AT is the tangen t at A as shown in which BC = a , CA = b a nd AB = c. I f a circle touches
figure. Prove that L BAT = L ACB. the sides BC , CA , AB a t D, E , F ,re s pect ively. Prove
c t ha t BD = s b.
-

2 2 . (HOTS] In the adjoining fig ur e , t h ree


circles with cen t res, A, B and
C, respect ively touch each
other externally.
T I f AB = 5 cm, BC = 7 cm and
CA = 6 cm, then fi nd the
NCERT Exemplar

19. radius of t he circle with


cen tre A.
Prove that the tangents drawn at the ends of a chord
of a circle make equal angles with the chord .
NCERT Exemplar
2 3 . IHOfSJ AB is a diameter of a circle and A C is its
20. If a number of circles pa ss through the end poin ts P chord such that LBA C = 3 0 °. I f the tangent at
and Q of a line segment PQ, then show that their C intersects AB extended at D, then prove that
centre l ie on the perpendicular bisector of PQ. BC = BD.
NCERT Exemplar

� I Long Answer Type Questions [ 4 M a rks each ]

24. :H oTsl In a right MBC in which L B = 90 °, a circle is 26. !HOTS] In a figure the common tangen t s , AB a n d CD to
drawn with AB as diameter intersecting the two circles with centres 0 and a intersect a t E . Prove
hypotenuse AC at P. Prove that the tangent to the that the points 0, E and a a re col linear.
circle at P bisects BC.
A
NCERT Exemplar

25. �ofS) Prove that the tangent drawn at the mid -point
of an arc of a circle is parallel to the chord joining the
end poin ts of the arc. NCERT Exemplar
N CERT Exemplar

� l Value Based Questions (VBQs) (4 M arks eac h ]

2 7. A village Panchaya t 28. For an inaugration of the eco A


con structed a circular tank c friendly week in a DAV Pu blic
to serve as a bird bath. A School, badges were given to
fencing was made in the volunteers. Meena made
shape of a quadrila teral. these badges in the shape o f a
Sides of the quadrilateral triangle with a circle inscribed
in it. A message "supporting 8 L-- _...;:: �----.Jo c
-

D
tou ched the circle as shown _

in t he figure. If AB = 5 m, tree plantation" wa s writ ten


CD = 6 m and BC = 7 m, then in the circle. Suppose, BD = 6 cm, CD = 8 cm and
AE = 3 cm .
( i ) fi nd AD.
( i ) Find the leng t h s of AB a n d A C
( ii ) wh a t va lues docs vi l lage Pa nchaya t depict
t h ro u g h this action? ( ii ) W h a t values are shown by h av i n g s u c h week i n
t h e school?

Answers
1 . 5 0° 2. 55° 3. 6 cm 4. 50°
5. 40° 7 . 100° 8. 3 cm 9. 3 cm
1 o. ] 2 0° 1 1 . 30° 1 2. 9 cm 1 3. 10 cm

1 7. 40 cm 22. 2 27. (i) AD = 4m (ii) Care towards nature and love for creature.
28. (i) AB = 9 cm and AC = 1 1 cm (ii) Team work, importance of tree plantation, social responsibility.
CHALLE N G E RS*
1 . In the given figure. thre e circles with centres P, Q and R are drawn, such that the circles with centres Q and R touch each other
externally and they touch the circle with centre P, internally. If PQ = 1 O cm, PR = 8 cm and QR = 1 2 cm, then find the d iameter
of the largest circle .

2. If A. B and C are three points on a circle. The tangent C meets BA produced at T. Given that, LA TC = 36° and LACT = 48°.
Calculate the angle subtended by AB at the centre of the circle.
c

3 . In the given figure, I is the incentre of LlABC. Al, when produced, meets the circumcircle of Ll ABC. If LBAC = 66° and
LACB = 80°, then calculate LDBC, LIBC and LBID.

4. A is a point at a distance 1 3 cm from the centre O of a circle of radius 5 cm. AP and AO are the tangents to the circle at P and Q.
If a tangent BC is drawn at a point R lying on the minor arc PQ to intersect AP at B and AO at C , then find the perimeter o f t h e
MBC.
5 . In the given figure , diameters of two wheels have measures 4 cm and 2 cm . Determine the length of the belts AP and BC t h at
pass around the wheels , if it is given that belts cross each other at right angles.

6. Find the locus of centres of circles which tou ch two intersecting lines.
7. In the given figure , common tangents AB and CO of two circles with centres 0 and O ' i ntersect at E. Prove that the points
0, E and O' are collinear.

* These questions may be or m ay not be asked in the exa mination, have been g iven just for a ddition a l p ractice.
Note Solutions to these q u estions are available at page number 386-388.
CHAPTE R 4

CO N STRU CTI O N S

� Topic Checklist In the earlier classes, we have learnt bisecting an angle, drawing perpend icular bisector
of line segment etc. In this chapter, we shall study some more constructions using the
� Division of a Line Segment and knowledge of earlier constructions.
Construction of a Triangle

TO PI C 0
0 Division of a Line Segment
• Construction of a Triangle
Similar to a Given Triangle
Division of a Line Segment and
��· Construction of Tangents to
a Circle
Construction of a Tria ngle
" Construction of a Tangent to
a Circle at a Point lies on it Di.visio n of a Line Seg ment
• Construction of Tangents to a Here, we will divide a given line segment AB (say) in two segments AC and CB (say),
Circle from a Point Outside
such that C divides AB internally in the given ratio.
the Circle
• Construction of Tangents to a e.g. If AC : CB = 2 : 3, then C divides AB internally in the ratio 2 : 3 .
Circle when Angle between 2 3
Them is Given Here, A C = - AB and CB = - AB. 2 3
5 5 A------ B
" Construction of Tangents to a. c
Circle when Angle between a And if the ratio is 3 : 2, then 3 c 2 -- a
Tangent and a Line Passing 3 2
A ----
Through Centre is Given A C = - AB and CB = - AB.
5 5
Also, the j ustification of this division is given by using the basic p ro portionality
theorem and the concept of similar triangles. To divide a line segment in given ratio,
we have the following constructions.

Constru ctio n 1
To Divide a Line Seg ment I nterna lly in the G iven Rat io
We have to divide a line segment AB (say) internally in the given ratio m : n, where m
and n are both positive integers.
Then, we use the following steps
[Step I J Draw the given line segment AB and any ray AX, making an acute angle
with the line segment AB. This ray AX can be draw above o r below AB.
[Step I IJ Mark m + n = p points (A1 , A2 , • • • , A P ) on the ray AX such that
AA ) = A 1 A2 = ... = A - 1 A P
p
(Step I J IJ ] oin BA P .
Constructions

(Step IV] Th ro ugh t h l' p o i n t A 111 , d raw a line parallel to A P B Justification


( b y ma k i n g a n a n gle equal to LAA ,. B at Am ) .which (v) Since, A3C I I A5B.
i n tl' rsccr s t l1 L' l i n l' sL"gm c n r AB at point C. Thus, So, in MBA5' by basic proporcional i ry theo rem, we get
po i n t C d i v i d l' s r h l' l i n e segm e n t AB internally in


AA3 = A C
. . . (i)
r h l' ra t i o m : 11, i . L'. A C : CB = m : n. A3A5 BC
x (vz) By con s rruc r i o n, we have
AA 3 --
3
= = . . . (ii)
A3A S (5 - 3 ) 2
(vii) On equating Eqs. (i) and (ii) , we gee
Am .AC 3
1\ .
\ - =-
'

A,
, ,
\
'
BC 2
\

A
B
This shows char C divides AB incernally i n the ratio
c
3:2.
Note
Justification
[Step _� S i nc l' , 11,,, C 1 1 A l' B. so use the basic proportionality
According co basic proporc ionality theorem . if a l i n e i s
drawn parallel t o o n e side of a triangle in tersecting t h e
ch corl'm i n �A BA r . ocher cwo sides at distinct poi nts. then other t w o s ides a re
divided in the same ratio.
AA ,,_
Th e n ,
__, _ _ _
AC
. . . (i)
A,,, A r BC Alternate Method of Construction 1
To divide a line segment in the given ratio m : n, where
Step V Wri te t h e ra t i o by u s i n g construction.
m and n are both positive integers, we can also use the
---
AA111 m
= --- . . . (ii) following steps
A 111 A r (p - m ) Draw the given line segment AB (say) and any
[Step VI� Eq uate Eqs . ( i ) a nd ( i i ) to justify the construction ray AX m aking an acute angle with the l in e
(or d ivisio n ) . segment AB.
This cons c rucc i o n ca n b e understood with the help of Draw another ray BY I I AX by making
fo ll owi ng exam p l e .
LABY = LBAX .
Example 1. Dmw a line segment AB = 8 cm and (Step IIIJ Mark off m points A 1 , A , . . . , A,,, on AX
divide it internally in the ratio 3 2 and also justify it.
2
: and n points B 1 , B2 ' . . . , B,, on B Y such that
Sol. Steps of Co11stmctio11 AA, = A . A i = . . . = A m - I Am
(1) F i rst, d raw l i ne segment, A B = 8 cm and draw a ray AX = BB1 = B1 B2 = . . . = B,, _ 1 B,,
which m akes an acu te a n g le with l i ne
(ii) M a rk m + 11 3 + 2 = 5 points Al ' A.!' A." _ A4 and A5 on
=
segment AB.
[Step IYJ Join A,,, B,, which intersect AB at the point C.
the ray AX such t h a t Now, C is required point which divides AB
AAI = A, A! = A2 A.\ = A., A4 = A-1A5 internally i n the ratio m : n.
(iii) J o i n BA� .
(iv) T h ro u g h t h e poi nt A_" d raw a l i ne A3C II A5B (·: m = 3 )
which i n t e rsect t h e l i n e segment A B a t C.
T l rn s p o i n t C divides the line segment AB internally in
,

the ra tio 3 : 2 .
x

A £.._----�- B
c
---- B ern ---
\ 1 30 \ AllinOne MATH E MATI CS Class 1 0th Te rn Hl

Justification Construction 2
(Step v) Use the conditions of similarity of two triangles To Construct a Tria n g le Sim ila r to a G iven

6.AAm C and t:,.�BnC. Then, AA m = AC Tria n g le as per g iven Sca le facto r


BB,, BC I f a triangle co be constructed simi lar co a given triangle
. . . (i)

then it may be smaller or larger than chc given triangle So,


[Step VI) Write the ratio by using construction,
there are two different situations which a rc depend on the
.

AAm =m ( 11 ) scale factor which is defi n ed b el o w .


BBn
• • " "

n Scale factor The ratio of the sides o f the tri a ngl e co �


constructed with the co rresponding sides o f the given
(Step VII) Equating Eqs. (i) and (ii) , we get AC = m
BC n triangle is called the scale factor. G enerally, ic is wri tten as m
1l
This construction can be understood with the help of
which may be less than I or greater than l .
following example.
Thus, two different situations to constru c t a triangle,
Example 2. Draw a line segment AB = 6 .5 cm and depends on scale factor arise which a rc discussed below
divide it internally in the ratio 3 : 5 and justify the
then the sides o f t h e triangle to be
m
< I or m < n,
construction.
(t) I f
n

Sol Steps of Construction constructed will be smaller than the c o rrespondi ng


(1) Draw a line segment AB = 65 cm and a ray AX making sides of the given triangle.

1 or m > n, then the sides o f t h e triangle to be


an acute angle with the line segment AB. m
{ it) If >
(it) Draw another ray BY 11 AX such that LAB Y = LBAX. n
(iiz) Mark off 3 points A P A 2, A 3 (·: m = 3) on AX and 5 constructed will be larger than the corresponding sides
points B. , B2, B3, B4, B5 (·: n = 5 ) on B Y such that of the given triangle.
AA, = A1A2 = A2A3
= BB, = B1 B2 = B2B3 = B3B4 = B4B5 Cond ition I
(iv) Join A3B5 which i ntersect AB at point C. Thus, C divides
AB in che ratio 3 : 5, i .e. AC:CB = 3 :5. Construction of a Tria n g le S im ila r to G iven
m
x Triangle when < 1 or m < n
n

To construct a M' BC' similar to gi ven MBC when m <I


n

or m < n, we use the following steps


[Step I J Take BC
as base and draw the MBC of given
meas urement.

y
(Step II) Draw any ray BX
m aking an acute a n gle with BC
on the side opposite to the vertex A .
Justification [Step I I I) Find the values of m and n, then mark off n points
(v) By the condition of similarity of MCA.� and ll.BCB5,
AA3 AC (the greater of m and n in m ) o n BX such th ac
we have -- = - ... (i) n
BB5 BC

� B y construcnon
( v1,
. AA3 3
- =- ... (ii)
BB5 5
(viz) On equating Eqs. (i) and (ii) , we gee
AC �
=
BC 5
This shows that C divides the line segment AB internally in the
ratio 3 : S .
No te Two rriangles a re s i m i lar. if
• rhe i r corresponding angles are equal and
• rhe i r corresponding sides are in che same racio

(or p roporrional) and vice�versa.


Constructions
f 131 f
(Step IV) Join B,,C an<l d raw a line through Bm ( m being (iv) Join B4C and chen draw a l ine B3C' through B3 parallel t o
m B4C c o intersect B C a t C'.
smaller of m and n in ) parallel to BnC to (v) Now, draw a line through C' parallel to the l i ne CA to
n
intersect BA at A', i.e. C' A' II CA.
intersect BC at C '. Thus, Bm C 'l l B C. II Then, AA ' BC' i s the required triangle similar t o given
Draw a line through C' parallel to the line CA MBC.
which intersect BA at A ', thus
CA ' l l CA .
Then, M ' BC ' i s the required triangle similar to
m
given MBC as per scale factor, < 1.
n

Justification
(Step VI) Write the ratio of BC' and C' C by construction,
BC' m
t.e. x
C' C 11 - m
Justification
(Step VII) Now, BC BC' + C' C
- By construction, we have
BC' BC'
-- = -- = -
BC' 3 3
CC' C' C 4 - 3
= 1 + --
BC' BC
-- BC' + C' C C' C
Now, = = 1 + --
n -m n BC' BC' BC'
= 1 + -- = -
m m
1 4
=1 + - =-
3 3
Also, C'A' l l CA.
BC'
-- = - 3
So, M ' BC' .... MBC BC 4
A ' B A' C' BC' m Also, C' A' II CA, so M' BC' .... MBC.
-- = -- = -- = -
AB AC BC n A ' B A ' C' BC' 3
AB
-- = -- = -- =
Construction in this condition can be understood with the AC BC 4
help of following example.
Condi.ti.on 1 1
Example 3. Construct a triangle similar to given
Construction of a Tria ngle Si.mi.ta r to Gi.ven
MBC, where AB = 6cm, BC = 7 cm and AC = 8 cm, m
Tri.angle when > 1 or m > n
with its sides equal to � of the corresponding sides of n
4
MBC. Also, justify the construction. To construct a M' BC' similar to a given MBC when m > 1
Sol. Steps of Construction n
( z) Draw a line segment BC = 7 cm . Then, wich B as centre or m > n, we use the following steps
draw an arc of radius 6 cm and wich C as centre draw an
arc of radius 8 cm which cues che previous arc ac A.
(Step IJ Take BC as base and draw the MBC o f given
measurement.
Join AB and AC. Thus, MBC is obtained.
(ii) Draw a ray BX making an acute LCBX with ics base BC ( Step II ) Draw any ray BX making an acute angle with B C
on che side opposite co che vercex A. on the side opposite t o the vertex A.
(iiz) G.1ven sea I e raccor
c
-
.1s 3 = -.
m .
(Step II� Find the values of m and n, the n mark m points
(the greater of m and n i n m ) B1 , B 2 , , Bm o n
4 n
• • •

. . m = 3 and n = 4
n
Here, 3 < 4 BX such that
So, mark 4 points B1 , Bi• B3 , B4 on BX such char
BB1 = 81 Bi = BiB3 = BJ B4 BB. = B1 B2 = B2 B3 = . . . = Bm_ I Bm
\ 132 1 Allin.One MATH EMATICS Class 1 0th Term �

Join B,,C (n being smaller o f m and n in m ) and


(iv) Now, 3 being smaller of 5
(Step IV) n and 3 in �.
3
draw a line through Bm parallel to B,,C to intersect
So, join B_\ C. Then, draw
the extended line segment BC at C ' .
a line through B., parallel �----""'--<.,_____.�
to B3C to intersect 8
extended line segment BC
at C'.
(v) Draw a line through C'
parallel to CA intersecting
(
8 �---- the extended line segmen t
x
81 BA at A' .
82
83
Thus, M' BC' is the req uired triangle s i m i lar co given
8n 5
8m - 1
MBC as per scale factor -.
3
8m
x justification
llABC .... JlA ' BC'
(Step v) Draw a line through C' parallel to CA intersecting
the extended line segment BA at A'. ..
AB
-- =
A' B
-- --
AC
A' C'
=
BC
BC'
Then, M' BC' is the required triangle similar to
BC BB\ 3 BC' 5
per scale factor m
By construction, -- = --· = - =>
given MBC as > 1. BC' BB., 5 BC 3
n
Justification Hence, --
AB
= -- = -- = -
A' B A' C'
AC
BC'
BC
5
3
(Step VI) MBC - M' BC'

AB
Note If we construct M'BC' ,... MBC with sca l e fac c o r
m
-- = -- = --
AC BC
> l (i.e. m > n> . then M'BC' is a n e n l a rg e m e n c o f !!. ABC.
A' B A' C' BC' n

BC = BB,, = !!_ => BC' m • If we conscruct M'BC',... MBC with scale factor m < I
But .n
BC' BBm m BC n (i .e. m < n) , then M'BC' is reduction o f M8C.

Hence,
A' B
-- = -- = -- = -
A' C' BC' m Examp le 5 . Construct a triangle similar to given
AB AC BC n MBC whose sides are AB = 1 cm, B C = 3 cm and
Construction in this condition can be understood with the AC = 1 cm with its sides equal to i of the
help of following example. 3
corresponding sides of MBC.
Example 4 . Construct a triangle similar to given
MBC whose sides are 6 cm, 7 cm and 8 cm, with its Sol. Steps ofConstruction
(1) In llABC, AB = 1 cm , BC = 3 cm and A C = 1 cm .
sides equal to � of the corresponding sides of MBC. Take BC as base and then draw MBC .
3
(iz) Draw a ray BX making a n acute angle with BC on the
Sol. Steps of Construction side opposite to the vertex A .
(t) Draw a line segment BC = 7 cm. Then, with B as centre
draw an arc of radius 6 cm and with C as centre draw an
arc of radi us 8 cm which cuts the previous arc at point A .
Join AB a n d A C.
Thus, MBC obtained.
(ii) Draw a ray BX maki ng an acute angle with BC on the side
opposite to the vertex A.
m 5
(iit) Here, -;; = 3

:. m = 5, n = 3 and 5 > 3
S o , mark 5 points B1 , Bl' B3 , B,.. a n d B5 on BX such chat x
BB1 = B1 B2 = B2 B3 = B3 B4 = B4B5
Constructions f 1 33 I
m
:. m = -- =
AB AC BC
= 4, n = 3 and 4 > 3
4
(iiz) Herc,
1l
-=-
3
Therefore
,-
A'B A'C' BC'

mark 4 poinrs
B1 , B2, B3 and B-t on BX, such chat By construction,
BB1 = B, B2 = B2 B3 = B3B". BC BB3
= =�
(iv) Join B_,C, draw B4C' II B3C intersecting the extended line BC' BB4 4
segment BC a t C ' . BC' -4
=:) --
(v) Draw CA' II CA inrcrsccting the extended line segment =
BC 3
BA at A '.
A'B A'C'
Thus, M'BC' is chc required criangle. Hence, - = --
AB AC
]ustificatio11 BC' 4
(vi) Herc, fl.ABC - fl.A 'BC' [by M similarity] = -- = -
BC 3

I mporta nt Construdi.ons of a Tri.angle in Different Cases

Before constructing a triangle similar to given triangle say MBC, we have to construct the M.BC. For this, let us review
some constructions in different cases are given below
Thus, MBC is the required triangle.
Case I When three sides ofa triangle are given.
A
Let three sides of MBC be AB = c cm, BC = a cm
and CA = b cm are given. Then, we use the following
steps of construction.
� First, draw the base say BC =a cm.
( Step II) By taking B and C as centres, draw two arcs of radius
c cm and b cm, respectively which intersect each

(Step III)
other at A.
Join AB and A C. Q
Thus, M B C is the required triangle. Case III When two sides and angle between them
are given.
A
Let the two sides of M.BC be AB = a cm,
BC = b cm and angle between them,
i.e. LABC = x 0• Then, we use the following
steps of construction

� Draw base say AB = a cm.


a cm

Case I I When base and altitude ofa triangle are given. ( Stepll ) D raw a ray BX making an angle x 0 at B and

[Step III)
cut-off BC = b cm from BX
Let the base of MBC be BC = m cm and altitude is
n cm. Then, we use the following steps of Join AC.
construction Thus, MBC is the required triangle.
x
Draw the base BC = m cm and draw its perpendicular
bisector OQ (say) which intersects BC at P (say) .
By taking P as centre, draw an arc of radius n cm
which intersect the line segment PO or extended line
segment of PO at A .
�tep 111) Join AB and AC.
a cm
A ---- ----'-� B
134 \ Allin.One MATH EMATICS Class 1 0th Term I

Draw base AB = a cm .
Case I V When one side and 'tWo angles are given. �
Let one side of MBC be AB = a cm and angles (Step II) Draw two rays AX and B Y m aking an angle
LA = x 0 and LB = y0• x0 at A and y 0 at B, which in tersect each
Then, we use the following steps of other at C.
construction (Step III) Join AC and CB.
Thus, AA.BC is the req uired triangle.
Note In case IV, i f one side (say AB> . fi rsc a ngle (say A > and
second angle (say G) are give n . c h e n firsc we calculate
third angle (say B> . by using angle s u m propercy and che:i
use above seeps.

NCE RT FO L D E R 4.1
,,.
' I I

�· . , : '.
' �

' •
l'iJ ·'1.�"4·�·
• •

1 Draw a line segment of length 7 .6 cm and divide By construction, --


AA5 5
A5A1 3 = -8
it in the ratio 5 : 8. Measure the two parts.
Sol. Steps of Construction Hence, AO = -5
-
(t) Draw a line segment AB = 7.6 cm. OB 8
(ii) Draw a ray AX, making an acute LBAX with AB. or AO : OB = 5 : 8
(iii) Mark 5 + 8 = 1 3 points, i.e. A A 2 ' A 3 ,
I'
On measuring, we found chac
A .. . , A 1 2 , A 13 on AX, such that
4, AO = 2.9 cm
M 1 = A1A 2 = A 2 A 3 = ... = A1 2 A13 and OB = 4.7 cm
(iv) Join A 1 3B. 2 Construct a triangle of sides 4 c m , 5 cm and
(v) From A 5 , draw A 5 0 II A 13 B which intersect AB 6 cm and then a triangle similar to it whose
at 0 [ by making an angle at A5 equal to LAA 1 3B].
Then, 0 is the point on AB which divides it in the sides are �
of the corresponding sides of the
3
ratio 5 : 8. first triangle.
So, AO : OB = 5 : 8.
x Sol. Given, a /lABC, in which BC = 5 cm, AB = 4 cm and
AC = 6 cm.
Here, scale factor = � < 1.
3
Steps of Construction

Justification
In MBA 13, we have
A P II A1 3 B Draw a line segment BC = 5 cm.
AA
(t)
,I
i
' - = -- [ by b as1c
AO s
. propomona
. 1 ·icy t h eorem] (iz) Wich B as centre and radius equal to 4 cm, draw an
OB A 5 A 1 3 arc.
Constructions

( fit) Wich C as ccncrc and radius equal co 6 cm, draw Thus, llNBM is the required triangle whose sides are
?.._ of the corresponding sides of 6.ABC.
anochcr arc which inccrsccc chc previous arc ac A.
( iv) Join AB and AC. Thus, MBC is conscrucced. 5
(v) Now, draw a ray BX making an acuce LCBX wich
base BC on chc side opposirc co rhe verrex A (say 60°).
(vi) Along BX. mark three points B. , B2 and 83 such char
BB, = 81 B2 = B2By

(viii) From /J2 , draw B2N II B3C which inrersecc BC ac N


[ by making an angle ac 82 equal co LBB3C] .
(ix) From N, draw NM I CA which inrersecc AB at M
[by making an angle ac N equal co LBCA] .
Thus, � MBN is che required criangle, each of whose
sides is � of the corresponding
. sides of llABC.
3
justification
By conscruccion, B2N II B.,C
BN 2 2 NC I x
-- = -- = - :::::> - = -
NC 3 - 2 BN 2 Justification
BC BN + NC NC _!_ � By construction, B7M II B5C
Now, = =l + =I + = 5
-- = -- = -5 => -- = -2
BN BN BN 2 2 BC CM
BN -2 CM 7 - 5 2 BC 5
i.e. -= BM BC + CM
BC 3 Now, -- = ----
. Also, NM 11 CA , so we have !1MBN - MBC. BC BC
CM BM 7
MB NM BN 2 = l + -- = l + -2 = -
7
=> -- = -
Hence, -- = -- = - = - BC 5 5 BC 5
AB CA BC 3
Also, MN ll CA
3 Construct a triangle with sides 5 cm, 6 cm and llNBM - 6.ABC
7 cm and then another triangle whose sides are NB BM MN 7
Hence, - = -- = -- = -
?_ of the corresponding sides of the first triangle. AB BC CA 5
5 CCE 2008 C
4 Construct an isosceles triangle whose base is
Sol. Given, a MBC, in which BC = 5 cm, CA = 6 cm and 8 cm and altitude 4 cm and then anothe r
AB = 7 cm. Here, scale faccor = - > l.
7
5 triangle whose sides are 1 ..! times the
2
Steps of Construction corresponding sides of the isosceles triangle.
(z)Draw a line segment BC = 5 cm. Sol. Given, a MBC, in which base
{iz) Taking B and C as centres, draw two arcs of radii 7 cm BC = 8 cm and altitude = 4 cm .
and 6 cm, respectively co incersecc each ocher at A.
Here, scale factor = 1 - = - > I
I 3
(iit) Join BA and CA.Thus, MBC is the given triangle. 2 2
(iv) Now, from B, draw any ray BX making an acute Steps of Construction
LCBX with base BC on the side opposite co the (1) Draw a line segment BC = 8 cm.
vertex A. (iz) Draw perpendicular bisector of BC which incerseccs BC ;.:.: ""
(v) Mark seven points B1 , B2, B3, B,p B5, B6 and B7 on at P. .
BX, such chat (iit) Along PO, cue-off PA = 4 cm.
BB1 = 81 B2 = B2B3 = B3B4 = B4B5 = B5B6 = B6B7• (iv) Join BA and CA. Thus, MBC is the given isosceles
triangle.
(vz) Join B5C and from B7, draw a line B7M I B5C
(v) Now> from B, draw any ray BX making an acute
intersecting the extended line segment BC at M. LCBX on the side o pposite to the vertex A.
( viz) From point M, draw MN II CA intersecting the (vz) Mark three points B, , 82 , and B .1 on BX such chat
extended line segment BA at N. BB I = B , B :! = B '!.B .I
136 \ AllinOlze MATH EMATICS Class 1 0th Term 1i

(vii) Join B2C and from B 3, draw a line B 3N I I B2C (v) Mark four points B1 , B B and B4 on AX such that
2, 3

intersecting the extended line segment BC at N. AB I = Bl B 2 = B 2 B = B B


3 3 -1 .

(viii) From point N, draw NM 1 1 CA inters�cting the (vi) Join B4B and from B ·" draw B _,M I I B.1 B intersecting
extended line segment BA at M. AB at M.

B N

Thus, �BN is the required triangle.


Justification (viz) From point M, draw MN I I BC intersecting AC at
By construction, B 3N ll B 2C N.
Thus, MMN is the required triangle whose sides arc
=>
BC
- = 2 = -2
--

i of the corresponding sides of MBC.


CN 3 -2
CN 1
4
=> - - -

BC 2 Justification

Now, BN BC + CN
- = = CN
l+- By construction, B3M II B4B
BC
BN
BC
1 3
BC AM
-- = 3 = -3
MB 4 -3
--

=> - = l + -2 = -2
BC MB 1
Also, NM l l CA => -- =-
AM 3
=> dMBN - MBC
AB AM + MB
Hence, MB NM BN 3
- = -- = - =- Now, -- = AM
AM
AB CA BC 2
Thus, the new triangle is similar to the given triangle whose MB
= l + -- = l + -31 = -34
sides are �.2 i.e. l .!_2 times of the corresponding sides of the AM
AM 3
isosceles MBC. => -- =-
AB 4
5 Draw a MBC with sides BC = 6 cm, AB = 5 cm Also, NM l l CB
and LABC = 60°. Then, construct a triangle !iAMN - MBC
whose sides are � of the corresponding sides of AM AN MN 3
4 Hence, -- = -- = -- = -
the MBC. CCE 2008 AB AC BC 4
Sol. Given, a MBC, in which BC = 6 cm, AB = 5 cm and 6 Draw a MBC with side B C = 7 cm, LB = 4 5 °
J LA.BC = 60°. Here, scale factor = i < 1. andLA = 105°. Then, construct a triangle
4 whose sides are i times the correspondin g
J Steps of Construction 3
(t) Draw a line segment AB = 5 cm. sides of MBC.
(iz) From point B, draw LABY = 60° and cut-off Sol. Given, llABC, in which BC = 7 cm
a
BC = 6 cm from BY.
L B = 45°, LA = 105 °.
(iiz) Join A C. Thus, MBC is the given triangle. LC = 180° - ( LA + LB )
(iv) Now, from A, draw any ray AX making an acute = 180° - (105° + 45°)
L BAX with base AB on the side opposite to the
vertex A . = 180° - 1 50° = 30°
Constructions f 137
Herc, scale factor = 4- >1 Sol. Given, a righc angled MBC, i n which B C = 4 cm and
3 AB =3 cm.
5
Here, scale faccor = > 1 -

I 3
I
I
I

M
�!c=::==:li:::===� C '
B
B1
82
83
84
Bs
Steps of Co11stn1ctio11
(1) Draw a line segmcnr BC = 7 cm.
(ii) M ake an angle of 45° ac poinr B and 30° ac poinc C,
Steps ofConstruction
(z) Draw a line segmenc BC = 4 cm.
which inrersccc each ocher ac poinc A. Thus, MBC is
c he given criangle. {iz) Make an angle of90° ac poinc B and cue-off
(iii) Now, from B, draw a ray BY by making an acuce AB =3 cm from ic.
L.CBY wich base BC on che side opposice co vercex A. (iii) Join AC. Thus, MBC is che given righc angled
(iv) Mark four poinrs B " B 2, B J and B 4 on BY such criangle.
thac BB 1 = B , B 2 = B 2 B _, = B JB 4• (iv) Now, from B, draw a ray BY making an acuce L CB Y
(v) J oin B_,C. on che side opposice co che vercex A .
(v1) From B.p draw B4M II B ,C incerseccing che excended
·
(v) Mark five poincs B, , B2 , B3, B4 and B5 o n B Y such
line BC ac M . chac BB, = B1 B2 = B2B3 = B JB 4 = B4B 5 •
(vii) From M , draw OM I A C incerseccing che excended (vz) Join B JC.
line BA ac 0 . (viz) From poinc B5, draw B5C' II B J C incerseccing che
excended line segmenc BC ac C'.

Thus, /J.OBM is che required criangle whose sides are
(viii) From poinc C', draw C' A' II CA incerseccing the
rimes of che corresponding sides of �ABC. excended line segmenc BA ac A ' .
Thus , M' BC' is che required criangle.
Justification Justification
By conscruccion, B4 M II B J C By conscruccion, B5C' II B _,c
BC 3 3 BC 3 3
--= --
=-
CM 4 -3 -- = = -- -

CC' 5 -3 2
=>
CM 1 CC' 2
=>
-- =- =- --
BC 3 BC 3
BM BC + CM CM 1 4 BC' BC + CC'
Now, =1 + --
=1 + - =- Now, -- = ----
BC BC BC 3 3 BC BC
Also, MO l l CA CC'
=1 + -­
.. /J. OBM - MBC BC
OB BM MO 4 2 5
Hence, = --
= =- --

AB BC CA 3 =1 + - =-
3 3
7 Draw a right angled triangle, in which the sides Also, C'A ' II CA
( other than hypotenuse) are of lengths 4 cm and M'BC' - MBC
3 cm. Then, construct another triangle whose A' B BC'
Hence,
sides are � times the corresponding sides of AB BC
3 A' C' 5
= -- =
given triangle. AC 3
TO P I C EXERCISE
::r�:::-:.=-�.'._==..=-�. ..-��---;-..,-��·.·=-�:·_;�-=-�=-- =���--:_· - -���---=�--�-- �- -�---- ·----�-- . __

"
L·�-=-�._":'.'"--:'�;t...= :· ..:..:. _"::_:.-:.· ·=:-;·:.-=::'!"'"..·--.:.�--: .-_::- -:......- �.":':.:�..-:·.--::,· :-- . ·":..
:·:.;=---="'°.'"C.7lo.O:C:-:-:-;�...,..::.-.:..i.:.-.=- .. I• -•
• •

Very Short Answer Type Questions 7 Draw an equ ilateral MBC of each side 4 c m .
1 To d iv ide the l ine segment AB i n the ratio 6 : 7 . draw a
Construct a triangle similar t o i t a nd o f sca le factor
�. Is the new triangle also an equ i la t e ra l ?
NCERT Exemplar
ray AX such that LBAX is an acute angle, t hen draw
5
a ray BY parallel to AX and the point A1 , A , A 3 ,.. and
2
B1 , B , B3 . ... are located at equal distances on ray AX
2 8 Construct a 6ABC sim ila r to a given isosceles D.PQR
and BY, respectively. Then which points should be
with QR = 6 cm. PR = PQ = 5 c m . such that each of
joined?
its sides is �th of the correspond ing sides of LlPQR.
2 In t he following figure, find the ratio in which 0 7
divides PQ i nternally.
Long Answer Type Questio n s
x
9 Construct. a triangle similar t o a given LlABC such
that each of its sides is � rd of the corresponding
3
sides of 6ABC. It is given that AB = 4 c m . BC = 5 cm
and AC = 6 cm. CCE 2008 c

10 Construct a 6ABC. in which AB = 6 c m .


y
LB = 60° and L A = 45°. Construct a LlPQR similar to
Short Answer Type I Questions 6ABC such that each side of 6PQR is 1 . 5 times that
of the corresponding sides of 6ABC.
3 Given a line segment PQ and divide it in the ratio 1:3.
11 Draw a 6ABC with side BC = 7 c m . LB = 45° and
4 Divide a line segment of length 10 cm in the ratio
6 : 5.
LA = 105 °. Then, construct a nother tria ngle whose
side.s are � times the corresponding sides of 6ABC.
Short Answer Type I I Questions 4
CCE 201 1
5 Divide a line segment of length 9 . 6 cm in the ratio 5:3. 12 Draw a right angled triangle, in which sides (other
Measure the two parts and given justification.

j
than hypotenuse) are of length s 8 cm and 6 cm.
6 Draw a line segment of length 7.7 cm and divide it in Then, construct another triangle whose sides are
the ratio 3 : 4. Measure the two parts and justify the
construction. times the corresponding sides of the given triangle.

Answers
1 . A6 and B, 2. 6 : 5 5. 6 cm, 3.6 cm 6. 3.3 cm, 4.4 cm 7. Yes
TO PI C 0 Co nstruction of Tangents to a Circle
A tangenc co a circle is a straigh t line wh ich couches the circle Example 1. Draw a circle of diameter AB = 6 cm with
at a poinc. This poi n t is cal led the point of contact and the centre 0 and then draw a tangent to the circle a t point
radius through the point of con tact is perpendicular co the A or B.
tangent. Sol. Given, diameter of circle = AB = 6 cm and centre is 0.
The number of tange nt s d rawn co a ci rcle from a point
:. Radius = OA = OB = �
depends on the position of the point with respect to circle. 2
( z) If a point l ies i nside the circle, then no tangent can = 3 cm
be drawn. Steps ofConstruction
(ii) If a point lies on the circle, then only one tangent at (1) Take a point 0 as centre and draw a circle of radius 3 cm.
this poi nt can be drawn. (it) Draw diameter AOB and cake a point A on the circle.
(iit) If a point lies outside the circle, then two tangents (iit) Take OA as base and construct LOA T = 90 ° at A.
from this poi nt ca n be drawn . T
Now, construction of tangents to a circle i n different cases
are given below

Constructio n of a Ta n gent B
to a Circle at a Point lies on it
We can construct a tangent to a circle at a point lies on it by
T'
two methods which are given below
(iv) Produce TA co T' co gee che required tangent TA T'.
Similarly, we can draw a tangent at point B or any point
Method I By using the centre ofcircle on the circle.
To construct a tangent co a circle by using the
centre we use che following steps. Method II Without using the centre ofcircle
Take a point 0 as centre and draw a circle of To construct a tangent to a circle without using the
given radius. centre of circle. We use the following steps.
Take a point P on the circle, at which we want to Draw a circle of given radius r cm and take a point
draw tangent. P (at which we want to draw tangent) o n the
circle. We use the following steps.

�tep II ) Draw any chord PQ


through the given point P Q
on the circle.

(Step 1 1 1) Join P and Q to a point R x


on the circle either in the
(Step I I� Join OP, which is the radius of circle. major arc or i n the minor arc.
(Step IV) Taking PQ as base, construct LQPY equal to
(Step IVJ Take OP as base and construct LOPT = 90° at P. LPRQ and on the opposite side of R. Produce
�tep VJ Produce TP to T ' to get the required tangent YP to X to get the required tangent YPX.
TPT'.
' �

i 140 j Allin.One MATH EMATICS C la s s 1 0th Te rm JI

Exam p le 2. Draw a circle of radius 5 cm. Take a Justification


point P on it. With o u t using the centre of the (v) Join OQ. Then , LPQO = 90°, s i n ce i r i s rn n s r ru c red i n the
semi-circle. Ir shows chat OQ 1- PQ .
circle, draw a tangen t to the circle at point P.
Sol. Given , radi us of circle = 5 cm Also, OQ is radius of given ci rcle, so PQ h a s to be a tangent
of given circle. Similarly, PQ ' is also a r a n ge n r ro rhc g i v en
circle.
Note The tangent to a ci rcle is perpencl i c u l ;l r 10 ! I H' r . 1 c l 1 1 1 --; 1 1 l r ough rh
Q poi nt of contact . I ts conve rse is also t rue .
x Example 3. Draw a circle of radius 1 . 0 0 c m . Fr o m a
point P, 2 . 2 cm apart from the cen tre of th e circle, dra w
tangents to the circle. Also, justify the con s t ru ction .
y Sol. Given, radius of c i rc le = 1 .0 0 cm and d i sr a n n.· hcl WL'L' l l p o i n r P and
centre = 2 .2 cm.
Steps of Construction
(i) D raw a circle of radius 5 c m and cake a point P on
Steps of Co11structio11
the circle. (i) Draw a circle of ra d i us I cm wirh c e n r rL' 0.
(ii) Draw a chord PQ through the point P on che (ii) Take a poinc P o u cs ide ic , such char i rs d i s c a n n.· fro m cenr re 0 is
circle. 2.2 cm.
(iii) Take a point R in the major arc and join PR and (iii) Take 0 and P as centre and
RQ. draw arcs of radius more rhan
(iv) Taking PQ as base, construct LQPY equal co
half of OP on boch sides of OP
LPRQ on che opposite side of the point R. which intersecc each ocher ar R
(v) Produce YP co X. Then, YPX is che required
and S. Join RS which bisecc OP
tangent at point P.
ac M. Then, MP = MO
(iv) Taking M as cencrc an d MO as
radius, draw a dorced ci rcle
Construction of Tangents to a which incerseccs given circle ac
Circle from a Point Outside Q and Q'.
(v) Join PQ and PQ'. Thus we gee rhc req u i red ra ngL'll rs draw n
from poinc P co che given ci rc l e .
the Circle
If a point lies outside the circle, then there will be two Justification
tangents to the circle from this point. These tangents Join OQ. Then, LPQO is an angle in c h e sem i-ci rc l e and rhcrcfore
can be drawn in two cases which are given below LPQO = 90°.
Case I When centre ofcircle is known. Since, OQ is che radius of given ci rcle, so PQ ha s ro be a c a n gc nt of
given circle.
If centre of circle is known, then to draw tangent from Similarly, PQ' is also a cangenc co rhe g i ve n c i rc l e .
a given external point, we use the following steps of
construction. Example 4. Construct a tangen t to a circle of radius 1 . 8
(i) Draw a circle with centre 0 and take a point P cm from a point on the concen tric circle of radius 2 . 8 cm
outside it. and measure its length . Also, verify the m eas urem ent by
(ii) Join OP and bisect it. Let its mid-point be M. actual calculation.
Then , MP = MO. Sol. Given, two concencric circles of radi i 2.8 c m
(iii) Taking M as centre and 1 .8 cm wich common cencrc say, 0.

,
Steps of Construction
,, (1) Draw cwo circles wich common ccncre p -_,�,_.
and MO or MP as
radius, draw a dotted I

I
I 0 and radii 2.8 cm and 1 . 8 cm.
(ii) Take a poinc P on chc oucer c i rc l e and
circle, which
in tersects the given p ....."---
I

. -----<1....-
.. --
1
I join OP.
(iii) Draw bisector of OP. Lee mid-poinr of OP be M.
\
circle at Q and
\,' , ,
(iv) Taking M as cencrc and PM as radius, draw a Jocred cirdt'.
Q ' (say) .
(iv) Join PQ and PQ '. which incerseccs che inner circle ac cwo point s say A and B.
Thus, P Q and PQ ' are the required (v) Join AP and BP.
tangen ts drawn to the circle from the external Then, AP and BP are required cangencs . On me as ur i n g lhc lengths,
point P. Here, we observe that PQ = PQ ' . we get PA = PB = 2. 1 4 c m
Constructions f 1 41 f
Calculatio11 Justification
Join OA. Then, OA = 1 .8 cm [radius of inner circle C,] Join OT. Then, LPTO = 90°
OP = 2.8 cm [radius of outer circle C2 ]
[angle in semi-circle of dotted circle]
This shows that OT .l P T.
and LPA O = 90°
[·: angle in semi-circle of constructed circle] Also, OTis radius of given circle, so PThas co be a tangent of given
So, in llPAO, by Pythagoras theorem, circle. Similarly, PT' is also a tangent of given circle.
2
OP 2 = OA + AP 2 => ( 2.8) 2 = (1 .8) 2 + AP 2
Alternate Method
=> 7.84 = 3.24 + AP 2 => AP 2 = 7.84 - 3.24 = 4.6
If centre of circle is unknown, then we can draw tangents
AP = 2. 1 4 cm => PA = PB = 2. 1 4 cm without finding centre of the circle. For this, we use the
Hence, the length of tangents is 2. 1 4 cm. following steps of construction
(z) Draw a circle ofgiven radius and take a point P o utside i t.
Case II When centre ofcircle is unknown.
D--r---
'
..
If centre of the circle is unknown, then co draw ..

tangents to the circle, first we find the centre of ,, ,,


,
�...-
�==
I
I

-����--t- --
- r-

, �-_;J_/:,
the circle by using these two steps.
First, draw the circle and then draw two

\�'
non-parallel chords of the circle.
(Step II ) Draw the perpendicular bisectors of both -

chords which intersect each other at a point, (it) Through P, draw a line (i.e. secant) intersecting the given
say 0. Then, this point 0 gives the centre of circle at points A and B, respectively and produce it to C
given circle. in opposite direction of AB s uch that AP = CP.
(iii) Now, bisect the segment CB. Let its m id-point be K.
Then, take K as centre and KB (or KC) as radius, draw a
Now, we use the steps given in case I to draw
tangents.
semi-circle.
Example 5. Draw a circle with the help of circular (iv) At point P, draw PD 1- CB which cuts the semi-circle
solid ring. Construct a pair of tangents from a point at D.
P outside the circle. (v) Take P as centre and PD as radi us and draw a dotted circle
Sol. Steps of Constr11ctio11 which intersects the given circle at points T1 and T2 •
(z) First, draw a circle with the help of given circular (vz) Join PT1 and PT2 which are the required tangents.
solid ring and then draw two non-parallel chords AB
and CD of the circle. Example 6. Draw a circle of radius 2 . 8 c m . From an
A external poin t P, draw tangents to the circle with o u t u sing
,,,, ..
the centre of the circle.
D
I

I
I
I
Sol. Given, a circle of radius 2.8 cm and we have co draw tangents
p ....
. ,...-
.- --+-i----
without using the centre.
B
1
I
I Steps of Construction
\,,
....
(1) First, draw a circle of radius 2.8 cm and take a poin t P
c outside the circle.
D
(iz) Draw perpendicular bisectors of AB and CD which
intersect each ocher at point 0. Then, 0 is the centre
of the circle.
(iit) Now, cake a point P outside the circle and join OP.
(iv) Draw bisector of OP. Lee its mid-point be M.
(v) Taking M as centre and MP as radius, draw a dotted
circle which intersect the given circle at T and T'.
(vi) Join PT and PT'. (ir) Through P. draw a secant PAB which intersects the circle at A
Then, PT and PT' are the required pair of tangents drawn and B and extend it co C in opposite direction of AB such
to the circle from P. that PC = PA .
' 1 42 ' AllinOne MATH EMATICS Class 10th Term

(iii) Now, b isect BC and take its mid-poin t as 0. Draw a Also, LAOR = 1 80 ° - L QOR [·: A OQ i s a srraighc line]
semi-circle with centre 0 and radius OB (or OC ) .
(iv) Draw PD .L B C which i ntersects the semi-circle a t D. = 1 80 ° - a
(v) Draw a dotted circle with centre P and radius PD which Now, in quadrilateral A ORP,
intersects the given circle at points M and N.
(vt) Join PM and PN Thus, PM and PN are the required LAPR + LPA O + L A OR + L PRO = 360 °
tangents to the given circle.
=> LAPR + 90 ° + 1 80 ° - a + 90 ° = 360 °
Construction of Tangents to a Circle L A PR = a
when Angle between Them is Given
Sometimes, angle between two tangents {or pair of tangents) Example 7. Draw a pair of tan gen ts to a circle of
is given and we have to draw these tangents. Then, we use radius 3 cm which are inclin ed to each other at an
the following steps of construction angle of 45°.
Sol. Given, a circle of radius 3 cm. We h ave w co n s t ruct a pai r of
(Step I ) F irst, draw the given circle with centre 0 and
tangents which are inclined co each o t h e r at an angle of 60°.
radius r em.
Steps of Construction
(1) Draw a circle wich 0 as ce nt re and ra d i u s 3 c m .
(ir) Draw any diameter POQ of chis c i rcle.
(iit) Draw che radius OR meets ch e c i rc l e at R s u c h chat
LQOR = 45 °.

(Step I I) Draw any diameter say AOQ of this circle.


(Step 1 1 1) Make given angle a. at centre 0 with OQ (say) as
E
base which intersect the circle at point R (say) or
draw the radius OR meets the circle at R such that D
LQOR = a . (iv) Draw PD .1 PQ and RE 1- OR.
which intersect each ocher at poi n t N .
[Step r\Jj Now, draw perpendiculars to OA at A and to OR Then, NP and NR arc the required tangents co the given
at R, which intersect each other at a point say P. circle inclined co each ocher at a n ang l e o f 4 5°.
Then, AP and RP are the required pair of tangents to given ]ttstification
circle, inclined at an angle a., i.e. angle between pair of By conscruccion, LOPN = 90 ° and OP is radius.
tangents is a. . : . PN is a cangenc co the circle.

Justification
Similarly, NR is a tangent co the circle.
Now, LPOR = 1 80° - 4 5 ° = 1 35 °
By construction, LOAP = 90 ° and QA is radius. So, PA is a [·: POQ is a scraight line and L QOR = 4 5 °)
tangent to the circle. In quadrilateral OPNR,
LOPN = 90°, LPOR = 1 35 ° and L ORN = 90°
Similarly, PR is a tangent to the circle. LPNR = 360° - (90 ° + 1 35 ° + 90 ° ) = 4 5 °
FO L D E R 4 . 2
1 Draw a circle of radius 6 cm. From a point 10 cm Steps of Co11structio11
away from its centre, construct the pair of (1) Draw cwo concentric circles C1 and C2 with common
tangents to the circle and measure their lengths. centre 0 and radi i 4 cm and 6 cm, respectively.
Given, a circle of radius 6 cm whose centre is 0 (say) and
(ir) Take any poinc P on oucer circle C2 and join OP.
Sol. (iii) Draw the biseccor of OP, which bisect OP at M ' .
a point P, I 0 cm away from its centre.
(iv) Taking M ' a s centre and OM' a s radius, draw a dotted
Steps of Co11str11ctio11
circle which curs che inner circle C1 ac rwo points M
(1) Draw a circle with 0 as centre and radius is equal ro and P'.
6 cm. Take a poinr P such char OP = I O cm
(v) Join PM and PP'. Thus, PM and PP' arc required
(ii) Draw chc bisccror of OP which incersecc OP at M. cangcncs. On measuring PM and PP',
wc gee
PM = PP' = 4.47cm
Calculatio11
In right angled tiOMP, L PMO = 90°
PM 2 = OP '!. - OM 2 [by Pythagoras theorem]
=> PM 2 = (6) 2 - (4 ) 2
= 36 - 1 6
= 20
(iii) Taking M as centre and MO as radius, draw a dotted => PM = J20
circle. Lee ch is circle cues the given circle at A and B.
(iv) Join PA and PB. = 4.47
Hence, che length of tangent is 4.47 cm.
Thus, PA and PB arc rhe required rangencs. By
measuremcnc (using scale), PA = PB = 8 cm. ]11stificatio11
]11.stificatio11 Join OM
Join OA and OB. Then, LOMP is an angle lies i n che semi-circle.
Then, L OAP is an angle in che semi-circle. Therefore, L OMP = 90 °
Therefore, L OAP = 90° => OM ..L MP
=> OA ..L AP Since, OM is radius of the circle, so MP has to be a cangenr
Also, OA is radius of che circle, so AP has co be a cangenc co ro che circle. Similarly, PP' is also a tangent to the circle.
rhe circle. Similarly, BP is also a cangent to the circle.
3 Draw a circle of radius 3 cm. Take two points P
2 Construct a tangent to a circle of radius 4 cm · and Q on one of its extended diameter each at a
from a point on the concentric circle of radius distance of 7 cm from its centre. D raw tangents
6 cm and measure its length. Also, verify the to the circle from these two points P and Q.
measurement by actual calculation. Sol. Given, rwo points P and Q on the excended diameter of a
Sol. Given, cwo concentric circles of radii 4 cm and 6 cm wich circle wich radius 3 cm such thac OP = OQ = 7 cm
common centre 0. We have to construct the cangents co the circle from the
Here, we have co draw two tangents co inner circle C1 from a given points P and Q.
poinc on che oucer circle C2 • I I
I I

_ _ ..L _ _ _ _ ..L _ _
.,,,- : - .... ... .... M p � , .... - : .... ... .... ,

I
,'/ , 7r' ... �_...._ , I!' ... ... ... \
\
I \
I I
I I

------- - ,:o
I I
\ I
\ I
\ I
' /

'' ,____ __., � ,/


... .... .... ... .... _ � _ _ _ ,. ,. , "' N O' .... .... .... .... _ _ _ : _ _ _ , , ,"

I I
I I
\ 144\ AllinOne MATHEMATI CS C la s s 1 0t h Term

Steps of Construction Now, LPOR = 1 80° - 60 ° = 1 2 °


(i) Draw a circle of radius 3 cm with centre at 0. [·: POQ i a r raight l i ne and L. QOR =

(ii) Produce its diameter on both sides and take points P In quadrila te ral OPNR
and Q on this diameter such that LOPN = 90°,
OP = OQ = 7 cm LPOR = 1 20°
(ii:) Draw bisector of OP and OQ which intersect OP and and L ORN = 90°
OQ a t E an d F, respectively. LPNR = 360° - (90° + 1 20 ° + 90 ° ) = 60 °
(iv) N ow, take E as centre and OE as radius, draw a
dotted circle which intersects the given circle at two 5 Draw a line segment AB f length 8 c m . Takin�
points M , N. Again, take F as centre and OF as A as ce n t re , draw a cir 1 o f rad i u s 4 m an
radius, draw another dotted circle which intersects taking B as centre, d raw another ir 1 of radi ,
the given circle at two points P' and Q'. 3 cm. Construct tangents to a h c i rcle from th
(v) Join PM , PN, QP' and QQ'. These are the required centre o f the other circle.
tangents from P and Q to the given circle.
Sol. Given, a line segment AB = 8 cm , two c i rcle with c nt ·

Justification
Join OM and ON. The LOMP is the angle lies in the A and B of radii 4 cm and 3 cm, re pectively.
semi-circle. Therefore, LOMP = 90° We have to construct two tangent t each ci rcl from
=> OM .L PM centre of the ocher circle.
Steps of Construction
(1) Draw a line segment AB = 8 c m .
Since, OM is radius of the circle. So, MP has to be a tangent
Draw a circle with centre A and radiu 4 c m
to the circle. Similarly PN, QP' and QQ' are also tangents to
(it}
another circle with centre B and radius 3 c m .
the given circle.
4 Draw a circle of radius 5 cm. Construct a pair of (iit} Now, bisect AB. Let 0 be t h e m id-p o i nt o f AB .
tangents on it, so that they are inclined at 60 °. (iv) Ta ke 0 as centre a n d A O a s radius a nd d raw a do
Measure the lengths of the two tangents . circle which i ntersects the two given circl
CCE 2015 N, Q, M and P.
Or
.. ...... - - - - I� - - - - .... .... ..
,.... ',, '

(
Draw a pair of tangents to a circle of radius

"�°"'
5 cm which are inclined to each other at an
\' N
M'
angle of 60° . CBSE 201 1
-----�
l
'
?!'\
Sol. Given, a circle of radius 5 cm. We have to construct a pair
of tangents which are inclined to each other at an angle
of 60°.
Steps of Construction
(t}
Draw a circle with 0 as centre and radius 5 cm.
(it}
Draw any diameter POQ of this circle.
(iit) Draw the radius OR meets the circle at R such that
LQOR = 60°.
(iv) Draw PD .L PQ and RE .l OR.
(v) Join AN, AQ, BM and BP. These a re the requ_i
Let PD and RE tangents to each circle from the cen t re of the oth
intersect each circle.
other at point N.
Then, NP and NR Justification
are the required Join AM and AP.
tangents to the Then, LAMB is an angle lying in sem i-circle. Therefo ,
given circle E --+-..i.._-
_ ---==-'--""-- LAMB = 90°
inclined to each AM .L BM
other at an angle Since, AM is radius of the circle with cencre A . So, BM h
of 60°. to be tangent of the circle with cencre A . S i mi l a rly, BP i nl
Also, the measure of lengths of two tangents are a tangent of the circle with centre A .
8.65 cm each. Again, join BN and BQ.
justification Then, LANB is an angle lyi ng in semi-circle
By construction, L OPN = 90° and OP is radius.
: . PN is a tangent to the circle.
LANE = 90° and BN is radius of the circle with c ntr
So, AN has to be tangent of the circle with centr
Similarly, NR is a tangent to the circle. S i mil arly , A Q is also a tangent of the circle with centr
Constructions J 1 4s I
6 Let ABC be a right angled triangle, in which 7 Draw a circle with the help of a bangle. Take a
AB = 6 cm, B C = 8 cm and L B = 90°. BD is the point outside circle. Construct the pair o f
perpendicu l a r from B o n AC. The circle through tangents from this point t o circle .
B, C and D is d rawn. Construct the tangents from Sol. Given, a circle drawn by using a bangle and a poin t R
A to this c i rc l e . oucside the circle.
Sol. G iven , ABC is a righ r angled rriangle, in which AB = 6 cm , Steps ofConstruction
BC = 8 cm, L 13 =
90° a nd BD is perpend i cu l ar co AC. (t) Draw a circle by using a bangle.
Then, LADB = = 90° (ii) Draw two chords AP and MTof the circle.
(iit) Draw perpendicular bisectors of AP and MT
L CDB
Steps of Co11structio11
(I) Draw the l i ne segmenrs AB =
which intersect each other at 0, which is the centre
6 cm an d BC = 8 cm
perpendicular ro each orher. J o i n AC. Thus, MBC is of the circle.
the given righ r a ngled triangle.

(ir) Take m i d-poi nr F of BC as


cen rre and d raw a circle
with rad i us BF (or CF) ,
pass i ng rhrough 13, C and
D.
(iir) Now, join AF a nd bisect ic.
Lee m id-po i n c of AF be 0.
(iv) Take a point R outside the circle. Join OR and bisect
(iv) Take 0 as cenrre a n d OA as it.
(v) Let Q be the mid-point of OR. Taki ng Q as centre
rad i us a n d d raw a dotred
circle wh ich i nr ersecrs the
and OQ as radius, draw a dotted circle which
given c i rcle (which passes
through B, C a n d D) at B and M. intersects the given circle at S and N.
(v) Now, j o i n A B and AM, which are r he requ i red (vt) Join RS and RN Thus, RS and RN are the required
.

tangenrs. tangents from R.


Justification
}ustificatio11 Join OS and ON.
Join FM and F/3.
Then, L OSR is an angle lying in the semi-circle.
Then, L AMF is a n angle lying in semi-circle.
Therefore, LOSR = 90 °
OS ..L SR
Therefore, LAMF = 90° =>
=> FM ..L AM
Since, FM is rad ius of che given circle. So, AM has co be
Since, OS is the radius of the circle with centre 0. So, SR
has co be a tangent co the circle with centre 0. Similarly, RN
cangenc of che circle wich cencre F. Similarly, AB is also a
is also a tangent to the circle with centre 0.
rangenc ro che c i rcle wich centre F.
TO P I C EXERCISE . 7 -· r---- - - - :- -... --� - - -. ""' -
' . . . -
- - -- - · - - ·

. - : _ ·!,; . ..,A. - ·� · - : · : _ . · .-• . • - -

Very Short Answer Typ e Quest ions Short Answer Type I I Questio n s

1 To d raw a pair of tangents to a circle which are 8 Draw a circle o f diameter 3 . 5 c m . Le t t he re b e a point
i nclined to each other a t an a ngle of 45°, it is P outside circle. Draw tange n t s to c i rcle f ro m P
requi red to d raw tangents at end points of those two without using the centre o f circle. CCE 20 1 2, 1 1

9 Draw a circle of radius 3.0 c m . Fro m a n e x t e r n a l


radii of the c ircle. What shou ld be the angle

point P. draw tangents to t h e c i rc l e w i t hout u s i ng


between them?

2 It i s possible to construct a pair of tangents from the centre of the circle.


point P to circle of radius 4 cm situated at a distance
10 Construct a pair of tange n t s t o c i rcle w hose rad ius is
6.5 cm are inclined to each o t h e r at a ngle o f 30°.
of 3.9 cm fro m the centre?

3 What is t h e relation between the length of the


11 Draw a circle of radius 3.5 cm and t he n d raw
tengents to this circle making a ngle o f 45° with a
tangents d rawn from an external point to a circle?

Short Answer Type I Questions line passing through the cen t re.

4 Draw a circle of radius AB = 45 cm w i t h centre 0 Long Answer Type Questio n s


and then draw a tangent to the circle at point A 1 2 Construct a tangent t o a circle o f rad i u s 3 c m f ro m a
5 Draw the tangent to a circle of diameter 5 cm at a point on the concentric circle of rad i u s 5 c m a n d
point P on it. measure i t s length. Also, verify t h e measure m e nt by
actual calculation.
6 Draw two tangents from the end points of the
diameter of a circle of radius 4.0 cm. A re these 13 Draw a circle of radius 4 cm. Ta ke a point P o u tside
tangents parallel? the circle at a distance 8 cm f ro m the centre. D raw

7 Draw two tangents to a circle of radius 4 cm from a


two tangents from P to the circle. M easu re t h e
length o f tangents. Write the steps o f constructi on
point P at a d istance of 7 cm from its centre.
and justify the construction.

Answers
1 . 1 35° 2. No 3. Both tangents are of equal length.
6. Yes 1 2. 4 cm
Very Sho rt A nswer Type Questions [ 1 Mark each]

1. To divide a line segment AB in the ratio 4 : 7, a 7. Given, a triangle with side AB = 8 c m. To


ray AX is d rawn first such that LBAX is an get a line segment AB' = � of AB, at what
acute angle and then points A1 , A2 , A3 , . . . are 4
located at equal distances on the ray AX. At ratio the line segment AB s hould b e
which point, B is joined? NCERT Exemplar divided?

2. To divide a line segment AB in the ratio 2 : 5, 8. Construct a MBC with sides


first a ray AX is drawn, so that LBAX is an AB = 4 cm, B C = 5 cm and CA = 7cm .
acute angl_e and then at equal distances, how Then, construct a triangle to it whos e
many points are located on the ray AX? sides are � times of t h e corresponding
CCE 201 1 7
sides of the given triangle . First, we d raw
3. I n the given figure, find the ratio when P
a ray BX such that LCBX is a n acute
divides AB internally.
angle and X lies on the opposite side of A
with respect to BC. Then, locate points
B1 , B 2 , B 3 , . . . on BX at equal distances .
What is the next step to j oin?
CBSE 2008 C

83 9.
84
To construct a triangle similar to a given
MBC with its sides � times of the
4. To divide a line segment AB in the ratio 5 : 7, 7
first a ray AX is drawn, so that LBAX is an corresponding sides of MBC, first d raw a
acute angle and then at equal distances, how ray BX such that LCBX is an acute angle
many points are located on the ray AX? and X lies on the opposite side of A with
. NCERT Exemplar respect to BC. Then, locate points B1 , Bi. ,
� , . . . on BX at equal distances . What is
5. To dr�w a pair of tangents to a circle which are : '-
next step to join? NCERT Exemplar
inclined to each other at an angle of 60°, it is
required to draw tangents at the end points of 10. To construct a triangle similar to a given
those two radii of the circle. Find the angle MBC with its sides 8/5 times of the
between them. NCERT Exemplar · corresponding sides of MBC, d raw a ray
BX such that LCBX is an acute angle a n d
6. To divide a line segment AB in the ratio p : q, X is o n the opposite side of A with respect
draw a ray AX so that LBAX is an acute angle.
, to BC. How many minimu m number of
How many points mark on ray AX? points to be located at equal distances on
NCERT Exemplar ray BX? NCERT Exemplar; CCE 20 1 4
' 1 48 , All-inone MATHEMATICS Class 10th Ter11t

� \ Short A nswer Type I Questi ons [ 2 Ma rks eac

I
1 1. PO i s a line segment o f length 6.4 cm. 15. Draw a circle of radius 3 c m . Ta ke a point a ,

G eometrically, obtain point R on PO such a distance of 5 . 5 cm from the centre of th-e


circle and from the point P, draw �,
that QR = �. tangents to the circle .
PO 8 CCE 20 1 5

1 6. Draw two tangents at the e n d points of the


12. Draw a line segment AB o f length 7 cm.
diameter of a circle of radius 3 . 5 cm . Are
Using ruler and compasses, then find a
these tangents parallel?
point P on AB such that AP : AB = 3 : 5 .
CBSE 201 1 1 7. Draw a circle of radius 6 c m and draw a
tangent to this circle m a king a n a ngle of 30:
13. Construct a triangle with sides 4 cm, 5 cm with a line passing through the centre.
and 6 cm. Then, construct a similar triangle
to it whose sides are �times of the 1 8. Construct a tangent to a circle of radius
3 4 cm from a point, which is a t a d istance of 6
corresponding sides of the given triangle . cm from its centre .
CCE 2013
19. Draw a line segment of length 7 c m . Find c
14. Geometrically divide a line segment of point P on it, which d ivides it in the rati

� I Short A nswer Type I I Questi bns


length 8 . 4 cm in the ratio 5 : 2 . CCE 20 1 5 3 : 5. NCERT Exemplar

[ 3 Marks eac ]

20. Draw a circle of radius 4 cm. Take a point P - 25. Draw a line segment A B o f length 9 cm.
outside the circle . Without using the centre Taking A as centre , d raw a c ircle of radius
of the circle, draw two tangents to the circle 5 cm and taking B as c entre , d raw anothe
from P. circle of radius 3 c m . C o n struct tangents
to each circle from the c e ntre of the other
2 1. Construct a MBC, in which AB = 5 cm, circle.
LA = 30° and AC = 6 cm. Then, construct a
triangle whose sides are
4
of the 26. Let ABC be a right angled triangle, in which
3 AB = 15 cm, BC = 8 c m and LB = go 0• BD i$
corresponding sides of MBC. CCE 20 1 5 the perpendicular from B on AC. The cirdt'
through B, C and D is drawn . Construct thr
22. Draw a circle o f radius 3 cm . From a point tangents from A to this circle .
6 cm away from its centre , construct the pair
of tangents to the circle and measure their 27. Draw a right angled MB C, in whkh
lengths. CCE 201 4 BC = 1 2 cm , AB = 5 cm and LB = go 0• Then
construct a triangle sim ilar to it and of scale
23.
Is the new triangle also a right
Construct a pair o f tangent t o a pair o f circle
of radius 4 cm from a point on the factor �.
concentric circle of radius g cm and
3
ang led triangle? N CERT Exemp lar
measure its length . Also, verify the
measurement by actual calculation.
CBSE 201 0
28. Draw a MBC, in which B C = 6 cm,
CA = 5 cm a nd AB = 4 c m . Construct "
24. D raw a circle o f rad ius 4 c m . Construct a triangle similar to it and of scale factor � .
pair of tangents on it, so that they are 3
inclined at 60°. CCE 20 1 6 NCERT Exe mplar
Constructi ons
f 149 I


29. Construct a triangle with sides 5 cm, 5.5 cm, 33. Draw a MBC, in which AB = 4 cm ,
and 6 . 5 cm . Now, construct another triangle BC = 6 c m and A C = g c m . Construct a
whose sides are times the corresponding triangle similar to .1ABC with scale factor
5 3 (2. Justify the construction. Are the two
sides of the given triangle CCE 201 4 triangles congruent?
[Note : that, all the three angles and two
3 0. Draw two concentric circles C1 and C2 of sides of the two triangles are equal .]
radii 3 cm and 5 cm . Taking a point on outer NC ERT Exemplar
circle C2 , construct the pair of tangents to
the other. M easure the length of a tangent 34. MBC is a right angled triangle, in which
and verify it by actual calculation. AB = 3 cm BC = 4 cm and LB = g o 0• B D
NCERT Exemplar i s perpendicular from B on A C . The circle
through B, C D is drawn. Construct the
3 1 . Draw a MBC, in which AB = 5 cm, tangents from A to the given circle.
BC = 6 cm and LABC = 60°. Construct a 35. Construct a pair of tangents PQ and PR to a
triangle similar to MBC with scale factor � . circle of radius 4 cm from a point P outside
7 the circle 8 cm away from the centre.
Justify the construction. NCERT Exemplar Measure PQ and PR. CCE 20 1 5

3 2 . Construct a MBC, in which AB = 5 cm, 36 . Construct a triangle o f sides 5 cm, 6 c m and


LB = 6 0 ° and the altitude CD = 3 cm. Then, 7 cm and then a triangle similar to it whose
construct another triangle whose sides are i sides are i of the corresponding sides of the
5 5

� I Long Answer Type Questions


times of the corresponding sides of MBC. first triangle. CCE 20 1 6

[ 4 Ma rks each J

3 7. Draw a �ABC with BC = 7 cm, LB = 45° 4 0. Draw a right angled �PQR, right angled at Q
and LC= 6 0 °. Then, construct another in which the sides PQ and Q are of lengths
triangle, whose sides are �
times of the
4 cm and 3 cm, respectively. Then,
construct another triangle whose sides are
5
corresponding sides of �ABC and justify 3 I 5 times of the corresponding sides o f
your construction. CBSE 2010 given triangle. Justify your construction.
CBSE 20 1 1

3 8. Construct a .1ABC, in which BC = 5 cm,


41. Construct an isosceles .1ABC with base
BC = 6 cm, AB = A C and LA = g o 0• D raw
LCAB = 120° and LABC = 30°. Then,
construct another triangle whose sides are � another similar triangle whose sides are
5
times of the corresponding sides of .1ABC. i times of the sides of �ABC. Justify your
5
Justify your construction.
construction. CCE 20 1 4

39. Draw a right angled triangle, in which the


sides (other than the hypotenuse) are of
4 2 . Draw a right angled �AB C with side B C = 7
lengths 8 cm and 6 cm. Then, construct cm, LABC = 60° and AB = 6 cm. Then,

another triangle, whose sides are � times of construct another triangle whose sides are �
4 4
the corresponding sides of given triangle. times of the corresponding sides of �ABC.
Justify your construction . CBSE 2009
Justify your construction. CBSE 2009
l 1so l AllinOne MATHEMATICS Class 10th Term 1 f

4 3. D raw a circle of radius 4 cm. Take two 44. Construct a 6AB C, in wh ich BC = 6 cm.
points P and O on one of its extended LA = 60° and media n, A D = 5 cm. Al so. ,
diameters , each at a distance of 9 cm construct another 6BPQ simila r to 6BCA such , I
from its c en tre. D raw tangents to the
I
3
that the side BP = - BC.

\ Va l ue Based Questi on (VBQ)


circle fro m these two points P and Q. 2 ;
I
I
I

� [ 4 Mar
l'
45. Sanjeev h ave a piece of cloth of 8m long. He decided to divide this piece in two persons A and B '
internally in the ratio 3 : 4 . I
(i) Draw a construction of above problem. I
1
(ii) If Sanjeev give 4th part of the piece to the person A, then what value is violated by S anjeev?
I
Sol u tions I
1 . Here, 4 + 7 = 1 1 points are located at equal AB' = � of AB = � x 8=3 x 2 = 6 cm
distances on the ray AX, so B is joined to last 4 4
point A 1 1 • [1 ] to---- 6 -----+- 2 -I
A 8' B
---- B cm ----
2. We know that, co divide a line segment AB i n che
ratio m : n , first draw a ray AX which makes an
acute LBAX, then marked m + n points at equal and BB' = AB - AB' = 8 - 6 = 2 cm
distance. AB' : BB' = 6 : 2 = 3 : I
Here, m = 2 and n = 5 [1 ] Hence, the line segment AB should be divided in 3 : 1. (1 , ·

Hence, number of points on che ray AX 8. Here, scale factor = � . So, we locate points B1 , B2, B3 , B4•
= 2 + 5 = 7 7
B5 , B6 and B7 on BX at equal distances and i n next st�
3 . From given figure, ic is clear that there are three join the last point B7 co C. [t)
points at equal distances on AX and four points at 9. Do same as Q. 8. [Ans. Join B7 to Cl
equal distances on BY. Here, P divides AB on
joining A3 B4 • So, P divides AB internally in che 1 0. We know chat, co construct a triangle s i milar co a gi\"tl1
m
ratio 3 : 4. [1 ] triangle with its sides of the corresponding sides of gh'tl1
n
4. Do same as Q. 2. [Ans. 1 2]
7 triangle, the minimum number o f points to be locaced at
w
5 . The angle between chem should be 1 20°, because
in chat case, che figure formed by the intersection equal distances is equal to the greater o f m and n i n 111 .Hert.
n
� point of pair of tangents, che two end points of
those two radii (at which tangents are drawn) and
m 8
- = - and 8 > 5. (112]
n
: ,F -.
5
'·.J �·
the centre of the circle, is a quadrilateral. [1 /2]
So, the minimum number of points to be located at equ:il
distances on ray BX is 8. (1/21
=�
QR
1 1 . Given, PQ = 6.4 cm and
PQ 8
0 PQ � =>
PQ - �
1: 1< =
'I
1-��;�I
or I = -1
l, ,j QR 5 QR 5
�!)
r.:.:.-=-��
ii =>
PQ - QR
---- - --
8 -5
=>
PR 3

:�
- = -
�! 1I
From figure, OPRQ is a quadrilateral.
QR 5 QR 5
:. LPOQ + LPRQ = 1 80°
ii
:.����l :1 Steps of Constrnction
! ./
I
[·: LRPO = 90 ° = LRQO]
. -�
__-:-�

-r:::.:_---.
I

I
=> 60 ° + e = 1 so0
=> e = 1 20° r 1 121 (t) Draw a line segment of length PQ = 6.4 cm.
I-�; ; [Ans. p
· -� · - ,

6 . Do same as Q. 2 (
{it) Draw any ray PX making an acute L QPX with PQ.
I
+ q)]
--- - �
!I (iit) Draw a ray parallel to PX by making
·-;-=-::!)
,, /:: 7. Given , AB = 8 cm LPQY = L QPX.
Constructions � 151 f
(iv) Mark three
points B1 ,BPA12points A1, Ai, � on PX and five
B4 and B5 on QY such char [1 ]
, B3,= A1A2 {it) Mark a poinc
centre and joinP acOP.a distance of 5 . 5 cm from the
0

QB1 = B1B2 == Bi� Ai� =


�B4 = B4B5
x

[1 1
(iit) Draw che bisector ofOP, incerseccing OP ac Q.
(iv) Taking
docced Qrasclecentre
ci whi c handintersects
OQ = thePQ giasvradius,
en circldraw
e at a
T
y andT'.
(v) Join PT and PT' .
(v)Join
Thus,A.�Rto iB5•s theThen,
pointic ofintersect
di v i d i nPQg PQat asuch
poinccharR. Thus, PTand PT' are che required tangencs from P. [1 ]
1 6.
QR = -or
- 5 QR = -5
- [1 ]
Steps ofCo11stn1ctio11
(t) Draw a circle wi c h centre and radius 3 . 5 cm.
0
PQ 8 PR 3 {it) Draw che di a meter POQ.
1 2. Do same as 1 1 . Q. (iit) Construct an angle of 90° ac both end points
PandQ.
x L

y
1 3. Steps o/Constmction y M [1 1
DrawCAa = cm.wich AB = 4 cm, BC = 5 cm Thus,the XPY and
wichLQM are the two tangents at P and Q
(t) MBC
and 6 toHere, cilinesrcleXPY centre
0.
{it) Draw an acute LCBX and mark chree poincs
B1, Bi and � such char BB1 = B1Bi = Bi� . [1] LP
11 LQM, because
+ L Q = 1 80° [linear pair] [1 ]
(iit) Joi n �C and draw BiC'l l B3C meecingBCacC'. 1 7. Steps o/Construction ·��-;:
·:�-.::-_ L
(iv) Through C', draw Cj4' 1 1 CA meeting AB ac A' . (t) Draw a circle with centre and radius 6 cm.
0

. -
I I'
·-�-=-
'
:

8 ..., .c
'
,,, , :
'
I
:
8(',a : [1 1
11
ib �cl
','*
82 /�:
(ii) Draw a radius OA and produce it to B. i: j ':::!:�
:.

(iit) Construct an LAOP equal to the complement of


' - · .J
' I
,

' :' -�.

'� 30°, i. e . equal to 60°.


I

:·; j•1 \ --'· '


• . • . --


83 '· x
(iv) Draw a perpendi c ul a r OP ac P whi c h intersects OB ;! ��
k
;;
Then, M'BC' is che required triangle. (1] acQ. I, .! '.1°1:;.
14. Do same as 1 1 . Q. Hence,
LOQP PQ= 30°.is che required tangent such that[1 ] ii
'i
0� j
c�j
1 5. Seeps of Construction cent Do same as Q. at page 1 1 47.
f!
ii
:-�=---.:.., :
(t) Draw a circle with as re and radius 3 cm. 1 8. -
!! ·
0
.� ..:_i.:
\ 1s2 \ Allinone MATH EMATICS Class 10th Term I

1 9. Do same as Q. 1 at page 1 47.


A8 X
A1
Ae
As
A4
A3
A2
A1

p B
---- 7 cm ---
A
24. Do same as Q. 4 at page 1 47.
20. Steps of Construction 25. Do same as Q. 5 at page 1 47.
(t} Draw a circle of centre 0 and radius 4 cm. - - - - - - -

(ii} Take a point P outside the circle and draw a secant


PAB, intersecting the circle at A and B.
D ... ... )'(\- - - -
--

-<' I ,
,,.,.,. I I ...
''

I
,,

/ �
/ I :
I
I
I

c
26. Do same as Q. 6 at page 1 47.

[1 1
(iit) Produce AP to C such that AP = CP.
(iv) Draw a semi-circle with A as centre and AC as

radius.
(v) Draw PD .L CB,
intersecting the semi-circle at D.
(vt) With P as centre and PD as radius, draw arcs to
intersect the given circle at T and T'.
(viz) Join PT and P T'.
Thus, PT and PT' are the required tangents. [ 1 ] 27. Do same as Q. 2 at page 1 47.
21 . Do same as Q. 6 at page 1 47. A

E
0
LO

[1)
28. Do same as Q. 3 at page 1 47.
N
22. Do same as Q. 1 at page 1 47.

The lengths of the tangents are 5.20 cm each. x


23. Do same as Q. 2 at page 1 47. 29. Do same as Q. 2 at page1 46.
Constructions f 1s3 I
30. Do same as Q. 2 ac p age I 47. 35. Do same as Q. I ar page 1 47.

The lenghr of rangenrs is 7 cm.


36. Given, a MBC, in which BC = 6 cm, AB = 5 cm and
3 1 . Do same as Q. 5 ac page 1 47.
AC = 7 cm. Here, scale factor = - < I.
4
32. Steps of Co11str11ctio11 5
(l) Draw a l i ne scgmenc AB = S cm and conscrucr Steps of Construction
LB = 60°. [1 ) (t) Draw a line segment BC = 6 cm.
(ii) Draw AN .l A B and cue AM = CD = 3 cm
from AN.
(iii) Through M, draw a line I parallel ro AB co
inrersecr ray BX ar C
(iv) Join A C
Thus, M B C i s a required triangle.
x

[1 1
(it) Wirh B as centre and radius equal co 5 cm, draw an
8
arc.
(iir) Wirh C as centre and radius equal co 7 cm, draw
another arc cutting rhe previous arc at A.
(iv) Join AB and AC. Thus, MBC is constructed.
(v) Now, draw a ray BX making an acute LCBX with
[1 1 base BC on rhe side opposite to the vertex A
(v) Draw a ray AX co make an acute LBAX and mark (say 60°).
five points A1 , A 2 , � , A4 and A5 on ray AX such (vi) Along BX, mark five points B1 , B2 , � , B4 , and
char AA1 = A1 A2 = A 2 � = � A4 = A4 A5 • B5 such chat BB1 = B1 B2 = B2 � = � B4 = B4 B5 .
(vz) Join A5B. (viz) Join B5C.
(viz) Through point A4 , draw A4 B' 1 1 A5B. (viii) From B4 , draw B4 N 1 1 B5C meeting BC at N.
(viii) Through B' , draw B' C' 1 1 BC. [by making an angle at B4 equal to LBB5C]
Thus, MB' C' is rhe required triangle similar co (ix) From N, draw NM 1 1 C4 meeting AB at M .
MBC wirh scale factor i. [by making an angle at N equal to LBCA]
5 [1] Thus, 11MBN is the required triangle, each of
33. Do same as Q. 3 ar page 1 47 .
i
3 4. Do same as Q. 6 a r page 1 47.
whose sides is of rhe corresponding sides of
5
c llABC. [11
justification
By consrrucrion, B4 N 1 1 BsC

I
-- = -- = - => --
BN 4 NC 4 I
= -
NC 5 - 4 BN 4
- = ----
BC BN + NC
Now,
BN BN
NC l 5
= l + - = 1 + - = -
(1 1 BN 4 4
! 154 ' Allinone MATH EMATICS Class 10th Tena

I.e. BN = -4
-- 38. Given A MBC. in whid1 BC = I) LCAB cm. = ll
BC 5 and LABC = 30°.
Also, NM I I MB
CA, so we have llMBN - MBC.
BN = -4
Then, LBCA = 180 ° - 30° - 120°
Hence, AB = -- NM = --
--
= 30°
CA BC 5 [1 ]
37. Given A MBC, in whi c h BC = 7 cm, LB = 45°and
LC = 60°.
Required Draw LlA'BC' ... 6.ABC with scale factor
� < 1.
s
Steps of Construction
(t) Draw a line segment BC = 7 cm.
(ii} Ac B, construct an LCBY = 45°. x
(iii} Ac
ac C, construct an LBCA = 60° intersecting BY
A. Now, do same as Q. 37.
39.
cm and 6rigcmht making
8Given A angled atriangle with sides of lcngt
right angle.
Required Tri a ngle whose sides arc � rimes of c
4
corresponding sides of given tria ngle.
Steps o/Constructio11
(1) Construct a right angl e d
6.ABC right angle ac wirh sides BC = 8 cm
B
AB = 6cm.
[1 1 (iz} Through
opposite coB,checonsrrucc
vertex A.an acme LCBX on the si.
[
(iv) Join AB and CA.
Then, MBC is che required triangle. [1 ]
(v) Through
side opposiB,teconstruct
co che verteXian acute
A.
angle LCBX on che
(vt} Mark
such charfiveBB1points= B1BBi1 , Bi,= Bi�� , =B4Band B5 on BX
3 B4 = B4 B5•
(vit} Join B5C.
(viii}
ac C'. B3 , draw� C' 1 1 B5C, intersecting BC
Through
(ix) Through C', draw CA' 1 1 CA, intersecting AB ac
A'. Hence, M'BC' is the required triangle. [1 ]

( ;. "·_
justification
By conscrucci o n, BsCI I� C '
(iit) Mark
char four points B. , Bi, B_,, and B4 on BX su1
BB1 = B1 Bi = BiB_,, = B3 B4 •

." ,/--, ==>


BC' = -- 3 = -3 :::::) C' C = -2
--
(iv) Join B4 C.
C' C S 3 2 - BC' 3 (v) Through � , draw B_,, C' 1 1 B4 C intersecting BC
Now, BC BC'+C' C ac C' .
BC' BC' (vi} Through
A'.
C', draw CA' 1 1 C inrersecring
A AB

�;±)!
= 1 + -- = 1 + -32 = -S3
CC' Hence, 6.A'BC' is che required triangle.
' ..i:>.:
BC'
.-�-�
:.J:U'.31
i.e. BC' = -3
-- Justification
l ·L� BC S By conscruccion, B4 C 1 1 B3 C'
�:? � AJso, C' A' I I CA .. --
BC ' = 3--
3
::: -

::::::> MBC ... A' BC' C' C 4 - 3


:""' ... . ;_ BC' = A' C' = -- A' B = -3 i.e. --
CC' = -I
Hence, -- BC AC AB 5
-- [1 ] BC' 3
Constructions j 1 55 �

Now, BC BC'+CC' (v) From B4 , draw B4 C' 1 1 B5C intersecting BC at C'.


BC' BC' (vi) Through C', draw CA ' I I CA intersecting AB at
CC' A '.
= I + --

BC' Hence, !l.A'BC' is the required triangle. [11


I
= ] + - =-
4 Justi cation
fi
3 3 By construction, BC'B4 C ' 1 1 B5C
4 = -4
i.e. BC'
= -
3 --
= --

5-4
Also, BC 4 C' C
--
= -
C' C
..
A' C' I I AC
M ' BC' - MBC
i.e. BC'
I
4
Hence, A' B
-- = --
A' C' BC'
= -- =
3
- [1 ) Now, BC
-- =
BC'+ C' C
-----

AB AC BC 4 BC' BC'
C' C
40. = I + = + _!_ 2_
=

BC' 4 BC'
I
4 4
i.e. -- = -
BC 5
Also, M BC' '-l l llABC A
C' CA
'
Hence, A' C'
=
BC'
=
A' B
=
4 [1 ]
AC BC AB S
42. n w
Given A ll.ABC, i hi c h BC = 7 cm, L ABC 60° =
and 6cm. AB =

Required - MBC wi t h scale factor �-


41 . Givm A n isosceles MBC wich base BC 6 cm, =
M 'BC'
4
AB = AC and L A 90°.=
Steps of Construction
(t) Draw a line seg me nc BC 7 cm. =
Then, L B = LC = 2 x 90° 45 °
I
- = (iz) Ac B, constru ct an acute LCBY 60° and =

Required !l.A'BC' - !l.ABC with scale factor i.


cue-off BA = 6 cm from it. [1 ]
(iiz) Joi n AC.
5 Thus, MBC is the required given triangle.
Steps of Co11structio11 y
(z) Construct an isosceles Dr.ABC with base
BC 6 cm, AB AC, LA 90°, LB 45°
= = = =
and LC = 45 °. [1 ]
(it) Through B, construct an acute angle LCBX on
the side opposite to the vertex A.
t A
I
I
I
I
I c
I
[1 ]
71"
I t_.( �
I (iv) Throughte coB,theconstruct
opposi vertex an acute LCBX on the side ;T
�....
·:

A. ·�,\.
( ;1
(v) Mark four points B1 , B2 , � and B4 on BX such c: �
chat
BB1 =
B1 B2 = B2� = � 84 .
(vt) Join B4 C.
(viz) Through � , draw � C' 1 1 B4 C intersecting BC at
[1 ] C'.
(iiz)MarkchatfiveBB1points= B1B1B2, =B2,B2�� , B4B3andB4 B5= B4onB5BX• (viiz) Through C', draw Cj<t' I I CA intersecting AB at
such =
A'.
Hence, AA.'BC' is the required triangle.
(iv) Join B5C.
' 156 \ AllinOne MATH EMATICS Class 10th Term

Justification
By construction, � C' 11 B4 C
BC'
-- = -- = -
3 3
CC''CC 4 -3
1
i.e. -- = -
BC' BC'+ C'C 3
Now,
BC
-- = ----
BC' BC' 1 4
= l + - = -
3 3
C'C
= 1 + -­
i.e.
BC' BC'
-- = -
3 (viii) Now,
opposiconstruct te an char
teonroBXve such acme
Then,an e Lch rec points
gl CBX on theBi,side
BC 4 r x A. mark
BB1 = /J1 B2 = 8213_, .
B:,

Also, A'C'llAC (ix)


and �
Join liandne segment chcn draw B_,,acP P.11 B2 C inrerseccing thr
=> 6.A' BC' .. 6.ABC BiC
extended BC
Hence,
A' B A'C' BC'
-- = -- = -- = -
3 (x) om poi n t
Fr P, CA
draw PQat 11Q. inrerscccing the
AB AC BC 4 (1 ] extended line segment BA
43. Do same as Q. 3 at page 147. Then, llBPQ is the required ro llBC�
triang l e s i m i lar

'
........ ----...... M
..
p ..
,
I .. .. -
-
- -
- ....
.. ..
such that BP �2 = BC .
,, ,\
[1]
/ \
I \
I I (t) Steps of Construction
:
45.
P, \Q
I
(a)
segmentDraw a line
cm.
AB of length 8
44. Given, scale factor = � > 1.
(b)
AX
Draw any ray
2
Steps of Construction
(z) Draw a line segment = BC 60°6
cm .
acutemaki
LB AXw
ng an
ith Bp A .
(iz) At construct B,B, = LCBX
in downwards. (c) Draw a ray B Y
(iiz) At draw BY 1-BX. AX by
1-
BYOB BC
(iv) Draw the perpendicular bisector of
BC at 0.
[1 ]
which makingto
arallel
LAB Y =
L BAX . [1 ]
(d) Mark4 poithreents points andand on such thac �, on AX
2
intem;ct at D and Ai , Ai
l.J.J
(v) Taking 0 as a centre and as a radius, draw a Bi , Bi , B3 B4 BY
circle passing through B C. 5 and (1 ]
and
M1 = A1 A 2 = A 2 AJ
� (vt) Taking D as a centre and radius cm, draw an arc
A. = BB1 = B1 B2 = B2 B3 = B_1 B4 •
Cf) to intersect the circle at
)
(e Joi n � B4• Then, it intersects at a point
(f) (vii) Join and AB AC. Thus, P i s the point di v iding AB
internal l y i nP.
the
u.J
AB
ratio 3 4 : [1)
CJ)
CJ)
Thus, 6.ABC is the required given triangle. (1 ]
(iz) He violated the equality.
value of [1)
_,,,,-
FO RMATIVE ASS ESS M E N T
• •

Act1v1ty
Topic Covered Construction of a quadrilateral. 4. In the given figure, ratio of d ivision is ... .

Objective We wish to construct a quadrilateral AB'C'D'


. � (reduction).
. 3
in quadrilateral ABCD with scale factor

Skill Developed Creativity

Time Req u i red 15 min

Steps of Construction
1. Construct a q uadrilateral ABCD. True/False
2. Draw a ray AX downwards to AB 1 . A triangle similar to &ABC is to be d rawn. If scale
making an acute LBAX. factor is greater than 1, then we get enlarged fig u re.
�--1'-i----LI, 2. In a llABC, we d raw AAB ' C ' - AABC with scale factor
A
3 . Mark three points A1 , A 2 and A
on AX such that 13
3
. Then, perimeter of AABC > perimeter of MB ' C '.
AA1 = A1 A 2= A 2 A 3 • 15
4. Join A B and d raw A2 8 ' 1 1 A 8
3 3 3. In the given figure, if AP = 6 cm and BP = 8 cm, then
m : n= 3 : 4.
which intersect AB at B'.
5. Join AC.
6. Draw a line parallel to BC from B' to meet AC at C'.
7. From C', draw a l ine parallel to CD to meet AD at D'.
Then, AB'C'D' is the required quadrilateral.

Fill in the Blanks


1. If we have to construct MB ' C ' ,.. llABC with scale
factor �' then MB'C' is a/an ...
3
4. In the given figure, if scale factor is � , then the next
1-
(reduction/enlargement) of llABC. step would be join A to B 5 • z
2. llAB'C' was constructed similar to MBC by taking LU

L
scale factor �. Now, if we construct a triangle similar
7
2:
to llAB'C' with scale factor ..., we get back AABC.
A
(/)
3. A triangle similar to llABC is to be drawn with scale B LU
a'-.,
1 ......,,
factor �. The resulting triangle will be ... ,
82 ''.c.
_
3 83 ........

(smaller/bigger) than llABC. 84 .... "ii..,


,

8s .....,.

Answers
Fil l in the blanks
:!. 3 . Smaller 4. 3 : 2
4
1 . Reduction 2.

True/Fa lse
0
4. False I.I.
1 . True 2. True 3. True
CHAPTE R EXE RCIS E
� \ Very Short Answer Type Questi ons [ 1 M a rk each ]

1. I t is possible to divide a line segment in the ratio 6. To construct a triangle similar to the given MBC, with
.J3 : )J by geometrical construction. its sides �
7
times of the correspo�d ing s ides of the
MBC first draw ray BX s uch th at LCBX i s an acute
2. In the given figure, 8BPQ is similar to 8BCA with its angle and X lie on the opposite sides o f A w.r.t BC.
sides � o f the corresponding sides of .6.BCA. Then, find Then, locate points 8 1 , B2 , B3 , on BX at equal
• • •

y distances write the next s tep.


x
the value of -. 7 . Find the ratio of division of the li ne segment AB by
y CCE 20 13 the point P from A i n the following figure.
A CBSE 2012

8 . In the figure of the given con s truction, if


3 . To divide a line segmentAB in the ratio 7 : 5, first a ray 8QST - !1QRP, then find QT : QP. CCE 2013
AX is d rawn so that LBAX is acute angle and then at
equal distance points are marked. Find the minimum
number of these points.
4. To d ivide a line segment AB in the ratio 3 : 5, first a
ray AX is drawn making, an acute angle LBAX and
then on the ray AX, points at equal distances are
marked . Find the min im um number of these points.
5. To d ivide a line segment AB in the ratio 4 : 5, first a
ray AX is d rawn making LBAX an acute angle and
then points A 1 , A 2 , A 3 , at equal distances are
••

marked on the ray AX . At wha t point is point B

� / Short Answer Type I Questi ons


joined?

[ 2 M a rks eac h

9 . Draw a circle of radius 8 cm. From a point I O cm away from its centre, construct the pair of tangents t o t he circle.
1 O . Cons truct a pair of tangen ts to a circle of radius 7 cm from a point on the concentric circle o f rad iu s 1 3 cm.
1 1 . Draw a line segmen t of length 1 0.4 cm and divide it in the ratio 5 : 8 .
1 2 . Draw a circle o f rad i us 3 . 2 cm . Draw any diameter of the circle. At the end points of the dia meter of the circle, draw
tangen ts. Are they parallel?
1 3 . Divide a line segmen t of length 8 cm in the ratio of 2 : 5 internally.
1 4 . Draw a line segment o f length 5 cm and divide it in the ratio 2 : 3 internally.
1 5 . Dra w a line segmen t o f length 6 cm and divide it in the ratio 1 : 3 . Measure the two parts.
Constructions 159 i! i

1 6. Draw a pa i r o f ta nge n t s lo a circle of rad ius 4.5 cm, 1 8. To loca te a poi n t Q on PR s uch t h a t QR = � PQ, at
which a rc i ncl i ned lO each other a t 60°. 3
wh at ra tio the line segment PR s h o u ld be divided?
1 7 . Con s t ru c t an equilatera l triangle of side 5 cm and CCE 20 1 3
con s t ru c t a no th e r t riangle whose sides are ± times of 1 9. Draw two concentric circles w i t h cen tre 0 and radii
5
given t riangle. 3 cm and 5 cm. From a point on t he o uter circle, draw

� I Short Answer Type II Questions


a tangent to the inner circle. Measure the l engt h of
the tangent . C C E 20 1 3

[3 Ma rks ea c h ]

20. Draw a circle o f ra d i u s 4. l cm. Draw any l ine through 24 . D raw a righ t angled M B C , i n which A B = 4 cm,
the cen t re o f the circle. Draw a tangent to the circle BC = 6 cm and LB = 90 °. From C, d raw two tangen t s
making an a ngle of 45° with the l ine. What is the t o the circle drawn on AB a s d iameter. F i n d t he l eng t h
length or t he ta ngen ts? o f each tangent .
21. Draw a c i rcle o f rad i u s 3 cm with centre 0 and take a 25. Draw two concentric circles o f rad i i 3 cm a n d 6 cm.
poi n t P o u t s i d e t he circle, such that OP = 5 cm. Draw From a point on the outer circle, cons truct a pair of
two t an gen t s to t h e circle and measure their leng ths. tangents to the i n ner circle. Meas u re t he ir l engt h s .
22. Draw a circle of rad i u s 5 cm. From a point 13 cm away CCE 20 1 3

Construct a l ine segment PQ


from i t s cent re, con s t ruct a pair of tangents to the
26. =
7 . 8 c m a n d d ivide i t in
circle and mea s ur e their lengths.
the ra tio 7 : 1 0. Mark the point of d iv i s ion as R a nd
23 . meas ure t he leng ths of PR and QR.

� \ Long Answer Type Questions


Draw t he l i ne segment of length 7.8 cm and divide it
in the ra t io 5 : 8. Mea sure the two pa rts.

[4 Ma rks ea c h ]

27. Con s t ruct a n equ ilateral MB C of side 5 cm. Now, 31. Const ruct a triangle similar t o g iven D. ABC with
AB = 4 cm, BC = 6 cm a nd CA = 7.5 cm such t hat each
constru ct a not her s i m i lar triangle �hose s ides are �
4 of its sides is equal to � t imes of the corres ponding
3
s ides of !).ABC .
times of the corresponding sides of MBC· CBSE 201 2
28. Con stru c t a !).PQR, in wh ich QR = 6 c m , LQ = 60 °
and LR' = 4 5 °. Construct another triangle similar to 32. Draw a D.LMN w i t h L. M = 90 °, MN = 6 c m and
!).PQR such t h a t i t s sides are 5/6 of the corresponding NL = 8 cm . Construct a n o t her t riangle s i m i l a r t o
s ides of D.PQR. CBSE 201 0
!).LMN , such that each of i t s s ide is
4
t ime s o f
Draw t w o equal circles o f radius 2 c m with centres A 7
corresponding sides of D.LMN .
29.
and B and d i s tance between A and B is 6 cm.
Construct a pair o f tangents from centres A and B to 3 3. Draw a circle of rad i u s 3 . 5 cm. Take a point T ou t s i d e
each other. t he circle a t a distance o f 7 c m fro m t h e centre and
30 . Draw a circle of rad ius 2 cm. From a point 8 cm away constru c t a pair of ta ngents from this point T t o t he
from the cen tre of the circle, draw two tangents to the circle and j u s t i fy you r cons t ruc t io n .
circle. Find the leng ths of the tangents. Justify your
cons truction.

Answers
2. � 3. 1 2 4. 8
1 . No 5
6. Join B 7 to B 7. 3 : 2 8. 2 : 3

1 5. 1 . 5 c m , 4.5 cm 1 9. 4 cm
1 2. Yes
1 8. 3 : 2
cm 23. 3 c m . 4.8 cm
20. 5.80 cm
21 . 4 cm 22. 1 2
26. PR = 3.2 cm. QR = 4.G cm
25. 5 .2 c m
24. 6 cm
31 . zJ15 cm
C H A PT E R S

SO M E APPLICATION S O F
TRI GO N O M ETRY
� Chapter Checklist Measuring heights and distances is an important application of trigonometry. In this
chapter, we shall study the use of trigonometry in measuring the heights and distances
;�0 Heights and D istances of towers, buildings and other objects.
• Line of Sight
Before starting this chapter, we shall overview some important result of trigonometry

0
• Angle of Elevation
i.e. trigonometric ratios and its table, which are widely used in solving the p roblems of

( !....J
• Angle of Depression
application of trigonometry such results are given below
In right angled MBC,

( .!!_J
Perpendicular
(t) sin A or = BC c
Hypotenuse H AC .'
AB .

( J
.
B ase =
{ it) cos A = or
Hypotenuse H AC

( J
( .. :\ Perpendicular P BC
ttt1 tan A = or - = -
Base B AB

( J
B ase B = AB
(iv) cot A = or
Perpendicular P BC

( J
Hypotenuse H = AC
( v) sec A = or A "--'-�-----____.'--' B
Base B AB Base (8)
:\ Hypotenuse
or - = -
H AC
( vt1 cosec A =
Perpendicular P BC

Trigonometric Ratios of Some Specific Angles

Angles 0° or O 30° or � 45° or � 60° or � 90° or �


6 4 3 2
sin 0 0 -1 1 .J3
2 ./2 2
.J3 1 -1
cos 0
2 ./2 2 0
1
.J3 00
tan 0 O
.J3
oo = Undefined
Some Applications of Trigonometry J 1 61 !
Heig hts a n d D ista n ces Some Important Points
(I) The a ngle of elevation of a
- - - --AngTe cr
In solving problems of heights and distances of different objects,
poin t P as seen fro m a p o i n t o
we need co defi ne few terms as discuss below. is always equal co the a ngle o f depression
depression of 0 as seen
Line of S ig ht from P.
�cf"
d-
'>�e,
The line of sigh t is the l i ne drawn from the eye of an observer co (ii) The a ngles o f elevation a n d Angle of A
_le_
e a_
v_tio n �
A _
H o riz ntal line
__
_
the point in the object viewed by the observer. depression a re a lways acute
angles. o
Ang le of E levation (iii) In solv i ng problems. observer is represented by a
poin t and objects a re rep resented by l i n e segm e n t o r
The angle of elevation of an object viewed, is the angle formed by
che line of sight with the horizontal, when it is above the horizontal
poi n t .

level i.e. the case when we raise our head co look at object. Example 1. If 3 00.../3 m high tower makes a n gle
P (Object) of elevation a t a point on gro und which is 3 00 m

away from its foot, then find the angle of
�C>J
°'
elevation .
-0�
Q,
Sol. AB
Let = 300be.J3them. tower whose height is 300./3 m, i.e.
Angle of
e l evat ion
AB Agai n , l e t C be the point at a distance
of 300 m from the foot ofthe tower, i.e. A C = 300 m.
1�
t.
. �
� Eye
H o r i zon tal line A
B

j �c
Lee P be the position of the object above the horizontal line OA and
0 be the eye of the observer. Then, OP is the line of sight and 300�
LAOP is called an gle of elevation, because the observer has to
elevate (raise) h is/her line of sight from the horizoncal OA to see the
object P. A - 3 oo m -

Here, 9.
Some I mportant Points nce, thehere
SiThen, anglbasee ofandeleperpendi
in right angled llBA C,
vation iscunknown,
ul a r are gi vsoen.let it be
( i) A plane level parallel co Earch surface is called the horizontal plane
level and a l i ne d rawn parallel co horizontal plane is called a
horizontal l i n e . tan = Perpendi
e
cular = AB = 300./3
( ii) I f
Base A C 300
e
tan = .J3 = tan 60°
c h e observer m oves cowards che perpendicular l i ne
(cower/build i ng) . t hen angle of elevation i ncreases and if the
observer moves away from the perpendicular l ine (tower/building) , e =60°
then angle of elevat i o n decreases.
(iii) If chc heigh t of tower i s doubled and the distance between the
Hence, the required angle of elevation is 60°.
observer and foot of che cower is also doubled, chen the angle of Exa m p l e 2. A bridge on a river makes an a n gle
elevation rem a i n s same. of 45 ° with its edge. If the length along the
(iv) I f che a ngle o f eleva c ion of Sun. above a cower decreases, then the bridge from one edge to the other is 150 m, then
length of shadow of a tower increases and vice-versa .
find the width of the river.
Angle of Depression Sol. Let
river suchbe that
BC the width of the river and be the ends of
A, B
('(. O ..--.--A-gl-
n- of- A
e-
The angle of depression of an object
viewed, is the angle formed by the line depression AB = 150 m =Length of the bridge [given]
(�
<90v
of sight with the horizontal, when it is

below the horizontal level i.e. the case �
�I'
when we lower our head to look at the (Object)
Horizontal line P
object.
Let P be the position of the object below the horizontal level OA
Also, let at the end, bridge makes L.BA C = 45°with the
and 0 be the eye of the observer. Then, OP is the line of si ght and
LAOP is called an an gle of dep ression, because the observer has to
edge of the river.
depress (lower) his/her line of sight from the horizontal OA to see
In right angled MCB,
. 45 ° = -
S i ll p
H
the object P.
1 162 I Allinone MATH EMATICS Class 10th Term H

1 BC = BC Sol. Let the aeroplane be at D an<l, D


two cars be at B and A such
i
=> =
.J2. AB 1 50
that their angles of depression
1 50 .Ji. [by rationalising] from D are 60° and 30°, 1 500 r.i

!
BC = x

.J2. .J2 respectively. ""-----"-_..___.___

=
1 50 .J2 = 75 .Ji. Given, DC I 500
= m A +- x m + B +- Y m+ C
2 Let AB = x m and BC = y m

= 75 x 1 .4 1 4 [·: .Ji. = 1 .4 14] In right angled llBCD,


= 106.05 m (approx.) DC
tan 60°= -
Hence, width of the river is 106.05 m. BC
.J3 = 1 500
Example 3. A straigh t tree is broken due to
y
th understorm. The broken part is ben t in such a way
that the peak of the tree touches the ground at an
1 500 .fj
y = -- x - r rarionalising)
60° .Jj .J3
y = 500.fj 111
angle of at a distan ce of 2.J3 m. Find the whole
. . . (i)
h eigh t of the tree.
In right angled llACD,

[-: can30° .fi and ]


Here, angle of elevation and base of triangle is given and we
have to find the whole height of the tree, i.e. AB for this we tan300= DC => __ =
I 1 500
have to find AC (or CO) and BC.
AC .fj AB + BC
Sol. Let AB be the tree whose part AC breaks A = AC = AB + BC
and touches the ground at D.
Then, BD = 2 .../3 m [given] I
=
1 500
and AC = CD .fj x + y
In right angled llCBD, 1 1 500 [from Eq. (i)J
cos 60° = !!_ = BD
=
.fj x + 500.Jj
H CD

[-: cos 60° = � and ]


x + soo.fj = 1 500.fj
=>
I 2 13
- = --
2 CD x = 1 500.fj -500.fj = 1 000../3
= 1 000 x 1.732 r ·.· .f3 = 1.7321
BD = 2 .Jj m = 1 732 m
=> CD = 2 x 2 ../3 = 4J3 Hence, the distance between two cars is 1 732 m .
= 4 x 1 .732 = 6.928 m [·: 13 = 1.732] Example 5. Determine the heigh t of a m o un tain, if
AC = CD = 6.928 m the elevation of its top at an unknown distance from
Again, in right angled llCBD, 30° 10

h
the base is and at a distance km furth er off from
tan 60° = p = BC the mountain along the same line, the a n gle of
elevation is 1 5°. (Take tan 15° 0.27)

B BD =
=> 13 = B [·.· tan 60°= .J3 and BD = 2 .../3 m] Sol. Let AB = km be the height of the mountain and angle of
2v 3 elevation ofits top from a point (say C) at unknown distance is
=> BC = ..J3 x 2.../3 = 6 m 30°, i.e. L BCA = 30°.
Now, AB = AC + BC B
= 6.928 + 6 = 1 2.928 m (approx.)
Hence, the height of the tree is 1 2.928 m. r
h km
Exam p le 4. An aeroplane at an altitude of 1500 m
fin ds tha t two cars on the ground moving forwards it in
l
the same direction . If the angles of depression of the
cars as observed from the aeroplane are and 30°, 60° Also, the angle ofelevation of its top from a point (say D) is 1 5°
respectively, then fin d the distance between the cars. i.e. LBDA = 1 5 ° and point D is such that DC = 1 0 km.
(Take, .J3 = 173 2) Let AC = x m
Some Applications of Trigonometry f 1 63 f
A D = A C + DC = (x + l O) m Now, AC = BC - AB= 34.64 - 20

[-: )J]
P AB
In 6.BAC. tan 30 ° = - = -
B AC Hence, rhe hei= 14.ght64ofmthe(approx.
flag staff) is 14.64 m.
Examp le 7. The distance between two vertical pillars
1 h
ran 30° =
J3 - -; is 1 00 m an d the height of one of them is double of th e
x = h f3 ... (i) other. The angles o f elevation o f their tops a t the
In !l BAD.
AB mid-point of the line 1ommg their feet are
tan 1 5 ° = -
AD complementary. Find their heights.
" Sol. Lee AB and be che two vertical pillars m apart such
CD l 00
0. 27 = -- [given, ran 1 5 ° = 0.27 ] that BD = 1 00 m. Let be the mi d -point of the line joining
0
x + 10
=> 0. 27 (x + 1 O) = h ... (ii)
the feet of rhe two pillars.
Then, BO = OD = 50 m
On purring x = J3h from Eq. (i) in Eq. (ii), we get Also, ler the height of pillar AB be h m.
.J3
0.27( h + 1 O) = h Then, CD = h 2 m [given]
=> 0.27 x .J3h
+ 0.27 x 1 0 = h Again, lee LAOB = 0
=> h(l - 0.27 x .J3) = 0.27 x 10 Then, LCOD = (90° - 0)
h(l - 0.27 x 1.732 ) = 2.7 [·: fj = 1.732] [since, both angles are complementary]
c

1
=> h (1 - 0.47) = 2.7
=> 0.53h = 2.7
27
h = · = 5.o9 � 5 km
0.53 2h m
i
Hence, chc he g ht of mountain is 5 km.
hm
Example 6 . Th ere is a flag staff on a tower of height
20 m. At a point on the ground, the angles of elevation l8 --�.l.-----' O
so m -0-- so m

·: can0 = �]
-----1 00 m
of the foot and top of the flag are 45 ° and 60 ° , - - ­

----

respectively. Find the height of the flag staff.


In right angled MBO,
1 J . . (i
h [
Sol. Lee AB be r c rower and A C be rh
flag staff on chc rower. Let D
point on chc ground, where rhe
a be e c
tan 0 = BO AB !!__
50

angles of elevation of foot A and top


[
C of rhe flag staff are 45° and 60°,
A
l cot 0 = -50h ·: 0 tan0 cor )== - .

respectively. E
0 and in right angled � CDO ,
Then, we have C\J

i ____ ____,'--'-=-....__.. CD 2 h
tan (90° - 0) = -
[ ;]
AB = 20 m , LADB = 45° B D =-
and LCDB = 60° OD 50
In right angled MBD, => cot 0 = -250h [·: tan (90°- 9) = cot 0] ... (ii)
AB
can 45°= - -: ran 0 =
BD From Eqs. (i) and (ii) , we get
-50h = -250h
J5 �
20
=> I=
BD
=> BD = 20 m [ ·: ran 45° = l] 2h 2 == 50 x 50
and in right angled �CBD, o 50
ran 60°= ­
BC => h= [taking positive square root]
h = 2 5 .fi.
BD
.J3 = BC [·: can 60 ° = .J3] =>
AB = 25 .fi.
=>
20 m
=> BC = 20 .J3 = 20 x 1.732 [·: .J3 = 1 .732] and CD = 2 25.J2 = 50 .J2 m
x

= 34.64 m (approx.) Hence, the height of the pil ars are 25 ..Ji. m and 50..Ji. m.
1 1 64 1 Allin.one MATH EMATICS Class 10th Term II

Example 8. An a eroplane is a t an altitude of 1200 m. Sol. Lee A be the aeroplane and AD be ics hcighc.
Find tha t two s hips are sailing towards it in the same Again, lee B and C be two co n s ccu ci vc kilometre scones o n the
direction. The a n gles of depression of the ships as road on the left and right of p lane A a nd t he angles of
observed from the a eroplane are 60°
and 3 0 °, depress i on of C and B from p lane A arc and 45•,
60°
respectively.
respectively. Fin d the distance between both ships.
Sol. Let the aeroplane be at B and, two ships be at C and D such
Then, LABC = LPAB = 4 5 ° c
[ al e rn ac e angles]
60° = 60°
that their angles of depression from B a re and
respectively. Then, the angles of elevation of D and C
are 30° and 60°, respectively.
B from30°, and
LACB = LQA C
BC = I km
[alcernacc angles]

1
Let BD = x km
B -
DC = BC BD = ( I - x ) km . . . (i)

l
1 200 m

D
---- y m ---
We have, AB = 1200 m
Let AC = x m and CD = y m
In right angled f).BA C , we have
tan 60 0 = P =
AB => .J3 = 1 200 [·: can 60° .J3J
=
In right angled MDB,
B AC x
tan 45° = - =
P AD
-

=> 1 200
x = -- x - .J3 [rationalising]
B BD
AD
.J3 .J3 I=
x
[·: tan 4 5 ° = 1]
=>
x = l 200.J3
3 = 400 .J3 m
... (i) :::) AD =x
and in right angled MDC,
.J3 = _2._

[ )3]
In right angled !).BAD, we have
AB
AD
30° AD AB tan 60°
= =>
1 -x
DC + CA
tan = -- = ---- [·: AD = DC + CA] DC
[·: tan 60° = .J3 and from Eq. (i)]
=>
Ir;; = 1 200
-- => x + y = I 200 vr;;3 ·: tan 30° = ::::> .J3 -.J3x = x => .J3 = J3x
+x
v3 x +y
=>
1 200./3 -
y= x . . . (ii) ::::> (.,fj + I ) x = .J3
On putting the value x from Eq. (i) in Eq. (ii) , e gee
of w
:::) x = .Jj
--

y = 1 200.J3 - 400 .Jj = 8oo.J3 .J3 + l


= 800 x 1.732 [·: .J3 = 1 .732] = -r:;- .J3 - I ::- [b y ranona
.J3 x ---,: . · . g]
1 1sm
= 1 38 5.6 m v3 + I v 3 - 1
Hence, the distance between both ships is 1385.6 m. - 3 -2 .J3 2 3 - .J3
Example 9. The angles of depression of two <.J3 > -(1) 2
[· . . .J3 = 1. 732]
consecu tive kilome tre stones on the road on right and 3 - 1 .732
lei t of an a eroplane are 60° and 4 5°, respectively as 2
observed from the a eroplane. Find the height of the = 0.634 km
a eroplane. Hence, the height of the aeroplane is 0.634 km.
FO D E R 5 .1
Now, AC = + = AB + fj BD [·.· BC = BD]
Jj8 + Jj16 = fj24 x fj
1 A circus artist is climbing a AB BC
20 m long rope, which is
=
tightly stretched and tied
from the top of a vertical = B f3 m
pole to the ground. Find the
height of the pole, if the
Hence, the height of the tree is sfj m.
angle made by the rope with B c 3 A contractor plans to install two slides for the
ground level is 30° (see the figure) . children to play in a park. For the children
Sol. In the given figure, AB is the height of the pole and below the age of 5 yr, she prefers to have a slide
AC =20 m is the length of rope, which is tied from the top
of the pole to the ground at point C.
whose top is at a height of 1 .5 m and is inclined
at an angle of 30° to the ground, whereas for
In right angled MBC, elder children, she wants to have a steep slide
0 at a height of 3 m and incline d at an angle of 60°
sm30
• P
= - =- =-
AB
H
AB
AC 20 to the ground. What should be the length o f the
-
I - slides in each case?
=> =
AB
2 2200 Sol. In rigyr.ht angled ic is slide for the children below age
of5 /:J.ABC,

= - = IO m
2 Let
inclined at1.5LCA.B = to the ground.
m be che height of the slide and slide is
AB
BC = AC

LJ1sm
Hence, che height of the pole is I m. 0 30°
2 A tree breaks due to storm and the broken part
bends , so that the top of the tree touches the
ground making an angle 30° with it. The

[·: sin9 = ;J
distance between the foot of the tree to the
A
point, where the top touches the ground is 8 m. B

Find the height of the tree. In right angled ll.ABC,

[·: in °= �]
Sol. Let che height of the tree be AC. When che storm come, the • 0 BC
tree breaks from point B. The broken part BC of the tree sm 30 = -
AC
touches che ground ac point D making an angle 30° on the 1 1.5
ground. c
.. .. .. ==> s 30
.. .. ..
..
2 AC

tan 30° : [· : tan 9 �]


... ... ... ...
In right angled aPQR,mit i the slide for the elder children.
So, BC = BD
==>
i n, 8 m
Also g ve AD = ,
',,,''
AC = 3
s
In right angled !::. BAD, B

= =
\
\
\
Let
incliRQ be the hei60°
ned at anm LRPQ
=3 ght toofthethe ground.
=
slide and slide PR is

I
\
\
0 \
In right angled MQR,
[·: an 0 }3J
- B m ___.:. o
sin 60° = RQ
R

..f3 2_ [·: sin60° fj ]


A
=> =
I AB
J3 s PR 0:::
8 u.J

[·: !J
=> = t 3 °= 3m

I
==>
AB m
Jj 2 PR =
2 = Cl
-'
J./
Again, in right angled /:J.BAD,
AD cos 9 = ==>
3 p
0

�]
LL-
cos 30° = - PR =
BD
6 x fj
� 1�
=>
.J3
- =-
2
8 [·: cos 30° = - -

- .J3 Jj
-

BD

=> =
= 2.J3 m [by rationalising]
BD
Hence, length slides in each case m and 2.J3 m .
of are 3
AlllnOne MATH EMATICS Class 10th Term II

4 The a n gle of elevation of the top of a tower from Sol. Lee AB = 30 m be che height of the building and
DC =1.5 be rhecheheilineghtDFof lchel CAboy. The point D
30°. 30
a point o n the ground, which is m away from m b e the
the foot of the tower, is Find the height of boy'
Then,s eye. Draw .
the tower. CD = AF = 1.5 m
Let BC be the height of the tower c The angle of elevation is LBDF = 30 °.
the
Sol.
which
AB. LetisAstanding
be a pointononthethe ground
groundof
Lee xm
= 60°. che building. Then,
anglhee ofwalelekvatied oDEn is=LBEFtowards
which makes an elevation
30° ro the rop of the rower. angle Lee EF =y m

Also, = 30 m given] Now, BF = AB - AF =30 - 1.5 = 28.5 m

In right anglABed MBC, [ B


t
I
tan 30° = - BC [·: cane=�] I
28. 5 m
AB o F
I = BC l � i
30° 6 0' J 30 m
.J3
30 .J3 [·: can30° = �] - x m -· -- y m -- ·


!
1 .5 m , 1 .5 m
:

=> BC = .J330 .J3 =I Ov3


x
r:;
m [by rationalising] c
In right angled l:lBFD, P F BF

[ 1]
Hence, rhe height of rhe rower is o.J3 I m.
tan 30° = -B = - B = ---
5 A kite is flying at a height of 60
m above the
DF DE+ EF
ground . The string attached to the kite is => I - 28.5 can 30° = r.; ·.·
temporarily tied to a point on the ground. The f3 + x y 3 v:>
inclination of the string with the ground is 60°. => x + y = 28.S ./3 m . (i )
. .

1
Find the length of the
string, assuming that there c and in right angled l!:r.BFE, BF
is no slack in the string . tan 60° = EF
be rhebe position of theof kithece .J3 = 28.5

I
Sol. Lee C
and AC the length 60 m => y [·.· can60°=.J3]
sering
60° on which
the makesThean heiangleght ofof
ground. 28.5 m
the kite m.from the ground 1s => y =--
.J3
A
BC =60 B
0
28. 5
n pumng y = .J3 1 , we get
. .
m E q.
(")
In right angled MBC, BC

=>x=28.5 (,/3 - �)= 2s.s ( 3�1 ) [·.· can60° = �]


sin 60° = .!..._ = AC
H x + 28..J35 = 28.S v3 r:;

.J3 60
2 AC
2
Ac -- 60.J3 .J3.J3
x
x [by rationalising] 28.J35 2 .J3 x
x [by rationalising]
.J3
= 1203.J3 = 4o.J3 m
= S?.J3 =I9J3 m

Hence , chc length of rhe sering is 4o.J3 m.


3
Hence,m.che distance he walked cowards che building, is
6 A 1.5 m tall boy is standing at some distance 19 .J3
from a 30
m tall building. The angle of
elevation from his eye to the top of the building
7 From a point on the groun d , the angles of
elevation of the bottom and the top of a
i ncreases from 30° to 60° as he walks towards
transmission tower fixed at the top of a 20 m
�-
the b uilding . Find the distance he walked
toward s the building .
high building are
Find the height of the tower.
45°
and 60°, respectively.
Some Applications of Trigonometry

Sol Lee BC = 20
DC = h m be chc heighc of che building and
m be chc hcighc of cransmission cower, which is
- "\/r-::>3 = h +x 1.6 h = v3x - 1.6 . . . (i)
=> r-;

scanding on chc building.


In right angled 6.CBA , tan 45° =
BC
Again, A be a fixed po inc on chc ground such char che angles AB
of elcvacion of chc boccom and chc cop of che cransmission
tower arc LBAC 45° and LBAD =60°.
=
I = j:x [·.· tan 45 ° = I ]
D =:> x =h
h m (Tower) On putting x = h in Eq. (i) , we get
h = .J3 h -1.6
.J3 - I ) = 1.6
c :=) h(
20 m (Building)
h = ( .J31.6- l) x .J3
.J3 + I [ by rationalisi ng]
+1

AB [·: ran 9 = %]
1. 6 r-;
= -2 ( v3 + = 0.8 ( v 3 + I) m
1) r-;

Also, let AB = x m
Hence, the height of the pedescal is 0.8( J3 + I) m .
In righc angled M BC , can 5 ° 4 = BC 9 The angle of elevation of the top of a building
l = 20x [·: tan 45° = I ] from the foot of the tower is 3 0 ° and the angle of
elevation of the top of the tower from the foot of
:=) x 20 mBD BC +CD. . . (i)
=
the building is 60 ° . If the tower is 5 0 m
high, then find the height of the building.
and in right angled MBD, tan 60° = = AB
AB CCE 2009

I
BC =50
v:> = -- 20 + h [·: tan 60° = v3] Sol. Let m be the
r-; r-;
:=) height of the tower an·d C·
r-;v3:> --20x+ h [from Eq. (i)] AD =h m be the height of
the building. Angle of
20 elevation of the top of the D E
20 + h = 20J3 building from the foot of
LDBA = 30° r
h = 20 ( J3 -1) = 20(1.732-1) [·: J3 =1.732] the tower is hm
= 20 x 0.732 =14.64 m and angle of elevation of
the top of the cower l 1L....1..:....----'--..::io
Hence, the height of the tower is 14. 64 m. from the foot of the A ·-- x m

8 A statue, 1 . 6 m tall, stands on the top of a


pedestal. From a point on the ground, the angle
building
Also, let ABLCAB= =60°.
x m be the distance between foots of che

[·: ran30° )3]


tower and the building.
of elevation of the top of the statue is 60° and
In right angled 8BAD, tan 30° =
AD
from the same point, the angle of elevation of
the top of the pedestal is 45 ° . Find the height of
AB
the pedestal. =

Sol. Let BC = hm be the height of D


the pedestal and CD = 1.6
be the length of the scatue,
m . . . (i)

which is standing on the


pedestal. and in right agnled 8CBA, can 60° = BC
AB
A
Again, let point be a fixed
point on the ground such that [ ·: can60° = .fj ]
45°
the angles of elevation of the __
top of the statue and bottom Ac;.___,_...__ x _
_ m -
__ -"-..&...J a =:> .\" =
50
.J3 Ill

. = J350 .
of the statue (i.e. top of the
pedescal) are LDAB = 60° andLCAB =.45°. 0 n putting x 111 Eq. (')
1 ,
we get
AB = x m
h = v5�3 v3� = S3O �3 m
Also, let
In right angled MBD , tan 60° =
BD x = 16
AB
BC
.J3 = x + CD [·: tan 60° = J3] Hence, the height of the building is 1 6 �
,,
m.
1 68 ( AllinOne MATH EMATICS Class 1 0th Term R

1 0 1\vo poles of equal heights are standing the tower, the angle of eleva tion o f the top of the
opposite to each other on either side of the road, tower is 30°(see the figure) . Fi nd the height of
which is 80m wide. From a point between them the tower and the width of the ca nal.
on the road, the angles of elevation of the top of Sol Let = x m be the widch of chc canal and AB = h m be
BC
the poles are 60° and 3 0°, respectively. Find the the height of the cower.
h eight of the poles and the distances of the Given, LACB = 60° and LADB = 30°
point from the poles. CBSE 201 5 A
Sol. Let AB = 80 m be the
width of the road. On
both sides of the road,
poles AE = BD = h m
are standing. Let C be
any point on AB such
that from point C,
angles of elevation are
LBCD = 60 °, and
LACE = 30 °. In right angled MED, can 30° = A B
Let BC = x m.
DB

[·: ran 30° = ..)J and DB = DC +CB]


h
Then, AC = AB - BC = (80 - x) m
I
=
In right angled /l CAE,
.J3 DC + CB

1 = h
[·: �]
AE
tan 30° = ­
AC

20 + x = ../3h
I = h. mn 3 0° =
.J3 8 0 - x .J3 20 + x

80 - x = h./3 (i) . ..

[·: tan60°= ../3]


h .J3 + x = 80 . . . (i) and in right angled MBC,
and in right angled !lCBD,
AB => .J3 = !!_
tan 60° = [·: tan 60° = .J3 J
BD ../3 !!_ BC x
tan 60° = => = => h = .J3x
BC x . . . (ii)
=> h = .J3x . . . (ii) On putting h = .J3x in Eq. ( i), we gee
20 + x = .J3 (./3x)

h
On putting h = ../3x in Eq. (i), we get
=>
../3x ( .J3) + x == 80 20 + x = 3x
=>
=>
2x = 20 => x = 1 0 m
=>
3x + x = BO
4x = 80 On putting x = 10 m in Eq. (ii), we get = .J3 (1 0)
=> x = 20 m => h = IO .J3 m
On putting x = 20 m i n Eq. (ii), we get Hence, the height of the cower is 1 o.,/3 and width of the
m

h = 20../3 m canal is IO m.
Now, AC = 80 - x = 80 - 20 = 60 m 1 2 From the top of a 7 m high building, the angle
Hence, height of the poles is 20 .J3 m and the distances of of elevation of the top of a cable tower is 60° and
the point C fro m the poles are 60 m and 20 m. the angle of depression of its foot is 45°.
11 A
Determine the height of the tower.
A
c
TV tower stands
vertically on a bank of a Sol. Lee AD = 7m be the
canal. From a point on
height of the building
the other bank directly
and BC = h m be the
�-+-------1 £ hm
height of the cable
opposite the tower, the tower. From the top

j
angle of elevation of of the building D, the t
?m
the top of the tower is
60 ° . From another point
angles of elevation l
and depression are
2 0 m away from this B LCDE =60° and
p oint on the line joining this point to the foot of LEDB = 45°.
Some Applications of Trigonometry J 1 69 J
the poinc D, draw a line DE 11 AB. =>
x = 75 mxin= Eq.75 m(i), we get
From
Then, L.EDB = L..ABD = 45° [alcernaceangles] O n p u rring
Also,ler AB = DE = x m be che becween
w
di ranee b u i ld i ng 75 + y = 75J3
and r r.
angled D.BAD, 7 => y = 75 (.J3 -l) m
(
I n righ t

ran 45° = - AD ::::::> I = - [·: can 45°= I] Hence, the distance between two ships is 75 .J3 - 1) m.

AB x 1 4 A 1 .2 m tall girl spots a balloon m oving with the


:::::> x=7m ... (i) wind in a horizontal line at a height o f 88.2 m
. 0
from the ground. The angle of elevation of the
I c fl. CED,
right ang d can 60 = - DE = ---
CE CB - BE
and i n

- 7 AB balloon from the eyes of the girl at any instant is


60° . After
:::::> .J3 = h x h -7 = x.J3
=> sometime, the
angle of elevation
h = x.J3 + 7 h = 7.fj + 7 [from Eq. (i)]
::::::>
reduces to 30° (see
:::::> h = 7(J3 + 1) m the figure) . Find
Hence, the height of the cower is 7(J3 + I) m. the distance
travelled by the
1 3 As observed from the top of a 7 5 m high balloon during the
lighthouse from the sea level, the angles of interval. CCE 2009
AD =1.2 be
/
depression of two ships are 3 0° and 45 ° . If one Sol. Let m

online thel giABrhori /


the tal l standing
�i 8811 .2 m
ship is exactly behind the other on the same
side of the lighthouse, then find the distance
between the two ships .
Lee CD = 75 m be che FH =EB =88.
zontal
beballotheon fromheigcheht lineof /���) - /
2 and
m ,
'
I I
I
: )Ti , '
,,
''
' I
:
I

I
Sol. 30°
heifromghttheof chcsea light house 0 ---------------- - 0

l e vel 45·
LeepositiAonsandof twoB ships be AC.theon AB.giAtrl D,thetheeyesanglofes o/.;1 1'j_.2 )��
the - -----------
m �Q-
- �C

l
I · I

75 m ofL.FDCele=60°vation are A -x m ym B -

the sea level. H

From
ldepressi pointtheDanglofestheof
ighthouse, and L.EDC =30°. = EC =88.2 -1.2 =87m
o n of two ships A ,
Nowthe distance FG
and B arcLODA 30° and L.ODB = 45°. AH =x m. travelled by the balloon, HB = y and
Lee m

. . L.CAD = LODA =30°


=

[alcernaceangles] :. DG = x m and GC = y m FG 0

and L.CBD = LODB = 45° [alcernaceangles] In right angled D.FGD, tan 60 = DG


Let distance betweenBCtwo=xshim.ps be AB = m
and
y r:;
v :> = 88.2x-1.2 [·: tan60° = .J3]
In right angled MCD, CD ... (i)
can 30°= - AC In right angled MCD, EC
1 = 75 can 30°=­
.J3 AB + BC DC87
[·: tan30° = )J and AB+ BC]
AC =
I
.J3 - DG + GC
1 = 75
.fj y + x [·: tan30° = )J and DC = DG + GC J
x + y = 75.fj ". (i) l
=
87
fj x + y
yx+ =B7.f3 ... (ii)
1 = 75x [·: tan 45°= l ] On putting x =� from Eq. (i) in Eq. (ii), we get
\ 170 ' AllinOne MATH EMATICS Class 1 0th Term

( - ]3 J
£ + y = 87 .J3 x + .Y = h./3
x + y = .fjx ( JJ ) [ from Eq . (i)]
.J3

=> y = 87 ,/3 =:) x + )' = 3x . . (ii)


.

It is given that a car moves from poi n c A ro B i n six seconds.


=> y=
87 (3 -1) Let its speed be k km/s.
.J3 . = Discance
Tame
87 x 2
.J3 87 x 2 .J3 S peed
-- .J3
-- x- = ----

.J3 3 6 = -) => _y = 6 k
'

[by rationalising] k

= 29 x 2 .J3 = 58 .J3 m
From Eq. (ii), we gee
x + 6 k = 3x =:) 6 k = 2x x 3 I.: => =

Hence, the distance travelled by the balloon during the Tame = Speed = -I.: = I.: = .J S
. Discancc x 3 k ').
-
interval is 5 8.J3 m.
1 5 A s traight highway leads to the foot of a tower. A Hence, the moves from poi nr !J po i n t C i n 3 s.
car ro

man stand ing a t the top of the tower observes a 1 6 The angles of elevatio n of the top of a tower
car a t a n a ngle of d epression of 30°, which is from two points at a dista nce of 4 m and 9 m
a pproaching the foot of the tower with a from the base of the tower a n d i n the same
uniform spee d . Six s econds later, the angle of straight line with it are com pl e m e n ta ry. Prove
d epres sion o f the car is found to be 60°. Find the that the height of the tower is 6 m .
time t aken by the car to reach the foot of the D
tower from this p oint. CCE 2008
Sol. Lee CD = h m be the

h
Let CD = h m be the height of the tower. At point D of the height of the cower.
Sol.
rower, a man is standing and observes the car at an angle of AC be a horizontal
depression of 30°. After six seconds, the angle of depression line on a ground. A m
and B be the two
j
of the car is 60°. points on a line at
i.e. LODA = 30°and LODB = 60° a distance of 9 m and 0
=> LDA C = LODA = 30° [alternate angles]
4m
4 m respectively from A a 4- C -

and LDBC = LODB = 60° [alternate angles] the base of the 9


4- - - - - . -- m ----

tower C.
Let LCBD ·= 9, then LCAD = (90° 8 ) -

[·: complementary means the sum of rwo angles is 90°)


In right angled MCD,

j
hm tan (90° - 0) = - CD
AC
=> cot e = !!... [·: tan (90°- 0) = cot 0 ] . . (i)
so•
.

30° 9
A y m -- a
- - x m- C CD
Let AB = y m and BC = x m
and in right angled llBCD, ran 8 = - BC
In right angled l:iBCD, tan e = - h . . . (ii)
CD
tan 60° = ­ 4
BC On multiplying Eqs. (i) and (ii), we gee

[-: __!__]
ct::: h CO[
h h
0 X tan 0 = - X -
u.J .J3 x
=- [·: tan60° = vf3] 9 4
Cl h = .J3x . . . (i) h2
_jj =:) I =- cot 0 = can e
=>
In right angled MCD, 36
0

[·: tan 30° = ]31


LL tan 30° = CD = CD
-- ---­ [·: AC =AB + BC] => h 2 = 36
AC AB + BC => h =6 m

�, h [on caking p ositive square root]


l
= Hence, che height of the cower is 6 m. Hence proved.
iULJ
.fj x+y
I Very Short A nswer Type Questi ons [ 1 M a rk e a c h ]

I
1 . I f the length of the shadow o f a tower is 1 1. The figure shows the
A

r
increasing, then the angle of elevation of observation of p oint C
the Sun is also increasing. Is it true? Justify from point A. Find the
your answer. NCERT Exemplar angle of depression from
A. CCE 2013
2. A ramp for disabled people in a hospital C'"-
-4-�
-=3-m _,__, a
__

have slope not more than 30°. If the height


of the ramp be 1 m , then find the length of 12. The angle of depression of a car standing o n
ramp. the ground, from th e top o f a 75 m high
tower, is 30°. Find the distance of the car
3. Find the height of a tree, if it casts a shadow from the base of the tower. CCE 201 3

17 m long on the level of ground, when the


angle of elevation of the Sun is 45°. 13. The angle o f elevation of the top o f a building
150 m high, from a point on the ground is
4. From a point 20 m away from the foot of a 45°. Find the distance o f the p oint from foot o f
tower, the angle of elevation of the top of the building.
the tower is 30°. Find the height of the
tower. [take , .J3 = 1 . 732 ) 14. The tops of two poles of height 20 m and
14 m are connected by a wire. I f the wire
5. A ladder, leaning against a wall, makes an makes an angle of 30° with the horizontal,
angle of 60° with the horizontal. If the foot then find the length of the wire.
of the ladder is 2 .5 m away from the wall,
then find the length of the ladder. 1 5. Find the length of the shadow on the ground
CCE 2016 of a pole of height 6 m when the angle of
6. If the height and length of the shadow of a elevation 0 of the Sun is such that tan 0 = �.
man are the same, then find the angle of 4
CCE 201 3
elevation of the Sun. CCE 2012, 1 1
16. Find the angle o f elevation o f the Sun when
7. A pole casts a shadow o f length 2../3 m on
the shadow of a pole h m high is .J3h m long.
the ground, when the Sun's elevation is
CCE 2009; NCERT Exempl ar
60° . Find the height of the pole. CCE 2015
8. If the Sun's angle of elevation is 60° and 1 7. A circus artist is climbing o n a 1 0 m long
height of the pole is 9 ../3 m, then find the rope which is tightly stretched and tied from
length of the shadow. the top of a vertical p ole to the ground. If the
angle made by the rope with the ground
9. The angle of depression of car parked on level is 45°, then find the height of pole.
the road from the top of a 150 m high tower
is 30°. Find the distance of the car from the 18. A kite is flying at a height of 30 m from the
tower. CCE 2014 ground. The length of string from the kite to
the ground is 60 m. Assuming that there is n o
10. A tower stands vertically on the ground. slack i n the string, find the angle o f elevation
From a point on the ground which is 20 m of the kite at the ground.
away from of the foot of the tower, the angle
of elevation of the top of the tower is found 1 9. Find the length of the shadow of a tree 8 m
to be 60°. Find the height of the tower. long when the Sun's angle of elevation v�
CCE 2010 is 45 °. N CERT Exemplar f��)

� �··:-�
172 ) Allinone MATH EMATICS Class 10th Term '

2 0 . A circus a rtist is climbing on a rope 12 m long, 23. A tower s t a n d s v e rt i ca l l y o n the gro und
which is tightly stretched and tied from the From a point on t h e 9 ro u n d , w h i c h is 60 m
top of a vertical pole to the ground. Find the away from t h e fo o t o f t h e t o wer the ,

height of the pole, if the angle made by the angle o f e l eva ti o n o f t h e t o p o f the tower
rope with the ground is 60°. is found to be 3 0 ° Fi n d t h e h e igh t of the
.

tower.
2 1 . A tower stands near an airport. The angle of
elevation 9 of the tower from a point on the 24. A circus artist i s c l i m b i n g f ro m t h e ground
ground is such that its tangent is � . Find the along a rope s t re t c h e d f ro m t he top of
12 a vertical pole a n d t i e d a t t h e ground.
height of t h e tower, i f the distance o f the The height o f t h e pole i s 1 2 m and the
observer from the tower is 120 m . CCE 201 5 angle m a d e by t h e rop e w i t h ground
level is 3 0°. C a l c u l a t e t h e distance
2 2 . I n t h e given figure, AB = 1 0 .JJ cm, DC = 8 cm covered by the a rt i s t i n c l i m b i n g to the top
and BD = 6 cm, then find 9. CCE 2015 of the pole.
c
25. A ladder 15 m long j u s t r e Cl c h e s the top of
a vertical wall. I f the l d Ci cl e r makes an
D angle of 60° w i t h t h e wa l l , t h e n find the
height of the wall . CCE 2013

� I Short A nswer Type I Questions


A

[ 2 M a rks each

26. In right angled MBC, AC is hypotenuse,


29. A player sitting o n the top o f a tower of
height 20 m obs e rves t h e angle of
AB = 1 2 cm and LBAC = 30°. Then, find the
l e ngth of the side B C.
depression of a ball l y i n g on the ground as
2 7. A tree is b roken by the wind . Its top struck the 60°. Find the d i s ta n c e between the foot of
the tower and the ball.
ground a t a n angle of 60° and at a distance of
3 0 m from the root of the tree. Find the whole 30. An observer, 1 .5 m tall, is 2 0 . 5 m away
from a tower 22 m h igh . Determ ine the
height of the tree. CCE 20 1 1
angle of elevation of the t o p of the tower
28. A window i n a building is at a height of 1 0 m from the eye of the obse rver.
from the ground . The angle of depression of a NCERT Exemplar; CCE 201 2
point P on the ground from the window is 30°. 3 1. If two towers of heights x m a n d y Ill
The angle of elevation of the top of the subtend angles of 3 0 ° and 6 0 ° respectively
building from the point P is 60°. Find the at the centre of a line j oi ning their feet ,
height of the building. then find the ratio of x : y. CCE 20 1 5
CCE 2007
Some Applications of Trigonometry f 1 13 I

� I Short A nswer Tvpe I I Questi ons [ 3 M a rks e a c h ]

32. A peacock is sitting on the top of a tree. It 40. The angles of elevation and d epression of the
observes a serpent on the ground making top and bottom of a lighthouse from the top of
an angle of depression of 30°. The peacock a 60 m high building are 3 0° and 6 0°,
catches the serpent in 1 2 s with the speed respectively. Find the difference b etween the
of 300 m/min. What is the height of the heights of the lighthouse and building.
tree? CCE 2015
41. From the top of a building 60 m high, the
angles of depression of the top and b ottom o f
33. A person standing on the bank of a river a tower are observed t o be 4 5 ° a n d 6 0°,
observes that the angle of elevation of the respectively. Then, find the height of the
top of tree standing on the opposite bank tower. [take, .J3 =1 . 732] CCE 20 1 4
is 60°. When he moves 40 m away from the
bank, he finds the angle of elevation to be 42. The angle o f elevation of an aeroplane from a
30°. Find the height of the tree and the point on the ground is 45°. After flying for
width of the river. [take, .J3 = 1 . 732) 15 s, the angle of elevation changes to 3 0°. If
CCE 2008 the aeroplane is flying at a constant height of
2500 m, then find the average speed of the
34. The angles of depression of the top and aeroplane. CCE 201 3
bottom of a tower as seen from the top of a 43. A person standing on the bank of a river
60 .J3 m high lift are 45° and 60°, observes that the angle subtended by a tree
respectively. Find the height of the tower. on the opposite bank is 60°. When he moves
35. From the top of a hill, the angles of
10 m away from the bank, he finds the angle
to be 30°. Find the height of the tree and
depression of two consecutive kilometre
breadth of the river.
stones due East are found to be 30° and
45°. Find the height of the hill. CCE 2015 44. The shadow of a flag staff is three times as
long as the shadow of the flag staff, when the
36. The shadow of a tower is 30 m long, when Sun rays meet the ground at an angle of 60°.
the Sun's angle of elevation is 30°. What is Find the angle between the S un rays and the
the length of the shadow, when Sun's ground at the time of longer shadow.
45. Two ships are there in the sea on either side o f
elevation is 60°?
37. A tree is broken by the wind. The top a lighthouse i n such away that t h e ships and
struck the ground at an angle of 30° and at the base of the lighthouse are in the s ame
straight line. The angle of depression of two
a distance of 30 m from its root. Find the
whole height of the tree. [take, .J3 = 1 . 732)
ships as observed from the top of the
lighthouse are 60° and 45°. If the height of the
38. As observed from the top of a lighthouse, lighthouse is 200 m, then find the distance
100 m high above sea level, the angle of between the two ships. CCE 20 1 4

depression of a ship sailing directly 46 A man standing on the deck of a ship, which
• .

towards it, changes from 30° to 60°. is 10 m above the water level. He observes the
Determine the distance travelled by the angle of elevation of the top of a hill is 60° and
ship during the period of observation. the angle of depression of the base of the hill
[take, .J3 = 1 . 732] CCE 2008 C is 30°. Calculate the distance of the hill from
the ship and height of the hill. CCE 20 1 6
39. From the top of a tower of height 50 m, the
angles of depression of the top and bottom 4 7. An aeroplane, when flying at a height o f
of a pole are 30° and 45°, respectively. Find 4000 m from the ground, passes vertically
(i) how far the pole is from the bottom of
above another aeroplane at an instant when
the tower. the angles of elevation of two planes from the
(ii) the height of the pole. [take, .J3 = 1732]
same point on the ground are 60° and 45°,
respectively. Find the vertical distance
CCE 2015
between the aeroplanes at that instant.
Alllnone MAT H E MATI CS C la s s 1 0th Term I

48. The re is a s mall island in the middle of a 5 1 . !!i.OTSl The angles o f elevd tion o f the top of a
1 00 m wide river and a tall tree stands on tower from two points on t h e ground at a
the islan d . P and O are points directly distance a m a nd b m from the bas e of the tO\'tW
opposite to each other on two banks and and in the s a m e s t ra ig h t line aze
i n line with the tree. If the angles of complementary. Prove t h a t t h e height of th'e
elevation of the top of the tree from P and tower is Jab m .
Q are respectively 3 0° and 45°, then find
the height of the tree. [take, ..J3 = 1732] 52. [[ors! A vertical tower s ta n d s on a horizontal
plane and is surmou nted by a vertical flag staH
of height- h . At a point on the pla ne, the angles

.( - }
49 . The a ngle of elevation of a tower at a
p oint is 45°. After going 40 m towards the of elevation of the bottom a n d the top of the Dag
foot of the tower, the angle of elevation of staff are a. and J3, res ectively. Prove that the
tower becomes 60°. Then, find height of . h tan a
h e1g h t o f th e tower i s
tower. tan f3 tan a

50. The a ngle of elevation of the top O of a 53 . [HOTS) A window o f a house is h m above the
vertical tower PO from a point X on the ground. from the win dow, the angles of
ground is 6 0° . At a point R, 40 m elevation and depression o f the top and the
vertically above X, the angle of elevation bottom of anoth e r house s ituated on the
o f the top Q of tower is 45°. Find the opposite side of the lane a re fou n d to be a. and
height of the tower PO and the distance p, respectively. Prove that the h eight of the

I
PX. [take , .J3 = 1732] CCE 2016 other house is h (1 + tan a cot J3) m .

� Long A nswer Type Questions [ 4 M a rks each ]

54. The angle of elevation of the top of a 5 8. A man in a boat rowing away from a lighthouse
tower from certain point is 30°. If the 100 m high takes 2 min to c h a n ge the angle of
obse rver m oves 2 0 m towards the tower, elevation of the lighthouse from 6 0° to 45°. Find
the a ngle of elevation of the top increases the speed of boat.
by 1 5° . Fin d the height of the tower.
59. The angle of elevation 0 of the top of a

55. The length of the shadow of a tower lighthouse as seen by a p e rs o n o n the ground is
5
standing on level ground is found to be such that tan 8 = - · Whe n the p erson moves a


2 x m longer when the Sun's altitude is 12
3 0° than when it was 45°. Prove that the distance of 2 4 0 m towar d s the lighthouse, the
h eight of tower is (../3 + 1) x m. angle of elevation b e c o m e s <f>, s uch that
tan <f> = . Find the height o f the lighthouse.
56. The a ngle of elevation of the top of a 4 CCE 2013
tower at a distance of 120 m from a point
60. The angle of elevation of the top of a vertical
tower from a point o n the g ro u n d is 60°. At t\
A o n the ground is 45°. If the angle of
elevation of the top of a flagstaff fixed at
the top of the tower, at A is 60°, then find point 40 m vertically a b ov e the first point of
the height of the flagstaff. [use, ..J3 = 1 .73] observation, the angle o f elevation is 30°. Find
the height of the tower a n d the distance

,: ij·1
between the tower a n d the first point of
CCE 2014
j :;
� !
I 1, 5 7. A balloon is connected to an electric
observation.
p ole . I t is inclined at 60° to the horizontal 61. From a window 15 m high above the ground i n
: n by a cable of length 2 1 5 m . Determine a street, the angles of elevatio n and depression
I

I
ii
II
the height of the balloon from the of the top and foot of a n other house on till'
1J grou n d . Also, find the height of the opposite side of the street a re 3 0° and 45''.
respectively. Show that the h eight of tht'
'I
balloo n , i f the angle of inclination is
;j
,,
I, I'
chan g e d from 6 0° to 30°. CCE 2015 opposite house is 2 3 . 66 m . [take , .J3 = 1 .732 ]
Some Applications of Trigonometry 175
62. Two ships are sailing in the sea on the 65. A straight highway leads to the foot o f a
either side of the lighthouse. The angles of tower of height 5 0 m . From the top o f the
depression of two ships as observed from tower, the angles of depression o f two cars

( 3; )
the top of the lighthouse are 60° and 45°, standing on the highway are 3 0° and 6 0°,
respectively. If the distance between the respectively. What is distance between two
ships is 1 00
f 1 m , then find the height cars and how far is each car from the tower?

66. fHOTSJ At the foot of mountain, the


of the lighthouse. elevation of its summit is 45°. Afte r
ascending 1 000 m towards t o mountain u p
63. The angle of elevation of the top of a vertical o f a slope of 3 0° inclination, the elevation is
tower from a point on the ground is 60°. found to be 60°. Find the height o f the
From another point 1 0 m vertically above mountain.
the first, its angle of elevation is 45°. Find
the height of the tower. CBSE 201 1 67. [HOTSJ A ladder against a vertical wall at a n
inclination a t o the horizontal. Its foot i s
64. The angle of elevation of the top of a pulled away from the wall through a
building from the foot of the tower is 30° and distance p, so that its upper end slides a

-
the angle of elevation of the top of the tower distance q down the wall and then the
from the foot o f the building is 60°. If the ladder makes an angle J3 to the h orizontal.
tower is 60 m high, then find the height of p cos J3 - cos a
Sh ow th at = .

sin a - sin J3

�I Value Based Questions (VBQs)


the building. CBSE 2013 q

[ 4 M a rks ea c h ]

the ladder that she should u s e which, when


68. Residential Welfare Association of a colony inclined at an angle of 60° from the
plans to install two slides for the children to horizontal, would enable her to reach the
play in a park. For the children below the required position ? Further, how far from the
age of 5 yr, it prefers to have a slide whose foot of the pole should she place the foot of
top is at a height of 1 .5 m and is inclined at the ladder ? What value is indicated from
an angle of 30° to the ground, whereas for this question?
elder children, it wants to have a steep slide
at a height of 3 m and inclined at an angle of 70 . An army pilot is flying an aeroplane at a n
. ...-
60° to the ground. What should be the ,.,. �? •
altitude o f 1 800 m observes some suspicious
length of the slide in each case? What value activity of two ships which are sailing towards ' � · .•,3 "":':. •

is indicated from this action? it in the same directions and immediately ·-�.
�o.,.=-1

report it to the navy chief. The angles o f


69. Sunita is an electrician and she has to repair depression of the ships as observed from the
an electric fault on a pole of height 5 m. She ��L.--- '
aeroplane are 60° and 3 0 °, respectively.
needs to reach to a point on the pole 1 . 3 m ( ·)
(i) Find the distance between two ships . ' -�
below the top of the pole to u11dertake the "' ' ;;
·��
(ii) What value of the pilot is shown ?
repair work. What should be the length of
So l u ti o n s
1 . False, because when angle of elevation
length of shadow decrease and vice-versa.
increases, the
[1 ] h = 20 ./3
Let XZ b e the length of the ramp and height of the
3
2.
[·: .J3 = 1.73!]
ramp l"Z = I m. = 20----
x 1 .732
z 3
= l l .55 m
Hence, the he ight of the cower is 1 1 .55 m. [ 1 12)

/c
5. Lee the length of the ladder, A C = h m
Given , AB =25m and LCAB =60 °

Also, let LZXY = 30°


In right angled axJ'Z,
[1 /2]
hm
= 1'Z
d
. 0 Perpendicular -

A m ----+
Sin 3 0 =
Hypotenuse XZ
=>
I
- =- I
B
2 xz +-- 2.5
[1 /'2]
=>
H -.
j
.XZ = 2 m In right angled MBC,
3.
Hence, the length of the ramp is 2
Let BC = h m be the height of the tree
m. [ 1 /2] sec60°= AC
AB [
·: sec e = B'

and AB = 1 7 m be the shadow of the tree. => 2 =- h [·: sec60° =2}


c
2.5
h =2.5 2
r o.
=> x

Bl
=> h =5m
hm Hence, the length of the ladder is 5 m. [1/'2]

A 17 m
6. Lee SQ be the height and PQ be the shadow ofa m an.
According to the question,
SQ = PQ
In right angled MBC,
s o
1
0 BC
Perpendicular = -
tan 4 5 =
Base AB [1 /2]
=>
h
I =- [·: can 45°= 1] Height

l
17

1 A�
=> h =17 m
Hence, the height of the tree is 17 m. [ 1 /2] 0
4. Let AB = h m be the height of P - Shadow -- 0
[ 1 /2]
the tower and C be the point
which is 20 m away from the h m Again, lee the angle of elevation of the Sun be
In right angled t:.PQS ,
9.

= Qs]
1
foot of the tower, i.e. BC = 20
m and LBCA = 30°. 30° tan 0 = Perpendicular = QS
a 20 m --c Base PQ

�]
can 0 = QS

[·:
I n right angled MBC,
QS
AB [·: PQ
tan 30° = ­
[ ·: tan 4 5 ° :::: l )
BC
1 = .h tan 0 = I = tan 4 5 °
=> tan 30° = 0 = 45°
.J3 20 Hence, the angle of elevation of the Sun is 45°. (1 12]
=> Jj h = 20
x [1/2) 7. Lee height of the pole, BC =h m
=> h
20-
..J3 [by rationalising] Given, length of shadow, AB = 2 .J3 m
- .J3 .J3
x
Some Applications of Trigonometry ' 1 77
of
and Sun's angle clevarion,
c 60° [1/2) 30°A [alternate angles]
I
LBA C = LA CB = LDA C =

i
- - - - -- - - - - - - - - - - - - - - - - - ---
0 30°

r
hm

60" 1 C "'----"---1.-.....1 B
In righc angled [1/2]
[·: can9 = �]
4 � 2\'3 m ---+ B M.BC,

75 [·: can30o J .
In righr angled we gcr
f'...A /JC ,
can30° =
AB
BC
ran60°= - BC
AB - I
- = �
.J3 BC v:>
3

Jj = _!_ [·: ran60° = .J3 J.


. 2 J3 75.fj m
BC =
= h 2 /3 x J3 Hence,m.che discance of rhe car from base of che cower is
75.Jj
=> h = 2x3 =6 m
Hence, rhe heighc of rhc pole is 6 m. (1/2) 1 3. Lee 150 m be che height of building and angl[1/2)
AB = e of
. 9 m]
9. Lee AB =150 be rhe height of che cower and[Ansangle of
8. Do same as 7. Q.
m
elevation is 45°.
LA CB =

elevation LACB =30°. A


1
r
m

�...__
_ ......
[1/2)
____

B (1/2) In right angled MBC,


In righcCangled M BC,
tan45° = p =
AB
tan30°= AB
B BC

150 I
=
BC
1 = 150 BC
[·: tan45° = 1]
.J3 BC BC =150of chem point from fooc of rhe
150 .J3 m
BC = Hence, the di s
building is 1 50 m. tance (1 /2]
Hence, the distance of car from the tower is 150 .J3 m.
Lee 20 m be the height of first pole and
[1/2) 1 4. AB" =
14 m be the height of second pole.
1 0. Do same as 4.
Q. 0.J3 m] [Ans. 2
CD =
A
In right angled = 90°
Then, can 9 = �� = 4� [·: can9 =iJ
i
11. MBC, LB
e
Let LDA C
== = [alternate angles] [1/2]
e
6m
lE

can 9 .)J = can30° [·: can30°= .}JJ


LA CB LDA C

Im
20 m
1--'-----"-

c
14
,0
=

e
Hence, the angle of=30°
depression from point is 30°(1/2] A AB
B
Then, AE == 20 -14BE= - A B - CD [·: BE = CD]
1 2. Lee 75 m be the height of cower and the angle
AB =
of depression is LDA C = 30°. In right angled MED,=6 LADE
m = 30° [1/2]
l 11s t AllinOne MATH EMATICS Class 1 0th Term I

H AD . A
cosec 30° = - =-
p AE
D
2 =A
6
[·: cosec 30° = 2] 30 m
=> AD = 1 2 m ·

[·: sin = �]
H ence, the length of the wire is 12 m.
[1 /2] B
In right angled MBC,

c
1 5 . Lee AB be che length of the shadow and height of che
pole BC = 6 m and angle of elevation LBAC = 0. AB
Sin O = - (1 1"

1
.
0
AC
0
. 0
sm =-3

l
60
6m 1
Sin e = - = Siil
· 2
· 30 ° [·: i 3 °= .!.l
..
sn 0
2

�] .
_.__.B 0 = 30°
A ..__.....__
___

. [1 /2) Hence, the angle of cl�vation o f rhc kirc at the ground


·

%• ]
(1 12
·

BC [ is 30°.
I n righ·c angled MBC, can 0 = ·: tan 9 =
AB 1 9. Do same as Q. 3 . [Ans. 8 m)
3 - 6 20. Do same as Q. 1 7. [Ans. 6./3 m]
- = tan 0 = given[· :
·c
4 AB 21 . Let h m be the height of rhc rowe r i.e. BC = h m
6x 4 Given, A B = 1 20 m
=> AB = -- = B m
3
Hence, the length of the shadow is 8 m. ( 1 /2] 1

l.
1 6. Do same as Q. 6. Ans . 30°] [ 11
1 7. Lee PR = 1 0 m be the l � ngoth of rope and PQ = h m be
·

the heigh t of pole.


0
Given , angle o f elevaci�n ( LPRQ) = 45° B
A -- 1 20r[l _ ..
P.
In right angled MBC,

� h1 m
[
BC 0

�.
,<;::, �
tan 0 = �· = -
h
. -
AB 1 20
[·.· tan = �] (112)

I 1
=
1 0 � � ·: tan 0 = , giv en
1

R Q

;]
h = 5 x 1 20 = 5 x 1 0 = 50 m

[
[1 /2]
I n right angled MQR, 12

}i-]
PQ Hence, the height of che cower is 5 0 m.
sin 4 5 ° = ·:
[112]
sin 0 =
PR 2 2. Given, AB = 1 oJ3 cm, DC = 8 cm and BD = 6 cm
1 h In right angled ABDC,
=> [·:
= sin 45 °= BC 2 = BD 2 + DC 2 [by Pythagoras theorem)
.J2 1 0
=> BC 2 = 62 + 8 2
h = ]!!__ = � X .Ji. => BC 2 = 36 + 64 = 1 00 [1 2 )
.J2 .fi. .fi. => BC = I O cm

[ p'
[on taking positive rootl
= 5.fi. m [by rationalising] Now, in right angled MBC,

[:. �
Hence, the height of pole is 5.fi. m. BC 10 1
tan e = ·: tan 0 =
[1 /2]
= =
1 8. Lee AB = 30 m be the height of che kite from the- level AB 1 0.J3 .J3 B
o f ground and AC = 60 m be the length of sering
without slack. => tan e = tan 30° = tan 30°
Let angle of elevation be e, then
=> e = 30°
LACB = 9
Hence, the value of angle 0 is 30°. [1 12)
•1 179
[Ans.
Some Appl i cat i ons of Tr igonometry :I

23. Do same as Q. 4 . 20J3 m] Again, in right angled MBC,


0AC = - y y
24. Clearly, the distance covered by ehe circus arrisc is equal sec 60 = - ::::::> 2 = - [·:sec 60° = 2]
co the length of ehe rope AC. BC 30 30
Lee AB be the vertical pole ofheighe 12 m and angle of => y = 60 m
clevaeion LA C/3 30°.
= [1/2] Now, BD = AB + AD = x + y
A = 30./3 + 60 = 30(./3 + 2) m
l
12 m
Hence, the whole height of the tree is 30(.Jj + 2) m.
[1 1
28. Lee QS be the building and R be the position of

BJ
[·: sin 0 <J
window.
""--...._
..._ ___.__. Given, height of the window, QR = 10 m
C ___

[·: sin 30°= �]


AB LQPR = LXRP = 30° [alternate angles]
In right angled M BC, sin 30° = AC and L.SPQ = 60°
s
·

R10
- =- 12
=> 2 AC

[Ans. � ] I
=> AC = 24 m
Hence, the distance covered by circus arrisc is 24 m.
[1/2] (Window)
m

[
25. Do same as Q. 1 7. J3 m
Q

I
p ..r.::...i;�L.....-L_J
[1/2)
ln righi angled APQR, ran 30° = � -: ran 0 = ; J
[ J
26.
c
Lee x m be the length of the side
BC.
.
=>
1 = 10 ·: tan30° = �3
.J3 PQ v:>
=> PQ = I0./3 ... (i) [ 1 /2)
QS
In right angled !lPQS, tan 60° = - PQ
.J3 = QS [·: tan 60° = .J3 and from Eq. (i)]
I0./3
[1 1 => QS = 10 ./3 x ..Jj = 10 x 3 = 30 m
1 2 .J3 Hence, the height of the building is 30 m . [1 1
=> x = .Jj x [by rationalising] 29. Lee AB = 20 m be the height of cower and the ball lying
.J3 �=·1
on the ground at point C. �:--
��.;.�

= 12.J3
3
= 4.J3 cm Given, angle of depression,
LTAC = 60° = LACB [alcernace angles] [1]
Hence, the length of the side BC is 4 Jj cm. T A + - - - - - - - - - -
[1 ] In right angled MBC,
27. Let the initial height of the 60"

m
can60° = ­ AB
tree be BD. BC
Lee AD = AC = y m be its 20 20
broken pare and remaining => .J3 = BC [·: tan60°= .J3J
pare of the tree be AB = x m.
Then, => BC = .J320
BD = AB + AD = x + y
Given, c

BC = 30 m and LACB = 60° _- 20 x .J3 [by rationalising]


In right angled MBC, .J3 x .J3
can60° = BCAB = .!-. C 30 m - B
-
= 20 x31 732.

[·: .J3 = 1.732)


30
=> .J3 = -2_ [·: can 60° = J3J = 1 1.55 m
30 Hence, the distance between foot of the cower and the
x = 30./3 m [1] ball is 1 1 .54 m. [1 ]
: 1so ·
J
AllinOne MATHEMATI CS Class 1 0th Tenn l

30. Let BE = 22 m be the height of the cower and 32 . Le eCbe the position of peacock a n d A be the posim.l
AD = 1.5 m be the height of the observer. The point D of serpent.
be the observer's eye. Draw DC llAB.
:. AB = 20.5 m = DC
Let 8 be the angle make by observer's
. eye to the top of
the tower i.e. LEDC = 0.
Now, EC = BE - BC = BE - AD
= 22 - 1.5 = 20.5 m [·: BC = AD] A

1 .5 m
B
E
Given, LDCA = 30°
=> LBAC = L DCA = 30° [alccrnate angles) ( 1

[-: �� min]
Also, given speed of peacock is 300 m /min and tim�

T
is 12 s
Distance = Speed Time x

A c = 300 < � .
22 m
I 2s =
m
[· : r
1 .5 1 .5 m => AC = 60 m (1
A - 20 . s m - 8 [1 1 In right angled MBC,
CE 205
In ng. h t ang1 ed .6DC'£, tan 0 = -- = -- = 1 sin 30° = BC =:> _!_ = !!___ sin30°=
DC 20.5 AC 2 6 0
can e = tan 45° [·: can 45°= 1] h=- 60 = 30 m
2
0 = 45° Hence, the height of the tree is 30 m . (1
Hence, the angle of top of the tower from the eye of the 33. Lee DC be the tree of height h m, BC be the river o:.·
observer is 45° . (1 ) width x m and A be the position of man after movin�
31 .

�1
Let AB = x m be the height of the cower 40 m i.e. AB = 40 m.
Also, LDAC = 30° and LDBA = 6 0 °

r
and CD = y m be the height of the ocher cower and
LAEB = 30°, LCED = 60°. 0
D

l
x i ym hm
1�1 A

a-:_-:_-x-m __.c
30° 60°
_
am-E a m- C

[ �]
__,__
___..____
_
___
A 40 m
Let E be the mid-point of the line AC. _ _ .....

:. AE = EC = a m {let) ln righc anglcd llBCD. can60° = CD

[·: tan 30°= )3]


In right angled .6BAE, tan 30°= AB ·: tan 0 = BC
AE
x [·: can60° = j
.· .r
=> h = ..f3x ... (i ) [ t )
Now, in right angled MCD, tan 30° = AC DC
�· ,;,
=:> x= a . . . (i) [1]

[
J3
and in right angled llDCE, can 60°= DC
I
= x +h 40
.J3
·: tan 30°= )3 and AC = AB + BC = 40 +
-__:::
- :""
CE

a
[·: ..'l.n 60° = .J3] .\·

y = J3a ... (ii) => J3h = x + 40 ... (ii) [ 1 )


x : y = � : J3a = 1 : 3 [from Eqs. (i) and (ii)] On putting h = .J3x from Eq. (i) in Eq. (ii), we gee
Hence, the required ratio ofx :y is I : 3. J3 J?,x = x + 40 =:> 3x - x = 40
x
(11
Some Applications of Trigonometry

2x = 4 0 BD = BC + CD (x + I ) km
=

=> .\· = 20 m 30°


From Eq. (i), h = J3 x 20 = 20 J3
= 20 x I .732 [·: .J3 = 1.732, given]
= 34 .64 m

l
Hence, rhe heighr
chc river is 20 m .
of chc rrcc is 34 .64 m and widch of
[1]
h km

34. Lcr AB be rhc rower of hcighr h m and CD be the lift of


hcigh r 6o jj m . D
G iven , a ngles of
depression arc 4 5° and 60°.
i.e. LXDA = L DAE = 4 5 ° [alternace angles]
[1 1
and LXDB = LDBC = 60 °
G iven, L XAD = 30° and L X4 C = 4 5 °
L ADB = LXAD = 30°

[ �]
[alccrnace angles] 1 .e. [alternate angles]
x-----.----...,; and L A CB = LXAC = 4 5 ° [alcernate angles]

h
;�
In righc angled MBC,

ran 4 5 ° = ·: ran 0 =

h
I =- [ ·: can 4 5 ° = I ]
x
=> x= . . . (i) [1 1

h [-: ]3J
Now, in right angled MBD,

[
AB
can 30° = -

.J3h
BD
B c [1 1

h J3h
I r an 30 ° =
DC -: ran 0 =
�] .Jj - x + I

h(J3 - I)
In right angled l!! DCB, tan 60°=
BC
x +I=

h .J3-1 .f3J3
[ ·: can 60° = .J3J +I= [from Eq. (i)]

=>

h [1 1
=:>
=I
BC = 60 m

h Jj-+1 Jj +
Now, BC = AE = 60 m = _I_ x +1
[by ration alisi ng]
and DE = DC - CE +1
jj - h) m

( .J3 1 )
= (6 o [·: CE = AB = m] I I
= = km

-h
3 2

--..,....
and in righ c angled l!! DEA ,

Al 0
DE +
can 4 5 ° = - Hence, the height of rhc h i l l is km. [1]
36. Do [Ans. 0 m]
EA 2
.[3
same as Q. 3 5 . I

h
=:>
60 _
I= [·: ran 45° = I ] __ . .

60
=:> 60 .[3 - h = 60 ' •.. �.

- j
=:> .Jj - 60
= 60

h
hm

---- -----
=:> h = 60( .Jj - 1 ) m
Hence, the height of the tower is 60 ( J3 l) m.
[1] ��

35. Let AB = km be che height of the hill and C, D be c 0- x m -B •, '1

cwo consecurive scones such thac CD = I km .


30 m
Lee BC b e x km,then 37. Do same as Q. 27. [Ans. 5 1 . 96 m] ,_._. __
-
-
182 1 Class 10th Term I

Let DC = I 00 m be the height oflighchouse. A and


!
Allin.One MATH EMATI CS

38. Le e Given,heigLDEC=30°,
and ht of che LDAB = 4 5 °
cower, DB = 5 0 m
BGibevenpositions
angl e s ofdepressi
of the ship.on are 30° and 60°. .. DC=D B BC
- =
- IJIJ AE
[·: BC = AE
.and. LXDA = LDAC =30° [alternate angles] =>
In right DC=(50-y)m
angled 11/IBD. . .. (i) (1 )
LXDB = LDBC = 60° D [alternate angles] (1)
BIJ
can45° =
I
AB
x

50
1 00 m
=> I= x
=>
Hence, che requiredx =distance
50m of rhc
pole from tht
bottom of a cower be 50
[-: co<0 = !]
m.
A
(1 ]

[can30° = Jj ]
B c
[1 ]
(ii) In right angledDC!:lECD.
In right angled !:lBCD,BC can30°= -­
[·: cot 60°= �]
cot 60°=-- �£C
50-y 1
DC =>
I
.J3
=
=>
1 = BC EC
.fj 100 I = 50-y (i) andx=50 =mAB] =.\· m]
[ from Eq .

BC = 100
13
m ... (i)
50- F3=50-y50
[·.·

=>
In right angled MCD, cot 30°= - AC J3
13 = AC
DC y = SO - - x -
50 ..fj [by rationalising)
100 [·: cot30° = .J3J ..fj .J3
=> AC .J3 m = l OO ... (ii) [1] = 50- 50.[3
3
Now, diABstance= ACbetween
- BC1 two observations, =50 -28. 8 7 = 21.1 3m
Hence, che required height of the pole is = 2 l.1 3 m. (1 )
( F3 � ) = 100 ( 3�I )
= 1 oo.fj - .J3 [from Eqs. (i) and (ii)]
OO
40. Lee AB = 60 m be chc height of chc build ing and CD �
= J OO _
the. . ligLEAC
hthouse=such LACBchar= LDAE
60° = 30°.[alternate angles]
= 2v0:3 = ioo.fj 200
=
1.732 = 115.47 m
x
D
3 3
[·: J3=1.732)
60 l
period of observation is 1 15.47 m. p during the
Hence, the distance travel l e d by che shi [1 )
A tff.-..,..._____ . E
_...--1

39. Let distance of the pole from the bottom of the cower

and height of the= AB=x


pole, m = m
AE y 1B

1
D AB [·.· can ]
lnrightanglcdMBC,can60°= BC O=p
B (1 )

50 m
=> .J3 =
;� ·: can 60 ° = -{3"] [

ym
I => BC = �
J3
1 Go .J3
= 3 20.J3 m = [1 ]
Some Applications of Trigonometry

Now, in ig r �JED, ran30°= DE


h c a n gl ed
42. Let tOXionsbe ofthe rhehorizplontal
posi a ne ground;
and0
AbeandtheB bepoint
the twoof

Il
AE
observation.
[·: 30°= .fi nd BC = = 20./3J
= DE I
J3 20../3

1r m
can a AE

· => Dh� = 20m


Hence, rhc d i fference hcrwccnrhe heights oflighrhouse ""'"--'--'-�--..JL...�
.:...- ��-LI....:__
0 C D
x
and building is 20 m.
In right angled ti.OCA, cot 45°= �� [-: cot = !J
[1 )
41 . Lee CD = x m he chc hcigh r rhc
of rower and disrance Here, AC = BD =
and LBOC = LBOD =30° 2500 m, LAOC = LAOD = 45°
BD b c rwee n rhe b ld
ui in and the rower be y m.
g
X A 0

OC
l = AC [·: cot 45 = l]
OC = AC = 2500 m OD
60 m
=> [11
O>

In right angled llODB, cot 30°= BD
CD
·5
J3 = ODBD [·: cot 30° = .J3J
xm
=> OD = J3BD
· => OD = 2500.Jj m [·: AC = BD = 2500 m]
D
Now, CD =OD -OC = 2500J3 - 2500
ym -- B => CDCD==2500 (fj -1) = 2500 (1. 7 32 -1 )
Herc, LADB = LXAD = 60° [alrernare angles] => 1830.mby plane in 15 s is 1830 m.
Thus, distance covered
l
and LACE = L.xi!C = 45°
Also, heighr of the building, BA =BA60 m[alrernare angles]
[1]
[1] :.Speed of plane =-- 1830 x 60 x 60 = 439.2 km/h
In righr angled MBD, ran 60° = DB 15 1000 disranee
=> .J3 = 60y [·: ran60°= .J3J [·.· speed = nme . and
60 x -.J3 [by rarionalising] l m =--km,1
1000 l s = 60x1 60 h
=> y =- .J3 .J3 Hence,2 km/h.the average speed of the aeroplane is
Go.J3 439. [1 ]
=> y =-- 3 43. Do same as Q. 33. [Ans. 5./3m, 5 m]
44. Let AB be the flag staff and AC = x be length of its
=> y = 20.Jj m ...(i) shadow,
60°. lewhen the Sun rays meet the ground at an angle
[1 )

In right angled MEC, tan 45° = CE AE ofAgain, 0


t bethetheleangl e between the Sunof theraysflagandstaff
the
1 = ABDB-EB ground, when ngth of the shadow
is AD = 3x, and be the height of the flag staff.

h [1 ]
[·: tan6045°=-xl, = AB -EB and CE= DB]
AE
=> 1 =-- y
[·: -x
AB =60, EB = CD =x and DB = y] h
60
1 = ---;=:;- [from (i)] I
In right angled ti.CAB, tan 60° = :C [·: tan = �J
Eq.
20 v 3 1.J"-�"'--����c=--'-�..J.....JA
=> 60 - x = 20J3 -2x- -x-

=> x =60 - 20!3 0


x =60-20xl. 73225.36 m [·: ..f3 = l732]
=60-34. 6 4 =
Hence, the height of the tower is 25.36 m.
[·: tan60° = .J3 ]
[1 ]
1 84 1
ti
Allinone MATH EMATI CS Class 1 0th Term I

h = J3x . . ( i) Lee a chac


manABis scanding on chc chc.:ofhill.
a ship ac point A

AD = ----
=> 46. <leek
[1 1 such = 10 m a n d CD ht.·
Now, in rightAB angledABll.DAB, che anglfrome ofAdepression of
chc.: baseofC of
tan 9 = --
Given, the ·
i s 3 0° elevation of
tan 9 = -- DC +hCA h [·: AD = + CA] CD observed and the: a n g le:
DC
top ofLEAD
D
Then, chc =60°
h i CD ob
ll is 60°.
s e rv e d fro m A

=>
2x+x =-3x and LCAE = LBCA = 30° [alrc.:rnace angles}
J3 Lee BC = m = h = m
x x A E. DE

DI
=> tan9 =-- 3xI [from Eq. (i)] and CE = AB = 0 m I

[·: "3 = tan 30°]


=> tan9 = J3
=> e
tan = tan 30° h
e
= 30°
Hence, the angle
at the time of longer between
shadowtheisSun30°.rays and the ground [1 ]
45. Let the height of light house, PM = 200 m and let A
and
such Bthatbe thetwo angl
shipses onof depressi
the eitheron atsidAes andof liBghthouse
are 60°
can 60° = - [-: can 0 = �
and 45°, respectively. In righc angled MED, DE
.J3 = !!_x [·.· can60°= .J3J
EA

can 30 - AB ,-;; = -10 [-: can30 °= .,5]


h = J3x ...(i) (1 )
In right angled ll.ABC,
Let LXPB == LMBP y
x m and=BM45° = m[alternate angles] 0
=>

In right angled MMP, tan 60° = : [-: tan 9 =�J


=
AM I
Then, BC v 3 x
and L YPA = LMAP = 60° [alternate angles] =:> x =IO .J3 m ... (ii) [1 )
On putting x = 10..fj in Eq. (i), we get
h = .J3 10 .J3 =30 m
x

J3
=>
x200 [:.ran 60° = .J3] Now, CD = CE + DE= I 0 30 = 40 m +
Hence,heigtheht ofdistance of the hill from the ship is 1 o/j m
x =-m ... (i) [1 ] and the hill is 40 m. (1 )

In right angled ll.BMP, 47. Letis vertiPandcallyQbelbeothew Ppositions


and OP of= 4000
two aeroplanes, when Q
m.
PM Gionven,thetheground anglesbeof60°elevation
tan 45° = BM A and 45°,of respecti
P an d Qvelfromy. a point
I = 200 [:.tan 45° =I]
I
y
p

200m=BM+ .. . (ii)
Now, width of the river,= 200
=> y
1\1.A
[1 ]
AB 0 4000 m

A
y

J
AB = x + = .J3 + 200
= 200 ( "3 + I ) = 200 (0.5774 + I )
=>

[from Eqs. (i) and (ii)] �---J.....I


In right angled MOP,
tan60°=­ OP
= 200 (I.5774)=315.48 m [·: J3 =1.732] AO
.J3 = 4000
· -
Hence, rhe distance between two ships is 3 I 5.48 m. [1 ]
AO [·: can 60° = v'JJ
/
Some Applications of Trigonometry

AO = 4000 ...(i) [1 ] 100 = (Jj + l)h [·: = 100 m, given] PQ

In
J3
'1/JOQ,OQ h =� x .J3 - I [by rationalising]
righ t a ngled
J3 + 1 .J3 -1
h = 50 (.J3 -1)
tan 4 5 ° = OA h = 1 00 ( .J32 -I)
I = OQOA [:.can 45°=1]
=> Oil = OQ t ... (ii) ::::) h = 50 (1.732 -1) [·: J3 =1.732]
From Hence, the hhei=36.
ght 6ofmthe tree is 36.6 m.
Eqs. (i) a n <l (ii),4 0we ge =>

OQ = '5°3 m 49. Do same as Q. 33. [Ans. 20.J3( .J3 +1) m] [1 ]

I
[1 1
:. Venical dista n ce between che aeroplanes,
=4000 (1 - � ) = 4000 ( F�t J
p
PQ = OP - OQ = 4000 - J3 4ooo

= 4000( J3J3 l x JjJ3 J [by rationalising] r


= 4000( 3 -3.J3 J = 4000( 3 -�732 )
-

i.ee. h be ehe heiPQ


50. L e ght =of tower,
h m and PX = y m
[·: J3=1.732] G iven , RX =
and LQRS 45Q 40 m =SP, LQXP =60°
= 4000x0.4227 =1690 . 8 m = [1 /2]
Q
ehae inseam is 1690.4ism.an between the aeroplanes at
Hence, the vertical d t ce
[1 ]
48. Lee
ven,OA =bPQ30°=and100 LOQA
GiLAPO e ehe tree
mof heiandght= 45°.
hanglm.es of elevation are
(h - 40) m

h
R -------1

40 m
f
J
[": tan9=;] In ri.ght angled 6.XPQ, tan 60 ° PQ [ 9=;]
.____.._
X ym-P [ 1 /2]

J3 = -yh [ ·: tan 60 °= .J3 ]


In right angled 6.POA, OA �· .·

J3 OP [·: tan30°= �J
= XP ·: tan
ran30° =-0P '-·-

"'-:.: -
' -....,

I=h ::::) -
OP = J3h ...(i) [1 ) ::::)
y = .J3h .. . (i)
Now, in right angled 6.QOA,
OA 1 =-[· h : tan45° =1] In right angledQS6.RSQ,
tan45°=- ·

OQ OQ ::::>
ran 45° = RS ran 45° = PQ -SP
::::>
XP
=> OQ = h . . (ii) [1 ] [ · : QS = PQ -SP and RS = .XP ] :-- ' =-�

1=h y
---··

On adding Eqs. (i) and (ii), we get =>


- 4o [ ·. ·
ran 5 ° = l] 4
OP + OQ =J3h + h . l .
-
-:-: - •
.

=> PQ =(.f3 + l)h [·: OP+OQ =PQ] y = h - 40 . . , ( i i) [1 ] I


Allinone MATH EMATI CS C lass 1 0th Term l

From Eqs. (i) and (iih ) , we get Lee H


chc cower be a n<l OR =
Giandven,cheLPheiheiROgghtht= ofofflag x.
52.
-
.J3
h -h =-40
= h - 40 [:.can 45°=1] � LFROscaff J3 h PO
=
= = Ff>

u H
- Now, in righc angled tl.POR, can RO = =
.J3 :c

h - ./3h = - 40 => h(l - ...; 3 ) = - 40...; 3


� � => x = H
-- . . . (i) [1
.J3 40.[3 and in righc angled !J.fOR.
ca n a
, FO FP + PO
=
� � can = -R-0-
- h( v � -1) = -40...; 3 => h = -,:-- J3 -
R0
-
/-/
h=� 40.J3 x -,:::-
.J3
v3 -1
+ 1 -- [by ranonal'IS. lng]
. .
( •...
..
n.
p
_

n h + " hca+n p ... (ii) [1.]


_ -..
-
_,.
=
H

}
x
v 3 -1 v3 + 1 F

h = 4o.J33c -.J31 + I ) = 20.J3c.J3 + 1) Flag staff

= 20 x 1.732(1.732 + 1) [·: .J3 = 1.732] p

= 20 x 1 .732 x 2.732 = 94 .64 m


On purring h = 94.64 m in Eq. (ii), we gee
y
H
= 94.64 -40 = 54.64 m
distance PXis 54.ght64ofm.the rower PQ is 94.64 m and che
Hence, the hei
Let the height the cowersuchABthat
be h m and C, D be two
[1 ] R L.......U-
__
---- x ----
___, J
51.
positions ofACcheofobserver
= am and =bm. From Eqs. (i) and (ii),H
we h H gee

ran can+ J3
AD
Let LACB = 0, then LADB (90°-9) = a
[since, the complementary means theanglsumes ofis 90°]
two => H can J3 = h ca n + H can a. a
=> H can J3 - H can = can a. h
In right angled !J.BAC, AB h => H(canf3-can a.) = hcan H = ----
a
/Jeana
can e = -=>
AC can e =-a a =>
can J3 - can u

h = a can 0 can .
81h m
... (i) [1 ] Hence, che requi. red h . h c o f cower 1s canf3-mna
e1g
. h a

Hence proved. (1 )

(90°- 9) I
53. Let che height of the ocher house OQ be H
MW
and OB = x m. =

C L--L----�---A Giandven,LQWM
height =a.,
of cheLOWM
first house= f3 == LWOB= h = MO WB
D- bm -
...:,...:..>
-.:.�

Lo..; ".:;. ;_�,

.1 :F : In right angled !J.BAD,


am [alcernace angles)[1)
-&' Q
('"
ll

.$1
;

'":""' _.;;-i�
,.
can (90°- 0) = - AB
� .... DA h

,-.
.
• .P�
· => core =-[·b : can(90-0) =coc0]
-·- ..;::: -
=> h = bcoc 0
On mulciplying Eqs. (i) and (ii), we get ... (i i X 11

[·: can 9 = -1-]


h x h2 = (a tan 0)x (b cot 0)
� h = ab can 0 x cot 0
=> b2 = ab x = abI
cot0 WB
h = J;b [caking positive square root] WB
h
r;·��:-
=> Now, in right angled tl. O , canf3 = OB = ­ -
x
Hence, che height of rhe cower is J;b m.
=> x = -- h
. . . {i\

' . ' .'
.

i
Hence proved. [1 ] canp
Some Applications of Trigonometry J 1s1 J
1 -- h [·: can30°= �J
-- WM
-h
and in right angled !lQM\'(I,
ran a = QM = --"---
OQ -MO ..J3 20 + x 3 v;)
\'(/Af 20 + x = ..J3 h [1]
=> 20 + h = ..J3 h
H
ran a = --
x
[from Eq. (ii)]
H -h ..f3 h - h = 20
x = -- . . . (ii) [1] h(../3 - 1) 20
ran a =
From Eqs. (i) and (ii), we gee ..J3 + l
h H -h h --�x
../3 - 1 ..J3 +1
can p can a racionalising] [1 /2)
[by
h ran a = ( H - h)can P

)
h = 20(../3 + 1)
(
h ran a = H can J3 - h can p 3 -1
H can J3 = h (can a. + ran J3)
20(../3 +I)
{ )
H =h
can a + can J3
can p 2
=10(../3 + 1) m
[·: ]
= 1. 1 + can a ·-tan1-J3 Hence, the requiredheighc of the tower is(I0../3 + l )m.
(1 1
= h (I + can a · cocJ3) - 1 - = coc a 55. Do sam e Q. 44 at page 1 80.
as

can 9
Hence, che required heighc of che ocher house is
h(l + can a · cocJ3). Hence proved. [1]
Lee che heighc of che cower be h m and che angle of
1
- r
54.
clevacion ac point Q is 30° i.e. LPQR = 30°.
Given, QS = 20 m and SR = m (lee) x

According co che question, ym -


B
LPSR
LPSR =LPQR
=30°+15°+ 15°
=> 56. Lee height of the tower, BC =h m and height of the
=> LPSR = 45°
Now, in righc angled llPRS,
. . . (i) [1/2) 'flagstaff CD =H m .
..Given, AB=120
BD=BC+CD=( h+H)mand LDAB=60° . . . (i)

D
can LPSR = - PR =-h LCAB=45°
m,
SR x
c
f
ran45°=� [from Eq. (i) ] Hm

..
x

I =�x [·; can 45°=1] (h +H) m

----- B I -
x =h . (i i )
hm
p (._;/:i
A ..c..--J�...L._----l
t ·i·:·
uj

[-.- tan 0 = ; ]
._..;.,---- 1 20 m
( �"::
In right angled MBC, we get v �
BC
tan45°= - <(
AB
+-- -
l =- h [ '.·
t n 45° =1]
a

Q
120
h=l20 m . . . (ii) [1 ) I
20 m S Xm R
(11 Now, m right angled MBD, we gee tan 60°= ­ BD
and in righc angled llPRQ, AB
PR = PR ..J3- h+H
can 30° = -QR QS SR + [·: QR = QS +SR] 120 [1 ]
[·: can60° = j3 and from Eq. (i)] ri
�:
Allin one MATH EMATI CS Class 1 0th Term ft

v:Jr.:3 1 20120+ H [from Eq. (ii)] [1] 58. Let the height of the l igh thouse A B be l 00 m and C D
be the positions of man when angle of elevation
l .73x l 20 =1 20 + H [ ·: .J3=I.73, given] changes from 60° co 45°, rcs pt:c r i vc:ly. The manlw

207.6-1 20=H covered a distance CD i n 2 m i n . (1)

B7.6=H ·· S peed = Ti me
D i sra nce

=> H =B7.6 m
Hence, the required height of the flagstaff is 87.6 m. [1] => S peed = -
CD
... l

,#'
2 (1
57. Let D be the initial position of the balloon and AB be
the height of the pole.
D (Balloon)
1 00 m
Lighthouse
1

[-: can 9=�


B
In right angled 6.ABC,
AB
ran 60° =
BC
A [1]

[·: sin 9=�]


Given, length of cable, DE = 2 1 5 m .J3 = 1 00
In right angled 6.EAD, => [:. ran60° = .Jj

[·: sin60°= �]
/JC
sin 60° = AD
ED => BC =
1 00 x J3 f by ration alisin...
.J3 AD J3 J3
2 215 => BC = 100 J3 ... (ii
3
AD = 2 1 5.J3
2 m In right angled MBD, can 45° = BD AB

Hence , initial height of the balloon from the ground is => I = 1BD
00 [·: can 45° = I ]
2 1 5..fj m .
2 [1] => BD = l OO m
Again, in right angled 6.EAD , Now, CD = BD - BC

[-: cos 9 = ! and DE = 21 5 m ]


AE AE
[1)

( �J
cos 60° = -
DE = - 215 = 1 00 - 1 003 J3. [from Eq. (ii)]

[·: cos 60" = i] = 100 ( 3 -/3J


= 1 00 1 -

-=2.

=>
i = :�
·'.f .) => AE = 2 1 5 m . . . (i)

[-: ]
2 From Eq. (i),
Now, the angle of inclination is changed, i.e.
(3- �.JlJ
[
:_;:: ., L CEA = 30°. 1 00.
can 9= �
] ( -/3 J m/min
r'
4' t.,'
3 .
In right angled 6.0EA C , tan 30° = AC CD
Speed = -2 2
EA

__
. : ii
-;:_'.J�.1 =>
1 = A C ·: tan 30°= 1 and from Eq. (i)
.J3
2�5 = 50 3
.J3
2_ Hence, required speed of che boat is
=> AC = 2 1 5 m S O (3 - .J3) m/min.
-;-.:::.TJ 2.Jj
c;::
[1 1
Hence, height of the balloon is � m, when an angle
--?;:·,
3 [1)
Lee che height of lighthouse AB be h m and C, D ht•
-_ ·J 21 59.
--.-.;I'-
2v3 two points of observation, such chat D C = 240 m. [1 )
1 .::-_ :: ""'�· I
makes 30°. [1 1 Le e BC =x m.
·,/
· ,:.,,: , :
Some Appl i cat i ons of Trigonometry J1s9 ·

In righc angled 11ABC, AB 61 . Do same as Q. 53 at page 174.

BC [-: ran �= �·givenJ


tan $=­ D
BC
:::) 3
-4 = - AB
hm
:::) =-34 x x
h . . (i)

62. Let CD= m be the height ofthe lighthouse and A, B


h
be twoes ofshidepression
angl ps on eitherareside60of° and
lighthouse,
4 5 °. such chat the
i.ande. LXCB = LCBD = 45° [alternate angl] es]
Given, theLYCAdistance
= LCAD
between=60°the shi[alternaceangl
ps A and B ises [1]
and in right angled MBD, AB AB = 100(.Jj.J3 + 1) m
can 9 =- BDAB In right angled llBDC,
:::)
5- = --- tan 45°=­ CD
12 DC + CB BD
[·: ran 9 = 1� . given and BD =DC + CBJ
-125 = --- 45° 60°
:::) I = BD CD [ ·: can 45° = l ]
y ... (i) [11
..
:::) h ...(i[1]i) :::) x
BD = CCD ___

240 + x
On putting h = �4 x from Eq. (i) in Eq. (ii), we gee
5- = 3
5 ( 240 + x) = 12 (% )
- x

tan 60°= AD .J3 = [·.· can 60 ° = J3 l


----'4=---
12 240 + x [1]
and in right anglCDed MDC CD
x
:::)
:::)
AD
5 x 240 + 5x = 9x �
...
:::)
r;; l3
:::) S x 240 = 4xSx 240 :::) CD =v AD :> = CD
:::) AD(ii) [11 �
.. x= 4 LU
On adding Eqs. (i) and (ii),CDwe get "S'

AB =CD (1 + )3) [·: AD + BD = ] , /,


From Eq. (i), we get =300m AD + BD = 13 +cD t/)
tl)

AB =CD ( Fjt)
w
h = �4 x 300 = 225 m => A B
Hence, the height of the lighthouse is 225 m. (1]
t/)

10o ( Fjt) =cD( F�t J


60. Do same as 50 at page 181.
Q.

�1- 40) m l
=>
[Ans. 60 m and 20.§m]

[·.· AB = 100 (l3.J3 + I ) , given ]


B
=>

r
.
__

.__._1_._._.J
CD =100 m
Hence, the height of lighthouse is 100 m. [1]
' 1 90 \ Alli-none MATH EMATICS Class 10th Term (
I

63 . Do same as Q. 50 at page 174. [Ans. 5(3 + .J3 m] 66. LetOMS.F be the foot and S be the summit of the mounta.
)
oj Then,
Consequently, LOFS = 45°OFand= OStherefore
=
h km L OSF =45°
(h - 10) m soLetDrawthat
FP = 1000 mLOFP
1-
= 1 km 30°
=
1-
be the slope,
E__..-----i C
l hm
PM OS and PL OF
JoiIt ins giPS.ven that, LMPS = 60° (1)

l l
10 m 10 m
s

Le� BC = 60 m and = h m be the heights of the


1A -
60°
xm- 8
1
64. AD
tower
From andfoot
the building,
of the respecti
tower vely. building A, the angles
Band
of elevation
Letx m be aredistance
LDBAbetween
= 30° andtowerLBAC
and =bui60°.lding i[1].e.
-.J

AB = x m. F -----'---� o
In right angled fl.DAB, L

J31 - -;h [·.- tan30°= �3]


tan 30° =- [ J ;)
h km
PL
AB
AD
· : ran0=: In right angled !l.FLP,sin30°= PF

= h-/3c
=> => PL = PFsin30°= 1 .!_2 =.!_2 km [·.- sin 30°:::::: .!_-
x
l

Now, MS = OS -OM = ( h - � ) km . (i)


=> x . . (i)
. [1 ] => OM = PL = -km I
2
l .. [1]

Fl = PFcos30°= ( 1 x � ) =�km
60 m FL
Also, in right angled !l.FLP, cos 30° = PF
l
Now, h = OS = OF = OL LF
h = OL + .J32
=>
,.__.____�....-""'
...__ B

[·: U=il
+
In right angled MBC,
OL + - � )
tan 60° BC AB =-
:=)
2
.J3 = 60

PM = h -� [·: OL = PM] .. (ii)


x
[· tan 60° = .J3]
.' =>

X=- 60 X - .J3 60-/3 [by rationalising]


= --
=> [1 ]
.J3 .J3 3 .

20-/3 = h-/3 h = 20 m
x
= 20-/3 m [1 1 SM
=>
x=
On putting 20-/3 in Eq. (i) , we get In right angled !:iSMP, tan60° = PM
=>
Hence, the height of the building is 20 m. [1]

h - I_2 = h../3 - 2-2


65. Do same Q. 15 ofNCERT folder.
as

[Ans. Distance between two cars is J OO3 .J3 m. The [·.- tan60° = .J3 and from Eqs. (i) an d (ii )

J3h-h = �2 -I_2
d istance of each car from the rower are so.fjm
and --so../3 m, respecti.vely. ] [1]
3
Some Applications of Tri gonometry 191 J
=> h( .J3 - I) = 1 On dividing Eq. (v) by Eq. (vi), we gee
1 -p = AB (cos J3 - cos a) = cos p - cos a
-----

q
--
h= AB (sin a -sin J3) sin a - sin J3
.fj - 1
p cos J3 -cos a
l .f3 + 1 => - = ----
- --
x --
[by rationalising] q sin Cl -sin J3
- J3 - I .fj + I
= J3
H e nce proved. [1 ]
1.732 + 1
+ I = --- [·: .J3 = 1.732 )
---

3 -1 2 68. For below 5 yr,


Let length AB of che slide be m. x
A
2·732 = 1.366 km
= 1.5 =sm. 30 0
2 So, - x
Hence, the height of mountain is 1 .366 km. [1 ) 1.5 1 1 .5 m
=> -=-
67. Lee AB and SQ be cwo positions of che ladder having x 2
angles of elevation a and p, respectively. => x =3 m [2]
Let OQ = and OA = y
x For elder children,
Given that, BQ = q, SA = p and AB = SQ = length of Let y m be the length XY of the slide.
ladder. � =sin60°
Also, L.BAO = a and L.QSO = p [1 ]
So, x
y
Now, in llBAO, .J3
3 =-
OA
cos a = -
=> -
y 2
AB 3m

=> cos a = _l_ => = 2 ./3 m


y
AB Value Participatory role of residents in
=> y = AB cosa. = OA ... (i) che society, need of proper care of y..__......__ ___ Z

(Wall)
children, preservation of environment. [2]
B
69. Let AB be the pole and C be the point 1 .3 m below the
top A of rhe pole. Also, let D be the point on the
ground, where ladder CD be placed.
So, LCDB = 60° and BC = (5 - 1 .3) m = 3.7 m [1 ]
Now, BC
CD = sin 60°
x

=>
3.7 .f3
CD 2
=> CV - - 7.4 m .,
·-

..J3 (

·�

So, length of the ladder= 7� m = 7.43..J3 m


nd sm. a = OB "';..._,
; ' f
a -

AB [1 ]
=> OB = BA sin a ... (ii) v3
Now, in llQSO , cos p = OS Also, BC
- = tan60°
SQ BD
A :
OS SQ
== cos p AB cos f3 [·: AB = SQ] . . (iii)
=
E_
.

BD = ..J3
=> . =>
[1/2] c � ; "')
BD = �
and sin � = OQ 3 m
·� _ ;'")
SQ v3 �
=> OQ = SQ sin � = AB sin f3 [·: AB = SQ] .. . (iv)
[ 1/2]
Thus, foor of the ladder must be placed � m,

SA = OS -AO
Now, . 37./3 m away from the foot of the pole. B I�
: ; · · 1 -:-..
p = ABcos P-ABcos Cl 1.e. --

3 !�
=> p = AB(cos f3-cos a ) . . (v) [1/2] .

Value Women can work in any field without gender I


. .....�
�.�
.,

and
=>
BQ = BO QO - bias. Also, it indicates 'dignity for labour'. [2]
'�
- _ - :w;."J
q = BAsin a -ABsinf3 70. Do same as Q. 38. [Ans. (1) l 200.J3 m ]
=> . q = AB (sin a. -sinf3) ... (vi} [1/2] (ii) Presence of m i nd, quick decision making. [1 ]
FO RMATIVE ASS ESS M E N T
• •

Act1v1ty
Topic Covered Angle of elevation. 3. What change wil l be observed in the a n g le of
elevation as we move away from the o bject?
O bjective
4. An object is placed below the o bserver 's horizontal.
To fi nd the a ngle of elevation of Sun at a
particu l a r time on a sunny day.
Then, what is the angle betwee n l i n e of sight and
Ski l l Developed Analysis, knowledge of trigonometric
observer's horizontal?
5. What change will be o bserved i n the a ngle of
ratios.
M aterial Req u i red A metre rod, measuring tape, table elevation as we a pproach to the foot of a n object?
for tangent of a ng les.
M ethod True/False
(t) On the particu l a r s u n ny day at the given time, put the 1 . Angle of elevation of the top of pole from a point on
metre rod on the level g round with one end on the the ground is 45°. Then, heig ht of the pole is equal to
g ro u n d a n d the other vertica l ly u pward. its distance of the foot from the o bserver.
(it) Measure the length of the shadow of the metre rod from 2. From the figure, heig ht h is 173.2 m .
the beg i n ni n g to the end.
A

I
Let the length of the shadow be 1.16 m = 116 cm (say).
(iit) The length of metre rod = 2 m = 200 cm (say).
Observation hm
(t) If 0 be the angle of elevation of the Sun at the given
moment, then we have the fol lowing figure on a sheet C
l
of paper by taking a suitable scale. --- 1 00 m --- B
B 3 . The length of the shadow of a tree 7 m high, when
the Sun's elevation is 45°, is 7 m .
4. A bridge across a river makes a n a n g le o f 45° with the
river bank. If the length of the bri d g e across the river
is 150 m, then width of river is 1 00 m .
5. The angle formed b y t h e line of s i g ht with horizontal,
when the point being viewed is a bove the horizontal
level, is called angle of elevation.

(it) From the right a n gl ed llBAC, we have Fill in the Blanks


tan 0 = 1 . In the given figure, the imag i n a ry l i ne th rough the


Perpendicu lar AB
=
Base AC object and eye of the observer is cal led ......... .
Object
200
=> tan 0 =
116
=> tan 0 = 1.724 (approx.)
tan 0 = .J3 (approx.) = tan 60° ---
i$i
i. e.
Horizontal
Eye
0 = 60 °

Oral Questions 2. In the given figure, a man o n the top of cliff observes
1. A n o bject i s placed a bove the observer's horizontal. a boat coming towards him. Then, 0 represents the
What do we cal l the angle between the line of sight angle of ..........

-- ------------ - - - - - -

a nd o bserver's horizontal ? 0
2 . A n g l e of elevation o f t h e top o f a building from a
p o i nt o n the g round is 30°. What will be the angle of
d e p ression of this point from top of the building?
'
!

Some Applications of Trigonometry J 1 93 :


3. In the g i ve n f i g u r e , if AB = 10 m and AC = 20 m, Match the Columns
then 0 = . . . . . . . . . .
Column I Col u m n I I
1 . A player sitting on the top of a tower of (a) 36.6 m
height 40 m observes the angle of
depression of a ball lying on the
ground is 60°. The distance between
the foot of the tower and ball is
40
4. A pole 6 m h i g h, casts a s hadow 2.J3 m long on the
2. A tower is 50 m high. Its shadow is x m m
.J3
(b)
shorter, when the Sun's altitude is 45°,
g round. Then, t h e S u n 's elevation is ... . . . . . . . then when it is 30°. Then, x equals to
5. In the g iven f i g u re, i f AB =

A
10./3 m, then CO = 3. The length of the string of a kite flying (c) 60°
[take, J3 = 1.732) at 1 00 m above the ground with the
elevation of 60° is

1
4. If the ratio of height of a tower and the 200
.J3
(d) m
length of its shadow on the ground is
J3 : , then the angle of elevation of

the Sun is

Answers
O ra l Q u esti o ns
1 . Angle of elevation 2. 30° 3. Decreas e 4. Angle of depression
5. Increase

True/Fa lse
1 . True 2. False 3. True 4. False 5. True

Fi l l in t h e B l a n ks
1 . line of sight 2. depression 3. 30° 4. 60° 5. 7 . 32 m

Match the Col u m n s


1 . (b) 2. (a) 3. (d) 4. (c)

S U M M A RY
• • The angle of depression of a n obj ect viewed, is the a n g l e
T he line of sight is the l i ne drawn from the eye of an
ob se rve r to the point in the object viewed by the observer. formed b y t h e line o f s i g ht w i t h the horizontal, when i t i s
• The angle of elevation of an object viewed, is the an gle below t h e horizontal level, i .e. t h e c ase w h e n we lower o u r
formed by the l i ne of sight with the horizontal, when it is head t o look at t h e object.
above the horizonta l level, i .e. the case when we raise our ·

B
head to look at the obj ect.
(Object)

�cs

°'
"'
�<::'
0
6'
Eye
Angle of
elevation
Horizontal line
A
CHAPTE R EXE RCIS E
� I Very Short A nswer Type Questions [ 1 Mark eac ]
1. A 6 ft t all man find s that the a ngle of eleva tion of a Estimate how f o r the boa t is fro m t h e base of the ligha
24 ft h i g h pillar and the angle of depres sion of its base house?
are complemen tary angles. Find the dis tance of the 9. A boy, flying a kite, h a s kt out 60 m of s t ring. If the
man from the pillar. angle of eleva t ion of t hL' k i t L' is 60°, t h e n ca lculate the
2. When t h e length of the s hadow of a pole is equal to .J3 height of the kite a bove the g ro u n d .
times the h eig h t of the pole, then find the angle of 1 0. Air balloon is con nected t o a m L· t c o ro lo g ical ground
elevat io n of s ource o f light. station by a cable o f lengt h 2 000 m a n d incl ined at 60
3 . Wha t will b e the angle o f eleva tion of the top of a to the ground. Find the h e i g h t o f t he b a l l oon from the
1 5 m h i g h tower at a p oin t 1 5 m away from the base ground, assuming n o slackness in i t .

A hill slopes upwa rd s a t a n a n gle o f 30° with the


of the tower?
11.
4. Find the angle of eleva tion of the Sun, if the length of horizontal . Wha t hei g h t d ocs a m a n ri se, when he
the shadow of a tower is l times the height of the walks 1 00 m up the h i l l ?
1 2. The tops o f two poles o f h eig h t s 1 6 m a nd IO m are
t ower. connected by a wire. I f t h e w i re: m a kes a n a ngle of 30
5. From the poin t P on the level ground, the angle of with the horizontal, then fi n d t h e l e n g t h of the wire.
eleva tion of the top of a tower is 30°. If the tower is CCE 2012, 1 1
I
00 m high, then how far is P from the foot of tower?
1 3. A bridge in the shape o f a s t ra i g h t pa t h, across a rivet
6. A k i t e is flyin g at a height of 75 m from the level makes an angle of 60° w i t h t h e w i d t h of the river. I f
ground, a ttached to a s t ring inclined at 60° to the the length of the bridge is l 00 m, t h e n fi nd the width
h orizon tal . Find the length of the string to the nearest of the river. CCE 20 1 2

A ladder is placed aga i n s t a wa l l o f a h o u se such that


metre.
14.
7 . The angle o f eleva t ion o f the top of a tower a t a its upper end is touching the top of the wall. The foot
dis tance of 1 5 0 m from its foot on the horizontal of the ladder is 8 m a wa y from t h e foo t o f the wa ll and
plane is found to be 3 0°. Find the height of the tower. the ladder is making a n a n g l e of 30° w i t h the level of
. the ground. Determine the heig h t o f t he wa ll.
8 . From t h e top of a lighthouse 40 m above the water, [ take, .J3 = 1 .732 ]
t h e a ngle o f d epres sion of a small boat is 45°.

� I Short Answer Type. I Questions [2 Marks each ]


1 5. From a p oin t on the ground 40 m away from the foot 1 9. The string of a kite is I 00 m long. I f the s t ring is in the
o f a tower, the angle of eleva tion of the top of the form of a stra ight line ( there is no slack in the string )
tower is 3 0° . The angle of elevation of the top of a and makes an angle of 0 w i t h t h e level ground such
water tank ( on the top of the tower ) is 45 °. Find tha t sin 0 = �. then find the heig h t of the kite.
( i ) heig h t of the tower ( ii ) depth of the tank. 15
1 6. If two towers o f h eigh t s h1 and hisubtend angles of 20. A statue 1 .46 m tall is s tanding on the top of a
45° and 3 0°, respectively at the mid -point of the line pedestal . From a point on the g round, the angle of
joining their feet, then fi n d the ra tio of h1 : hi . elevation of the top of the s t a t ue is 60° a nd from l he
1 7 . If two towers of heigh t s h1 andhi , subtend angles of
same point, the angle of eleva t ion .o f the top of t he
pedestal is 45°. Find the height of ped estal.
6 0 ° a n d 3 0 °, res pectively a t the mid - point of the line
s e gment j oinin g their feet, then find the ratio of [ take, .J3 = 1 .732 ]
h1 : h 2 • CCE 201 2 2 1. From the fi rs t floor of Qutab Minar which is at a
1 8 . The angles o f eleva tion of the top of a rock from the height of 25 m from the level g round, a man obsen es
t o p and foot o f a 60 m high tower are 45° and 60°, the top of a building at an angle of el eva tion of 30°
respectively. Fin d the height of the rock . and the angle of depression o f the ba s e o f the building
[ take, .J3 = 1 . 7 3 2 ] to be 60 °. Calculate the heig h t of t h e b u i lding.
Some Applications of Trigonometry 195

�I Short Answer Type I I Questions [ 3 M a rks e a c h ]


22. The s hadow of a tower, \vhcn the angle of elevat ion same poin t o n the grou n d a re 60° a n d 45°, respectively.
of the S u n is 4 5 °, is fo und to be 40 m longer than Find the vertical dista nce between the two a e ropl a n e s .
when i t i s 60°. r: i n d the height o f tower. CCE 20 1 2, l l , 08

2 3 . An observer s t a n d i n g 60 m a way from a building 2 8. The angle o f eleva tio n of an aeroplane from a poin t A o n
not ice s t h a t t h e a n g l e s o f eleva tion of the top and the ground is 60°. After a fli g h t o f 3 0 s , t h e a n g l e o f
bo t t o m of a fl a g s t a ff o n the bu ild ing are 60° and eleva tion change to 3 0°. I f t h e pla ne i s flying a t a
4 5 °, re s pec t ively. F i n d t h e height o f the flagsta ff. constant heigh t of 3 600 .J3 m, t he n fi n d t h e s peed i n
km/h o f the plane. [ ta ke, .J3 = I . 732 ]
2 4 . From a w i n dow 60 m high above the ground of a CCE 2008
house i n a s t reet, the a ngles of eleva tion and 2 9 . The angle of eleva tion of a jet figh ter from a poi n t A o n
depression o f t he t o p a n d t he foot of another house the ground i s 60°. After a fl igh t o f I O s , t he a ng l e o f
on oppos i t e s i d e of t he s t reet arc 60° and 45°, eleva tion change t o 30°. I f the j e t is flying a t a s peed o f
respect ively. S how t h a t the height of the opposite
house is 6 0 + ./3) m .
432 km/h, t hen find t h e cons t a n t heig h t a t which t h e
(I jet i s flying. [ ta ke, .J3 = 1 . 7 3 2 ]
2 5 . From a 6 0 m h igh b u i ld i ng, the angle o f depression 3 0 . The angle of eleva tion o f a jet fighter from a poi n t A o n
of t he top a n d bot tom of a la mppost arc 30° and 60°, the ground is 60°. A fter a flight o f 1 5 s, t h e a ng l e o f
respect ively. F i n d t he d i s ta nce between lamppost eleva tion cha nge t o 3 0°. I f t h e j e t i s flying a t a speed o f
and b u i l d i n g . Al so, fi nd the di fference of heights 720 km/h, then find the constant height a t which t h e
j e t is flying. [ ta ke, .J3 = l.73 2 ]
between b u i l d i ng a nd lamppos t .
CCE 20 1 2, 08
26. The angle o f eleva tio n o f a j e t plane from a point A
on the gro u n d is 60°. After a fl ight of 1 5 s, the angle 3 1 . The angles of eleva t ion o f t h e top o f a tower from t wo
of eleva t io n cha n ge s to 30°. If the jet plane is flying poin ts distant s and t from i t s foo t a re complemen t a ry.
at a con s t a nt height of I soo/3 m, then find the Prove that the heigh t o f the tower i s JSi.
speed of t h e jct plane. CCE 2008 3 2 . The angle of eleva tion o f a cloud from a poin t 60 m
2 7. An aerop la n e, when 3000 m h igh, passes vertically above a lake i s 3 0° a nd the a ngle o f depre s s ion o f the
above a nother aeroplane at an instant, when the reflection o f the cloud i n the l a ke is 60°. Find the height
angles o f eleva t ion of t he two aeroplanes from the of the cloud.

� I Long Answer Type Questions [ 4 M a rks ea c h ]

33. A spherica l bal loon of radius r subtends an angle 9 3 7. A bird is s it ting o n t h e top o f a t ree which i s 8 0 m h i g h .
at the eye of a n observes. I f the angle of eleva tion of The angle of eleva t ion of t h e b i rd from a poi n t o n t h e
its cen t re is <t>, t hen fi nd t he height of the centre of ground i s 45°. T h e b ird flies a w a y from the point o f
t he bal loon . observa tion horizontally a n d remains a t a con s t a n t
34. I f the a ngle of eleva tion of a cloud from a point h
height . After 2 s , t h e a n g le o f eleva t io n o f t h e b i rd fro m
the point o f observa t ion becomes 30°. Find t h e s peed o f
me tres above a l a ke is a and the angle of depression flying of t h e bird . CCE 2008

. h( tan j3 + tan a)
of i t s reflection in the lake is 13. Prove that the height
of the cloud is ------ 3 8. A boy whose eye level is 1 . 3 m from the g rou nd, spots a
t a n j3 - tan a CCE 2003 balloon moving with the wind in a horizo n t a l level a t
some heigh t from t he groun d. The a ng le o f e leva t ion o f
3 5 . Two pill ars o f equ a l height are on either s ide of a t h e balloon from t he eyes o f the boy at a ny i n s t a n t i s
road, .which is 1 00 m wide. The · angles of elevation 60°. After 2 s , t h e angle of eleva t io n reduces to 3 0° . I f
of the top of t h e pil lars are 60° and 30° at a point on t he speed of t h e wind a t t h a t moment i s 2 9 .Ji m /s,
the road between t he pillars.
then find the heig h t o f t h e ba l loon from g ro u n d .
Find the pos i t ion of the point between the pil lars. CCE 2009
Also, find t he height of each pillar.
3 9. [f[OTS! Two sta tions due South of a ka n i ng t owcnv h k· h
loans towa rd s the North, a re a t d i s t il nccs ,z a n d b fro m
36 . At a poi n t on level ground, the angle of eleva tion
of a vertical tower is fou nd to be such that its its foot . I f a a nd 13 a re the elcvil t io n s o f t he top o f t h e
tangent is 2-. On walking 1 92 m towards the tower, tower from t hese s t a t ions, t hen p rove t h a t i t s
12 inclinat ion e t o t h e horizon t a l i s g iven by
�. Find the bcot a - a co t �
cot e =
the ta ngen t o f t he angle of elevation is
b - ,7
.
4
heigh t of the tower.
196 j AllinOne MATH EMATICS C lass 1 0th Term If

40 . 1«>TSl T h e e l evat i o n o f a hill from a point P due East 42. 1HOTS The angle o f devil l i on o f l h e t o p o f the cliff A
o i t is 0 a n d at a place Q d ue South of P, the rom a fixed point t is 0. A fk r go i n g u p ii d i s t ance of
elevat i o n is qi. I f t h e d i s ta nce PQ be d, then find the k m towards the top of t he cl i ff a t a n a n gle � it is found
h e ig h t of t h e h i l l . tha t the a ngle of eleva t ion i s u. S h ow t h a t t h e height of
)" f ( "
4 1. lHOjSl From a n a eropl a n e vertically above a s t raight t h e c 1 f m metres ) 1 s
. k ( co s lf> - s i n lf> · co t u )
.
�nzo n t a l p l a n e , the a n g l e s o f depression of two n>t 0 - co t u

is 11[�]
If th e a ngle of e leva t io n o f a cloud from a
consecu t ive k il o metre s on the opposite sides of the
a e ropla n e a re fou n d to be a and Jl S how that the
43. lH OTSl
point h m above a l a ke i s a u n d t h e a n g l e o f d e p re ss i on
of its reflection in t he la kL· i s r3, t h e n p rove t h a t the
h e i g h t of t h e a eroplane is
tan a tan J3
tan a .
tan a + tan J
f height of the clo u d
1-r
m, w h e re r
ta n J3 =

�I Va l ue Based Questi ons (VBQs) [4 Ma rks eac ]

to the foot of 300 m high tower. An


( i ) Which s ta t ion s h o u l d s e n d i t s t e a m and how
44 . A h ighway l ea d s much will this team h a ve t o t ra ve l ?
o b s e rva tory i s set a t the top of the tower. It sees a car
movi n g toward s it wit h an angle of depression of ( ii ) Wha t accord ing to you, a re t h e va l u es d i s played by
3 0°. Afte r 1 5 s angle of depression becomes 60°. the teams at fi re s t a t io n s P a n d Q.

4 6 . A tree breaks due to s torm a nd t h e b ro ken p a r t bends,


( i ) F i n d t h e d i s ta n ce t ravelled by the car during
this time.
so that the top of the tree t o u c h e s t he g ro u nd making
( ii ) H o w t h i s o b s erva tory i s helpfu l t o regulate an angle 60° with it. The d i s t a nce b e t w een t he foot of

is 1 0 m. The teacher a s ked t he s t u d e n t s to find the


the t r a ffic on the h i g hway? CCE 2009 the tree to the poi n t where t he t o p t o u c h e s t he ground .

45. A fire in a b u il d in g B i s reported on telephone to two height of the tree. All t h e s t u d e n t s fa i l e d b u t Necraj
fire s t a t i o n s P a n d Q, 20 k m apart from each other on took initia tive and calcu l a t e d i t co r rec t l y u s i ng
a s trai g h t road . P observes that the fire is at an angle trigonometry.
( i ) Wha t height N cc raj q1 1 c u l a t e d ?
of 60° to t h e road and Q ob serves that it is at an angle
( ii ) Wha t quality o f N c e r aj i s d e p i c t ed h e re?
of 45° t o the roa d .

Answers
1. 6J3 m 2. 30° 3. 45° 4. 60° 5. 1 73.2 m
6. 86.6 m 7. 86.6 m 8. 45 m 9. 30./3 m 1 0. 1000J3 m
11. 50 m 1 2. 12 m 1 3. 50 m 1 4. 4.62 m 1 5. (i) 2 3 . 1 m (ii) 1 6. 9 m
1 6. .J3 : 1 1 7. 3 : 1 1 8. 1 4 1 .96 m 1 9. 53.33 m 20. 2 m
2 1 .33.33 m 22. 94.64 m 23. 43.92 m 25. 34.64 m; 20 m
26. 720 k m/h 27. 1268 m 28. 864 km/h 29. 1039.2 m 30. 2598 m
32. 1 20 m 33. r sin cf> cosec �
2
35. Position of the point from angle of elevation 60° is 25 m.
Height of each pillar is 4 3 . 3 m .
d sin 0 sin cp
3 6 . 1 80 m 37. 29.28 mis 38. 88 . 3 m 40 _
�sin 2 0 - sin 2 cp
44 . (i) 1 4.66 m m
(ii) Presence o f mind, ability t o take promote decision.
45. (i) Station P should send the team, its distance covered 14.66 km.
(ii) Presence of mind, ability to take promote decisions.
46 . (i) 3 8 . 32 m
(ii) Lea dership, logical and reasoning.
CHALLE N G ERS*
1. A tower stands at the centre of a circular park. A and B are two points on the boundary of the park, such that AB = a m s u btends
an angle of 60'' at the foot of the tower and the angle of elevation of the top of the tower from A or B is 30°. Find the height o f the
tower.

2. In the followi n g fig u re . from the top of a building AB , 60 m high, the angles of depression of the top and the bottom of a vertical
lamppost CO are observed to be 30° and 60°, respectively.
(1) Find the horizontal d i stance between BA and CD.
(ii) Find the height of the lamppost CD.
(iii) Find the rad i u s of the circle, if Y-axis and AB are the tangent to the circle.
y
50
45 A F
40
35
30
25 D
20
15
10
5 0
X' x
5 5 10 15 20 25 30 35 40 45 50
10
15 al \Jc
20
Y'

3 . Firstly, plot the poi nts A(2, 4) , B (6. 4) , C (6, 2) and D (2, 2) and join all adjacent points. A pole BE of height h is standing on point B .
Ang le o f elevation of t h e t o p o f a pole from point A i s 30°. Further, when a point A moves to position F in t h e direction o f B at a
distance 2 m , the angle of elevation is 45°
(1) Find the height of the pole and distance between B and F.
(ii) Also, find the total area formed by the figure.

4. The angle of elevation of the top of a tower from a point A due South of the tower is a and from B due East of the tower is fl I f
d
A B = d , show t h a t height o f t h e tower is ) .
cot a + cot 2 �
2

5 .' A bird flies in a circle on a horizontal plane. An observer stands at a point on the ground. Suppose 60° and 30° are the maxi m u m
and minimum angles of elevation o f the bird and they occur when the bird is at t h e point P a n d Q, respectively o n i t s pat h . Let 0
be the angle of elevation of the bird when it is at a point on the arc of the circle exactly mid way between P and Q. Find t h e
numerical val ue of tan 2 e. (assume that the observer i s not inside the vertical projection of t h e path of the bird )

* Th ese questions m ay b e or m ay n o t b e asked i n t h e examina tion, have been g iven j u s t for additio n a l p ractice.
Note Solu tions to these q u estions are available a t page number 388-390.
CHAPTE R 6

P RO BAB I LITY

<;;) Chapter Check_�st In Class IX, we initiated the study of probability, where we scud icd about meaning of
probability, experiment, trial, event, occurrence of an event and the experimem-.il
Meani n g of Probabil ity
approach of probability. All these concepts are basics of p robabil i ty.
In this chapter, we shall first recall the study of probability done i n earlier classes and
Experiment
0 Random Experiment
then continue it to study theoretical (classical) definition of pro bability and problems
" Determi n i stic Experi ment
based on it.
Trial
Event M ea n ing of Probability
·· Occurrence of an Event Probability is the study of the chances (or likelihood) of events happening. By means of
" Probabil ity of an Event probability the chance (or likelihood) of events is measured by a n um ber lying between
Probab i l ity-An Experimental 0 and 1 .
Approach
P robability-A Theoretical Experiment
Approach An operation which produces some well-defined outcomes, is calJed a n experiment.
·· Elementary Event e.g. Tossing a coin, throwing a die, etc.
.. Compound Event
There are two types of experiment
• Equally l i kely outcomes
Favourable outcomes (l) Random experiment
° Complement of an (iz) Deterministic experiment
EvenVNegation of an Event
Some Special Events
Random Experiment
0 I m possible Event If an experiment, when repeated under identical conditions, do not p ro duce the same
., Sure Event or Certain Event o utcome every time but the outcome produced in each trial is one of the several
possible outcomes, then it is known as a random or probabilistic experiment.
In other wo�ds, an experiment whose outcome has to be among a collection of possible
outcomes, which are completely known but whose exact outcome is unknown, is
called a random experiment. e.g. Tossing a fair coin is a random experiment because if
we toss a coin, then either a head or tail will come up. But if we toss a coin again and
again, then ea�h time outcome will not be the same, but it will s u re be either head or
tail.
Note An experiment whose outcome is known , is not a random experi m e n t . e .g. Th rowing a
scone upwa rd is not a random experimen t because when a s c o n e is t h rown u p ward . it is
sure chat the scone will fal l downward.
Probability

Determlnistic Exp e rlment Probability-An Ex p erim enta l Approac h


An experi ment which when repeated under identical The empirical (or experimental) probabi l i ty o f an eve n t E is
conditions produce the same rcsulc or outcome, is known given by
as detenninistic experi m en t. e . g. Tossing of an unfair coin Number o f times event E occurred
P(E) =
(having either rail or head on both sides) is a deterministic Number of times the experiment was perfo rmed
experiment, as ic gives same result while tossing the coin
any number of ti mes. e.g. If a coin is tossed I 000 times with the fo llowing
frequencies : Head : 455 times, Tail : 545 times and if E1 be
Note Uncert a i m y occurs in random experi ment. rherefore we the event of getting a head and E 2 be the eve n t o f getting a
deal wich ranclorn expcr i rnc n l o n l y .
tail,
Trial then P(E1 ) = Probability of occurrence o f head
A trial is an action which resu l ts in one or several outcomes. 455
= = 0.4 5 5
e.g. If a coin is tossed 20 rimes, then each toss is called a 1 000
trial. and P(E 2 ) = Probability of occurrence o f tail
Event =
545
= 0. 545
An event for an experi ment is the collection of som e 1 000
outcomes of the experi ment. We generally denote it by Proba bility-A The o retica l Appro ac h
capital letter E.
We have seen that the experimental p ro bability is b ased o n
e.g. (z) Getting an even n u mber in a single throw of a die is
the results o f actual experi ments a n d adequate recordi ngs o f
an eve n t. This even t would consist of three
outcomes, namely 2, 4 and 6. the happening of events.
But this approach o f probability has some l i m i tations, l i ke
(iz) Getting a head in the toss of a coin is an event. This
repetition of experiment is not possible in every s i tuation as
event would consist of only one outcome, namely
it may be very expensive or u n feasi ble.
H (head) .
e.g. (z) Repeating the experiment of launching a satel l i te i n
Occurrence of a n Event order to compute the empirical probability o f i ts
An event E associated to a random experi ment is said to failure during launching, will be very expensive.
be occur (or happen ) i n a trial, i f the outcome of trial is (iz) Repetition of the phenomenon o f a n earthquake to
one of the o u tcomes that favou rs E. compute the empi rical probability o f a multi-sroreyed
building getting destroyed in an earthquake, is
e.g. If a die is rolled and the outcome of a trial is 4, then we unfeasible.
say that each of the fol lowi ng events has' happened (or In many experiments, the repetition o f an experiment can b e
occurred) avoided by making certain assumptions. These assumptions
(z) getting an even number. help us in calculating the probability directly ( o r by
(iz) getting a n umber greater than 2. theoretical approach) .
(iiz) getting a n umber less than 5 . Let us first look at the following terms, which help us m
introducing the theoretical concept of p robability.
Probability o f a n Event
(or Proba bility of occu rrence of an Event) Elementary Event
By probability of an even t we mean, measure of chan ce of An event having only one outcome of the random
occurrence of event, i.e. it is a measure of the chance that experiment is called an elementary event.
the event will occur as a result of an experiment. e.g. I n tossing of a coin, the possible o u tcom es are head (H )
If E is an event associated with a random experiment, then and tail ( T ) .
probability of E, denoted by P(E) , represents the chance of Let, £ 1 be the event of getti ng head (H ) o n the upper face o f
occurrence of event E. the coin and E 2 b e the event o f getti ng tail (T) o n the upper
e.g. If E denotes the event of getting an even number in a face of the coin.
single· throw of a die, then P(E) represents the chance Then, £1 and E 2 are elementary even ts associated with the
of occurrence of event £, i.e. the chance of getting 2, random experiment of tossing o f a co i n .
4 or 6.
\ 2 00 Allinone MATH E MATICS C lass 1 0th Term I

Compoun d Event Difference between Experimenta l


A collection of two or more elementary events associated and Theoretical Approach
with an experiment is called a compound event. The basic difference between chcsc rwo approaches to
e.g. I n the random experiment of tossing of cwo coins probability is chat in che experi mental approach, �
simul taneously, if we define the event of getting exactly one probability of an event is based o n whac has been actually
head, then it is a collection of elementary events happened while in theoretical approach , we t ry co predict
(or outcomes) HT and TH. So, it is a compound event. what will happen w itho uc a c cu a l ly performing cht
experiment.
Equa lly Likely Outcomes
The o utcomes of a random experiment are said to be equally I mportant Results Related to Probability
likely, when each outcome is as likely to occur as the other, I· The experi mencal p roba b i l i r y o f a n evc n c a pp roaches to i t s
i.e. when we have no reason to believe that one is more likely cheorecical proba b i l i c y. if che n u m b e r o f t rials of �.
to occur than the other. experi menc is very l a rge.
2 . The proba b i l i c y o f h a p p e n i ng of .in eve n c always res
e.g. When a die is thrown, all the six outcomes, i . e. l , 2, 3, becween O co I (0 a n d I i n c l us i ve) i . c . O :$ PC. £) � I. Also.
4, 5 and 6 are equally likely to appear. So, the outcomes probabi l i ty can be ex p ressed a s a p e rc c n c a g e by rnulciplying
l, 2, 3, 4, 5 and 6 are equally likely outcomes. il by 1 00. Thus. proba b i l i r y ( i n p e rc e n t age) l i es becween O �
t o 1 00 % (0 and 1 00 i nc l u s i ve ) .
Favoura ble Outcomes 3 . Probabil i ty of an eve n t c a n n eve r be n ega 1 i ve.

The outcomes which ensure the occurrence of an event are 4 . The sum of the proba b i l i c i cs o r c o m p l e m e n ta ry evencs of a
experimenc is I .
i . e. I f E and E are corn p l e m e n c r y eve n c s .
called favourable outcom es to the even t.
an
P(£) + P( E ) = I P( E) -
e.g. The favourable outcomes to the event of getting even
Then , or = I P( E)
P(E) I - P( l:: )
number when a die is thrown are 2, 4 and 6.
or =

Complement of an Event where, P(E) represen t s t h e occ u r renc e o f a n evenc E


/Negation of an Event and P(E) represen t s t h e n o n -o c curre n ce o r a n event E.

Let E be an event associated with a random experiment. justification


If m outcomes are favoura b l e co an eve n c 1:: ouc of n possibl�
ouccomes. then the n u m be r o f outco m e s w h i c h ensure the
Then, we can define the �omplement of event E or negation
of event E, denoted by E , as an event which occurs if and non-occurrence o f eve n t £ i s n m. -

only if E does not occur. Thus, we have


-
1- 1 - P( E>
e.g. Let E be the event of getting an even number in a single n - m m
P(E) = -- = - = � P(E) + P(E) = I
t:E row of a die. Then, i ts com plement can be define as event n n

E of getting an odd n um ber, as E is consisting 2, 4 and 6. Note • The rat io P (£) : P (Q, i . e . m : n - m.
Therefore, E would consist 1 , 3 and 5 .
lr nown as odd in
favour of occu rrence o f eve n t £.
Note
E and E are c a l l e d com p l e m e n ta ry even ts. • The ratio P(EJ : P(E). i . e . n - m : m k nown as odd in again sc
of the occurrence of even t £.
T h e o retica l D efin itio n
(C la ss ica l d e fin itio n) of P ro�a bility Some Specia l Events

Let us assume a11 the outcom es of an experiment are equally I mpossible Event
likely and E is an event associated with the experiment, then An event which is impossible . to occ u r, 1s called an

the theoretical p robability (or classical probability) of the impossible event .


event E is given by e . g. In throwing a � ie, there are only six possible outcomes I ,
N umber of favourable outcomes
2, 3, 4, 5 and 6. Let we are interested in getting a number 7
P (E ) = on throwing a die. Since, no face of the die is marked with 7,
Total n umber of outcomes
So, 7 cannot come in any throw. Hence, getting 7 is an
e . g. When a die is thrown , there are six possible outcomes. impossible event.
So, total n umber of outcomes is 6. Let E be the event of Then, P (getting a number 7) = Q = 0
get ti ng 4, then favourable outcome to E is only one, i.e . 4. 6
So, pro bability of getting 4 , P (E) = _!. Thus, probability of impossible event is always zero.
6
j 201 j
' -

Probability

Sure Event or Certain Event


An event which is s u re co occur , is called a sure event or certain
event. e.g. Suppose we want to find the probability of getting Two unbiased coi ns are tossed simu ltaneously.
a number less than 7 in a single th row of a die having numbers Find the probability of getting
I co 6 on its six faces. (i) one tail. (ii) atmost one head.
We are sure chat, we shall always gee a number less than 7,
whenever we th row a die. S o , getting a number less than 7 is a Step 1
First, write the total number of outcomes according
to number of coins tossed.
sure event.
Here, two coins are tossed simultaneously.
Then , P (getti ng a n u mber less than 7) =�= I So, possible outcomes are
6 (H, T) , (T, H) , (H, H) and (T, T).
Thus , probability o f sure event is always I . :. Total number of outcomes = 4

Step 2
Write the given events and find the number of
Different Types of Problems favourable outcomes.
Let £1 = Event of getting one tail
Related to P ro b a bility of an Event
To solve these problems , we required the number of possible Then, favourable outcomes related to £1 are (H. T) and
outcomes and n u mber of o utcomes favourable to given event (T, H).
which depend on the obj ect like coin , die , etc. , given in :. Number of outcomes favourable to £1 = 2
Again, let E2 = Event of getting atmost one head,
problem . Let us discuss the p rocedure of finding probability i.e. getting one head or no head.
for che problems o f followi ng types : Then, favourable outcomes related to E2 are

(Type 1] Problem Based on Tossing a Coin (H, T) , (T. H) and (T. T).
:. Number of outcomes favourable to E2 = 3
A co i n has two sides, head (H) and tail (T) . If we toss one or Find the required probabilities by using definition of
Step 3
probability,
more coins , then thei r possible outcomes are as follows : Number of favourable outcomes related to E
(z) In case of one coin , possible outcomes are H and T. P(E) = .
Total number of outcomes
(ii) In case of two coi ns , possible outcomes are
( H , T ) , (T , H), (H, H) and (T, T) . (1) Probability of getting one tail, P(E1 ) = � = .!
4 2
(iii) In case of th ree coins , possible outcomes are (ii) Probability of getting atmost one head, P(E2 } = �
(H, H , H ) , ( H , H , T ) , (H, T, H), (T; H, H), 4
which are the required probabilities.
(H, T, T), (T , H, T), (T, T, H)"and (T, T, T) .
Now, if given p roblem is related to toss of one or more , i.e. 2 E x ample 1 . Thro coins are tossed sim ultaneously.
o r 3 coins , then to find the required probability, we use the
fo l l ow ing steps :
Find the probability of getting
(i) exactly two head. (ii) atleast one h ead.
(Step IJ First , write the total number of outcomes Sol. When two coins are tossed simultaneously, then possible
according to the n umber of trial. outcomes are
Write the given event say E and find the number of
(H, }{), (H, 7), ( T, JI) and ( T, 1).
� favourable outcomes related to E. .. Total number of outcom es = 4
(Step III) Find the required probability
(z) Let £. = Event of getting exactly two head.
by using che Then, the outcomes favourable to £. is ( H, H).
definition of probability, i.e. . . Number of outcomes favourable to £1 = 1
Number of favourable outcomes
P (E) = P(EI ) = 4
I
Total number of outcomes
(ir) Let £1 be the event of getting acleast one head, i.e. o ne
Note • head or two head.
(H, 7), ( T. 7), a n d
When we talk about a coi n . we assume that it to be ' fair',
i .e . sym m e trical so t h a t there is no reason for i c to come Then, favourable outcomes are
down m o re often on one side than the other. This property ( H, 7) .
of the coi n is sa id to be ·unbiased' . : . Number of outcomes favourable to E� = 3
• Ra ndom toss of a co i n le is a phrase which means that the
P(E, ) = �
-
coin is a l lowed to fall freely without any bias or
i n terference.
4
. 2 02 i AllinOne MATHEMATICS Class 1 0th Te rm I

(Type I I ) Pro blem Based on Throwlng a Die Ex a mple 2. S uppose we th ro w a die o n ce.
n n umber greater
A die has s ix faces marked as 1 , 2, 3, 4,
(i) What is the probability of getting
and 6. If we have 5 than 4?
(ii) What is th e probability of ge t t ing n n u m ber Jess than
more than o ne die, then all dice a�e considered as distinct, if
n o t o therwise stated. If we throw one (or two) die (or dice) , or equal to 4 ?
then their possible outcomes are as follows : Sol. Possible outcomes of t h e expcri mcnr a rc I , 2, 3 . 4 , and 6. 5
(z) In case of a die, possible outcomes are 1, 2, 3, 4, and 6. 5 :.Total number of outcomes = 6
(iz) In case of two dice, possible outcomes are (1) Let E be the event of gcrcing a n u m ber grcacer than 4.
Then, outcomes favourable ro F a rc 5 and 6, as and 6 5
( I, 1) , ( I, 2) , ( I, 3) , ( I, 4) , ( 1, 5), (1, 6), (2, 1) , (2, 2),
are greater than 4. Therefo re, rhc n u m ber of outcomes
(2, 3) , (2, 4) , (2, 5), (2, 6) , (3, 1), (3, 2) , (3, 3), (3, 4) , favourable to E = 2.
2
:.P(E) = P ( num ber greater than 4 )
I
(3, 5), (3, 6) , (4, 1) , (4, 2) , (4, 3) , (4, 4) , (4, 5), (4, 6)
6 3
(5, I) , (5 , 2) , (5, 3) , (5, 4) , (5, 5), (5, 6), (6 , 1), (6 , 2) , (ii) Let F be the event of gc r c i ng a n u mber less than or
·

(6, 3) , (6, 4) , (6, 5) and(6 , 6) equal to 4.


· Outco�es favourable co t h e evcn r F a rc I , and 4.
2, 3
..
Now, if given problem is related to throw of one or two dice,
So, rhe number of o u c co rn cs favou rable co F = 4
then to fi n d the required probability we use same seeps as in P ( F) = P (number less chan or equal c o 4)
Type I . 4 2
=- =-
123 � Black Board Example 6 3
Two d ice a re th rown si m u ltaneously. Fi nd the
. .
pro babi lity of getti ng a m u lti ple of 2 on one die TRY YOURSELF
and a m u lti ple of 3 on the other die.
Q. l Three coins are tossed simultaneously. Find the
Step 1
First , write the total number of outcomes according to probability of getting
(t) two tail.
n u m ber of dice thrown .
Here, two dice are thrown , so possible outcomes are (it) atleast one tail. [Ans. (1) 3/8 (ii) 7/8]
(1, 1 ) , (1, 2 ) , (1 , 3 ) , (1, 4) , (1 , 5 ) , (1, 6) Q. 2 Two coins are tossed si r:n ultaneously. Find the
(2, 1 ) , (2, 2 ) , (2, 3 ) , (2, 4) , (2, 5 ) , (2, 6) probability of getting
-
(3, 1 ) , (3, 2 ) , (3, 3 ) , (3, 4) , (3, 5 ) , (3, 6) (t) two head .
(it) three head.
(iit) no head. [Ans. ( 1) 1 /4 (ii) o (iii) 1 /°' )
(4, 1 ) , (4, 2 ) , ( 4, 3 ) , (4, 4) , (4, 5 ) , (4, 6)
( 5, 1 ) , ( 5, 2 ) , (5, 3 ) , (5, 4) , (5, 5 ) , (5, 6)
Q.3 Suppose we throw a die once. What is the probability of
. CCE 20 1 3
(6, 1 ) , (6, 2 ) , (6, 3 ) , (6, 4) , (6, 5 ) and (6, 6)
getting an even number?
: Total number of outcomes = 36 [Ans. 1 /2)
Q.4 When a die is thrown , then what is the probability Of
Step 2 [Ans. 1 /.3)
Write the given event and find the number of
favourable outcomes related to this event. getting an odd number less than 5?

Let E = Event of getting a multiple of 2 on one die and a


Q. 5 Two dice are thrown at the same time. Find the
probability of getting
(t) same number on both dice.
multiple of 3 on. the other die.
(it) different numbers on both dice. [Ans. (1) 1 /6 (ii) 516)
Here , multiples of 2 are 2, 4 and 6, and multiples of 3 are
3 and 6.
So, favourable outcomes for event E are (2 , 3) , (4, 3) ,
(6, 3) , (2, 6) , (4, 6) , (6, 6) , (3, 2) , (3, 4), (3, 6) , (6, 2) and (Type I l l J Problem Based on Playing Cards
(6, 4) . A deck of playing cards consists of 5 2 cards out of which
: . Number of favourable outcomes = 1 1
26 are black cards and ocher 26 are red cards, where red
Step 3 Calculate the required probability.
cards consist of 1 3 cards of heart ( ¥ ) , 1 3 cards of diamond
11
1 1 ty = P(E) = -
. d pro ba b·1·
Require ( • ) and black cards consist of 1 3 cards of spades ( • ) and
· - - - · - -- - - - - - - - ----
36
1 3 cards of club ( "' ) .
N o te
a n d a m ul t i ple o f 3
Acco rd i n g ro r h e q u e st i o n , a m u l t i p l e of 2 occurs on one die
Thus, 52 playing cards are divided into fo u r pares (called
(2 . 4 , 6) (3, 6) suits) of 1 3 cards each, namely heart, diamond, spades and
o c c u rs o n ocher. So. here we take cwo

(3 , 6)
c a se s . I n fi r s t c a s e . we w i l l ca ke on fi rst d i e and
o n s e c o n d a n d i n s e c o n d c a s e . we w i l l cake on first die club.
a n d (2 . 4. 6) o n o t h e r . Thus. we gee I I favourable outcomes.
Probability f 203 f
Also, 1 3 cards in each s u i r arc ace, ki ng, queen, jack, 1 0, 9, 8, Example 3. One card is drawn from a well-sh u ffled
7 , 6 , 5 , 4 , 3 and 2 . deck of 52 cards. Find the probability of
Tree diagram of playi ng cards
(i) a king of diam on d.
(ii) a black ace card.
Tot a l card
(iii) an ace or a jack.
(iv) neither a king n or a jack.
Sol. Tocal number of cards in one deck of cards is 52, out of which
one card can be drawn in 52 ways.
:. Number of possible outcomes = 52
(1) Let £1 be the event of getting a king of diamond.
Then, number of outcomes favourable to £1 = I
[·: only one king card is present in diamond cards]
Heart
D iamond :.Required probability = P( E1 ) = __!__
52
Face Cards King, Queen and Jack are call �d face cards. So, (ii) Let £2 be the event of getting a black ace card.
total face cards = I 2 .
Then, number of outcomes favourable to £2 = 2
[4 aces are present in a deck of cards, out of which 2 are
Now, if gi"ven problem is related to select one or more cards, red and 2 are black]
then co find the requ i red probability, we use same steps as in
:.Required probability = P( E2 ) = _3_ = _l_
Type I. 52 26
�123} B lack Board Example -----.
(iii) Lee £3 be the event of getting an ace or a jack.
Then, number of outcomes favourable to £3 = 8
[ ·: 4 aces and 4 jacks are present in a deck of cards]
The ki n g, q u e e n a n d j a c k of s p a d es a re re moved
from a d ec k of 52 p layi n g cards and th en :.Required probability = P(E3. ) = �
well-s h u ffle d . O n e ca rd is selected fro m the
52 = 213
rem a i n i n g ca rd s . Fi n d t h e p ro ba b i lity of getti ng (iv) Let £4 be the event of getting a card, which is neither a
(i) a s p a d e . (ii) a k i n g. king nor a jack.
Then, number of outcomes favourable to £4 = Total
�tep 1 Write the total number of outcomes. number of cards - Number of kings - Number of jacks
Here, king, queen and jack of spades are removed = 52 - 4 - 4 = 52 - 8 = 44
. .. 44 1 1
:.Required probability = P(E4 ) = - 52 = -
from a deck of 52 playing cards.
So, remaining cards in deck = 52 3 = 49 - 13

[Type IV) Problem Based on Selection of a n


: Total number of outcomes = 49
.

[·: out of 49 one card can be drawn in 49 ways]

Step 2
Write the given event and find the-number of Object from a Bag/Box
favourable outcomes to the given event.
In this type of problems, we have to fi nd the probability of
(1) We know that, there are. 1 3 cards of spade. After selecting an item/object of particular colour/type from a
removing king, queen and jack of spades, only bag/box containing items/objects (like 5 balls, 4 marbles,
1 o spade cards are left iri the deck.
Let f, = Event of getting a spade 8 oranges, 7 bulbs etc.) in more than one colour/type. For
:. Number of favourable outcomes to E1 = 1 0 finding the required p robability of such problems we use the
(it) We know that, there are 4 kings in a deck. After following steps :
removing a king of spade, we left with 3 kings.
Let f2 = Event of getting a king
(Step I) First, find the total n umber of outcomes which
is generally obtained by adding all i tems/objects
:. Number of favourable outcomes to f2 = 3
in the given bag/box.
Calculate the required probability by using the Find the n umber of favourable outcomes to
formu I a, Pro ba b·i·t
Step 3 Number of favourable outcomes
11 y= given event or selection.
Total number of outcomes
Now, calculate the p robability by using the

:�
Now, required probability, formula,
(1) P(E, ) = Number of favo urable o utcomes
i tty =
P ro b ab.1. ----

(ii) P(E2 ) = 2_ Total number o f o utcomes


49
2 04 !, l
Allin one MATHEMATI CS Class 10th Te rm I

1V23� Black Board Example ---�


Sol. Here, rocal number of bulbs = 600
Our of600 electric bulbs, one bulb can bl.· chosen in 600 way� ..

Th ree bags co ntai n i ng 10 orange, 10 green and :.Total number of outcomes = 600
10 red balls, respectively are m ixed together in Number of defective bulbs = 1 2
o n e la rge bag. If one of the balls is taken out at
ra n d o m without loo ki ng i nto the bag, then what :. Number of non-defective bulbs = 600 - 1 2 = 588
i s the p ro babi lity that it is Lee E be chc cvcnc of selecting n o n - d e fect ive bulb. ch��
(i) o ra n ge? (ii) not orange?
number of ouccomcs favourable /;· 5 8 8
co =

Step 1 Hence, P (geccing a non-<lefccrive bulb) = - � 8 � 4 9 =


Find the total number of outcomes. (100 50
Bag I Bag 1 1 Bag Ill
10
Example 5. Cards n umbered 1 , 2 , 3 , 4 , 5 , . . . , 1 7 afe
10 10
green balls red balls put in a box and mixed th oro u ghly. O n e person draws
a card from the box. Find th e probability that the
number on the card is
(i) an odd n umber. (ii) a prime n umber.
Large bag (iii) divisible by 3. (iv) divisible by 2 and 3 both.
(v) a m ultiple of 3 or 5.
Sol. There are 17 cards numbered I , 2, 3, 4. 5 , . , 17 which are in
. . a

Given, number of orange balls = 1 0 box.


number of green balls = 1 O Our of 1 7 cards, one card can be drawn in 1 7 ways.
and number of red balls = 1 O
Since, all balls are mixed in one large bag.
:. Tocal number of ouccomcs = 1 7
So, total number of balls in large bag (t) There are 9 odd numbered cards, namely
= 1 0 + 1 0 + 1 0 = 30 l , 3, 5, 7, 9, 1 1 , 1 3, 1 5 and 1 7.
:.Total number of outcomes = 30 :. Number of favourable ourcomcs = 9
Step 2 Find the favourable number of outcomes. Hence, P (geccing an odd number)
(1) Let E be the event of selecting an orange ball. = Number of favourable ouccomcs 9
Tocal number of ourcomes 1 7
Then, number of outcomes favourable to E = 1 0
[· : there are 1 0 orange balls] ( it) There are 7 prime numbered cards, namely 2, 3, 5, 7 , 1 1 ,
(ii) Let F be the event of selecting a ball which is 1 3 and 1 7.
not orange.
Since, there are 10 green and 10 red balls.
:. Number of c:ivourable ouccomes = 7
:. Number of outcomes favourable to F = 20 Hence, P (geccing a prime number)
Step 3 Now, calculate the probability. Number of favourable ouccomcs 7
(1) Required probability of getting an orange ball,
Tocal number of ouccomes 1 7
10 {iit) If a number is divisible by 3 , chen i c is a mulriple of3. The�
= _:!_
P(E) = are 5 numbered cards, which are mulriple of 3. na meh· ·
30 3
(ii) Required probability of getting a ball not
3, 6, 9, 1 2 and 1 5 .
orange, :. Number of favourable outcomes = 5
20 � Hence, P (getring a card bearing a number divisible by 3 )
P(F) =
30
=
3 Number of favourable ouccomes 5
Alternate Method Total number of ouccomes 1 7
P(F) = 1 P (getting an orange ball) (iv) If a number is divisible by 2 and 3 boch, chen ir is a mult ipl e
of 6. In cards bearing number 1 , 2, 3, 4, . . . , 1 7, there
-

=1 P(E) a rt­

[·.· E and F are complementary events] only 2 cards which bear a number divisible by 2 and 3 both,
-

= 1 -2=� i.e. by 6. These cards bearing numbers 6 and 1 2.


3 3 :. Number of favourable ouccomes = 2
Hence, P(geccing a card bearing number divisible by 2 and J
--- - -- - -- -- --- - - - - - - - ----- --- ----- --------'

Exam p le 4. It is given that a box of 600 electric boch)


b ulbs con tains 1 2 defective b ulbs. One bulb is taken Number of favourable outcomes = -2
o u t a t ran dom from this box. What is the probability Tocal number of outcomes 1 7
tha t i t is a n on -defective b ulb?
Probability f 2os I
(v) There arc 7 numbered cards, which arc multiple of 3 or S Q. 2 A card is drawn at random from a wel l-shuffled deck of
na mely, 3, 6, 9, 12, I 5, S and 10 . 52 cards, find the probability that the card d rawn is
:. Number of favourable outcomes = 7 CBSE 20 1 1
(t) non-ace card.
[
Hence, P (getting a card bearing a num ber m ultiple of3 or 5) (it) a king or a jack. 12 2
Number of favourable outcomes = -7
------

Toca) number of outco mes 17


Ans . (z)
13
- (it) -
13 J
Q.3
(Type V ] Problem Based on Geometry
A bag contains 2 red balls, 3 green balls, 5 black balls.
One ball is drawn from the bag. What is the probability of
getting
(t) a red ball .
In this type of problems, a geometrical figure is given to us
and we have to find the probability that, a given element lies (it) neither red nor black ball.
inside the small part of the geometrical figure. For this, we
first find the area/volume of both geometrical figures
(iii) either red or green ball.
[Ans. (1) .!. (it) 2_ (iii)
5 IO
.!.]
2
(small and big) separately and then find the required Q.4 Cards marked with numbers 5 to 75 are placed i n a box
probability by taking area/volume of small part as number of and mixed thoroughly. One card is drawn from the box,

[Ans.��]
favourable outcomes and area/volume of whole part as total find the probability that the number on the card is eve n .
number of outcomes.
Method to solve such type of problem, can be understand
easily with the help of following example. Q. 5 In the figure, a square dart board is shown.The length of a
side of the larger square is 1 .5 times the length of a side of
Example 6 . A missing helicopter is reported to have the smaller square. If a dart is thrown and lands on the
crashed somewhere in the rectangular region shown larger square. What is the probability that it will land in the
in figure. What is the probability that it crashed inside
interior of the smaller square?
D C

45ktl
the lake shown in the figure ?

a
-
6 km -

- 9 km
:=] �
---
km
A B

Sol. Here, the helicopter is equally likely to crash anywhere in the


region. The given geometrical figure is a rectangle and its small [Type VI ) Miscellaneous Problems
part, i.e. lake is also a rectangle. Sometimes the given problem is not of the type I to V whi ch
For rectangular region, Length = 9 km and Breadth = 4 .S km are discussed before. To solve these problems, first find the
:. Area of entire rectangular region, where the helicopter can possible outcomes for these problems very carefully and then
crash = 4 . 5 x 9 = 40.5 km 2 find the outcomes favourable to the given event for which
For rectangular lake, we want to find the probability.
Length = 9 - 6 = 3 km Now., use the definition of probability, i . e.
Breadth = 4 . 5 - 2 = 2.5 km
Probab111ty =
. . Number of favourable outcomes
Arca of the lake = 2.5 x 3 = 7.5 km 2 --------­

Total number of outcomes


Hence, probability that helicopter crashed inside che lake
7.5 = 75 =-5
= 40.5
-- -
to calculate the required probability.
Ex a mple 7. A Jot consists of 4 8 mobile phones of
405 27
which 42 are good, 3 have only minor defect an d
TRY YOURSELF 3 have m ajor defect. Varnika will b u y a phone, i f i t is
good but the trader will only buy a mobile, if it has n o
Q. l One card is drawn from a well-shuffled deck of 52 cards.
major defect. One phone i s selected a t ran dom from
Find the probability of getting the lot. What is the probability tha t it is
(t) a red face card. (i) acceptable to Varnika ?
(it) the queen of diamonds.
[Ans. (iz)
(1) i_
26
_!_]
52
(ii) acceptable to the trader? NCERT Exemplar
t 20 6 j AllinOne MATHEMATI CS Class 10th Term l

Sol. Given, total number of mobile phones is 48. So, number of Sol. Since, the music is stopped at any time within 2 min. So, di
possible outcomes 48 = possible outcomes are all real numbers (representing minmd
between 0 and 2 which arc represented on rhe number line.
Let E be the evenc char rhe music is stopped within the fusl
half-minute. Then, outcomes favourable ro E arc points on the
number line from 0 co .!.__2
0 t 1t 2
(i) Let £, be the evenc chat phone is acceptable co Varnika, Here, all the outcomes arc equally likely.
i .e. £, be the event of getting a good phone. :. Total number of outcomes = Total distance from 0 to 2 = 2
:. Number of favourable outcomes co £, = 42 Number of outcomes favourable co event E
Hence, probability chat phone is acceptable co Varnika, 1 1
P(E ) = 42
I 48 = ?_8
= D 1srancc
. firom 0 co - = -
2 2
Lee £2 be the evenc chat phone is acceptable co the I / 2 .!._
(ii)
trader, i.e. £2 be the evenc of getting a good phone or a Now, required probability = P(E) = =
2 4
phone with minor defect.
:. Number of favourable outcomes co £2 Example 10. '1\vo players Neh a and Shivani play a
= Number of good phones tennis match. It is known that the probability of Neha
+ Number of phone with minor defecc winning the match is 0 . 62 . Wha t is th e probability of
= 42 + 3 = 45 Shivani winning the match ?
Hence, probability chat phone is acceptable co the Sol. Lee E and F denote the events chat Ncha and Shivani win the
trader, match, respectively.
P(E ) = 45
2
48 = !2 le is clear chat, if Ncha wins the match, then Shivani losses the
16 macch and if Shivani wins che match, then Ncha losses cht
Example 8. Savita and Hamida are friends. What is
macch. Thus, E and Fare complemcncary events.
the probability tha t both will have
.. P(E) + P(F) = 1
(i) the same birth day? (ignoring a leap year) Since, probability of Neha's winning che match,
i .e. =
P(E) 0.62
(ii) differen t birth days ?
Sol. There are 365 days in a year, so Savica's birthday can be at :. Probability of Shivani's winning the match,
any day of 365 days in che year. P(F) = P (Neha losses che match)
Similarly, Hamida's birthday can be at any day of365 days in = 1 -P(E) [·: P(E) + P(F} = I)
the year. = 1 -0.62 = 0.38
So, total number of outcomes = 365 x 365
(1) If both have same birthday, chen the number of TRY YOURSELF
favourable outcomes for her birthday = 365
:. P (Savica and Hamida have same birthday) Q. l
[Ans. �
In a lottery, there are 1 O prizes and 25 blanks. What is the
= 365365 = I
- probability of getting a prize?
x 365 365
{it) P (Savi ca and Hamida have different birthdays)
Q. 2 Find the probability that a number selected at random from
= 1 - P (Savi ca and Hamida have same birthday)
[·.· P(E) + P(E) = I]
the numbers 1 , 2, 3, . . . , 35 is
(1) a prime number.
= 1 - __I (ii) a multiple of 7.
1 1 . 1 .. 16
[
365 (iii) a multiple of 3 or 5.
Ans. (t) - (11) - (m) -
364 35 7 35
365 Q .3 If the probability of an event is 0.65, then find the probability
Exam p le 9. In a m usical chair game, the person CCE 2013
[Ans. 035
of not happening of that event.
playing the m usic has been advised to stop playing
the m usic a t any time within 2 min after she starts Q.4 In a cricket match, a batsman h its the boundary 8. times out

[ �
pla ying. Wha t is the probability that the music will of 40 balls he plays. Find the probability that he did not hit
Ans.
s top within the first h alf-min u te after starting? the boundary.
·.
NCE RT FO L D E R 6.1
1 Complete the following statements Sol. When we toss a coin, we get either head or tail which are
(i) Probability of an event E + Probability of the equally likely outcomes.
event ' no t E' = ... Hence, the result of the toss of a coin is completely
(ii) The probability of an event that cannot happen unprediccable or unbiased. So, cossing a coin is a fair way of
is . . . . Such an even t is called . . . . deciding.
(iii) The probability o f an event that is certain to 4 Which of the following cannot be the
happen is . . . . S u ch an event is called . . . . probability of an event?
(iv) The sum o f the probabilities of all the
elementary events o f an experiment is . . . . (i) � - 1 .5
(ii)
3
(v) The probability o f an event is greater than or (iii) 15% (iv) 0.7
equal to . . . and less than or equal to . . . .
Sol. {it) The probability of an event cannot be negative in any
Sol. (t) 1 , because the sum of probabilities of complementary case.
events is equal ro one.
{it) 0, impossible event. (e.g. Suppose a die is thrown and 5 If P(E) = 0.05, what is the probability of ' not E'?
we have ro determine che probabilicy of geccing 8. Sol. Given, P(E) = 0.05
This cypc of evenc is called an impossible event and
probability of this event is 0.) We know chat, P(E) + P(E) = I
(iit) l , sure event. (e.g. The event of getting a number less :. P(E) = I - P(E) => P(E) = I - 0 . 05 = 0.95
than 7 in a single throw of a die is a sure event and its
probability is 1 .) 6 A bag contains lemon flavoured candies only.
(iv) I , (e.g. Suppose a die is thrown, then Malini takes out one candy without looking
P(gectingl ) + P(geccing2) + P(geccing 3) into the bag. What is the probability that she
+ P (geccing 4) + P (getcing 5) + P (geccing 6) = 1) takes out
( v) 0, l, since, che probabilicy of an event always lies from (i) an orange flavoured candy?
0 co I . (ii) a lemon flavoured candy?
2 Which of the following experiments have Sol. (t) Lee E be an event of getting an orange flavoured
equally likely outcomes? Explain. candy. All possible outcomes are against the event E
(i) A driver attempts to start a car. The car starts or because in a bag, all candies are lemon flavoured.
does not start. So, P(E) = 0
(ii) A player attempts to shoot a basketball. She/He (ii) Lee F be an event o fgening lemon flavoured candy.
a

shoots or misses the shot. All possible outcomes are favourable co event F
(iii) A tria l is made to answer a true-false question.
because all che candies in che bag are lemon flavoured.
The answer is right or wrong. So, P(F) = I
(iv) A baby is born . It is a boy or a girl. 7 It is given that in a group of 3 students, the
Sol. (t) The car scares normally but when there is some defect, probability of 2 students not having the same
then car does not scare. So, the outcomes are not birthday is 0.992. What is the probability that
equally likely. the 2 students have the same birthd ay?
{it) The outcomes in chis situation are not equally likely
because the outcomes depends on many factors such Sol. Lee E be the event chat 2 students not having the same
training of the player, quality of basketball, etc.
as birthday. Then, E be the event that 2 students having the
(iit) The outcomes in trial of true-false question is either same birthday.
true or false. Hence, the two outcomes are equally Given, P(E) = 0.992 _n
likely. We know that, P(E) + P(E) = 1
(iv) A new baby can be either a boy or a girl, so both the 0
outcomes are equally likely. 0 . 992 + P(E) = 1 LL.
3 Why is tossing a coin considered to be a fair P(E) = 1 - 0.992 = 0.008 1-
way of deciding which team should get the ball So, the probabilicy of 2 students having the same birthday , tX
LI.I
at the beginning of a football game? is 0.008.
u
z
2osl All{nOne MATHEMATICS C lass 1 0th Term l

8 A b ag contains 3 red balls and 5 black balls. A :. Total number of coins = 1 00 + 50 + 20 + 10=180
b all is d rawn at random from the bag. What is
the probability that the ball drawn is
(i) red?
(ii)not red?
Sol. +
Total number of balls = 3 5 = 8
:. Number of all possible outcomes = 8
E
Let be the event of drawing a red ball.
Here, number of red balls = 3 (':'\ p
'' (SO .
pa1se com. ) = Number of 50 paise coins
:. Number of outcomes favourable to £ = 3 1
Toral
00 5
number of coins
- -
= 1 80 = 9
(iz) Number of coins which arc nor of' 5 == 180 170
-10
:. P (that the coin wi l l nor be a ' 5 coin ) = =!I
l ?O
180 18
Hence, 11 Gopi buys a fish from a
(t} Probability ofdrawing a red ball, P(E) = � shop for her aquarium.
8 The shopkeeper takes
(it) Probability of not drawing a red ball out one fish at random
-P
= 1 (drawing a red ball) = 1 - P ( E) from a tank containing
=1 - �8 = �8 5 male
8 female
fishes and
fishes (see
9 A b ox contains 5 red marbles,
figure) . What is the
probability that the fish taken out is a male fisht
8 white marbles
and 4 green marbles . One marble is taken out
of the b ox at random. What is the probability Sol. Total number of fishes in the rank
that the marble taken o ut will be +
= 5 male fishes 8 female fishes = 1 3 fishes
(i) red ? :. Probability of takenof male
our afishes
male fish
(ii) whi e ?
Number 5

[Ans. :7 J
t Total number of fishes 1 3
(iii) not green?

[Ans. I�J
1 2 A game of chance consists of
Sol. (t} Do same Q. 8as
spinning an arrow which comes
to rest pointing at one of the
(it) Do same Q. 8as
numbers 1, 2 , 3 , 4, 5 , 6 , 7 a n d 8
(see figure) and these a re
(iit} Let £3 =Event of drawing a marble which is not green equally likely outcome s . What is
= Event of drawing either red or white marble) the probability that it will p oint at
:.Number of favourable outcomes to £3 = 5 + 8 = 13 (i) 8?
So, probability of taken out not a green marble, (ii) an odd
number?
P(E)= � (iii) a �umber greater than 2 ?
17 (iv) a number less than 9 ?
1 0 A piggy bank contains hundred 5 0 paise coins, Sol. Total number of points on the circle = 8
fifty f f
1 coin s , twenty 2 coins and ten 5 coins.f ( t) £1
Let = Event of getting arrow at number 8
If it is e qually likely that one of the coins will
fall out when the bank is turned upside down,
: Number of outcomes favourable to £1 = I
.
Probability that arrow comes at number 8,1
E =
then what is the probability that the coin
will 50 paise coin? s P( 1 )
-
(i) be a
(ii) will not be a ' 5 coin? Let £ = Event of getting arrow at odd number
Here,2 odd numbers are 1, 3, 5 and 7.an = 4
(ii)
Sol. Given, number of 50 paise coins = I 00 20
Number off coins = 50, Number off 2 coins
I = :.Number of outcomes favourable to E2
Number off 5 coins = 10
Probability J 209
Probabiliry that arrow comes at an odd number, Sol. Tora) number of cards in one deck of cards is 52.
P(E, ) = _i8 = _!_2

:.Total number of outcomes = 52
(t) Lee £1 = Event of geccing a king of red colour
(iii) Let £_, = Event of geccing arrow at a number greater
:.Number of outcomes favourable co£.of=playing
2
:.than 2, i. e . at 3, 4, 5, 6, 7 or 8
Number
Probabiliry ofthatoutcomes
arrow favourablac ea ronumber
comes £3 =6 greater [·: there
out are
of four
which kings
two i
aren a deck
red and two are cards
black]
than 2, Hence, probability of2gettin1 g a ki ng of red colour,
P( ) = =
P(E,). = � = � £. 52 26
8 4
(iv) Lee £4 = Event of geccing arrow ac a number less
{it) Lee £2 = Event of geccing a face card
:.Number of outcomes favourabl e co £there
2 = 1 2are 1 2 face
:.than
N 9, i.e. ofat outcomes
umber 1, 2, 3, 4, 5, 6, 7 or 8

Probability chat arrow comes favourabl e co £4 =less8 than 9,


ac a number
[ · :
cards,ofnamelin a deck of cards,
y 4 akings, 4 jacks, 4 queens]

P(�I ) = = 1
Hence, probability
P(E2) = 1 2 = 3
getting face card,
52 1 3
1 3 A die is thrown once. Find the probability of (iiz) Lee £3 = Event of getting a red face card
getting
(i) a prime number.
:.Number of[·:outcomes
inouta deckof whioffavourabl
cards, e to £3are=162 face cards
there
Hence, probability of6gettingch3 6a areredredfaceandcard,6 are black]
(ii) a number lying between 2 and 6 .
(iii) an odd number.
Sol. OnTotal
a die,number
there areof sixpossibl
numbers 1 , 2, 3, 4, 5 and 6. P ( E3) = =
:.(z) Lee £1 Event of gettie noutcomes
g a prime =number
6 52 26
(iv) Lee £4 = Event of getting a jack of heart
=
Then,
3 and 5.
£1 would consist of three outcomes namely 2, :.· Number of outcomes favourable co £4 = 1
:.Probability
Number ofofoutcomes favourablnumber,
e co£, = 3 [ : there are four 1jackof club, cards1 inofaspade
deck, andnamel1 ofy diamond]
I of heart,
geccing a prime Hence, probability of geccing1 a jack of heart,
P(E ) = 3 = 1 P(E4 ) =-
, 6 2 52
(iz) Lee £2 = Event
2 and 6
of geccing a number lying between (v) Lee £5 = Event of getting a spade
Then, £2 would consist of chree outcomes, namely :. Number of outcomes
[·: in a deck of cards,favourabl
there are e1co3 spades,
£5 = 1 3
1 3 clubs,
3, 4 and 5. 13 hearts and 1 3 diamonds]
Number ofof outcomes favourabl e co £2 = 3 Hence, probability of getting a spade,
Probability
2 and 6,
geccing a number lying between P(E ) = _!i = _!_
5 52 4

=
P(E., ) = � _!_ (vz) Let £6 = Event of geccing a queen of diamond
6 2 :. Number of outcomes favourable to £6 = 1
(iiz) Lee £3 Event of getting an odd number
£3= would consist of three outcomes namely 1 ,
Then,
andNumber Hence,[·:probabi
in 1 3 diliatymondofgetticards,
ng a there
queenisofonldiyaonemond,queen] 1 :Y
"'
:.3Probability
5.
ofofgeccing
outcomes
an oddfavourabl
number,e co £3 = 3 P(E") =-I
52 LW
t' �:

P(E3 ) = 3 = 1 1 5 Five cards-the ten, j ack, queen, kin g a nd ace of �


6 2 diamonds are well-shuffled with their fac e c
1 4 One card is drawn from a well-shuffled deck of downwards. One card i s then picked up at
52 cards. Find the probability of getting random.
(i) _a king of red colour. (ii) a face card. {i) What is the probability that the card is the
(iii) a red face card . (iv) the jack of heart. queen?
(v) a spade. (ii) If the queen is drawn and put a side, what is the
(vi) the queen of diamond. CCE 201 5
probability that the second card picked up is
(a) an ace? (b) a queen? CCE 20 1 4
\ 21 0 \ Allt�nOne MATH EMATICS Class 10th Tenal

Sol. (i) Total number of cards = 5 Lee £1 = Event of sclcccing a good bulb, then
:. Number of all possible outcomes = 5 Number of ouccomcs favourahlc co £1 = 1 5
P (picking a queen card) = .!... :.Required probability = P( E1 ) = !_2
5 19
(ii) Suppose a queen is draw.n and put a side. Then, four
1 8 A box contains 9 0 discs , which are numb
cards are left. from 1 to 90. If one d i s c i s d rawn at randoa
Now, number of all possible outcomes = 4 from the box, find the prob a b i l i ty that it bears
(a) P (the second card picked up is an ace) = .!_
4 (i} a two-digit number.
(b) P (the second card picked up is a queen) (ii} a perfect square n u mber.

= �4 = 0 [·: queen is drawn before]


m
(iii} a nu ber divisible by 5 .
Sol. (1) Total number of discs in a box = 90
16 1 2 defective pens are accidentally mixed with :.Number of all possible outcomes = 90
1 3 2 good ones . It is not possible to just look at a Lee £. = Event of gcccing a disc bearing a

pen and tell whether or not it is defective. One cwo-digic number


pen is taken out at random from this lot. Here, two-digit numbers I 0, 1 1, ... , 90
a rc

D etermine the probability that the pen taken out :.Number ofouccomcs favourable co Ei = 8 1
is a good one. Hence, probabiliry of gcccing a disc bearing a

cwo-digic number,
Sol. Total number of pens = 1 2 defective + 132 good = 144 pens 81 = 9
Let E = Event of selecting a good pen. P(E. ) = 90 IO
Then, number of outcomes favourable co E = 132 (ii) Lee £2 = Event of gcccing a disc bearing a perfect
square number
Here, perfect square number arc 1 , 4, 9, 16, 25, 36.
49, 64 and 8 1 .
:.Number of ouccomes favourable to £2 = 9
:.Probability of selecting a good pen, Hence, probabiliry of gcccing disc bearing a per&ct
a

P(E) = 1 32 = !..!_ square number, P(EJ = - 9 1


1 44 1 2 90 = -
10
(iit) Let £3 = Event of getting a disc bearing a number
17 (i) A lot o f 2 0 b ulbs contain 4 defective ones. One
divisible by 5
bulb is d rawn at random from the lot. What is the
probability that this bulb is defective?
Here, the numbers divisible by 5 arc
5, 10, 1 5, 20, 25, 30, 35, 40, 45, 50, 55, 60, 65, 7�
(ii) Suppose the b ulb drawn in (i) is not defective 75, 80, 85 and 90
and is not replaced. Now, one bulb is drawn at
random from the rest. What is the probability
:. Number of outcomes favourable to £3 = 18
90 !5
Hence, required probabiliry = P(E3 ) = .!_!
that this bulb is not defective? =

Sol. (i) Total number of bulbs is 20.


. . Number of all possible outcomes =20 1 9 A child has a die whose six faces show the
letters as given below
0 []] @] [Q] [fil 0
The die is thrown once. What is the probability
of getting
Let E = Event of getting a defective bulb, then (i) A?
Number of outcomes favourable to E = 4 (ii) D?
:. Probability of selecting a defective bulb, Sol. Total number of outcomes in a single throw of six fact$a

P(E) = _i_20 = .!._5


die =6
( 1) Let Ea = Event of getting a letter A
(ii) Suppose one good bulb is drawn and put outside. Then, number of outcomes favourable co event
Then, 1 5 good bulbs and 4 defective bulbs are remain £1 = 2
in a lot. Hence, probability of getting a letter A,
Now, total number of bulbs = 19 P(E1 ) = -2 = -1
:.Number of all possible outcomes = 19 6 3
Probab i lity f 211
(it) Let , number
£2
Then = Event ofofoutcomes
getting a lfavourabl
etter e co event = I
D
£2
(ii) A student argues that there are 11 p ossible
outcomes 2, 3, 4, 5, 6, 7, 8, 9, 10, 1 1 and 1 2 .
Hence, probability of getting a letter D, Therefore, each of them has a p robability _..!:._ , D o

3m
- = ]_6
P( E, )
you agree with this argument? Justify your
11

@]2m
answer.
20 Suppose , you d rop a die at Sol. (1) Total possible outcomes on throwing two dice are
random on the rectangular
(I, 1 ), (I, 2), (I, 3 ), (1, 4) , (1, 5 ), (I, 6)
Area of rectangle = 3 2 = 6 m 2
region shown in figure . What
is the probability that it will (2,1 ), (2, 2), (2,3 ), (2, 4 ), (2, 5 ), (2, 6)

and area of circle of radius �2 m = (�)2 2= �4 m


land inside the circle of diameter 1 m? (3, 1), (3, 2), (3, 3 ) , (3, 4 ) , (3, 5 ) , (3, 6)
Sol. x ( 4, 1 ), (4, 2), ( 4,3), ( 4, 4), ( 4, 5), ( 4, 6)
(5, 1) , (5, 2 ), (5, 3 ) , (5, 4) , (5, 5 ), (5, 6)
[·: diameter = Im => radius = � m ]
2
7t (6, 1 ), (6, 2 ), (6, 3 ),(6, 4),(6,5) and (6, 6)
(a) Then:.LetNumber=Sum
£1
of allonpossitwobldie coutcomes
e i s 3. = 36
Now, probability that4 the die land inside the circle , 2),and£,. (2,woul
(1:. Number I). d consist of two outcomes, namely
= -- 7t /
6 = -24
7t of outcomes 2 favourable to 2E1 =

21 A lot consists of 1 44 ball pens of which 20 are


Hence, P (E ) =-36 I

defective and the others are good. Nuri will buy (b) ThenLet = Sum
£2 on two disct eofis three
4. outcomes namely
a pen if it is goo d , but will not buy if it is ,
£2 would consi ,
defective. The shopkeeper draws one pen at (1:.N, 3),umber(2, 2of) and (3
outcomes , I). favourable to = 3
£2
random and gives it to her. What is the
probability that Hence, . = -36P ( £2 )
3
(i) she will buy it? (ii) she will not buy it?
Total number of ball pens =144 Let = Sum
(c) Then £3
would
on two dice is 5. outcomes namely
, , 2)consi
andsfavourabl
(4t of, I)four
. e to ,
Sol.
£3
:.Number of al l possible outcomes=
Let = Event of getting a good ball pen 144 , (2, 3)of, (3outcomes
(1:.,N4)umber
(1) E
4 £3 =4
:. Then
= Total, number
number ofofoutcomes
bal l pensfavourabl
-Defecti evetoball pensE Hence, = -36 P ( £3 )

= 144 - 20 = 124
She will buy a pen , if it is a good pen. (d) Let = Sum on two dice is 6.
£4

:. Required probabiliry=P(.E) = 144 124 = �


36 (1,:.Then
5), ,(2, 4)woul
£4

Number of outcomes, (3 , d3)consi


, ( 4, s2)t ofandfive(5, I).outcomes, namely
favourable to = 5
£4
5
{it)
:.SheR=Iequired
will notprobability
buy a pen, if it is not a good pen.
- Probability of getting a good pen
Hence, P = -36 (E4 )

[·: (E) =I]


(e) Let = Sum on two dice is 7.
£5

31 5 P (E) + P
(I,:.Then ,,(2, 5),ofwould
£5
6)Number , 4 ),consi
(3outcomes s),t (5,of six2 ) andoutcomes
( 4, 3favourabl (6 , I). , namely
=1 -P(E)=I --36 = -36 £5 e to = 6 a:
u.
22 Two dice, one blue and one grey, are thrown at Hence, P(E.;) =-36 6 c
the same time. Then
(i) Complete the following table: (f) Let = Sum on two dice is 9.
£6 c
namely w..
Event : Sum (3Then, 6) , (4, 5),would
,:. Number £6
(5, 4)consi andst(6,of3).four outcomes,
rn
2 3 4 5 6 7 8 9 10 11 12
on 2 dice
1
of outcomes
4 favourable =
to E6 4
Probability -
1
36
-
5
36
-

36 Hence, = -36 P(E6 )


Alllnone MATH E MATICS C lass 10th Tem

(g) Let E7 = Sum on two dice is 10. (ii) Probability char 5 will nor come up eicher rime,
Then, E7 would consist of three outcomes, namely 5
( 5, 5 ) and
(4, 6), (6, 4). P( E ) = I - P( E ) = I _ !_!_
36
=2
36
:. Number of outcomes favourable to E7 =3
3
Hence, P ( E7 ) = - 25 Which of the following a rg u ments are coned
36 and which are not correct? Give reason for yo�
(h) Let Ea = Sum on two dice is 11. answer.
(i) I f two coins a re tossed s i m ultaneously, then
Then, Ea would consist of two outcomes, namely
5 ) and ( 5,
(6, 6). are three possible outcomes-two heads, �··
:. Number of outcomes favourable to 2 Ea = tails or one of eac h . There fore, for each 01
2 !
Hence, PCEa ) = - these outcomes, the probability is .
36 3
(ii) If a die is thrown , there are two possib!c
(ii) No, we do not agree with the given argument because outcomes-an odd number o r an eve n numoo
the events of eleven sums are not equally likely. Therefore, the probability of getting an odd
2 3 A gam e consists of tossing a one rupee coin num b er is 1
. -.
3 times and noting its outcome each time. 2
H anif wins, if all the tosses give the same Sol. (1) Incorrect. We can classify che o utcomes like this �
chey are nor chen 'equally likely' . Reason is char •ant
of each' can resulc in two ways-from a head on first
result, i . e . three heads or three tails and losses
coin and rail on che second coin or from a rail on th·
otherwise . Calculate the probability that Hanif
will lose the game . firsc coin and head on che second coi n. This makes ii
Sol. The total possible outcomes on tossing a coin three times twicely as likely as cwo heads (or cwo rails).
( HHH ) , ( HHT) , (HTH), (THH) , (HIT), (it) Correct. Since, che cwo o uccomcs considered in th
(THT) , (ITH) and (TIT). question, are equally likely, chcrcforc we can cl�·
: . Number of all possible outcomes = 8 che outcomes as given in the qucscion.
Let E be the event that Hanif will loss the game. So, we have, P (getting an odd number)
Han ifwill lose the game, if all tosses do not have same result. =P (getcing an even number) . . . (i)
i.e. i f outcomes are [·: ouccomes are equally lik�� '
( HHT) , ( HTH ) , (THH), (HTT), (THT) or (ITH). Also, we know chat sum of probabilicies of '
:. Number of outcomes favourable to E =6 elementary events of an experiment is 1.
Since, we have, P (getting a n odd number)
Hence, required p robability
6
= P(E) = - + P (geccing an even number) = l
8 => 2P (getting an odd number) = [using Eq. (i)
1
3
4 => P (gecting an odd numbe r) = .!._
2
2 4 A die is thrown twice. What is the probability
that Additional Question
(i) 5 will come up atleast once? 26 A girl calculates that the probability of her
(ii) 5 will not come up either time? winning the first prize in a lottery is 0.08. If
!
Throwing a die twice and throwing two dice
6000 tickets are sold , then how many tickets
has she bought? NCERT Exemplar
1
simultaneously are treated as the same experiment.
Sol. Total number of outcomes = 36 Sol. Given, tocal number of sold tickets = 6000
(t) Let E = Event of getting 5 on adeast one die Let she bought x tickets.
Then, E would consist of 11 (6,
outcomes, namely Then, probability of her winning the fi rsc prize
5 ) , ( 2, 5), 5),
(1, 3), (3,5), (5, 5), 5), ( 5, 1 ), (5, 2),
(4, x [given
( 5, (5, and ( 5,
4) 6). 6000_ = 0.08
_
:. Number of outcomes favourable to E 1 1
x = 0.08 x 6000
Hence, probability that 5 will come up adeast once,
=
x = 480
P(E) = .!..!_
36 Hence, she has boughc 480 tickets.
NCE RT F O L D E R 6.2 (O pti o n a l)
- - --- -- - --- ---- - · ---- - - - - - - - -- - � ----··� ---- - �-- - .. - - -�-- - · · - - -- - - - - ---·-- - - - ---! . L _,, _ -- - �-�--· -

1 Two customers , Shyam and Ekta are visiting a Sol. The complete table is
3:
particular shop in the same week (Tuesday to N u mber in first throw
Saturday) . Each is equally likely to visit the
shop on any d ay a s on another day.

-
'C
+ I1 2 2 3 3 6
c:
1 2 3 3 4 4 7
What is the probability that both will visit the 0
(..) 2 3 4 4 5 5 8
Q)
shop on rn
.5
2 3 4 4 5 5 8
(i) the same day? (ii} consecutive days? ...
Q) 6 6
3 4 5 5 9
(iii) different d ays? ..0
E 3 4 5 5 6 6 9
::::s
Sol. Number of days from Tuesday to Saturday = 5 z 6 7 8 8 9 9 12
Total number of possible outcomes for both = 5 x 5 = 5 2
(z) If both visit the shop on the same day, then there are
Total number of possible outcomes = 36
5 days. So, number of favourable outcomes (z) Let £1 = Event that total score is even which can be
=5 2, 4, 6, 8, 12
Now, P (both will visit on the same day) = 1._ = .!..
:.Number o f outcomes favourable to E, = 18
2 5 5 Hence, P {total score i s even) = � = .!_
(it) If they visit the shop on consecutive days, then 36 2
favourable outcomes are (it) Let £2 = Event that total score is 6
( T, W ) , ( W, Th ) , ( Th, F ), (F, S ) , (S, F), :.Number of outcomes favourable to E2 = 4
( F, Th ) , ( Th, W') and � T )
·:.Number of favourable outcomes = 8 Hence, P (total score is 6) _!36 .!_9 = =

Hence, P (both will visit on consecutive days) = -


8 (iiz) Let £3 = Event that total score is atleast 6,
(iiz) P (both will visit on different days)
5 2 i.e. 6, 7, 8, 9, 12
:.Number o f outcomes favourable to E3 = 1 5
= 1 - P (both will visit on the same day) . 15 5
1 4 Hence, P (total score 1s atleast = - = 6) 36 -2
=1 -- =- 1
5 5
3 A bag contains 5 red balls and some blue b alls.
2 A die is numbered in such a way that its faces If the probability of drawing a blue b all is
show the numbers 1 , 2 , 2 , 3 , 3 , 6. It is thrown double that of a red ball, determine the number
two times and the total score in two throws is of blue balls in the bag.
noted. Complete the following table, which
Sol. Lee the number of blue balls be n.
gives a few values of the total score on the two Tocal number of red balls = 5
throws :.Total number of balls = 5 + n
3:
N umber in first throw Now, P(drawing a red ball)
0
... +
=
2 2 3 3 6 Number of red balls
------ = --
5
"O 2 3 3 4 4 7 Total number of balls 5 + n
c:
0
(..)
2 3 4 4 5 5 8 . Number of blue balls
Q) an d P (d rawmg a bl ue ball) = ------
en 2 5
.5
Total number of balls
...
Q) --
3 n
..0 3 9 = --
E 5 S+n
::::s
z
6 7 8 8 9 9 12
According to the question,
What is probability that the total score is P (drawing a blue ball) = 2 x P (drawing a red ball)
n 5
(i) even? => -- = 2 x -- => n = 2 x 5 = 10
(ii) 6? 5+n 5+n
(iii) atleast 6? Hence, the number of blue balls in the bag is 1 0 .
214 1 All{none MATH EMATICS C lass 1 0th Term

x+6 x
4 A b ox contains 1 2 b alls , out of which x are => -- = -
black. I f o n e b all is drawn at random from the 18 6
b ox , wha t is the p robability that it will be a => 6 (x + 6 ) = 1 B x
black b all? => x + 6 = 3x => 2x = 6 => x =3
5 A jar contains 24 marbles, s o m e a re green and
I f 6 m o re black b alls are put in the box, the
probability o f d rawing a black ball is now
others are blue. If a marble is d rawn at random
from the jar, the probability that it is green, is
d ouble o f wha t it was before. Find the value
o f x.
Sol. Given, number o f black balls = x
�. Find the number of blue m a rbles in the jar.
3 CCE 2013
and total number of balls = 1 2 Sol. Toral number of marbles = 24
: . Probability o f drawing a black ball, Lee che number of green marbles = x

P.1 =
N umber of black balls x Then, number of blue marbles = 24 - x
=-
Total number of balls 1 2 Now, probability of drawing a green marble
If 6 more black balls are put in the box, then total number _- Number o f green marbles = � [given]
of black balls = x + 6 Total number o f marbles 3
and total n umber of balls = 1 2 + 6 = 1 8 x 2
=> -=- => 3 x = 24 x 2
x+6
Then, pro bability of drawing a black ball, Pi = la
24 3
24 x 2
=> x= --
=> x =16
Now, according to the question, Pi = 2P. 3
x+6 :. Number of green marbles = 1 6
=> =2x�
18 12 Hence, number of blue marbles = 24 - 1 6 = 8
�I Very Short A nswer Type Questions [1 M a rk e a c h ]

1. What i s the probability o f a n impossible event? 13. A die is thrown once. Find the p robability o f
getting a number which i s not a factor o f 3 6 .
2. What is the probability of a sure event? CCE 201 5
CCE 2013, 12
1 5. Th e probability o f getting a rotten e g g from
3. Find the sum of probabilities of all the a lot of 400 eggs is 0.035 . Find the number
elementary events of an experiment. of rotten eggs in the lot.
CBSE 20 1 1 ; NCERT Exemplar
4. What is the probability of non-occurrence of
an event that is certain to happen? CCE 2015 16. A single letter is selected at random from
the word 'PROBABILITY' . Fin d the
5. The probability that it will rain today is 0.07. probability that it is a vowel. CBSE 20 1 2
What is the probability that it will not rain
today? 1 7. A letter o f English alphabet i s chosen at
random. Determine the probability that the
6. In a simultaneous toss of two coins, find the letter is a consonant.
probability of exactly one head. CCE 2013 CCE 201 4; NCERT Exemplar

18. A letter is chosen at random from the


7. A die is thrown once. Find the prob�bility of
English alphabet. Wh at is the probability
getting an even number less than 5. that it is a letter of the word
CCE 2013
'RAMANUJAN ' ? CCE 201 5

8. A die is thrown once. Then, find the


probability of getting a number less than 3 .
19. A coin i s tossed two times. Find the
CBSE 2008 probability of getting atmost one head.
NCERT Exemplar
9. If a die is thrown, what is the probability of
getting a n umber less than 3 and greater 20. Find the probability that a non-leap year
than 2 ? CBSE 201 1 selected at random will contain 53 S undays .
CCE 20 16; NCERT Exemplar
10. An ordinary die is thrown once. What is the
probability that the number appearing on 21. A school has five houses A, B , C , D and E. A
the ordinary die is greater than 3 ?CBSE 2010 class has 23 students, 4 from house A ,
8 from house B , 5 from house C, 2 from
1 1. Cards marked with numbers 5 to 7 5 are house D and rest from house E. A single
placed in a box and mixed thoroughly. One student is selected at random to be the class
card is drawn from the box. Find the monitor. Find the probability that the
probability that the number on the card is selected students is not from A, B and C.
odd. CCE 2015 NCERT Exemplar

12. A number is chosen from 1 to 100. Find the 22. The probability of guessing the correct
probability that it is a prime number. answer to certain question is p I 1 2 . If the
NCERT Exemplar probability of not guessing the correct
answer to same question is 3 I 4, then find
1 4. A game of chance consists of spinning an the value of p. CBSE 20 1 1
arrow which comes of rest pointing at one of
the numbers 1 , 2 , 3, 4, 5, 6, 7, 8 and these are 23. A number x is chosen at random fro m the
equally likely outcomes. Find the numbers - 4, - 3 , - 2 , - L 0, L 2 , 3 , 4 . What
probability that the arrow will point at any is the probability that I x I < 2? CCE 20 1 6
factor of 8 . CCE 2015
AllinOne MATH EMATICS C lass 1 0th Term I

�I Short A nswer Type I Questions [2 M a rks eac

24. A die is thrown once. What is the probability 33. Aninteger is chosen between 0 and 100.
of getting a p rime number? CCE 2014 What is the probability that it is
(i) d ivisible by 7 ?
2 5 . A card is d rawn from a well-shuffled pack of (ii) not d ivisible b y 7 ?
52 card s . Find the probability that .the card CCE 2 0 1 5; NCERT Exemplar
d rawn is neither a red card nor a queen.
CCE 2016 34. A carton of 24 bulbs contains 6 defective
bulbs. One bulb is drawn at random. What
2 6 . A bag contains 2 green, 3 red and 4 black is the probability that the bulb is not
balls. A ball is taken out of the bag at defective? If the bulb s elected is defective
random. Find the probability that the and it is not replaced and a s econd bulb is
s elected ball is CBSE 201 1 selected at random from the rest, then what
(i) no t green. (ii) not black. is the probability that the second bulb is
defective? N CERT Exemplar
2 7. A card is drawn at random from a
well-shuffled pack of 52 cards. Find the 3 5. Box A contains 25 slips o f which 1 9 are
probability of getting marked ' 1 and other are marked ' 5 . Box B
(i) a red king. (ii) either a queen or a j ack. contains 50 slips of which 4 5 are marked
' 1 and others are marked � 1 3. Slips of
28. One card is drawn at random from a both boxes are put into a third box and
well-shuffled pack of 52 cards. What is the reshuffled. A slip is d rawn at random. What
probability that the card drawn is either a red is the probability that it is marked other
card or a king? than ' 1 ? N CERT Exemplar

29. From a group of 2 boys and 2 girls, two 36. If probability of success is 63%, what is the
children are selected at random. List the probability of failure?
possible outcomes. Find the probability that
one boy and one girl are selected. CBSE 201 1 3 7. A number is selected at random from first
50 natural numbers . Find the probability
3 0 . Two friends were born i n the year 2000. What that it is a multiple o f 3 and 4 . CBSE 2009
is the probability that they have the same
birthday? CCE 2013, 12 38. [[QJ1J A number is selected from the
numbers 2, 3 , 3 , 5 , 5 , 5 , 7 , 7 , 7 , 7 , 9, 9, 9, 9, 9
3 1 . Cards bearing numbers 2, 3, 4, . . . , 1 1 are at random. Find the probability that the
kept in a bag. A card is drawn at random number selected is CCE 2015
-

2:.
from the bag. Find the probability of getting (i) their med ian .
f)
a card with a prime number. (ii) their mode.
f)
.LJ 3 2. Apoorv throws two dice once and computes 39. [HOTS] A die has its six faces marked 0, 1 , 1 , t ,
n the product of the numbers appearing on the 6, 6. Two such dice are thrown togeth er
f) dice. Peehu throws one die and squares the and the total score is recorded .

�I
:( number that appears on it. Who has the (i) How many d iffe rent s co re s a re equal?
b etter chance of getting the number 36? (ii) What is the probabili ty of getting a to ta l
Why? NCERT Exemplar of 7? NCERT Exemplar
Probability 217

� I Short A nswer Type I I Questions


R
[3 M a rks e a c h ]

40. Two dice are thrown at the same time and 4 7. A square of side 5 o c

O
the product of numbers appearing on them cm is drawn in the
s
0 1 0 cm
is noted. Find the probability that the interior of another
product is less than 9 . NCERT Exemplar square of side 1 0 cm
and shaded as
4 1 . A bag contains 5 red balls, 8 white balls, shown in P
the L....- ----L- ---' B
following figure. A A
- -

4 green balls and 7 black balls. If one ball is


drawn at random, then find the probability point is selected at random from the interior
that it is of square ABCD. What is the probability
(i) bla ck. (ii) red . (iii) not green. that the point will be chosen from the
shaded part? CCE 201 5
42 . A carton consists of 1 00 shirts of which
88 are good, 8 have minor defect and 4 have 4 8. A child has a die, whose s ix faces show the
major defect. Jimmy, a trader, will only letters as given below
accept the shirts which are good but Sujata, F G H I J F
another trader, will only reject the shirts The die is thrown once. What is the
which have major defect. probability of getting
One shirt is drawn at random from the (i) F?
carton. What is the probability that it is (ii) H?
acceptable to (iii) a vowel?
(i) Jimmy? (ii) S ujata?
49. A die is thrown once. What is the
43. One card is drawn from a well-shuffled deck probability that it shows
of 52 cards. Find the probability of getting (i) 3 ?
(i) the ki n g of heart.
(ii) 4?
(ii) a diamond.
(iii) number greater than 4 ? CCE 201 5
(iii) the j ack of spade.
5 0 . Three
44. The king, queen and jack of clubs are
unbiased coins are tossed
simultaneously. Find the probability of
removed from a deck of 52 playing cards and getting
then well-shuffled . Now, one card is drawn
(i) exactly 2 heads. (ii) atmost 2 h e ad s .
1-
at random from the remaining cards. Find
the probability of getting a card of 5 1 . Two dice are thrown simultaneously. What 2
(i) a heart. (ii) a king. NCERT Exemplar is the probability that the sum of the u..

45 There are 1 00 cards in a bag on which



numbers appearing on the dice, is 2
numbers from 1 to 1 00 are written. A card is
(i) 7? v
(ii) not a prime number? v

taken out from the bag at random. Find the
(iii) 1 ? CCE 20 1 4; NCERT Exemplar
probability that the number on the selected
V
5 2 . In a game, the entry fee is � 5 . The game u .
card CCE 2016
(i) is divisible by 9 and is a p erfect square. consists of tossing a coin 3 times. If one o r <:=t
(ii) is a prime number greater than 80. two heads show, Sweta gets h e r entry fee
46. All kings , queens and aces are removed
back. If she tosses 3 heads, she receives
double the entry fee . Otherwise, she will
from a pack of 52 cards. The remaining loss. For tossing a coin three times , fin d the
cards are well-shuffled and then card is probability that she
drawn from it. Find the probability that the ( i ) losses the entry fee.
drawn card is CBSE 2012
(ii) gets double entry fee.
( i ) a b l a c k face card .
(iii) just gets her entry fee. NCERT Exemp lar
(ii) a red queen card .
21s t AllinOne MATH EMATI CS C lass 10th Term I

53. I n the given figure, a cross-mark is 5 7. One card is drawn from a pack of 5 2 cards, each
placed at random on the rectangular of the 52 cards being equa lly likely to be drawn.
region. What is the probability that it Find the probability that the card drawn is
will b e placed inside the circle with (i) a red king . (ii) ' 2 ' of spade.
radius l m? (iii) ' 1 0 ' of a black suit . CCE 2016
6m
58. A pair of dice is th rown once. Find the
3m probability of getting
(i) doublet of prime numbers .
(ii) a doublet o f odd nu mbers . CCE 2015

5 4. A game of chance c 59 . Ankita and Nagma a re two friend s . They were

A B both born in 1 99 0 . What is the probability that


of an

ffi
c onsists
a rrow which they have CBSE 201 1
comes to rest 0 (i) same birthday? (ii) different birthdays?
pointing at one of 3
the regions 1 , 2 or 60. A bag contains 2 4 balls o f which x are red, 2x
3. 0 is the centre of the circle, OC .l AB. are white and 3x are blu e . A ball is selected at
Find the probability that random. What is the p robability that it is
(i) arrow is resting on 3 . (i) not red? (ii) white? N CERT Exemplar
(ii) arrow i s resting on 1 .
61. Three different coins are tossed together. Find
(iii) arrow is not resting on 2 . CBSE 2012
the probability of getting
55 . Cards with numbers 2 to 1 0 1 are placed (i) exactly two heads .
in a b ox. A card is selected at random. (ii) atleast two heads.
Fin d the p robability that the card has (iii) atleast two tails . CCE 2016
(i) an even number.
(ii) a square number. NCERT Exemplar
62. [HOTS) A bag contains 1 8 b alls o ut of which
x balls are red .
56. Two dice are thrown together. Find the (i) If one ball is drawn at random from the bag,
p robability that the product of the what is the probability that it is red ball?
numbers on the top of the dice is (ii) If 2 more red balls are put in the ba g, the
(i) 6 . probability of drawing a red ball will be
(ii) 1 2 .
� times that of probability of red ball coming

�I
(iii) 7 . CBSE 201 1 ; NCERT Exemplar 8
in part (i) . Find x. CCE 201 5

>
- Long A ns wer Type Questions [4 M arks each]
,
J
.J 63. Two unbiased coins are tossed 65. (i) Find the probability o f getting 53 Fridays
, simultaneously. Find the probability of in a leap year.
') g etting (ii) A die is thrown, find the p robability of
( (i) no head . getting an odd prime number.
(ii) atmost one tail .
.I
:. (iii) one tail. 66. A child 's game has 8 triangles of which 3 are
- (iv} one head and one tail. CCE 2015 blue and rest are red and 1 0 squares of which 6
- are blue and rest are red . One piece is lost at
c- 64. In a single throw of two dice, find the random. Find the probability that it is a

-- probability of getting
(i) triangle.
(i) a total of 1 1 .
• (ii) square .
..
(ii) doublets .
) (iii) six as a product.
(iii) square of blue colour.
) '
(iv) triangle of red colour. NCERT Exemplar
Probab i lity f 219
67. In the given figure, a game of 71. In the given figure, points
chance consists of spinning an A, B, C and D are the centres
arrow which comes to rest of four circles that each
pointing at one of the numbers have a radius of length one
1 , 2 , 3 , 4 , 5 , 6 , 7 , 8 and these unit. If a point is selected at
are equally likely outcomes. random from the interior of
What is the probability that it will point at square ABCD, what is the probability that
(i) 6? (ii) an even number? the point will be chosen from the shaded
(iii) a number greater than 4? region?
(iv) a number less than 8?
72. A square dart board is placed in the first
68. All the three face cards of spades are removed quadrant from x = 0 to x = 6 and y = 0 to
from a well-shuffled pack of 52 cards. A card y = 6. A triangular region on the dart
is drawn at random from the remaining pack. board is enclosed by the lines y = 2, x = 6
Find the probability of getting and y = x. Find the probab µ_ity that a dart
(i) a black face card . (ii) a queen. which randomly hits the dart board will
(iii) a black card . (iv) a spade. CCE 2013 land in the triangular region formed by
the three lines.
69. A bag contains white, black and red balls
only. A ball is drawn at random from the bag. 73. A number x is selected at random from the
The probability of getting a white ball is � numbers 1, 4 , 9, 1 6 and another n umber y
10 is selected at random from the numbers 1 ,
�. 2 , 3 , 4 . Find the probability that the value
of xy is more than 1 6.
and that of a black ball is Find the
5 CCE 201 6
probability of getting a red ball. If the bag
contains 2 0 black balls, then find the total 74. IHOTSJ A die is rolled twice. Find the
number of balls in the bag. NCERT Exemplar probability of
(i) 4 turning up atleast once.
70. There are 1 000 sealed envelopes in a box, 10 (ii) 4 not turning up either time.
of them contain a cash prize of � 100 each, 100
of them contain a cash prize of � 50 each and 7 5. (HOTSJ A traffic signal displays green light
200 of them contain a cash prize of .� 10 each for two minutes to allow passage of traffic
and rest do not contain any cash prize. If they on a particular road. If the signal is
are well-shuffled and an envelope is picked currently displaying green light, then fin d
up out, what is the probability that it contains the probability that i t will turn red within
no cash prize? NCERT Exemplar the next half a minute.

�I Value Based Questions (VBQs) [4 M arks eac h ]

76. A group consists of 1 2 persons, out of which 3 77. Ram goes to school either by a car driven
are extremely patient, other 6 are extremely
by his driver or uses his bicycle.
honest and rest are extremely kind. A person
from the group is selected at random . Probability that he will use the car is � .
7
(i) What is the probability that he will use
Assuming that each person is equally likely to
be selected, find the probability of selecting a
his bicycle for going to the school?
person who is
(ii ) Shyam is the best friend of Ram .
( i ) e x t re me ly patient. Which mode of transport should
( ii) extremely kind or honest. Shyam suggest to Ram for going to
Which of the above values you prefer more? the school and why?
CCE 2013
Allinone MATH EMATI CS C lass 1 0th Term I

7 8 . A group of students from a school decided to


7 9. Anu, Priya and Jyoti were fighting to get
d on ate blood. The blood group of
first chance in a game. Anu s ays, "Let us
1 6 students of Class X are recorded as
toss two coins. I f both hea d s appear, Priya
will take first chance, if b o th tails appea�
follows
A, B , O, O, AB, O, A, 0, B, A, 0, B, A, Jyoti will ge t it and if one head and one tail
0, 0, AB . appears, I will get t h e c ha n c e " .
(i ) Find the p robability o f blood group (i) What is the p robability of Anu, Priya and
(a) 0. Jyoti getting the first cha nce?
(b) A. (ii) Is her decision fa i r?
(ii) What value is d epicted from this ac tivity? (iii) What quality of her character is being
depicted here?

So l u tions
1 . The probability o f an impossible event is 0. [1 ] :.Number of possible ouccomes = 6
2 . The probability of a sure event is 1 . [1 ] Let E be the event that the number appearing on the
ordinary die is greater than 3, then E would consist of
3 . The sum of probabilities of all the elementary events of
an experiment is 1 . [1 ]
4,
three outcomes, namely 5 and 6. [1/2)
:.Number of outcomes favourable to E = 3
4. Probability of non-occurrence of an event that is
certain to happen is zero. [1 ] Hence, the required probability, P ( E ) = � = _!_ [1 /2]
5 . Given, P (E) = 0.07
6 2
1 1 . Given cards marked with the numbers 5 to 75 .
. . P ( not E) = P(E) = 1 - P(E) :. Total number of cards = 7 1
[·: P(E) + P(E) = 1) Let E = Event of getting a n odd number from 5 to 75,
= 1 - 0.07 = 0.93 [1 1 i.e. event of getting = 5, 7, 9, 1 I , . . . , 75 [1/2]
6. Possible outcomes in a toss of two coins are So, the number of outcomes favou rable to E = 36
HH, HT, THand TT 36
:. P (drawn card is of odd number) = [1/2]
.·. N umber of all possible outcomes = 4 71
Let E = Event of getting exactly one head 1 2. Total number of outcomes = 1 00
So, E would consist of two outcomes, namely HT and Let E = Event of getting a prime number, i.e. event of
TH. getting 2, 3, 5, 7, 1 1, 1 3, 1 7, 1 9, 23, 29, 3 1 , 37, 4 1 ,
:.Number offavourable outcomes = 2 [1 /2] 43, 47, 53, 59, 6 1 , 67, 7 1 , 73, 79, 8 3 , 8 9 or 97
� _!_ :. Number of outcomes favourable to E = 25 [1 /2)
So, required probability = =
25
4 2 Hence, P (getting a prime number) = = _!_ [1/2]
1 00 4
Hence, the probability of exactly one head is ..!... [1 /2]
2 1 3. All possible ouccomes are I, 2, 3, 4, 5, 6, 7, 8.
7. Possible outcomes in a single throw of a die are :. Number of all possible outcomes = B
1 , 2, 3, 4, 5, 6. The factors of B are 2, 4 and 8 .
:.Total number of outcomes = 6 :.Number o f favourable outcomes = 3 [1 /2]
There are two even numbers, 2 and 4, which is less
than 5 . Hence, required probability = � [1/2]
B
:. Number o f favourable outcomes = 2 [1 /2]
1 4. On throwing a die, possible outcomes are
So, required probability = � = _!_ [1/2] I, 2, 3, 4, 5, 6
6 3
-
:. Number of possible ouccomes = 6
I

- 8. Do same as Q. 7 [Ans. _!_] The factors of 36 are l , 2, 3, 4, 6, 9, 1 2, 1 8, 36.


3
Here, we see that 5 is not a factor of 36.
9. There is no such number lies on a die which is less than
:. Number of outcomes favourable to E = I [1 /2]
3 and greater than 2.
:. Probability will be zero. Now, P (getting a number which is not a faccor of 36)
[1 ]
= -
1
1 0. On throwing an ordinary die, possible outcomes are (1/2]
I , 2 , 3 , 4 , 5 and 6. 6
Probab ility f 221
Probability (guessing the correct answer),

(1/2]
1 5. Given, total number of eggs = 400
P(E) = .!..._
[1121
·: Probabiliry of gcccing a rorren egg, [given]
P( E ) =
N umber of rorren eggs 12

(1 /2]
Probability (not guessing the correct answer),
Tora! number of eggs
-
P (E) =
3
4" [given]

12 p �4 �4 12
:. Number of rot ccn eggs
= Tora! number of eggs x P (E)
= 400 x 0.035 = 14 We know that, I - P(E) = P(E)
Hence, rhc number of rorren eggs are 14. [1/2] => 1 -L = => l - = L => p = 3
1 6. Total number of ouccomes = 1 1
There arc three vowels and one is repeated. So, number
Hence, the value of is 3. (1 /2)
2

of favourable outcomes = 4 23. Number of all possible outcomes = 9
Numbers favourable to I x I < are - 1, 0 and 1 .
[1]
. . Required probabiliry
N umber of favourable ourcomes 4 :. Number of favourable outcomes = 3
Total number of outcomes 11
1 7. We know rhar, in English alphabet, there are
So, required probability =
9
= .!.
3
(1]
26 lcctcrs (5 vowels + 2 1 consonants).
So, total number of ourcomes = 26 [1/2] 24. Do same as Q. 1 3 part (t) of NCERT folder 6. 1 at page
209. [Ans. 1 /2]
Let E = Event of choosing a consonant, i.e. choosing 25. Total number of cards in a pack = 52
b, c, d, f, g, h, j, k, /, m, n, p, q, r, s,
t, v, w, x, y or z
Total number of red cards = 26 (including 2 queens)
:. Number of ouccomcs favourable to E= 2 1 Total number of queen cards is 4, out of which 2 are

[1 /2]
black queen cards and 2 are red queen cards.

28 (1 ]
Hence, required probabiliry = P(E) = Total number of red cards and queen cards
26
= 26 + 2 =
1 8. Number of all possible outcomes = 26
Let E be the event of getting neither red nor a queen
Lerrers in the word ' RAMANUJAN' are R, A, M, N, U,

[1/2]
card, then
- 28 24
24 �
J. Number of outcomes favourable to E= 52 =
:.Number of favourable outcomes = 6
i_ = i_ [1/2] :.Required probability = P(E) = [1 ]
=

+3+4
Hence, required probabiliry = 52 13

Z
26 13

-2
26. Total number o f balls in the bag = 2 =9
1 9. Do same a s blackboard Ex. part (it) at page 20 1 .
(1) Number of non-green balls = 9 = 7

(1]
[Ans. 3/4]
20. In a non-leap year, total number of days is 365. :. Required probability =

4
9
Out of them, there are 52 weeks and 1 day extra.
(ii) Number of non-black balls = 9 - = 5
1-
[1 ]
Thus, a non-leap year always has 52 Sundays. The
remaining 1 day can be Sunday, Monday, Tuesday,
:.Required probability = � 2
LL
· Wednesday, Thursday, Friday and Saturday. 9
Out of these 7 cases, we have Sunday in one case.
:.Total number of ourcomes = 7 [1/2] 27. Total number of cards in a pack = 52
( 1) Let £1 = Event of getting a red king
2
v
Number of favourable outcomes = I
Hence, required probabiliry = .!_ [1/2] :.Number of outcomes favourable to £1 = 2
[·: only king of heart and diamond are red]
Cf
u..
v
7
2 1 . Total number of srudents = 23
:.Number of all possible outcomes = 23 P(E1 ) = =
2
Hence, probability of getting a red king,
I
[1 l
Cf
<l
52
[1/2]
26

4 8 -
Number of srudents in houses A, and C B LL
4 4
{it) Let £2 = Event of getting a queen or a jack
= + + 5 = 17 ......

8
--
There are queen and jack in a pack of cards. -
:.Number of outcomes favourable to E2 =
:. Remaining students = 23 17 = 6
Thus , number of favourable outcomes = 6
Hence, probability of getting a queen or a jack, �
[1/2]
So, probabiliry that the selected student is not from A, B
and C = i_
8
P(E2 ) = - = -
52
2
13
[1 ] :ii
23
22. Let E = Event of guessing the correct answer to a 28. Total number of cards i n a pack = 52
:ii
-
certain question :. Number of all possible outcomes = 52
-
u
Allinone MATH EMATICS C lass 1 0th Tenn I

Let E be the event of getting a red card or a king. :.Number of outcomes favourable co £2 = 99 - 14
Then, number of outcomes favourable to E
:�
= 85
(1 )
= Number of red cards
Hence, required probability = P( £2 ) =
+ Number of kings in black cards
34. Do same as Q. 1 7 ofNCERT folder 6. 1 ac page 2 1 0.
= 26 + 2 = 28 [1 ]
H ence, required probability = P (E) =
28
= !_ (1 ] [Ans. 3/4 and 5/23]
52 13
35. Total number of slips in third box = 25 + 50 = 75
29. Let the 2 boys and 2 girls be denoted by B1 , B2 , Box A Box B
G1 and G2 , respectively.
Here, two out of four children are selected.
:. The total possible outcomes are
B1 B2 , B1 G. , B1 G2, B2 G. , B2G2 and G1 G2•
:. Number of all possible outcomes = 6 (1 ]
I f one boy and one girl are selected, then favourable
outcomes are B1 G1 , B1 G2 , B2G1 an d B2G2 •
:.Number of favourable outcomes = 4 From the chart, it is clear chat there are 1 1 slips which
H ence, required probability = = 3. i (1 ] are marked other than � 1 .
6 3 Let E be the event of getting a slip which is marked
30. Total number of days in the year 2000 = 366 other than � 1.
[·: year 2000 is a leap year] Then, number of outcomes favourable co event E= 1 1
Total number of ways in which two friends may have :. Required probability
their birthday = 366 x 366 [1 ] Number of favourable outcomes 11
= = - (1 )
N umber o f ways i n which two friends have same Total number of o utcomes 75
birthday = 366 36. We know chat,
:. Probability that both have same birthday Probability of success + Probability of failure =: 1 [1 ]
366 1
=
366 x 366 366
[1 ] => _§_ + Probability of failure = 1
100
3 1 . D o same as Example 5 part (it) at page 204. [Ans. 63 37
=>
112) Probability offailure = 1 - = = 37% [1 ]
32. Apoorv throws two dice once. 1 00 1 00
So, total n u�ber o f outcomes = 36 37. Number of all possible outcomes = 5 0
N umber of outcomes for getting product 36 = 1 l,
[·: outcome can be anyone of 2 , 3 , . . . , 49, 50] ( 1 ]
[·: only (6, 6) gives product 36) Number o f numbers which are multiple o f 3 and 4
1 = 4 [·: multiple of 3 and 4 are 12, 24, 36 and 48]
:.Probability for Apoorv = - [1 ]
36 Probability that the selected number is multiple of 3
4 2
Also, Peehu throws one die. and 4 :::: - = - (1 ]
::=::' So, total number of outcomes = 6 50 25
w Number of outcomes for getting 36, which is square of 38. (t) Median of the numbers is 7. So, P (Median) = _!
>
number 6 = 1 [·: only square of 6 gives 36] 15
') So, probability for Peehu = .!_
= i_ [·: there are four 7s and total n umbers are 1 5] [1 ]
5 1
f)
6 36
(ii) Mode = 9 P(Mode) = - = -
u
[1 ]
·: Probability for Apoorv < Probability for Peehu l
5 3 [· : there are five 9s]
f)
:. Peehu has better chance of getting the number 36. 39. Given, a die has its six faces marked 0, l, 1 , 1, 6, 6.
,.. [1 ]
·: Two such dice are thrown together, therefore number
3 3 . The total number of integers between 0 and 1 00 = 99 of all possible outcomes = 36
(t) The different scores which are possible = scores,
u
(t) Let £1 = Event of getting an integer which is 6

divisible by 7, i.e. event of getting l,
namely 0, 2, 6, 7 and 1 2. [1 ]
IK:I 7, 1 4, 2 1 , 28, 3 5 , 42, 49, 56, 63, 70, (iz) Le t E = Event o f getting a s u m 7 , i .e. event of
77, 84, 91 or 98 getting
:. Number of outcomes favourable to £1 = 1 4 (1, 6), (1, 6), ( 1 , 6), ( 1 , 6), (1, 6), 6), ( I,
Hence, req ui red probability = P(E1 ) :::: .!_i [1 ]
(6, 1), (6, 1 ) , (6, 1 ), (6, 1 ) , (6, 1 ) or (6, 1)
99 :. Number of outcomes favourable to E = 12
(it) Let £2 = Even c of getting an integer which is not Hence,
12
P(E) = - = -
1 (1 ]
divisi ble by 7 36 3
Probability f 223 1
40. When cwo dice arc thrown simultaneously, then Area of che square PQRS
=
number of possible ou tcomes = 36 [1] Area of the square ABCD
·: Produce of numbers is less than 9, therefore 25
favourable outcomes arc = - = 0.25 [1 ]
1 00
( 1 , 1 ) , ( 1 , 2) , ( 1 , 3), ( 1 , 4) , ( 1 , 5), ( 1 , 6), (2, I ), (2, 2),
(2, 3), (2, 4), (3, 1 ), (3, 2) , (4, 1 ), (4, 2), (5, 1) and (6, I ) 48. Do (z) and (ii) same as Q. 1 9 NCERT folder 6. 1 at
i.e. 1 6 outcomes [1] page 2 1 0-2 1 1 . [Ans. {i) 113 (ii) 116]
(iii) Let � be the event of getting a vowel, i.e. event of

[ ]
16 4
Hence, P {produce < 9) = - = -
36 9
[1] getting I.
:. Number of outcomes favourable to = 1 �
41 . Do same as Q. 9 of N CERT folder 6. 1 at page 208. 1
Hence, P(� ) = [1 ]
7 " 5 '" ' 20
An s. ( l ) - ( ll ) - (Ill ) - G
2 24 24 49. Total number of outcomes = 6
42. Do same as Example 7 at page 205-206. (z) Lee £1 be the event of getting 3.
[Ans. (1) 0.88 (ii ) 0.96] Then, number of outcomes favourable to £1 = I
43. (1) Do same as Q. 1 4 , pare (iv) of NCERT folder 6. 1
1
P (E1 ) = G (1 ]
at page 2 09. [Ans. 1152]
(iz) Do same as Q. 1 4, pare ( v) of NCERT folder 6. 1 at (ii) Do same as part (t) . [ Ans. 1/6]
page 209. [Ans. 1/4] (iii) Do same as Example 2 part (t) at page 206. [Ans. 1/3]
(iii) Do same as Q. 14 pare {iv) NCERT folder 6. 1 at 50. When three coins are tossed simultaneously, all
page 209. [Ans. 1152] possible outcomes are
44. Do same as blackboard Example at page 203. HHH, HHT, HTH, THH, HTT, THT, TTH and TTT.
[Ans. (i) 1 3/ 49 (ii) 3/ 49] Total number of possible outcomes = 8
45 . Total number of cards in the bag = 1 00 (1) Lee £1 be the event of getting exactly 2 heads.
:.Number of all possible outcomes = 1 00 Then, favourable outcomes are
HHT. HTH and THH
(z) Lee £1 be the event of getting a number which is
divisible by 9 and is a perfect square. Then, Number of outcomes favourable to £1 = 3
2 2 2 . . P (getting exactly 2 heads)
outcomes favourable to £1 are 3 , 6 , 9 , i.e. 9, 36
and 8 1 . = P (E1 ) = � [1 %]
:. Number of outcomes favourable co £1 = 3 8
(iz) Let £2 be the event of getting atmost 2 heads, i.e.
Hence, required probability =P(E) = 2_ %] [1 E2 be che event of getting 0, 1 or 2 heads.
1 00
{iz) Let £2 be che event of getting a prime number So, the favourable outcomes are
greater than 80. TTT, HTT. THT. TTH. HHT. HTH and THH .
Then, outcomes favourable co £2 are 83, 89 and Number of outcomes favourable to E2 = 7
97. :.P (getting atmost 2 heads) = P ( E2 ) = ?_ [1 %]
:. Number of outcomes favourable co E2 = 3 8
5 1 . Two dice are thrown simultaneously.
Hence, required probability =P(E2 ) = [1 %] � So, cocal number of possible outcomes = 36 [1]
1 00
46. Since, all kings, queens and aces are removed. (z) Lee £1 be the event of getting the sum 7.
Total number of cards lefc = 52 - 12 = 40 [1] :. Favourable outcomes are ( I , 6) , (6, I ) , (3, 4),
(t) Number of black face cards = 2 (4, 3), (5, 2) and (2, 5).
Number of outcomes favourable to £1 = 6
P {a black face card) =
40
=-
1
20
� [1] Hence, required probability
{iz) Number of red queen card = 0 6 1
0
= P (E1 ) = - =
36
-

6
(1 ]
P (a red queen card) = - = 0
40
[1]
(iz) Let £2 be an event of getting sum of the numbers,
2 2 appearing on che dice, a prime n umber, i.e. getting
47. Area of the square ABCD = (BC ) 2 = 1 0 = 1 00 cm
2 2 [1 1 sum 2, 3, 5, 7 or 1 1 .
Now, area ofthe square PQRS = (Side) = (5)
:. Favourable outcomes are { I , 1 ) , ( I , 2 ) , ( 2, 1),
[·: side = 5 cm, given] 5) ,
( 1 , 4) , ( 2 , 3) , (3, 2), (4, l ) , ( 1 , 6) , ( 2, (3, 4) ,
= 25cm 2 [1] (4, 3), (5, 2), (6, 1 ) , (5, 6), (6, 5 ) .
:.P (the point will be chosen from che shaded pare) Number o f outcomes favourable to £ = 1 5
2
Allinone MATH EMATICS C lass 1 0th Term I

Hence, required probability = 1 - P(E) = 1 - !236 (2)2 , (3)2 , (4) 2 , ( 5 )2 , (6)2 , (7)2 ,
= -
7 [1] i.e.
(8) 2 , (9)2 or (I0\2
4, 9, 16, 25, 36, 49, 64, 8 1 or 100
:. Number of outcomes favo urable co £2 9 (1]
12 =
(iii) As 1
cannot b e the sum o f the numbers appearing
on the dice. So, its probability is zero. [1] Hence, required probability = P(E,) - = _.2_100 (1)
3
S2. Possible o utcomes on tossing a coin times, are
HHH , TIT , HTT , THT , ITH, THH , HTH , HHT 56. Number of all possible outcomes = 36
:. Total number of outcomes = 8 (1) When produce of chc n u mbers on the cop of the
(t) Let £1 = Event that Swet:i losses the entry fee dice is 6.
i.e. She tosses tail three times, i.e. TTT So, favourable outcomes a re ( l , 6), (2, 3), (3, 2)
:. Number of outcomes favourable to £1 = 1 and (6, I ) .

Hence, required probability = P (£1 ) = .!.


8 [1] :.Number o f favourable outcomes = 4
Hence, required p robability = � = .!.
(iz) Let £2 = Event that Sweta gets double entry fee 36 9 (1)
i.e. She tosses heads three times i.e. HHH (ii) When product of the numbers on che cop of the
:. Number of outcomes favourable to £2 = 1 dice is 12.

Hence, required probability = P (E2 ) = .!_


8 [1 ] So, favourable outcomes are (2, 6), (3, 4), (4, 3)
(6, 2).
(iiz) Let � = Event that Sweta gets her entry fee back :. Number of favourable outcomes = 4
i.e. Sweta gets heads one or two times, i.e. event of Hence, required probability = � = .!. [1)
getting 36 9
HTT , THT , TTH , HHT , HTH or THH (iit) Produce of the numbers o n the top of the diet
:.Number of o utcomes favourable to E; = 6 cannot be 7. So, ics probability is zero.

8 = ![1]
[1)
� ways.
Hence, required probability = P (E; ) =

[Ans. 1�]
57. Out of 52 cards, one card can be drawn in 52

[Ans._!_26
4 So, total number of possible outcomes == 52 [1)
as
S3. Do same Q. 20 of NCERT folder 6.1 at page 21 1. (t) Do same Q. 14, pare (z) o f N CERT folder 6. 1
as

at page 213-214 .
S4. Total angle m ade by the circle at 0 is 360°. (ii) There is only one '2' of spade.
Angle subtended by region 1 at 0 = 9 0° :.Total number of favourable outcomes = 1
Angle s ubtended by region 2 at 0 = 90° 1
Angle subtended by region 3 at 0 = 180° Hence, required probability = -- (1)
[1 ]
52
(iit) There are two suits of black cards, namely spades
180° 1
(t) P (arrow resting on 3) = -- = -
360° 2
[1 ]
and clubs. Each suit contains one card bearing
(iz) P (arrow resting on 1) = -- 90° = -1 number 10.
> 360° 4 :.Total number of favourable o utc.o mes = 2
, (iit) P (arrow resting on 2) -- ==
90° = -1 Hence, required probability = � == --
1 [1 )
, 360° 4 52 26
.J 58. Total number of outcomes = 6 x 6 = · 36

== 1 - P (arrow resting on 2) = 1 - � = %
: . P (arrow not resting on 2)
,
,
[1 ] (t) Doublets of prime numbers are (2, 2), (3, 3)
and (5, 5). They are three in numbers.
( So, number of favourable outcomes = 3

36 12 [1 'f.i]
SS. Number of all possible outcomes = 100
.. (z) Let £1 == Event of selecting a card which is an even Hence, required probability = � = _..!.._
:II
- number, i.e. event of selecting
- 2, 4, 6, .. . , 100 (iz) Doublets o fodd numbers are (1, 1), (3, 3) and
c Then, number of outcomes favourable to £1 = 50 (5, 5). They are three in number.
- So, number of favourable outcomes = 3

- :. Required probability = P(E1 ) 2Q_ = .!_
==
-
..
100 2 [1] Hence, P(a doublet of odd numbers) = -
3 1 = -
- (iz) Let £2 == Event of selecting a card which is a square 36 12
) number, i . e. event of selecting crn1
Probability

[ ]
59. Do (1) and {ii) same as Ex. 8 on page 206. ( it) Let £2 = Event of getting atmosc one rail, i .e. even t
1 364 T
o fgetting H , TH o r HH
Ans. {i) -- (ii) :. Number of outcomes favourable to £2 = 3
365 365
60. Total number of balls in a bag = 24
Hence, P (getting acmosc one rail) = P(E) = � [1]
4
Number of red balls = x (iii) Let � = Event of geccing one rail, i.e. event o f
Number of white balls = 2x getting HT or TH
Number of blue balls = 3x :. Number of outcomes favourable to �
= 2
:. x + 2x + 3 x = 24 => 6x = 24 => x = 4 [1 1 Hence, P (getting one tail) = P(� ) = �4
= _!_ [1]
Thus, number of red balls = 4 2
number of wh ite balls = 8 (iv) Let £4 = Event of getting one head and one tail, i . e .
and number of blue balls = 1 2 event of getting H T or TH
( 1) P (nor red) =
20 2_ :. Number of outcomes favourable co £4 = 2
24
=
6
[1 1
Hence P (getting one head and one tail)
� = _!_ = P(E4 ) = 4 = l
2 I

[ �]
(ii) P (white) =
24 3
[1 1 [1 1
6 1 . ( 1) Do same as Q. 50, part ( 1) at page 2 1 7.

[ �J
[1] 64. Total number of outcomes, on throwing two dice = 36
[1 1
( 1) Do same
Ans.
as Q. 22 of NCERT folder 6. 1 at
(ii) Let £2 be the event of getting adeasr two heads, i.e. page 2 1 1 -2 1 2. Ans
event of getting HHT, HTH, THH HHH.
or 1
:.Number of outcomes favourable to £2 = 4 (ii) Let £2 be the event of getting a doublet, i.e. event of

[ �]
Hence, required probability = P(£2 ) = i = _!_
getting (I, I),
(2, 2), (3, 3), (4, 4), (5, 5) or (6, 6).
8 2 [1 ) Thus, number of outcomes favourable to £2 = 6
(iii) Do same as pan (ii) .
Ans. :.P (getting doublets) = P( E2 ) = � = .!. [ 1]
36 6

62. (t) Total number of balls are in the bag = 1 8



(iii) Let be the event of getting 6 as a product.
Since, I x 6 = 6 , 6 x I = 6,
Total number of red balls in the bag = x 2 x 3 = 6,3 x 2 = 6
:. Probability (red ball) = � [1] Thus, �
be the event of getting
18 (1, 6), (6, I ) , ( 2, 3 ) or (3, 2 )
(ii) Now, 2 red balls are added to the bag. : . Number o foutcomes favourable c o event £3 = 4
:.Total balls in the bag = 1 8 + 2 = 20 Hence, P (getting 6 as a product) = P ( E3 )
Total number of red balls in the bag = x + 2 4 I
[1]

()
x + 2 -- =-
:.Probability of drawing red ball = [1] 36 9
65. ( t) Number of days in a leap year = 366 days
20
According to the question, we get
=(52 x 7 + 2) days =52 weeks and 2 days
x + 2 _ 9 - x
--

20
- -

8 18
Thus, a leap year always has 52 Fridays. [1]
The remaining 2 days can be
x + 2 x {a) Sunday and Monday
=> --
= -
20 16 (b) Monday and Tuesday
=> 1 6x + 32 = 20x (c) Tuesday and Wednesday :
.;;.;;:. ..

=> 4x = 32 (d) Wednesday and Thursday


=> x =8 [1] (e) Thursday and Friday
63. Wh en two coins are tossed simultaneously, (f) Friday and Saturday
all possible outcomes are HH, HT, TH, TT. (g) Saturday and Sunday
. __ :. - ·
Total number of possible outcomes = 4 Out of these 7 cases, we have Friday in two cases.
(t) Let £1 = Event of getting no head i.e. event of [1 ]
getting TT So, total number of possible outcomes = 7
:. Number of outcomes favourable to E1 = 1 and total number of favourable outcomes = 2
P (53 Friday) = � [1 ]
Hence, P (getting no head) = P(E1 ) = ..!._
.. - - -..... -- '\ 1

4
[1] 7
, -·-
AllinOne MATH EMATICS C lass 1 0th Term I

( ii) Number of possible outcomes = 6 69. Balls in the bag are of colours white, black and red.
Let E = Event of getting an odd prime number,
i.e. event of getting 3 or 5 Given, probability of gerring a white ball = i_
IO
:. Number of outcomes favourable to E = 2 2
Probability of getting a black ball = -
Hence, required probability = P(E) = � = .!_ [1 ] 5
6 3 Clearly, probability of getting a red ball
66. Total number of figures = 1 - [P(white ball) + P(black ball)]
= 8 triangles + 10 squares = 18 [·: sum of probabilities of all elcmencary events is I )
=1 -- 3
- -2
10 5
10 - 3 - 4 3
= (2)
10 10
Now, lee the number o f balls in the bag be x.

Since, it is given that, the number of black balls in the


4
8 =-
. l e) =-
. is. a tnang bag is 20.
(z) P (lost piece
18 9
[1 ]
:. Probability of getting a black ball = 20
. . 10 5 x
(iz) P (losr piece is a square) = 18
=
9 (1 1 -2 20
= - =::::> x = 50
5 x
(iiz) P (lost piece is square of blue colour) = i_ = .!_ [1 ] Hence, the total number of balls in the bag = 50 (2)
18 3
70. Total number of envelopes in the box = I 000

[Ans.
P ( lost piece is triangle of red colour) = 2_

Z]
(iv) 18
[1 ] Number of envelopes containing cash prize
= 1 0 + 1 00 + 200 = 3 1 0 [2]
67. Do same as Q. 1 2 at of NCERT folder at page 6. 1 Number of envelopes containing no cash prize
208-209. (i) .!_ (ii) _!_ (iii) .!_ and (iv) = 1000 - 3 1 0 = 690
8 2 2 8
68. Total number of cards = 52 :. Required probability = 690 = 0 . 69 (2)
1000
Since, three face cards of spades are removed, then 71 . Since, radius of each of the given circles is 1 unit.
number of remaining cards = 52 - 3 = 49 :.Side of square
: . Total number of possible outcomes = 49 ABCD = 1 + 1 = 2 units [1]
(z) Let £1 = Event of getting a black face card Now, area of square = 2 2 = 4 sq units (112 ]
Then, number of outcomes favourable to £1 = 3
3
: .P(getting a black face card) = P(E) = 49 [1 1

(iz)
2
Let £2 = Event of getting a queen
.u Then, number of outcomes favourable to £2 = 3
>
- : . P( getting a queen) = P(E2 )= _i_ [1 ]
49
,.., Clearly, area of shaded region = area of square
,.., (iiz) Let � = Event of getting a black card - area of quadrant (I)- area of quadrant (II)
J.J Then, number of outcomes, favourable to E; = 23 - area of qudrant (III)- area of quadrant (IV) ( 1 ]
n [·: remaining black cards 7t(14-
)2
n = 1 3 of clubs + 10 of spades = 23] = 4 - 4 x area of a quadrant = 4 - 4 - x

::( Hence, P( getting a black card) = P(E; ) = 23 [·: each quadrant is of equal �a]
49 [1 ] l
.....
[·: area of a quadrant of circle with radius �]
r =
>
-
(iv) Let £4 = Event of getting a spade 4
=
- Then, number of outcomes favourble to £4 10 = 4- 7t (1 )
[·: remaining cards of spade = 1 3 - 3 = IO] Hence, P (that the point chosen from the shaded
Hence, P (getting a spade) = P(E4 ) = 49
10
[1 1 . ) = Area of shaded region 4 - 7t
region [ 1 12)
Area of square 4
Probabi lity f 221 j
72. According to the :. Probability of the green light turning red within the
given information, 1
we have a square next halfa minute =P(E) = 22 = ..!..4 [2]
OABC of side
6 units and a ll.DEB 76. Given, a group consists 1 2 persons.
with side DE = 4 :.Total number of outcomes = 1 2
units and (1) Given, number of extremely patient persons = 3
BE = 4 units. [2] :. Number of favourable outcomes = 3
Clearly, area of
square OABC = 6 2 :. P(excremely patient) = i_ = .!_4
= 3 6 sq units 12
(ii) Given, number of extremely honest persons = 6
and area of ll.DEB = I - x base x height and number of extremely kind persons
2
= 12 - 6 - 3 = 3
= .!_2 x DE x BE = .!_2 x 4 x 4 = 8 sq units


:. Number of favourable outcomes = Number of
Hence, P(darc lands in the triangular region) extremely kind persons + Number of extremely
honest persons = 6 + 3 = 9
= Arca of D.DEB = -8 = -2 :. P {extremely kind or honest) = _2_ = 4
Arca of square OABC 36 9 [2] 12
73. Clearly, number x can be selected in four ways and Value preferred : Honest
corresponding co each such way there are four ways of 77. ( 1) Given, P(using che car) = �
selecting number y.


7
:. Two numbers can be selected in 1 6 ways as listed Probability that Ram will use bicycle for going to
below school = 1 -P(use the car)
(I, 1 ) , (1 , 2), ( 1 , 3), ( 1 , 4), (4, I), (4, 2), (4, 3), (4, 4), (9, I),
(9, 2), (9 3), (9, 4), ( 1 6, 1), ( 1 6, 2), (16, 3) and (16, 4)
•.
= 1 - = _±7 [·: P(E) = 1 - P(E)]
7
:.Number of all possible outcomes = 1 6 [1 %]
Now, the product of xy will be more than 16, if xand y (ii) Shyam should suggest use of bicycle because ic is
are chosen in one of the following ways good for
(9, 2 ), (9, 3) , (9, 4), ( 1 6, 2), ( 1 6, 3) and ( 1 6, 4)
(a) health. (b) saving petrol.
:.Number of favourable outcomes= 6 [1 %]
(c) save the environment from pollution.
78. (t) Number of all possible outcomes = 16
Hence, P(that product will be more than 1 6) = � = �8 (a) Lee E1 be the event ofgetting blood group 0.

[ ]
16
[1 1 Then, number of outcomes favourable to
74. Do same as Q. 24 of N CERT folder 6. 1 at page 2 1 2. E1 = 7
Ans (i) .!..!. (ii) 5
2 :. Required probabilic)r = P(E1 ) = ]__
16
36 36
(b) Do same as part (a). [Ans. 1 /4]
75. Here, the possible outcomes are all numbers between {it) (a) Habit of donation.
0 and 2. The same can be represented using the (b) Social works.
number line {see the given figure). 79. The possible outcomes of the experiment of tossing
two coins are HH, H T, TH and TT.
( 1) Outcomes favourable co Anu are HT and TH.
2 2
0 1 1 3 2

P(Anu getting first chance) = � = .!_


Let E be the event that the green light turning red 4 2
within the next half a minute.
The outcomes favourable to E are points on che Outcomes favourable to Priya is HH.
number line from 2 to 2. :.P(Priya getting first chance) = .!_
4
The distance from 0 to 2 is 2 units while the distance Outcomes favourable to Jyoti is TT.
rrom -3 to 2 1s· -1 umc..
c. [2] :.PQyoti getting first chance) = .!_4
2 2
Since all outcomes are equally likely, we can say that out (ii) No, the number of cases favourable to each one of .
of the total distance of 2 units the distance favourable to them are not equal.
the event E 1s. -1 umt.. (iii) Dishonesty, as she kept two cases favourable for ":,· ; ��
. .. .
·

2 herself and one each for the other two friends. fj ·


• •

Act1v1ty
Topic Covered To get familiar with the idea of probability P(getting one head) = � = ]_
of a n event thro u g h the toss of two coins. 4 2

Pre - knowledge The formula of probability of an event E, =�= .!


2
P(getting one tail)
N um ber of favou rable outcomes to E
4
P(getting two ta ils) = ..!
P (E) =
Total nu mber of possible outcomes
4
Tim e Req u i red 1 5 min
Material Required Two one rupee coins, white paper, a Oral Questions
yellow m arker and a blue marker. 1 . Define elementary events.
Proced u re 2. Define compound events.
1 . Take two coi ns and shade one coi n with yellow marker 3. What is complement of an event?
4. What is the probabil ity of an event that is sure to
a n d a n other coi n with blue marker to distinguish
between fi rst and second coins.
2 . Let blue coin rep resents first coi n and yellow coin
happen?
5. What is the probabil ity of an i m possible event?
rep resents second coin.
3 . Analyse the possible outcomes on tossing two coins 6. If the probability of occu rrence of a n event is 0.9,
then what is the probability of its non-occurrence?

I
si m u ltaneously.
4 . P repare a result sheet as shown below 7 . In a single throw of a d ie, what i s the probability of
getting 3?
Possible
outcomes _
8. Why do we say that the p ro ba bi lity of an event
cannot be negative?
5 . Calculate the p robability of some events associated

I
with the experi ment. True/ False
O bservation 1 . Probability of an event lies between zero and one.
Possible 2. The probability expressed as percentage of a
outcomes _
HH HT TH TT
particular occurrence can never be less than zero.
HH represents head o n both coi ns. 3 . The sum of probabilities of all the outcomes of an
HT rep resents head o n fi rst coin and tail on second experiment is g reater than one.
coi n . 4. For an event E, P(f) = 1 - P(E)
TH represents tai l o n fi rst coin a n d head o n second 5. If a random experiment is perfo rmed, then each of
coin. its outcome is known as favo u ra ble event.
TT rep resents tai l o n both the coins.
From Table
Fill in the Blanks
N u m ber of outcomes favourable for getting two heads 1 . When each outcome of the experiment is as likely
= 1 to occur as the other, then outcomes a re ... .
N u mber of outcomes favourable for getting one head 2. When the sum of probabilities of two events is 1,
=2 the events are called ... of each other.
N u m ber of outcomes favourable for getting one tail = 2 3. On a single roll of a die, the p ro bability of getting a
number 8 is ... .
N u m ber of outcomes favourable for getting two tails
= 1 4. On a single roll of a die, the p robability of getting a

Ca l cu lation number less than 7 is ... .


5. An event having only one outcome is called ... .
P(getti ng two heads) = ..!
4 6. A fair die is rolled. Probability of g etti ng a nu mber
[·.· total nu mb�r of possible outcomes = 4] x such that 1 � x � 6, is ... .
robability f 22 9
7. The pro ba b i l ity expressed as a percentage of a Down
pa rticu l a r occu rre n c e c a n n ever be ... . 2. Probabil ity of a sure event is ... .
8. If P(A) d e n otes the p ro b a b i l ity of an event A, then ... . 3 . Drawing a card from a deck of wel l -shuffled ca rd s i s
9. An eve nt is very u n l i ke l y to h a p pen. Its proba bil ity is cal led ... .
closest to . . . . 5. Probability of an i m possible event is ... .
10. In tossing of a c o i n , th ere is o n ly head and ... . 7 . Number of face cards i n a deck of card s is . . . .
8. Colour of hea rt and d iamond ca rd s is . . . .
Crossword Puzzle Match the Columns
2
8 A card i s drawn from a wel l -sh uffled pack o f c a rd s .
7 Column I Colum n I I
1. (a ) -
4
Probability o f getting a red card.
13
3 1
2. Probability of getting an ace. (b) -

2
(c) 1
6
3. Probability of getting one jack and -

one queen card. 13


4. Probability of getting a number 4 on (d) 1
9
-

the card. 5
Across
1. To measure the deg ree of certa i nty of occurrence of
Project Work
an event is ca l led . . . . To appreciate that f i n d i ng p ro ba b i l ity t h ro u g h

4. G etti ng a 6 i n a th row of a die is an ... .


experi ment is d ifferent from f i n d i ng p ro ba bi l ity by
ca lculation. Stu dents s h o u l d o bs erve the fact that, if
6. The plura l of d i e is . . . . .
tney i ncrease the n u m be r of tri a l s of the experi m e n t,
9. Probabil ity of g etti n g a n even nu mber when a die is proba bil ities fou n d t h ro u g h experiment a p p roaches
thrown, is . . . . the calcu lated p roba b i l ity.

Answers
O ra l Q u esti o n s
1 . An outcome o f a trial i s a n elementary event.
2. A collection of two or more elementary events associated with an experiment is called compound event.
3. Let E be an event, then complement of event E is E, which occurs if and only if E does not occur.
4. One 5. Zero 6. 0. 1
�'== �-
7 . .! 8. Because the probability is always lies between 0 to 1 (0 and 1 inclusive) . �'
,_-., .4
6 �

r�J
True/Fa l se
1 . False 2. True 3. False 4. True 5. False

Fil l in the B l a n ks
1 . equally likely outcomes 2. complementary events 3 zero 4. one
5. elementary event 6. 1 7. greater than 100% 8. 0 � P(A) � 1
9. zero 1 0. tail

Crossword P uzzl e
Across
1 . probability 4. event 6. dice 9. h alf
D own
2. one 3. trial 5. zero 7. twelve
8. red

Match the Co l u m n s
1 . (b) 2. (c) 3 . (a) 4. (c)
S U M MARY

• An event for a n experi ment is the collection of some outcomes of the experiment. e . g . Getting head i n the toss of a coin is
a n event.
• An event E associated to a random experiment is said to occur (or happen) in a trial, if the outcome of trial is one of the
o utcomes that favou rs E.
• P robability is the study of the chances (or likelihood) of events happening.
• An event having only one outcome of the random experiment is called an elementary event.
• A collection of two o r more elementary events associated with an experiment is called compound event.
• Equally li kely outcomes a re those outcomes that have the same probability of occurrin g .
• I f E is a n event t h e n i t s complement defined as event E (or E' o r Ee ), which occurs i f and only i f E d o e s n ot occur. Here, the
events E a n d E a re ca lled complementary events.
• The p robabil ity of a n impossible event is 0.
• The p roba bility of a s u re event (or certain event) is 1 .
• The e m pi rical (or experimental) probability of an event E is given by
p (E) =
Number of times event E occurred
N umber of times the experiment was performed
• I f all the outcomes of an exr:"eriment are equally likely and E is an event associated with the experiment, then the
theoretica l probability (or classical probability) of the event E is given by
P (E) =
Number of favourable outcomes
Total number of outcomes
• The p robability of happening of an event always lies between 0 to 1 (0 and 1 inclusive), i . e . O � P (E) � 1. Also, in
percentage, it lies between 0 % to 1 00% (0 and 1 00 inclusive) .
• The s u m of the probabilities of complementary events of an experiment is 1 , i.e. P (E) + P (E) = 1
where , P (E) represents the occurrence of an event E and P (E) represents the non-occurrence of a n event E.
• P robabil ity of a n event can never be negative.
• The s u m of the p robabilities of all the elementary events of an experiment is 1 .
• If m outcomes a re favo u ra ble to an event E out of n possible outcomes, then the number of outcomes which ensure the
non-occu rren ce of event E is n m . Then,
-

Probability of the happening of the event E,


=
Number of favourable outcomes m
P (E)
Total number of all possible outcomes n

Probability of the n on-happening of the event E, P (E) or P(not E) =


n - m

n
• The odds i n favour of the occu rrence of the event E are defined by m : n - m,i . e. P (E) : P (E).
• The odds i n against of the occurrence of the event E are defined by n m - : m, i .e. P (E) : P (E).
CHAPTE R EXE RCIS E
� I Very Short A nswer Type Questions [ 1 Mark e a c h ]

1 . In a throw of a d ie, find the probability of getting an odd 7. Find the probability o f get ting a perfect s q u a re
number less than 6 . CCE 2013 number from the numbers l to 1 0. CBSE 20 1 2
2. A student says that, i f you throw a die, it will show up l 8. I n a leap year, find the probability that t here are
or not l . Therefore, the probability of getting not l each 5 3 Sundays in a year.
. I . .
is equa l to -. I s t h is correct? Give reasons. 9. If in a lottery, there are 5 prizes and 2 0 blanks,
2 NCERT Exemplar then find the probability o f getting a prize.
3. In a throw of a pair of dice, wha t is the probability of 1 0. What is the probability o f getting exactly two tails,
getting a doublet? CCE 201 3, 12, 1 1 when two coins are tossed together? CCE 20 1 3
Or
1 1. I f P(E) = 0. 1 5, then find P( not E ) . CCE 201 3, 1 2
Two dice are thrown together. Find the probability of
getting the s a me nu mber on both die. CBSE 2009 1 2. I f I toss a coin 3 times and get head each time,
Or then I should expect a tail to have a higher chance
in the 4th tos s. Is it true? NCERT Exemplar
A pair of dice is thrown once. Find the probability of
getting the same number on each die. CBSE 2008 1 3. In a family having three children, there may be no
4. A box contains 90 d iscs, numbered from l to 90. If one
girl, one girl, two girls or three girls. S o, t he
disc is drawn a t random from the box, find the probability of each is ..!... . I s it true?
probability that it bears a prime number less than 23. 4 NCERT Exemplar
CCE 201 3 1 4. In a family of 3 children, find the probability o f
5. A bag contains 4 red, 7 white and 2 yellow balls. If a ball
having atlea st o n e boy. CCE 20 1 4
is drawn from it, then find the probability that it is not 1 5. A girl calculates that the proba b ility o f her
red. winning the first prize in a lottery is 0.06. If 6000
6 . Find t he probability that a number selected at random
tickets are sold, then how many t ickets has she
from the numbers l , 2, 3, ... , 1 5 is a multiple of 4. bought?
CBSE 2014

� I Short Answer Type I Que�tions [2 M a rks e a c h ]

1 6. A ticket is d rawn at random from a bag containing 1 9. A bag contains 6 red, 3 black and 6 white b a ll s . A
tickets numbered from l to 40. Find the probability that ball is selected at random from the bag. Find t he
the selected t icket has a number, ( i) which is a multiple probability that the selected ball is
of 7 ( ii ) which is a multiple of 5 . ( i) red or black.
CBSE 201 1; NCERT Exemplar
( ii ) not black. CBSE 20 1 2
1 7. Cards marked with numbers 5, 6, 7, ... , 74 are placed in a
bag and mixed t horoughly. One card is drawn at random 20 . A c c¥n i s tossed 3 t imes. Find t h e probability o f
getting
from the bag. Find the probability that the number on
the card is a perfect cube. ( i ) all head s . ( ii ) a tlea s t 2 hea d s .

1 8. A box contains 3 blue, 2 white and 4 red marbles. If a


CCE 201 4; NCERT Exemplar
marble is drawn at random from the box, then what is 21. Two dice are rolled once. Find t he pro b a b ility o f
the probability that it will be getting such numbers on two dice, whose product
( i ) white. ( ii) blue. is a perfect square.
� 23 2 1 AlllnOne MATH EMATICS C lass 1 0th Term I

2 2 . 1\vo di fferent d ice are tossed together. Find the 2 6 . Five cards, ten, jack, queen, k in g and an ace of
probability diamond are shu ffled with their face downwards.
( i ) t h a t the number o n each die is even . One card is picked at ra ndom. Whil t i s the probability
tha t
( i ) the card is ten o r j ack? ( ii ) t h a t the card is king?
( ii ) t h a t t h e s u m of numbers appearing on the two
d ice i s 5 . CCE 2014
2 7 . 1\.vo dice are thrown simu l t a neou sly. What is the
2 3 . F ro m a well-shuffled pack of cards, a card is drawn at probability that
random. Find the probability of getting a black
( i ) 5 will not come up on a t lca st o n e d ie?
queen.
( ii ) 5 will come u p on b o t h d ice?
2 4 . A letter is chosen a t random from the letters of the
2 8. A box contains 80 discs numbered from I to 80. If one
disc is drawn at random from the box, find the
word 'AS SAS SINATION' . Find the probability that
the letter chosen is a vowel.
probability that it bears
2 5 . A letter of English alphabet is chosen at random. (i) a perfect square n u m b e r.
What is the probability that it is a letter of the word ( ii ) a number divisible by 2 a n d 3 . CBSE 201 1
'MATHEMATIC S' . NCERT Exemplar

� I Short Answer Type I I Questions [ 3 Marks each]

2 9 . A card is drawn from a well - shuffled pack of 52 cards. ( i ) a blue card .


Find the probability of getting ( ii ) not a yellow card .
( i ) a red face card . ( iii ) neither yellow nor a b l u e c a rd . CBSE 2009
( ii ) a b lack k in g . 3 7 . A box contains 1 2 balls out of which x are black. If
( iii ) either a spade o r a n ace. CCE .2012, 10 one ball is drawn from the box, what is the
probability that it will be a black ball. I f 12 more b la ck
3 0. A game consists of tossing a coin 3 times and noting balls are put in the box, the probability of drawing a
its o u tcome each time. H anif wins, if he gets three
black ball now is double of wh a t it was before. Find
the value of x.
head s or three tails and losses otherwise. Calculate
the probability that Hanif will win the game.
3 8. Anita, Sita, Gita and Rita a re four friends having
31. A number is selected at random from the numbers same birthday. Wha t is the proba bility that ( in a
3, 5, 5 , 7, 7, 7, 9, 9, 9, 9. Find the probability that the non-leap year)
selected number is their average. CBSE 2012 (i) their birthdays fal l in the m o n t h of October?
32. A number x is chosen from the numbers l , 2, 3 and a ( ii) their birthdays fal l on I 0 t h d a y of the months?
n umber y is selected from the numbers l , 4, 9, 6. Find
the probability that xy = 1 0 .
( iii ) their birthdays fal l in J a n u a ry o r February?
CCE 201 3

33. A number is chosen at random among the first 3 9. I n a game of musical chair, t he person playing music
1 20 na tural n umbers . Find the probability of the has been advised to s top playing the music at any
number chosen being a multiple of 1 1 or 1 5 .· time within 1 min after she starts playing. What is the
probability that the music will s top within the first
3 4 . Three u nbiased coins are tossed together. What is the 20 s after starting?
probability of getting
40. A traffic signal displays green light for 3 min t o allow
( i ) two tails? passage of traffic on a particu lar road . If the signal is
( ii ) a tlea s t two head s ? CBSE 201 1 currently displaying green light, then find the
probability that it will turn red within the next half a
3 5 . Two dice are thrown a t the same time. Determine the minute.
41. From the numbers l , 2, 3, one nu mber x is select ed
probability that the difference of the numbers on the
and from the numbers l, 4 , 9, second number y is
two dice is 2 . NCERT Exemplar

3 6 . A box con tains 1 00 red card s, 200 yellow cards and selected. Find the probability that the
50 blu e card s . If a card is drawn at random from the (i) prod uct of two numbers i s more than 1 3 .
box, then fin d the probability that it will be ( ii) sum o f two numbers i s 1 2.
Probability f 233 f

�I Long Answer Type Questions [4 M a rks e a c h ]

42 . A box con t a i n s 1 9 balls bearing numbers ( ii ) will not be a � 5 coin?


l, 2, 3, . . . , 1 9. A ball is d rawn at random from the ( iii ) will be eit her a 5 0 p or a � 2 coin?
box . Wha t is t he probability that the number on the CBSE 20 1 0

4 6 . A number x i s selected a t random from the


ball is
( i ) a pri m e n u m b e r? ( ii ) divisible by 3 or 5? numbers 1 , 2, 3 and 4. Another number y is
( iii ) nei t h er d iv i s i b l e by 5 nor by 1 0? selected at random from the numbers 1, 4, 9 and
( iv ) a n eve n n u m b e r? 1 6. Find the probability tha t product o f x a n d y is
less than 1 6.
43.
CCE 20 1 6
A box con t a i n s card s bearing nu mbers from 6 to 70.
If one card is d rawn at ra ndom from the box, find 4 7. A game consists of spinning a n
the proba bility t h a t it bears CCE 201 5 arrow which comes to res t
( i ) a o n e - d i g i t n u m ber. pointing at one of the regions ( 1 ,
2 o r 3 ) . Are the ou tcomes 1 , 2 a n d
( ii ) a nu m b e r d i visible by 5 . 3 equally likely to occur? G ive
( iii ) a n o d d n u mber less t h a n 30. reasons. NCERT Exemplar
( iv ) a com pos i t e n u mber be tween 50 and 70.
44 . A bag con ta in s 40 balls out of which some are red, 4 8. In the given figure, a dart is thrown and lands i n
some arc blue a n d remaining are black. If the the interior o f the circle. What is the proba bility
probability of d rawing a red ball is _!_!_ and that of that the dart will land in the shaded region?
20
blue ball is .!., t hen fi nd the number of black balls.
5 NCERT Exemplar
45 . A piggy b a n k con tains hundred 50 p coins, seventy
� 1 coins, fifty � 2 coins and thirty � 5 coins. If it is
equally likely t h a t one of the coins will fall out when
t he b an k is tu rned upside down, what is the
probab ility that t he coin
( i ) will be a � l coin?

�I Value Based Questions (VBQs)


4 9 . Videocon Electronics has launched two new mobile ( i ) Find the probability of n umber of people
hand set s : Set I and Set I I . Set I is cheaper as ( a ) u sed car and scooter o n ly.
compared to Set I I . But Set II has built-in device to ( b ) used cycle only.
recharge the battery with auto-cut power supply
( c ) used a tleas t one k i n d o f m o d e s o f
when it is fully charged. In a lot, there are 250 pieces
of Set I and 1 00 pieces of Set I I . If a mobile is picked transport .
at random, then ( ii ) Which value wou ld you learn fro m a b ove
( i ) fin d t h e prob a b ility of getting Set I . data?
( ii ) fin d t he probability o f getting Set I I . 5 1 . There is a group of 80 people who are pat riotic, 40
( iii ) i f y o u a re actively participa ting i n Save people believe in violence. What is the probabil ity
E nvironment campaign, which mobile set of people who believe in non-violence? Which
you wou ld prefer and why? value you will develop i n your character?

50 . For travelling different mode of transport is used by 5 2 . In answering a question of MCQ Tes t with
500 people are as follows :
Mode of transport Number of people
4 choices per question, one of them being correct,
a student knows the answer, guesses or copies the �
F�
answer.
Car 40
(i) What is the probability that his a nswer
!'! ?'
Scooter 1 00 is correct, if Shivam does not know the
Bus 1 60 answer to one of the questions i n the
Train 70 test? �
�a:;
Cycle 110 li2!S>!..,'.
(ii) Which value would S hivam violate , if �I
No mode of transport 20
he copies the answer? (h�
AllinOne MATH EMATICS C lass 1 0th Term I

Answers
1
1. 2. No, because the outcomes (1) and (not 1) are not equally likely
2

3. - 4. _i_
1 1
5. 6.
6 45 13 5
8. �
1
9 . .!
3 1 0.
7.
10 7 5 4
1 3. False 1 4.
3


1 1 . 0 . 85 1 2. False
4
1 6. (i) .! (ii) .! 1 7. � 1 8. (i) (ii)
2
1 5. 360 -
9
1
8 5 70
(u ) -
" '

1 9. (i) � (ii ) �
' 1
20. (i) .! (ii) .! 21 . � 22. ( z. ) -
5 5 8 2 9 4 9
24. � 26. (i) - (ii) .!
1 4 2
23. 25.
26 13 13 5 5
30 . -
;'\
27. (i) (ii) _..!_ 2 8 . (i) _!__ (ii) 29 (i ) - ( ii ) ( m, -
25 13 . 3 .. 1 . . 4 1
• -

33. �
36 36 10 80 26 26 13 4
31 . � 32. 0 34. (i) � (ii) .!
;'\ 3 1 " ";'\ 59
10
36
2
( ( ui,
20 8
x
35. � ( z. ) - ( z.z. ) - ( u z. ) - 37. = 4 38 l1 -
" 1 ." 3 ..' 2 "' 12
• -, x • - ( H) -

9
9 7 7 7 12 365 365 365
;'\ . . ;'\
40 . .! 41 . (i) � (ii) .! 42 ( i) - ( u, - ( iii, - (iv) -
1 8 8 . 16 .
.
19
3 9. •

3 6 9 9 19 19 19
(i) .!__ (ii) (iii) �
" 4 . " 1 . . " 12 . 3 22 1
65 z.z. 5 m 65 (w) 1 3
43 ( ) ( ) ( ) 44. 10 45. 46.
• z. 25 25 5 2
47. No, the outcomes are not equally likely. The outcome '3' is more likely to occur than the others.
2 57t - 4 8
48. ---

� �
2 57t

49. (z.) (ii) (iii) Prefer Set II because it will save energy and leads to less green house effect.

50. (i) (a) 2 (b) .!.! (c) (ii) Maximum people using public transport. So, we should use public transport.
24
25 50 25

5 1 . .! ; Non-violence


2

52. ( i) (iz.) hone s ty

(iii) Cheating may get him marks in this test but this habit may not let hime develop an integrity of character in the long
run.
CHAL LE N G E RS*
•••i.!
NE:
Q!:!
· -�
·- :_.:: :::-.. -- · ·---� �--� -_- - _-.---=-����:.....:.... _!.:;:
.M* rl
*••••••••••••••••+
m1w;
s:J!
·WDM1
RDim"
:l
'Z
'�
{
·�
'3!2IT
�·�·c:
-
··
::.
.::
·-=
. -
·� -
··-= -

1 . (1) If odds i n favou r of an event is 2 : 3, then find the probability of non-occurrence of this event.
(it) If odds i n against of an event is 3 : 4, then find the probability of occurrence of this event .
2. A graph is shown below. The intersection points of two lines are said to be the integer points.

:g
y
4
rj rjE (4,ir3)
__

D : : :

3 - - - - - -- ----·· -··· ··-- ----- ----

2 - -- - - - - ·····-· 1 ··--·-· f · · - - -- - 1 ------- ----

1 ......A i i 1 B
----

5
--�--�+-----�--�- x
0 (0, 0)
----t �

2 3 4
Find the probability of E with respect to the integer points in the rectangle ABCD.

3. Two dice are n umbered 1 , 2, 3, 4, 5, 6 and 1 , 1 , 2, 2, 3, 3, respectively. They are thrown and the sum of the n umbers on them i s
noted. F i n d t h e probability of getting each sum from 2 t o 9, separately.

4. At a fete, cards bearin g n umbers 1 to 1 000, one number on one card, are put in a box. Each player selects one card at -�ndom
and that card is not replaced . I f the selected card has a perfect square number greater than 500 , the player wins a p rize. What i s
the probabil ity that
(1) the first player wins a prize?
(it) the secon d player wins a prize, if the first has won?

5. A box contains 54 marbles each of which is blue, green or white. The probability of selecting a blue marble at random from t h e

box is i �
a n d t h e probabi l ity of selecting a green marble at random is · How many white marbles does t h e box contain?

6 . On the d isc s hown below, a player spins the arrow twice. The fraction � is formed, where a is the number of the sectors , where

the arrow stops after the first spin and b is the number of sectors where the arrow stops after the second s pi n . On every s p i n ,
each o f t h e n umbered sector has a n equal probability o f being the sector on which t h e arrow stops. What i s the probabil ity that
the fraction � is greater than 1 ?
b

• These questions m ay be or m ay not be ashed in the examination, have been g iven just for a dd itio n a l p ra ctice.
Note Solutions to t h es e q uestions are available at page number 390-391.
C H A PT E R 7
� ' n •• I f ...
I
' j& ' • .

COO RD I NATE G EO M ETRY

In this chapter, first of all, we will review the concepts of coord i nate geometry that w e
learnt in Class IX . Further, i n this chapter, we will learn how c o find che distance
between two points whose coordinates are given and che coordinates of the po in t
Review of Concepts of which divides a line segment joining two given points in a given ratio. We will also
Coord i nate Geometry study to find the area of a triangle formed by three given points.
., Cartesian System
.. Coord i nates of a Point i n
Cartesian Plane TO PIC O Review of Concepts of
Coordinate Geometry
• Sign Conventions of
Coordinates in Various
Q uadrants
D istance Formula Coordinate geometry is the branch of Mathematics which deals with the position of
0 Distance between any Two an object lying in a plane, described with the help of two perpendi cular lines.
Points i n a Cartesian Plane
Section Form u la C a rtesia n System
0 I nternal D ivision of a Lin e The system used to describe the position of a point 10 a plane, IS called
Segment cartesian system.
Area of a Triangle
In cartesian system, there are two perpendicular y
., Area of Quadrilateral straight lines XX' and YY' which intersect each 3
2 --
° Col l i nearity of Three Points other at point 0. The point of intersection of these
lines is called origin and it is denoted by 0. In other
goo
words, the point from which distances are measured,
-2 2
is called origin. X'
-3 -1 0 3
The horizontal line XOX ' is called X-axis. The -1
vertical line YOY ' is called Y-axis. Directions OX -2
and 0 Y are called the positive directions of X and
-3
Y-axes, respectively. Similarly, directions OX ' and
Y'
0 Y ' are called the negative directions of X and
Y-axes, respectively.

N o te • The angle between the horizontal and vertical axes is 90°.


• Any l i ne perpendicular to the X-axis is parallel to the Y-ax i s .
• A n y l i ne perpendicular t o t h e Y-axis is parallel t o the X-a x i s .
Coordinate Geometry f 231 I
Quad ra nt :. The point is P(x-coordinate, y-coordinate) o r P(x, y) .
y
The intersection of X a n d Y-axes divides the plane into Perpendicular distance

/
four parts. Each part is called quadrant and these from Y-axis
quadrant are numbered I , I I , I I I and IVth in (x-coordinate, y-coordinate)
anti-clockwise directio n from OX. Thus, the plane N -------------
�p
consists of the axes and four quadrants, is known as l Perpendicular distance
XY-plane or cartesian plan e or coordinate plane and the y-coordinate :-
I
from X-axis
1
I
I
axes are known as coordinate axes. I
I

y O ---- I M
X' -----+--L-- X

II
x-coord i nate
Quadrant Quadrant Y'
Note The order of x and y is imporcanc in (x. y). Posicion o f (y, x )
x· x wi ll b e differenc from (x, y).
0
Ill
Quadrant
IV
Quadrant
Important Points
I . Every poi nt on the X-axis has no perpendicular d i s ta n ce
from the X-axis, so i ts ordinate is 0. Thus. the
Y'
coordi nates of every poi nt on the X-axis a re o f the form
(t) XOY is called the I quadrant. (x. 0) .
2.
{it) X'OY is called the I I quadrant. Every point on the Y-axis has no perpend icular d i s tance
from the Y-axis, so i ts abscissa i s 0 . Thus, the
(iit) X'OY' is called the I I I quadrant. coordinates of every point on the Y-axis a re of the for m
(0, y) .
(iv} XOY' is called the IV quadrant.
3 . The coordinates of a point at origin is (0, 0) . because it

Coord inates of a Point


has zero discance from both the axes. So , i ts abscissa
and ordinate both are zero.
in C a rtes ia n Pla n e
To locate a point in the cartesian plane, its perpendicular
Example 1. In the given figure, O is � L-----.. 8
the intersecting point of OA and QC �
distances from X and Y are required. These distances

A
and OABC is a square of side �-
are called coordinates of the point. Each point in 4 units. Find the position of A , B en
cartesian plane has two coordinates; x-coordinate and
O
and C. X
4 units
y-coordinate.
(t) The x-coordinate of a point is its perpendicular
Sol. Here, che poinc A lies on side OA, which is ac a discance of uni cs 4
from che incerseccion poinc 0 of che lines OA and OC, so ics
distance from the Y-axis measured along the posicion will be ( 4, 0) and poinc C lies on che side OC ac a
X-axis (positive along the positive direction of discance of 4 unics from che incerseccion poinc 0, so its position
the X-axis and negative along the negative direction will be (O, 4). Point B lies 4 units away from both the sides
of X-axis) . The x-coordinate is also called the (or axes), so ics position will be ( 4, 4 ).
abscissa.
(iz) The y-coordinate of a point is its perpendicular Sig n Conventions of Coord inates
distance from the X-axis measured along the in Various Quad ra nts
Y-axis (positive along the positive direction of
Some useful rules to determine the signs ofcoordinates ofa point in
the Y-axis and negative along the negative direction
aquadrant are given below.
of Y-axis) . The y-coordinate is also called the
ordinate. (t) If a point is in the I quadrant, then the sign of the
Coordinates x and y taken together are called coordinates of a point will be in the fo rm ( +, +). As the
cartesian coordinates or coordinates of a point and I quadrant is enclosed by the positive X-axis and the
denoted by (x, y) . Here, (x, y) is called an order pair. positive Y-axis.
(ii) If a point is in the II quadrant, then the sign of
.
Let PM ..l OX and PN ..l OY
the
coordinates of a point will be in the form ( -, +). As the
:. PN = OM = x-coordinate II quadrant is enclosed by the negative X-axis and the
and ON = PM =y-coordin ate positive Y-axis.
\ 238 1 Alli'none MATH EMATI CS Class 10th Term "

(iit) If a point is in the III quadrant, then the sign of the For point R
coo rdinates of a point will be in the form (-, -) . As the III. (1) Abscissa of the point = - 5
quadrant is enclosed by the negative X-axis and the (ii) Ordinate of the point = - 3
negative Y-axis. (iii) Coordinates of the point = (-5, -3)
(iv) If a point is in the IV quadrant, then the sign of the (iv) Point of the type (-, -) lies in III quadrant, hence
coordinates of the point will be in the form (+, -) . As the the poinc (-5, -3) lies in I I I quadrant.
IV quadrant is enclosed by the positive X-axis and the For point S
·

negative Y-axis (see below figure) . (1) Abscissa of the point = 5


y (ii) Ordinate of the point = - 1
4 Coordinates of the point = (5, - 1 )
II
(iii)
3
Quadrant
(iv) Paine of the cypc ( +, -) lies in I V quadrant, hen�
Quadrant
(-, + )
2
( + , +)
the point (5, - I ) lies in IV quadrant.
0
X ' ---+--+---+--+-�+---+---+-+--1-- X
-4 -3 -2 -1 2 3 4 TRY YOURSELF
-1
Ill -2 IV Q. 1 I n the given figure, 0 i s the intersection point of OP and
Quadrant -3 Quadrant OR and OPQR is a square of side 6 units. Find the
(- , -) (+ , -) positions of P, Q and R.
-4 Y'
y
Tabular form ofsign conventions ofcoordinates in various
quadrant are given as Q R
Region Quadrant Nature of Sign of Sign of Coordinates
x- coordinate y- coordinate
6 units
X and of a point
x
p 0
Y-axes
__
I
x•- ___..
_ __

XOY x > 0, y > O + + (+ , +)


-
Y'
X •OY II X < O, y > O (-, +)
Ill
+
X'OY' X < 0, y < 0 - - (- , -) [Ans. Position of P = (-6 , 0), position of Q = (-6, 6),
XOY' IV x > 0, y < 0 + - (+, -) position of R = (0, �

Q. 2 Write the abscissa and ordinate of the points Pand Q.


Examp le 2. Write down the (i) abscissa (ii) ordinate
(i)
(iii) coordinates for the poin ts P, Q, R and S (iv) quadrant 4 y
in which these poin ts lie. 3

1i �
y p
Q ,- - - - - 4
-2 - 2

--------, p x·--------------1----11--1--+-+X
-2 -
2 3 4
:t 1
-4 -3 -2 -1 0
-1
Q
l3
ll
X' --+---+--+--4--+----+�
!!
0--+--+--+--t--1,-.
, X
-2
-3
-5 1-4
-==:--5 -� s
-3 -2 -1 2 3 4 5:
Y'
1 -4
i3 --21
R
:i !
._ _ _ _ _ _ _ _ _ _ _ _ _ _ _ _
-3 (ii) y
-5 - Y' p 4
3
Sol. For point P 2
(t} Abscissa of the point = 2 => x = 2 1
(it} Ordinate of the point = 3 => y = 3 x
0
-4 -3 -2 -1
X'
(iit} Coordinates of the point = ( 2, 3) -1
1 2 4
(iv) Point of the type (+, +) lies in I quadrant, hence the point -2 Q
(2, 3) lies in I quadrant. -3
-4
For point Q Y'

(t} Abscissa of the point = - 2


(it} Ordinate of the poinc = 4 [Ans. (1) For point P, abscissa = - 2, ordinate = 2 and
(iit) Coordinates of the poinc = ( -2, 4) for point Q, abscissa = 2 , ordina�

(it) For point P, abscissa = - I , ordinate = 4 and


= -2
(iv) Point of the cype (-, +) lies in II quadrant, hence the poinc
( -2, 4) lies in II quadrant. for point Q, abscissa = 3, ordinate = - 2]
EXERCIS E
� - �- - -
-----="�™-- : �-- . - :

y
1 In the figure given below, ABCD is a rectangle Sol. According co the question, B---- 4 A
with length 6 cm and breadth 3 cm, 0 is the The figure would be given as 3
mid-point of AB. Find the coordinates of A, B,C and D. Coordinate of point 0 = (0, 0) �
y Coordinate of point A = (O, 4 ) x· ....
c_.._
._ ...._.___._
. ______ x
E - 6 - 5 -4 -3 -2 - 1 1 °
D c Coordinate of point B = ( -6, 4 ) 2
1 Coordinate of point C = ( -6, 0) 3
3 cm Y'
l 4 Plot the following points, join them in order a n d
A B identify the figure thus obtaine d .
P(3 , - 2) ; 0 (3, 2) ; R(-3, 2) ; S {- 3, - 2)
6 cm
Y' Write the coordinate of the point of intersection of
Sol Given, 0 is the mid-point of AB and 0 is the origin (from figure). diagonals. -
y
:. OA = OB = � cm == 3cm Given points are 3
Sol.
2 p = (3, -2), R (-3. 2) Q ( 3 , 2)
Similarly, DE = EC = 3cm [·: DC = AB = 6cm] 2 ._ ___

Q = (3 , 2), R = (-3, 2), ... ...


s = (-3. - 2). X-----+--- ... _._ q ... ...----<
.� �
... ... ..-""" ... ... ...--- ...
2 "------ X
:. Coordinates of A = (-3, O)
Coordinates of B = (3, 0) , Coordinates of C = (3, 3) -3 ... 1 3
..... .... ..... ..... ....
and coordinates of D == ( -3, 3) Placing these points we
have che figure chus (-3 . _2J 5 -3 p (3 , 2)
2 From the given graph, write
-

obtained is a rectangle.
Y'
(i) the coordinates of the points B and F. :. Coordinate of point of
(ii) the abscissa of points D and H. intersection of diagonals = (0, O )
(i) Plot the points M (5, - 3) and N ( 3 3)
(iii) the ordinate of the p oints A and C.
5
the perpendicular distance of the point G from the
- , - .
(iv)
y
X-axis. (ii) What is the length of MN?
7 (iii) Find the coordinates of points A, B and C lying o n
MN such that , MA A B B C = CN.
= =
5
6

4 H •
0G Sol. (1) Given, coordinates of point M = (5,-3 ), it mea:n 5 units
3 distance from Y -axis and 3 units distance from X-axis i n
2 IVth quadrant. And coordinates ofpoint N = (- 3 ,-3 ), it
0A c 00 F
means 3 units distance from Y-axis and 3 units distance
from X-axis is IIIrd quadrants .
x
-7 -6 -5 -4 -3 -2 - 1 0 1 2 3 4 5 6 7 4 y
x· ·

-1 3
-2 2

E0 -4-3 1
0B -5
-6
-7
Y'
Sol. From che graph,
(t) Coordinates of B = ( 5 - 4), Coordinates ofF = (6, O)
- , (iz) Length of MN
(it) The abscissa of point D = 1, The abscissa of point H = 0 = Perpendicular distance of M fro m negative Y-a..xis
(iit) The ordinate of po inc A = 1, The ordinate ofpoint C = 0 + Perpendicular discancc of N from n ega tive Y-axis
(iv) The perpendicular distance of the point G from X-axis = Abscissa of M + Abscissa of N (w i rhout sign) = 5 + 3
= y-coordinate of G = 4 = 8 units
3 Write the coordinate of the vertices of a rectangle (iiz) Here, points A , B a n d C divide MN i n to 4 equal parts o f
whose length and breadth are 6 and 4 units, length �, i.e. 2 u n its.
respectively. Its one vertex is at the origin. The 4
longer side is on the X-axis and one of the vertices :. Coordi nares of A = (3, - 3 ). Coord i narcs of B = (1, -3 )
lies in second quadrant. and coordinates o f C = ( - 1 , - 3 )
Alllnone MATH EMATICS C lass 1 0th Term II ·

[Type II ) Problems Based on Equ idistant Points Example 6. Do the points (3, 2 ) , (-2 , - 3) and (2,
form a triangle? If so, name the type of triangle
3)
To find a point say X which is equidistant from two points
say A and B, we take AX = BX and then simplify. formed.

Note A p o i n t P equidista n t from two points say A and B. lies on


Sol. Let the given points be A(3,2), B(-2,-3) and C(2,3)
the perpendicular b i sector of A B . Here, AB = � -3 )2 + (-3 -2 2
(-2 )
2
Example 5. If the point P(x, y) is equidistant from the - x1 )
[·: distance= �(x2 + (y2 -y. )2 ]
points A (5 , 1 ) and B ( l , 5 ), then prove that x = y. = �(-5 ) + (-5 )2 = .J2 5 + 2 5 = .J50
2
Sol. Given point P(x, y) is equidistant from the points A(5, l)and = 7.07 units (approx.)
)2
B(l,5). BC = � + 2 )2 + (3 + 3
(2
=> [on squaring both sides]
;( -5)2AP2+ (y=-BP21 )2 = (x-1)2 + (y-5)2 = �(4)2 + (6)2 = .J16 + 36 = .Js2

= 7.21 units (approx.)
[ ·.· distance = ��(x2 x1-)2 + (y2 --y1--)� ]
____ ____
_

.:_
and = �(3 2 )2 + )2 = .J(l)2 + (- 1)2 = .J1+i
CA (2 -3
=> -1 -
x2 + 25 Ox + y2 + 1 2y = x2 + 1 -2x + y2 + 25 -10y
=> -lOx + 2x = -10y + 2y Hence proved. = .J2 =1.41 (approx.)
=> -Bx = y -8 => =
x y Gisidvesenispoigreater
nts A,than
BandthiCwill
r d form a triangle, ifsum of
side. any two

(Type m J Problems Based on Here,


and CA
AB++BCAB CABC ; BC + CA AB
>
>
>

Geometrical Fig u re So, given points form a triangle. )2 )2


Sometimes, three or four points are given and we have to Also, (.Jsi. )2 = (.J50 + · (.J2.
show that the given points form given geometrical figure or ==> (BC )2 =(AB )2 +(CA )2
not. To show that a given figu.re is a So, converse of Pythagoras theorem, = 90°
by LA
(z) triangle, prove that sum of lengths of any two sides is Hence, llBAC is a right angled triangle.
Example 7. Show that the points (2 , 3 ) , (3, 4 ) , (5, 6)
greater than the length of third side.
(iz) isosceles triangle, prove that two sides are equal.
and (4, 5) are the vertices of a parallelogram .
(iii) right angled triangle, prove that sides oftriangle satisfy
Pythagoras theorem. Sol. l..etA(2,3), B(3, 4), C(5, 6)andD(4, 5)bethegivenpoints.
(iv) equilateral triangle, prove that all three sides are equal. Then, by�distance formula .J
(v) parallelogram, prove that the opposite sides are equal. AB = (3 - 2)2 + ( 4 -3 )2 = (l)2 + (1 )2 ..fi. units =

(vt) rectangle, prove that o pposite sides are equal and the
·

BC=�(S-3)2 +(6-4)2 = �(2)2 +(2)2 = .J4+4
diagonals are also equal. = Fa = 2.fi units
(viz) parallelogram b!-Jt not a rectangle, prove that its
opposite sides are equal but the diagonals are not equal.
CD = �(4-5)2 + (5 -6)2 = �(- 1 )2 + (-1)2 = .J2 units
(viii} square, prove that the four sides are equal and the and AD = �(4-2)2.JS+ (5 -3)2 = .J(2)2 + (2)2
·
. diagon.als are also equal. = .J4 + 4 = = 2../2 units
(ix) rhombus, prove that the four sides are equal. Thus, ABopposi
=CDteandsides are=equal.
AD BC
(x) rhombus but not a square, prove that its all sides are Hence, the
So, it is a paralle..Jlo�gram.
equal but the diagonals are not equal.
Calculate the length of different sides of a figure by using
Also, AC = (5:...-_;_... 2-)2_+_(_6_--3-)2
distance fo rmula and then show with the help of above = �- =.J9+9 = 3J2 units
results . and BD = �(4-3)2 + (5 - 4)2
N ote To show that given quadrilateral ABCD (say} is a square, we = �(1)2 + (1)2= -J1+i = .f2 units
also use the following method, firstly show that all four
sides say AB, BC, CA and AD are equal and then find diagonal
A C. Now. show that AB2 + BC2 = AC2 • Then, by converse of
So,Hence,the AC -:;; BDare not equal.
diagonals
given vertices represent a parallelogram.
Pythagoras theorem . LB = 90° Thus, given quadrilateral have
four sides equal and each a ngle of 90°, so it is a square. Hence proved.
Coordinate Geometry f 243

TAY YOU RSELF


Q. 1 Show that points ( 1 , 2) , (3, 4) , (5, 6 ) are collinear.
Q. 2 Show that the points (a, a), (- a, - a) and (- ../3a., ../3a.) are vertices of an equilateral triangle.

Q. 3 Which point on X-axis is equidistant from (5, 9) and ( - 4, 6) ? [Ans. (3, O)]

Q. 4 Show that the points A (1, 2 ), 8(5, 4), C (3, S) and D(-1, 6) are the vertices of a square.

Q. 5 If the point (x, y) is equidistant from the points (a, b) and (b, a) = 1. Prove that x = y.

NCE RT FO- ·- �;4·L��- -TD E R 7.1 .


.

� .::�
L; . ·�- . _; . �- - . - ···. . --_ · · · : -:ip)' •l1 Yi_ar.. 'l /1·· :.,.
. '. · , - ,, , ) ·....!· �, ',,.,·•·." "· , � , ' ,. ,
,
. , •. ·... .. ,·, 1 , 1#. � :1> . · · ··"-' ·
� -,. _, .rio .... .r.. � �· ' � •..,
. �:l·l � "1 'M_�\·
., ,.-.., •... �_,
;-i .r.�"."!k
,J;.. �:· ::, ..
.
r - -- ·· -
••
' 1 � '. ...,.., .
·
. .
· .•

· · · ··· - · ·
· • · . · ·� .. � . �'
.

-·-----�
.

1 Find the distance b etween the following pairs = �(1)2 + (- 2 ) 2 = .Jl+4 = JS = 2.236 units
of points. BC = �(- 2 - 2)i + (-1 1 - 3)i
(i) ( 2, 3) , (4, 1) (ii) (- 5, 7) , (- 1, 3)
(iii) (a, b) , ( - a, - b)
= �( - 4)2 + (-14) 2
Sol (i) Let A(2, 3) and B(4, 0 be the given points. = �16 + 196 = .fiii.
Here, x1 = 2, y, = 3 �nd x2 = 4, Ji =1 = � = 2.J53 = 14.56 units
Now, AB = �(xi - x, )i + (Ji - y, )i and AC = �(-2 -l)i + (-1 1 - 5) 2
[by distance formula] =�( -3)2 + (- 16)i
= �(4 - 2) 2 + (1 - 3) 2 = .J9 + 256 = .J265 = 1 6.279 units
= �(2 )2 + (- 2)i = �4 + 4 = .JS = 2.J2 units Now, AB + BC = 2.236 + 1 4.56
{it) Do same as part (i) [Ans. 4.J2.] = 16.796 "# 16.279
� AB + BC "# AC
(iit) Do same as part (i) [Ans. 2 .Jai + bi units] Similarly, BC + AC AB :;1:
and AC + AB BC :;1:
2 Find the distance b etween the points (O, O) and
(36 , 15) . Can you now find the distance Hence, A, B and C are non-collinear points.
between the two towns A and B ? 4 Check whether (5, - 2) , (6, 4) and (7, - 2) are the
Sol Let M(O, O) and N(36, 15) be the given points. vertices of a isosceles triangle.
Here, x1 = 0, y, = 0 and Xi =36, 12 =15 Sol. Let P{5, -2), Q(6, 4) and R(7, - 2) be the given points..
Now, MN = �(x2 - x, )i + (1i - y,)i Then, PQ = �(6 - 5) 2 + (4 + 2) 2
-------
[by distance formula]
= �(36 - o )i + (15 -o)i [·: distance = �(x2 - x.) 2 + (Ji - y.) 2 ]
---

= �(36) 2 + (15)2 = �1296 + 225 = �(l)i + (6) 2 = .J1+36 = .J37 iinits


c::
= .J1 5 21 = 39 units QR = �(7 -6)2 + (- 2 - 4)2 LL
Since, the position of towns A and B are (O, O) and (36, 1 5) , = �(l)i + (-6)i c.
I
respectively. So, the distance between them is 39 km. = Jl+3G = .J37 units -

3 Determine, if the points ( 1 , 5) , (2, 3) and c


and RP =�(5 - 7) 2 + (-2 + 2)2 u..
(-2 , - 1 1) are collinear.
= �(-2) 2 + 0 = � = 2 units

I�
Sol Let A 5 ) B( and C - 2,
(1, , 2, 3) ( - 1 1) be the given points.
Then, AB = �(2 -l)i + (3 - 5) i Thus, PQ = QR
Since, two sides of a triangle are equal.
[ ·: distance = �(x2 -x, )i + (12 - y.}2 ] Hence, flPQR is an isosceles triangle. Hence proved.
Allinone MATHEMATICS Class 10th Term II

5 I n a classroom, 4 friends are seated at the 6 Name the types of quadrilateral formed, if any.
points A, B, C and D as shown in figure. by the following points and give reason for your
Champa and Chameli walk into the class and answer.
after observin g for a few minutes, Champa asks (i) (- 1 , - 2) , (1, 0) , (- 1, 2) , (- 3, O)
Chameli, "Don' t you think ABCD is a square? " , (ii) -
(- 3, 5) , (3, 1) , (O, 3) , (- 1 , 4)
Chameli disagrees. Using distance formula, (iii) (4, 5) 0 6) (4, 3 ) (1 , 2)
I I I I

find which of them is correct?


10 Sol. (1) Let A(-l, - 2), B(l, O), C(-1, 2) and D(-3,0) bc
9 the given points..---_______
8
(/) 7 B�
Then, AB = �(1 + 1 )2 + (0 + 2 )2 = �(2)2 (2)2 +


a: 6 [·: distance = �(x2 - x1 )2 y1)2 )
+ (Ji -
5
4 A c {i i,
= .J4+4 = .JS = 2 J2 units
n

D�
3 BC = �(-1 -1)2 + (2 -0)2.J2 = .J,,_(--2-)2_+_(_2_)2
2 = .J4+4 = .JS = 2 units.--------
1 2 3 4 5 6 7 8 9 10
CD = �(-3 + 1)2 + (O - 2)2 = .J(-2 )2 + (- 2)2
Columns = .J4+4 = JS = 2./2 units
From the given figure, the coordinates of points A, B, C
[·: distance= �(xi -x1 )i + (y2 -y1 )2 ]
Sol.
and D are (3, 4), (6, 7), (9, 4) and (6, 1), respectively. and DA = �(-1 + 3)2 + (-2 -0)2
Now, AB = �(6 - 3)2 + (7 -4)i = �(2)2 + (- 2)2 = .J4+4 = JS = 2.fi.units
Thus, AB = BC = CD = DA
= �3 i + 3 i = .J9+9 = .JIB =3J2 units Now, diagonal..-s-, -____---
BC = �(9 -6)2 + (4 - 7)i AC = �(- 1 + 1 )2 + ( 2 + 2 ) 2
= � = 4 units
= �(-3 )2 + (-3 )2 = .J9+9 = .JIB = 3J2 units
= �3 i + (-3)i = .J9+9 = .JIB =3.J2 units and BD = �(-3 -1)2 + (0 -0)2
CD = �(6 -9 )2 + (1 -4)i = �(- 4)2 + 0 = 4 units
and DA = �(3 -6)2 + (4 -1)2 Thus, AC = BD
= �(-3)2 + 3 2 = .J9+9 = .JIB =3.J2 units and the
equal.DA are eqwl
Since,also diafourgonalsids esACAB,andBC,BDCDare and
Thus, AB = BC = CD = DA So, the quadrilateral ABCD is a square.
Now, diagonals, ( ii) Let
givenApoi(-3,nts. B(3, 1), C(O, 3) and D(-1;
5), -4) be the
AC = �.--(9---3-)2_+_{_4_--4-)2 = �36 + 0 .= .J36 = 6. units . Then, AB = .Jc3 + 3)2 + (I - 5)2
and BD = �(6 6)2 + (1 - 7)i = �O + 36 = {36 = 6'units
·_

[·: distance = �(xi -x1 )2 +(Ji - y1 ): ]


Thus, diagonals, AC = BD =6 units
HSince,
ence,allABCD
the fouris asides
square.andSo,diagonals
Champaareisequal
correct.. = .J(6)2 + (-4)2 = .J36 + 16
= ../52 = 2.,Jl3 units
Alternate Method

= 18 +18 = 36 = (6)2
Findsame
fourassides AB, BC = �(O -3) i + (3 -1)2
AC above. BC, CD andJi.DA andJi.one diagonal say = .J(-3)2 + (2)2 = .J9+4 = .Jl3 units
Now, AB2 + BC2 = (3 )2 + (3 )2 CD = �(-1 -0)2 + (- 4 -3) 2

and DA = �(-1 + 3 )2 + ( -4 - = .J(2 )2 + (-9 ):


AB2 + BC2 = AC2 = �(-1)2 + (-7)2
Therefore, by converse of Pythagoras theorem, LB = 90°
A quadrilateral wi ch all four sides equal and one angle 90°, is = .JI+49 = .J50 = s./2 units..-------
a square. 5 )2
So, ABCD is a square. = .p;+8l = .J8s units
Coordinate Geometry f 2 45
Now, diagonals, => (y + 9) (y - 3) = 0
AC = �,..-(0_+_3_t_'+--( 3---5-) 2 = �(3)2 + (-2)2 => y = - 9 or y = 3
Hence, the required values of y are -9 and 3.
= � = .Jlj units
9 If Q (O, 1) is equidistant from �(5, - 3) and
and BD = �(-1 -3) 2 + (-4 -1)2 = �(-4)2 + (-5) 2
R (x, 6), then find the values of x. Also, find the
= .J1 6 + 25 = .J4i units distance QR and PR.
Here, we sec that from above sides, Sol. Given point Q(O, 1) is equidistant from P(5, -3) and R(x, 6).
AC + BC = AB . :. QP = QR => QP 2 = QR2 [on squaring both sides]
i.e. .Jl3 + .J1j = 2.Jlj
=> A, B and C are collinear. => (5 -0)2 + (-3 - 1)2 = (x -0) 2 + (6 -1) 2
Hence, points A, B, C and D does not form a
quadrilateral.
+ (_y_2 _
[ ·: distance = ��(x_2____x1-)-2 - _ y_1)-2 ]
(iii)Do same as pan (i). => 5 2 + 4 2 = x 2 + 52
[Ans. The quadrilateral ABCD is a parallelogram because' => 25 + 16 = x 2 + 25
AB = CD = .JIO, BC = DA = 3.fi. and AC :1: BD] => x 2 = 16
7 Find the point on the X-axis which is => x=± 4
-
equidistant from ( 2 , 5) and (- 2, 9) . Thus, coordinates of R is ( 4, 6) or ( - 4, 6).
Now, QR = Distance between Q(O, 1) and R(4, 6)
Sol Let A( x, O) be any point on the X-axis, which is equidistant
from B( 2, - 5) and C ( - 2, 9 ). = �(4 -0)2 + (6 - 1)2 = �4 2 + 5 2
.. AB = AC = �16 + 25 = .J4i units
=> AB2 = AC2 [on squaring both sides] or QR = Distance between Q(O, 1) and R(- 4, 6)
=> (x - 2)2 + (0 + 5 )2 = (x + 2) 2 + (O -9)2 = �(- 4 -0) 2 + (6 - 1)2
[·: distance = �(x1 -x2 ) 2 + (y1 - 12 ) 2 ] = �(-4)2 + 5 2 = .J16 + 25 =J41 units
Also, PR = Distance between P(5, - 3) and R( 4, 6)
=> x 2 - 4x + 4 + 25 = x 2 + 4x + 4 + 81 = �(4 - 5)2 + (6 + 3)2
=> - 4x - 4x = 81 - 25
=> - Bx = 56 = �(-1)2 + (9)2 = .JI+8l = .J82. units
=> x = -- 56 = - 7 or PR = Distance between P(5, - 3) and R(- 4, 6)
8 = �(- 4 - 5 .)2 + (6 + 3 ) 2
Hence, the point equidistant from given points on the
X-axis is (- 7, 0). = �( -9)2 + 92 = .J81 + 8 1 = .Jl62 =9...fi. units
8 Find the values of y for which the distance
between the points P (2, - 3) and Q (10, y) is 1 0 Find a relation between x and y such that the
10 units. point (x, y) is equidistant from the points (3, 6)
and (- 3, 4).
Sol. Given points are P(2, -3) and Q(IO, y ).
According to the question, PQ = 10 Sol. Let the point A(x, y) be equidistant from the points B(3, 6)
and C(-3, 4).
=> �(10 - 2)2 + (y + 3)2 =10 Then, AB = AC
[·: distance = �(x - x1 )2 + (Ji - y1 ) 2 ]
=> AB2 = AC2 [on squaring both sides]
2 => (x -3)2 + (y -6)2 = (x + 3)2 + (y - 4)2
=> �(8 )2 + (y + 3)2 =10 [·: distance = �(x1 - x2 )2 + (y1 - y2 ) 2. ]
=> �64 + y2 + 9 + 6y = 10 => x 2 -6x + 9 + y 2 - 12y + 36
=> �l + 6y + 73 = 10 = x -., + 6x + 9 + y -., - By + 16
On squaring both sides, we get y 2 + 6y + 73 = 100 => -6x -6x -1 Zy + 8 y + 36 -16 = 0

� y2 + 6y - 27 =0 => -12x - 4y + 20 = 0
� y 2 + 9y -3y - 27 =0
=> - 4(3x + y - 5) = 0
. [by factorisation] => 3 x + y -5 = 0 [dividingby- 4]
. which is ch� required relation between and y.x

y (y + 9) - 3 (y + 9) = 0
TO P I C EXERCISE
Very Short Answer Type Questions 7 Find the points on the X-a x i s which a re at a distance
1 Find t he d i stance of the point P(3, - 4) from the of 2../5 from the point ( 7. - 4). How many such points

orig i n . are there? CBSE 201 1

2 Fi n d the d i stance between t h e points P 8 I f the distance between the points A (l. 1) and B(x. 1)
(cos a , - si n a ) a nd Q ( - cos a.sin a ). is 5 units, then find the value(s) of x.

Short Answer Type I Questions Long Answer Type Questio n s


3 Show t h a t t h e points ( 4. 2), ( 7,5) a n d ( 9, 7) d o not form 9 Prove that the points A (O . 1). B(l. 4), C (4. 3) and
a t riangle. D(3 . 0) are the vertices of a squa re.

4 Find the relation between x and y such that the 10 Show that the points A (2, -2). B(14, 10), C(ll, 13) and
point (x, y) is equidistant from ( 7, 1) and (3, 5). D(-1. 1) are the vertices of a rectangle .
1 1 If two vertices of an equilateral triangle be (0, 0)
Short Answer Type II Questio ns
(3, J3). Find the third vertex .
5 Show that ( a, 1), ( b, 1), ( c, 1) are collinear points.
1 2 If A (x, y) is a point on the line joining the points
6 Show that the points ( 3, 2), 4, ( %) and (5, 3) are
.
which are equidistant from B(2, -3) and C(-4,
then prove that 4x + 3y + 1 = 0.
5),

coll inear.

Answers
5 2. 2 units 4. x - y = 2

- -
1. units
7. (9, O) and (5, O); Required number of points = 2
8. x = 4 6 ,
1 1 . (0, 2'/3) or (3, .J3)

TOP I C 0 S ection Formula


I n this topic, we will develop a formula to find the This formula is known as section formula for internal
coordinates of a point which divides the given line segment division.
y
· ·

internally or externally in a given ratio.


Proof Consider any
two points A(x 1 , y 1 )
I nt e rn a l D iv is io n of a Line Seg ment ·
and B(x 2 ' y 2 ) and
Let A (x 1 , y 1 ) and B(x 2 , y 2 ) be the two points and P (x , y) assume that point

=
is a point on the l ine segment j oining A and B such that P(x, y) divides AB
AP : BP m 1 : m 2 , then point P is said to divide line internally in the ratio
segment AB i nternally in the ratio m 1 : m 2 •

m2 �2' Then,
(x,, Y'
A

( )·
. . . (i)
Then, coordinates of point P are
Draw the perpendiculars AR, PS and BT on the X-axis from
m 1 X2 + m 2 X1 m 1 Y 2 + m 2 Y1 the points A, P and B, which intersect X-axis at R, S and T.
respectively. Now, draw A Q ..l PS and PC ..l BT, which arc
,
m1 + m2 m1 + m2
parallel to X-axis.
Coordinate Geometry f 247 '

By M similarity criterion, M QP ,.. !lPCB Special Case


PA AQ PQ If a point divides the line segment joining the points (x 1 , y 1 )
- = -- = -- . . . (ii)
and (x 2 , y 2 ) internally in the ratio I : l , then we get

(
BP PC BC
Now,

J
mid-point of chis line segment. Thus, coordinates o f the
AQ = RS = OS - OR = x - x 1
mid-point P of the line segment joining the po in ts A (x 1 , y 1 )
PC = ST = O T - OS = x 2 - x

= + xi , y i 2
. xI 2 + yi .
PQ = PS - QS = PS - AR = y - Y1 [·: QS = AR] and B(x 2 , y 2 ) is
and BC = B T - C T = B T - PS Ji - y [·: CT = PS]
On putting these values in Eq. (ii) , we get Example 1. Find the coordinates of the point which
PA x - x1 - divides the line segment joining the points (4, - 3) and
= = y Y1
BP X2 - x Y2 - y (9, 7) internally in the ratio 3 : 2.

=>
m1 = x - x1 . = Y - Y 1 [from Eq. (i)] . . . (iii) Sol. Let P(x, y) be the required point.
Then, P divides AB internally in the ratio 3 : 2.
Ill

On taking I and II terms of Eq. (iii) , we get


A 3:2 B
(9. 7)
x - X1
p
m1
=

Then, P(x, y ) p ( m.. xl ++ mmix, ' "1iYl ++ miJ, )


(4, -3)
m..
m2 X2 - X 3 and (x. , y,) (4, -3); (x2 ' yl )
Here, - = - = (9, 7 ) =
m 1 (x2 - x ) = m 2 (x - x 1 ) m2 2

(p 3 3 ++ 22 4 ' 3 3+ 2[by (section


-3) )
- m 1 x - m 2 x = - m 1 x 2 - m 2 x1
=
x (m 1 + m 2 ) = m 1 x 2 + m 2 x 1 l m,. ml m,.

( 5+ 2\-6 ) = p( 3; ' n (7, 3)


=> x
m 1 X 2 + m 2X 1 formula]
x 9 x x 7 x
m1 + m2 =

On taking I and I I I terms of Eq. (iii) , we get


+2
=
p 27 8' I p
m1 - =
= y Y1
mi Y2 - Y
Therefore, ( 7, 3) is the required point.
m 1 CJ2 - y) = m 2 (y - J1 )
=> m1 Y 2 - m 1 y = m 2 y - m 2 J1 Example 2 . In what ratio, does the poin t (- 4, 6)
divide the line segment joining the points A ( - 6, 1 0)
=> Cm 1 + m 2 ) Y = m 1 Y 2 + m 2 J1
m 1 Y2 + m 2 J1
-
and B (3, 8)? Also, verify the ratio.
=> y= Sol. Let point P ( - 4, 6) divides the line segment joining the points
m1 +m2 A (- 6, 10) and B (3, 8) in the ratio 111i ml .
·J
- :

(
Hence, the coordinates of the point P are
A ---- m1 m2

p
)
m 1 X 2 + m 2 X 1 m 1 J2 + m 2 J1 ----+-- a

(
, (-6, 1 0) (-4 ,6) (3, -8)
m1 + m2 m1 + m2
By using section formula, we get
No te (I) In case of i nternal division. poi nt dividing the line
segment lie on it. But in external division. point dividing (- 4, 6) 3 111i - 6m2 , - 8 Tni ++ ml 0m2
= • • • (i)
m,. + m2 m.. 2
(ii) I f a poi nt P divide l i ne segment AB externally in the ratio
the line segment does not l ie on it.

- ----- � p
On equating x-coordinate both sides of Eq. (i) , we get
m, : '71:! ·
4 3 11Zi - 6mm2
_
=

..
m.. + z
- 4 (m. + m2 ) 3 m,. - 6m2 =

( m, m2y1J.
1
.::::----a ,.- _-- m 2 �I
A 1 =>
- 4m. - 4m2 = 3 m1 - 6m2
Then . coordinates of point P are
=> - 4m. - 3nli - 6m2 + 41112 =

m,x2 - �x, m,y2 -


- 7m. - 2m2 =

. m. 2
- m2 m,- � - = -

This formula is known as section formula for external m}. 7


division. or . . . (ii)
Alllnone MATH EMATICS C lass 1 0th Term Ii

Verification On equating they-coordinate from both sides of D ifferent Typ es of P ro b le m s


Eq. (i), we get
Based o n Sectio n Fo r m u la-
6 = - Sm.m. ++ m10m2 With the help of section formula, we can solve many types of
m. 2 problems. Some types ofproblems and methods to solve them
- a + 10 are given below
6 = 111im2
m2 + 1 ( Type I ) To show Collinea rity Using
[on dividing the numerator and denominator of RHS by m2 ] Section Form u la

- s ( �7 J + 10 - 16 + 70 54 6
On putting 111i = � from Eq . (ii), we get To show collinearity of having points say A , B and C by
m2 7 section formula, first we assume that C divides AB in the
ratio ofk : 1 Then, write coordinates ofC by section formula
-- -
...;. .- and then equate x and y-coordinates separately. If value ofIt
6 = -----
�+1 = 2+7 = 9 = from both equations are same, then A , B and C are collinear,
7 otherwise not.
Hence, the point ( - 4, 6) divides the line segment joining the
points A(-6, 1 O) and B(3, - 8) in the ratio 2 : 7. Example 3. Using section form ula, sh ow that the
Alternate Method points A (- 3, - 1 ) , B (L 3) and C ( l 1) are collinear.
-
Let point P(-4, 6) divides the line segment joining the points CBSE 20 1 1
A(-6, 10) and B(3, - 8 ) in the ratio k : 1. Sol. Lee C (-1, 1) divides AB in the ratio k : 1.

( kk -3+ l , 3kk + l )
A -- k -- B A--�-k��--��1�--B

k + 10 1 )
(-3,-1 ) C(- 1 ,1 ) ( 1 , 3)

( ' 6) - ( k k+ +(-6)
(-6, 1 0) p ( 3, -8)
(- 4, 6) Then,
. by using section formula, we get
c (- 1, 1) = c

+ 10 )
By using section formula, we get -1

(-4, 6) = ( kk +- l6 ' -Bk


-
4
3 x x1' -8 x x

. 1 k�1 , On equating x-coordinate from both sides , we gee


3
. . . (i) - l = kk -+ 31
k+l
� -k -l = k -3
On equating x-coordinate both sides of Eq. (i), we get - 2k = - 3 + 1
3k - 6
-4 =

I = --
k+1 -2k = -2
k =1

k = -27
=> -4 ( k + 1) = 3 k - 6 => -4 k - 4 = 3 k -6
On equating y-coordinate from both sides, we gee
=> 3 k + 4 k =6 - 4 3k - 1
=> 7k = 2
k+I

we get 6 = ---

=> . . . (ii) k + I =3k - 1
� 2k = 2
Verification On equating y- coordinate both sides of Eq . (i),

On putting k = -2 from Eq. (ii), we get


k =l
-Bk + 10 Since, in both cases value of k is same. So, C divides AB in th�
k+1 ratio 1 : 1, i.e. C is the mid-point of AB.

-8 - + 10
He'iice, A, B and C are collinear.
7
2 x
(Typ e II J Problems Based on Finding the

6= 27 Section Ratio and Section Point


-7 + I Sometimes, end points of a line segment and an axis which
divides this line segment (or any other condition) is given
6 = -162 ++770 = 594 = 6 and we have to find section ratio and section pofot both. In
this type of problems, first we find section ratio by using
Hence, the point (-4, 6) divides the line segment joining the given condition (or property of axes) . Then, find section
points A(-6, 10) and B(3, - 8) in the ratio 2 : 7. point (or point of intersection) by using section formula.
Coordinate Geometry f 249 I
Example 4. Find th e ratio, in which the Y-axis
- (Type J III Problems Based on Fi.ndi.ng the U nknown
divides the line segment joining the points (5, 6 ) and Vertex of the Geometrical Figure
(- � - 4). Also, find the point of in tersection.
Sometimes, for finding the unknown vertex or end point of
Sol Lee che end points of che line segment AB be A(5, -6) and a line segment in given geometrical figure (e.g.
B (- 1, 4) P
- and point divides it in the ratio k: (say) . 1 Parallelogram, circle etc.) we use the mid-point of this line
y segment and properties of given geometrical figure.

Exam p le 5 . If the points A (6, 1) , B (B, 2) , c (9, 4) and

X' ._-..+---+--+---11--+-+-+-+-+-+-+-� X
-1 5 D (p, 3) are the vertices of a parallelogram, taken in
order, then find the value of p.
B
(-1 . -4)
Sol. Given vertices of a parallelogram are
and D (p, 3). A (6, 1), B (8, 2), C(9, 4)
Y'
A
(5, -6)
Here, we have co find the value of p.
We know that, diagonals of a parallelogram bisect each other.
Then, by using section formula, we gee :.Coordinates of mid-point of diagonal AC
Coordinates of P = ( -k+5 , -4k-6
k+1 k+1 )
=Coordinates of mid-point ofdiagonal BD
( 6 � 9 · 1 : 4 )= ( 8 : p . 2 � 3 )
('� · %) =( 8 : p · %)

Given, che Y-axis divides che line segment AB and we know
(O, y).
0.
that, on che Y-axis, abscissa is So, the coordinate of P are �

-k +5 -4kk +-6l )
(O, y) -- ( �, .. . (i) On equating x-coordinate from both sides, we get
On equating x-coordinate both sides of Eq. (i), we get -152 = --
8 +2 p
-kk ++1 5 =0 -k + 5 =0 => => 15p =8=15+-Bp
=>
::)Thus, the requiredk =5ratio is 5 : 1.
=> p =7
Hence, the required value ofp is 7.
Now, putting k = 5 in Eq. (i), we get Exam p le 6. Find the coordinates of point A where
0 =( -55 ++1 5 , -4(5)
( , y)
5 + 1-6 ) =(0, -266 )=(0, -133 ) AB is the diameter of a circle whose centre is (3 , 4)
and B is (1, 4 ) .
-

which is the required point of intersc:;ction (i.e. section point). Sol. Let
AB be the diameter and C be the centre of the
circle. Let coordinates of be (a, and C be the mid-point
A b)
TRY Y6URSELF of
AB.

Q. 1 Find the coordinates of points which divides the line


-
segment joining (1, - 2 ) and ( 3, 4) in the ratio 1 : 1 .

( )
[Ans. (- l , l)]

Q. 2
Coordinates ofC = a ; 1 • b : 4
Find the coordinates of the point which divides the line

2115)]
[·.- mid-point = ( x, : )]
segment joining the points (6, 3) and (- 4, 5) in the ratio
3 : 2 internally. [Ans. (O,

Q. 3
:
(a; : 4)
In what ratio does the point C(3 I 5, 1 1 I 5) divide the line

[Ans. 2 : 3]
x, .
segment joining the points A(3, 5) and B(- 3, - 2'[? y, y,

Q. 4 In what ratio does Y-axis divides the line segment joining


the points P(- 4, 5) and 0(3, - 7'[? Also, find the section
I, b
� =

( -;3 )]
(3, 4)
[Ans.
_

4 :3 or 0,
point.
[given, coord inates of C = (3 , 4 )] -

Q. 5 .
Using the se ction formula, show that the points On comparing the coordinates of and y, we get -2- 3
x
a+l =

A(2 , 3) , 8(1, 4) and C (- 2, ?) are collinear.


1 2so l

+ )
AllinOne MATH E MATICS C lass 10th Tenn \1

b + 4 =-4 'lniY2 + m-i],


P (x, y)

)
and
2
Now, = P( m1 x 2 + m2x1
m1 + m2
,
m1 "'2
=> a + 1 =6
[by sectio n formula]
b + 4 =-8

) i � -;)
,/ 1 x 4 + 2 x 2 , 1 x
{;
and
=>
(- 1 ) + 2 x ( - 3)
a

and b = - 12
=5 =
.. l 1 + 2 1+2
Hence, the coordinates of point A are (5, - 1 2). 4 4 -1 -6

(Type IV) Problems Based on Finding Points. of


=1 . = ·
3

2 : 1 , then 111i = �I
m2
(ii) Let Q divides AB in the ratio
Trisection
Trisection means a line is dividing into three equal parts. y
Here, A (x. , 1 ) = ( 2 , - 3 ) a n d B (x2 , Yi ) = (4, -1 )

Q (x, y ) = Q( m1 Xi ++ mm2X1 , "1iTniY2 ++ mm21, )


A P Q B Q
A �����-1-��___, B
This can be done by finding two points P and Q on the line (2 , -3} 2 :1 (4, -1 }

2 x 4 + 1 x 2 2 x (- 1) + I x ( - 3) )
segment AB, such that AP = PQ = QB.
Now,
Let AP = PQ = QB = x i. 2

J (
m1

)
= rl
Then, AP = x and PB = PQ + QB = x + x = 2x
AP : PB = x : 2x = 1 : 2 "'l. ,
2+1 2+1
Also,
= nr
A Q = AP + PQ �. - 2 - 3 � s -

=Q .
= x + x = 2x '<\. 3 . 3 3 3

If points P. Q and R divide the l i n e segment AB into four


QB = x
Note
and
Then, A Q : QB = 2x : x = 2 : 1 equal pares, then P divides A B in the ratio I : 3, R divides
AB in the ratio 3 : I and Q divides AB i n the ratio I : I, i.e.
mid-point of AB.
Hence, to find p oints of trisection, we find two points P and

I II
Q (say) such chat P divides AB in the ratio 1 : 2 and Q divides
I II I
AB in the ratio 2 : 1 . A p Q R B
Example 7. Fin d the coordinates of the points of
trisection of the line segmen t joining (2, - 3) and (4, - 1). TRY YOURSELF
Sol. Let P and Q be the points of trisection as shown below
A B Q. 1 If (1 , 2) , (4; y), (x, 6) and ( 4 , 2) are the vertices of a
(2, - 3) p Q (4, -1 ) parallelogram taken in order. Find x and y.
Then, AP : PB = 1 : 2 and A Q : QB = 2 : 1 [Ans. x = 7, y = 6 1
(t) Let P divides AB in the ratio 1 : 2. Q. 2 Find the coordinates of the points which divide the line
11ti segment joining A(- 2, - 2 ) and 8(2, 8) into four equal
( I , �I )
1

[ Ans. ( n
Then, - =-
i (0, 3),
m2 2
- !,
parts.
p
A B
1 :2 Q. 3
.
(2, -3) (4, -1 ) Find the lengths of the medians of the triangle whose
Here, A ( xi ' y1 ) = ( 2, - 3 ) and B (x2' Yi ) = (4, -1) vertices are (5, 6) , (3, 8) and ( 1, 2 ). [Ans. sJ2 .Jl?. M l
-
NCE RT F O L D E R 7.2
= [ 2 x (-22)+ +1 1 x 4 , 2 x (-3)2 ++ 1I x (- l) J
..
.---,-.
--� - · -
'

.... - l ,

1 Find the coordinates of the point which divides

= ( - 43+ 4 . - �- l ) = (o. - fl
the line segment j oining of (- 1, 7) and (4, - 3) in
:

So, the cwo poincs of criseccion are ( 2, -�) and 0, -f)


the ratio 2 3.

(
Sol. Lee the coordi nates o f a point be (x, y).

( x, y ) - ( Tn,X2 ++ m,.x, , Tni.Yi ++ miY, )


We have, x1 = - 1, y1 = x2 = 4, y2 = -3
7,
and m1 m2
= 2, = 3
By using section formula, we have

= [ 2 x 4 2+ +3 x3 (- 1) , 2 x (-3)
2+3 ]
3 To conduct Sports Day activities in your
m2 Tni m2
"'1 rectangular shaped school ground ABCD, lines

= ( 8 -5 3 , -6 5+ 21 ) = ( �5 , �5 ) = (I, 3)
+3 x 7 have been drawn with chalk powder at a
distance of lm each. 1 00 flower pots have been
placed at a distance of 1m from each other
along AD, as shown in figure.
r--r-r-r-r-.--.--.--..--..--....... C
Hence, coordinates of the point are (1, 3).

2 Find the coordinates of the points of trisection


of the line segment joining (4 , - 1) and ( 2, - - 3).
Sol. Lee - 1) and
A { 4, B ( - 2, - 3)
be the end points of line
segment AB
and points of trisection of the line segment be
P Q
and .
Then, AP = PQ = BQ = k [say]
Now, PB = PQ + QB=k + k = 2k
k k k
(4, -1 } A ------ a (- 2,- 3)
p Q
and AQ = AP + PQ = k + k =2k 2 3 4 5 6 7 8 9 10 8
AP : PB = k : 2k =1 : 2
and AQ : QB = 2k : k = 2 : 1 Niha rlka runs .!.th the distance AD on the 2nd
Since, P divides AB internally in the ratio 1 2. :
4
2 line and posts a green flag. Preet runs .!.th the

= [1 x (-12)+ +2 2 x 4 , 1 x {-3)1 ++ 22 x ( - l)
A 5
(-2, -3)
B
p distance AD on the 8th line and posts a red

[(
(4, -1 }

J
So, the coordinates of P fla�. What is the distance between both
the flags? If Rashmi has to post a blue flag

]
exactly halfway between the line segment

+ m2 J M ( 2 x I, � x 100 ) i. e. M (2, 25 ) and red flag


joining the two flags, where should she post her
·: using section formula

= ( - 23+ 8 , �J ( ) ( -�J3
flag?
· e. "'1X2 + m2X1 "'1 J2 + 1ni.J1

posced by Pree< is N ( 8 x I, � x 100 J i.e. N (8, 20).


Sol. From the given figure, the position of green flag posted by
1. , _...;.-"--
.;;.. -- 0
"'1 + m2 "'1 Niharika is
� ' -5 = 2,
3 =3 3
·: Q divides AB internally in the ratio 2 : 1.
2 1 Now, MN = �(8 - 2)2 + (20 - 25) 2
] )2
A ----
Q
(4, -1 }
--1�- s
(-2 , -3)
2 1 >__+_(_J_2-_-Y-1-
�,...( x_.. 2____x_
[·.· distan ce =

So, the coordinates of Q = �(6)2 + ( - 5)2 = �36 + 2 5 = .J6l


i 2 s2 I Allin.One MATH EMATICS Class 1 0th Term 11

Hence, the distance between flags is .J6i m. According co the question, line segment joining A(l, -5)

)
Let P be the position of the blue flag posted by Rashmi in and B(-4, 5 ) is divided by X-axis, i.e. point M lies on
X-axis.

( )
:
Then, coordinates of P = x, : x, , y, y,
the halfway of line segment MN.

= ( 2 +2 8 , 25 +2 20 ) = ( 120 , 452 ) = (5 , 22.5)


:. Coordinates of M = (x, O ) ...(ii)
( From Eqs. (i) and (ii), we gee
(x, 0) = -4 k + l , --
Sk -5
... (iii)
k+l k+l
. On comparing y-coordinaces both sides, we gee
Hence, the blue flag is on the 5th line at a distance of
22.5 m above it. 5k - 5 = 0
4 Find the ratio, in which the line segment
k+1
5k - 5 = 0

( ) ( )
joining the points (- 3, 1 0) and (6 , -8) is divided
by (- 1 , 6) . => k =1
Let the point A(- 1, 6) divides the line joining B(-3, 10) On putting k = 1 in Eq. (iii), we gee

( )
Sol.
and C(6, - 8) in the ratio k : 1. (x, 0) = -41 x+l1+ l , S x1 +l -1 5 = -42+ 1 , 5 -2 5
1 0) A(-1 , 6) C(6,- 8)

(x, 0) = ; , 0
8(-3,

=>

( }
-

So, the required ratio is 1 : 1 and che po inc of division M is


- Then, the coordinates of A are 6k - 3 , - 8k + 10
-- ----
( -; . o)
[-:
k+1 k+l

( ++ , ++ )]
6 If (1, 2) , (4, y) , (x, 6) and (3, 5) are the vertices of a
by section formula, coordinate of a point
parallelogram taken in order, then find x and y.
Sol. Lee A(l, 2), B( 4, y ), C(x, 6) and D(3, 5) are che vertices of

(-l, 6) = ( 6k.- 3 , -Bkk ++l10 )


p ntiX2 m2X1 11tiY2 m211
= a parallelogram.
1ni m2 1ni m2
0(3, 5)
=>
k+l
On comparing coordinates both side, we gee
6 k - 3 = 1 and - Bk + 1 0 = 6
_

k+l k+l A( 1 , 2) 8(4 , y)


� 6k - 3 = - k - l and - 8k + 10 =6k + 6
� 6 k + k = - 1 + 3and - Bk -6k =6 - 10 Since, ABCD is a parallelogram.
:. Diagonals AC and BD will bisect each other. So, chc

( )( )
� 7 k = 2 and -1 4 k = -4 mid-point of AC and mid-point of BD are same point.
=> k = -2 :. Mid-point of AC =Mid-point of BD
1:x . 2 ; 6 = 4 ; 3 . y ; 5

[·: coordinates of mid- point = ( x, : x' . y, ; )]


7
So, the point A divides BC in the ratio 2 7. :
=>
5 Find the ratio, in which the line segment
j oining A('L - 5) and B ( - 4, 5) is divided by the y,
c� X-axis . Also, find the coordinates of the point of
w division .
0 Sol. Let M divides che line joining the points A (1, -5) and On comparing the coordinates, we gee
,__J B ( -4, 5) in che ratio k : 1 I+x 4+3
= 2+6 =5+y
2 and 2 2
0 k
A -----1--- B
2
LI� (1 , -5) M (-4 , 5)
1 + x = 7 and 8 = 5 + y

( J
t-� Bysection formula, we gee x =6 and y = 3
� Coordinates of 7 Find the coordinates of a point A, where AB
i�_!.JI IJ -4k + l , 5k - 5 is the diameter of a circle whose centre is (2 , 3) -
M =
;� ... (i) and B is ( 1 , 4) .
k + 1 k +1
Coordinate Geometry f 2s3 I
9 Find the coordinates of the points which divide
Sol.
ofGicentre
vcheen, AB is ahaving
circle
ac C(2
diameterics
, -3) and y)A the line segment joining A(- 2, 2) and B (2, 8)
(x, 8(1 , 4) into four equal parts.
coordinates of end point
are 4 ).of LeeA chebe
Bcoordinates
(1, Sol. Lee P, Q and R be the points on line segment AB such
that AP =PQ =QR =RB

Then, coordinates of = ( ; , y ; 4 J
( x, y). Since, AB is Lee AP = PQ = QR = RB = k
diameter.

[ · coordinates of mid-point ,;, ( : , y, : y, J]


k k k k
:. C is the mid-point of AB. (-2, 2) (2 , 8)
x 1 A p Q R B
C AP =-k = -1

( 2, 3) = ( x ; I , y : 4 ) :. Coord.1naces of = [ x 21 + 3( - 2) , ----
Now, -

[
x'
PB 3k 3
1 + 3 -J
x,
Therefore, P divides internally
AB in the ratio 1 : .3.
p 1 +3+3x2 1 x 8

( � . ++ , ++ =--"- J
J
_

using section formula,

= ( 2 �6· : 6 ) = ( �4· l: J = ( -1. �)


[given, coordinates of C = (2, -3)]
y
On comparing coordinates, + = 2 weandgee + 4 = -3 1. e. . X2 m2X1 111i Y2 mi_Y,
"1i "1i
---

x 1 m2 m2

y
=>
2+ = 4 and +2 4 = -6
1
8

x "= 3 and y = -10


x

. AR
=>

:. Coord.1nates of R = [ 2 +3 1+ x1 ( - 2) , ----
Hence, che required coordinates of A are (3, - 10). Agam, - = -3kk = -31

RB
8 If A and B are (- 2, - 2) and (2, 4 ) respectively, - ,
Therefore, R divides 3internally AB i n the ratio 3 : 1.
= ( 6 :2 , 24�+ 2 ) = ( �· 246 ) = (I, In
then find the coordinates of P such that x 3 x + 2-] 8 1 x

AP = AB and P lies on the line segment AB. 3 +1


7
According co the question, AP = 73 AB 7 A!1 ::::>
:::= 3
Sol.

AP + PB = -7 [·: Plies on .·.AB =APAP +PB]


PB = -7 PB = Z = 7
AQ =-2k =-1 · '

.. Coordinates ofQ = ( 2
=> A.B
Also, - QB 2k 1
AP 3 => Q is che mid-point of - 2 + 2, 2; 8)
[·.- coordinates of mid-point ( : , y, : y, )]
AB.
=>
3
=> +- APl
34 APAP 3 3 -l -

Q = ( % . 1�) = (0,
=>
PB
- = -3 PB = �4 or AP : PB = 3 4 :
=
AP
=> x x2
,
P(x, y)

So, the required points are ( -I; �} (I, 1: ) and (0, .


A(-2, -2) 8(2,- 4)
� 5)
3:4
)
5

Coordinates of P = ( J
joiSuppose
ning the poiy)ntsbeA(-the2,poi- n2)t whi
P( x ,
andcB(2,
h divides
-4) ithen thelinratie segment
o 4. 3: 1 0 Find the area of a rhombus, if its vertices are
By section formula, we have (3, 0) , (4, 5) , (-1 , 4) and (-2 , -1) taken in order.

= [3 2 4 ( -2> , 3 (- 4) + 4 (- 2)]
'11i X2 + m2x1 , "1iY2 + mi], [Hint Area of a rhombus = .!_ (Product of its 2

·
"1i + m2

= ( 6 ; . - l�- 8 ) = (.� -�O )


"1i +m2 diagonals) ]
=>
y) x + x x x Sol. Lee A(3, O), B(4, and D(- 2, be the
5 ) , C( - 1,
4) - 1)
( x,
3+4 3+4 vertices of the rhombus ABCD.

Hence,· the required coordinates of the point P are


2 :. Diagonal, AC = �(-1 -3)2 (4,-----

H - ;)
8 -0)2 +
[·: distance= �(x2 - Y + (Ji-- -)2 ]
)
x
- -

y1

2 = �(- 4 2 + 42 = /32 = 4.J2 u nits


Diagonal, BD = �(- 2 - 4)2 + 2 (- 1 - 5)
254 � Allinone MATH EMATICS C lass 1 0th Term n

= �(-6 ) 2 + (-6 ) 2 = �36 + 36 = JTi. = 6.J2 units =>


3 - 4 = 111
-- => 111 = ( - l ) x
---
2
: . Area of the rhombus ABCD 3+2 5
2
= ..!.._ x Produce of irs diagonals -2
2 Hence, the value of is -.
5
m

= ..!.._ x AC x BD = ..!.._ x 4.J2 x 6..fi.


2 2 1 2 Show that A (6, 4), B (5, - 2) and C(7, - 2) are the
= 2 x 6 x J2 x J2 = 1 2 x 2 = 24 sq units
vertices of an isosceles triangle. Also, find the
length of the median through A. CBSE 2010
Add itional Questions
Sol. Given points are A (6, 4 ), B ( 5, - 2) and C(7, - 2).
11 Determine the ratio in which the point P(m, 6)
divides th� line segment joining the points
Then AB = Jc6 - 5)2 + (4 + 2)2 -
A (-4, 3) and B (2 , 8) . Also , find the value of m y2---,-.) 2)
[·: distance = ��(x-2---x,-)-2-+-(-
Sol. Let P(m, 6) divides the line >"� �
= J0)2 + (6)2 = J1+36 = J37 units

(
k /'- 1
segment AB in the ratio k : 1. ( A 3) B BC = J( 7 - 5 )i + (-2 + 2)2
The!}, by section formula, we -:4. P (2, 8)

J
have = Jc 2)2 + 02 = .J4 = 2 units

( J
Coordinates of P = 2 k - 4 , Bk + 3
.
-- --
and AC = JC6 - 7)2 + ( 4 + 2)2
k+l k+l
= Jc- 02 + (6)2 = �l + 36 = J37 units
2k - 4 B k + 3
= (m, 6 ) . . (i)
.

k+l , k+l
[given, coordinates of P = (m, 6) ]
0 n comparing y-coor cl"mates, we get B
k+3
. =6 --

k +I
B k + 3 = 6k + 6
B k - 6k = 6 - 3 B (5, -2) 0 (6, -2) c (7 , -2)
=>

)
k=

AB = AC

(
2k = 3
2 So, M.BC is an isosceles triangle.
O n comparing x-coordinates i n Eq. (i), we get

(%)
2k - 4
Lee D be the mid-point of BC. Then, coordinates ofD arc

--- =m 5 + 7 - 2 - 2 1.e.
. (6, - 2).
k+1 - 2- , 2
2 -4
AD = J(6 - 6) 2 + (4 + 2) 2 = ./36 = 6 units
On putting k = �. we gee ----'
"'--'-
- =m-
..
2 -� +1
2

TO P I C EXERCISE
¢'1

Very Short Answer Type Questions 2 If point P divides the line segm e n t joining the
1 I n t h e f o l l ow i n g figu re.AD i s a

(�, 2)
median of fl.ABC. points A ( 2, 1) and B ( 6, 5 ) i n the ratio 1 : 3, then
F i n d t h e coord i n a t e s of D. find the coordinates of P.

BJS:C
3 If the centre of a circle is - and one end of

.;
the diameter is ( 3, 2), then find the coordinates of
:::' ·

(4 , 7) 0 (6, -5) the other end.


Coordinate Geometry f 2ss I
Short Answer Typ e I Qu estio n s
- 2, - b) divides l ine segment join i ng
Long Answer Type Questions
4 If t h e centre of a circle is (2a. - 7), then find the a 9 I f P (9 a
values of a. if the circle passes through the point A ( 3 a + 1. - 3) and B(Ba, 5) in the ratio 3 : 1 , then
(11. -9) and h a s diameter 10../2 units. find the values of a and b. Also, determine the value
NCERT Exemplar of a2 + b2 • NCERT Exemplar
5 Find the coordi nates of the point C, if the point
1 0 The points A, B and C are coll inear and AB = BC. If
B (l ) divides the line segment joining the points
6 the coordinates of A. B and C are (3, a). ( 1, 3) and
(b. 4) respectively, then find the values of a and b.
A ( 3, 5) and C in the ratio 1 : 3. CCE 2013

6 Using section formu la, show that the points 1 1 Find the coordinates of t he point Q on the X-axis,
P ( 2, - 1). Q ( 0. 1) and R (-3, 4) are collinear. which lies on the perpendicular bisector of the l i ne
segment joining the points A ( - 5, -2) and B (4, -2).
Short Answer Type I I Questions
-
Name the type of triangle formed by the points Q, A
and B. NCERT Exemplar
7 If the points (7, 2), ( 5 , 1) and (3, k) are collinear, then
find the value of k. CCE 2013 1 2 Find the coordinates of the point R on the line

(% . ) divides the line


segment joining the points P(- 1, 3) and Q ( 2, 5 ) such
8 In what ratio, does the point 6 that PR =
3
S
PQ.
NCERT Exemplar
segment joining the points (3, 5) and (- 7, 9)?
CCE 2013, 12

- ·- · - - · -
- · - - - · - · - · - · - · - · - · -
- - - - · - - - - - - - - - - - - - - - - - - - - - - · - · - · - - · - · - - - · - - - · - · - · - · - · - · - · - - · - · - · - - · - - · - - · - · - · ·

Answers
2) 4. a = 5 and 3 5. (-7, 9)

- 2
1 . (5, 1) 2. (3,

9 . a = l and b = 1 0. a = and b = - 1

(�, 2;)
7. k = 4 3 , 10

Q(-;, 0) ;
8. 1 : 3

11. isosceles 1 2.

TO PI C 0 A rea of a Triangle
In previous classes, we have learnt to find the area of triangle when its base and corresponding height (i.e. altitude) are given
by using formula

x Base x Altitude
_!_
Area of triangle =
2

and in class IX, we have learnt Heron's formula to find the area of triangle when length�f its all sides are given.
In this topic, we will learn to find the area of triangle when the coordinates of its vertices are given.

Area of Tria n g le When the Coordinates of Its. Vertices a re G iven


The area of M.BC whose vertices are A(x 1 , y 1 ), B(x2 , y2 ) and C(x3 , y3 ) is given by

..!.
l x 1 ( Ji - y3 ) + x 2 (y3 - y1 ) + x3 ( y 1 - y2 ) l sq units
Area of (MBC) =
2 .•

or
AllinOne MATHEMATICS Class 1 0th Term II

Proof Given, ABC be any triangle whose vertices are It can be learnt easily with the help o f
following arrow d iagra m . Com pute the sum
A (x1 , Y 1 ) , B (x 2 , Y 2 ) and C (x3 , Y3 ) .
of produces of numbers ac the ends o f the
Draw AL, BM and CN perpendiculars from A, B and C to the lines pointing dc;>wnwards and chen sut?cracc
X-axis, respectively. Thus, ABML , ALNC and BMNC are all che sum of produces of numbers ac che ends
trapezium. of the l ine pointi ng upwards. Lastly. m ul c i ply

y the di fference by ..!..


2

C (xa. Ya) Example 1 . If A (- 5, 7), B (- 4 , - 5) and C(- 1 , - 6l


! f
: Ya
then find the area of MBC.

!1
Sol. Given, A(- 5, 7), B(...:.. 4, - 5) and C(- 1, - 6)
Here, x1 = - 5, y1 = 7, x2 = - 4, y2 = - 5, x3 = - 1
X' ..--�+-____.�____.����·�- X and y3 = - 6
- x1 -
0 M N

X3 ---
Y•
. . Area ofMBC
. . Area of MBC = Area of trapezium ABML = 21 l x, (y2 - J3 ) + X2 (J3 - Yi ) + X3 (y, - Y2 >l
+ Area of trapezium ALNC
- Area of trapezium BMNC = .!. 1- 5 (- 5 + 6) - 4 (- 6 - 7) - 1(7 + 5) 1
2
= _! ( BM + AL) ML + _! (AL + CN) LN
2 2 = .!. 1- 5 (1) - 4(- 1 3) - 1(12)1

[ J
_!_ (BM + CN) MN 2
2 ·
_

= !2 l- 5 + 5 2 - 1 21 = .!_2 35 = 1 7. 5 sq units
� (sum of parallel sides)
x

· . area of trapezium
" = x height
Example 2. If the area of a triangle with vertices
(a, 0), (5, 1) qnd (3, 5) is 8 sq units, then fin.d the value
.
: _! (Yi + Y 1 Hx1 -: x2 ) + _!_ (Y1 + y3 ) (x3 - xi )
2 2 qt a
1
I

- - (y2 + Y3 ) (x3 - xi ) ' Sol. Let A (a, O), B(5, I) and C (3, 5) be the vertices of MBC.'
2
Here x1 = a, y, = 0, x2 = 5, y2 = l, x3 = 3 and y3 = 5
Area of MBC = -1 la(l - 5) + 5 (5 - 0) + 3 (O - 1) 1
2
+ x3 y3 - xl y3 - X3 Y2 + X2 Y2 - x3 y3 + X2 J3 ]
1
= - [ X 1 Y 2 - X 2 Y1 + x3 y1 - X 1 Y3 - X3 Y2 + X2 J3 ]
2
1
= - [ (X 1 J 2 + x 2 y3 + x3 y1 ) - (X 1 J3 + X 2 J1 + X3 J z ) ]
2

[
1 (3, 5) C ------- 8 (5, 1)

J
= - [ (x 1 Yi - x1 Y3 ) + (x z y3 - X 2 Y 1 ) + (x3 Yi - x3 J2 ) ]
2
·: area of triangle
= � lx,(y, - y,) + x, (y, - y, ) + x, (y, - y,JI
= _!_ [ x 1 <Y2 - y3 ) + x z (y3 - y1 ) + x3 (y1 - y 2 )J sq units
2
B ut area of a triangle cannot be negative. So, we write a .
modulus sign. i.e. = 2I I a(-4) + 5(5) + 3 ( -1) 1 = 2I l-4a +25 31 -

:::: .!.2 J-4a + 221 sq units


I
to remember The above formula can be written as
N"te Trick
=> 8 = -2 l-4a + 221
Area of MBC = ..!. 1c x1 y2 + x2y3 + X3 y, ) - (x2y1 + x3 y2 + x, y,) I [·: area of MBC = 8sq units, given]
2
=> 16 = l-4a + 221 => ± 1 6 = - 4a + 22
Coordinate Geometry f 2s1 I
On taking positive sign, we get 16 = - 4a +22 Collinearity of Three Points
=> 4a = 22 - 1 6 4a = 6
=> Three points are said to be collinear iff area of the
=> a = -6 => a = -3 triangle formed by these three points is zero.
4 2 Let A(x1 , y 1 ) , B(x 2 , y 2 ) and C(x3 , y3 ) be three points .
On taking negative sign, we get - 1 6 = - 1 4a + 22 Then, A, B and C will be collinear iff
=> 4a = 22 + 1 6 Area of AA.BC = 0
=> 4a = 38
[x. (J2 - J3 ) + x 2 (y3 - Y i ) + x3 (J i - J 2 ) = 0 )
�2
38 19 or

Examp le 4. Prove that the points (2, - 2) , ( - 3 , 8)


a=- a =-
4 2
Hence, the required values of a arc and !.2..
2 and (- L 4) are collinear.
Sol. Let A(2, - 2) , B (- 3, 8) and C(- 1, 4).
Area of a Quad rilateral Now, area of MBC


To find the area of a quadrilateral, first we divide it into 1
= - l x1 (Yi - y3 ) + Xi ( y3 - Y1 ) + X3 ( y, - Ji ) I
triangular region, which have no common area. Then, find the 2 .
area of each triangular region separately and add them to get the = 1 2 (8 - 4) - 3 (4 + 2) - 1 (- 2 - 8�
required area of given quadrilateral.
= .!.. 18 - 1 8 + i 01 = .!.. 118 - 1 8 1 = o
Example 3. Find the area of the quadrilateral ABCD . 2 2
whose vertices are respectively A ( 1 , 1), B (7,-3) , C(1 2, 2) Since, che area of MBC is zero.
and D (7, 2 1). Hence, given points A, B and C are collinear.
. Hence proved.
Sol. Given vertices of quadrilateral ABCD are A(l,1), B(7, - 3) ,
C(12, 2) and D(7, 2 1). Method to Find a Un known
On joining AC, we get two triangular regions ACD and ACB. when Three Points are Collinear
c (1 2, 2)
�--'-----.,, Sometimes, three collinear points are given, in which
one or two or three points contain an unknown or
constant value. Then, for finding this unknown value,
we use the concept that "area of triangle formed by three
collinear points is zero" and simplify to get required
value of given unknown.
B (7, -3)

-
A (1 , 1 )
Ex a mple 5 . Find the value of k, for which points
For MCD, (k, 2 - 2 k) , (- k + L 2k) and (- 4 - k, 6 2k) are
x1 = 1, y1 = l, Xi = l 2 , Yi = 2 and x3 = 7, y3 = 2 1
collinear.
Area of MCD = -1 l x1 (y2 - y3 ) + Xi (y3 - Y1 ) + x3 (y1 - Jz ) I Sol. Let A(k, 2 - 2 k), B (- k + 1, 2k) and
2
1
= - 11(2 - 2 1) + 12 ( 21 - 1) + 7(1 - 2)1
C( - 4 - k, 6 - 2 k). Here, k is an unknown.
2 Since, given points are collinear.
= -I 1- 1 9 + 240 - 71 = -1 12141
So, area of MBC = 0
2 2 i.e. x, <Yi - J3 ) + x2 (y3 - y, ) + x .1 (y, - J2 ) = 0
= 1 07 sq units ·
k (2k - 6 + 2k) + (- k + 1 ) (6 - 2k - 2 + 2 k)
For MBC, x, = l, y1 = l, Xi = 1 2, Jz = 2 and x3 = 7, y3 = - 3
=>
+ ( - 4 - k) (2 - 2 k - 2 k) = 0
Area of MCB => 4 k2 - 6k + ( - k + I) ( 4) + ( - 4 - k) ( 2 - 4 k) = 0
=1 2 Ix, (Ji - y3 ) + Xi ( y3- y, ) + X3 ( y1 - Ji )I => 4k2 -6k - 4 k + 4 - 8 + 1 6 k - 2k + 4k2 = 0

-=
-
=> 8 k2 + 4k - 4 = 0
= .!. 11<2 + 3 > + 12 < - 3 - 1 > + 7 o 2 > I => 2k 2 +k - 1 = 0 [divide by 4]
2 => 2 k2 + 2k - k - 1 = 0 [by facto risation]
l l
= - 1 5 - 48 - 71 = - 1- 501 =
50 25
sq units 2 k( k + I) - 1 ( k + I) = 0 => ( 2 k - I) ( k + I ) = 0
-2
=>
2 2 2
:. Area of quadrilateral ABCD => 2 k - 1 = 0 o r k + l = 0 => k = 1 k = -1
or

=Area of MCD + Area of MCB Hence, fo r k = ..!_ and k = -1 , given po ints are collinear.
= 1 07 + 25 = 1 32 sq units 2
2s s l AllinOne MATH EMATICS C lass 10th Term

TRY YOURSELF
Q. 1 Find the area of triangle whose vertices are ( - a 4� (- 6, 6) and (- 3. 9i [Am. O
Q. 2 Find the value of k, if area of triangle with vertices (S. - 1� (k, 4) and (6, 3) is 5.5 sq units
[And=9ori
Q. 3 Find the value of k, if the points A(k + 1, 2ki Bl_3k, 2k + 3) and C(Sk - 1, Sk) are collinear. NCERT Exemplar

[Am2.i
[Am. ::]
Q. 4 Find the value of m, if the points (5, 1 ) , (- 2, - 3) and (a 2m) are collinear.

N CE RT FO L D E R 7. 3

Xi +-X2 , Yi + )�1J
(-
1

[
Find the area of the triangle whose vertices are
(i) (2, 3) , (-L O) , (2, - 4) (ii) (-5, - 1 ) (3, - 5) , (5, 2)
,

(i) Let A (xi ' Yi ) = (2, 3) , (xv y2 ) = (-1,


A B O) 11
·:
Sol. B
-2-
. =
. o f nud-pomt
.

N = ( o : 2 . - 12+ 1 ) = 0. o J
and C (x3 , y3 ) = C (2, - 4) coordmace
2
:. Area of MBC Coordinate of mid-point of AB,
= -21 l xi <Y2 - y3 ) + X2 (y3 - Yi ) + X3 (yi - Y2 ll

= ( 0 : 2 • 3 ; 1 ) = (I, 2)
= .!_2 12 (O + 4 ) + (-1) (- 4 -3) + 2 (3 -0) 1 and coordinate of mid-point of BC ,
= -21 I8 + 7 + 61 = - p
21 sq Units.

and M (x3 , y3 ) = M (O, 1)


2
(ii) as
Do same part (i) [Ans. = 32 sq units]
Lee N (x., Yi ) = N (1, O), P (xv Ji ) = P(l , 2)
2 In each of the following, find the value of k, for
which the points are collinear.
(i) (7, - 2) , ( 5 , 1 ) , ( 3 , k) (ii) (8, 1 ) , (k, - 4) , (2, - 5) A (0, -1 )
Sol. (i) Let A (xt> Ji ) = ( 7, - 2), (x2 , y2 )
A B B= (5, 1) and
CSince,
(x3 , y3 ) = (3, k)
C
the points are collinear.
:. Area of 6.ABC = 0
� -21 l xi ( Ji - y3 ) + X2 ( y3 - Yi ) + X3. <Yi - J2 ) I = O
� 7 (1 - k) + 5 (k + 2) + 3 (-2 -1) =0 C (0, 3) P ( 1 , 2) B (2, 1 )

= = .!..2 1 1 (2 -1) + 1 (1 - 0) + 0 (O - 2) 1
[multiplying by 2] :. Area of ANPM
� 7 - 7k + 5k + 10 -9 =0
� �.
- 2k + 8 = 0 2k = 8 k = 4 ::::>
= .!_2 I Xi (Ji - y3 ) + X2 (y3 - Yi ) + X3 <Yi - '2) 1
c:::
w
(ii) Do same pan (i)
as [Ans. k 3]
::l 3 Find the area of the triangle formed by joining
.J the mid-points of the sides of the triangle whose = -21 I 1 + 1 + 0 I = -22 = 1 sq umt.
) vertices are (O, - 1 ) , (2 , 1 ) and (0, 3) . Find the
ratio of this area to the area of the given . Again, let A (xp,yi ) = A (O, - 1 ), B (x2 , y2 ) = B (2, 1 )
- triangle .
and C (x3 , y3 ) = C (0, 3)
� Sol. Let A (0, -1), B(2, 1) and C (O, 3) be the vertices of a AABC
and M, N and P be the mid-points of A C, AB and BC,
:. Area of ll.ABC
.I = .!..2 l x1 (y2 - y3 ) + X2 (y3 - Yi ) + X3 (Ji - Yl ) I
, respectively.
..
-
:. Coordinate of mid-point of AC,
Coordinate Geometry ; 2s9

= 21 I O (1 - 3 ) + 2 (3 + 1 ) + 0 (- 1 - 1} 1 Here, A (4, - 6) = A (x., y1 ), D (4, 0) = J? (x2 ' Ji )


and C (5, 2) = C (x3 , y3 )
= -21 IO + 8 + O I = 4 sq units Now, �rea of !!:.AD C

[·: area of triangle = � lx1


:. Required ratio = Area of l!:.NPM: Area of MBC = 1 4 :
= .!.2 (4 (0 - 2) + 4 (2 + 6 ) + 5 (- 6 - 0)]

]
4 Find the area of the quadrilateral whose (Ji - y3 ) + x2 (y3 - Y1 )
vertices taken in order are (-4 , -2) , (-3, -5),
(3, - 2) and (2, 3). + X3 (y. - Y2 >I
Sol. Lee A (-4, -2), D (2, 3) =, -1 l-8 + 32 - 301
B (-3, -5), C (3 , - 2) and c (3, -2)
2
D (2 , 3) be the vertices of the = .!.2 x I- 61 = �2 = 3 sq units
quadrilateral ABCD.

[ ]
:.Area of quadrilateral ABCD Area of MED
= Area ofMCD B (-3, -5) = -21 14 (-2 - 0) + 3 (O + 6 ) + 4 (- 6 + 2)1
+ Area of MBC
lee A (4, - � ) = A ( x., y1 ) , B (3 , - 2) = B (x2 ' Ji )
A (-4, -2)

= -I 1 - 4 (-2 -3) + 3 (3 + 2) + 2 (- 2 + 2)1 and D (4, O) = D (x3 , y3 )


2
+-1 1-4 (- 5 + 2) - 3 (- 2 + 2) + 3 (- 2 + 5)1 = -1 l- 8 + 1 8 - 161
2
2
[·: area of triangle = .!. 1x1 (y2 -y3 )
2 •
= .!.2 l -61 = �2 = 3 sq units
+ X2 (y3 Yi > + X3 (yl - Yi> l l
-
Thus, area of !!:.ADC = Area of MBD
= 21 1- 4 (- 5) + 3 (5) + 2 (O) I Hence, the median of a triangle divides ic into cwo triangles
of equa.l areas.
I
+ -4 ( -3(0) + 3(3) 1
2 1 - 3) 6. If P(x, y) is any point on the line joinin g the
points A(a, O) and B (O, b), then show that
= -21 1 20 + 1 5 + o I +' -21 I I 2 - o + 9 1 �a + rb = i . CBSE 2009
= -21 I 3 5 + 2 1 I = -21 x 56 = 28 sq units So l. le is given chat the point P( x, y) lies on che line segment
5 You have studied in Class IX (Chapter 9,
joining points A ( a, O) and B(O, b ).
Example 3 ) that a median of a triangle divides it :. Points P (x, y), A ( a, 0) and B(O, b ) are collinear points.
:. Area of l!:.PAB = 0

[ ·: area of triangle
into two triangles of equal areas. Verify this
=>
result for MBC whose vertices are 1
A (4, - 6) , B (3, - 2) and C (5, 2). 2 1 x(O - b) + a(b - y) + O(y - O) I = O
Given, A ( 4, - 6 ) B (3, - 2)

]
Sol. , A(4,- 6)
and C (5, 2)
According co the question,
AD is the median of MBC, = � l x, (y, - y3 ) + x2 ( y3 - y1 ) + x3 ( y1 - y, ) 1
therefore D is the mid-point 0::
of BC. - xb - ay + ab + 0 = O u..

= ( 3 ; s . - 22+ 2 )
:. Coordinates of D [on multiplying both sides by 2] c

[ ( )]
xb + ay = ab
xb + ay = 1 [on
= (4, O) ab ab dividing both sides by ab] w.
, ,
·: coordinates of mid-point = x, ; x, , y ; y x + y =1
- -

a b He nce proved.
N CE RT FO L D E R 7.4
1 Determine the ratio, in which the line Sol. Let C (x, y) be the centre R(3, 3)
2 x + y - 4 = 0 divides the line segment joining of the circle passing
the points A (2, - 2) and B (3, 7). through the points
Sol. Let the line 2x + y - 4 = 0 divides the line segment joining p (6, 6 ) Q (3,
- , - 7)
and R (3, 3).
the points A ( 2, - 2) and B (3, 7) in the ratio k : 1 at the
point P. Then, PC = QC = CR
[radii of
circle]
0 Now, PC = QC => PC 2 = QC 2 [on squaring both sides]
(2, -2) .. !---..,B (3, 7)
A -k-....P- => (x - 6)2 + (y + 6)2 = (x - 3 ) 2 + (y + 7)2
I
. :::....

)
�(x2 - x1 )2 + (y2 - y1 )2 ]

( 3kk ++ 2
+ (: distance =
c.?\i 1 2x + 36
+ y 2 + 12y + 36
=> x2 -

( "1iX2 ++ mm2x1 , "1iY2 ++ mimlYi ]


= x2 - 6x + 9 + y2 + 1 4y + 49

[
.
:.Coordina tes of P = -- , --
7k - 2
l

J
k+I => - 12x + 6x + 12y - 14y + 72 - 58 = 0
=> - 6x - 2y + 1 4 = 0
·: using sectio n fo rmula = => 3x + y - 7 = 0
m. m.

2 ( 3k + 2 J + 7k - 2 - 4 = 0
[dividing by - 2] . . . (i)
2 and QC = CR
But P lies o n 2x + y - 4 = 0. P
So, coordinat�s of satisfy the
=> QC2 = CR2 [on squaring both sides]
equation 2x + y - 4 = 0
=> (x - 3 )2 + (y + 7) 2 = ( x - 3 )2 + (y - 3 ) 2
k+I k+I => (y + 7) 2 = .(y -3)2
=>
=> 6 k + 4 + 7k - 2 - 4k - 4 = 0 y2 + I4y + 49 = y2 -6y + 9
9k - 2 =0 40
=> 20y + 40 = 0 => y = - = - 2 . . . (ii)
20
9k = 2 On putcing y = - 2 in Eq. (i) , we get
k = -2 3x - 2 - 7 = 0
9 => 3x = 9 => x = 3
So, the point P divides the line segment in the ratio 2 : 9. Hence, the centre of circle is (3, - 2).
2 Find a relation between x and y , if the points 4 The two opposite vertices of a square are (-1. 2)
(x, y) , (L 2) and (7, O) are collinear. and (3, 2). Find the coordinates of the other two
Sol. If points A (x, y), B (I, 2) and C (7, O) are collinear, then vertices.
area of M.BC = 0. Sol. Let PQRM be a square and P ( - 1, 2) and R (3, 2) be the
I vertices. Let the coordinates of Q be (x, y ).
2 l x1 (y2 - y3 ) + x2 (y, - y1 ) + x3 (y1 - Ji ) I =O All sides of a square are equal .
1 PQ = QR
- Ix ( 2 - 0) + 1 (O - y) + 7 (y - 2)1 = O =>
2 PQ2 = QR2 [on squaring both sides]
2x - y + 7y - 1 4 = 0 [multiplying by 2] => (x + 1)2 + (y - 2)2 = (x - 3)2 + (y - 2)2
2x + 6y - 14 = 0 [·: distance = �(x2 - xY + (y - y1 ) 2 ]
x + 3y - 7 = 0 [dividing by 2]
2
=> (x + 1)2 = (x -3) 2
which is the required relation between x and y.
=> x2 + 1 + 2x = x 2 + 9 - 6x
3 Find the centre of a circle passing through the
points (6, - 6} , (3, - 7) and (3, 3) .
2x + 1 = - 6x + 9
Bx = B => x = 1 . . . (i)
Coordinate Geometry
In !iPQR, PQ 2 + QR2 = PR2 [by Pythagoras theorem] Then, area of MQR
2 = ...!.1 . 14 (2 - 5 ) + 3 (5 - 6) + 6 (6 - 2)1

[·: area of riangle k


. => (x + 1) 2 + ( y - 2)2 + (x - 3)2 + (y - 2)
= (3 + 1)2 + (2 - 2)2 2
� lx1 (y - 13 ) + x (y3 - y1 )

J
=
=> x2 + 1 + 2x + y2 + 4 - 4y + x2 + 9 - 6x
2 2
+ y2 + 4 - 4y = 42 + 02 + X3 (yl - hi
··
=> 2x2 + 2y2 + 2x - 6x - 4 y - 4 y + 1 + 4 + 9 + 4 = 16 = .!_ 14 (-3) + 3 (-1) + 6 x 41
=> 2x2 + 2y2 - 4x - By + 2 = 0 2
=> x 2 + y 2 - 2x - 4y + 1 = 0 . . (ii) .
. = .!_ l- 1 2 - 3 + 241 = 2. sq units
[dividing by 2] 2 2
On putting x = 1 from Eq. (i) in Eq. (ii),'we get Case II When Cis taken as origin and CB and CD
as coordinate axes.
l + y2 - 2 - 4y + l = O Then; area pf !iPQR
.
=> y 2 - 4 y = 0 => y (y - 4) = 0 . 1.
· = - 1 12 2> + 1 o c 2 - 6> I
c6 - 3 > + 1 3 (3 -
=> y = 0 or 4 2
Hence, the required vertices of square are (1, O) and (1, 4). 1
= -1 1 2 x 3 + 1 3 x 1 + 1 0 x (- 4)1
5 The Class X students of a secondary school in 2
Krishinagar have been allotted a rectangular = .!_ I 36 + 1 3 - 401 = 2. sq units
plot of land for their gardening activity. 2 2
Sapling of Gulmohar are planted on the Hence, we observe that the area of triangle in both
boundary at a distance of 1 m from each other.
cases is same.
There is a triangular grassy lawn in the plot as 6 The vertices of a MBC are A (4, 6) , B (L 5) and
shown in the figure . C (7, 2). A line is drawn to intersect sides AB and
The students are to sow seeds of flowering AC at D and E respectively, such that
_plants on the remaining area of the plot. AD = AE = .!..Calculate the area of the MDE
.

llr
AB 4AC
.:..i-.+---4--4---4--4--4-....-+-
.. -+-+-+-+-+---.__."-i
, c
t and compare it With the area of MBC.
'

Q,
t · Sol.
AD
Given, - = -1
' AB 4
' t
' t
AB 4
- = - => AD + DB 4
I
=>
'
AD 1 AD
t '
=>
l
+ DB _i
= => DB _i
= - l = � => AD = _!_
A 1 2 3 4 5 6 T 8 9 10 1 1 12 13 14 15 D AD 1 AD 1 1 DB 3
(i) Taking A as 9rigirt, find the coordinates of the So, D divides AB internally in the ratio 1 : 3.
vertices of the triangle. A(4, 6)
(ii) What will be the coordinates of the vertices of
b.PQR, if C is the origin? .
Also, calculate the area of the triangle in these
cases. What do you observe?

When A is taken as origin, AD and AB as

6)
Sol. (t)

(1
3
coordinate axes, i.e. X and Y-axes, respectively.
Coordinates of P, Q and R are respectively (4, 6), B (1 , 5) c (7, 2)
(3, 2) and (6, 5 ) .

[ =(
When C is taken as origin and CB as X-axis and CD as . ofD =
:. Coord1nates +3x 4 1x 5 +3
x 1 x

l])
(it) , -----
· �.:
Y-axis. 1 +3 1+3

( : : ) ( : �)
Then, coordinates of P, Q and R are respectively
( 12, 2), ( 13, 6) and (10, 3). ·: section formula m,, x2 + m2x1 , n�y2 + m�1
111.i + m2 111.i + m!.
·

Area of triangles according to both cases are as follows


Case I When A is taken as origin and AD and AB = 1 12 . s 1 s = 1 .
as coordinate axes.
262 Allinone MATH EMATICS C la ss 1 0th Term I \

(6 ; 5 ; 4 ) = (H)
AE (1) Given, AD is
Similarly,
AC
AE = 1
-- -4 - =-31
EC
=>
:. D
the median of MBC.
is the mid-po inc of BC.
1,

[·: coordinates ( ; y, ; Ji)]


1 : 3.
= ( :1 2 ' 2 :1 8 ) = (1;' 5 )
So, E divides A C internally in the ratio Then , coordin a tes of D =

:. Coordinates of E = ( l x 71 ++ 33 x 4 , l x 2+3
----
x 6)
1 +3 mid- point = x
,
x

[ ]
,,
7
of

P
(ii) Given, divides AD in the ratio 2: 1.
Now, area of MBC :. Coordinates of P

l
= -21 14 (5 - 2) + 1 (2 -6) + 7 (6 - 5)1
[" ·
[
1 - 2x 2!2 ++ 1l x 4 , 2 x-=.-922 ++ 1I x 2
__ ____

2 l x1 ( Yz - J3 )
] ( ++ "'211 )
area of triangle =

+ +
X2 (y3 - Y1 > X3 (yl � Yz ) I using section formula

=P ( 7 ; 4 . 9 ; 2 ) = P (�l · �!)
m!x, TniY2 +
= � l l 2 - 4 + 71 = 1�
= � 14 ( 2: - 5 ) + 1: (5 - 6) + 1; ( 2: )
m1 x 2 ,

m1 1111 + mi
sq 1:1nits =
mi

and area of MDE


6 -

(iii) Given, BE is the median of MBC. So, E is the


= _!_2 1 4 x �4 _ 134 + 4 x _!J41 = 2 1 3 _ 134 161 9 1
!2. ..!._ +
mid-point of AC

:. Coordinates of E = ( 4 ; 2 ; 4 )=( % . 3}
1,

[
=. .!_2 1 48 - 1652 + 1 9 1 = 32 !2_ . -1 units

]
Also,Q divides BE in racio 2 1 :

Area of MDE
= 15/15/322 = l :16
I
:. :. Coordinates of Q
Area o f MBC
Hence, the ratio of area of MDE to the area of ll.ABC = 2 x %2 ++ 1i x 6 , 2 x 23 ++ 1l x 5
= (5; 6 ' 6 ; 5 )= (131 ' 131 J
is
16.
:

7 Let A (4, 2) , B (6, 5) and C (1 , 4) be the vertices of


MB C.
(i) The median from A meets BC at D. Find the
Also, given CF is the median of MBC. So, F is the

= ( 4 ; 6 , 2 ; 5 )=(s. D
coordinates of the point D.
(ii) Find the coordinates of the point P on AD such mid-point of AB.

]
that AP : PD = 2 : 1. :. Coordinates of F

[
(iii) Find the coordinates of points Q and R on
medians BE and CF respectively, such that
BO : OE = 2 : 1 and CR : RF = 2 : 1.
Since, R divides CF in ratio
:. Coordinates of R
2 : 1
(iv) What do you observe?
(v) If A (x1 1 Y1 ) , B (Xi , y2 ) and C ("3 , y3 ) are the = 2x 5 + I x 10 2 x � + I x 4
(103+ I ' 7 ; 4 ) ( 131 ' 131 )
vertices of MBC , then find the coordinates of
the centroid of the triangle.
2+1 2+1
Sol. Given, A ( 4, 2) , (6, 5)
B and C ( 1, 4) be the vertices of

f; , �I}
MBC. = =
A (4 , 2)
I'
,,,,1
(iv) We observe char the points P , Q and R coincide at

I
rhe poin

(v) The poinc, which is common to all the three medians,


is called the cencroid and this point divides each
median in che ratio 2 : 1.
Let A ( x" y1 ), B ( x2, Ji ) and C ( x3 , y3 ) be the
vercices of MBC whose are AD, BE and
medians
B (6 , 5)
Coordinate Geometry f 263 I
+ 4) = (-1, l3 J
[·: coordinates of nu"d pomt· x, +2 X2 , Yi +2 Y2 )J
( x, ; x, y, ; y, } .·. Coordinates of P = (-1-2-.-1'-, -
CF, respectively.ofThen,CA andE and are r�spectively
D, F -1 -
the mid-points AB.
BC, 2 (
4 + 4 J =(2, 4)
:. Coordinates of are D , . - =

.: ofQ = (5--, -1 --
A (X1 , Y1)
:. Coordmates 2 2 J( J
and coordmates of R = -+2-5 , -
. (5 4 -l 5, 3
2- 2 =

R (5 . � )

]
D (5 , -1 ) .---_ _,,,,,.,----.. c (5, 4)

Coordinates ofa point G dividing in the ratio 2 : 1 AD


S (2, -1 ) Q (2, 4)
[l ) + 2 . (xi +2 X3 ) 1 (Ji ) + 2 . (Ji +2 J3 )
_,;:o;:.::;__
(x
( x1 + ; + x' . y, + � + y3 )
I
= 1+2 1+2 A (-1 , -1 ) '----- __ ,
----' B (-1 4)
P(-1 , � )

The coordinates of E are x, x'. ; y, }


and coordinates of = (- -- -1-- lJ = (2, -1)
+5, -
Now, PQ = (2 + 1)2 + ( 4 -%)'
= S
1
( ; y,
2 2
-:-
The
2 : 1 coordinates of a point dividing BE in the ratio
H
,.-------- 2
are [ · di' stance= �(xi -x, ) i + (Ji - Yi ) ]
:

[1( x2 ) + 2 (x, +2 x3 ) , 1 ( 2 ) + 2 . (y, +2 Y3 ) l


. y )' (%) J
+ = 9+�
= 4 = v4
1+2 1+2
= x1 + ; + x' . y, + � + y, )
t6 + 25
QR = �(5 - 2)' + ( % - 4)' ) + -
( [61 = .J61 sq units
2 ' ( %)'
Since,
coordi coordi
n ate nates
of a of G and
point di v i I
d i nareg CFsame,in thesimratiilarloy2the: 1
H
( x1 + x2 + x3 y, + Ji + J3 ) J 25 J36 + 25 {61 .J6l
= �( 2 - )2 + (- 1 - %)' = (-3)' + -
are 3 , 3- �---'- �
= 9 + 4 = 4 = v4 = 2 sq units'
x1 + x2 + x3 , y1 + Y2 + Y ) and medians f a
� 25 �36 25 [61 .J61
(Hence, coordinates of centroid of the MBC is ( %)
= 9 + 4 = 4 ='v4 = 2 sq units
3 ·
RS 5
o
3. 3

and SP = (-1 - 2)2 + % + = (-3)2 + (%)


triangle are concurrent. +

Note When vertices of a triangle are given and we have to find '
the coordi nates of centroid of the triangle, then we use ( )
( ) I

= 9 + 254 36 +4 25 = �4
above result , i . e . Coordinates of centroid of the triangle
x 1 + X 2 + x J Yi + Y2 + YJ er::
l
=

3 3
I
J J {61 = .J6l sq uni ts
= c
8 ABCD is a rectangle formed by _ the points 2 ...
� ..;_-_)
:1

A (- L - 1) , B (- L 4) , C (5, 4) and D (5, - 1). P, Q, R Now, diagonals, ( J


.....---

Given, vertices of rectangle ABCD are A (-1, -1),


and S are the mid-points of AB, BC, CD and DA,

PR = (5 + 1 ) , + l3 - 23 2 = -v G· + O =6sq units •

B (-1, 4), (5, 4) and (5, -1).


i' . · - ··,
respectively. Is the quadrilateral PQRSa square,
a rectangle or a rhombus? Justify your answer. • �- ___.,. _ _ . ..
..��"' - ·
l:::.c-=.. I
Sol. and SQ =�( 2 - 2)i + ( 4 + 1)2 =�0 + 52 =5 sq units I� � l:L
C
P, Q, Rand
D
Thus, PQ = QR = RS = SP and PR SQ * \ _ _ ''\
AlDA,so,respecti vely. Sare the mid-points ofAB, BC, CD and Hence, PQRS is a rhombus.
Allinone MATH EMATICS Class 1 0th Te

( : : J
[·: coordinates= ( of centroid of triangle -J
g Find the coordinates of the centroid of a Then, coordinates of centroid of a triangle

� )
triangle whose vertices are (O, 6) , (8, 12) and x+ +3 y- +6
= 0

( )
( 8 , O) .
(x1 , y1 ), (x2 , Ji )

( : : J
Sol. We know that, when vertices of triangle are
and then coordinates of centroid of a triangle are
(x , y ),
3 3 x, + + x., • y, + + ,,
3 3

( : s . ; ) (1: :) (': )
x1 + Xi + X Yi + Yi + Y3
3,
(3 , 5) +3 y-
- .
x+ +6
=> =
:. The coordinates of centroid of a given triangle
o+ + 6+1 +o ' centroid of given triangle = given]
[·: (3, - 5),
= = . = .6
On comparing coordi nates, we gee
4 and 8 =
x+
3
+3 3 y - +6 -5
1O If centroid of triangle is (3, - 5) and two vertices =
3
y-
of triangle are (4, - 8) and (3, 6) , then find the
third vertex.
=> 7 = and 2 -15
x+ 9 =

Sol. Let coordinates of third vertex be (x, y). . => = 2 and -13 x y=

TO P I C EXERCISE
·Hence, the required coordinates ofchi rd vertex are (2, - 1 3).

Very Short Answer Type Questions


. 8 Show that 2
' the points (at , 2at). (�, ) -2a
and (a, 0
1
t t
Find the a rea of a triangle with vertices
A { 3, 0), B ( 7, 0) and C ( 8. 4). NCERT Exemplar are collinea r for all values of a and t.

2 Find the area (in sq u nits) of the triangle formed 9 ABCD is a paral lelogram with vertices A(xl ' y 1 ),
by joi n i ng the points (0, 0). (0, 2) and (2. 0). B(x2 , y2 ) and C ( x3 , y 3 ). Find the coordinates of
3 Find t he a rea of triange whose vertices are (3. 2), fourth vertex D in term s of x1 , x2 • x3 : y 1 y 2 and y 3 •
(-2, - 3) a n d (2. 3). NCERT Exemplar
4 Show that t he points (0, 0), (1. 1) and (2, 2) are Long Answer Type Questio n s
collinear.
10 I n a MBC, the coordinates o f poi nts A. B and C
Short Answer Type I Questio n s are (3, 2), (6, 4) a nd (9, 3), respectively. Find the .
5 Find t h e area o f t h e quad ri lateral whose vertices coordinates of centroid G. Also, find the areas of
are taken in order a re ( -1. 6 ) . ( 3 - 9), (5, - 8) and
-
,
AABG and AACD. CCE 2013
(3, 9). · 11 If the points A ( 1. - 2), B ( 2. 3), C( - 3, 2) and
6 Find the value of k, so that the line joining (3, 4) D {- 4, - 3) are the vertices of a parallelogram
and (7, 7) passes t h rough { k , 10). ABCD, then taking AB as the base, find the heig ht
of this parallelogram . · CCE 201 3
Short Answer Type I I Questions
12 Find the area of a parallelogra m ABCD, if three of
7 Determine t he value of A. such that the points its vertices are A ( 2, 4), B ( 2 + .J3. 5) and C(2, 6).
( 2/..., 2/... + 2). ( 3. 2/... + 1) a nd ( 1. A. + 1) lies on the CCE 201 3
same line.
- - - - - - -· - - - - - - - - -.--- -.-.- - - · - - -
- · - - -
· · - - - - · - - - - - - - - - · -- - - - - - .
Answers
1 . 8 s q units 2. 2 sq units 3 . 5 sq units 5. 96 sq units 2 and·6. 1 1 7. -l
2
9. (.x; + X3 - X2 , Y1

2 4 umts
+ Y3 - Yz)
1 0. Centroid = (6, 3), ar (M.BG )
2 4
= � sq units and ar (MCD) = � sq units

11 , /1n 1 2. z.13 sq units


v16
• ·
�I Very Short A nswer TVp� Questions [1 Mark e a c h ]

1. Find the perpendicular distance of 1 1. If the points (O, O), (L 2) and (a, b) are collinear,
A (5, 1 2) from the Y-axis. then find the relation between a and b.
NCERT Exemplar
2. If A is a point on Y-axis, whose ordinate is
3 and coordinates of point B is (- 5, 2), 12. Find the radius of the circle whose end p oints
then find the distance AB. of diameter are (24, 1) and (2, 23) . CCE 201 5

3. If the point P (x, y) is equidistant from 1 3. Find the centroid of a triangle whose vertices
L (5, 1) and M ( 1 , 5), then find the
- are (3, - 7), ( - 8 , 6) and (5, 1 0).
relation between x and y. CCE 2014
14. AOBC is a rectangle whose three vertices are
A (0, 3), 0 (O, O) and B (5, O). What are the

( �)
4. Find the distance of the point P (2, 3) from
NCERT Exemplar length of its diagonals? NCERT Exemplar
the X-axis .

5. 1 5.

( i)
If 3, is the mid-point of the line Find the perimeter of a triangle with vertices
(O, 4 ) , (0, O) and (3 , O) . NCERT Exemplar
segment joining the points (k, O) and
[Hint The perimeter of a triangle is the sum of
7, then find the value of k. CCE 2015 lengths of its three sides, so first find the
length of three sides and then �dd them. ]

6. Find the coordinates of a point A, where 1 6. Find the value of a, so that the point (3, a) lies
AB is the diameter of a circle, whose on the line 2x - 3y = 5. CCE 2009
centre is (4, - 3) and B is (2, 6).
1 7.

.
If the point P(2, 1 ) lies on the line segment
7 . Find the point which Ues on the joining the points A (4, 2) and B (8, 4), then
perpendicular bisector of the line find the. relation between AP and AB.
segment joining
. the points A(-2, 5) and -
NCERT Exemplar 1-
B (2, 5). NCERT Exemplar ·, . z
· 1.8 � .If the distance. between the points (4; p) and · UJ
8.· Find the coordinates of the point 'p as
·

( 1 , O) is 5 , th�n find the valµe(s � of p. . ·�


. shown in �he d �agram. CCE 2013 ·
NCERT Exemplar <l1

-
y (f)
u.J

·
1 9. What is the distance between the p oints
0 1 2 3 (1 0 cos 30°, · o) and (O, 1 0 cos 6 0 ° )? (./)
U")
-1
X' .,+- _
--;;;...,_... ..,. _ X
:
-2 ------ p
20. If the point C(k, 4)divides the join of points
c:::(
Y' A(2 , 6) and B(5, 1) in the ratio 2 : 3, find the
9. Find the distance of the . point P ( - 6, 8) value of k? CBSE 20 10
from the origin. NCERT Exemplar
21. If the coordinates of one end of a dia meter of a
10. Find the area of a triangle with vertices circle are (2, 3) and the coordinates of its
( a, b . + c) , (b , c + a) and ( c, a + b). centre are (- 2, 5). Find the coordinates of the
NCERT Exemplar other end of the diameter? CBSE 2009
2661 Allin One MATH EMATICS Class 1 0th Term I :

22. I n the following figure, find the area 23. Find the point on the X-axis which is
of MBC. CBSE 2013 equidistant from points ( - L O) and (5, O) .

24. If the centroid of triangle formed by points


P (� b), 0 (b, c) and R (c, a) is at the origin.
What is the value of (a + b + c)?

25. Find the value of k, for which the distance


between the points A(k, - 5) and B (2, 7) is
13 units .
-2 -1 1 2 3 4 5
Y' 26. [[ors] Find the distance between the points
(a

� I Short A nswer Type I Questions


cos 35° , O) and (O, a cos 55 ° ).

[ 2 Marks each]

2 7. Prove that the points (3, 0) , (6, 4) and (-L 3) 35. Point P divides the line segment joining the
a re the vertices of a right angled isosceles points A(- L 3) and B (9, 8) such that
= !!.. . If P lies on the line x - y + 2 = 0,
AP
triangle. CCE 2016
BP 1
28 . Che ck, whether the points (- 4, O) , (4, O) and then find the value of k.
(0, 3) are the vertices of an isosceles triangle CCE 201 0
or equilateral triangle. 36. Show that the points A ( - 6 , 1 0), B ( - 4 , 6)
NCERT Exemplar, CCE 2014 and C (3, - 8) are collinear, such that
2
2 9. If the points A (4, 3) and B (x, 5) are on the
AB = - AC.
9
NCERT Exemplar

circle with centre 0 (2 , 3), then find the


2 37. Show that A (6, 4) , B (4, - 3) and C (8, - 3 )
value o f x + 5.
are the vertices of an isosceles triangle.
3 0 . T h e x -coordinate o f a point P i s twice its Also, find the length of the m edian through
y-coordinate . I f P is equidistant from A. '
0 (2 , - 5)
If the points (P, q), (M, n) and (P - M, q - n)
and R (-3, 6) , then find the
38.
coordinates o f P.
are collinear. S how that Pn = qM.
3 1 . Show that if a circle has its centre at ' the CBSE 201 0
origin and a point P(5, O) lies on it, then the
p oint Q (6, 8) lies outside the circle. 39. [HOTS] Two vertices of a triangle are (3, - 5)
NCERT Exemplar
and ( - 7, 4). If its centroid is (2, - 1), then
3 2. I f P and Q are the points o f trisection of the find the third vertex.
line segment j oining the points A (2, - 2) and 4 0. !HOTS] Two vertices of triangle are (L 2), (3, 5 )
B (- 7, 4) such that P is nearer to A. Find the
coordinates of P and Q. CCE 201 6 and its centroid is at the origin, then find
the coordinates of the third vertex.
3 3 ° I n what ratio , does the X-axis divide the line
.
s egment j oining the points (2 , - 3) and (5, 6)? 4 1 . [HOTS) Prove that the area of triangle whos e
Also, find the coordinates of the point of vertices are (t, t - 2), (t + 2, t + 2) and
intersection . (t + 3, t), is independent of t.

340 In what ratio, does the Y-axis divide the line 42 . lHOTS) Show that MBC with vertices A (- 2 , Ol
s e gment j oining the points P (- 4, 5) and B (O, 2) and C (2, 0) is similar to ll.DFE with
Q (3, - 7)? Also, find the coordinates of the vertices D ( - 4, 0), E (4, O) and F (0, 4).
point of inters e c tion . NCERT Exemplar
Coordinate Geometry f 267 f
� I Short A ns w er Type I I Q uesti ons [3 M a rks e a c h ]

43. If the point P (x, y ) is equidistant from 51. Show that the points A (5, -
1) , B (B , 3) , C (4, O) and
the
B(a - b, a
points
+ A (a +b, b - a) and
b), then prove that bx = ay.
D (L - 4) are the vertices of a rhombu s .
CCE 2016, 12 CCE 201 5

44. Let the angular points of opposite sides 5 2. If the area of MBC formed by A(x, y), B (1 2) and
of a square be (3, 4) and (1 - 1). Find the C(2, 1) is 6 sq units, then prove that x y = 15 +
'
coordinates of remaining angular or x + y + 9 = 0. CBSE 201 3
points .
53. I n the given figure, i n 8 ABC, D a n d E a r e the
45. Prove that the points A (-3, 0) , B (l , - 3)
and (4, 1) are the vertices of an isosceles mid-points of the sides BC and AC respectively.
right angled triangle. Find the area of Find the length of DE. Prove that DE = .!. AB.
this triangle.
2
CBSE 201 1
C(-6, -1 )
46. Show that the points A(O, 1), B (2, 3) and
C(3, 4) are collinear. CCE 2015

47. The coordinates of one end point of a


diameter of a circle are (4, - 1) and
coordinates of the centre of the circle are 8 (2, -2) A(4, -2)
(L - 3). Find the coordinates of the other
end of the diameter. Also, find the _ 54. Find the area of a parallelogram ABCD, if tl . ee
diameter of the circle. of its vertices are A(2 , 4) , B (2 + .J3, 5) and
C(2 , 6). CBSE 201 3
48. If the point A (2, - 4) is equidistant from
P (3, 8) and Q (- 1 0, y), then find the 55. l!lOTS] If a :F- b :F- c, then prove that the
value of y. Also, find distance PO. points (a, a ), (b, b2 ) and (c, c 2 ) can never be
2
NCERT Exemplar collinear.

49. Show that quadrilateral PQRS formed by 56. (H9� Four points A(6, 3 ) , B (- 3, 5 ) ,
vertices P (22, 5), Q (7, 10), R (12, 1 1) and
C(4 , - 2 ) and D(.x, 3x) are given such that .....
z
= .!., find x.
8DBC
LL.I

(� �)
S (3, 24) is not a par�llelogram.CCE 201 5
MBC 2
50. Find the ratio, i n which the point
2:
57. [HOTS! The vertices of a 8

( - � i)
ABC are A(7, 8) , Cf)
P , divides the line segment B(4, 2) and C(B, 2) . The mid-point of the side B C U1
4 12
is (6, 2) . Show that the median AD divides the u.J
joining the points A , and U1
V1
MBC into two triangles equal in area. Also ,

B (2, - 5). NCERT Exemplar..


fin d the area o f MBC t o verify your answer.
er
CCE 201 5
,,,,,,,
;;;-
r ·· --

-
2681 Allin.one MATH EMATICS Class 10th Term I

� I Long A nswer Type Questions [ 4 Marks 11ch

58. The vertices of MBC are A (-2, 0) , B (2, O) parallelogram ABCD, then taking AB as the
base, find the height of the parallelogram.
and C (O, 2) and that of �PQR are CBSE 2013
P (-4, 0) , 0 (4, O) and R (0, 4). Verify that
0 (-4 , -r
3..) __________ -----;
,',
the ratio of the areas of the two triangles '
I
I
is equal to the square of the ratio of their :h ......
I
I
' ,'
,' ,
corresponding sides . CCE 2015 '
I
I

'
'

,,
I
I
59 . I f P (2, - 1) , 0 (3 , 4) , R (- 2, 3) and
(1 , -2) A M
S - 3, - 2 are four points in a plane,
B (2. 3)

then show that PQRS is a rhombus but 63. Observe the graph g iv e n below and state
not a square. Find the area of the whether MBC is s calene, isosceles or
rhombus. equilateral. Justify you r answer. Also, find the
60.
area of MBC. CCE 2015
y
A (6, 1), B (8, 2) and C (9, 4) are three

(-4, 6) B e-
vertices of a parallelogram ABCD. If E is ------

the mid-point of D C, then find the area of


MDE. NCERT Exemplar

61. The coordinates of A, B, C are (7, 4) , (-2, X'·.---�-+--4---i-++-+-�....._�--t-t---+-+---+- X


-1 0 -5 5 10
A

point (x,of S how


6) and {S, - 1) , respectively and P is any
y). that the ratio of the
( 1 . -1 )

( �' n G' fl
areas �PBC and �ABC is

l x + ; - 4 1·
Y'

64. !HOTS) If D - E (7, 3 ) and F


62. I f the points A{ l , - 2) , B{2 , 3 ) , C(- 3, 2) are the mid-points of sides of MBC, then find
D(- 4 , - 3)

� I Va l ue Based Questi ons (VBQs )


and are the vertices of the area of MBC.

(4 Marks each ]

65 . To rai s e s ocial awar.e n�ss a bout 66. In a sports day celebration, Pushpraj and
hazards of smoki n g , . a s chool de�ided Shani are standing at positions A and B whose
. c � ordinates are (2 , -2 ) and .{ 4 , 8) , respectively.
to s ta rt " N O S MO KING" campaign . °
1 0 students a re asked t o prepare Th e teacher asked Deepanshi to fix the country
campaig n b a nn e rs in the shape of flag at the mid-point of the line j oining poin ts
triangle ( a s s hown i n the figure) . A and B .
n
(i) If cost of 1 cm 2 of banner is � 2, then (i) Find the coordinates of the mid-point.
n
� find the overall cost incurred on such (ii) Which mathematical concept is used to
campaig n . solve the question?
(ii ) Which mathematical concept is used (iii) What type of value is d epicted here?
in this question?
(iii) Which value is depicted in this
question?
Given points are (k, 0) and ( 7. % )
Sol u ti o ns
-s-
1.
.... : A (S, 1 2)
y 5.
12 �

10
8 �
Now, coordinates of mid-point
6 :
l 0
4
= [k ; 7 . : %]

= (� 2 , �4 )
2 l
X-
' ��--------....-..._�-X
- 4 - 20 2 4 6

["· coordinates of mid-point = ( ; xz , Yi ; Yz )]


-2
Y'
Itfromis cltheearY-axis figure, perpendicular distance of A
from isthe5 units. [1 ]

But coor<Jinates of mid-point are given (3, �)


x,
2. Here,
its ordinatepoint iAs 3.liesSo,ontheY-axis,
pointsoA ionts absci
Y-axisssaisisA(O,zero3).and

=( ; �
Here, A = (x1, y1 )= (0, 3) [1 /2]

H
. . AB = �(x - x1)2 + (Ji - y1)2
and B = (xi ,i Yi ) = (-5, 2) [1 /2)
J k 7 J
[by distance formula] .

= �-(--5---0-)-2-+-(2---3)-2 On comparing x-coordi k 7nate both sides, we get


= �(-5)2 + (- 1)2 3= 2 +
= J25+l => k+7 =6
= � units => k= -1 [1 /2]
Hence, the distance between A and B is .J26 units. 6. �
Do same Example 6 at page 249.
[1 /2) 7. We knowdithat,
segment v i d es thetheperpendi
line cularinto
segment bisector
two of anyparts
equal line
3. Given points are (x y), L(5, l)andM(-1, 5).
P ,

i. e. the passes
perpendithroughcular bithesectormid-point
of the liofne thesegment
Since, is equidistant from L and M.
..
P
PL = PM always line
(PL) i = ( )2 [on squaring both sides] segment. [ 1 /2]

( �(x - 5 )2 + (y - 1)2 r
=> PM :.Mid-point of the line segment joining the points
=> -5 + 5 J
A( -2, - 5) and B (2, 5) = ( -2 ; 2 ,

[ ·: by distance formula, distance [·: coordinates of mid-point = ( x, ; Xz Yi ; Yz )]


2
= ( �(x + 1)2 + (y - 5)2 )2 = (O, O)

=> (x - 5)2 + (y - 1)2 = (x + 1)2 + (y - 5)2


= ( �(x, - xz )z + (y, - Yz l' )] 8. From the given figure, from [1/2]
x = Perpendicular distance Y-axis = 3

[·: (a + b)2 = a2 + b2 + 2ab and


[ 1 /2]
=> x2 + 25 - 10x + y2 + 1 - 2y
·
y = Perpendicular distance from X-axis = -2

= x2 + 1 + 2x + y2 + 25 - 10y Hence, the coordinates of the point is (3, - ). [1]


P 2
9. Here, (x1, y1 ) = (- 6, 8) and (x2 , y2) = (O, O)
(a - b)2 = a2 + b2 - 2ab] [·.· coordinates of origin = (0, O)J
=> -10x - 2x - 2y +,J Oy + 25 + 1 - 1 - 25 = o :.Distance from the origin = �(O + 6)2 + (0 - 8) 2
=> -12x + 8y = 0 [by distance formula, d istance
- x1 > 2 + (Ji

= .JlOO = un�rs
=> -12x = -By = �(x2 Y1)21 -

which is the required rela3xtion=betweerixan


=> 2y,
dy. [1/2] = �(6)2 + (-8 ) 2 = $+64
4. Do same as Q. 1 [Ans. 3] 10 [1]
Alllnone MATH EMATICS C lass 1 0th Term I

1 0. Here, ( xp y1 ) = (a, b + c), (x2 , y2 ) = (b, c + a) 1 4. In a rectangle, both


and (x3 , y3 ) = (c, a + b) diagonals are same .
Now, area of a triangle So, length of the diagonal
AB
= -1 (X1J2 + X2J3 + X3J1 - X1J3 - X2J1 - X3J2 ] = Distance between the
2 points A(O, 3) and B(5, 0)
[1 /2]
Here, x1 = 0, y1 = 3,

-
= -1 [a(c + + b(a + b) + c(b + c)
a) [1'2]
2 x2 = 5 and Yi = 0

- -
-a(a + b) - b(b + c) - c(c + a)]
1
:. By distance formula,
= [ ac + a 2 + ba + b2 + cb + c 2
2 AB = �(x2 - X1 ) 2 + ( Ji - Y1 ) 2
- a 2 - ab b 2 - be - c 2 - ca] = �(5 - 0) 2 + (0 3 ) 2
1
= �(5)2 + ( -3 ) 2
= - (O) = 0
2
Hence, the area of a triangle with vertices (a, b + ), c

(b, c + a) and (c, a +· b) is zero. [1 /2] = .J2s + 9


11. If(O, O), (1, 2) and (a, b) are collinear, then = .J34 units
area of triangle formed by these points = 0 Hence, the length of each diagonal is .J34 units. [ 1/2)
� .!_ [0( 2 - b) + l(b - O) + a(O - 2) ] = 0 1 S.

-
It is clear from the figure that,
2 Perimeter of MOB

[� ��,::��;:�. ]
= Distance (AO) + Distance (OB) + Distance (AB)
+ x,{y3 - y, ) + X3 (}1 J2 )1 (0, 4) A
y

� b - 2a = 0
2a = b, which is the required relation. [1 ]
1 2. Given end points of diameter of a circle are
(xp y1 ) = (24, 1) and (x , y ) = (2, 23). - -- -
(0, 0) 0
X'-- -+- -- ---X
Now, diameter of a circle,2 2 (3, 0) B

-
d
Y'
= �(2 - 24)2 + (23 - 1)2
[by dista11ce formula, distance = �(O - 0)2 + (4 - 0)2 + �(O - 3)2 + (O - 0)2
= �(x2 - x1 )2 + (J2 J1 )2 ] + �(O - 3)2 + (4 - 0)2
[1 /2] ( 1 /2)
[by distance formula, distance
= �(-22)2 + (22)2 _____--_ _

-
= �(22)2(1 + 1) = �,...( X. 1 _ X2 )2 + (Jl Ji )_2)
___
J = 22.Jl+i = 22../2 units = �(4)2 + �(3 ) 2 + .J9 + 16

- 6)
. l e, r = -d = --
> � .
. = 4 + 3 + .J2s
:. Ra dius o f a c1rc 22J2 = l lv2 units
, 2 2 [1 /2]
=4+3+5

-
j ( 3
= 12 units
, v J2 (

( : 36 )
.J 1 3. Hcre, ( x1 , y1 ) = - 7), (x ) = 8 ,
Hence, the perimeter of given triangle is 1 2 units. [1 /2)
1 6. Given, (3, a) lies on the line 2 x - 3 y = 5.

- 6
, and (x3 , y3 ) = (5, 1 0)
, : . Coordinates of the centroid of a triangle Then, 2(3) 3(a) = 5
(

( � )]
= 3 - + 5 ' - 7 + + I 0 =(O, 3) => -3a = 5 -
' ff'� => - 3a = 1
[ ·.· coordinates of centroid of triangle
;
a = -1

iI
=>
3
=
XI + + X3 ' }I + } + }3
Hence, the value ofa is .!... (1]
3
[1]
Coordinate Geom etry

(- 2, S) - ( x 2+ 2 , y 2+ 3 )
1 7. P(2 , 1 ) ------- 8 (8, 4) 21. Lee A(x, y) be che other end of a diameter. Ir is given
A (4, 2)
char centre is ( - 2, 5) and one end of diameter is 2, 3 ).
[·: by distance= �(formulx -a, di):cance+ (
(
Here, AP = �(2 - 4) 2 + (1 - 2)2
2 Xi
= �(- 2 ) 2 + (- 1)2
Yi - Yi )2 J [·: centre of a circle is the mid-point of the diameters]
:. Coordmates . of m1'd-pomt. = ( xi + X2 , Yi +2 Y2 J
2
= J4+l = .JS unirs On compari+ng2coordinates, wey gee+ 3
2 = - 2 and 2 = 5
x

AB = �(8 - 4)2 + (4 - 2)2 = �(4)2 + (2)2 => x + 2 = - 4 and y =+ 73 = 10


= .J16 + 4 = J20 = 2.JS units [1 /2] => x = - 6 andy (1 ]
From thenatefigofure,A = (x , Y ) = 3)
1 i
22.
On putting JS = AP from Eq. (i) and Eq. (ii) , we get Coordi (I,
AB = 2.JS = 2AP Coordi
and coordi n ate
n of
ate Bof=C = 2 i ) = ( -1, O)4,
(x , J
y3 ) = ( 0 ) [1 /2]
AP = AB2 :. Area of MBC (x3 ,

= �lxi(Y2 - + - y1 ) +x3 (Y - Y2 > I


[1 /2]
Do same as Q. 4 at page 265. [ An s. ± 4] i

Here, Xi = 10 cos 30°, Yi = 0 and x2 = 0,


1 8. J3 ) x2 (J3
1 9.
Yi = 10 cos 60°. = .!.2 . 1 1(0 -0) + (-1 ) (0 - 3) + 4(3 - O) j
:.Distance berween the points = .!_2 I O + 3 + 12 I = _!_2 I 5 I = !22 = 7.5 sq units[1/2]
t
= �(O - 10 cos 30° )2 + 0 cos 60°-0)2
(I
[ ·: by distance formul a, distance Do same as 3.
= �(X2 - Xi >2 + (J2 - Yi )2 ]
23. Q.

P(x, O).Theuse PAcoordi= PB] nates of points P on X-axi P (2,s O)are]


= (10), ( ¥r + (10),G Centroid of Ll = ( i X23 X3 Yi + Y32 + )
[Hint
[1 /2] [Ans.
J 24. We know char, x + +
· J3

= - 300 + -100 Gi.ven, (0, O) = ( a + 3b + c , b + 3c + a J


4 4[ �
·: cos 30°= and cos 60°= �]
= �4�0 = ,JWo = JO units
On comparia +ngbx-coordi + c = Onaces both sides, we get
3
(1/2]
=> a +b +c =O [1 ]
20.
��
25. Do same s . 4. a Q = 7, -3]
[Ans. k

4 ) = ( 3 22 ++ 32 5 , 3 26 ++ 32 1 )
B (5, 1 )
26. Let the points be A(a cos 35°, 0) and
(
B O cos 55° ).
A (2, 6) c (k, 4)
, a
Using section formula,
( k,
x x x x
:. I AB I = )�(O__ _ a__co_s_3_5°-)-2-+-(a-co_s_5_5_0____0 )_2
,�-
., �-2����, ---"-�
, -
[by distance formula]
= ( 6 � 10 . 1 s ; 2 ) = (': �o )
(k, 4) = c: ,4 )

= -ya- cos 35° + a- cos- 55
.,
= -ya-I ., [cos- (90° - 55°) + cos-., 55)
=> = �a2 (sin 2 55° cos2 55)0 -
+
8) == si n
On comparing16x-coordinace both sides, we gee
[·: cos (90 cos2 e e111 =
· (I )
= �a 2 =.f:? [· : s i n e +
.2
k= ��;��'. ·
\'
[1 ]
=a [1 ]
5 ti .
I..."':>
212 1 Alllnone MATHEMATICS Class 1 0th Term I

27. Given, vertices of a triangle are A (3, 0), B (6, 4) and 4 = (x - 2)2 + 4 � (x - 2) 2 =0
c
Dis(-1,
tance3).between A and B, � x - 2 = 0 => x = 2
x2 + 5 2 2 + 5 9
[11'2)

[·: by distance= �(xformula,- )idi+stance( - )i


= = uni cs
AB = �(6 - 3)i + (4 - O)i x2 + 5 9
Hence, the value of is uni cs.
Ji Ji ]
(11'2)
30. Do same Q. 4 as

i Xi [Hint Lety-coordinaces of P = 11
= �(3)i + (4)i = .J9 + 16 = J25 [1/2) Then, x-coordinacc of P = 211

Here, OP = �(5 - 0)2 + (0 - 0)2


= 5 units :. Coordinates of P = (211, 11)
Now, BC = �(-1 - 6)i + (3 - 4)i .. PQ = PR] [Ans. (1 6, 8))

= �(-7) 2 + (-1)2 = .J49 + 1 31 .

= .J50 units = 5.Ji units [1/2) [·: by discancc formula, distance


= J(.\'2 - X1 )2 + (Ji - Ji )2 )
Now, OQ = J<6 - 0)2 + (8 - 0)2

and CA =· (3 + l)i + (O - 3)i
= �( 4) 2 + (-3) 2 = .J16+9 = N = 5 unirs =Radius of circle [1/'2)
= .fi5 = 5 units [1 /2]
Here, AB = CA = �(6)2 + (8)2 = .J36 + 64 [1)
and AB 2 + CA 2 = BC 2 � 25 + 25 = 50 = Mo= 1 0 units
So, BCLAis an=is90°
Hence,
angled atMA. osceles right angled triangle, ri[1/2)ght Hence,OQQ(6,> 8)OPlies outsiY de the circle.
Hence proved. __-,,_�Q (6, 8)

28. Let A = (xp Yi ) = (- 4, O), B = (x2 , y2 ) = (4, O)

[..· by distance formula, distance


and C = (x3 , y3 ) = (O, 3) P (S, 0)
x-·----- x
Now, AB = �,...(4- ... --(--4)-]2-- + (0--
- 0)-i

= �(X2 - Xi )i + (J2 - Ji )2
= �(4 + 4}2 .JB2 = = 8 units
] 32.
Y' [1/2]
Given, - 2) and B (-7, 4) be the end points ofline
( 2,
Hence proved.

segment AB and poincs of trisection of the line


·
BC �( - 4) 2 + (3 - 0)2 = �....-(--4-)2_+__
= O (3_
1� 1 segment be PAPand=Q.PQ = BQ = k

-:.
= .J16 + 9 Then,
Now, PB = PQ + QB = k + k = 2k [say]
= .fi5 5 = units
J
k k k
and AC = �[O
- (- 4) ] 2 + (3 - 0) 2 (2 , -2) (-7, 4)
= �(4) 2 + (3) 2 A P 0 B

= .J16 + 9 = .[i5 = 5 units and AQ =APAP: PB+ =PQk : =2kk =+ 1k: 2= 2k


BC = AC
MBC
So, is an isosceles triangle.
A( 4, 3) B(x, 5)
[1] Since, P diviAQ
and des :ABQBincernall
= k : 2 ky in= the2 : 1 ratio I : 2.
[1/2]

- = [ 2 21++1 2 (-7) , 2 (-2)1 + +2 1 4 ]


29. Since, and lie on the circle having centre 1 2
0(2, 3). (2, -2) A p B (-7, 4)
.. OA =OB [radii ofcircle] [1/2] So, the coordinates of P
=> �( 4 - 2)2 + (3 - 3)2 = �(x - 2)2 + (5 3)2

I.e.. ( m1m1x2 ++ mm2x1 m1Jm12 ++ )


x x x x

l

c
[·.· by distance formula distance
J

= �(Xi - Xi ) 2 + (Ji - J1 )i
[·: using section formula
miJ1


== m2


• -� J4 �,..
(x---- 2 + -4
2 )- [1/2) 2
) On squaring both sides, we get
\
Coo rdinate Geometry

= ( 4 ; 7 . -\+ 4 ) = c: ,0 ) = (-1,0) [1 ]
37. Given vercices of a triangle are A(6,4),
C(8,-3).
B(4,-3) and
·: Q divides AB internally in the ratio 2 : 1.

�C
2 1
A (2. -2) Q B (-7, 4)
So, the coordinates of Q
= [ 2 x (-7)1 + +2 1 x 2 , 2 x 4 1++l 2x (-2) ] B( (S , - 3 )

= ( -14/ 2 . 8 ; 2 ) [
We have, AB = �(6 - 4) 2 :+ (4 + 3) 2
·: by discance formula, distance ])2
Now,AC = �(6 - 8) 2 + (4 + 3) 2 = �(-2) 2 + ( 7 ) 2
= �(XI - X2 )2 + (JI - J2
= ( --123-, 63 ) = (-4, 2) [1/2] = �(2 )2 + (7)2 = .J4 + 49 = J53 units -----

So, the two points of triscccion are (-1, O) and (-4, 2).
33. Do same as Q. 5 ofNCERT Folder 7 . 2 at page 252. = .J4 + 49 =.J53 units
[Ans. 1 : 2; (3, O)] and BC = �(4 - 8) 2 + (- 3 + 3) 2
34. Do same as Example 4 at page 249. = �(-4)2 + 0 = 4 units
[Ans. 4 , 3; R ( - 1:)]
[Ans. = �]
o. Here, AB = AC
So, LiABC is an isosceles triangle. (1 ]
Lee

[·: ]
35. Do same as Q. 39. k
D be che mid-point of BC.
Then, the coordinates of D
= ( 4 ; 8 . -3 2- 3 ) =(6, - 3)

J
36. Here, coordinates of A (x. , y1 ) = (-6, 10),
=

Coordinaccs of B (x2 , y2 ) = (-4, 6 ) and


=
coordinaces of mid-point
( x1 ; Y1 +2 y_�,
Coordinates of C (x3, y3 ) = (3, -
= 8).
We know chat, = Xi •

Area. of triangle = .!_2 I x1 (y2 - J3 ) + x2 (J3 - Yi )


+ X3 (y. - Y2 > I Now, AD = �-(6---6-)- 2 +-(4-+-3)-2= �O + (7)2
:. Area of MBC = .!.2 . 1 - 6 {6 - (- 8)} + (- 4)
= 7 units
Hence, length of the median is 7 uni cs. AD [1 1
- 10) + 3(10 - 6) 1
(- 8 38. Let coordinates of A (xp y1 ) = (P, q),
=

= ..!..2 1- 6(14) + 18) + 3( 4> 1


< - 4)(- Coordinates of B (x2 , Ji) (M, n) and
= =

Coordinates of C (x3 , y3 ) = (P - M, - n). q


= .!_2 1- 84 + + 121 = o
= .

n r1 1 Since, A , B, C are collinear points. [1 /2)


:. Area of LiABC = 0
Since, area of MBC is zero. So, points A , B and C are
2 l x1 (J2 - J3 ) + x2 ( J3 - Y1 )
I
collinear. + - = o
Now, AB = �(- 4 + 6) 2 + (6 - 10) 2 P(n - q + n) +MM) (q(q -- n)-=q)0 [dividiYi > I ·"·' < 11
=> n [1 /2]
[·: by distance formula, distance (P - ng by
= �(X2 - X1 )2 + - Yt )2 ] Mq + Mn 2]
P(2n -qP+q) -- MMnn ++ PqPq -- nPllP -- Mq
l -.�
< 12 => = o
2nP -
=> + Mn = o
= �2 2 + (-4)2 = .[4+16 = .J20 => nP - Mq = o
= 2.JS units => nP = Mq

[·: = x1 +
Hence proved. [ 1 ]
39. Let the coordinates of the chi rd vertex be ( x, y )
AC = �(3 + 6) 2 + ( - - 10) 2 6

= ( -� + � - 7 , ; 4)
.

Then, (2, - \) Y -
= .J92 + (-18)2 = �81 + 324

J
+

= ,J405 = � = 9.JS uni cs coordinates of cenrro id of A,

AB = 2../5 x 9" = 92 AC
9 Hence proved. [1 ] l!i.
( ;· i + x3 • y1 + .v; + _v, )
274 f I
-
• Allln' one MATH EMATICS Class 1oth Term 1

3 x - 4 = and 3 - I --31 [1]


On comparing coordinates, weyget- + 4 :. MBC !:i DFE [by SSS criterion]
5
x + 3 - 7 = 2 and

LS
==

-
:::) 6 y ==

:::) x = IO and y = - 2
Thus, coordinates ofthe third vertex are (10, - 2). [1] A (

Given points are P(:c, y). + b, b -


40. DosameasQ.39. [Ans.( - 4, - 7)] 2, 0) c (2, 0) D (- 4. 0) E (4, 0)
(1 /2)
=
Let coordinates ofA (xp Yi ) = (t, t - 2)
41 .
Hence, both the triangles arc similar. Hence proved.
a) and
=
Coordinates of B (x2(x, J2, ) =) (t= + 2, t + 2) 43. A ( 11
=
and coordinates of C 1 3 Y3 (t + 3, t ).
-
B(a - b, + b). 11

=
.. Area of MBC = -2 l xi (Yi - Y3 ) + x2 (Y3 - Yi ) Since, is equidistantPA from
..
P
= PB
A and B.
+ X3 ( Ji - Y2 > l
� �[x - (a + b)] 2 + [y (b - 11)] 2

=
.!... l t(t + 2 - t) + (t + 2) {t - (t - 2) }

-
2 = �[x - (a - b)]2 + [y - (a + b)]2
+ (t + 3 ) {(t - 2) - (t + 2) }! [1] [·: using distance formula, distance
.!_ \2t + (t + 2) (2) + (t + (- 4)1. 3)
J2)l )

[x - (a + b )] + [y - (b -
2 = �( Xi - X2 )2 + (JI
= -1 \2t + 2t + 4 - 4t - 12\ ( 1)
2 On squaring both sides, we get 2
.!...2 1- s \ = .!...2 x s a)]2

- 2y (b - a) + (b -
= =4

= x 2 - 2x (a - b) + (a - b)2 + y 2
. Area of MBC = 4 sq units = [x - - b )] 2 + [y - (a
(a + b)] 2
:Hence, � x - 2x (a + b) + (a + b ) 2 + y 2
2
a) 2
area of MBC is independentHence
oft. proved. [1]
42. Given, vertices of MBC arc A (-2, 0), B(O, 2)

- xa
and C(2, 0) and l:iDFE are D(-4, 0), E(4, O) and - 2y (a + b) + (a + b) 2 (1 )
F(O, 4) . => -2x(a + b) + 2x(a - b)

Now, for MBC, AB = �(O + 2)2 + (2 - 0)2 = - 2y (a + b) + 2y (b - a)


� - xa
xb + - xb = - ya - yb
= .J4+4 = 2.J2 units + yb - ya [dividing by(1]2]
[·: using distance formula, distance � - 2bx = - 2ay => bx = ay [dividingby2]
=
= �(x2 - xi ) 2 + (y2 - J1 ) 2 ]

[An·-G· � } (- H)]
Hence proved.
BC = �(2 - 0) 2 + (0 - 2)2 44. Do same as Q. 4 ofNCERT folder 7.4.
2
.J4+4 � 2.J2 units [1/2)
J.J CA = �(- 2 - 2) 2 + (O - 0) 2

=
� = �(- 4) 2 + 0 = 4 units 45. Do same as Q. 27.
.11 Now, area of MBC = _!_2 x Base Height
) } sq units [1]
x
f) F o r l:iDFE, we have
,J.J = -1 x AB x BC
.
(0_ + 4-)2_+_(4- --0-
DF �,... ._ _ )2
.n
=
2
/) �( 4) 2 + (4)2 = 4.fi. (� 2

units (1/2) = x5x5 =
·.·.-!'
FE = �( 4 - 0) 2 + (O - 4) 2
46. Here, A(x. , Yi ) = A(O, 1), B(x2 , Y2 ) = B(2, 3) and
= �( 4 ) 2 + (-4 ) 2 = 4./2 units C(x3 ; y3 ) = C(3, 4)
and ED = �( - 4 4) 2 + (O 0)2 _ _ Condition for three points to be collinear,
= �( -8) 2 = ./64 = 8 units Area of MBC = 0
Now, area of MBC
- 3) 1
Here, we see rhat, (1/2]
AB BC = -
CA = .!.. l x1 CY2 - y3 ) + X2 ( Y3 - Yi ) + x3 (Y1 - y2 ) I
-
DF
=-
FE ED
=-
2
2 [1 ]
= .!... 10(3 - 4) + 2(4 - 1) + 3(1
2
Coordi nate Geom etry f 21s f

= .!.. 1 0 + 2(3) + 3(-2) 1 [1 1


49. Giv2en vertices ofa quadrilateral are P(22, 5 ), Q(7, 1 0),
2 R(1 , 1 1) and 5(3, 24).
= .!_ l6 - 6 l = _l_2 x 0 = 0
2
Now, PQ = �,_..(7---2-2_) 2_+_(1_0____5 )_2
Since, =0 [·: by distance formula, distance
Hence,areapointsof 6..A.ABC
Band C are collinear.Hence proved. [1 ] = �( X2 - Xi ) 2 + (y2 - Yi ) 2 J
47. Do same as Q. of NCERT Folder 7.2.
7
( --
= �,... . 1- 5)-2 = �225 + 25
5)_2_+_(_
[Hint Now, diameter of circle = Distance between = .J2SO = 5M units
(4, - 1) and ( -2, - 5 ))
QR = �(12 - 7) 2 + (1 1 - 10) 2 = �(5) 2 + (1) 2
= �(4 + 2) 2 + (-1 + 5) 2
[·: using distance formula) = �25 + 1 = .J26 units [1 ]
= .JG2 + 4 2 = .J36 + 16 .J52 = 2.J13 units [1 ]
= 2
RS = �(3 - 12) + (24 - 1 1) 2 = �(-9) 2 + (1 3 ) 2
[Ans. (-2, - 5), 2.J13 units] = .Js1 + 169 = .J2SO = 5.Jlo units
48. Given points arc A(2, - 4), P(3, 8) and Q(-10, y). and SP = �(22 - 3) 2 + (5 - 24) 2
According to the question, PA = QA = �(19) 2 + (-19) 2 = 1 9.JI+l = 1 9.Ji. units[ 1
=> �( 2 - 3) 2 + (-4 - 8) 2 1

= �(2 + 10) 2 + (-4 - y) 2


Here, we see that
not equal i.e. QR SP. opposi
-:¢:
t e si d es of a quadri l ateral are
[·: by distance formula, parallelogigram.
Hence, ven vertices of a quadrilateralHence formed[1a]
are notproved.
distance = �(x2 - Xi )2 + (y2 - Yi )2 ] 50. Do same as Q. 4 at page 255. [Ans. 5] I :

=> �( -1) 2 + (-12) 2 =�(12) 2 + (4 + y) 2 51 . Do same Q. 8 of NCERT Folder 7.4 at page 263 .
as

=> ..J1 + 1 44 = �144 + 16 + y 2 + By 52. Given, A(x, y), B(l, 2) and are the vertices of
C ( 2, I )
MBC .
=> .Jl45 = �160 + y 2 + Sy [1 ]
We know that,
On squaring both sides, we145get= 160 + l + 8y Area of M.BC
=> y 2 + 8 y + 160 - 14 5 0 = = -I l xi (Y2 - y3 ) + x2 (Y3 - Yi ) + x3 (Yi - Y2 )I
2 [1 1
y 2 + By + 1 5 = 0 :.Area of M.BC 2 1
= - l x(2 - I ) + 1(1 - y)
=> y2 + 5 y + 3y + 1 5 = 0 [ by factorisation] + 2(y - 2 )1
=> y(y + 5) + 3(y + 5) = 0 ( x1 ,
= (
[here, Ji ) (x, y); x2 , (xy2 ,) = )(1,= 2)(2, an d
=> (y + 5) (y + 3) = 0 3 J3 I )]

=> y+5 =0 y+3 =0 or [1 ] => Area of = -21 I x + - y + 2y - 41


MBC I
=> y = -5 y = -3
or
Now, PQ = �(-10 - 3) 2 + (y - 8) 2 => Area of aABC = -I I x + y - 31 sq units
2
Fory = -5, PQ = �(-13)2 + (-5 - 8)2 Given, area of M.BC = 6 sq units
x
.!. 1 + y - 31 = 6
=
= �(-13)2 + (-13) 2 2
= .J169 + 169 = .J338 units => x + y - 3 ± 12 [1 ]
x + y - 3 = 12 x + y - 3 = - 12
=>
= anandd =
= and
andfory = -3, PQ = �(-13)2 + (-3 - 8) 2 => x + y 15 x + y - 9
x + Y 15 x + y + 9 = o f1 l
. = �( -13) 2 + (-1 1) 2
H ence p ro d .

:. Coordinates of D and (2� , -


= .J169 + 121
v e
Given Dveland E and of BC AC,
= .J29o units 53.
respecti y. are the mid-poin-cs6
y -5, -3
Hence, the values of are and distance, - 2- --l \)
PQ = .J338 .J290
units and units. E are D
[1 ] 2
1 276 All{none MATH EMATI CS C lass 10th Term I

2 2 )
4 --6 , - 2 - 1 respectively
an d ....J. � - We know chac, area of �A BC
)'J ) +
= � lx1 (J12 - x2 (y, - )11 > + X3 (J1 - JiM
C (-6, - 1 )
1 , , ., 2
:. Area of MBC = - I a( b - c - ) + b(c-

= 2 I a<b - Hb + + b 2
- - a )
2
+ c(a2 b2)1(1) -

d
1

= ( X1 +2 Xi J1 : Ji J]
- c c

[
B ( 2 , -2) A (4, -2) - ba 2 + ca2 - cb2I
[·: 2 - B 2 = B (A - )( A +

= .!_l2 a(b - c)(b + c) - bc(b - - a 2 (b - dl


. A B)]

or D( - 2, =f) and � - I, =f) respec<ively.


·: coordmates o f "d -pomt
nu· •

- c)(b + c) + be-.., - cb 2 + ca 2 - a 21
"I1
I
= -la(b

= .!.2 1 (b - c)[ab + ac - be - 11 2 1
[1 ] 2
c)

Now, DE = �( - I + 2) 2 + ( -23 + % )
= _!_2 l (b - c)[b(a - c ) - a(a - c >I
'

= .!.2 . l<b - c Hb - a > <,, - c >I


l11
[ ·: by distance formula, d
= �(xi - X1 )i + (Ji - Y1 )i ]
�------�

= .Jl+o = unit I [1 ]
= _!_l2 (b - c)(c - a)( a - b)j
and AB = �(2 - 4) i + (- 2 + 2) i
= ..[4"+0 = 2 units It is given that a b c.
=t: =t:
[1 /2]
:. Area of MBC =t: 0
DE = -2I AB Hence, the given points can never collinear.

=D B(-(x4, 3,y45),) =CD(x,


Hence proved. [1 /2]
Hence proved. [1)
54. Given, three vertices of the parallelogram ABCD are 56. Given, A (x1 , y1 ) = A(6, 3), B (x-2 ,2)Jiand)
A (2, 4), B(2 + -/3, 5 ) and C( 2, 6). J3 ) = C(4,
(x3 , 3x).
Area of� = _!_ l [x1 ( Ji - J3 ) + x2 ( J3 - Y1 )
We know that,

:. Area of MBC = _!_ 1 (6(5 + 2) + (- 3)(- 2


2
+ X3 (JI Ji)JI -

3)

= .!.2 1 [6 7 - 3 < - 5 > + 4 x c-2>1 1


A (2, 4)
-
B (2 + "3, 5)

Join AC. We know that diagonal of the parallelogram 2


- - r divides it in two triangles of equal area. [1 )
+ 4(3 5)JI -

;2 Area of parallelogram ABCD = 2(area of MBC ) x x

�fi
:. Area of parallelogram ABCD
.•f j
:: �.l
= 2 x .!.2 I [ 2(5 - 6) + c 2 + -13H6 4) - = .!.2 1 (42 + 15 - 8)1
: :'P .·,
· .-:..-

. .f''',
..,)
+ 2( 4 - 5 )] I [ 1 ] = -2I 1 49 1 = -492 sq unit ·
[1 ]
�· /! [·: area of L\ = .!_ l [x1 (Yi - y3 ) + X2 (J3 - y1 )
2 and area of llDBC

= .!_2 17x + 6 + 9x + 1 2x - 201


+ 3( -
= 1- 2 + 2.fj - 21 X JI J2 )]1
4
= .!.2 lx(5 + 2) 3(-2 - 3x) + 4(3x - 5�
-

(x1 , Y1 ) = A( a , a 2 ). B(x2 , Ji ) = B(b, b 2 ) and


+
�, �� Area of parallelogram ABCD = 2.J3 sq units
["
[1 1
i��:t
'·�-. ) '
55. Le r us assume that the given points
A = .!.2 12sx - 1 41 = 7l2x - II (1 1
C (x3 , y3 ) = C(c, c i ) form a MBC . of llDBC
Now, it is given that area area o f ABC 2
A
u.n
Coordinate Geom etry I 211 I
7 l ( 2x - 1)1 = 1 = -21 [16 + 16] :::: -
32 = 1 6 sq units
.
49 2 2 [1 ]
2 Now, AB = �(2 + 2) 2 (O - 0 ) 2
+
=> 2 x 7 l (2x - 1)1 = .!._ [by distance formula]
49 2 = J42 = 4 sq units
=> 2 x 2 I (2x - 1)1 = 7

--
=> Bx - 4 = ± 7 [·: l xl = a=> x = ± a] and PQ = �(4 + 4) 2 (O - 0) 2 +
=> Bx - 4 = 7 and Bx - 4 = - 7 = .J82 = 8 sq units [1 1
=> Bx = 1 1 and Bx = - 3 (AB) 2
x=- 1 1 and x = - -3 We have to verify that, MBC = --
B 8 [1 ] llPQR (PQ) 2
57. Given points are A (7, 8), B(4, 2) and C(8, 2) Now, LHS = - MB C = - 4 = -1 . . . (i)
and mid-point of BC is D(6, 2) (say D). llPQR 1 6 4
We know that, and RHS = -- (AB) 2 = - 42 = -
16 = -1 . . . (ii)
(PQ) 2 8 2 64 4
Area of fl = -I l [x1 ( y2 - y3 ) x2 (y3 - Y. i )
+ From Eqs. (i) and (ii), we get
2
+ X3 (J1 - J2 )J I [1 ] LHS =RHS Hence proved. [1 ]
:. Area of MBD = .!_ I (7(2 - 2 ) + 4(2 - 8) 59. Do same as Q. 8 at page 265.
2 Now, area of rhombus PQRS
+ 6(8 - 2>11 = .!._ x (Product of lengths of diagonals)
= .!. l (o - 24 + 36 11 = 6 sq units

(.!. )
2
2
Area of MDC = -1 x (PR x QS)
2
= .!. 1 [7(2 - 2) 6(2 - 8) + 8(8 - 2)]1
+
= x 4..fi. x 6.Ji. = 24 sq units
2 2 [1 ]
= .!._ I (0 - 36 + 48)1 = 6sq units [1 ] 60. Given that, A(6, I), B(S, 2) and C(9, 4) are three
2 vertices of a parallelogram ABCD. Let the fourth
Area of MBD = Area of MDC vertex of parallelogram be D(x, y).
Also, area of MBC [1 ]
D (x, y) E C (9, 4)
= .!._2 I [7(2 - 2> + 4(2 - 8) + 8(s - 2) J I
= .!. no - · 24 + 48JI = 24 = 12 sq units
2 2
i.e. area of MBD = area of MDC = .!. (area MBC) A (6, 1 ) B (8, 2)
2 We know that, the diagonals of a parallelogram bisect

(8 : x . 2 ; y) = (6 ; 9 . 1 : 4 )
Hence proved. [1 ] each other.
58. Given vertices of a ll.A.BC are A(-2, O) , B(2, O) and :. Coordinates of mid-point of BD = Coordinates of
C(O, 2), respectively. mid-point of AC
:. Area of MBC �

= -1 l x1 ( J2 - J3 ) + x2 (J3 - yi ) + X3 (J1 - J2 )l

( ;
[·: coordinates of mid - point of a line segment

-
2
= .!._ I (-2) (O - 2) + 2 (2 - 0) + 0 (O - O)j joining the points (x1 , y1 ) and (x2 , y2 )

( 8 ; x 2 ; y ) = ('� % )
2
= -1 [ 4 + 4 ] = 82 = 4 sq units = x 1 x 2 , y1
2 [1]
Also, given vertices of a 6.PQR are � • •

P(-4 , O ), Q(4, O) and R(O, 4). [1 ]


:. .Aiea of /::,,.PQR

-
On equating the coordinates, we get
8 + x = !-2. and
= .!._ 1-4(0 - 4) + 4(4 - 0) + oco - O )I � �2 ===
2 2 2 2
.!.
= 1(-2)(0 2) + 2( 2 - 0 ) + 0(0 - O )I ::;> 8 + x = 1 5 and 2 + Y = 5
2 => x = 7 and · y = 3
Allinone MATHEMATICS C la ss 1 0th Term I

Now, mid-point of side DC = E ( 7 ; , 3 : )


So, the fourth vertex of a parallelogram is D(9 7, 3 ).4 62. Lee DM = be the height of parallelogram
h ABCD

= E ( 8. f l
when AB is taken as1 rhe base.
Area ofMBD = - ( AB DM) x

and E ( 8, � ) is given
2
[·: area of = - x base height)
/1
l
x
:. Area of MDE with vertices A{6, 1), D( 7, 3) (1 1
2
as
� Area ofMBD = .!._2 AB x x h

� H3 - �) + G - I ) + 8 (I - 3)1
h
ar ofMDE = 2(area MBD) ... (i) (1)
AB
= Now, find rhe lcngch of AB using distance
7
formulafirst, by
[·: area of triangle = .!._2 I x1 (y2 - )3 ) = Jc2 - 1)2 + (3 + 2) 2 = .J26

1 (�I) + {%) + 1
AB

:.Aiea of MBD = .!._ 1 (1(3 + 3) + 2(- 3 +


+ X2 (Y3 - Yi > + X3 (Y1 - Y2 >l l
= �6x
= 1 -3 + 3 - 1 61
Now, the coordinates of vertices of 11ABD are

� ;
8(-2) A (1, - 2), B(2, 3) and D(- 4, - 3).
2)
2
+ (- 4)(- 2 3)J I
= _!_ 1 3 5 - 1 9 1 = .!._ 1 -3 1 =
-

_!_ [·: area of /1 = -2I l [x1 (Ji )3 ) +


Hence, the required area ofMDE is �4 sq units.
� �
2 2 2 2 = 4 sq units
-
x
2 2
['.' area cannot be negative] .'\"2 (y3 - Yi ) + X3 ( y. 12>11 -

24
= .!_ 1 [6 - 2 + 20] 1 = = 1 2 sq units ( 1]
(1 ] 2 2
61 . We have, A(x1 , y1 ) = A(7, 4),B(x2 , y2 ) Now, putting the value of A B and area of MBD in Eq.
(i) , we get
h
= 2 x 1 2 = � x .J26
= B(- 2, 6), C(x3 , y3 ) = C{5, - 1) and P(x4 , y4 )

+
= P (x, y ) .J26 .J26 .J26
:.Area of MBC = .!._2 I [x1 (y2 - y3 ) + x2 (y3 - y1 ) h
= 24.J26 I .J26
l
26 = 1 3
units [1]
X3 (y. - Y2 >l l (1 ] Given vertices of MBC are A (1, - 1), B (-4, 6) and
Area of1 MBC 63.
C (-3, - 5), respectively

+
= 2 17(6 + 1) + (-2)(-1 - 4) + 5(4 - 6)1
Now, AB = J..-(--4---1-)2__ + (_6_+_1)_2
= 21 17 x 7 - 2 x (-5) + 5 x (-2)1 (1 ] [by distance formula) [1]
2 2
= �(-5) + (7) = .J25 49
= .!_ j49 + 1 0 - I OI= 49 sq units

+
2 2 [1 1 = .J74 = 8 .6 units

+
Here , P(x, y ), B(-2, 6) and C(5, - 1) BC = �(-3 + 4) 2 + (-5 - 6) 2
:. Area
1
of 11PBC = �(1) 2 + (-1 1) 2 = .J1 1 2 1 [1]
=
2 jx(6 + 1) + (-2)(- 1 - y ) S (y - 6)1 = Jill. = 1 1 .0 1 units

=
= -1 17 x + 2 + 2 y + 5 y - 301 and CA = �(-3 - 1)2 + (-5 + 1)2
2 •

.!._ j7 x + 7 y - 281 = Z j x + y - 4jsq uni cs


= �(-4) 2 + (-4) 2 = .J16 + 16
>"
l>C
2 2 = .J32. = 5.7 units
!;
7 x + - 41 Here, we2 see that2AB CA* BC ¢ CA [1 ]
I'; area of 11PBC = 2 l Y
�· Hence, and BC *- AB + 2
u
;
+
�'! :.i area of 11ABC 49
;, So, given triangle is a scal e ne tri a ngle.
.- , , 2 Now, area of MBC
Hence proved. [1 ]
= _!_2 l x1 (J2 - J3 ) + x2 (J3 - y1 ) X3 (J1 - y2 )I
·.'· i"i
Coordinate Geometry f 279
J

= 2I 11(6 + S ) + ( -4 ) ( 5 + 1) + (-3)(-1 - 6)1


-
So, rhe vertices of MBC are A ( I l, 4), B (- 4, 3) and
C(3, 2). [1 /21
I
Now, area of MBC = - x1 (Ji .,.... y3 )
= 2I 1• t - 4(-4) - 3(-7)1 = 21 11 1 + 16 + 21 1 2
+ Xi (Y3 - Y1 ) + X3 (yl - Y2 )I
= -21 148 1 = -1 x 48 = 24 sq units
2 [1 ]
= 21 II I (3 2) + (- 4) (2 - 4) + 3 (4 - 3)1
-

( %} ( H)
64. Lee the vcrriccs of MBC be A ( x1 , y1 ), B(xi • Yi ) and = -1 11 1 x 1 + < - 4) c - 2) + 3 o ) I
C (x3 • Y.d· 2
Given. D - i· = -1 ll l + 8 + 3 1 = -1 I 22 I = -
22 = l I sq units
E (7, 3) and F are 2 2 2

(�, )
[1 ]

(- i %)
the mid-points of the sides BC, CA and AB, 65. (1) Here, from the figure,
respectively. Coordinaces of A = (I, I), coordinates of B = (6, I)
Since, D , is the mid-point of BC. and coordinates of C= 2 7
[1]
• Xz + x."\ 5 Also, altitude of M.BC = CD = 7 - I = 6 cm
2 = - ..!...2 . and
Yi + Y3
2 2 and base of MBC, AB = 6 - I = S cm [1 /2)
=> X 2 + x3 = - 1 . . . (i) . . Area of one banner = Area !lABC
and Yi + Y3 = 5 ... (ii) [1 /2] = -21 x CD x AB
As, E (7 , 3) is the mid-point of CA .
X3 + Xi
= 7and Y3 + Y1 = 3 . = -2I x 6 x 5
2 2

(% , %)
=> � + � = 14 ... (iii) = 3 x 5 = 1 5 cm 2 [1 /2]
and Y3 + Yt = 6 ... (iv) [1 /2) Then, area of 10 banners = 1 5 x I O = I 50 cm 2
[ 1 /2)
Also, F is the mid-point ofAB. :.Cost of 10 banners at the rate of ' 2 per cm 2
- 7
7 and .::. = I S O x 2 = � 300 [ 1 /2]
...,:__
x, + Xi Yt + Yi -
· ·

2 = - .--:;....;;..
2 2 (ii) Coordinate geometry. [1 /2]
=>
2 ... (v)
XI + Xi = 7 (iit) Social awareness. [1 /2)
and Yt + Yi = 7 ... (vi) [1 /2) 66. (t) Let A(2, -2) A ( x1 , y1 ), B(4, 8) = B ( xi , Yi )
=
On addi ng Eqs. (i), (iii) and (v), we gee and mid-point ofAB be C (x, y) .
2 ( x1 + Xi + x3 ) = 20
=> X 1 + x + x3 = 10 . .. (vii) x =
x1 + x
2 and y = �--
Yi + Y i
2
On subrracring Eqs. (i), (iii) and (v) from Eq. (vii) 2 2 [1 1
2+4 -2 8
x = -- and y =
+
respectively, we get => --- ;l'.._·.·_

XI = 1 1, Xi = - 4 , X3 = 3 [1/2) 2 2 ·- �
:

and y = -62 :-
On addi ng Eqs. (ii), (iv) and (vi), we get => x = -
6 ...
. ..

"

2 (y, + Yi + Y3 ) = I 8 2
:::) y1 + Y2 + y3 = 9 .. (viii)
.
=> x =3 and y = 3
On subtracting Eqs. (ii), (iv) and (vi) from Eq. {viii) Hence, the mid-point of A and B is (3, 3) . [1 ]
respectively, we get (ii) Mid-point ofline segment. [1 ]
Y1 = 4 • Yi = 3 • Y3 = 2
_,
[1 /2) (iii) Enjoyment and intelligency. �
[1 ]
m 11 11
FO RMATIVE ASS ESS l\Jttt:··N T
• •

Act1v1ty
Topic Covered To find the distance between any two Oral Questions
points i n the plane. 1 . Give the formula t o find the d istance between the
Pre-knowledge Knowledge of plotting given points in a points A(x1 , y, ) and B(x2 , y2 ).
coord inate system. 2. Suppose the coord inates of a point lies on X-axis.
Time Req u i red 1 5 min Then, what are the coord inates of P?
Materia l Req uired G raph paper, scale, pencil/black ball 3. Points A, 8, C and D are fou r d istinct points in a
pen, rubber (eraser), sharpner. cartesian plane. How d o we show that ABCD is a
parallelogram?
Observation Let X'OX and Y'OY be the coordinate axes.
4. How will you show that three d i stinct points are
Let P(x1 , y1 ) a nd Q (x2 , y2) be two g iven points in the
plane. Draw PL a nd QM perpendiculars from P and Q on collinear?
X-axis respectively. From P, d raw PN � QM. 5. How will you find the area of q uadrilateral ABCD
y (using the formula area of triangle)?

O(x2. Y2)
True/ False
1. MBC with vertices A (-2,0), B (0, 2) and C (2, 0) is similar

YLI Y2
to llDEF with vertices D (-4, 0), E (4, 0) and F (0, 4).
2. Points A (-6, 10), B (-4, 6) a n d C (3, - 8) are collinear

P(x1 , Y1) _ x2 - X1 _ I such that AB = � AC.


9
3. The point P (-2 4) lies on a circle of radius 6 and
N
X1 Y1
,

= == =
centre (3, 5).
4. The points A (-1, - 2), B (4, 3), C (2, 5) and D (-3, oj

= =
in that order form a rectangle.
The n, OL = x1, OM x2 , PL y1 and QM y2 5. Point P (5, -3) is one of the two points of trisection of
.. PN = L M OM - OL x2 - x1 line segment joining the points A (7, - 2) and
and QN = QM - NM QM - PL = y2 - y1 8 (1, - 5).

Clearly, l:lPNQ is a right angled triangle, right angled


Fill in the Blanks

=
at N.
1 . The point which is equidistant from the points
(-2, 5) and (-2, - 3), is .... .
Therefore, by Pythagora s theorem,

=
PQ 2 = PN 2 + QN 2
2. If point (x, y) is equidistant from the points (a + b, b - a
=> (Xz - x,...) 2 + (Y2 - y, ) 2
PQ 2 and (a - b a + b , then bx is equal to .... .
=> PQ �(Xz -: x, ) 2 + (yz - Y1 ) 2 3. Points (3, 2), (-2, - 3) and (2, 3) form a .... triangle.
Hence, the distance between two points 4. If points (1, - 1), (5, y) and (9, 5) are collinear, then y is
equal to ....
(Difference of abscissae) 2
= 5. If points (0, 0), (3, ./3) and (x, y) form a n equilateral
triangle, then (x, y) is equal to ... .
+ (Difference of ordinates) 2
Coordinate Geometry

M atc h t h e Columns Across


Col umn I Column I I 3. The area of triangle formed by three poi nts is zero .
Then, poi nts are ... .
5 . One of the two mutually perpendicular l i nes i n the
1. The ratio, in which X-axis divides the
join of the points (2 , -3) and (5, 6) , is
If P and 0 are points on the line joining
coord inate system is cal led . . .
6. T h e point, which is eq u id ista nt from a l l poi nts o n a
2. (b) isosceles
A (-2, 5) and B (3, 1 ) , such that
AP = PO = OB. then mid-point of PO circle, is cal led . . . .

= =
is
7. Dista nce of a point from Y-axis is cal led . . . .
3. Type of a triangle having vertices (c)
(4, 0) , (- 1. - 1). (3, 5) , is 8. M id-point of the poi nts (3, - 5) a n d ( 3, 5) is . . . .
-

1 :2 9 . Quadrilateral formed by the poi nts A, 8, C a n d D,


a triangle having vertices A (1 , 4), B (3,
4. The le ng th of the m edia n throu gh C of (d)
such that AB = BC CO DA a n d AC = BO, is
0) a n d C (2, 1 ) , i s ca lled ...

Crossword Puzzle D ow n
Solve the following crossword puzzle, hints are 1 . Distance of a point from X-axis is ca l led . . . .
2. The point that divides each med i a n of a tri a n g l e i n

= =
given below.
2 the ratio 2 : 1, is cal led ... .
4. Quad rilateral formed by the poi nts P, Q, R a n d S such
6
4
that PQ QR = RS SP, is cal led . . . .
3

8 Project/Survey Work
To mark coord i nate axes on you r city map a n d fi nd the
7 distance between important landmarks l i ke you r h ouse,
school, bus stand, railway station, ai rport, etc.

Answers
O ra l Q u est i o n s

1. AB = (Y2 - Y1 ) 2
�(X-i - Xi )2 + 2. P(a, O), a e R
3. We show AB = CD and BC = AD (opposite sides are equal)
4. Area of triangle formed by these three points should be zero.
S. Divide the quadrilateral into two triangles and find their areas separately and then add them to get area of two
triangles.

True/Fa lse lf�


'"""
1. False 2. True 3. False S. True \..;:; .
l-�=
4. True

··� - ·
.� ,--=itro

F i l l in the B l a n ks
1 . (-2, 1) 2. 3 . right angled v.
.a.
ay 4. 2
5. (0, 2J3) or (3,- ../3)

M atch the Col u m ns ,�


1. (d) 2. (a) 3 . (b) 4. (c)

Crossword P uzzle
1. ordinate 2. centroid 3. collinear
5.
4. rhombus axis 6. centre
7. 9. square
8. origin r, 1.. _ _
abscissa
S U M MARY
• I f P (x1 , y1 ) a n d 0 (x2, y2) a re end points of the line segment PO, then by dista nce form u la the d i sta nce between P andQ ·


g iven by

=�
2 2


P O = (x2 - x,) + (Y2 - y,)
2 2
or (X1 - X2) + (y, - yi}

=�
i.e. 2 2
PO (Difference of abscissae) + (Difference of ordinates)

(
Let P (x, y) be a point a nd 0 (0, 0) be the origin . Then, distance of P from 0 is

)
2 2
OP (x + y )
• Coordi nates of the point, dividing the line segment joining the points (x1, y1) and (x2, y2) i nterna lly i n the ratio m 1 : m2 are
,x + m2X1 m1 Y2 + m2Y1
g iven by i nternal section form u la m 2 , .

( ; ; J
m1 + m2 m1 + m2
x, x2 y,
• The coordi nates of the m id-poi nt P of joining the points A(x1 , y1) and 8 (x2, y2) is Y2

=�
• Thre e or m ore than three points lie on a line is called a collinear poi�ts.
• If A(x1, y 2), B (x2 , y 2) a nd C (x3, y3) are the vertices of a llABC, then the area of dABC is g iven by the following formula,
Area of dABC I x1 (y2 - y3) + x2 (y3 - y1) + x3 (y1 - y2)j sq units.

• Th ree poi nts a re said to be collinear, iff area of the triangle is zero.
• The a rea of quadri latera l , is finding by dividing it into triangular region, whictJ have no com mon a rea . Then, find the area of
each triangular reg ion sepa rately and add them to get the required area of given quadrilatera l .

CHAPTE R EXE RCIS E


�I Very Short Answer Tvpe Questions [ 1 Mark each]

1 . Find the distance of the point P(2 , 3) from the X-axis. 6 . Find the distance between P (b + c, c

-
+ a) and
NCERT Exemplar Q (c + a , a + b) .
2. A line interesect the Y-axi s an d X-axis at the points P and 7. Find the coordinates of the point, where the line
Q, respectively. If ( 2, - 5 ) is the mid - point of PQ. then find
x y = 5 cuts Y-axis . CCE 2013
the coordin ates of P and Q. NCERT Exemplar
B . Find the point o n the X-axis, which i s equidistant from 8. Find the value ( s ) of x, for which the distance AB
t he p oints ( 5, 4 ) and

-
(-2, 3 ) . between the points A (x, - 5) and B (3, 2) is
t he po int on the X-axis which is equidis tant from .J58 units.
4 . fi n d t s (- I , O)
t he po in and ( 5 , 0 ). CCE 201 3
9. The mid-point of the line joining (3a , 4) and
5 . The d is t an ce b etween the points P(x, 1 ) and Q (3, 2) is
(-2, 2b) is (2, 2a + 2). Find the values of a and b.
5 u nits. Fin d the value of x.
Coordinate Geom etry f 283 f

�I Short A nswer Type I Questions [2 Ma rks ea c h ]

1 0 . MBC is a rig h t a n gled t riangle, in which A ( 0 , 2) and 2 0 . I f the points ( -2 , - 1 ) , ( I , 0 ) , (x, 3 ) and ( I , y ) form a

=
B ( 2 , O) are g iven . Then, find the coordinates of C. parallelogram, then find the values o f x and y.

21. If the mid-point of the line joining ( 3, 4 ) a n d (k , 7 ) i s


CCE 2013

=
1 1 . Name t he type o f triangle formed by the points (x, y ) and 2 x + 2y + I 0 , then find the val u e o f k .
A( O, 4 ) , B ( 3 , 0 ) an d C ( O, 0 ) .
2 2 . Determine the ra tio, i n which the s traigh t line
1 2. Prove that t he points ( 0, 0 ) , ( 5, 5 ) and ( -5, 5 ) are the x - y -2 O d ivides the line segment j oining poin t s
vert ices of a right angled isosceles triangle . ( 3, - I ) and ( 8, 9 ) .
CCE 2012, 1 1
2 3 . Check whether the points ( l , 5), ( 2 , 3 ) and (-2 , - I I)
1 3 . The points A ( 0, 3 ) , B ( -2, a ) and C ( - 1, 4 ) are the
vertices of MBC, right angled at A. Find the value of are collinear or not, by u sing distance formula.
a.
24. Find the perimeter of the triangle formed b y points
1 4 . Find the coordinates of the point, which divides the (0, 0), ( I , 0) and ( 0 , I). CCE 201 1

(: }
line segme n t j oining the points ( 1 , -3 ) and b3, 9) in 2 5 . Check whether the points ( 20 , 3), ( I 9 , 8 ) and ( 2 , - 9 )
the ra tio I : 3 internally.
are a l l equidistant from the point ( 7 , 3).
1 5 . What will b e the value of y. if the point
2
, y ivides 2 6 . The vertic�s of a triangle are (a , b - c), (b, c - a) a n d
(c, a - b) . Prove that i t s centroid lies on X-axi s .
the line segmen t joining the points ( 5 , 7) and (4, 5 ) in
the ratio 2 : 3 internally. 2 7. Find the angle subtended by these points P( ..Ji., ..Ji.),
1 6 . The centroid o f a triangle is the point ( 6 , - 1). If two Q(- .Ji., - .Ji.) a n d R (- .J6, .J6). NCERT Exemplar
vertices are ( 3, 4 ) and ( -2, 5 ), then find the third
vertex.
28. If ( 5, 2 ), ( - 3, 4) and ( x , y) a re collinear, s how tha t
x + 4y - 1 3 = o. CCE 201 5
1 7 . Find the coord ina tes of a points which trisect the line

=
segment j oining ( I, -2 ) and (-3 , 4) 2 9 . (ijO"fS] The line segment j oining the points A (2 , I) and
CCE 2008 B(5 , - 8 ) is trisected a t the points P and Q such that P
is nearer to A. I f P also lies on the line g iven b y
1 8. Find the angle subtended at the origin by the line

P ( si� ) andQ ( co;e}


2x - y + k 0, then find the value of k. C C E 201 5
segment whose end points are ( 0, 1 00 ) and ( I O, 0 ) .
3 0 . fHQ'fj] Find the dis tance between the points
1 9 . Find a point o n the X-axis, which i s equidistant from
the points ( 7, 6 ) and ( -3, 4 ) . a, 0 o,

�I Short Answer Type II Questions [ 3 Marks ea c h ]

3 1 . Prove that ( 5, 6 ) , ( 2, 8 ) , ( 0, 6) and ( 3, 4) are the Find the value o f k an(! the distance AB.
vertices of a parallelogram. Is it a square? NCERT Exemplar
3 2 . If two opposite vertices of a square are ( 5, 4) and a :, . �� If the centroid of the triang le formed b y t h e
( 1, - 6 ), then find the coordinates of its remaining two pomt s A(a , b), B(b, c) and C (c, a) is a t the origi n. Wha l
vertices.
is the value of
a2
- +
b2
_ + _?
c2
3 3 . Show that the points A ( I , 0 ) , B ( 5, 3 ) ,
C ( 2, 7) and
be ca ab ·
D (-2, 4) are the vertices of a parallelogram. 3 6 . (ijOfSl Points P, Q, R, s a nd T d ivi d e we lin e s eg m ent
3 4 . If (a , b) is the mid-point of the line segment joining joining the po in t s A ( l, 2) an d B(6 , 7) in 6 equal part s .
F in d coordinat e s o f the p oints P, Q, R, S and T .
the points A( l 0, - 6 ) and B(k, 4) lies on a - 2 b = 1 8 .
CCE 20 1 4
Allin.One MATH EMATICS C lass 1 0th Term I

� I Long Answer Type Questions [4 M arks each]


3 7 . Find the a rea o f Af'QR formed by joining the 40. �
� S tudents o f a school a rc s t a n ding in rows and
m i d -points of the sides of the triangle, whose vertices columns in their playg ro u n d fo r a d ri l l practice. A, B.
a re A (1 , 2), B (3, 2) and C (
-
-
1 ,
4). Al so, find the area C and D are the positions o f fou r s t uden t s as shown in
of MBC . CCE 2013 figure. Is it pos sible t o place J a s pal in t h e drill i n such
a way that he is equid i s t a n t fro m each of the four
38. J:Lq_j� The poin t s (3 , - 4 ) and (-6 , S) are the end points of a students A, B, C and D? I f so, wh a t should be his
d i agonal of a paral lelogram. If one of the end points posit ion? NCERT Exemplar
of the s econd d ia gonal is (-2 , 1), then find its other
B
10
end point. 9

39. �of� Ayu s h s tarts walking from his house to office.


8
7
I n s tead of going to the o ffice directly, he goes to a Rows 6 'A c;
5
bank first, from there to his daughter's school 4
3
and then reaches the office. What is the 2 D
ex tra dis tance tra velled by Ayush in reaching 1
his o ffice? [ a s sume that all distances covered are
in s traight lines, if the house is situated at (2, 4),
Columns Teacher �
1 2 3 4 5 6 7 8 9 10 1 1 12 13

bank a t (5 , 8 ), school at ( 1 3 , 1 4) and office at ( 1 3 , 26)


4 1 . [HOT51 A(6 , l), B(8 , 2) and C (9 , 4) a re three vertices of a
parallelogram ABCD. I f E is the mid-point of DC.
and coordinates are in kilometres ] .

I
NCERT Exemplar Find the area of the MDE. CCE 2015

� Value Based Question (VBQs) [ 4 Marks each]

4 2 . To ra ise social awarenes s about the hazards of ( ii) which value is depicted in the question?
s moking a school decided to start " NO SMOKING" y
campaign. A s tudent is a s ked to prepare a campaign
b anner in the shape of a triangle shown in the figure.
If cost of l cm 2 of b anner i s � 2, then ( i ) find the cost
o f the banner. (1 , 1 )
------- x
~ (6, 1 )
0

Answers
1. 3 2. (0, - 10) and (4, 0) 3. (2 ; O) 4. (2, 0)
5. 7 6. .Ja2 + 2b2 + c2 - 2ab - 2bc 7. (0, -5 )
= 0, 6 a 2, b <!
= = 1 0. (0,

( - � 0) . ( : )
8. x 9. O) 1 1 . Right a ngle d triangle
, ,2 (0, 0)
31
5
1 2. - 1 3. 1 1 4. 1 5.

1 6. (1 7, - 1 2) 1 7. ( � 0) (-� 2)
- , and , 1 8. i 1 9. (3, 0)

2 0. x = 4, y = 2 21 . k - = 15 22. 2: 3 23. Not collinear


24 . (2 + ./2) 25. Yes 27. 60°, equilateral 29. k = - 8
1
31 . No 31 . (-2, 1) and (8, - 3) 34. k = 22, = 2J6I
2
30. AB

35. 3 36. (6, 7), (2, 3), (3, 4) , (4, 5) an d (5, 6)

3 7 . ar(f).PQR) � sq unit s ; ar(M.BC)


= = 1 0 sq u nits
38. (-1, j. O) 39. 2 .4 k m
2
40 . Yes, Jaspa l shou l d be placed at the point (7, 5).
4 1 .'3 sq u n i t 22. (i) 1 0 (ii) Social awareness
. 2
CHALLE N G E RS*
---;l!t'.:::.;.-:... . _ _-.
!IE::

1 . If the vert ices o f a triangle have integral coordinates, prove that the triangle cannot be equilateral .
2. The angle of elevation of a lower at a point S (3, 2) is 45°. After going your towards the foot of tower from S to R(6, 2 ), the ang l e o f
elevation of tower becomes 60° . Then, find the coordinate o f the tower P.

3 . Join two points P(2, 2) and 0 (4, 2) in a plane. Fixed the point P and rotate the line PQ in anti-clockwise d i rection at an ang l e of
270°. Find the n ew coordinates of point Q. Also , find the area formed by this figure.
4. If G is the centroid of a MBC and P is any other point in the plane, then prove that
PA2 + PB 2 + PC 2 = GA2 + G B 2 + G C 2 + 3 GP 2 .

DLJC (3,3)
5 . A figure is shown adjacent.
y
:
1
X'--+---+---+-=+---+--+-
-3 -2 -1 0 1 2
(3 3,
A (1 1 1 )
B
-4-- X
1)

-1
-2
-3
Y'
If we rotate this g raph about 0 at an angle of 1 80° in anti-clockwise direction , then find the new coord inates of A, B, C and D. Also ,
find the intersection point of diagonals .
6. If (- 4, 3) and (4, 3) are two vertices of an equilateral triangle, then find the coordinates of the third vertex, Given that, the o ri g i n
lies in t h e interior of t h e triangle.

7. Using analytical geometry, prove that the diagonals of a rhombus are perpendicular to each other.
8. Suppose there are four points A(2, 4), 8(6. 4), C(6. 6) and 0(2, 6), which lie in the first quadrant.
(t) If we rotate only the axes at an angle of 90° in anti-clockwise direction, then find the new coordinates of the g iven points .
(it) Write the name of the figure when we join adjacent points.
9. Suppose, PQ be a pole whose coordinates be P(1, 3) and 0(3, 3) and point A be the position of a man whose coord inates
are (1, 1) .
(t) I f a pole makes a n angle of elevation to the point A , then find the angle a.
(it) Also, if we shift the origin at (1, 1). then find the angle a.
1 0 . (t) A circle is i nscribed i n a squar, find the centre and radius of a circle.
y

3 DOC
5
4

3
2
A B
x· x
0 1 2 4 5 6
Y'
(it) Also, find the area between the square and inscribed circle.
* These q uestions m ay be or may not be asked in the exa m ination, have been g iven just for a d ditional prac ti ce.
Note Solu tions to these questions are available at page number 397-394.
CHAPTE R S

A REAS RE LATE D TO C I RC L ES

( � Topic_ Checklist _ Many obj ects that we come across in our daily life , are related to ci rcular shape in
some form or other. In this chapter, we shall review the concepts of perimeter
Review of Concepts of Area (or circumference) and area of a circle and apply this knowledge i n fi nding the
and Peri meter of a C i rcle areas of two special parts of a circular region known as sector a n d segment.
� Circle
, Chord
, Secant
TO PI C 0 Review of Con ce pts of Area
" Diameter
� Arc and Perimeter of a C i rcle
� Sem i-circle
,.. Quadrant of a C i rcle
CI RCLE
A circle i s the locus of a point which moves i n a plane in s uch a way that its distance
· Peri m eter of a C i rcle
• Area of a Circle
from a fixed point is remains the same. The fixed point is called the centre and the
Areas of Sector and Segment
of a Circle
given constant distance is known as radius of the circle.
. Sector of a C i rcle
0 Area of Sector of a C i rcle TE RMS RELATED TO CI RCLE
" Length of an Arc of a Following terms related to circle are as follow
Sector
.. Segment of a Circle Chord
Areas of Combi nations of A line segment joining any two points on a circle is called a chord of the circle.
Plane Figures E D

F
c

Here, AB, CD and EF are three chords of circle with centre 0.


Secant
A line which intersects a circle at two points, is called
a secant of the circle. Here, PQ is secant of circle. 0

Note In case of a chord , end points of a line segment is

Q
always lie on a circle. Bue in case of a secant, end
poi nts of a line segment lie oucside of che circle. P
Areas Related to Ci rcles f 287 /
Diameter Perimeter of a Circle
A chord of a circle passing thro ugh the centre is called a The distance covered by travelling o nce around a circle i s
diameter of the ci rcle. called the perimeter or the length of boundary o f a circle i s
called the perimeter. Usually, i t is known as circumference.
The ratio of the circumference and diameter of a circle is a
0 constant. This, constant ratio is denoted by the Greek letter
A B 1t (pi) .
Circumference
Thus, ------ = 1t
Diameter

Thus, I Diameter =2 x Radius or Circumference = 7t x Diameter = 1t x 2r


Note
where, r is the radius of circle.
Diameccr is the l ongest c h ord o f a c i rcle.

Arc
=> j Circumference of circle = 21tr

,, , ----- Some Important Formulae


Arc is a continuous piece o f a circle. Q
...

-- ''

( +
9 �,)
In the adjoining figure, APB is a arc and .. .. .. ,
/ Major arc \ (1) Circumference of semi-circle = (rrr + 2r)
( ; 2r)
'

it is written as APB. The remaining part I I

of the circle is denoted by JiQA. (it) Circumference of quadrant of circle = +

An angle subtended by an arc at the \// (iii) Distance moved or covered by a wheel in 1 rotation

an e �
centre of a circle is called its central A ' , /B = Circumference of the wheel
gl . Here, central angle of arc (iv) Number of rotations of a wheel in travelling a certain distance
APlJ = LAOB = 8° . Minor arc Distance travel led by wheel

-
Circumference of wheel
In the circle, smaller arc (APif) is called the minor arc and
the larger arc (AQB ) is called the major arc. The great I ndian M athematician Aryabhatta (476 -550 AD)
gave the approximate valu e of 7t.
For minor arc, the central angle is less than 1 80° while for 62832
7t = :::: 3.1 41 6
major arc, the central angle is greater than 1 80°. 20000
The great Mathematical Genius Srinivas Ramanujan (1 887-1 920)
Semi-circle of India calculated the value of 7t correct to million places of

-- -
decimal. 7t is an irrational number and its decimal expansion is
A diameter divide a circle into two equal arcs. Each of these non-terminating and non-recurring (non-repeating) . We generally
7t
22
two arcs is called a semi-circle. Here, A CB and ADB are take the value of as or 3. 1 4, approximately.
7
two semi-circles.
c Area of a Circle
The area of a circle is 1tr 2 , where r is the radius of the circle.
We can verify it by cutting a circle into a n umber of secw r�
A B and rearranging them as shown in figure.

Quadrant of a Circle
One-fourth part of a circle is called a quadrant of the circle.
Here, A OB is a quadrant of the circle. The central angle of a
quadrant is 90° . 8 . (I)

-
(ii)
l
The shape in fig. (ii) is nearly a recta ngle of length x 2nr
,,
.I...

and breadth r.
Then, area of the circle = _! x 2rtr x r :::: rtr 2
2
\ 2 ss \ Allinone MATH EMATICS C lass 10th Term n

Some Important Formulae Exam p le 3. The cost of fencing a circular field at the
rate of ' 24 per m is � 5 2 8 0 . Th e field is to be ploughed
Let a circle of radius be r, then
at the rate of ' 0.50 per m 2 • Find th e cost of ploughing
(1) Area of semi-circle = ..!.. 1tf 2
2 the field. [Take, rc 2 2 / 7)
2
=
(it) Area of quadrant of circle = � . . field = Total cost
Let r and R be the inner and outer radii of a ring .
4 Sol. Length of the fence of a circular ----

Rate
m
5280
= 24 = 220
So,Let rthebecithercumference m.

radius of theof2rcrthefield,=fiel220then
d is 220
2 227 r = 220
x -x

r = 220 x 7 = 35
m

(R2 - r2 ) = r) (R
Then, area of the ring = 1t 1t (R + - r)
x 2 22
So, the radius of the field is 35 m.
N o te • I f two circles touch i n ternally, t�"' C :1 t ! 1 e distance between
thei r centres is equal to the difference of their radii.
Now, area of the field = rcr2
• If two circles touch externally. then the distance between
their centres is equal to the sum of their radii.
=-227 35 35
x x

Example 1 . The radii of two circles are 13 c m and


= 22 5 35 2
x x m

6 c m, respectively. Find the radius of the circle which


Given, cost of ploughing 1 m 2 of the field = 0.50 �

has circumferen ce equal to s um of the circumferences So, total cost of ploughing the field
of the two circles. = 0.5 22 5 35 = 1925
x x x �

Sol. Circumference of the= 2rccircle13having radicmus 13[·cm: radius= 13 cm] Exam p le 4. Circular footpath of width 2 m is
x = 26rc constructed at the rate of f 2 0 per m 2 aro und a
and circumference of =the2rccircle6 =12rccm
x
having radius [6·:cmradius =6 cm] circular park of radius 1 500 m. Find the total cost of
construction of the footpath . [take, 7t 3 . 1 4 ]=
:. Sum of the circumferences of the two circles Radius of the park, i.e. inner radius of the park,
Let r be the radius of =the26rcrequi12rc
+
r ed =r38rc
ci cle cm circumference is
whose
Sol.

38rc
Then,cm (i. e . sum2rcrrof==l9the38rccircumferences of two given circles). Width of the footpath raround
=1500them park = 2 m
=>
Lee R be the outer radius of the park including the footpath.
Hence, the radius of the new circle is 19 cm. Then, outer radius of the park,
R =(1500 2) m =1502 m +
Example 2. A bicycle 's wheel makes 5000
revolu tions in m oving 1 1 km . Fin d the diameter of the
wheel. [Take, re 2 2 I 7]
=
Sol. Dis. tance covered in 5000 revolutions =11000
11 =1111000 m km
.
Distance covered i. n one revoluuon = 5000 = -5 m --

Now, circumference of wheel


11
= Di22stance covered
11 in one revol
7
ution Now, area oft.he footpath = rtR2 - rcr2
::::> = 5 2 -7 r = -5 r = -m
2 1tr - => x x =>
20 =rt{R2 -r2 } =3.14 {(1502)2 - (1500)2 }
· Oi'arnerer
.. of bicycle ' s wheel = 2r = 2 .!_ = !_ x =3.14 {(1502 + 1500) (1502 - 1500)}
20 10 =3.14 6004 =18852.56 m 2
x

= 210 1 00 cm = 70 cm [·: I m=lOOcm]


x the footpath at the rate of
t
cost
18852.56on =of" 37705
20 per m 2 =of20constructi
:. Total x 1 .2
f 289

[·: 25 100 ]
Areas Related to Ci rcles

Example 5. If the area of a semi-circular field is


= 550x100 cm = � m
30800 sq m, then fin d th e perimeter of the field.
Sol Let the radius of che field be r.
2x -227 x 25
Then, -rrr-1 , I 22
=:> -
i
550 x 7 x 100
2 = 30800 2 x -7 x r = 30800 2 x22 x 25
=350
r2 = 30800 x 2 .!_22 = 1 9600
x => r = 140
[on taking square root] TRY YOURSELF
Thus, perimeter of the field = rtr 2r
Q. 1
+

[Take,
The perimeter of a semi-circular protractor is 1 08 cm.
= -227 x 140 2 x 140 = 440 280 = 720 m 22
J
7t
7
+ +
Find the diameter of the protracto. =

E xample 6 . The radius of the wheel of a bus is [Ans. 42 cm]


25 cm . If the speed of the b us is 33 km/h, then how Q.2 The diameter of the driving wheel of a bus is 1 40 cm.
many revolutions will the wheel make in 1 min ? How many revolution per minute must the wheel make
Sol. In 1 h discance covered by wheel = 33 km in order to keep a speed of 66 km/h? [Ans. 250]

:. 1 min, distance covered by wheel =


33x1000 m = 550 m Q .3 Find the area of a circle whose circumference is 22 cm.
60 [Ans. 38.5 cm 2]
Now, number of revolucions made in 1 min
Q. 4
revolutions must it take to cover 792 m.
Distance covered by wheel A wheel has diameter 84 cm. Find how many complete
= Circumference [Ans. 300]
of the wheel

NCE RT FO L D E R 8.1 -- - . . :T

. k
Unless stated otherwise, ta e 7t =
22
7· 56rt =28 cm
r =-
27t
1 The radii of two circles are 1 9 cm and 9 cm, Hence, the radius of new circle is 28 cm.
respectively. Find the radius of the circle which 2 The radii of two circles are 8 cm and 6 cm,
has circumference equal to the sum of the respectively. Find the radius of the circle
circumferences of the two circles. having area equal to the sum of the areas of the
Sol. Given, 1j = 19 cm and r2 = 9 cm two circles.
Let r be the radius of new circle. Sol. Given, 1j = 8 cm and r2 = 6 cm
:. Circumference of first circle, Let r be the radius of new circle.
c1 == 227t1x j7t x 19 = 38 7t cm 1 cm
Area of first circle, A = 7t1j2 = 7t (8)2 = 64 7t 2
and circumference of second circle, Area of second circle, A 2 = 7tr22 = 7t (6) 2 = 36 7t cm
2
c2 == 221trx 27t x = 18 7t cm According to che question,
9 Area of new circle = A + A 2
1

According to the question, rtr22 = 64rt 36rt


+
Circumference of new circle => rtr = 1 OOrt
= Circumference
+
of first circle
Circumference ofsecond circle
=> r2 =100
2rtr => r = 10 cm since :: ��
2rrr =387t
=567t 187t
+
[neglect - ve sign, radius
Hence, the radius of new circle is I 0 cm .
cannor be negative)
�\:.1 'j�-�
" r,
1! t�
\ 29 0 \ Allinone MATH EMATICS Class 10th Term II

3 I n the given figure depicts an archery target 4 The wheels of a car a re of diameter 80 cm each.
m arked with its five scoring regions from centre How many complete revolutions does each
outwards as gold, red , blue, black and white. wheel make in 1 0 min when the car is travelling
The diameter of the region representing gold at a speed of 66 km per h ?
score is 2 i cm and each of the other bands is
Sol. Given, diameter of wheel of the car, d1 = 80 cm
d
1 0 .5 cm wide. Find the area of each of the five
scoring regions . Then, radius of wheel of the car, r = __!_ =-BO = 40 cm
2 2
:. Circumference of wheel of the car
= Circumference of circle

[ ]
= 2 rtr = 2 22 40 = --
x - x
1760 cm
7 7
Also given, speed of a car = 66 km/h
= 66 1000 mI man
x .

·: I km I h =
1000 m I
-- nun
.

60 60
= l lOO m/min
= 1 10000 cm/min
Sol. Given, diameter of gold region = 21 cm [ ·: 1 m I min = 100 cm / min]

Then, radius of gold region = � = 1 O.S cm Now, wheel of car moves in I min = 1 10000 cm
2 ca r
:. Wheel of moves in 1 0 min = 1 1 00000 cm
: . Area of gold region = 7t r2 = 22 (10.5 )2 Then, number of complete revolutions
7 car
= Distance moved in 1 0 min by wheel of the
= x 1 10.25 = 346.5 cm 2
22 Circumference of wheel of the car
7 1 100000 = 7700000 = 4 375

[·: l
Radius for gold + red region = 10.5 + 10.5 = 21 cm =
1760 / 7 1760
[by given condition] Hence, the wheel makes 4375 complete revolutions in
22
:. Area for red region = [(21)2 - (10.5)2] 10 min .
7 5 Tick the correct answer in the following and
area of a ring = 7t (R2 -r2) justify your choice . If the perimeter and the
Where, R = radius of outer ring area of a circle are numerically equal, then the
and r = radius ofinner ring radius of the circle is
rt units
= 22 (21)2 - 22 (10.5)2 = 1386 - 346.5 = 1039.5 cm 2 (i) 2 units (ii) (iii) 4 units (iv) 7 units
7 7 Sol. (1) We know that, area of circle = rtr 2
Now, radius for gold + red + blue region = 21 + 10.5 and perimeter of a circle = 2 7tr
= 31.5 cm [by given condition] According to the question,
:. Area for gold + red + blue + blue region Area of circle = Perimeter of a circle
22 22
= 22 [(3 1.5 ) 2 - (2 1)2] = 7 (31.5)2 - 7 (21)2 7tr2 = 2rtr
7 r = 2 units
=>
= 31 1 8.5 - 1386 = 1 732.5 cm 2
Now, radius for gold + red + blue + black region Additional Questions
6 A boy is cycling such that the wheels of the
= 31.5 + 1 0.5 = 42 cm [by given condition]
� 22

·
. .Area for black region = [( 42)2 - (31.5)2 ) cycle are making 140 revolutions per minute. If
7
c.:;; 22 22
= (42)2 - (31 .5)2 = 5544 - 31 18.5 = 2425.5 cm 2
the diameter of the wheel is 6 0 cm, then find the
---1.d 7 7 speed per hour with which the boy is cycling.

0 Radi us for gold + red + blue + black + white region Sol. Given, diameter of wheel = 60 cm
LI- = 42 + 1 0.5 = 52.5 cm [by given condition] . of wheel (r) = Diameter = -
60 = 30 cm
. . Radms

�I
:. Area for whi te region =
22 2
[(52.5) - (4 2)2] 2 2
7 30 m = -3m
=-
= 22 (5 2.5 )2 22 (42 )2 = 8662.5 - 5544 = 31 18.5 cm 2
_
1 00 10
r_J �
7 7
Areas Related to Ci rcles � 2 91 i
We know chat, 1j2 + r22 :::: 130 . . . . (ii)
(lj + ri ) :::: 'i + r2 + 21jr2
Circumference of che wheel = 2 7tr = 2 x -22 x -3 = -132 m Now, 2 2 2
7 10 70 0 4) 2 = 1 30 + 21jr2 [using Eqs. (i) and (ii)]
Also, ic is given chac wheel makes 140 revolutions in 196 = 130 + 21jr2
1 min. 21jr2 = 196 -130
:. Distance covered by wheel in I min 21jri = 66 =:> r.r2 = 33 . . . (iii)
= N umber of revolutions in 1 min (1j -r2 ) 2 = 1j 2 + r22 - 21i r2
x Circumference of circle (1j -r2 ) 2 = 130 -2 x 33 = 64
1 32 264 km
= 140 x 70 m = 264 m = 1000 =:> 1j -r2 = 8 . . . (iv)
On solving Eqs. (i) and (iv), we have
264 x 60 km = 15.84 km
:. Distance covered in 1 h = 1000 r. ='l l , r2 = 3

=
8 The short and long hands of a clock a re 6 cm
Hence, the speed per hour wich which the boy is cycling, is and 8 cm long, respectively. Find the sum of the
15.84 km/h. distance travelled by their tips in 1 day.
7 1\vo circles touch each other externally. The [Take, n 22 I 7]
sum of their areas is 1 3 0 n cm 2 and the distance Sol. In 1 day, i.e. 24 h shore (hour) hand of the clock make 2
between their centres is 14 cm. Find the radii of revolutions and long (minute) hand make 24 revolutions.
the circles . In 1 revolution, distance travelled by cip of hour hand
Sol. Lee 'i• r2 be radii of che given circles and 0, O' be their = Circumference of circle of radius 6 cm
centres. =2x - 22 x 6
7
In 1 revolution, distance cravclled by rip of minute hand
= Circumference of circle of radius 8 cm
=2x- 22 x 8
.. 00' = 1j + r2
7
:. Sum of distances travelled by tips of boch hand in I day
[·: if cwo circles couch each ocher externally, then distance 22 x 6 + 24 x 2 x -22 x 8 = 2 x -(12
22 + 192)
between cheir centres is equal co the sum of their radii] =2x - 7
:::::> 'i + r2 = 14 . . . (i) 7 7
le is given char sum of areas of circles is 130n. 22
= 2x -7 x 204 = 1282.29 cm (approx.)
1t1j 2 + nr22 = 130 7t =:> 7t(1j 2 + r22 ) = 1307t

TO P I C EXERCISE
\&., .iL,... _ . ...... . ...__ _ , _ - - · • • . - • - - ·- • - ... . .. _
' .- · - -· -

Very Short Answer Type Questions 4 The outer and inner diameters of a circular ring

1 What will be the primeter of a quadrant of a


are 3 4 cm and 32 cm. respectively. Then, find the
circle of radius r?
area of ring. CBSE 2 0 l l
CCE 20 13

2 A garden roller has a circumference of 4 m. Find Short Answer Type I I Qu estio ns


5 The shaded area in the adjacent
the number of revolutions it makes'in moving
40 m. CCE 20l l
figure between the
Short Answer Type I Qu estions circumference of two
concentric circles is 346.5 c m � .
·. ! . '..'..: : .
3 If the perimeter of a circle is equal to that of a
:dL!
The circumference o f the inn er
11
square, then find the ratio of their areas. circle is 88 cm. Calculate t he ij .. :-1
NCERT Exemplar CCE 20 1 2 "'
H
radius of the outer circl e.
\ 2 92 \ AllinOne MATH E MATICS Class 1 0th Term 11

6 The short a n d long hands of a clock are 8 cm and 9 The diameter of a cycle wheel is 21 c m . How many
12 cm long, respectively. Find the sum of the revolutions will it make to travel 1.98 km? CCE 20l l

[ ]
d istances travelled by their t ips i n two days i.e. in
48 h. take, 7t 7
22
=
10 A wire when bent i n the form o f a square en c l ose an
area 121 sq cm. If the wire was bent i n the form of a

[
CCE 201 1
circle, then fi nd the area e nclosed by the circle.
7 Two parallel lines touch the circle at points A and B, 22-
respectively. If a rea of t h e c ircle is 257t cm 2 , t hen
take. 7t = 7
J CCE 201 1
f i n d the lengt h of AB. CCE 201 1
Long Answer Type Qu estio n
8 I f t he circum ference of a circle and the perimeter of
a squa re a re equal, then check whether it is correct 11 The ratio of the outer and i n ner circum ference of a
t hat a rea of c i rcle > a rea of the square. circular path is 23 : 22. I f the pa t h is 5 m wide, then
NCERT Exemplar find the diameter of the i n ner circle. CCE 2013

(% + }
Answers
1. 2 2. 1 0 3. 1 4 : 1 1 4. 337t 5. 17.5 cm

6. 382 1 .72 m 7 . l O cm 8. Yes 9. 3000 1 0. 154 cm 2


1 1 . 220

TOPIC 0 Are as of Sector and Segment of a Circle


S ecto r o f a Circle Area of Secto r of a C irc le
The region enclosed by two radii and the corresponding arc Let there be a circle of radius r having a minor sector with
of a circle is called the sector of circle. angle e ' then
c Q

p Minor sector p

_e x
In the above figure, shaded region OAPBO is a sector of the
_
Area of the sector = 1tr 2
circle with centre 0. Here, LA.OB is called the angle of 360 °
sector.The smaller sector is called minor sector and the Also,
bigger sector is called major s ector.
Area of the major sector = 1tr 2 - Area of minor sector
Here, in the above figure, unshaded region OA CBO is called
th e major sector and shaded region OAPBO is called the
mi n o r sector of a circle. Angle of minor sector is less than Area of minor sector = 1tr 2 - Area of major sector
1 80 ° an d angle of major sector is more than 1 80°. The sum
of angl es of major and minor sector is 360°. N ote • I f 0 = 1 80°, then sector becom es a sem i-c i rcular region
N o te • T h e su m o f arcs o f m ajor and m i nor sectors of a circle is and i ts area = � nr 2 •
2
e q ual co rh e ci rc u m ference of the circle.
• T he sum o f t h e a rea s of major and m i nor sectors of a • If 0 = 90°, then sector becomes a q uadrant of a circle
1 2
c i rcl e is e q ual co rh e a rea of the ci rcle. and i ts area - nr .
4
Areas Related to Circles f 2 93 /
Example 1. The length of min u te hand of a clock is
14 cm. Find the area swept by the min ute hand in one
TAY Y6UASELF
n = 22 Q. 1
[
minute. Take, 7 J Find the area of the sector of a circle with radius 4 cm and
of angle 30°. Also, find the area of the corresponding
major sector. [Ans. 4. 1 9 cm 2 , 46. 1 cm 2 J
The length of hour hand of a clock is 7 cm. Find the area
Sol. Clearly, minute hand of a clock describes a circle of radius Q .2
equal co its length, i.e. 14 cm. We latow char angle described swept by the hour hand in one hour. [Ans. 1 2. 83 cm 2 ]
by minute hand in 60 min = 360°.
Q .3 In a circle of radius 21 cm are subtends an angle of 60° at
:. Angle described by minute hand in 1 min = 3600
60
= 6° - the centre. Find the length of the arc. [Ans. 22 cm]

So, che area swept by che minute hand in 1 min is the area of a Seg ment of a Circle
sector of angle 6° in a circle of radius 14 cm.
8 , 6 ° 22
:. Reqmre d a rea = -- x n r· = -- x - x ( 1 4 )
. 2 The region bounded by a chord and the corresponding arc
360° 360° 7 of the circle is called the segment of circle.

=- 1 x- 22 x 14 x 1 4 = 10.27 cm·, In the above figure, shaded region APB is o


60 7 segment of circle containing the minor
-

Length of a n A rc of a Sector arc APB.


The segment containing the minor arc is
The arc corresponding to a sector is called the arc of the
called a minor segment and the segment
sector.
containing the major arc is called the
If the radius of a circle is r and the angle of sector is 0, then major segment.
M inor segment
In other words, we can say 'that, the
segment which is less than semi-circular regio n is called a
minor segment and the other segment is called major segment.
Area of a segment (ar a circle) = Area of corresponding
sector -Area of triangle formed by chord and the radii of
the circle.
a
Length of an arc of a sector, I = -- x 21tr Also,
360°
Area of major segment = 1tr 2 - Area of minor segment
Area of sector in terms of length of arc = .!. Ir
Area of minor segment = 1tr 2 - Area of major segment
Also,
2
Example 2. In a circle of radius 28 cm, an arc
Note When we wri te segment and seccor. we will mean the
subtends an angle of 45 ° at the centre. Find the length 'minor segmen t' and the ' m i nor sector' respectively. un less
of the arc and using it, find the area of the sector. stated otherwise.

[
Given, r = 28 cm and 8 = 45° Example 3 . Find the area of the segment AYB, if

J
Sol.
radius of the circle is 2 1 cm and LAOB = 1 2 0 °.
n 22
=
7
Y
Take,

22
8
:. Arc length = 360°
-- x 2 1tr = 45° x 2 x -
--
.
x 28
360° 7
.!.. x 2 x 22 x 4 = 22 cm
==
Sol. Given, r = 21 cm and L.AOB = 120°
8 : Area of the sector OA YB
.

Now, area of sector == ..!.. x l x r [where, I = arc length] = 120° x -22 x 21 x 21 = 462

[
cm
2 --

e x rc r 21
360° 7
_!_ x 22 x 28 = 22 x 14 = 308 cm 2
==
2 ·: area of sector = --
360° ...
· 294 Allin.One MATH EMATICS C lass 10th Term 11

p
Now, in tiOAB, OA = OB = 2 1 cm, LAOB = 120° . . Area of sector OAPBO = -l ( 22 x 14 )
Draw OM .l AB, then by RHS congruence, 2
MMO : l:l.BMO = 1 54 cm :.

[·: area of a seccor = 360° x ]


e
So, M is the mid-point of AB. => -- x nr-, = l 5 4
Also, LAOM = LBOM = -- LAOB = --
2
1 20° = 60 0 360°
2
Let OM = x cm y __Q_ 1tr 1

[ -i]
In right angled l!iOMA , cos 60°= OM e 22 x l4 x l 4 = 1 54
QA => -- x-
360° 7
0
- -- cos 60°= = 1 54 x 360° = 90 0
22 x 2 x 1 4
x =:>
=:> -1 = -
21
x =- => 0M = 3..!_
2 21 2 2 Now, area of right angled MOB = _!_ x OB x QA
2
AM x
= 1 4 1 4 = 1 4 x 7 = 98 cm ·'
Also, sin 60 ° =
OA
.J3 = AM 21.fj cm 2
=> AM = . . Area of segment APB
2 21 2 = Arca of sector OAPBO -Area of MOB
r.;
AB = 2 AM = 2 x -- = 2 1v 3 cm
2 1.[3
= 1 54 - 98

[:. J
2 = 56 cm 2
.!_
Now, area of l!iOAB = AB x OM = ! x 21 .J3 x 3..!_
2 2 Hence, the area of the segmcnc of the i rcl e is 56 cm 2 •
c

triangle= -i
2
x base x height
area of
TRY Y6URSELF
= 44 1 .Jj cm 2
4 Q. 1 A chord of a circle of radius 20 cm subtends an angle
Hence, area of segment A YB = Area of sector OAYB 90° at the centre. Find the area of the corresponding majOi

�(88
-Area of 6.0AB segment of the circle. [take , 7t = 3.1 4] NCERT Exempl':'
[Ans. 1 1 42.85 cm '
= 462 - 44 1 fj = - 21 .J3) cm2
4 4 Q.2 A chord 1 0 cm long is drawn in a circle whose radius is
Example 4. Find the area of the segment of a circle s.J2. cm. Find area of both the segments.
of radius 1 4 cm, if the length of the corresponding arc [take, 7t = 3.1 4] [Ans. 1 42.85 cm 2 , 14.28 cm =
APB is 22 cm. [Take, 7t = 22 /7]
Q.3 A chord of circle of radius 35 cm subtends an angle ot
Sol. Area of sector OAPBO = .!.. tr , where I is the length of arc and 1 20° at the centre. Find the area of corresponding minor
2 segment of the circle. [take, 7t = 22 7 and ../3 = 1 .73]
r is the radius of the circle. [Ans. 755.825 cm :

Unless stated otherwise, take 7t = 22 .


7

Fi n d the area of a sector of a circle with radius 2 Find the area of a quadrant of a circle whose
6 c m , if an gle of the sector is 60°. circumference is 22 cm .

Sol. We k n o w rha r. area of seccor of a circle = -- x 7tr 2


9 Sol. Given, circumference of a circle = 22 cm
360° 22 x r = 22
=>
G i ven , rad i us o f circl e, r = 6 cm and angle of sector, 9 = 60° => 21tr = 22 2x-
7
:. Arca o f sec to r of a c ircle == -
60° 22 6) 2 132 2
6 x - x ( = - cm 22 x 7 = -7 cm
3 0
0
7 7 r = --
2 x 22 2
Areas Related to Circles

2
2
Now, area of a quadrant of a circle = 1tr (ii) Area of major sector OAFGCO

(fl'
4 =Area of circle - Area of sector OAECO
= 7tr 2 - 78.5 = 3.14 x (10) 2 - 78.5
= 314 - 78.5 = 235.5 cm
= 227 x 4 = !! 49 77 '
14 x 4 = 8 cm 5 In a circle of radius 2 1 cm, an arc subtends an
3 The length of the minute hand of a clock is
angle of 60°at the centre. Find
14 cm . Find the area swept by the minute hand (i) the length of the arc.
in 5 min. (ii) area of the sector formed by the arc.
(iii) area of the segment formed by the
Sol. Given, length of the minute hand of a clock = 14 cm
le means radius of circle, r = 14 cm
corresponding chord .
Given, arc ABC subtends an angle 60° at the centre.

e
Sol.
· : Angle moves by the minute hand in 60 min
:. e = 60° and radius, r = 21 cm
= 360°
:. Angle moves by the minute hand in 1 min (t) Length of arc ABC

= 3600
60 = 60 = --
360° x
2rtr
Then, angle moves by the minute hand in 5 min 60°
= -- 22 x 21
360° x 2 x -

,
= 5 x 6° = 30° 7
Now, area swept by the minute hand 44
= - x 3 = 22 cm
= Arca of sector of the circle 6
0 30° x -22 x 14 x 14
= --
360°
x 7tr · = --
360° 7
(ii) Area of the sector formed by a
the arc
22 x 14 x 2 616 = -1 54 2 e 2 60° 22 x (21) 2 .
= --
360° x rrr = --
= 12 - - 1 2 3 cm 360° x -

=
7
22 9702
4 A chord of a circle of radius 1 0 cm subtends a
42 = 231 cm 2
= --
6 x 7 x 21 x 21 = --
right angle at the centre . Find the area of the
(iii) Given, LAOC = 60° and OA OC (radii of circle)
corresponding
(i) minor segment.
Lee LOAC = LOCA = x
(ii) major s�ctor. [Take, 7t = 3 . 1 4 )
In .6.0AC,
Sol. Given, radius of a circle, AO = 10 cm and LAOC =90° LOAC + LAOC + LOCA = 180°
=> 60° + x + x = 1 80°
We know that a perpendicular drawn from the centre of => 60° + 2x = 1 80
circle co the chord of the circle bisects the chord. => 2x = 1 20=> x = 60°
F__
_
LA = L 0 = LC = 60°
:. .6.0AC is an equilateral triangle.

[.·. area of equilateral criangle = � (side)'- ]


Now, area of .6.0AC = .J3 x 2
4 (21)

- -
e_
= 44 1 .J3 cm�
E

Area of MOC = 21 x OA x OC = 1 x 1 O x 1 O = 50 cm 2 4

( "'3}m'
2 Hence, area of the segment
_
Area of sector OAECO = 360° x 7tr 2 =Area of sector formed by the arc - Area of .6. 0A C
90° x
= -- 2 = 231 - 44�
360° 3. 14 x (10)
=-314 = 78.5 cm 2 6 A chord of a circle of radius 1 5 cm subtends a n
4 angle of at the centre . Find the are as o f the
60°
(z) Area of minor segment AECDA corresponding minor and major s egme nts o f
= Area of sector OAECO - Area of .6.AOC the circle . [Take, 7t = 3 .1 4 a nd J3 = 1 . 7 31
= 78.5 - 50 = 28.5 cm 2
296 \ Allt�none MATH EMATICS C lass 10th Te rm 1 1

2
(I )�
3 0° 3.1
o
x x
I
Sol. Given, chord AB subtends an angle 60° ac the centre. Area ofseccor OA CBO = --

60° rc r = -- x

.. L AOB = 60° � 2. l
and OA = OB [radii of circle] = = 150.72 cm
=::> LOBA = LOAB = (say) x
. . Area of t h e co rrespo nd i ng gmenr
Now, = Area of s c c r 'A CBO - Area of 6 A OB
LAOB LOBA LOAB = 1 80°
60°
+
+x+x = 180°
+
= 60°=> x
= 150.72- 62. 2 = cm .44 2
:. �OAB is an equilateral triangle. 8 A horse is tied to a p g t n orner o f a squ are
shaped grass field of s i d 1 5 m b y means of a
Now, area of �AOB = J34 (1 5 ) 2 x
5 m long r o p e {see th fi g u r ) . Find
[·: area of equilateral triangle
= .J3
4 (side)2]
= = 97.3125 cm2
56.25 x 1.73
a 2
Area of sector OACBO =_ 360°_ 7tr x

600 3.14 (1 5 ) 2
= 360° -- x x
(i) the area of the part of the field in which th
= 3. 14 225 = 706.5 = 1 17.75 cm 2
x horse can g raze.
6 6 (ii) the increase in the g ra z i n g a rea if the rope w

= 225
. . Area of minor segment ACBDA m long instead of 5 m . [Take, n = 3.1 4)
10
=Area of sector OACBO -Area of LiOAB Sol. Given, side of a square = 15 m
= 1 17.75 - 97.31 25 = 20.4375 cm 2 . . Area of square = ( 1 5) 2 m2
[· : area of square = ( ide
Now, area of major segment -

= Area of circle - Area of minor segment Also given, length of rope = m 55


= (15 ) 2 - 20.4375 = 3.14 225 - 20.4375
7t x .. Radi us of arc = m
706.5 - 20.4375 = 686.0625 cm 2
= (t) ea
Ar of the field graze by the horse,
e 90° 3.1 4 (5
2
x x )·

= 3.1 4 4 25 78.4 5 19.625


A 1 = -- x 7t r = --
7 A chord of a circle of radius 12 cm subtends an 360° 360°
}
angle of 1 2 0° at the centre. Find the area of the
[ · : each angle of a square i 9
corresponding segment of the circle.
x
[Take , 7t = 3.1 4 and .J3 = 1 .73) = = cm i
Sol. Given, chord AB subtends an angle 120° at the centre. {it) If length of rope = l O m = tj (say)
1.e. L AOB = 1 20°
..L AB
OD l LAOB such
bisects =ABLBODwel=60°. as as
Then, area of the field graze by the horse,
e
Draw
chat AD = BD and itLAOD
and 90° (I O)·
Also radius ciO,rcle, = 1 2 cm r
A2 =
360° 360° 3.1 4
-- x nr. 2 = -- x x

In rightgiven,angled �AD
of
=
3.1 4 100 314 785
4 4
x
= = cm 2

= AD =::> J3 = AD
sin 60 0 . . Required increase i n the grazing area
cm
AO 2 12 = A i - A1 = 78.5 -19.625 58.875
=
2

=> AD = 6./3 = 6 1.73 x

= 1 0 .38 cm 9 A brooch is made with s ilver


and co s 60° = OD => - OD
1 =- wire in the form of a circle
AO 2 12 with diameter 35 mm. The
:::::> OD = 6 cm wire is also used in ma kin g
:. Lengrh of chord, AB = 2 AD = 2 10.38 = 20.76 cm x 5 dia met e rs which divide the
-=�1 fl
. ::;; r,1 and area of MOB :::: .!.. AB OD circle into 10 equal s ectors a s
;;:' 1 ' x x
shown i n the figure . Fin d
. , 1·!
2
(i) the tot a l length o f the s ilver wire required.
= .!.. x 20.76 6 =62.28 cm 2
x
2 (ii) the area of each sector of the brooch.
�; I .
f 2 97
-- --
Areas Related to Circles
I

e
Given, diamcrer of circle, d = 35 mm = 360° x rcr = 1 1 5° x -
2 22 x 2

360° 7 (25)
Sol.
. . Circumference of circlr· = red [·: d = 2r]
=- 22 x 35 = 1 1 0 .,mm ·
= 237 xx 7222 x 625 = 23 x7 1x1 x 625
7 36
2
· Now, lengrh of 5 diamerers = 5 x 35 = 175
(1) Toral lengrh of rhc silver wire = red + 5d
mm
= 1 58125
252 cm
2
= 1 1 0 + 175 = 285 mm :. Toral area deaned by both blades
(ir) Here, we sec rhac rocal circle is divided inro = 2 x Area cleaned by one blade
1 0 seccors. = 2 x 1 58125 1 581261 25 cm 2

--
360° = 36°
:. Angle of each seccor = -- 252
10 1 2 To warn ships for underwater rocks, a light
Then, area of each sccror of rhe brooch = e x rcr 2
:60 0 ; ( n [- : f ;
house spreads a red coloured light over a s ector
360° of angle 80° to a distance of 1 6 .5 km . Fin d the

--
'

.
= 6° x 2 3 r = = 3 mm J area of the sea over which the ships are warned.
[Take, re = 3.1 4)

--- - --
=-22 x 1 225 Sol. Given, sector angle, e = 80°
70 4 and disrance or radius, r = 1 6 5 km
26950 385 mm ·., e
:. Area of sector = 360° x rtr 2 = --so0 x 3.14 x (I 6 .5 > 2
= 4
= 360°
70 x 4
= 2 x 3.14 x 272 . 25 = 1 709. 73 = 1 8 9.97 km 2
1 0 An umbrella has 8 ribs which are equally 9 9
spaced (see the figure) . Assuming umbrella to
be a flat circle of radius 45 cm, find the area which is rhe required area of the sea over which the ships are
between the two consecutive ribs of the
warned.
umbrella. 1 3 A round ·t a b le cover h a s s ix
equal designs as s hown i n the
figure . I f the radius o f the .
cover is 28 cm, fin d the cost o f
making t h e d e s i g n s a t t h e
rate of � 0.3 5 per cm 2 •

-- =
[take , .J3 = 1 . 73]

-- =
Sol. Given, u mbrella to be a flar circle. So, the cencral angle of a
Sol. W e know that t h e central angle o f ci rcle is 360 °.
an u mbrella is 360°. 360°
Si nce, u m b rella has 8 ribs. . . Angle of each sector = 60 °
450
6
360 0
e
: . Angl b etween tw o n' b s ==

-- =
8
Area between two ribs

45°
= Area of one sector of the umbrella
e )
-- x rc r x - x ( 4 5 2 [ ·: r 4 5, given]
2 22
=360°
=
360 ° 7
= _E_ ( 4 5 )2 == 222 75 cm 2
7x 8 28 Now, we determine the area of one segment of a circle.
LAOC = 60°
1 1 A car has two wipers which do not overlap. and OA OC = [radii of circle]
Each wiper has a blade of length 25 cm => LOAC L OCA x
= = (say)
LAOC L OA C + L OC4
sweeping through an angle of 1 15°. Find the + = 1 80°
total area cleaned at each sweep of the blades. Now, 6 0° + x + x = I80 °

Sol. Given, lengch of wiper blade = 25 cm = r (say) => x :: 60 0

and angle made by this blade, e = 1 1 5 ° :. �OAC is an equilateral triangle.


: . Area cleaned by one blade
Area of MOC = ..J3 x ( 2 8) 2 :: 333. 2 cm
2
= Area of secror formed by blade 4
Alllnone MATH EMATICS Class 1 0th Term 11

Now, area ofsecror OABCO == -9- x 7tr 2 1 4 Tick the correct answer in the following question.
360° Area of a sector of angle p (in d egrees) of a circle
60° 22 x (28)2 22 x 4 x 28
= 360 0 x 7 = = 410.67 cm2 with radius R is
6 (i) _E_ x 2 7tR (ii) _.!!_ x rtR 2
. . Area of segment ABCA 1 80° 1 80 °
= Area of sector OABCO -Area of ll AOC (iii) _E_ ( iv) _E_ x 2 7tR 2

e
x 2 1tR
= 4 10.67 - 333.2 = 77.47 cm2 360° 720°
Now, area of six segments = 6 x 77.47 464.82 cm 2 == Sol. (iv ) Given, sector angle = p and ra<lius of circle = R
x 7t r - = P x 1tR - = --
P
Since, rhe cost of making the design is ' 0.35 per cm2• :. Area of sector = --
,
--
, l

:. Total cost = 464.82 x 0.35 = � 162.68 360° 360° 720° x 21tR

TO P I C EXERCISE
Very Short Answer Type Questions 4 Area of a sector of a ci rcle of rad iu s cm is 54n cm2 • 36
1 Find the length of the correspond i ng arc of sector.
I n a circle of radius 42 c m , an arc subtends a n CCE 201 1
a ngle of 120° at t he centre. Find the length o f the
a rc of sector. 5 The minute hand of a clock i s 1 2 c m long. Find the area

2 I n the g iven figure. 0 is the centre of the circle


of the face of the clock described by the minute hand
CCE 20 1 2
with radius equal to 14 cm. The length of the arc
between 9 am and 9 : am. 35
AB 13.2 cm. Find the area of the shaded sector
= Short Answer Type II Qu estio n s
6 A chord 1 0 c m long i s d rawn i n a c i rcle whose radius is
o f the c ircle.

.J50 cm. Find the a rea of segment. CCE 2013


7 Fi nd the area of the m i nor segment of a c ircle of radius
14 cm, when the angle of the corresponding sector is 60°.
NCERT Exemplar

Long Answer Type Qu estion s


8 A chord o f a circle o f rad i u s 3 0 c m subtends a n angle of
Short Answer Typ e I Qu estions 60° at the centre. Fin d the a rea of the corresponding
minor and major segments of th e c i rcle. [Take, 7t = 3.14
3 A cow i s tied w i t h a rope of length 1 4 m at the and .J3 = 1.73]
corner of a rectangular f ield of dimensions
20 m x 16 m. Find t h e area of the field in which 9 A chord of a circle of radius 10 cm subtends a right
the cow c an g ra z e. NCERT Exemplar angle at the centre. Find t he a rea of major segment.
[Ta ke, 7t = 3.14]

- - - - - - . - . - - - ·- - - - - . . - - - - - ·- - - . -· - . .. - - - - - - -

Ans wers
88 cm
2. 92 . 4 cm2 3. 154 cm2 4. 3n c m 5. 2 6 4 cm2
1.
6 2) :! 7. 1 7. 89 cm 2
. -- cm
2 5(rt -
8. 8 1 . 75 cm2 , 2744.25 cm 2
--

9. 2 8 5 . 5 cm2
TOPIC 0 Areas of Combinations of Plane Figu res

n
In our daily life, we co me across various plane figures which Given, side of square = 10 cm
are combination of two or more plane figures. e.g. window Diameter of each semi-circle = 10 cm
desig s, flower beds, d rain covers, circular paths, designs on :. Radius of each semi-circle = 10 = 5 cm
table covers, etc. For calculating areas of such figures, first

( )
2
we identi fy the plane figures wh ich are given in combination Now, area ofl region area oflll region =Area of ABCD
+

22
and then find area. -Area of two semi-circles each of radius 5 cm
= 1ox 10 - 2 x i "x 5 '
Methods to solve such problems

(Type ) When Combination of Circle/Semi-circle


according to given combin ation are given below
I
= (100 -3. 14 x 25) = (100 - 78.S ) = 2 1.5 cm
and Square/Rectangle are Given Similarly, area of II region + area of IV region = 2 1.S cm
If combination of circle and square or semi-circle and square .·. Area of the shaded design =Area of ABCD
or circle and rectangle or semi-circle and rectangle is given, -Area of (I + II + III + IV) region
then find the area of separate figures and then add/subtract = (100 - 2 x 21.5) = (100 -43) = 57 cm 2
them according to the situation given in the question to get
area of required figure.
2
[Type J When Combination of Two Concentric
II
Area of square = (Side) Circles are Given
Area of rectangle = Length x breadth Sometimes combination of two concentric circles, i . e. two
circles with same centre are given. Then, method to solve
Examp le 1. Find the area of the shaded region in such problems can be understood with the help of following
figure as shown below where ABCD is a square of side examples.
·

1 2 cm .
E x a m p le 3. In the given
figure, sectors of two
concentric circles of radii 7 cm
and 3.5 cm are shown . Find
the area of the shaded region .
Sol. Area of shaded region

J [ (7-)2 �J
2

[
Sol. ·.· Area of square = (S i de) [ by using the formula]
2 ::: (Area of sector having r = 7 cm, e = 30 °)
:. Area of square ABCD = 1 2 x 1 2 = 1 44 cm - (Area of sector having r = 3.5 cm, e = 30 °}

( ) ·:
[ o s t r=- ]
·: Diameter of each circle = 122 = 6 cm [here, length = 1 2 cm]

- =-
== 22 x ( 7) 2 x � - 22 x x
:. Radius of each circle (r) = 2 == 3 cm ·� 7 360 ° 7 2 360 °

8 x rtr 2

--
77 77
2 area f a ec o -

Area of one circle = rtr


• 2 22 1 98 = -
'7
= x3x3 cm 6 24 360 °
7
Therefore, area of four ci. rcles = 4 x cm
1 98 79 2
=
2 =
308 - 77 23 1
� 77 == 9.625 cm 2
24 24 8
7 7
Hence, area of the shaded region Examp le 4. In figure,
=Area of square - Area of four circles AOB is a flower bed in the
A
_ 216
= 30.86 cm 2
792 1 008 - 7 9 2
= 1 44 = = shape of a sector of a circle
7 7 7
of radius 40 m and
Examp le 2. Find the area of LAOB = 6 0 °. Also, a 1 5 m
shaded design in the given figure, wide concrete track is made
where ABCD is a sq uare of side as shown in the figure.
10 cm and semi-circles are drawn Flower bed is made at the
with each of the side as diameter. rate of � 2.40 per m 2 and

,
[Take, 1t = 3.1 4] CCE 20 1 1 rate of making the concrete
track is � 2 0 per m 2 • Find the total amoun t spen t for
0
Sol. Let the four unshaded regions be denoted by I, II, I I I and IV as
the job. [ ta ke n = 3 .1 4]
shown in the figure.
3 00 All{nOne MATH EMATI CS Cl ass 1 0th Te rm

Sol. Given, radius of circle = 40 and angle of sector = 60°


m Example 5. A mem ento is

Area of the sector A OB = -- 60 ° x 3.14 x ( 40 ) 2 made as shown in the


360° figure. Its base PB CR is
= -61 x 3.1 4 x 1600 = -- 25 12 m·., silver plated from the front
3 side at the rate of ' 2 0 per

[
Amount spent for mahlng the flower bed at the rate of cm 2 • Find the total cost of

J
'2.40 per m2 = ' 2.40 x 2531 2 = ' 2009.60 the silver plating.
"
B

{ } = 3 = 10 cm
Angles for the major sectors of both the circle at 0 is same, Take, = 272
i.e. 300°. From the figure, we have
Area of concrete track Sol.
300° X 7tX (4 Q )-., - --
300° X 7tX (252) AB 7 +
= --360° 360° AC = 3 = I 0 cm
7+
[·.· area of track = area of outer sector - area of inner sector, LBAC = 90°
Also, radius of circle = AP = AR = 7 m

=
radius of the outer circle = 40 m and radius of
the inner circle = 40 -15 = 25 m] . . Area of right angled MBC = .!_2 x AB x A
= �6 X 7t X (1600 -625) = �6 X 3.14 X 975 = 255 1.25 m 2 I
- x I x I = 5 cm
Amount spent for making the concrete track at the rate of 2
' 20 per 2 = 20 x 255 1.25 = � 5 1 025
m
90° x = -I x 7t x (7) -
Thus, the total amount spent for both the jobs Area of the sector APR = -- m· -

= ' 2009.60 + � 5 1 025 = ' 53034.60


360 0 4
I 22
= -4 x - x4
TRY YOURSE LF 7
1 1 x 7 = 85 cm·
= --
Q. 1 A paper is in the form of a rectangle ABCD in which 2
AB = 20 cm and BC = 1 4 cm. A semi-circular portion with Then, area of base PBCR (shaded) w h i ch i c be si lver plat
BC as diameter is cut-off. Find the area of a remaining = Area of the right a gled tJ. BA C
n

part. [Ans. 203 cm2 ) - A rea of che seccor APR

Q. 2
=50 -38.5 = 1 1.5 cm 2
A rectangular park is 1 00 m by 50 m. It is surrounded by
semi-circular flower beds all round . Find the cost of :. Total cost of silver plating at the ra ce o f � 20 per cm -
levelling the semi-circular flower beds at � 60 per m2. = '20 x 1 1.5 = ' 230
[Take, 7t = 3.1 4] [Ans. � 5887.50]
Example 6 . Find the area of the

[ ]
Q. 3
From each of the two opposite shaded region in figure, if AC = 2 0
corners of square of side 8 cm, a cm, AB = 1 5 c m and 0 i s the centre

7t
q uadrant of a circle of radius 1 .4 cm
is cut. Another circle of radius 4.2 cm of the circle. rake, = 272
CBSE 20 1 1
is also cut from the centre as shown
in the figure. Find the area of the Sol. It is clear from the figure, BC i s
remaining (shaded) portion of the diameter of the circle
�--_ sq uare. [take , 7t = 22/7] CBSE 201 0 [Ans. 5.48 cm 2 )
:. LBAC = 90°
[Type III ] whe n Com bi nation of Circle and [·: angle in a sem i-ci rcle i s a right an l
Now, in right angle !J.BAC,
(BC)2 = AB2 + AC2 [ u s i ng Pychagoras ch
Triangle are Given o re n1

BC = �(1 5)2 + (20)2 [·: AC = 20 cm , AB = 1 5 c m ]


So me time s comb i nat ion of circle and triangle is given, then
m eth o d to solve s uch problems can be understood with the
BC = .JG25
help o f fol lo wing examples.
:::> BC = 25
Area of an eq uila rera l trian gle = .,/3 (Side) 2
4 :. Radius of the circle, = 252 cm
r

Area o f a triangle = J. x Base x H eight Now, area of shaded region = Area of s e m i - c i rcle
2 - A rea of MB
Areas Related to Circles f301 '

= - - -2 x AB x AC = -21 x 227
nr 2
2
1 25 25 1 20 1 5
- x - x - - - x
2 2 2
x
Sol. ·: Diagonal of a square = OB = Radius = Side .Ji = 14.Ji cm
[·: diagonal of square = side J2.J
6875
=

28
- 1 50 = 245.53 - 1 50 :.Area of shaded region.
= Area of quadrant OPBQ -Area of square OABC
=95.53 cm 2 2 - (Side) = .!.. 22 (14./2) 2 - (14) 2
(Type IV) When Combination of Quadrant of
= .!_7tr
4 4 7
x x

Circle and Triangle/Square is Given = !..!. 14 14 2 - 196 = 308 - 196 = 1 12 cm 2


x x x
14
Note
Sometimes, a quadrant of circle is given and a
• The m id·poin c of the hypotenuse of a right a ngled triangle i s
triangle/square is formed in it. Then, to find required area,
equidistant from t h e vertices of a triangle.
we subtract the area of triangle/square from area of • Angle subtended at the circumference by a d i am eter i s
quadrant.

;�
always a right angle.

!�
Example 7. In the adjoining

,
figure, OACBO represents a
TAY YOURSELF

- -8
quadrant of a circle of radius
4.5 cm with centre 0. Calculate 3 cm Q. 1 Find the area of shaded region in the figure below if
the area of s h a d ed portion . PQ = 1 6 c m PR = 1 2 cm and 0 is centre of the circle.
[take, 1t = 3.14]
0 CBSE 201 1
[Ans. 61 cm2]
n
[Take, = 2 2 / 7 ] 4.S cm

Sol. Area of qu a ra
d nt OACBO = -7tr 1 2
4
= .!..4 227 (4.5) 2 =15.9 1 cm 2
x x

and area of llBOD

= .!_ BO OD = .!_ 4.5 x 3 = 6.75 cm 2


x x x
2 2
Hence, area of shaded portion Q. 2 In the given figure, PQRS is a rectangle of length 1 oJ2 cm
= Area of quadrant OACBO - Area of !lBOD and breadth s.J2. cm. If PEQ is an isosceles triangle
= 1 5.9 1 -6.75 = 9.16 cm 2 inscribed in the semi-circle with diameter PQ, then find the
area of the shaded region. CBSE 201 5

��
Example 8. A s q uare OABC p Q
is inscribed in a q uadrant

-
OPBQ of a cfrcle as shown in
'figure. If OA = 1 4 cm, then find
the area of shaded region. s E R 2]
[Take, 7t = 2 2 !7]
. 200 cm
Q
[ Ans
c
7

L D E R 8.3
Unless stated otherwise, take
7t 22
= .
7
1 Find the· area of the shaded We know that any angle made by the diam rer RQ in t he
region in the given figure, if semi-ci rcle is 90°.
PQ = 24 cm, PR = 7 cm and O is .. L RPQ = 90 °
the centre of the circle . In igh t a n gl ed llRPQ
r
RQ 2 = PR 2 + PQ [by Pythag ras theorem]
Sol. Given, PQ = 24 cm and PR = 7 cm
=> RQ 2 = 7 2 + 24
1 302 1 Alll110ne MATH EMATICS C lass 1 0th Term II

42
=> RQ 2 = 49 + 576 RQ 2 = 625 => RQ = �625 Hence, area of shaded region
= Area of square -( Arca of s i- ir l APD
=>
em c c
RQ = 25 cm [·: side cannot be negative]

2
=>
+ Arca of semi-circle BPC
. . Area of right angled 6.RPQ = ! x RP x PQ
2 = 196 - (77 + 77) = cm�

( )
= ! x 7 x 4 = 84 cm 2
4 Find the area of the shaded
2 region in figure, where a
Area of semi-circle circular arc of radius 6 cm has

--
been drawn with vertex 0 of
= 7tr = � 25 2
2
2 7x 2 2 an equilateral triangle OAB
of side 1 2 cm as centre .
= 1 1 x 625 = 6875 cm 2
28 28 Sol. Given, radius of circle, r = 6 cm
Hence, area of the shaded region and side of equilaceral triangle A 1 2 cm
= Area of the semi-circle -Area of right angled ll.RPQ = 1 2 cm

-
= 6875 84 = 6875 28- 2352 = 4523 cm 2

4
_ Since, OAB is an cquilaccral triangle.
28 28 .. LO = LA = LB = 60°

(12)-
2 Fin d the area of the
shaded region in figure, if
Area of an equilateral 6.0AB = .J3 (Sid )
radii of the two concentric .J3 6 .J3 ·
circles with centre 0 are
cm
4
=
7 cm and 14 cm A Area of circle oucsidc che triangle
=Arca of circle - Arca of OPQO

[22 J
sec t o r
respectively and
LAOC = 40°. 60° 1t -, 1t - - = 5 1tr.
= 7tr -, - --
360° x r = r 6 -6
_ 1tr

Sol. Given, OB = 7 cm, OA = 14 cm


andLAOC = 40°
e
=
2_
x (6) .? = 1 160xx 736 = 660
7 cm
2

Now, area of sector OED = -- i 6 7


360° x 7tr

( )
Hence, coca! area of the shaded region

--
= 40 0 x 22 x (7) 2 = 1 54 cm i = Area of an equilaceral D. OAB
+ Area of circle outside the rrian
360° 7 9
e h 660 = 1 56.56 cm ·.,
and area of sector OAC = 360° x 7tr 2 = 36v3 + ?
40 ° x 22 x 0 4 ) = 22 x 28 = 616 cm 2
2
5 From each '"'
A �--.-----.--·
= 360° 7 9 9 corner of a
square of side 4 c m , a
Hence, required area of shaded region quadrant of a circle of
= Area of sector OAC -Area of sector OBD radius 1 cm is cut and also a
=-616 - -1 54 <= --
462 = -
1 54 cm 2 circle of diameter 2 cm is
9 9 9 3 cut as shown in figure . Find
the area of the remaining 0 L-_.._ __.__J,_ _
_

3 Find the area of the shaded A r--::::---� a

2 22
portion of the square .
region in figure , if AB CD is a
square of side 1 4 cm and
·
Sol. Given, side of a square = 4 cm
APD and BPC are semi­ Radius of quadrant, 'i = 1 cm

--
circles .
and radius of circle of diameter cm, r2 = - = 1 cm

.
Sol.
.._ .. ,

( )
Given , side of square = 1 4 cm 0 c
Also, APD and BPC are semi-circles, therefore Area of one quadrant = - m: 2 22 (1) 2 = -
22 cm
2

4 =7x4
1
' r=- 14 = 7 cm.

22
28
Their rad ms,

--
2 22 = 22 2
No w' a rea of semi-circle APD Then, area of four quadrants = 4 28 7
cm

1t r 2 22 ( 7 )2 = 77 2
. . rc1 e BPC = -= cm 22 (1) 2 = 7 2
= Area o fsem i-ci
2 7x 2 Now, area of circle = 7tr22 = 7 cm

and area of sq uare ABCD = (Side) 2 = (14) 2 = 196 cm 2 and area of square ABCD = (Side) 2 = ( 4) 2 = 1 6 cm
Areas Related to Ci rcles
f303

)
:. Ar a of h a d e d region = Arca f quare : . Area of shaded region
- (Arca of fo u r q uad rants + Area of circle)
(
= Area o f circle -Area o f M BC
44 l 12 - 44
= 16 - = 16

(�
+
22 22 _ 68 [·: area of circle = n radi us 2 )

)
= = cm2
= (32) 2 - 76B.J3 =
7 7 7 7 7 22
n (1 024) - 768 .J3
6 In a circular table c ov r
7
of A
radius 3 2 cm, a d esig n i s
22 28
= - 768,J3 cm '
formed leaving an
equila teral MB C in t h e
7 In the adjoining fig ure ,
mid dle s shown in t h e
ABCD is a square of side
figure . Find t h e are a of the
14 cm . With centres A, B, C
desig n .
and D , fou r circles are
Sol. of ehc c i rc l e = 3 2 cm
ive n , rad i u drawn such that each circle
Lee the side o f ehc quil a ec ral MBC be
e a cm. touch externally two of the
Lee h be ehe he i ght of the e r ia gle . n remammg three circles.
A Find the a rea o f the shaded region .

aj \a
Sol. G iven, side o f square = 1 4 cm

j h \
i.e. AB = BC = CD = DA = 14 cm
o :. Radi us of circle = _!_ (Side of a quar ) = 1 4 = 7 cm
2

(�)
2
2
nr 2 = --
Area o f qua d rant o f one ci rcle = -
. 22 1 54
( 7) 2 = - cm
4 7x 4 4

We know t h ae in an equilateral criangle, centroid and


:. Area of four quadrants of fo u r ci rcles = 4 ' 4
circu mce n cre coincide. = 1 54 cm 2
AO = -2 hem Now, area of square = (S ide)2 = (1 4 ) = 1 9 6 cm 2
3
[".· centroid divides ehe median in the ratio 2 : 1] Hence, area of shaded r gion
which i s equal co the radius of circle. = Area of square - Area of fo u r quadrant
= 1 96 - 1 54 = 42 cm 2
.. 3. h = 32
3
h = 48 cm . . . (i) 8 Following figure depicts a racing track whose
Now, we d raw a perpendicular from vertex A to side BC left and right ends a re semi-circular. The
which b isects BC. distance between two inner p a rallel line

W + h'
I n right a ngled MDB, segments is 60 m and they a re each 1 06 m lon g .
AB 2 = BD 2 + AD 2 [by Pythagoras theorem] I f the track is 1 0 m wid e , fin d

�--))
a' =
� 2

h2 = a2 � = �
-

(i) the distance around the track along it inner edge.


4 4
( 48)2 = 3a2 [from Eq. (i)J (ii) the area of the track.
4
a2 = 3072 Sol. G iven , l ngrh o f a h parall l lin s = 1 0 m

and width of th era k = 1 0 m


=> a = .J3072 = 32J3 cm
. . Area of equilateral MBC 1 0m
A
= .J3 ( a ) 2 = ..J3 (32..J3)-
1 06 m -'----_.. B
4 4 1p t
T 60 rn
= J3 x 3072 t
4
D
= 768.}3 cm 2
All{none MATH EMATICS C lass 1 0t h Te rm I

[ ��)]
(t) Distance around the track along its inner edge Sol. Given, OA = 7 cm
= PQ + RS + 2 x Circumference of semi-circle POR .. OD = 7 cm [ radii f a m ci

(H
le then radius = ?.. an

:
2n ·: OD is the diameter of smaller c i r
2
= 106 + 1 06 + 2 Now, area of smaller circle
= __2 x __9 = _ cm 2
2 4 7
= nr ' = n
= 2 1 2 + 60 7t = 2 1 2 + 60 x - 1320
22 = 212 + -- 7
7 7 Now, area of llABC = I x AB x 0
-

1 484 + 1 320 2804


--
2
= = m
7 7 = _!_x 2 x OA x 0 [·: AB = 20
(ii) Area of the track =Area of outer track 2
m2
[·: QA = OC]
- Area of inner track = _!_ x 1 4 x 7 = 4 9
= [Area of rectangle ABCD 2

�( 60 : 20 J]
+ 2 x Area of semi-circle ATD] and area of semi-circle ABCA = 7tR 2 = � x (7)�
[Area of rectangle PQRS
- 2 7x 2

[
+ 2 x Area of semi-circle POR] = 77 cm 2
:. Area of minor segment corresponding to the chords BC
= 1 06 x (60 + 20") + 2 x and AC = Area of semi-circle - Ar f MBC
[ wm
a

= 77 - 49 = 28 cm 2
- 106 x 60 + 2 x Hence, area of coral shaded region
= Area of small circle + Aiea of minor segment

( : )( : )
corresponding co che chords BC and AC
= - + 28 = 38.5 + 28 = 66 . 5 cm 2
[·: area of rectangle = length x breadth]

= ( 1 06 x 80 + 2: x 40 x 40 ) - (1 06 x 60 + 1; x 1800 )
77
= 1 06 x 80 + 2 x 40 x 40 - 106 x 60 + 2 x 30 x 30 2

= ( 8480 + 35�00 ) - (6360 + 1 9�00 )


1 0 The area of an equilateral
.1 ABC is 1 7 3 2 0 . 5 cm 2 •
With each vertex of the
triangle as centre, a circle
is drawn with radius

= 2 1 20 + 1 5400 = 2 1 20 + 2200 = 4320 m 2


equal to half the length of
7 the side of the triangle
(see the figure) . Find the
Alternate Method area of the shaded region .

.Jf
Area of the track = 2 x Area of rectangle APQB [Take, 7t
= 3.1 4 and .J3 = 1 . 7 3]
+ 2 x Area of semi circular ring ATDROPA Sol. Let the side of an equilateral triangle be a.

[ ]
= 2 x 1 06 x 1 0 + 2 x �[(40) 2 -(30) 2 ]
2 :. Area ofan equilaceral MBC = ( a ) '
x
·: area of rectangle = l b and area of Bue given, area of equilateral MBC = 1 7320.5 cm 2
semi - circular right = �(R2 - r2)
2 17320. S - 1 .73205
4
(a 2 )
22 x 70 x 1 0
= 2 x 1 06 x 10 + - => a 2 = 10000 x 4
7 => a = 100 x 2 = 200cm
= 2 1 20 + 2200 = 4320 cm 2 Since, AA.BC is an equilateral triangle .
9 In the given figure, AB and B :. LA = LB = LC = 60°
Here, AB = a = 200 cm
CD are two diameters of a
circle (with centre 0)
. le = 200 = 100 cm
. . Rad.ms of c1rc -

en dicular to eac h other D t-----+--� c


2
-
perp [·: radius of circle = half the length o
a nd OD is th e diameter of the side of the LiABC]
the s maller circle . If Area of sector of a circle
A e_ x nr2 = 600 x 3. 14 x (1 00)2 = 5233.33 cm ·
=_
=
OA 7 cm , t h en fi nd the
area of th e sha d ed regi on. CCE 2013, 12, 1 1 360° 360°
Areas Related to Circles 3 05
Th n ar a three equal sectors
f .. Area of shaded region
=3 x
52 33 . 33 = 1 5700 cm 2 = Area ofq uad ranr OA CB - Area ofl:l OBD
: . Area of r quired shaded region = 77 _
35 =
28
=
49 77 -
cm 2
= Arca of MBC - Arca of three sectors 8 8 8
= 1 7320.S - 1 5700 = 1620.5 cm 2 1 3 In the given fig u re , a s q ua re OAB C i s i n s c ribed
=
11 On a s q u a re handkerchief, nine circular des igns in a quadrant OPB Q. If OA 20 c m , t h e n find
the area of the s h a d e d region . [ Take , n = 3 . 1 4)
ea ch of rad i u s 7 cm are made (see the figu re ) .
Fin d the rea of the remaining portion of the Q
handkerc h i e f.

A p
Sol. Given, OA BC is a square.
:. Diagonal of a square = J2 x Side = J2 xcm 20
[ -. · si de OA = 20 cm given]
Then , radius of c i rcl e r = Diagonal o f square = 2 0 J2 cm
,
Sol. o f each circle, r = 7 cm
i ve n , rad i u

:. D i a meter of circle, d = 14 cm [·: diameter = 2 x rad ius ]


rcr 2
:. Area of quadra nt OPBQO = -
In the given fi g ure, horizontal three circles touch each other. 4
:. Length o f side of square = 3 x Diameter of one circle
a 31 4 x c 2 0 J2)2 3.1 4 x 800 -

= = = 62 8 cm
= 3 x 14 = 42 cm 4 4
Now, area o f one circle = rcr 2 = re (7) 2 Now, area of square = (Side) 2 = (20) 2 = 400 cm 2
=-22 x ( 7) 2 = 1 54cm 2 Hence, area of shaded region
7 =Area of quadrant -Area of square
:.Arca of nine circles = 9 x 1 54 = 1 386 cm2 = 628 - 400 = 228 cm 2
Aiea of square ABCD = ( Side) 2 = ( 42) 2 = 1764 cm 2
1 4 AB
7
Hence, area of the remaining portion of the handkerchief and CD a re respectively arcs of two
= Area of square Area of n i ne ci rcles
-
concentric circles of ra dii 2 1 c m and cm and
= 1 764 - 1386 = 378 cm2 centre 0 (see t h e fig u re) . I f LAOB = 3 0 °, then
fin d the a re a o f t h e s h aded region.
1 2 In the given figure, OA CB is A
a quadrant of a circle with
centre O and radius 3 . 5 c m . If
OD = 2 cm, then find the a rea
0
of th e
(i) quadrant OACB.
(ii) shaded region.
Sol. Given, radius of quadrant, r =3.5 cm 0
(z) , Area of quadrant OA CB = _!_ x Area of the circle Sol. Given, L COD = 30° 'I = OC = 7 m and

= 4 = 7x 4 (3. 5) 2 = ? x 4 ( Z2 )
4
r2 = OB = 2 1 m
rcr 2 � � 2
8
:. Area of sector COD = --
360 0
x n: r.1 -

22 x 7 = -
= -- 77 cm 2
2x8 8 =
30°
x
22
( 7)- = !!:__!__ == !!. 1

1 1
360 ° 7 1
oo
(iz) Now, area of D. OBD = 2 x OB x OD = - ( 2 1)-
-
x 3.5 x 2
2 Area of sector A OB = -- x n: r - 2
=: -0
7
60 0
[·: radius = OB =3.5 cm and OD = 2 cm, given ] 22
=3.5 cm 2

12
3 06 � AllinOne MATHEMATI CS C lass 1 0th Te rm t

Hence, area of shaded region Ttr! 2 2 ( ) �


= Area of sector AOB -Area of sector COD =-4
=- x -
7
=- [ ·: r = A B =
=- 693 -- 77 = -308 cm 35 2
6 6 3 7 c m cm

B e rn
1 5 I n the given figure, ABC is a A -------... 8
B
quadrant of a circle of
radius 14 cm and a
E
semi-circle is drawn with (.)
co
BC as diameter. Find the
area of the shaded region.
A
CCE 201 2, 1 1
Sol. Given, radius of quadrant,
r = 14 cm = AC = AB Now, area of right angled AB
I
= - x AB
2
x BC
In right angled 6BAC,
BC 2 = AB 2 + AC2 [by Pythagoras theorem] = -21 x 8 x 8 = 2 cm 2 t·: BC = 8 given)
=> BC 2 = (14)2 + (14)2 B �--- . . Arca of shaded region A E
=> BC = 14 ./2 cm = Arca ofscccor BAE - A r a f right angled MBC
Now, area of right angled !1BAC =-352 - 32 = 352 - 224 = - I 28

= -21 x AC x AB 7 7 7 cm
Hence, required shaded region
= 2 x Arca of s haded region AEC
= .!..2 x 14 x 14 = 98 c 2 A �----
m
1 28 = 256 c m � ?

=2x - --

and area of quadrant ABDC = - 7tr2 = --


22
4 7 x 4 (14)2
7 7
Additional Questions
=154 cm2

(14J2J 2
1 7 In the given
Now, area of semi-circle BCE = �2 figure, boundary

= 22 x7 14
2 of the shaded
_E_
= region consist of
7x 2 2 four semi circular
x 14 x 2 arc, the smallest
x 2 x 4 = 22 x 7 = 154 cm
2
two being equal.
If the diameter of 14 cm
Hence, required area of shaded region the largest is 14 cm and o f the i smallest
=Area of semi-circle BCE -(Area ofquadrant ABDC 3 . 5 cm, then find
-Area of right angled !1BAC) .
= 1 54 - (154 - 98) = 98 cm 2 (i) the length of the boundary.
(ii) the area of the shaded region . CBSE 20 I O
1 6 Calculate the area of the designed region in Sol. From the figure, we have
figure , common between the two quadrants of 14 = 7 cm
circles of radius 8 cm each. AB = 14 cm, AD = - 2

BC = AE = - = -4 cm, GF = ? cm a n d DF = -27 = 35
a cm 7
3.S
n
2
(t) Length of the boundary = Ci rc u mfe rence of
E
(.) E
() semi-circle with radius AD + 2 x Circum ferenc f
co co semi-circle with radius AE + Ci rcumference of
semi-circle with radius DF
. � = 7t X 7 + 2 X 7tX -47 + 7tX -72
- B ern
I!

- S ol. Firs d y, determin e the area of the shaded regio n AEC . = l47t = 14 x -22 44= cm
"'

Now, area of sector BAEC 7


307 1
AB �( �B )
Areas Related to Ci rcles

( E)
(iz) Ar a f had ·d r gi n = A rca f sem i -ci rcle with rad i us
A D + A r a f c m i -ci rcl \ i rh rad i u DF '

HH HH
Now, area of sem i-circle with diamete r =

(� ) � ( ) AC�
- 2 x Ar a f m i -ci rcl with rad ius AE
2
44 n
2: x 2: x E x E i


2
= (?t + -2x = = = cm
2 2 2 2 2 8

(
(J BC
Area of semi-ci rcle with diam eter
= � x ( 7 ) 2 1 + _!_ _.!.J = !: x ( 7 ) (2- _.!.J '

= �( BC )2 ( )
2 4 8 AC 28 '
2 4 = = x = x 1 4 x 1 4 = 9 8 tt cm 2
2 2
= � x ( 7 )2 - = � x 7 x 7 x 2.
7x 2 8 Area of sem i-ci rcle with diameter
2

D.ABC
2
1 I x 2
35 35 35 1 225
=�x = !: x x = m2
-- - = 86.62 5 cm 7t c

AB
= =
2 2 2 2 2 2 2 8

18 I n t h e g iv e n fig u r , A B C Area of = ..!.. x 2 1 x 2 8 = 294 cm 2


i s a righ t a n g l d t ri a n g l e
2

[: : J
as
in which LA = 9 0 °,
::Area of shaded region = Area of semi-c i rcle with

AB = 2 1 c m a n d A C = 2 8
diamerer + Area of semi-ci rcle wi th A C as d i am eter
+ Area of MBC - Area of semi-ci rcle wi th BC as d i amete r

[( : � ) J
cm. S e m i - c i rc l s a re 4 1" 12 5
7t
drawn on A B, B C a n d AC a c = + 9 8 tt + 294 -

[( : ) ]
as d i a m et e rs . Find th a rea of the s haded re gion .
4 1" 12 5"
F rom the flgu r have = + 98 tt - + 294
Sol. w
LA = 90 °, A B = 2 1 cm, A C = 28 cm
44 1 tt + 78 tt - 1 22 5 tt
In righ t a ngled M BC , = + 294
BC = A B - + A C 2 = ( 2 1 )2 + ( 28 ) 2
= (O t 294) = 294 cm
2
==:> BC = .J1 22 5 = 35 cm

TO P I C EXE RCISE
Very Short Answer Type Questions Short Answer Type I Qu estion
1 Find the a rea �f the largest triangle that can be 4 A square park has each side of 100 m. A t each
inscribed in a sem i -circle of radius r un its.
NCERT Exemplar
corner of the park, there i s a f lower bed in t h e form
of a quadrant of radius 14 m as show n i n t h e figure
2 It is proposed to bu ild a single cir·c ular park equal i n below. Find the a rea of t he rem a i n i ng part of the
area to t h e su m of areas o f two circular parks of park .
diameters 16 m a nd 12 m i n a locality. Find the
radius of t h e new park. NCERT Exemplar

3 Find the dia meter of a circle whose circumference is


e qual to t h e su m of the circumference of the two
circles of diameters 36 cm and 20 .cm.
NCERT Exemplar
; 3os j Alllnone MATHEMATICS C lass 1 0th Te rm I

Short Answer Type I I Questions 8 Th e re a re t h ree sem i - c i rcles. A. B a nd C having


d i a me t e r 3 cm eac h a n d a no t h e r sem i - ircle E
5 7
In t h e given figure. ABCD is a having a c i rc l e D w i t h d i a meter 4.5 cm as sho n ·

square of side cm and A. B. C the figure below. Find t he a re a of t he sh aded reg·o .

and D are centres of equal circles CE OL


which touch externally in pa irs.
Find the area of the shaded
region.

6 In the given figure. diameter AB is 12 cm long. AB is


trisected at points P and Q. Find the area of the
shaded region.

9 In the given figure. 0 is t he cen t re a nd AOC is a


dia meter of the c i rcle. F i n d
p

Long Answer Type Questions Q


7 O n a circular table cover of A
radius 42 cm, a design is formed
by a girl leaving an equ ilateral
6ABC in the m iddle as shown i n (i) the sum of the a rea s of the two shaded
figure. It was decided that the segments made by the c hords AB and BC.
(ii) the area of the shaded seg ment made by the

[
�J
payment to the girl be made
chord PQ. take, " = [
proportional to the covered area 2

�J
of the design. Find the covered area of the design.
.J3 = 1. 73 and " =
2 CBSE 20 1 0
take .

- - - - - - - - - - - - - - - - - - - - - - - - - - - - - - - 4 - -
- - - - - - - - - -
-
- - - - - - - - - - - - - - - - - - - · -
-

Answers
1 . r2 uni ts 2. 10 cm 3. 56 cm 4. 9384 m 2 5. 1 0. 5 cm 2
cm2 (ii)
63 . 16
32
7 . 3255.2 1 cm2 8. -
7t Cm2
6. l 21t c m 2 16
9. (i) - cm -

7 7
�I Very Short A nswer Type Questions [ 1 Mark each]

1. What will b e the length of an arc of a sector of 1 4. The sum of circumference and the radius
a circle with radiu s r and angle 9? of a circle is 5 1 cm. Find the radius of
circle.
2. What will b e the area of a sector of a circle
with radius r and angle 9? NCERT Exemplar 15. The outer and inner diameters of a circular
ring are 34 cm and 32 cm respectively,
3. If the ratio of the circumferences of two then find the area of the ring.
circles is 3 : 1 , then find the ratio of their
areas. 1 6. A garden roller has a circumference of 4 m .
Find the number of revolutions, it makes in
4. The ratio of the areas of two circles is 4 : 9 . moving 40 m.
Then, find t h e ratio of their radii.
1 7. The circumference of a circle is 44 cm.
5. The length of the minute hand of a clock is Find the area of circle.
14 cm. Find the area swept out by the minute
hand in 1 h . 18. If the perimeter of a semi-circular
protractor is 36 cm, then find its diameter.
6. A pendulum swings tbrough an angle of 60° CBSE 2008
and describes an arc 8.8 cm in length. Find
the length of pendulum. 19. The circumference of two circles are in the
ratio 2 : 3. Find the ratio of their areas.
7. For a race of 1 5 40 m, find the number of
rounds one have to take on a circular track of 20. If the area of a sector of a circle is· �th of
18
radius 3 .5 m .
the area of that circle, then find the central
angle of the sector. CCE 201 5
8. Find the area o f sector of a circle of radius
28' cm and central angle 45 °.
NCERT Exemplar 21. Find the area of a square inscribed i n a
circle of radius 8 cm. NCERT Exemplar 1--
9. Z
u.J
What will be the perimeter of a quadrant of a
The radii of two concentric circles are c m
7
circle of radius r? 22.
and 5 cm. Find the area of the portion 5
10. If the area and circumference of a circle are between two circles. ti)
numerically equal, then find the diameter of
the circle. 23. A race track is in the form of a ring whose LU
inner circumference is 352 m and outer
1 1. Find the area of sector of central angle x 0 of a circumference is 396 m. Find the width of
circle with radius 4r. CBSE 201 1 the track.

12 . If the area of a circle is equal to sum of the 24. Find the radius of a circle whose
areas of two circles of diameters 1 O cm and circumference is equal to the sum of th e
24 cm, then find the diameter of larger circle. circumference of two circles of radii 15 cm
and 18 cm. NCERT Bxemplar
13.
·
Find the diameter of a circle whose area is
equal to the sum of the areas of two circles of 25. The circumference of a circle is 1 1 cm .
radii 40 cm and 9 cm. Find the area of its quad rant .
310 ': All/110ne MATH EMATICS C lass 1 0th Te r m I

� I S hort A nswer Type I Questi ons ( 2 Ma rks ea c

n = �2 ]
26. An 3 4 . The length of the minute han d of the doc ·

[
athlete runs on a circular track of radius
4 9 m and covers a distance of 3080 m along is 1 4 cm. Find the area swept by the minu e
its boundary. How many rounds has he hand from 9 : 0 0 am to 9 : 35 a m .
3 5 . The radii of two circles are 4 m and 3 c m .

21
Find the radius o f the circl whose area i
taken to cover this distance? Take,
CCE 20 15 equal to the sum of the ar a s of the h
circles. Also, find the circum f r nee of thi
2 7. The diameter of a cycle wheel is cm. How circle.
many revolutions will it make to travel
3 6. Difference between the circ u m ference an
7
1 . 9 8 km?
the radius of a circle is 4 cm. Find the al"'
28. The difference between circumference and of circle.

21
diameter of a circle is 1 3 5 m. Find the radius
37. The area enclosed between the concentri

I
77
of the circle.
A
circles is 0 cm2 . I f the radius of the out
29. Find the perimeter of

1<ij
circle is cm, then find the radius of th
the given figure, where inner circle.

[ n=-]
E
AED is a semi-circle and 5

�1
g
ABCD is ;
rectangle.
3 8 The short and long hands of a clock are 4 cm
.

and 6 cm long respectively. F i n d the sum of

o[Jc .5 5
2
7
Take, distances travelled by their ti p s i n 2 days .
B C
CCE 20 1 5
-1 4 cm - 39 . Find the area of the sector o f a circle f
radius cm, if the corresponding arc length
30. In the given figure, a is 3 cm N CE RT Exemplar
s quare is inscribed in a H G
circle of diameter d and 4 0. Find the area of the largest circle that

another square is can be drawn inside the given rectangle of


length ' a cm and bread th b cm (a > b).
' ' '

Exempler; CCE 20 1 5
circumscribing the circle.
E F NCERT
Is the area of the outer A B
square four times the area 4 1. If the length of an arc of a circle o f radius r i

0
of the inner square? Give reason for your equal to that of an arc of a circle of radius 2r,
answer. NCERT Exemplar
then the angle of the corresponding sector
of the first circle is double the angle of th
3 1. In the figure, a square of corresponding sector of the o th er circle . I
diagonal 8 cm is inscribed in a this statement false? Why? NC E RT Exemplar
circle. Find the area of the
shaded region. 42. In the given figure, arcs


NCERT Exemplar are drawn by taking
32 . Find the area of the sector of a circle with vertices A, B and C of an
equilateral triangle of side
radius 1 0 cm and of central angle 60°. Also, 1 0 cm, to intersect the B
find the area of the corresponding major 0

25
sides B C, CA and AB at
E a n d F. Fin d
sector. NCERT Exemplar
their respective mid-points D,
33 A
the area of the shaded region.
.. h orse is tied to a peg at one corner of a NCERT Exemplar
squ a re sha ped grass field of side m by
mea ns o f a 1 4 m long rope. Find th e area of 4 3 . The circumference of a circle exceeds th
the p art o f the field in which the horse can diameter by 1 6 . 8 cm. Find the radius of th
g ra z e [Take, rr = 2 2 /7 ]
. circle.
Areas Related to Ci rcles 31 1

I Short A ns wer Type I I Quest i ons [ 3 Marks each ]

rsz:r
44. Fin d th a r f th s h a d d 50. PQRS is a rectangle i n
reg i o n in t h g i v e n fig u re , if whose length i s two
PR = 12 m , PQ = 5 m a n d O times the breadth a n d L
is th ntr f th circl . is the mid-point o f PO .
p Q
rt
22 With P and Q as centres, L
7 CBSE 2009, 08
[ Ta k , = -]
two quad rants, a re d rawn as shown i n
figure . Find t h e ratio o f t h e a rea o f
45. Cal u l a t recta ngle PQRS t o the shaded p o rtion .
of
5 1 . A flower bed i s laid i n a
th
o ,.-----.-----. c
reg i o n s h ow n i n t h
square park of side 1 0 m
fig u r m m o n 6 cm 6 cm as shown (shaded ) i n
betw e n tw
quad ra n t s of ci r 1
t h e figure . Fin d the a rea

of rad i u s 6 cm a c h .
o f flower bed, i f the
6 cm portion left out a re the
A � 1 0 m ___.. 8
quadra nts o f a circle o f
46. I n t h e g iv e n fig u r ,
same radius . The diameter o f t h e circle i s
fin d the a rea o f shaded
regi o n . [ Ta k , rt = 3 .1 4]
equal t o t h e side o f s quare . CCE 20 1 5

c�B
52. Area o f a sector o f central a n gl e 2 0 0° o f a
circle is 7 7 0 cm 2 . Fin d the l e n gt h o f the
corresponding a rc of this sector.
47. I n the g iv n fi g u re , OABC N CERT Exemplar
is a s q u a re o f side 7 cm. I f
53. I n the given figure ,

[ ]
OAPC i s a q u a d rant o f a
AC = 24 c m , B C = 10 c m a nd
circle with c e n tre 0, then
A
0 is the centre o f the circl e .
fin d the a re a o f the shaded o
Find t h e a rea o f the s had d
n=
region. [Take , rt = 3 . 1 4]
22
regio n . Take ,
7 C BSE 20 1 0

5 4. I f the give n figure, a


48. S i d e s o f a tria n g ular field are 1 5 m, 1 6 m
OACBO repr sent a
quadrant of a cir l
a n d 1 7 m . With the three corners of the field f
a cow, a b u ffalo and a horse are tied radius 7 cm wit h c ntre
separa tely with ropes of length 7 m each to 0. I f OD = 5 cm , then
graze in the field . Find the area of the field find th ar a f th
w hi c h c a n n o t be grazed by the three shaded regio n .
animals . NCERT Exemplar; CCE 20 1 5
A
55. Fin d t h e a rea o f th
49. Fin d t h e a rea o f the shaded region given in
· NCERT Exemplar
haded region i n the g i n
figure .
fig ure, i f B C = BD = cm,
� 1 4 Cm -+ AC = AD = 1 5 c m a n d 0 i
the centre of the circl
[Tak , rt = 3 . 1 4]
B
56. A circular p n d i 1 7 .5 m i
of diameter. I t i d b 2 m wid
path . Find t h e t f n t ru t ing th ath
at th rat o f � 25 p r m2 · NCERT Exemplar
Alli'110ne MATHEMATI CS C lass 1 0th Term I I

5 7. In the figure given 61. In the given figure, find the area of the shaded
alongside, a circle is portion with given measure ment s .
inscribed in a square of
sid e 4 cm and another
circle is circumscribing
the square. Prove that the
area of the circumscribed circle is two - 1 4 cm -
times the area of the inscribed circle.
NCERT Exemplar 62. An arc of a circle is of length 5 rc cm and sector it
bounds has an area of 20rc cm . Find the rad i u s
58. I n the given figure, of the circle.
ABC is a quadrant of a
In a circle with centre 0 a nd radius 5 cm, AB is a
circle of radius 1 0 cm
and a semi-circle is
63.

-
chord of length 5.J3 c m . Find the a rea of sector

: :
drawn . with BC as
AOB.
diameter. Find the

..-... -
area of shaded region. CBSE 2008, 08 C
64. The length of the minute hand of a clock is
59 . The given figure
5 cm. Find the area swept by the minute hand
during the time period 6 05 am a n d 6 40 am.
consists of four A a
���Hffi� D __.,.E
NCERT Exemplar

0
c
small semi-circles of
equal radii and
65. A tool is prepared out of a circular m etallic disc
of radius 14 cm , by taking out two s e mi - circular
two big semi-circles
of equal radii (each 42 cm) . Find the
parts of radius 7 cm each form

]
area and perimeter of the shaded region.

[
·
the two sides as shown in the
60. Find the area of the shaded portion from figure. Find the area of the

-
metal used in making 50 such

1
the given figure .
re 22
D C tools. Take, =
7
CCE 20 1 5

� cm F 66. The radius of the circumcircle o f a ri gh t angled


G triangle, is 3 cm and the altitude drawn to th

lA "-------L---'
hypotenuse is 2 cm long . Find the s um of th

.
B areas of the three disj oint segme n ts of th
E
-+-- 1 4 cm----+ circumcircle formed by the sides of this
triangle. CCE 2015

) long A ns wer Type Que stions [4 ar s each]


)
J s R
)
67. A 2 1 widem 68. Find the difference of the areas o f a sector of
athletic track angle 1 2 0 ° and its corresponding maj o r sector
D c
consists of two o f a c ircle o f radius 2 1 cm. NCERT Exemp lar
straig ht sectio ns A a
1 50 m long 69. The central angles of two sectors of c ircles of
j oini ng P Q radii 7 cm and 2 1 cm are respectively 120° and
semi-circular ends whose diameters are 40°. Find the area of the two sectors as well a
84 m each (see the figure) . Find the area

[ ]
the lengths of the corresponding arcs . What d
of th e tra ck. you observe? NCERT Exemplar

22 a nd J3 = 1 3
n =
If three circles of radius a each, are d rawn su h
7
70 .
7
take I

CCE 201 5
that each touches the other two, then prove th t
I
Areas Related to Ci rcles 3 13

72 .
area include d between them is equal to
In the g iven fig u re, 0 is the centre of the
_i_ a2 . [Take, .J3 = 1.73 and 7t = 3.14) circle with AC = 24 cm, AB = 7 cm and

L.B OD 9 0 °. Fin d
25 CCE 20 13 == th e area of the shaded
n =
71 . AB CDEF is a regular region. [Take, 3 .1 4)
h exagon . With vertices ,
A , B, C, D, E a nd F as
th centres, circles of
sam e radius r are
drawn .
Find the area of the
D
shaded portion shown in the given figure.

I Va l ue Based Questi ons (V8Qs) [4 Marks e a c h ]

73. For the inaugration o f ecoclub to the school,


badges were given to teachers. Sangeeta [ take,
it = 22
7
]
(ii) find the cost of ploughing the field.

made thes e badges in the shape of an


equilateral A (iii) Which value is depicted by the farmer in
triangle of side 5 fencing the field?
cm with a circle of 76. A poor artist on the street
5
radius -- cm makes funny cartoons for
2 .J3 children and earns his
inscribed in it as living. Once he made a
shown in the comic face by drawing a
figure . B circle within a circle, the
[Take, 7t = 3 .1 4)
D
radius of the bigger circle being 30 cm and
(i) Find the area of the shaded portion. that of smaller being 15 cm as shown in the
(ii) Which value is depicted by Sangeeta? figure. What is the area of the cap given in
this figure, the base of the cap being the
7 4. Two farmers have diameter of the bigger circle? What
circular plots. The qualities of this artist are being reflected

A
plots are watered here?
with the same water

l.____L
source placed in the 77 . A farmer has two types


m
point common to both the plots as shown in
the figure. The sum of their areas is 130n and
of fields-in the form o f a v
triangle and a
u.
the distance between their centres is 14 m. 7m 7m
Find the radii of the circles. What value is rectangle. Rani is
depicted by the farmers? CCE 2015 allowed to cut grass of 7 m
triangular part (shaded)
75. A fa rmer has a field in the form of circle. He
and Ramu of 7 m
wants to fencing the field. The field is to be
ploughed at the rate of � 0 . 75 per rectangular field
m 2 • If the cost of fencing of a circular field at (shaded) . Calculate the areas of th h d e d
the rate of f 25 per m is 5500, then portion. What value is d pi t d by the
(i) find the length of fencing the circular farmer? CCE 201 5
field.
So l u tions

angle 1s -- x 2 7tr i. e . -- x 1tr.


1. Length ofan arc ofa sector of a circle with radius rand
e e
8. Given,central
radiusanglof ea, circle,= 45°
cm
r = 28
e . and e

--
[1 ]
1 80°
·
360°
Ic
. ca of scccor o f a circ = -- nr20
2. Area of a sector of a circl e wi t h radius r and angle is e . · Ar
3600
e
x 1tr i . 50
4
22 (28)2
360° [1 ] = x x = x 28 x 2822

3• Circumference of one circle ( 3 C1 )


7 360° 7 8
Circumference of ocher circle ( C2 ) = 11 x 28 cm
= 308 2 [1]

=>
21t7i = � =>
7i
= �1 [1 /2]
9. Perimeter of a quadrant = r + r + _!._ x 2nr
4
27tr 1 r

(-- �2 2 = -92 = 9 :ri1


Now, ratio of their areas = 1tr: = r: = (2_)
2
1t
2
2 = + -1 nr = -2r ( 7t + 4)
2r

1)
'i
7i 2 [1]
1 0. Lee r be chc radius of circle.
Accordingofcoa checirclquesti o n,
e = Circumference of a circle
1
[1 /2] Area ( 1/2)

4. Do same as Q. 3 .
=> 1tr2 = 2nr => =2
[Ans. 2 : 3] r

:::::::: ��=
Diameter=eter2r =of 2thex ci2rcl=c isuni4 uts4
5. Length of the minute hand
= Radius of a circle (r) = 14 cm F 2i?j
- = 1tr 2 = -7 x 1 4 x 1 4 :.LeedAreacmofbea ciche)r2cldiameter
Area swept in 1 h = Area o f sector = -- 7tr 29 ll. •·

(d
[1 /2]

., .,
3600
1tr2
---- x
360°
360° 22 1 2.
e =Arca ofof required
rwo smallcircl
er circle. es
= 22 x 28 = 6 1 6 c� 2 => 7t 2 = 7t (S)- + n (1 2) -
Hence, the area2• swept out by che minute hand in
[·: radius of firsr circle 1� = 5 cm and l
[1/2]
1 h is 6 1 6 cm [1 /2]

(2d )2 25 = 1 69 (-d2 )2 = (13)2


6. Given,central
and lengthangle,
of an arc,= 60°= 8.8 cm
e
l
=

Lee r be the length of the pendulum. radius of second circle 242 = l 2cm =

. ·.
Length of an arc, = 1 80° l m-0
=> =
(1 /2]
+ 1 44 =>
r = 1 80° x l
e
=>
n => :!_ = 13 => d = 13 x 2 = 26 cm
1 80° x 8.8 x 7 = 2
=> r= 8.4 cm Hence, che diameter of the larger circle is [26Ancm. [1/2]
22 x 60°
Hence, the length of the pendulum is 8.4 cm. 1 3. Do same as 1 2. Q. s. 82 cm]
[1 /2]
·: Sum of circumference of a5 circle and radius= 51 cm
( )
7 · x Circumferen ce of circular crack= Length of race
-n
1 4.
=> n x (of27tr)rounds= 1 540taken => 2 7tr + r = 1 [1 /2]
where, = number
n on a circular [1 /2] => r( 2 7t + 1 ) = 5 I

track. 22 =:> r 2 x 22 + 1 = 5 1 =:> r x 5 1 = 51


7
n x 2 x 7 x 3.5 = 1 540 [·: r = 3.5 cm, given]
=>
=>
7
r=7 cm
n = ---- 1 540 - Hence, che radius of rhe circle is 7 cm. (1 /2)

= ----- = 70 Lee = Outer radius = 342 = 1 7 cm


22
2 x - x 3.5 1 S. Given,innerouterdiadimeter
ameterof aofcircular
a circularring=ring=32 cm34 cm
7 and
1j
1 540 x 7
22 x 2 x 3.5
Hence, the num ber of rounds is 70. [1 /2] and r2 = Inner radius= 322 = 1 6 cm [1/2)
Areas Related to Ci rcles f 31 5 f
:. Area= ofrc(chc -circular ring Do same as Q. 15.
1/ rf) = rc(l72 - 16 2 ) 22.

= rc(289 - 256 ) = 33rc cm 2 2 23. Lee R andy.r be che radii of oucer and inner circles,
Hence, chc area of che circular ring is 33rc cm • (1 /2) respectivel
Then, inner circumference, 2 nr = 352
1 6. Here,
Distancecoca)covered distancein covered
one = 40onm
revoluci 352
21t
r=-
= Circumference40of a circle = 4 m ( 1 /2]
and oucer circumference, 2 nR = 396
:. Number of rcvolucions = 4 = 10 [1 /2]
=> R = 39627t [1 /2]

2 -7 r = 44 r = ---
1 7. Given, circumference= 44of a circle = 44 cm Widch of che crack = R - = 3962rc - 3522rc
.. 22 2rcr 44 7
x
r

=>
x x
22 2
x = 2rc44 = 244 227 = 7 m
x

=> r = 7 cm 22 [1 /2]
Hence, the widch of the track is 7 m.
x

[1 /2]
Now, area of a circle = rcr 2 = 7 (7)2 x
24. Lee che requi r ed radi u s be r.
= 227 7 7 = 154 cm 2
x ..
Then, 2nr == 152rc(l5)
r
+ 2rc(18)
+ 18 = 33 [1]

2 rcr = 1 1 2 -227 r = 11
x

Hence, che area of a circle is 154 cm 2 .' [1 /2]


25. Given, circumference of a circle = 11 cm
Perimeter=Perimeter
of a semi-circular procraccor ·: Circumference of a circle = 2nr

{ 2 + � ) = 36 => r = 36
1 8.
=>

2r + rcr = -36circle = (2r +36nr)[givcmen]


of a semi => x x

r = 221 1 72 = -47 cm
x
·
[1 /2]
7
x

= ..!._ 227 ( Z.4 )2 = 3277 cm


x

=>
[1 /2]
1
:.Hence,Diameter= 2r =
x 2 r7 == 147 cmcm .. Area of the quadrant= ., -nr 4 2
14 cm. che diameter of a semi-circular procraccor is (1 /2] 4
x x 2

1 9. Do same as Q. 3. [Ans. 4 : 9] Hence, area of che quadrant is 3277 cm 2 • ( 1 /2]


20. Leer be radius of circle. e
:. Area of a sector of circle = _ 360°_ rcr2 Given, radius of a circular track, r = 49 m
= 2nr = 2 -7 49
x
26.

and area of circle = nr2 Distance travel led in one round


= Circumference of a circular crack !­

le is given char, 22 z

e 7tr 2 = -5 nr 2
x
LLl
x

Area ofsector ofcircle = 2_18 Area ofcircle


x [1/2]
= 308 m
--
Bue coral di s tance covered by athl e te = 3080 m [given] �
(1]
l;0
::::>
360°x
185
x
. . NumberDiofstance roundscovered by athelece 3080 (/)
e 360°
::::> --
360° = -18 9 = 18
=>
s x
= Circumference of circular track = - 308 = 10 LW (j .

·Lee: Dixabegonalthe=ofsiDidsquare
::::> e = 100° [1/2] Hence; number of rounds taken by athlete is 1 O. (1 ] U1
4
21 . Here, diagonal= 2 8 = 16 cm 27. Do same as Q. 26. [Ans. 3000]
and diamerer , �
x

Leer be the radius.


= x
e of square. 28.

of.Gi. vaen,circldieffiserence
135m.2nrbetween circumference
==> x .J2 = 16 => x = 8Ji 2 ( � l) = �i �
t' f-
ameter ofcircle - = 135 �
:. Diagonal .fi 2r

=> 2r - 1 35 [1 ]
. r�
:. Area of square = ( Side)2 = x
a

= (B.Ji.)2 = 128 cm2 (1 ]


x 2r .!,2 135
7
==

I�
316 Alllnone MATH EMATICS C l ass 1 0th Te rm I

r
x

= 135 15 Given, radius of circle, = I cm 0

=_
7 32. 0
=> r x
2 and central
Area of minorangle, sector = 8 60°
63 31.5 m
--
=> r = -= e_ x
OAPBO

Hence, the radius of circl2 e is 31.5 m.


:. Rad1us. of a sem1.-c1. rc e, = --=
nr .?
[1 ] 360°
60°
29. In the given figure, BC = AD = 14 cm = 360° x
-
22 x
IO 10 x

p
I
AD -14 = cm
r 7 [1 ]
7
227tr = l lOO = 52.38 cm.!
= - 7 = cm
2 111
Now, circumrerence of a sem1-c1rc e = --= 1t r
. 21
c
2
. I
Area of rhe major scccor OA QBO
° 60
22 x

7
22
= 360° 7 x I )2 = =
300° x 22 5500
21
( 0
. =. Peri meterBCof a giCDven figure m (1] 26 1 .90

Do same as Q. 3 of NCERT fo der


AB
= 14
20 +
+
+
+
20 + 22
Circumference
+
= 76 cm of a semi-circle [1 ]
33. Do same as Q. 8 (1) of NCERT fo lder 8 . 2 .
m -J [Ans. 1 54
30. No 34. l 8.2.
Given, diameter of a circle is d. [Ans. cm - 35 9 .33
:. Diagonal of a =innerDiasquare
= 2 -7 5 = -- = cm
meter of a circle = d 35. Do same as Q. 1 2. [Ans. m) r = 5
Let side of a inner square EFGH be x. :. Circumference of a circle = 2 nr
In right angled !!:.EFG, x 22 220

o�c
x 3 1 .43
= EF 2 + FG 2 [by Pythagoras theorem]
r ( - I) = 74
EG 2 7

d2 = x 2 x2 + [1 ]
36. Let r be rhe radius of the- rcircl= 74e
.. 2 1tr
d2 = x 2 x 2 = d2 x

= (Side)2 = x 2 = -d
22
2 => => 2
2 7
:. Area of a inner square 2 H
r = 14cm [1 J
E
:. Areaofcircle = nr 2 = 14 x 14= cm 2 [1 ) 22

:. Area of outerDiameter
EFGH F x 616
2 A B 7
Bur side of =a outer square ABCD 37. Let rbecirclradie =us21ofcminner circle and i en radiu g v that of

square =ofda2circle = d outer


ItArea
is givofenbithat,
Hence,thetheareaareaofoftheouter ll r
times innersquare
square.is not equal to four [1 ] =:)
gger circle -Area - nr =
n( 2 1 ) 2
of
2 ma e circle = s [1)
770
770

31. Let thebe rsidecm.of a square be a cm and the radius of a


circle ::::> 7t (441 - r 2 ) = 770

Given, length of the diagonal = 8 cm =:) (441 - r 2 ) =


22
7
770
a .Ji. = 8 770 x 7
f :,
=>
441 - r2
a = -J2.8 x .Ji. .Ji.
= 4v cm �
2
[1 ] 441 - r 2 = 245 22

Now, =:) r2 = 196 = 14cm => r


Diagonal of rhe square = Diameter of the circle = 8 cm 38. Giandvenlelngth
engthoflong of shorehandhand= 6ofcmthe = 4 cm [1 ] clock

:. Radius of a circle = �2 = 4 cm We know that the short hand will 4 rou nd com p lete
Now, area of a circle = 7tr2 = 7t(4)2 = 167tcm2 inDis2tance
days.moved
= 2 - 4 = -cm
and area of a square = = ( 4..Ji.)2 = 32 cm 2 by its tipcircumference= 4
= .
a2
So , a rea of shaded=Arearegioofn circle -Area of square =Number of rounds 704 x x 27tr
4 x
x
22
x [1 /2 ]
7 7
cm 2 longmoved hand willby complete u n in
its2tip = 4848 Circumfer ds 2 da
:.andDithestance ro
( l 6 n - 32 )
Hence, th e req ui red area of the shaded region is enc x

(1 6 n - 32 ) cm 2 • [1 ] = 48 2 x 6 = cm [1 /2)
x 2
7
x 1 2672
7
Areas Related to Circles f 317
Hence, sum of disrancc 704 moved by rips
12672 42. Given, ll.ABC is an equilateral triangle. 10
= 7 + 7 = 1910.8 5cm [1 ] :. LA = LB = LC = 60° and radius, r = 2 cm
Given, radius of circle, (r) = 5 cm a
39. AFEA
Area of ..sector = 360° x 1tr --
i

and know
lengrhrhar, (/)
of an arc, = 3.5 cm
We 60° x 1t(5)2= -25 7t cm
= 360° 2

e
--

sectorson are equal. 6


= 3 x Area of sector = { � It)
Lengrh of an arc = 360° x 21tr --
Si:. Total
nce, areaareaofofallshaded
three regi (1]

e
3.5 = 360° x 2 7t x 5 = 3.5, r = 5]
-- [/
AFEA
0 x 360° 3.5 x 36 ... (i)
=> = 3.5 l07t 7t [1 1
= 3 x 256 x 3.14 = 39.25cm 2
e
Now, area ofsecror=-- 360° x 1tr 2 Hence, the area of shaded region is 39.25 cm 2 . [1 ]
35 x 36 x 7t x (5)2 [from Eq. (ii)]
= 360° 43. Lee the radius of a circle be r cm.
x 7t Then, diameter of a circle = 2r cm
= 3.5 x 2 5 = 17.25 = 8_75 cm2 and circumference of a circle = 21tr cm
Clrectangl
early, etheof lengrh
IO
largesr circlande breadth
. [1 ]

that canbbe drawnb) isinthesidcie rthecle byle is16.8givencm.chat the circumference exceeds the diameter [1 ]
40.
a (a >
:. Circumference of a=Di circlame eter of a circle + 16.8
with diameter b. => 21tr = 2r + 16.8
Then, its radius = �2 cm a
(1 ]
=>
22
2 x 7 x r = 2r + 16.8
=> 44r = l4r + 16.8 x7

. . Area of circle = 1tr2 = It ( � J = b:n


=> 44r - l4r = 117.6
=> 30r = 117.6 ::::) r = 117.30 6 --

[1 ]
r = 3.92 cm
Hence, the radius of a circle is 3.92 cm. (1 1
41 . No
Let 44. Do same as Example 6 at page 300. [Ans. 36.39 cm 21
centrestwo circl
0andes berespecri
C1 andvelCy2. of radius r and 2r with
O', 45. Do same as 16 at page 309.
Q. [Ans. 20.57 cm 21

46. Here, AB = 12 cm, BC = 5 cm


In right ·angled MBC,
AC = �AB2 + BC 2 [by Pythagoras ch.eorem1 [1 1
= �(12)2 + (5)2 = �144 + 25 = .Jl69 = 13 cm
Diameter, AC = 1313 cm
lItength
is givCDen that
of C2thei.e.arcABle=ngthCbAB of C1 is equal to...arc(i) =>
. r = -cm
Radius, (1 ]
Now, l e e.
t be the angl e subrended by arc AB and 0 2 2
be the angle subtended by arc CD at the centre. Area= ofAreashaded regi o n
of the circle -Area 13of the rectangle ABCD
.AB = � 360° x 27(/ = 7tr2 - 12 x 5 = 3.14 x 2 132 - 12 x 5 x
and CD = � 360° x 27t(2r)= � 360° x 47(f [1 ]
= 132.67 - 60 = 72.67 cm 2
From Eq. (i), AB = CD Hence, the area of shaded region is 72.67 cm 2 •
[1 ]
� x 2rrr = � x 47(f 01 = 2 0 2 Given, is a square of side cm
and = is =quadrant= r =a7
=> => r1 1 47. OAB C 7 cm .
360° 360° :::::> QA co ==7
i.thee. angl correspondingngsector
anglee ofofthethecorrespondi sectorofof • is double
C1
AB
OAPCO a
BC
of ci rcle wi th centre 0 . [1 ]

Hence, it is true statement. C


2 Now, radius of the c i rde, cm [1 /2]
318 � AlllnOne MATH EMATICS C lass 1 0th Term II

:.Area of shaded region So, the area of field which cannot be grazed by the
= Area of square OAB C - Area of quadrant OAPCO three animals = Arca of t ri a ngu la r field
[1 /2] - Arca of each seccorial fidd
= ( OA ) -., 4l 7tr-., = 7 7 - 4l - 22 7 x 7 = ( 2 4 J21 - 77 ) m 2
Hence, che req u i red a rea of rhe field which cannot be
- - x x - x x
7
77 21
= 49 - -2 = -2 = 1 0.5 cm - ., grazed by the rluee animals is ( 24 ·/21 - 77) m 2• [1 ]
Hence, the area of shaded region is 10.5 cm 2 . [1 )
49. There arc fo u r eq ually sem i - ci rc l es a nd }KIM formed
a square.
48. Given chat a triangular field with the three corners of FH = 14 - (3 + 3 ) = 8 cm
che field a cow, a buffalo and a horse are tied separately Lee che side of sq uare JKLNI be .\· cm.
with ropes. So, each animal grazed the field in each
corner of triangular field as a seccorial form. Then, FH = .:_2\' + + .:_2\'
x

Given, radius of each seccor(r) = 7 m [1 /2]


=> 8 = 2.\' => .\' = 4 [1 12)

//;17 md(
./�'in- - C:- ��)-16�m
So, rhe side of square should be 4 cm and radius of
semi-circle of both e n ds arc 2 c m each.
:. Arca of square JKLM = ( 4 ) 2 = 16 cm 2 [1 ]

77 m \1 m/ 7 7mm
2
Arca of semi-circle }HNI = 1tr = rr (l)2 = 2 7t cm 2 x

2 2
:. Area of four sem i -ci rcles = 4 2 rt = Brr cm 2

15 m
' , � x


l .,

't>-1
Now, area of square ABCD = (14 t = 196 cm · [ 1 12)
I I
:.Area of shaded region = Arca of square ABCD
Horse (H)
(B )Buffalo
- (Area of four semi-circles + Area of square ]KUvl)
Now, area of sector with L C = 196 - (8 7t + 16) = 1 96 - 16 - 8rt
= --LC .,
1tr- = -- rt (7) 2 m 2
LC
x x x
= (180- 87t) cm 2
360° 360° Hence, che required of rhe shaded region is
Area of the sector with LB [1 /2) (180 - 87t) cm 2 • [1 )

360° 1tr = -- 360° rt x (7) m 2


LB
= -- 2 LB
x 2 x 50. Lee the breadth of reccangle PQRS be x cm.
:. Length of rectangle PQRS 2x 2 =
and area of the sector with LH Area of rectangle PQRS = 2x x = 2x cm 2 [1/2)
= -- 2 LH rt
360° 1tr = --
L/-1 x
x x x ( 7) 2 m 2 Also, radius of each quadrant at P and Q, r = x cm
360° [·: L is mid-point :::::> PL = LQ = r =
Therefore, sum of che areas of three sectors .\·]
LC
( 7) 2 LB
2 m· 2
= 360° 7t x
360° rt ( 7)x + x x :. Area of two quadrants = 2 4 x --

+ --
LH
360° 7t ( 7)
2 x x = 2 4l 227 x - -171 x�., cm-.,
x - x - x
l
= [1 ]
_( _
L C + L_
__ ____
B + LH )
7t 49 x x Now, area of shaded region
360° [1 /2] =Area of rectangle PQRS - Area of two quad rants
x
1 80° 22 49 = 1 1 7 = 77 cm 2
x - x = 2x 2 - -I7I x 2 = -37 x 2 cm 2
360° 7
[·: sum of interior angles of triangle is 1 80°] Hence, required ratio = 2x 2 : �7 x 2 = 14 : 3 [H�]
G i ven , sides of triangle are a = 1 5, b = 16 and c = 17.
a + b + c
. I e, s = ----
N ow, scm1-. pcn. m e ce r o f cnang 5 1 . Given, side of a square, A B = 10 m
. . AB
= -I O = 5 cm
2
1 5 + 16 + I 7 48 :. Radms of a q uad ra nt (r) = -
=
2 =
2 = 24 2 2

Area of tria ngul a r fi eld Now, area of a quadranr, having cencre A


2
- a ) ( s - b) ( s - c )
== v �·(s [by Heron's form ula] =- rtr 2 = 3.1 4 (5 ) 2 = --
x 78.5 [1 1 m

9
-v24 J( x 7 = .J64
4 4 4
= -
8 x x 9 21
x
:.Area of four quadrants = 4 (Area of on� quadrant)
=8 x 3 J2 1 == 2 4 -vr:::
21 :-; m 2 [1/2] = 4 7845 = 78.5 2 [1 ]
x m
Areas Related to Ci rcles � 31 9
Now, area of a square A BCD = ( AB)2 x
17 17
= 3. 1 4 x - (60 + 60)
= (10) 2 = I 00 m 2 2 2
= 226.87 - 1 20
. . Arca of rhe sha<led region = Area of square = 1 06. 87 cm 2
- Area of four quadrancs
Hence, the area of shaded region is 1 06.87 cm 2 . (1]
= 1 00 - 78.5 = 2 1.S m 2 (1 )
56. Given chac, a circular pond i s surrounded b y a wide
52. Lee chc rad ius of rhe secror pa ch.
A OBCA be r cm. Also given, che diamecer of circular pond = 1 7.5 m
:. Radius of circular pond (r1 ) =
Given chac, cenrral angle of secror
A OBCA, 0 = 200 °
Diamerer
2
and area of che scccor A OBCA
= 770 cm 2
1 75
c I.e. QA = 'i = = 8.75 m
2
We know char,
Arca of chc scccor = -- x 1t1· -
8 .,
360 0
770 = -- x nr- => = ,. - [1 ]
200 ., 77 x 1 8 .,
360° 7t
,. 2 = 77 x 1 8 x 7 =:> ,. 2 = 9 x 49
22 and che widch of the pa ch = 2 m
=:> r = 3 x 7 = 2 1 cm AB = 2 m

[-.- fl
i.e.
So, radius of che seccor A OBCA = 2 1 cm (1 ] Now, lengch o f OB = QA + A B = 'i + AB
Now, lengch of che corresponding arc of chis seccor Lee r = 8.75 + 2 = 1 0.75 m [1 ]

[·: ]
2
= Cen cral angle x R.,dius . . Area of circular path = Area of outer circle i.e.
0
=

= nr -., - 1t1j-., [·: area o f circ


(Circular pond + Pach) - Area of circular pond
. l e = nr 2 ]
= 200° x 2 1 x � 1° = -2:._ radian 2
= 7t(r 2 - 1j 2 ) = n[ (I 0.75 ) 2 - (8.75 ) ]
1 80° 1 80° 2
2
20 22 220 1 = n[(I 0.75 + 8.75 ) (1 0.75 - 8.75)]
= - x 2 I x - = -- = 73 - cm
18 7 3 3 = 3J 4 x 1 9.5 x 2 = 1 22.46 m 2 [1 ]
Hence, che required lengch of che corresponding arc is Now, cost of construccing che parh per square metre
I
73 - cm. (1 ) = '25
3
: . Cost conscruccing the path ' 1 22.46 m 2
53 . Do same as Example 6 ac page 300. [Ans. 1 45.33 cm 2 ] = 1 22.46 x 25 = ' 306 1.50
54. Do same as Example 7 ac page 300. [Ans. 2 1 cm 2 ] Hence, required cost of constructing the pach is
55. Given, BC = BD = 8 cm and AC = AD = 1 5 cm ' 306 1 .50. [1 ]
AB is che d iamecer of che circle, LA CB = 90° 57. Let 1j = radius of the inscribed circle = - = 2 cm
4
[·: angle in semi-circle] 2
I n righc angled MB C , and r = radius of rhe circumscribed ci rcle
2
_ Now, in 6 0AB,
AB = �...-
A- C_2_+
_ B_ C2
( OA ) 2 = (OB)2 + ( BA ) 2
= 1 5 2 + 8 2 [by Pychagoras rheorem]•
� => rz2 = 'i 2 + 'i 2 [ · . · OD B'nA ]
=

= �225 + 64 .J289 = 17 cm
.J2;f
= [1]
=> r2 =
I e , r = - = - cm
.
. o f t he c1rc AB 17
22 = .J2 cm
. . Rad 1us
2 2 => r2 = �2 x 2

Now, area of M CB = .!._ x AC x BC Now, area of inscribed circle


2 = 7t1j2 = rr ( 2 ) 2 = 4 rr cm 2 . . . (i )
= _!_ x 1 5 x 8 = 60 cm 2 and area of circumscribed circle
2 = m22 = rr( 2 J2 ) 2 8 rr c rn 2
==

Similarly, area of tlADB = 60 cm 2 [1 )


. . Area of circ u mscribed circl e == 2( 4 7C)
:. Area of shaded region = Arca of che circle = 2 x (Area of inscribed c i rc le) l from Eq. (i) 1
- (Area of M CB + Area of tlADB) H ence proved .
= nr 2 - (60 + 60)
' 320\ AlllnOne MATHEMATICS C la ss 10th Te rm 11

58. Do same as Q. 15 at page 309. [Ans. 50 cm 2] Hence,cmthe2 •required area of the s a e porrion is h d d

59. It is given radius of bigger circles= 42 cm= R 96.25 (2)

08\\ncm
Let57t cmthesubtends
radius ofantheangle
circle beatrthecmcentre AB
:.
62. and arc oflen
Radius of smaller circles= R2 = 422 = 21 cm e of the circle.
. . Perimeter of shaded regiofon4 smaller semi-circle
= Circumference
Circumference (1 ]
+
of two biggerR)semi-circle A
+ = 0
= 4(7tr) 2(7tR) 27t(2r +
22 22
(42 42 )
x

Area ofsector OABO = 207t cm 2


=2 (2 21 42) = 2
x
7
x +
7
+
Given, arc AB = 5 7t cm
e
(1 1
= 528 cm (1 ) => 21tr = 5 7t . . . (i
Area= 2of[(Area
the shaded regionsemi-circle) Area of smaller
ofsemi-circl
bigger
-- x
360°

[·: area of sector = - 9- ]


+ [given
e -Area of smal l e r semi - ci rcl e ] e
1tr 2 = 207t
= 2 Area of bigger semi-circle
x
-- x
360°
(
= 2 .!_2 7tR2 = 227 42)2 = 5544cm 2
x x x [1 ]
360°
x nr2 • • • (ii) [ 1

60. Area of rectangle ABCD = 14 = 700 cm 2 )0 x

Area of two quadrants at [·: areaandofEBFrectangle =


AEG
On dividing Eq. (ii) by Eq. (i), we get
e .,
b] 7tr- 207t
= --
-- x
Ix 3600
--------
1 1 e
= 2 4 1tr2 = 2 7 7 7 = 77 cm 2 -
22
x - x x - x x -- x 21tr
360°
·: r = 7 cm, given]
Area of1 semi-circle1with22CD di[ ameter as
(1 ]
=> !....
=4
2
= cm
=> r 8

- x -
= 2 1tr 2 = 2 7 7 7 = 77 cm 2x - x x [1 ] Hence, the radius of the circle is 8 cm. [1 .]

Now, area of shaded 63. Given, AB = 5 .J3 cm and radius, 5 cm r =

-Area(Areaofregisemi-circl
ofotwon =Area of rectangleand EBF
quadrants
e with CD diameter) AEG
as
=> AL= BL = --5 .J3 cm
+
= 700 - ( 77 77) = 546 cm 2
+
[·: perpendicular 2drawn from centre to the cho .
Hence, the required area of the shaded portion is
546 cm 2 •
Q
bisects the cho
(1 )
61 . Radius of semi··circle 14with AC diameter, as

1i = - = ?cm (1/2)

and radius of semi-cir7cl2 e with AD diameter, as

r2 = 4 cm (1 /2]
Lee LAOBLAOL= 29, then [1]
0
In right angled = L BOL5 .J3=
6 0LA. ,

sm. 0 = -AL = z = - .J3 --

sin 60° 5 0A 2
Now=Area
, areaofofradius
shadedofportion sin == 60° e [1 ]
semi-circle with AC diameter as 0
4 Area of semi-circle with AD diameter
+ x as
LA OB = 2 60° = 1 20 °
= 1 1t1i2 4 7trf 2
x

=2
- x + x - Area of sector AOBA = 360°a 1tr -- x
2

Do same as Q. 3 at page 296. [ Ans. 45 � cm ]


2
1
- - 7 4 1 22 7 7 - --
25 7t cm 2 [1 J
2 7
x - x- x = 120° 7t 5 2 = -
2
x + x x
2 7 4 4
x x x -

7 360° 3
= 77 + 1 9. 2 5 9 .
= 6 2 5 cm 2 64.
Areas Related to Circles 1 321
65. t r be radius ofr semi-circle. :. Area of major sector ABOA = Area of ci rcle
= 7 cm - Area of m in o r sector AOBA
and R e rad i u sof =
c i rcular metall ic disc.
4 cm
= 1 386 - 462 = 924 cm 2
..
N w ar

of R 1
a
-
one sem i-ci rcle
22
So, rhe difference of the areas of m i nor sector
and i ts corres p onding major sector
= Area of sector ABOA
ABOA
AOBA
- Area of sector
AOBA
= - = -- x ( 7 ) 2 = 77 cm 2 [1 1
2 2 x 7 = 924 - 462 = 462 cm 2
i nce, he sem i-ci rcle has same rad ius. Hence, the req ui red difference of two sectors is
[1 1
:. A rea frwo sem i-circle = 2 x 77 = 154 cm 2 462 cm 2 •
69. Lee the l engths of the correspo n d i n g arcs be and
a n d a rea of c c l i r u ar disk = rtR 2 [1 /2]
11 12 •
= - x
22
7
(14) 2 616 cm 2 [1 /2]
=

: . A rea of m etofl circular


used fordisc-Area
a one cool ofcwo semi-circle.
= A rea
=616 - 1 54 = 462 cm 2
o, metal u n fed for 50 such rools 2 A [1 /2]
50 462 23100 cm
= x = [1 ] G iven , radius of seccor P01 QP, (1j ) 7 cm =

66. G iv n , radius of circle, r = 3 cm ad n radius of sector A02 BA, (r2 ) 2 1 :::: cm

l de of LlABC, CE = 2 cm
and a t i tu
Central angle of t h e sector P01 QP(91 ) = 1 20 °
[1 1 and central angle o f the sector A02 BA(9 2 ) 40° =

:. S u m o f chesegments
areas of ofche threeche :. Areanr 2 of t h e sector w i t h central angle 1 20°
disj o i n t 9 x ( 7 )2 x
X
-1-
7t 1 20 °
circumcircle
=Area offormed by theofsidMBC
circle -Area es =
360 0 I
= ------

360 °
[1 ] 7 x 7
( 1 121
22 1 54 22 x 7
x x 1 20 0 =
rtr 2 - _!_2 AB CE
= x x
=
7 360 ° 3
cm 2
3
=

2
and area of the seccor with central a ngle 40°

= =
rt(3) 2 - _!_2 6 2 = 22
= x 6 x x 9 -
9
x 40 o
= 7tr2 22 21 x 21
7 x = x
=
42 = 1 56 = 22.29 cm 2
1 98 - 360 ° 2 7 360 °
22 x 7 1 54 cm
7 7 [1 ] 22 x 3 x 2 1 = 2 (1 /2]
6 7. Do same as Q. 8 at page 307. [Ans. 10458 2] m 9
68. Giandvencentral
chat, angl
radiuse ofofthechesector = 21 cm = 120°
circleAOBA, e
Now, length of the arc of the sector P01QP 180nr0 = --
0
= 22 cm ( 1 121
120 ° = �
x
7 x 44 x 7 x
22
=
u_;
7
180° 3 3 7

<
nr0
and length the arc of the sector A02 BA 180°
of _.,-·

: ,: �
...,_., .
= 22 x 21180°40° � 21
7
x
=
9
x x
22
7
�; ,·
· ""''

22 63 = 1386 cm 2 [1 ]
1-
(1 1 2 22 44 1
= - x = - cm
:. Area che ci rcl rtr2 227 (21)2 227 441
of e= = x = x 3 3 2

= x two. wevenobserve
Hence,
ofequal gi circleths at 1 ngth of t h e ar
ar equal but c h ir
ccor

Now, area of minor sector AOBA with central angle


120° 70. FirEach
st, youanglneede oftoreg l fi nd the a ngl r gular h xag n.

7tr 2 360e 0 227 2 1 2 1 1 6200 0°


= x -- = - x x x --
:.
Sum ofall angl u a r h xago n

=
22 3 2 1 22 2 1 462 cm 2x x
= x =
=
Number of sid
120° [2]
3 [1 ]
322 Alli'none MATH EMATICS Class 1 0th Term 11

radms of a c1rc. 1
Now, area of shaded portion = 6 1 20°Area of one sector
x 73. (1) Given. side of an. equilateral triangle is 5 cm and

. Hence,
5
= 6 360 7tr2 = 27tr2
x -- x c 1s
2v3
r::; cm.

= J34 (5 ) 2 = 2s4.J3
the area of the sector is 2 7tr2• [1 ]
Now, area of equilateral ll.ABC = ..J34 (Side)2
71 Let the cen rres of the circles be A, B and C.
GiABC
·.·
ven, radius
i s an of eachateralcirtriclea=nglae
equil cm 2 [1]
ofSo,sidearea2a.of equilateral triangle
and area of circle = nr2 = " ( 2� J
an = _25 4 1.73 = 10.8 1cm 2
x

= .J34 (Side) 2 = .J34 (2a) 2


= 3.1 4 --
= .J34 4a2 = .J3a2
x [1 ] 25 = - 78.5 = 6.54 cm 2 [1 ]
Area of sector PARP =Area of sector PBQP x
4 3 12

3J 4 a 2 --
x

e :. Area of che shaded poreion


= Area of sector QCRQ = 7tr 2 3600 x --
= Area of MBC - Arca of circle
60° 1 = 10.8 1 - 6.54 = 4.27 cm 2
= x x
360° = 3.14 a 2 6x x - (1%]
(i1) Social responsibility.
[1 ]
(1 )
Now,=Areaareaof ofll.ABC
shaded- 3portiArea on of sector PARP 74. We knowbetweenthat if twotheircirclcentres
es couchis equal
externall y, thensumrhe
x
ditheistance to the of
= .J3a2 - 3 .!._6 3J4 a2
x x [1 /2]
Letand rtheradii.radiibeofthethecentres
cl , c
1j
two circlofesthebegivanden r2 respectively,
ci rcles.
= ..J3a2 - 1.57 a2 = a 2 (1.73 - 1.57) [·: ..J3 = 1 .73] 2
= O.I 6a 2 = - 16
a2 = - 4 a2
Hence proved. (1 ]
100 25
72. Given, AC = 24 cm, = 7 cm and LBOD = 900°
AB
BOC is a diameter of the circle.
LBAC = 90° (1 /2] (1 /2 )

In right angled·:ll.BAC
[ angle, in a semi-circle is a right angle] Then, 1j r2 = 14[·: distance between the cencres
+

two circles = 1 4 cm, given] ... (i) of


BC2 = AB2 AC2 +
[by Pythagoras theorem] Given,sqthe sum of the areas of two circles is equal co
1307t cm.
BC2 =(7)2 + ( 24)2 .. 7t7j2 7tr22 = l 307t
BC2 = 49 + 576 BC2 = 625 => +
=> 7t (1j2 rf ) = 1307t
=> BC = 25 cm (1 ] +
25
:. Radius (r) = 2 =12.5 cm 1j2 rf = 130
+ . . (ii) (1 /2]
(1j +
Now, r2 )2 = 1j2 r22 21jr2 + +
Now,=Area
area ofofshaded regi o n
circle -Area of quadrant COD => (14) 2 = 130 21jr2 [from Eqs. (i) and (i i ) ]
+
-Area of !1BAC (1/2] =>
=>
196 - 1 3066 == 21jr 2
= nr2 - 4r2 21 AB
n 21jr2
1jT2 = 662 = 33
- - - x
.
AC x
=> [1 /2]
-

��-.!:
I

=-34 nr 2 - -21 x AB x AC We know that,


:;.. ; J

- 7 )2 = 1 30 - 66
- ti
9m I .2 1
= 43 x 3.14 x ( 1 2 5 ) - 2 x 24 (7j - r2) 2 = r,_2 722 - 2r,_r2
+

:c 4
7 x
=> (r,. - r ) 2 = 1 30 - 2 x 33 [from Eq. (i)]
== 3 6 7.9 7

.
� ��i
- 84 => (r,_
2
= 283. 9 7 cm

2 2
=> (1j - r2 ) 2 = 64
�-, u Hence, the area of shaded region is 283.97 cm 2. . . .(ii )
1j - 72 = 8
[2]
1:-·� ,l
..
;
=> (1/2]
Areas Related to Circles :' 323
j

On adding Eqs. (i) and (iii) , we22get


21j = 22 =:::> 'i = -2 = I I cm
On putting r2
I I + = 14
the value of in =Eq.14(i), weI I =gee3 cm
=:::>
'i
r2 -

Hence,valuethedepicted
The radii of theby chccirclfarmers,
es arc 11arccmofandcooperati
3 cm. ve
[1 ] p

nacure and mucual understanding. [1 ] . . Area of cap =Area of llABC = _!_2 x AD x BC


75. (1) Given, coral cost of fencing = � 5500 2
and race of fencing per mecre Total = � 25cost I
= -2 x 30 x 60 = 900 cm [1]
. Arti
Length offcncmg = Race of fencing per metre
-------
respectst belandievesdignitin yselfoflacmpl
bour.oyemenc, which gives self
(1 ]
= 550025 =220m [1]
77. Area of triaLPngular shaded 22
= 360° x 7 x 72 + 360° x 7 x 7
region LQ 22 2
{it) Circumference of chc circular field LC 22
=> 21tr = 220
= Length of the22fence
2 x -7 x r = 220
=:::>
360° x -7 x 7 2
+ -- ( 1 /2]

r = 220
2 x 22
x7 => r
= 35 m [1 1

i.e. Radius of the circular field = 35 m


. . Area of the circular field = 22 7tr 2

[·: Arc of sector = 360° x 7tr2 ]


22 )2 LQ + LR) x 22 x 7 2
= 7 x (35 = 7 x 35 x 35
2
= (LP + 360° 7
= 22 x 5 x 35 = 3850 m _e_ ( 1 /2]
Now, cost of plxoughing at the rate of � 0.75 per m 2
= � 3850 0.75 = � 5
Hence, coral cost ofploughi2887. 180° x -22 x 7 x 7
[1]
ng the field is � 2887.5. = 360°--
7
(iii) Security and separate the boundary ofa field. [1 ] [· : LP +
2
LQ + LR = 180°]
76. Draw a radius
lineAP.of bigger circle= 30 cm
. Gandiven,radius = 11 x 7 = 77 cm
of smaller circl e = 20 cm . . Area of rectangula22r shaded region 22 [1 ]

ToBCfind _2Q_ x
= 360° x 7 2 + --2.Q._ x 7 x 72
and theAD.area of llABC, we need to find the value of [1 ]
7 360°
90 x -22 x 7 x 7 = 77 cm .,
Here, BCBC == 22 x(Radi u s
=>
of
30 DPBC = 60 cmbi gg er ci rcl e ) = 2 x 360° --
7 - (1 ]

=>
and also, AP60 == AD AD ++ 30 AD = 30 cm => [1 ] amount ofgrass (77 cm withoutandanyRamu
The farmer has al l o wed Rani
2) genderco cutbiasequal [1 ]
• •

Act1v1ty
Topic Covered Area of a sector of a circle. 8. With the help of fevistick, fix a l l the eight parts as
shown in figu re below.
Objective To verify that the area of a sector of a circle of
·

(-8- )
radius r which subtends a n angle e at the centre of the circle
is x 7tr
2 sq units. We will verify above statement for
3 60 °
e = 45 °.
Skil l Developed C reativity, having knowledge of area
of circle of radius r, i. e. Ttr2 and concept of a sector of circle.
Time Req uired 1 5 min
_

Material Req uired Sheet of glaze paper, geometry


box, a pair of scissors, black ball pen, paint brush and paint Observation We observe that the same circle is
box, fevistick. constructed. We measure the sector angle for each sector
Method and it is 45°.
1 . On the g l aze paper, d raw a circle of any radius r. :. Total area of eight sectors = Area of the circle = nr2
2. Mark the centre of circle as 0 and draw diameter AB.
- --
Then, area of one sector
45 0
x 1tf2 1tf 2
1
= = x
8 360°
Hence, the statement is verified for angle, 0 = 45°.

Oral Questions
1 . What i s meant by the a rc of a circle?
3 . Cut-out the circular disc out of the glaze sheet. 2. What is the formula to find length of a rc of a circle?
4. Fold the paper along AOB and cut the two 3 . What is meant by sector of a circle?
4. What is meant by angle of sector?
semi-circles a long AOB.
5. Paint one semi-circle with g rey colour and other with
5. Give the formula to find perimeter of a sector.
red colour as shown in fig u re below.
6. Write the formula to find the a rea of minor segment
of a circle of radius r and angle of sector 0, where a is
acute.
7. A figure A completely lies inside figure 8. How will we
find area of that part of fig u re B which is not covered
in figure A?

6 . Again, fold the two semi-circles and cut each in two True/ False
equal parts as shown in figure below. 1. Area of a circle is 1 54 cm 2 . Its diameter is 7 cm.
2. Ratio of radii of two circles A and B is 3 : 7. Then, the
ratio of their circumferences is 7 : 3 .
3 . OAB is a quadrant of a circle. Then, perimeter of OAB
is 25 cm.

7 . Now, e ach of these four quadrants are folded and then


cut equally into eig ht equal sectors.
l\reas Related to Ci rcles f325
4. A c i rcu l a r reg i o n is d ivided i nto equa l sectors by 4. In the g iven figure,
6 d i a m eters. The a ng l e of each sector is 60°. Area of shaded pa rt = ... x A rea

D�B
. of segment AB.
5. Area of a m aj o r seg m e n t is a l ways more than area of A
m i nor seg m e n t .

Fi l l i n the B l a n ks
1. A pizza of d i a m eter 60 c m was cut i nto 6 equa l pa rts
as shown in f i g u re . T h e a rea of each part is . . . . . . cm 2 •
[Ta ke, n = 3.14]
M atch t h e Colu m ns
Column I Col umn I I
1. If 'a' represents area of circle of 5:4
radius r arid c is the circumference,
(a)

r2 )
then relation is given by
2. If r is radius of a circle, where r = 20 (b) n(R 2 -

cm, then area of circle is given by


[Take, n = 3.1 4]
2 . Area of c i rc l e is 1 80 sq c m . The area of a sector wh ich 3. If ratio of areas of two circles is (c) 1 256 c m2
subte n d s an a n g l e of 77° at centre of circle, is .... cm 2 . 25 : 1 6, then the ratio of their
3 . Rad i u s of a m e rry- g o - g ro u n d is 7 m. An occupant i n
.
circumference is
4. If r is radius of inner circle and R is re

[ 2n
one of t h e baskets m oves . . . m i n ta king o n e circle. (d) = 2a
radius of outer circle. Area enclosed
Ta ke, it = between them is given by

nswers
O ra l Q uest i o n s
9
1 . A continuous piece o f circumference o f a circle. 2. Length of arc = -- x 21tr
360°
3. It is the region enclosed by an arc and the radii joining the ends of the arc to the centre of the circle.
4. Angle between the radii which are connected to the end points of the arc of the sector, is called angle of a sector and it

(i �)
is generally represented by 'fr.

5. Perimeter of a sector = 2n +
360° .
�. Area of minor segment = _e_ 1t r 2 - Area of triangle inclosed between radii and chord of segment. t--
3600 z
7. Required area = Area of figw·e B Area of figure A-
u..J

True/False
1 . False 2. False 3 . True 4. False
5. True

Fil l i n the B l a n ks
1. 471 2. 38.5 3. 4 4. 44

Match the Col u m ns


1 . (d) 2. (c) 3. (a) 4 . (b)
S U M MARY
• The dista nce covered by travelling once around a circle is called perimeter of a ci rcle. Usually, it i s known as circumference.
• Let the radius of a circle be r, then
(i) Area of circle = Tt r 2 (ii) Circumference of circle = 2rrr
1[(2
(iii) Area of semi-circle = Ttr 2
1
- (iv) Area of a quadrant = -
2
Let r a n d R be the inner and outer radii of a ring. Then, area of the ring = rr ( R 2 - r 2 ) = rr ( R + r ) ( R - r )
4

(i) Dista nce moved o r covered by a wheel i n 1 rotation = Circumference of the wheel
( 11") N um b e r o f rotations of a w h ee 1 to cover a certain d 1stance =
· ·
' . Distance covered
-------

Circumference
• A l i n e segment joining any two points on a circle is called a chord of the circle.
• A l i ne which i ntersects a circle at two points, is called a secant of the circle.
• Arc i s a contin uous piece of circumference of a circle.
• An a rc which ma kes an a ngle less than 1 80°, is called minor arc and other arc is called major arc.
• A dia m eter d ivided a circle into two equal arcs. Each of these two arcs is called a semi-circle .
• One-fou rth part o f a circle is c a lled a quadrant of the circle.
• The region enclosed by two radii and the corresponding arc of a circle is called a sector of ci rcle.
0

P M .i nor sector
0
• Length of a n a rc of a sector of an angle 0 = -- x 2nr
360°
oi Area of the sector = _e_ x Ttr 2 . Also, Area of the major sector = rrr 2 Area of minor sector
-

360 °
• The region bounded by a chord and the corresponding arc of the circle is called a segment of circle. The seg ment
conta i n i n g the m i nor a rc is called a minor segment and the segment containing the major a rc is called the major segment .
0

M i nor seg m e nt

') Are a of the segment A PB = Area of the sector OAPB Area of !lOAB = - _ e_ x rrr 2 - Area of !l OA B
360 °
Also, a rea of m ajor segment A OB = Ttr 2 - Area of the minor segment APB
e· Ar e a of squ a re = (Side) 2 • Area of rectangle = Length x Breadth

a Area o f a n equ il ate ral triangl e = .[3 (Side) 2


4

= �x Base x Height
2
w Area of a tria n gle
T E E Cl
I Very Short A nswer Type Questions [ 1 Mark e a c h ]

1. I f t h ar a f th ir I i 3 0 1 . 8 4 cm2 , then find radius 8 . I f the area o f a circle is numerically equal to twice its
o f c i rcle. circumference, then. find the d iameter o f the circle.
CCE 20 1 3
2. Find the a rea o f the circle that can be inscribed in a
. square of side 6 cm. 9. The area of a circular playground is 22 1 76 m 2 • Find
the cost of fencing this ground at the rate of � 50 per m.
3. If the perimeter o f a sector of a circle of radius 5. 7 cm NCERT Exemplar
is 27.2 cm, t hen fi nd the a rea of corresponding sector.
4. The area of a circle is 2464 cm 2 • Find the diameter of
1 0. If the perimeter of a protractor is 72 cm, then
calculate its area. CCE 20 1 3
circle .
1 1 . Find the area o f a quadrant o f a circle, whose
5. The difference be tween the circumference and the circumference is 44 cm.
rad ius of a circle is 3 7 cm. Find the area of the circle.
1 2. Find the area of a quadrant of a circle, whose
6. If the circumference of two concentric circles forming circumference is 22 cm. CCE 20 1 3
a ring are 88 cm and 66 cm, then find the width of the
ring . CBSE 20 1 2
1 3. The diameters o f front and rea r wheels o f a t ractor
are 80 cm and 2 m, respecth dy. Find the number of
7 . Find t h e a rea o f t h e sector of a circle o f radius 5 cm, i f revolutions that rear wheel will make i n covering a
the corresponding arc length i s 3 . 5 cm. distance in which the front wheel makes 1 400
NCERT Exemplar revolutions. NCERT Exemplar

�I Short A nswer Type I Questions [2 Mar ks e a c h ]


14. A road which is 7 m wide surrounds a circular park 1 8 . All the vertices of a rhombus lie on a circle. Find the
whose circumference is 3 5 2 m. Find the area of road. area of the rhombu s, if area of the circle is 1 2 56 cm 2 .
CCE 201 2 [Take, 7t = 3 . 1 4 ] NCERT Exemplar
1 5. A circular park is surrounded b y a road 2 1 m wide. If
the radius o f the park is 1 05 m, then find the area of 1 9. Area of a sector of a circle of radius 36 cm is 54 7t cm 2 •
the roa d. NCERT Exemplar Find the length of the corresponding arc of the sector.

1 6. A wire in a s hape of a square of side 88 cm is bent, so 20. Find the area of the square that can be inscribed in a
as to form a circular ring. Find the area of the circle. circle of radius 8 cm. NCERT Exemplar

I 7. A circular pond is 1 7. 5 m of diameter. It is surrounded 2 1. The areas of the two circles are in the ratio o f 4 : 9 .
by a 2 m wide path. Find the cost of constructing the Find the ratio between their circumferences.
path at the rate of � 25 per m 2 • NCERT Exemplar

�I Short Answer Type I I Questions (3 ar eacM

]
2 2 . A wire when bent in the form of a square enclosed an 2 4 . Circular footpath of w i d t h 2 m i c o n t ru e r d a t th

[
area 1 2 1 sq cm . If the wire wa s bent in the form of a rate of � 20 per m a round a circ ular park f rad iu
2,
circle, then find the area enclosed by the circle. 1 500 m. Find t h e t ta l c t o f c n r ru i n f t h
footpath. [ Take, n = 3 . 1 4 ] CBSE 20 1 2
Take, n =
22 CBSE 201 1
7
2 5. The minute ha n d o f a cl k i 20 m I n fi nd the ar a
23. A bicycle wheel makes 75 revolutions per minute to on the face of t h l k d d b t h minut ha n d
ma intain at a speed of 8 . 9 1 km/h. Find the between 8 am a n d 8 : 45 m .
diameter of the wheel. CBSE 20 1 2
All/none MATHEMATICS C la s s 1 0t h Te rm 11

2 6 . I n t h e g iven figure, 0 is the centre o f the concentric circles. Radius o f t h e i n n e r c i rc l e is h a l f t he rad i u f t he u t


circle. O f LDOE = 1 3 5 ° and OD = 14 cm, then calculate the area o f t h e s haded reg ion .
[ Leave your a nswer in terms of 7t]

�I Long A nswer Type Questions (4 ar s e c

2 7 . F rom a n aluminium plate, which is a square of side 32. Find the area of t h e s e g m e n t o f a c i rc l of rad iu
1 2 . 5 cm, a circular disc of diameter 7 cm is cut-off. 1 2 cm, wh os e corre s p o n d i n g sect r ha a centr 1
F i n d the weigh t o f the remaining part, if l sq cm of angle of 60 °. [ Ta ke, rt = 3 .1 4 ) NCERT Exemplar
Three circles each of rad i u s 3 . 5 c m ar drawn in
the plate weight 0 . 8 g.
33.
2 8 . Find the a rea of the shaded field shown in figure. such a way t h a t each of t hem tou h s t h other
NCERT Exemplar two. Find the a rea enclosed bet we n t he e cird .
NCERT Exemplar

t Four circular cardboard piece s of rad i u 7 cm ar


l E
3 4.
"<t" placed on a paper in s u c h a way t ha t each pie
E t touches other two piece s . Fin d t he a rea of t he
portion enclosed between t he s e piec s .
co

! NCERT Exemplar

s[7\Jo 1b cm
35. Floor of a room is of d i men s i o n s 5 m x 4 m and i t i
2 9 . Find the a rea of the shaded D R covered with circula r t i l e s o f d iamet r 50 cm each

Jl]B j
as shown in fi g ure F in d t h e a rea of floor t h t
C
region in figure, where arcs .

d ra wn wit h centre A , B, C and remains uncovered w i t h t ile s. [ Ta ke, 7t 3 . 1 4 ]


=

D intersect it pairs at NCERT Exemplar


mid-poin t s P , Q, R and S of
the s id e AB, BC , CD and DA, p
respectively of a square ABCD. [Take, 7t = 3J4]
3 O . In the g iven figure, arcs have been drawn with
radius 1 4 cm each and with centres P , Q and R.
Fin d the area of the s haded region.
p
3 6 . Find the area of the flower bed ( wi th semi-circul r

10t
end s ) shown in the fig u re . NCERT Exemplar

'----+----+-___. R
cm
NCERT Exemplar

3 1 . Find the area of the minor segment of a circle of


i
radius 1 4 cm, when the angle of the corresponding 3 7. A ceiling fan has three win g s a s shown in t h
sector is 60 ° . NCERT Exemplar figure. Find t h e length o f a rc d e s c ri b e d betw en
two consecutive wings, where the le n g t h of a h
wing is 0.98m. CCE 201

B
- Minor segment
Areas Related to Circles / 32 9

[ ]
38. I n llPQR o f t hc g i vcn fig ur
7
, LP = 58 ° , LQ = 42 °, a nd LR = 80 ° . With P, Q and R as cen t res,
circles of equ a l rad i i cm ar d rawn. Find ch area common to the MQR a nd the t h ree circles.
22
Ta ke, " = 7 .
CC E 201 5

� I Value Based Questions (VBQs) ( 4 M arks e a c h ]


3 9. An archery target has

{ - 1J .
perpendicular t o the radius OA a n d meets
three regions formed OP produced at B. Prove that the perimeter
by three concentric of shaded region is
circles as s hown in tan 8 + sec 8 + � CCE 2 0 1 6
the figure . If the 1 80 °
diameters of the B
concentric circles are
in the ratio 1 : 2 : 3,
then find the ratio of the areas of three
regions. NCERT Exemplar

40. For inauguration of A


Eco- Club of the
school, badges were
given to teachers by
42 . [HOTSJ In the given figure, PQR is a right
the students . S eema
made these badges
angled triangle at P. Find the area of
shaded region, if PR = 4 cm, RQ == 5 cm and
in the shape of an .
c
equilateral triangle
' I is centre of incircle of LlP QR. C BSE 200 9
of side 6 cm with a 8
circle of radius .J3 cm inscribed in it as R
shown in the figure.
(i) Find the area of t he shaded p o r ti o n .
(ii) Which value is de p ic t e d ? CCE 20 1 5

4 1 . [f[oj� In figure is shown a sector OAP of a


circle with centre 0, containing Le. AB is Q
t--
z
u.J
A nswers
.

:2:
1 . 9.8 cm 2. - cm 2
1 98
3. 45.03 cm2 4. 56 cm 5. 1 54 cm 2
7
6. a5 cm 7. 8.75 c m 2 8. 8 units 9. � 26400 1 0. 308 cm 2
1 1 . 38.5 cm 2 1 2. 9.625 cm 2 1 3. 560 1 4. 2 6 1 8 m2 1 5. 1 5246 cm 2
(./)
cm2
.
1 6. 985 6 c m 2 1 7. � 306 1 . 50 1 8. 800 cm 2 1 9. 37t 20. 1 2

21 23. 63 cm 24. � 37705 1 . 20 25 · 7 cm2


66 00
2 : 3 22. 1 54 c m 2

28. (32 27t) 29. 30.96 cm 2


147 .,
26 . - 7t cm -
4
27. 94.2 g + 30. 30 cm 2
31 . ( 1 0 2 . 6 + 49J3) sq cm 32. (75.36 - 36/3) cm2 33. 1 .97 cm 2
cm 2 35. 4.3 m 2 36. (280 + 25 7t) cm 2 37. 2.05 m 77 m2

(9J3 - )
34. 1 . 9625 38.
cm 2 (ii) Highligh ting the impo1·ta nce of an eco-fri ndly
66
39. 1 : 3 : 5 40. (i) nvir nm n t
7

7 cm 2 ::>
22
'"
42.
CHALLE N G E RS*
1. I f t h e hy pote n u s e o f a n i sosceles right angled triangle i s 7 2 cm, then find the area o f t h e c i r c l e i n s c r i b e d i n i t .

2. F i n d t h e d ifference o f t h e areas o f two seg ments of a ci rcle formed by a chord o f length 5 c m s u b t en d i n g a n a n g le of 90° at the
centre.

3. I n t h e g iven f i g u re , h av i n g two ci rcles which touch a t t h e point A, 0 is t h e centre o f the b i g g e r c i rc l e . I f C B = 9 cm, ED = 5 cm.
a n d MOO , COO , t h e n f i n d t h e area of the shaded region . [Take , rr = 3.1 4]
E

A B

4. I n t h e g iven fig ure, ABCO i s · a t rapezium with A B 1 1 DC, A B = 1 8 cm , D C = 3 2 c m a n d t h e d i stance between AB and
DC is 1 4 c m . I f arcs of equal rad i i 7 cm with centres A, B, C and D have been drawn , then find the a re a o f the s h aded region.

5. AB i s o n e o f t h e d i rect c o m m o n tangents o f two circles o f rad i i 1 2 c m and 4 c m respect ively t o u c h i n g e a c h other. F i n d the a re
of t h e reg i o n e n cl osed by the c i rcles and the tangen t .

6.

T'>�<l
I n t h e g iven f i g u re , ABCD i s a square of side 6 cm . Find the area o f t h e shaded region.

A� B
7 ·
ABCDEF is a reg u l a r h exago n . Consider each vertex as centre, draw a circle of radius 4 cm . F i n d t h e area of the shaded region.

8. D ra w a c i rcl e h avi ng centre A ( 4, 5) and radius J2. units. From any point P(2, 3), draw the pair of tangents to t h e c i rcle. Find the
area b e tween t h e p a i r of tan gents and cirscle.

* Th ese q ues tions m a y be o r may n o t be a s ked in the exa mination, have been g iven just fo r a dditio n a l p ra ctice .
Note q ues tio n s a re a va i l a b le
S o l u ti o n s to t h es e at page num ber 394-397.
C H A PT E R 9

SU RFAC E AREAS AN D VO LU M ES

In earlier classes, we have learnt to find the surface area and volume o f solid figures,
like; cuboid, cube, cylinder, cone, sphere and hemisphere etc. In this chapter, we will
1) Surface Area and Volume of learn how to find the surface area and volume of combination of solid figures,
Combination of Solids conversion of solids and frustum of a cone.
• Method to Find the Surface
Before starring topics of this chapter we will review some important definitions and
Area and Volume of
Combination of Solids formulae which will frequently use in this chapter.
0 Conversion of Solid from One
Shape to Another Solid Figu res
C> Frustum of a Cone The objects having definite shape, size and occupies a fixed amount of space in three
dimensions are called solids such as cube, cuboid, cylinder, cone, sphere and
hemisphere, etc.

Su rface Area
Surface area of a solid body is the area of all of its surfaces together and it is always
measured in square unit.
e.g. A cube has 6 surfaces and each surface is in a square shape. Therefore, its surface
area will be 6a 2 sq units, where a 2 is the area of each surface of the cube.

Volu me
Space occupied by an object/solid body is called the volume of that particular
obj ect/solid. Volume is always measured in cube unit.
e.g. Suppose, a cube has edge of length a units. Volume of a cube is equal to the
product of area of base and height of a cube i.e. a 2 x a = a 3 cu units.

I m po rtant Form u lae


(Related to So m e So l i d Figu res)
1 . Cuboid
A cuboi d is a solid body having 6 faces. L e t i ts length = I u n i t s
breadth = b units
and height = h un its. Then.
(I) Total surface a rea of cuboid = 2(/b + bh + hi) sq u n i ts
(il) Lateral surface a rea of cuboid = 2(/ + b)h sq u n i ts
(iii) Diagonal of the cuboi d = �{2 + b i + h i u n i ts
(iv) Volume of cuboid = I x b x h cu u n i ts

N o te Lateral surface area = Area of the 4 faces


.� 332 \ AllinOne MATHEMATICS ·
C lass 1 0th Term 1 .

2 . Cube 5. Sphere
C u b e i s a special case o f cub o i d which has A sphere is a sol id generated by t h e revolut ion of a semi-cirde
-----4---- about ics diameter. Let rad i us of sph e re be r un its. Then,
6 faces o f equal lengt h . Lee ics
l engch = b readth = heighc = a u n ics A
:. Each edge o f cube = a u n i t s
The n ,
( i ) Tocal surface a rea o f a cube
=6 x ( Edge )2 = 6a2 sq un i ts
( ii) La t e ral surface a rea of cube
= 4 x ( Edge )2 = 4a2 sq uni cs
( iii) Diago n a l of a cube = J3 x Edge = J3 a units B
( iv) V o l u m e o f a cube = ( Edge )3 = a3 cu u n i cs (i) Surface a rea of sphere = 4 nr .. sq u n i t s

(ii) Volume of sphere = � nr ' cu u n i t s


:>
3. Rig ht Circula r Cylinder
Cylinder i s a sol i d figure obtai ned by revolvi ng. (iii) Volume of a hollow sphere = � n< R 1 - r 1 ) cu un its
:>
A
che rectangl e . say A B CD. abouc i cs one side, say
where . r = i n n e r rad i us a n d R = oucer rad ius
B C. Lee base radius of righ c c i rcular cyl inder be

6. Hemisphere
r u n i ts a n d its height be h u n i t s . The n .
h
( I) C u rved surface a rea
= C i rcum fe rence o f che base x J'.eighc A plane passing through che cen c re
= 2rtrh sq uni cs cues the sphere in two equal p a r e s .
(ii) Toca I surface a rea = C urved surface area each pare is called a h e m i s p h e r e . L e e
D
+ A rea of two ends radius of hemisphere be r u n i c s . Th e n .
= 2rtrh + 2rtr2 = 2rtr(h + r) sq u n i ts (i) Curved surface a rea o f
hemisphere = 2nr � s q u n i ts
(iii) Volume of the cyl i n de r = Area of base x
(ii) Toca! surface area of h e m i sphere = 2nr 2 + nr 2
Height
2
= rtr2h cu u n i ts
= 3nr sq unics

4. Right Circular H ollow Cylinder (iii) Volume of hem i sphere = �


:>
nr 1 cu u n i c s
Lee R u n i ts a n d r u n i ts be che external and

7. Right Circular Cone


i n ternal rad i i o f che hollow cyl i nder and h units
be i ts heigh t . Then .
( i) C u rved surface a rea A right ci rcular cone is a sol id A
= CS A of outer cyl i nder + CSA of i n ner generated by the revolution o f a righc
cyl inder angled triangle about one o f its s i d es
= 2rtRh + 2rtrh = 2rt(R + r)h sq u n i ts containing the right angle as a x i s as
( ii) Total surface a rea shown in figure. Let height o f a right
= C urved surface a rea o f hol low circular cone be h units and i ts rad i us
cyl i n der + Area of both ends be r units. Then .
= 21t(R + r)h + 27t(R2 - r2 ) (i) Slant heigh t of the con e .
= 2rt(R + r)h + 2rt(R + r) (R - r) I = AC
= 2rt(R + r) [ h + R r] sq un i ts
- = r+h2 u n i t s
(iii) To cal oucer surface area = 27tRh + 2rt(R2 - r2 ) sq uni cs (ii) Curved surface area o f cone = rtr/ sq u n i ts
(iv) Volume o f hollow cyl i n der (iii) Toca! surface area of a co n e = Curved surface area
= Volume o f outer cyl i n de r - Volume of i n ner cyl inder + Area of the base = Ttrl + Ttr2 = nr(/ + r) sq units

(iv) Volume of cone = ..!.


= 1tR2h - Ttr2h = rt( R 2 - r2 )h cu un i ts
Ttr2 h cu u n i ts
3
Surface Areas a n d Vol u m es f333 f
TOPI 0 Su rface Area a n d Volu me of Com.bi nation of Solids
In our day-to-day l i fe w com across some complex solid Method of solving these types of p roblems can be
figures which are combi nation o f two or more similar or u nderstood wi th the help of the following examples.
different solid figu res. In this topic, we will learn co find ch�
surface area and v l u m e of combination of solids. Exa m p le 1. Three cubes each of side 5 cm are joined
end to end. Find the surface area of the resulting solid.
ethod to F i n d t h e S u rfa ce Area a n d Sol. Here, on joining three cubes, we get a cuboid whose
lengrh, I = 5 + 5 + 5 = 1 5 cm, breadth, b = 5 cm and
Volu me of C o m b i na t io n o f Solids height, h = 5 cm ·

Whenever we have co fi nd the s urface area of a solid, which

rJ [J [J QJm
is a combination o f o ther sol ids, we add the curved surface 5 cm 5 cm 5 cm

area of individual solids i . e.

Total surface area o f c o mb i n ation of solids = Sum of curved


surface area of i ndividual solids. II Ill

Volume of combination o f solids = Sum of vol ume of


individual solids. ,/

For fi n ding the s u rface area and volume of combination of : . Required surface area of the resulting solid
solids, first identify the solid which are i n combination and = Surface area of new cuboid
then apply above fo r m u l a. = 2(/b + bh + hf)
= 2(1 5 x 5 + 5 x 5 + 5 x 1 5 )
e.g. A solid formed by j o i n i ng two hemisphere at both the
ends of a right ci rcular cyli nder. = 2(75 + 25 + 75 )
= 2(1 75 ) = 350 cm 2

Example 2. An iron pole consists of a cylin der of


T oral surface area o f new solid
heigh t 240 cm and base diameter 2 6 cm, which is
surmounted by another cylinder of h eigh t 66 cm and
= Curved surface area of one hemisphere + Curved surface radius 1 0 cm. Find the mass of the pole given tha t
area of the cyl i nder + Curved surface area of other
hemispher� 1 cm 3 of iron has approximately 8 g m ass.
Volume of new solid [take, n = 3. 1 4]
= Volume of hemisphere + Volume of cylinder Sol. Here, solid iron p o le is a combination of two cylinders.
+ Volume of other hemisphere
ote In calcula t i ng t h e s u r face a rea . we do not add the surface

1
areas of t h e two i n d iv i d u a l solids because some part of the
1 0cm
surface a re a d i sa p p e a re d in the process of joi n i ng them. Bue
this will n o t b e t h e case when we calculate the volume.
The vo l u m e o f the s o l i d fo rmed by joi n i ng two basic sol ids

l
wi l l actu a l l y b e the sum of the volu mes of the i n d ividual
sol i d s . 1 3 cm
240 cm

Different Typ s of Problems Based on


Various o m b· nation of Solids

( Type I) When com bination of two or more than For.first cylinder,


two figu res of sam.e type is given !1eight = 240 cm
Base diamec r = 26 cm
Sometimes combination of two or more than two figures of
. · . Base radiu = - cm
26
same type such as combination of two or three cubes,
2
combinatio n of two cyl i nders etc are given and we have to
= 1 3 cm
find their surface area and volume. For finding surface area,
For second cylinder,
we remove area of that portion which is common in joining
Height = 66 cm
che figure.
Radius = 10 cm
[ 334 j All/none MATH EMATICS C lass 10th Term U

We know that, So, the total


= Total surfacesurface of the decorative block
a r a ofarea+cube - Arca of base of he mi ph ere
Volume of cylinder = TCr 2 h e s
:. Total volume of iron pole = Volume of first cylinder Curved surface area of hcmisphett
.+ Volume of second cylinder
240 + TC (10)2 66
= TC (1 3 ) 2 X X
= 150 - ' + 2 ' = 1 + ' = I 50 + -227 2 .1 2.1
TCr- TCr- SO m· - x x

==3.14
TC (169[40560240 6600)
X + 100 66]
X = 150+13.8 6 =163.8 6 cm �
+
=3.14 47160=148082.4
x cm 3 TRY YOU RSELF

[·: l g = _100 kg]


Hence, total mass
of the iron pole
Q. 1 A wooden article was made by scooping out
= 148082. 4 8g=1184659. 2 g [given, 1 cm 3 :::: g]
8 a
hemisphere from one face of a cubical wooden block..
x

1= 184659.2 kg = 1 184.66 kg 1 If each edge of cube is 1 O cm and diameter of base �


1000 hemisphere is 7 c m . then find the volume of wooden
article. [Ans. 9 1 0. 1 7 cm � ·
[Type II) When com bination of sphere
Q. 2 A hemispherical depression is cut out from one face oi
(or hemisphere) and cube/cuboid is given a cubical wooden block such that the diameter 4 of the
If combination of sphere (or hemisphere) and cube, combination hemisphere is equal to the edge of the cube..
[Ans. 4,, � (24 + n) sq units]
of sphere (or hemisphere) and cuboid are given in the question,
Determine the surface area of the remaining solid.
then find their surface area and volume separately and then find
surface area and volume of combination by adding them. Q. 3 Five cubes each of side 6 c m are joined end to end.
[Ans. 792
Find the surface area of the resulting cuboid.
Method of solving these types of problems can be understood cm:

with the help of following example.


[Type IIIJ When combination of cylinder and
Exa m p le 3. The decora tive block shown in the below sphere/hem isphere/cube/cuboid
fig u re is m ade of two s olids, a c u be and a hemisphere.
is given

[
The b a s e of the block is a c u be with edge 5 cm and the
. If combination of cyli nder and sphere/hemisphere as

J
h emisphere fixed on th e top has a diameter of 4 2 cm,
then fin d the total s urface a rea of the block and find the medicine capsule, vessel, etc, combi nation of cyli nd er
22 and cube/cuboid are given, then fo r finding the surface
total area to be painted. take, " = 7 area and volume of such combination, first we find the
4.2 cm surface area and vol ume of each figure separately and
then add them.

Exampl e 4. A solid is composed of a cylinder with


hemispherical ends. If the whole length of the solid

[
is 1 04 cm and the radius of each hemispherical
5 cm end is 7 cm, then find the cost of polishing its
surface at the rate of '!' 2 pe r d m 2
272 ]
• take, " =
S cm Sol. Given, whole length of the solid � - - - - ---•

=104
104 cm
Sol. Here, the decorative block is a combination of a cube and a cm
hemisphere. and the radius of each hemisphere
For cubical portion,
= ? cm
Each edge = 5 cm
For hemispherical portion,
Therefore, the length of the
cylindrical part of the solid
Diameter == 4.2 cm =(104 - 2 7)=90 cm
.
Rad 1us, r = - cm = 2.1 cm
4.2
x

For hemispherical portion,

Now , roral s u rfa ce


2
area of (Edge)
cube
rhe
Radius, = 7 cm
r

=6 2 = 6 5 5 = 150 cm 2 For cylindrical portion,


x
Radius, = 7 cm
x x

Here, the part of rh e cu be wh ere the hemisphere is attached, is not


r

included in rhe surface area. Height, = 90 cm


h
Surface Areas a n d Volu mes

:. Total surface
= 2 xareaCurved
of rhesurface
solid area of hemisphere Example 6. A toy is in the form of a cone mounted

= 2 [ 2 2; x ( 7)' ] + 2 x � x ( 7) (90)
on a hemisphere with the same radius. The diam eter
+ Curved surface area of cylin drical parr of the conical portion ,is 6 cm and its h eight is 4 cm.
= [ 2 ] + = 2 x ( 7)2 ] + ( 7) (90)
2 7t r 2 2 1trh [ 2 7t 27t Determine the surface area and the volume of the toy.
x x
[take, 7t 3.1 4)=
= 4 x 22 x 7 + 2 x 22 x 90 = 22 [28 + 1 80] = 4576 cm 2
Sol.
aHere,
hemigisphere.
ven coy is combination of a cone and
A

Then, rhe cosr of4polishing ar rhe race of� 2 per dm 2 For conicalportion,
Diameter = 6 cm
=� 576 x = � 9 1 .52 [·: l dm 2 = 1 00 cm 2 ]
1 00
2
:. Radius, = �2 cm = 3 cm
the
r
Example 5. A vessel is in the form of a hemispherical
.and
-. Slaheine height,ght,= 4 cm
h B c

[take, 7t = J
bowl mounted by a h ollow cylinder. The diameter of
hemisphere is 1 4 cm and the total height of the
l = ) + = .J9 + 1 6 = .Ji5 = 5 cm
r2 h2
vessel is 1 3 cm. Fin d the capacity of the vessel.
22 For hemisphericalportion,
7 Radius=3cm
Curved surface area= 3.Iof conical porti o[·n: =radius ofboch are same]
7trl
Here, che vesselof ais hemisphere
a [given, =314]
�1
Sol. x5
= 47.14 xcm23
? cm 7t
combination
and a cylinder. Curved surface area of hemispherical =porti56.52on
For hemispherical portion, = 27tr2 = x 3 . 14 x (3
2 )2 cm 2
Diameter = 1 4 cm Required surface area of coy= 47. 1 + 56. 52 = 1 03.62 cm 2
:. Radius, = !.! cm = 7 cm
r
2 Volume of conical portion = _!_3 7tr 2 h
For cylindrical portion,
)•
, �
Height, =To = -1 x 3 .I 4 x (3 x 4 = 37.68 cm·
h
= (1 3cal- 7)height of che vessel -Radi u s of
cm[giv=en,6 cmcocal height of che vessel = 1 3 cm] hemi s phere 3
and radius, R =radius of che hemisphere= 7 cm
. Volume of hemispherical portion = �3 1tr.l

:.Now,Capacicapaci
ty oftytheof chevesselvessel= Volwiul mebe ofequal
che cohemiics svolphereume.
+ Volume of che cylinder
= � x 3 .1 4 x (3)3 = 56.52 cm
3
Required volume of coy= 37.68 56.52 = 94.2 cm
+
·1

.1

= [( j x 272 x 7 x 7 x 7 ) + ( 2; x 7 x 7 x 6)]
m· 3
= 3 + 7tR2h

Example 7. A wooden toy rocket is in the shape of a

= ( 2�6 + 924 ) = (718.67 + 924) = 1642.67 cm'


cone mounted on a cylinder, as shown in figure. The
height of the entire rocket is 26 cm, while the height of
the conical part is 6 cm. The base of the conical

1
portion has a diameter of 5 cm, while the base
diameter of the

�)
Hence, the capacity of the vessel is 1642.67 cm 3• cylindrical portion is 6 cm
3 cm If the conical 26 cm --....---==-.::;'.....
rr_yp;iVJ When combination of cone and
.
portion is to be
sphere/hemisphere/cylinder is given painted orange and
Sometimes combination of cone and sphere or hemisphere the cylindrical portion s cm
as playing toy (spinning top) , ice-cream, etc. and other yellow, then find the

J
combination i.e. combination of cone and cylinder as area of the rocket
playing toy (rocket) , tent, etc. are given in the question and painted with each of ease of cylinder
for finding their total surface area and volume, we use the these colours. Base of cone
above given formula. Method of solving these types of [take, 1t = 3 .1 4]
problems can be understood with the help of followi ng
examples.
Sol. Here,a cylinder.
and che given wooden roy rocke r is co mb inatio n of a cone
\ 336 \ AllinOne MATHEMATICS Class 1 0th Term ll
For conicalportion, Sol. Here,a hemi givensphere.solid toy is a combinacio n of a right circular cone
Diameter = 5 cm and
Letthe base
BPCofbethethehemi hemisphere ABC be chc cone standing
andshown Oil
Radius, 7j = �2 cm 2.5 cm =
s phere as in chc figure.
Height, "1 = 6 cm---___ For conicalportion,
Height, h = 2 cm
Then, slant height, l = -'1,.< 2... . 5 )2 + 62 [·: l :;:: �r2 h2 ]
+
Diameter, = 4 cm
d
= �6.25 + 36 Radius, r = i2 cm = cm 2
= .J42.25 = 6.5 cm
For cylindricalportion, For hemisphericalportion,
Diameter = 3 cm Radius, r = 2 cm
Radius, r2 = �2 cm = 1.5 cm [·: radii of hemisphere and cone are same)
So, volume of the solid toy
=Volume of hemisphere + Volume of cone
= [� 314 )( ( 2 )3 + � )( 3.1 4 )( ( 2 )' )( 2]
Height, h2 =Total height of rocket -Height of cone
= 26 - 6 [·: total height of rocket= 26 cm]
= 20 cm
= �3 7tr3 .!..3 1tr2h
+

Here,
orangeweandhaveyellowto findcolours
the area of they. rocket painted with
separatel )(

Since,
cylinderradiandus cone
of baseis mounted
of cone isonlarger
cyl i than
n der. radius of base of = 16.75 + 8.37
:. Aiea to be painted orange = Curved surface area of cone = 25.12 cm3
+ Aiea of base of cone Hence, volume of the solid toy is 25.12 cm 3•
-Aiea of base of cylinder Now, solletidtheHP=
toy.rigThen,
ht circular cylinder EFGH circumscri berculthear
So, volume of the cylinder= 7t r 2h = 3.14 22 4 = 50.24 cm,
[·: area of= base of cyli n der is common in area of base of cone] gicylviennder= radius of the base
BO = 2 cm and its height, of the ri g ht ci
1t7jl + 7t 1j 2 - 1tr22
= 3.14 2.5 6.5 + 3.14 (2.5)2 -3.14 (I .5)2
x x x x
EH = AP = AO + OP = + 2 = 4 cm 2
x x
=3.14 [16.25 + 6.25 - 2.25]2
. x
= 3 1 20.25 = 63.585 cm
4
Now, area to be painted yellow
Now,
the solrequiid toy=red Volumedifferenceofofcylinder
the volume d toyand
of theofcylsolinider
-Volume
=Curved surface area of cylinder = 50.24 - 25.12 = 25. 12 cm·'
+ Aiea of base of the cylinder = 2rcr2h2 7tr22 +
TRY YOURSELF
x
= 2 3.14 1.5 20 + 3.14 (1.5)2
x x x

=3.14 [60 + 2.25 ] =3.14 62.25 x Q. 1 A spherical glass vessel has a cylindrical neck 7 cm long.
= 195.465 cm 2
[ 1
4 cm in diameter; the diameter of the spherical part is
21 .cm. Find the quantity of water it can hold . use, 1t = 22
Example 8. A s olid toy is in the 7 -
(Ans. 4939 cm �}
form of a h emisphere s urmo unted
by a right circular cone. The height Q. 2 A solid is composed of a cylinder with hem i sph e ri cal ends.
of the cone is 2 cm and the If the whole length of the solid is 1 08 cm and th e diameter
diam eter of the base is 4 cm. of the hemispherical ends is 36 cm, find th e cost ot
polishing the surface of the solid at the rate of 7 paise per
[
Determine the volume of the solid
toy. It a right circular cylinder
circ um scribes the toy, then find the
H �-��=----i G
f.. .._________ __
sq cm. use, =
7t 22
7 J [Ans. ' 85 5 .*l

differen ce of the volumes of the cylinder and the toy. Q. 3 A circus tent is cylindrical to a height of 3 m an d conica
[take, 7t = 3 . 1 4 ] above it. If its base radius is 52 .5 m and slant h e ig ht of ti�
conical portion is 53 m, find the area of canvas needed to
make the tent. (Ans. 9735 m :
NCE RT FO L D E R 9.1
1 2 cubes each o f volume 64 cm 3 are joined 3 A toy is in the form of a cone of radius 3 . 5 cm
end-to -end . Find the surface area of the surmounted on a hemisphere of same radius .
resulting cuboid . The total height o f the toy i s 1 5 .5 c m . Find the
Sol. Given, the volume o f each cube is 64 cm 3 • total surface area of the toy.

l� I� bJ
Lee each side of che cube be 11 cm. Sol. Here, coy is a combination of a

r
A
hemisphere and a cone.
Given, coral height of coy,
4 cm
AD = 1 5.5 cm E
(.)
For hemisphericalportion, LO
t.ri
4 cm 4 cm
Radius, OC = OD = OB = 3.5 cm
3 For conicalportion,
Height, OA = AD -
Volume of cube = (S ide}
64 =a ·' [given]
OD
= 1 5.5 - 3.5 = 12 cm
=> a = 4 cm and radius = 3.5 cm D
When two cubes arc joined end-co-end, then we gee a Now, coca) surface area of the coy
cuboid whose length , l ( 4 + 4) = 8 cm, breadth, b = 4 cm = Curved surface area of cone
and height, h = 4 cm.
+ Curved surface area of hemisphere
J J
:. Surface area of chc resulting cuboid
= 2 ( lb + bh + hi) = 2 (8 x 4 + 4 x 4 + 4 x 8) = rcrl + 2rcr2 = rcr h2 + r 2 + 2 rcr 2 [·: l = h 2 + r2]
= 2 ( 32 + 1 6 + 32) = 2 (80) 1 60 cm 2
=
= 22 x 3.s x Jo 2 >2 + c3.s f + 2 x 22 x (3.5) 2
7 7
2 A vessel is in the form of a hollow hemisphere
mounted by a hollow cylinder. The diameter of
= 1 1 �144 + 1 2.25 + 22 x 3.5 = I I .JI 56.25 + l l x 7
the hemisphere is 1 4 cm and the total height of = 1 1 (1 2. 5) + 77= 137. 5 + 77 = 2 1 4. 5 cm 2
the vessel is 1 3 cm. Find the inner surface area of
the vessel.
4 A cubical block of side 7 cm is surmounted by a
hemisphere. What is the greatest diameter of
Sol. Here, vessel is a combination of a 14 cm the hemisphere can have? Find the surface
hollow hemisphere and a hollow area of the solid.
A
cylinder.
7 cm
Sol. Given, a cubical block is surmounted by a hemisphere.
For cylindrical portion, Therefore, diameter of hemisphere muse be equal co the
Diameter = AB = DC = 1 4 cm E
(.) side of cubical block and ic is the greatest diameter of
co
:. Radius = OB = O' C = O' P E hemisphere.
(.)
AB 1 4 ("') ..-- Hemisphere
=- =-
2 2
= ? cm
Total length of vessel, PO = 13 cm Cube
:. Length of cylinder,
00' = PO - O'P = 1 3 - 7 = 6 cm p
For hemispherical portion, u..I
CJ.
__:!
Radius of hemisphere
-�
'
= Height of hemisphere = 7 cm
"=·
Now, the i nner surface area of �he vessel +--- ? cm --
= Curved surface area of cylinder For cubical portion,
E d ge
+ Curved surface area of hemisphere = 7 cm

2
= 2 rcrh + 2 rcr2 = 2 x 22 x 7 x 6 + 2 x
22 x (7) 2
7 For hemispherical portion,
7 D iame ter = 7 crn
= 2 x 22 x 6 + 2 x 22 x 7 7

= 44 (6 + 7) = 44 x 1 3 = 5 72 cm Rad ius, r == l cm
338 } AllinOne MATH EMATICS C lass 1 0th Term II

Now, total surface area of solid Sol. Given that, medici ne capsule is a

- - - ----
= Su rface area of five faces of the cube combination of two hemispheres
+ C urved surface area of hemisphere + (Area of face and one cylinder. -- - - -

(H { ( 7 )' - � x GJ}
E
ABCD PQRS)
- Area of circular space Also, given diameter of the capsule 2. 5 mm
E
= 5 x (Edge) 2 + 2 7t (r ) 2 + {(Edge) 2 - 7t (r ) 2 } = 5 mm '<:1'
E
E

= 5 x (7)' + 2 x 2; x
( )
Cl
:. Radius = 2_ = 2. 5 mm
+ 2
and length of the capsule = 1 4 m m
= 5 49 + 1 1 x 7 +
x 49 - 7; : . Length o f the cylindrical portion
= 1 4 - ( 2.5 + 2 . 5 ) = 9 mm 5 mm
= 245 + 77 + 49 - 772 Now, curved surface area of one hemispherical ponion
= 21tr-, = 2 x - 275 mm·,
22 x 2.5 x 2.5 = --
= 371 - 772 = 665
2 = 332.5 cm
.? 7 7
and curved surface area of chc cyl indrical portion
Hence, required surface area of the solid is 332.5 cm 2 22 x 2 . 5 x 9 = -
22 x 45 = -
990 ,
• ·
= 2nrh = 2x - mm
5 A hemispherical depression is cut-out from 7 7 7
one face of a cubical wooden block such that Now, required surface area o f medici ne capsule
the diameter I of the hemisphere is equal to the = 2 x Curved surface area of one hemispherical ponion
edge of the cub e . Determine the surface area + Curved su rface area o f the cylindrical ponion
of the remaining s olid.
=2x
27 5 + 990 = 550 + 990 = 1 540 = 220 mm 2
Sol. G iven , side of the cube 7 7 7 7 7
= Diameter of the hemisphere = I units
7 A tent is in the shape of a cylinder surmounted
by a conical top. I f the height and diameter of
the cylindrical part are 2 . 1 m and 4 rn,
respectively and the slant height of the top is
2.8 m, then find the area of the canvas used for
making the tent. Also , find the cost of the canvas
of the tent at the rate of � 5 0 0 per m.
p B
[Note The base of the tent will not be covered
:. Radi us o f the hemisphere, r = !.._ units with canvas. ] CCE 2016
2
N ow, required surface area of the remaining solid
= Area of one face ABCD of the cube - Area ofcircle PQRS
+ Surface area of rem aining five faces of the cube
+ Curved surface area of hemisphere
Sol. Given chat, tent is a combi nation
of a cylinder and a cone.
For conical portion,
l �.�
p

:I
I

Slant height, I = 2 .8 m
D -:::�:::c::::::
'0' - - - - -
C
= (Edge) 2 - 1tr2 + 5(Edge) 2 + 2 7tr 2 Radius, r = Radius of cyli nder i- -
...
__
_
_ _

2 2 .-
= 12 - n -1 + 5 12 + 2n -1
4 4
Diameter
2
= _! = 2 m
2 I
I
: 2m
E
I
I
12 I
For cylindrical portion, I
= -4 (7t + 24) sq units I
I r
Radius, r = i = 2 cm
,-- - - --- -----

Q
-
,

2 A _ _ _ _ _
_ _ _
_ _ __ __ ,a
6 A medicine capsule is in the shape of a
cylinder with two hemispheres stuck to each of Height, h = 2 .1 m 4m
:. Required surface area of che tent

)
its ends (see below figure) . The length of the

(
'
entire capsule is 1 4 mm and the diameter of Curved surface area of cone
th e capsule is 5 mm. Find its surface area. + Cur\red surface area of cylinder
22 2 x {2.8 + 2 x 2. 1)
= 7trl + 2 7tr h = 7tr ( I + 2 h) = - x
7
__
5 mm 44 44
= - (2.8 + 4.2 ) = - x 7 = 44 m
2

7 7
·: Cost of the canvas of the tent at che race of � 500 per m
� :
14
= Surface area x Cost per m = 44 x 500 = 22000
mm 2 �
Surface Areas a n d Volumes / 339
8 From a solid cylinder whose height is 2.4 cm 9 A wooden article · was made by
and diameter 1 . 4 cm, a conical cavity of the scooping out a hemisphere from
same height and same diameter is hollowed each end of a solid cylinder, as
out. Find the total surface area of the remaining shown in adjacent figure. If the
solid to the nearest cm 2 • height of the cylinder is 10 cm and
Sol. Herc, a cylinder is hollowed our in a its base is of radius 3.5 cm, then
conical caviry. find the total surface area of the
Given diameter of cylinder article.
= Diameter of conical caviry =
:. Radius of cylinder
cm 1.4 Sol. Given, wooden article is a combination of a cyli nder and
cwo hemispheres.
= Radius of conical cavity
=
Diameter
2 = .!.:_i =
2 0_7
cm
Height of chc cylinder
= Height of che conical cavity
= 2. 4 cm
:. Slane height of the conical cavity,
I = �h 2 + r 2 = �(2.4 )2 + (0. 7 ) 2
= �5. 7 6 + 0. 4 9 = .J6. 2 5 = 2. 5 cm
Here, height of the cylinder, h = I 0 cm
·: Radius of base of the cylinder
Hence, total surface area of remaining solid
= Curved surface area of conical cavity = Radius of hemisphere, r =3. 5 cm

++rcrl + 2rcr + rcr2 rcr(l + 2h + r)


Curved surface area of cylinder Now, required coral surface area of che wooden article
Area of che base of the cylinder = 2 x Curved surface area of hemisphere
= h= + Curved surface area of cylinder
22 = 2 x (2re r 2 ) + 2rerh = 2 rer (2r + h)
= - x 0. 7 x (2. 5 + 2x 2.4 + 0. 7 )
7 =
22 3.5 x (2x 3.5 +10)
2 x -x
= 22 x 0.1 x (2.5 +4.8 + 0. 7 ) 7
= 2. 2 x 8 22
-
2
= 1 7 .6 :::::: 18 cm 2 =
7 x 7 x ( 7 +IO)= 22 x 17 = 374 cm

T FO L D E R 9.2
( )
rer J = _!_ re (1)2 (1) + 3_ re l J
1 A solid is in the shape of a cone standing on a
hemisphere with both their radii being equal to 3re rerih2 + 3.3(re+
= _!_
3 rt 3
1 cm and the height of the cone is equal to its
radius . Find the volume of the solid in terms of
= -3 + -3 re= 3
2rc)
=- = rt n1

7t.
A
2 Rachel, an engi n ee ri n g student was asked t o
Sol. Given, solid is a combination make a model shaped like a cylinder with two
of a cone and a hemisphere.
cones attached at its two end by usi n g a thi?'
Also, Radius of the cone, r aluminium sheet. The di ameter o f th mo d el 1
= Radius of the hemisphere a e.---....-:i:r---� 3 cm and its length is 1 2 cm.
If each cone has a hei g ht o f 2 m, th en fi nd the
= l cm
h1
Height of the cone, = cm
:. Required volume of the solid volume of air c on t ai n e d i n th m odel that
Rachel made. (Ass u m th o uter an d inne r u
dimensions of the rood 1 t o b e ne a rly the sa me . )
= Volume of che cone
+ Volume of che hemisphere 2
3 40 : Allinone MATH EMATICS C la s s 1 0th Te rm 1 1

Height o f cylinderical p a r e,
(I
Sol. G iven , model is a combination of a cylinder and cwo
-
cones . Also, volume of che air will be equal co che sum of h = PQ - ( PR + SQ ) . + l. )
rhe volumes of rwo cones and one cylinder. = 5 - 2.8 = 2 . 2 c m

[- \] , -
= ED = 3 cm BC
r
We have, diameter of rhe model, :. Volume of one gulabjamun
= 2 x Volume of hemispherical a r
:. Radi us of cone = Radius of cyli nder, r = � = 1 .5 cm
f cyl i nd rical pan

]
+ Vo l u m e

[� ] [-
2
Height of cone, h1 = 2 cm 2 4 \ f.
=2x rcr· + 7t r- h = rtr + 1 v· - 'J

� � [± � ]
Toral length of che model, AF = 1 2 cm 3 3

x 1 .4 x I . + 2 .2
B D 22
\
= it r ' + h = x I .

CJ -
I

�]
I 7
: E

?_ [
_,_ - - - _ - - -
A
: U1
F
=
22
x x x + 22
I 7 10 10 3 1 0 1 0
I

------
I

B ern 1 54 � 9 94 c m
= 22 x .!._ x
28
+ x

= AF - ( A O + O' F)
c = =
------ 1 2 cm 5 5 15 5 25 1 5 375
:. Lengch of che cylinder, 00' Now, volume of 45 gulabjarn uns

( , )
= 1 2 - ( 2 + 2) = 8 cm = h2
9394 3 x 9394 28 1 82 1 1 2 8 c m
Now, volume of che air inside che model = 45 x = = = 27.
= Vol ume of air inside (cone + cylinder + cone) 375 25 25
Since, one gulabjamun conca ins sugar syrup u p ro about
= '3 rcr-h1 + rcr-, h2 + '3 rcr-,h1
1 1
of ics volume.
Hence, quancicy of syrup found in 45 gu labjamuns
= .!_ rtr2 ( h1 + 3 h-, + h1 ) = 1 1 27.28 x
30
= 1 1 27.28 x i_ = 338. 1 4 ::::: 3 8 c m
3 1 00 10
1 22
= - x - x 1 . 5 x 1 . 5 (2 + 3 x 8 + 2) 4 A pen stand made

--
3 7
22 2.25 of wood is in the
=-x x ( 2 + 24 + 2) shape of a cuboid
7 3
with four conical �
22
= - x 0.75 x 28 = 22 x 3 = 66 cm
7
3
depressions to LJ/"
hold pens. The
3 A gulabjamun contains sugar dimensions of the cuboid are 1 5 cm x 10 cm
syrup upto about 3 0% of its x 3.5 cm. The radius of each of t h e depression i
volume. Find approximately 0.5 cm and the depth is 1 . 4 cm. Find the volum
how much syrup would be of wood in the entire stand (see fig u re) .
=:=:=-s-:=�==�-s.�:=Y-=:=:=
Sol. Given, length of cuboid ( /) = 1 5 cm, b readth of cub
.
found in 4 5 gulabj amuns, each
s haped like a cylinder with two
hemispherical ends with length
5 cm and diameter 2 .8 cm
(see figure) .
ia1�
- - - - - - - - - - -- -
:: : : : :
: :-:
(b) = 1 0 cm and height of cuboid ( h ) = 3 5 cm
:. Volume of cuboid = I x b x h
= 1 5 x 1 0 x 3.5 = 525 c m
Also, radius of conical depression, r = 0 . 5 cm
Sol. Given, o ne gulabjamun is a combination of a cylinder and height of conical depression, h = 1 . 4 cm
and rwo hemispheres. H ere, coral length of one
:. Volume of one conical depression = .!_ rt x 1·2 x h

c r \+ -
gulabjamun = 5 cm and diameter = 2.8 cm
2.2 cm
3

1
0 . 5 cm
I 22
A B = - x - x 0.5 x 0.5 x 1 .4

-!R ---
3 7
P ?,� �
r
- - - - - - - _ _ - - 22 1 1 2 11
Q = - x - x - x - = - cm 3
I I
I I
3 2 2 1 0 30
D C Now, volume of 4 conical :i::.

l
Guiab jamun
depressions = 4 x Volume of one a
� �::i �i 5 cm conical depression 3
;:} � = 22 cm 3
:. Radius of cyli n d rica l pare = Radius of hemisphe rical pare =4x
30 1 5
= r = � = 1 .4 cm c
2
Surface Areas a n d Volu mes , 3 41
Hence, the volume of w od in the entire stand and radius of the first cylinder,
1j = - = 1 2 cm
= Volume of cuboid - Volume of 4 conical depressions 24
2 2
= 525 - 2 Height of the second cylinder,
15
3 h2 = 60 cm
= 5 2 5 - 1 .47 = 523. 53 cm
and radius of the second cylinder, r2 = 8 cm
5 A vessel is in the form o f an i nverted cone. Its . " Volume of iron pole
height is 8 cm a n d the ra dius of its top, which
= 7t 'i 2 h, + 7t r22 h2 = 7t (1j 2 h, + r2 h2 )
= Volume of first cylinder + Volume of second cylinder
is ope n , is 5 cm. It is fi lled with water upto the
2
1 6 cm
bri m . When lead s h o t s , each o f which is a = 31 4 (144 x 220 + 64 x 60)
sphere of radius 0 . 5 cm are dropped into the [·: 1j = 1 2 cm, h1 = 220cm
ves s e l , one-fourth of the water flows out. Fin d and r2 = B ern, h2 = 60cm]
t h e n u mber of l e a d s hots dropped in the = 3. 1 4 (3 1 680 + 3840) = 3. 1 4 x 35520
vessel. CCE 20 1 5 = 1 1 1 5 32.8 cm 3

� [ � J
Sol. Here, a cone a n d spherical lead Also, given that,
3
_ _ _
1 cm of iron pole has approximately mass = 8 g E
u
shoes arc given . Since, lead c
J----=-----'
0
N
shoes are dropped i nt o the
= - : lg = N
vessel, so lead shoes flow out the 1 0 0 kg 1 0 kg
water from the vessel equal to
3
. . 1 1 1 532.8 cm iron has approximately mass
its volume.
8 892262.4 24 cm
Given, height of the vessel,
h = 8 cm and radius of the vessel
= 1 1 1 532 .8 x -- = 892.26 kg Pole
1 000 1 000
r = 5 cm
:. Volume of water filled in a 7 A solid consisting of a right circular cone o f
vessel height 1 2 0 c m a n d radius 60 c m standing o n a
= Volume of con hemisphere of radius 60 cm is placed upright in a
= - x n x r X flI
1 2 right circular cylinder full o f water s u c h that i t
3 touches the bottom . Fin d t h e volume o f water left

= - x - x 5 x 5 x 8 = -- cm
1 22 4400 3 in the cylinder, if the radius o f the cylinder is
3 7 21 60 cm and its height is 180 cm .
Also, radius of a spherical lead shots = 0.5 cm S o l. Given, solid is a combination of a cone and a hemisphere and

[ � J
. . Volume of one spherical lead shoe = i x n x r 3
it is placed into a right circular cylinder.
3 Height of the cylinder h = 1 80 cm = l .8 m
= - x - x - x - x - = - cm
4 22 1 1 1 1 1 3
-: I cm = m
3 7 2 2 2 21 1 O
Lee n b e the number o f lead shots dropped i n the vessel Radius of the cylinder,
then according to the question, . r = 60 cm = 0.6 m
n x Volume of one spherical lead shot :. Volume of water filled in a right
= .!_ x
Volume of water filled in a vessel circular cylinder
2
= 7t r h = -
7 x 0.6 x 0.6 x 1.8
4 22
n x - = -- x -
1 1 4400 1
:::::> E
21 21 4 14.256 (.)

n x I 1 = 1 1 00 :::::> n = 1 00
= -- In 0
:::::> 7 co

Hence, required n u m b er of lead shot is 1 00. For conicalportion


6 A solid iron pole consis ts of a cylinder of Height, '1i = 1 20 cm = 1 .2 m
height 220 cm and base d i a m e te r 24 cm, Radius, 'i = 60 cm =0.6 m
For hemispherical portion,
which i s s u r m o u n t e d by a n o t h e r cylinde r
of h e ight 60 c m a n d ra d i u s 8 c m . Fi n d the Radius, 'i = 60 cm =0.6 m _ _ _ _ _ _ .,..

mass o f the pole , g iv e n that l cm 3 o f i ron


. . Volum of th solid
of ch h mi ph r
has approximately 8 g mass. [take, n = 3 1 4] = Volum of ch cone + Volum
= 3 x 1t'i 2hi + -2 mi
1
S o l. Here, the given solid is a combination of rwo cylinde rs.
2 22 (0. ) 3
(0 .6 )2 ( )+-
Given, height of the first cylinder, 22
1.
1
h, = 220 cm =- x - x 7
7
1 342 , AlJ.inOne MATH E MATICS C l a s s 1 0th Te r m t

= -
22
x (0.6)- [1 . 2 + 2 x 0.6 ] =
22
x 0.36 ( 1 . 2 + 1 . 2)
-
Sol. ot corr ct. i n, ph rical la c cl i a 2 cm
21 21 com bination f a ph r a i t ba
22 1 9 .008 6.336 cyl inder a it neck.
For cylindrical portion,
= x 0 . 36 x 2 .4 = = m
21 21 7
Now, on p utting the sol i d i n to cyl i nder, the volume of water H igh t f the cyl i nder h1 = cm
flow out from cyl i nder w i l l be equal to volume of solid. Radiu of the cyl i nd r
2
'i
Vol u me of water left in the cyl i nder = -
=1 m
= Vo l u m e o f water fi l led in a righ t circular cyl inder 2
-Volume of the solid For spherical portion,
1 . 131 m 3
1 4 .2 5 6 6 . 336 7.92
= 1 . 1 3 1 4 29 m 3
_ Radius of th ph r
= = �
7 7 7 8.5
r = - cm
2 2
N ote We know t h a t . when a solid i s placed i n the righ t circular :. Volume of war r filled in a phcri al gla
cyl i nder full o f water. then it flow out the water from
= Volume of t he cyl i nder + l u me
cyl i nder equal to i ts total volume. That means. if we need to
= 7t lj h. + - 1t 1i3
4
fi n d the v o l u m e of water l e ft in the cyl i nder. then we
subt ract the v o l u m e of t h e s o l i d . from volume of the 3
1
= 3.1 4 x 1 x 1 x 8 + - x . 4 x - x - x -
cyl i nder filled w i t h water. 4 8.5 8.5 8.5

8 A spherical glass vess el has a cylindrical neck 3 2 2 2


1 92 8 . 3 5 2 5
8 cm long, 2 cm in diameter; the diameter of the = 25. 1 2 +
spherical part is 8 . 5 cm . By measuring the amount 6
of water it hold s , a child finds its volume to b e 25 . 1 2 + 32 1 . 3 9
=

= 346. 5 1
3 4 5 cm 3 Check whether she is correct, taking the

above as the inside measurements and n = 3.1 4 . So, che correct answer i s 346. 5 l c m 3 •

TO P I C EXE RCISE ·
Very S h o rt Answer Type Questions Short Answer Type II Questio ns
1 T h e b a se radi i o f t w o r i g h t c i rcular cones o f the 6 From a circular cylinder o f dia m eter 1 0 c m and
height 12 cm , a conical cavity of the same base
sa m e h e i g h t a re in the ratio 3 : 5 , t hen find the ratio
radius and of the sa me height is hol lowed out. Find
of t h e i r v o l u m es .
the volume of the rem a i n i n g sol id . [take, 7t = 3.1
2 A sol id ba l l i s exactly fitted i n side the cubica l box of
7 A metal contai ner is in the form of a cylinder
side a. What is t h e volu me of the ball? surmou nted by a hem isphere of the same radius.
The internal height of the cylinder is 7 m and the
Short Answer Type I Questions interna l radius of the cyli nder is 3.5 m . Calcu late t h

3 I f t h e rad i u s o f a sphere i s r c m . It i s divided into two


total surface area of the contai ner.

equal parts. What w i l l be the total su rface area of 8 A vessel is in the form of a h e m isphere bowl

[
CCE 20 1 2
two parts? mounted by a hollow cylinder. The dia meter of the

J
hem isphere is 16 c m and the tota l of height of the
4 What w i l l be t h e approx i mate volume of the largest vessel is 15 cm. Find the capacity of the vessel.
rig h t c i rcular cone that can be cut out from a cube of " 22
take, =
ed ge 4 . 2 c m ? CCE 20 1 2, 1 1 7

A n empty cyl i ndrical contai ner of radius 7 m and


5 T h e ratio of th e volu mes of two spheres is 8 : 27. I f
9
height 10 m is covered by a conical cap of radiu s
r a n d R a r e ra d i i of two spheres respectivel y, then
10 . 5 m and height 9 m . Calculate the volume of th
f i n d ( R - r ) : r. CBSE 20 1 2
air trapped insider. .
Surface Areas a n d Volumes /343 I
Long Answer Typ e Q u estions 12 A building is in the form of a cylinder surmounted

10 A tent is in the shape of a right circular cylinder


by a hemispherical vaulted dome and contains
19
upto a height of 3 m a nd conical above it. The total 41
21
m of air. If the internal diameter of dome is
height of the tent is 13.5 m above the ground.
equal to its total height above the floor. Find the
Calculate the cost of pa inting the inner side of the
tent at the rate of f 2 per m 2 , if the radius of the base
height of the building. NCERT Exemplar
is 14 c m . CCE 20 1 2, 1 1 13 A solid is in the form of cone mounted on a
hemisphere in such a way that the centre of the base
11 A pen sta nd made of wood is in the shape of a cuboid of the cone just coincide with the centre of the base
of the cone is _! r, where r is the radius of the
with fou r con ica l depressions and a cubical
depression to hold the pens and pins. respectively. 2
The dimensions of cuboid are 10 cm, 5 m and 4 cm. hemisphere. Prove that the total surface area of the
solid is � ( 11r + 21) r sq unit.
The radius of each of the conical depressions is
0 . 5 cm and the depth is 2.1 cm. The edge of the 4 CCE 201 3
cubica l depression is 3 cm. Find the volume of the
wood in the entire stand. NCERT- Exemplar

Answers
1 . 9 : 25 2. .!. 7ta :i 3. 6nr2 4. 19.4 5. 1 : 2
6
6. 628 7. 384.85 8. 2480. 7619 9. 577.5 1 0. � 2068

1 1 . 1 70.8 1 2. 4 m

TOPIC 8 Co nversion of Solid from One Shape to Another

Sometimes, we need t o convert solid o f one shape to Method of solving problems based on conversion of solid
another. When we come across objects which are converted from one shape to another can be understood with the help
from one shape to another or when a liquid which originally of following examples.
filled in one con tainer of a particular shape is poured into
Example 1. A copper rod of diameter 1 cm and
another container of a different shape or size, the volume
length 8 cm is drawn in to a wire of length 18 m of
remams same.
uniform thickness. Find the thickness of the wire.

cylHere·indri, acal shape. shape is converted into


a '
� AM
e.g. Solid metallic sphere is melted and recast into more than
one spherical balls or recast into a wire of cylindrical shape,
Sol. rod of cylindrical
, l�ll
the earth taken out by digging a well and spreading it
For rod.
uniformly around the well to form an embankment taking
the shape of a cylindrical shell from its original shape of right
Diameter = 1 cm

the rod = 11r,'h = 11x ( � ) x 8 = 2 7!


circular cylinder, etc. => Radius, = _!._ cm
r1
2
Important Results or Formulae and length, h = 8 cm
(i) If solid (or solids) of one shape is convened into solid
:. Volume of ' cm -'

(or sol ids) of another shape. then


Total volume of the solids co be converted = Total For wire,
volume of the sol ids i n to wh ich che g iven solids is Length = 18 m =1800 cm
(are) to be converted
(ii) N umber of solids of a g iven shape in which a g iven
Lee 1· be the radius (in cm) of
c ross-secrion of the
w i re , then
om
V l u e of wire = ,. � 1 8 00 cm -'
x 7t X
sol id i s to be converted
Total volume of the solid co be convened
c n t d wifre,rod
Since, the rod i s o ver e inco so

Volume of one convened sol id Volume ofwire == Vol ume o


\ 344\ Allinone MATH EMATICS Class 1 0th Term 11

7t x r 2 x 1800 = 2 7t => r 2 = -=>


1 r = -1 cm 1tr,-21 = 34 7tr.,
" -
.l

1 )2
7t x ( -
900 30
Then, diameter=-30 =-cm 2 1
15 n(0. 1) -, x h = 34 x 7t x (3 ) ·'
-

Hence, the diameter of the cross-section, i.e. the thickness of x h = 4 x 7t x 27


-

the wire is_!_15 cm, i.e. 0.067 cm (approx.) . 10


1
3
Example 2. A well of diameter 1 0 m is dug 1 4 m· 100 x h = 36 7t
7t x -
deep. The Earth taken out of it is spread evenly all h = 367t x7t 1 OO 3600 cm= 36 m [·: 1 m= l OO cm)
around to a width of 5 m to form an embankment. Find
the height of embankmen t. Hence, che length of the wire is 36 m.
Sol. Here, a well is dug and Earth taken out of it is used to form an
embankment. Ex ample 4. Water is flowing at the rate of 5 km/h

. Radius = = S m ll!lllllilll�
1
through a pipe of diameter 1 4 cm into a rectangular tank
Given,
Diameter of well = 10 m which is 50 m long and 4 4 m wide. Determine the time
in which the level of the water in the tank will rise by
:
10 7 cm.

�4 m taken
and Depthof =Earth Suppose, the level of the water in che rank wil rise by in 7cm

I
Sol.
E x hour.
Volume �
Silength
nce, theof thewaterwateris flowi
out on digging the well
= nr2h = 227 x (5)2 x 14 flow ningxat theSxrace of=55000x
h = km
km/h. Therefore,
m

[·: 1 km = 1000 m)
= 1 100 m 3 1;4 cm ('

,.·���
.,.���
,______
The embankment is in the form of cylindrical shell, so area of ______

embankment
.l*f=71
�fa
(R 5 )
7t 2 - r 2 ) = n(l0 2 2 = n (l00 -25) = x 75m 2
= -
22
7
Since, Earth taken out
:. Volume of embankment from wel l i s used to form embankment. c
7 m J 50 m
m
=Volume ofEarth
=> Area of embankment taken out on digging the well
x Height of embankment We have, diameter of cylindrical pipe14 = 14 cm

x h = 7tr2h = 227 x ( -7 ) x 5000x = 77x m


=Volume
Vol u me ofofEarth
Earth dugout
dugout . . Radius of cylindrical pipe, = 2 = 7 cm = _}___
r
100 m

=> He1"ght of embankmenr = ---- Areal l of the--- ----


embankment Volume of the water flowi2ng through che cylindrical pipe in
= 22 OO = 4.67 m -
100
3

7 x 75 Also, volume of the water char falls into7 the tank in x h


[ I= 50 m, = 44 m and h =radius=_!___1 00 m ]
Examp le 3. The diameter of a metallic sphere is 6 cm . h = 50 x 44 x -100 = 154 m
=Ix bx 3
It is melted and drawn into a wire having diameter of the
cross-section as 0 . 2 cm. Find the length of the wire. ·: b
Sol. Here, metall ic sphere is converted into cylindrical wire.
For metallic sphere,
Diameter = cm 6
·: Volume of the water flowing through the cylindrical pipe in
x h =Volume of water that falls in the rank in x h
Radius, = �2 =3 cm
'i => 77x =154
wire, =>· x=2
For cylindrical
D iam eter of cross-section =0.20.2 cm Hence, rhe level of water in the tank will rise by 7 cm in 2 h.
. Example 5. A hemispherical tank full of water is
Radius of cross-section, r2 = -2 = 0.1 cm emptied by a pipe at the rate of 3 i L /s . How much
wire b e
Let thetedlength rhedri cal h ofcmle. ngth
ofcylin Since,h cm.
metallic sphere is �
[
7
conver into a wire time will it take to emp half the tank, if it is 3 m in
:. Volume ofVolu
:::::>
metalmeusedof cylind w ire ==
in er == Volume
Vo sphere
lume of
of sphere diameter? rake, " = 2 2- J
7
! �
Surface Areas and Volumes 13 45 �

()
For one cone,
Sol. Given, radius o f che hem ispherical cank, r = � m
2 Radius = 3.5 cm = ?_ cm and height = 3 cm

( )
. . Volume of che hemispherical cank 2
� 1tr. \ = � x
22 x � 3 = 99 m 3 Volume of one cone = .!.. 1tr2h = .!.. 7t x ?_ x ?_ x 3
= 3 3 2 2
= �n cm
. 3 3 7 2 14 4
Then, volume of c h e water c o b e emptied '

= _!_ x
99 m -' = 99 x I 000 L = 99000 L
2 14 28 28 .
Required num ber o f cones =
Volume of the sphere
[·: I m 3 = 1000 L] Volume of one cone
4 25 . .
3087rt x __±_ = 126
· given, 3 - L = - L o f water 1s empned 1 s.
. =
.
m
Agam
7 •7 2 49 7t

[-:
99000 . . 99000 7 Hence, from che given metallic sphere, 126 smaller cones
-- L o f waccr w1' I I b c empned m -- x -
:.
28 28 25 s, can be obtained.

i.e. in 990 s or 1 6.5 min. 1 s = 1 min J TRY YOURSELF


60
Example 6. A metallic sphere of radius 10.5 cm is Q. l A hollow sphere of external and internal diameters
melted and then recast in to smaller cones, each of 1 2 cm and 6 cm respectively is melted into a cone of
radius 3 .5 cm and h eight 3 cm. How many cones base diameter 8 cm. Find the height of the cone.
[Ans. 47.25 cm]
are obtained?
Sol. Here, a sphere is convened i nto smaller cones. Q .2 A cone of height 24 cm and radius of base 6 cm is
made up of modelling clay. A child reshapes it in the
For sphere, form of a sphere. Find the radius of the sphere.
Radius = 1 0.5 cm =- � cm [Ans. 6 cm]
2 Q.3 A hemispherical tank of radius 1 . 75 m is full of water. It
. . Volume of the sphere = - rt r 3
4 is connected with a pipe which empties it at the rate of
3 7 Us . How much time will it take to empty the tank

(�)
= -4 rt x -
21 x -
21 x -
21 completely? [Ans. 26.73 min]

3 2 2 2 Q.4 Water in a canal, 30 dm wide and 1 2 dm deep is flowing


with velocity of 1 O km/h. How much area will it irrigate in
= 30 n cm ' 30 mi n , if 8 cm standing water is required for irrigation?
[Ans. 225000 m 1 )

NCERT FO L D E R 9.3 '.


" I
' ... ' ·, ' t -�
.. "'-•
I • , • • f i1 •
'I ' , r, '
• ' ,; '
'
'•
'
'

1t 22
Unless stated otherwise, take =
.
7
1 A metallic sphere of radius 4 . 2 cm is melted and Let h be che height of the cylinder, chen
recast into the shape of a cylinder of radius Volume of cylinder = 7t 1j2 h
6 cm. Find the height of the cylinder. 22 x 6 x 6 x h
=-

=-
Here, a metallic sphere is converted into a cylinder. So, 7

I
volume of sphere will be equal to the volume of cylinder. Now, according co che question,
Sol. Given, che radius of che sphere (r) = 4.2 cm

=
Volume of the sphere = Volume of the cylinder

and radius of the cylinder = 6 cm (1j) =::) 310.464 22 x 6x6x h \.-

Then, volume of the sphere = - 1tr 3


4 7

3 3 1 0.464 x 7
h
4 22 . 22 x 6 x 6
= - X - x 4.2·x 4.2x 4.2
3 7 � == 2.744 cm =
= 4 x 22 x 1 .4x 0.6x 4.2 79 2
= 3 10 . 464 cm 3 Hence, che height o f the cylin der is 2 . 744 cm.
� 346 �
' I
AllinOne MATHEMATICS Class 1 0th Term 11

-
2 Metallic spheres of radii 6 cm, 8 cm and 1 0 cm => rtr.,!.h1
= I x b x h!.
respectively, are melted to form a single solid 22 x -7 x 7 x 20 = 22 x 1 4 x h,
:::::) -
sphere. Find the radius of the resulting sphere. 7 2 2 -
Here, three metallic spheres are melted to form a single :::::)
11x 7
-- x 20 = 22 x 1 4 x h,-
solid sphere, so volume of single solid sphere will be 2
1 I x 7 x 20 � l
h-, =
2 x 22 x 1 4 2 = .5 m
equal to the sum of volumes of three metallic spheres. =

Let 1j , r.? , r3 be the radius of given chree metallic spheres and


Hence, che height of chc plarform ( h!. ) is 2. 5 m.
Sol.
R be che radius of a single solid sphere.
Then, 1j = 6 cm, r.? = 8 cm and r3 = 10 cm 4 A well o f diameter 3 m i s d ug 1 4 m deep. The
Volume of first metallic sphere CV.) Earth taken out of it has been spread evenly all

= -34 7t I 3 = -34 1t(6) = -1t


864 cm 3
J
around it in the shape o f a circular ring o f width
r.
4 m to form an embankment. Find the height of
3
the embankment.
Volume of second metallic sphere (V.?)
2048 1t cm 3 Sol. Given, che height of deep
= -4 1tr.,3 = -34 7t (8)3 = -- well which is in che form Embankment
3 - 3 of a cylinder, (h1 ) = 14 m
and volume of third metallic sphere (\'3 )

Lee
and radius of deep well, �
Well
= i 1trj' = i 7t(l 0 ) 3 = 4000 7t cm
3
3 ( )=3 m 3m
3 3 3 1j 1 J
Now, volume of a single solid sphere (V) = i 1t R 3 che width and height
of che embankment which is in chc form of a circular ring be
d and h2 , respectively.
3
According co che question, Here, d = 4 cm [given]

-------
Volume of three metallic sphere Now, che volume of deep well (cylinder)
=Volume of a single solid sphere 22 x -3 x -3 x 14 = 99 m '
:::::. V. + � + it; = V = 7t1j 2h, = - ·
7 2 2
:::::. 864 + 2048 1t + 4000 7t = i 1tR 3
7t
Here, we observe chat che::

7m
3 3 3 3
691 2 = j_R3 embankment form a hollow � '
cylinder. So, for finding chc � 20 m

Lee
volume of embankment, we
1
3 3 ,_,
3
R = 1728 = (12)3 apply che concept of hollow Platform
:::::> R = 1 2 cm cylinder. Well
Hence, the radius of resulting sphere is 12 cm. the height of the
embankment be hl'
3 A 2 0 m deep well with diameter 7 m is dug and :. Volume of hollow cylinder (embankment)
= 7t(1j + d)2 h2 - 1t1j 2 h2

{% + 4 J x h2 -{%) h, = n[(1;)\-H
the Earth from digging is evenly spread out to •

form a platform 22 m x 1 4 m. Find the height of


the platform.
[here, radius of outer circle = 'i + J]

\ Here,

1
the Earth obtained from digging the well of
cylindrical shape is used to make a platform of cuboidal
= ' m'

j
shape. So, the volume of Earth will be equal to the Now, according co che question,

[( J -
volume of cylindrical well and it will be equal to volume of
Volume of deep well (cylinder)

]
! cuboidal platform.
Given, che height of deep well which is in the form of a = Volume of embankment (hollow cylinder)
99 = n � x h2 � h,
Sol.

- [·: 2;]
cylinder (h1 ) = 20 m =)
. ·.Radius of deep well, 1j = ?... m
2 x 99 = � x h 2. h
7
=>
Le n g rh of che platform, I = 22 m 2 4 2 n=
22 4
--
7 x 99 = --
an d brea d th of che platfo rm, b = 14 m

l.
I 12h2
Lee h e i gh r of rh e plat form = h2 =>
Now, according to rhe question, 22 4
;�

_,1:
d
;· ;
Volume of deep well (cylinder) 7
h2 = 99 x 4 = 2. = l 25 m
x
�. ! 22 x 1 1 2 8
�I
/

...
ii
;
= Volume of platform (cuboid)
Hence, che height of the embankment is 1 . 125 m.
Surface Areas a n d Volu mes / 3 47
5 A container shaped like a right circular cylinder Sol. We know char, every coin has a shape of cylinder.
having diameter 1 2 cm and height 15 cm is full
of ice-cream . The ice-cream is to be filled into
cones of height 1 2 cm and diameter 6 cm,
having a hemispherical shape on the top. Find
the number of such cones which can be filled
with ice-cre a m . Cuboid
Sol. Le e che hcighc and radius of cylindrical ice-cream container Given, diamecer of cylindrical coin = 1 .75 cm

be h1 and 1j , respectively.

[ J
Then, radius (1j ) of cylindrical coin = 1 · 75 cm
2
-- - - - - - - - - -
6 cm
----- and heighc ( h1 ) of cylindrical coin = 2 mm = 0.2 cm
E
(.)
-: 1 mm = �
1
cm

� 1 2 cm
:. Volume of che cylindrical coin = TC1j 2h
3
----
6 cm = 22 x 1 .75 x 1 .75 x 0 _ 2 = 1 3.475 cm
---------- 7 2 2 28
Container (cylinder) Also, given chac che lengch, breadch and heighc of che cuboid
are 5.5 cm, 10 cm and 3.5 cm, respectively.
Given, h, = 1 5 cm and 1j = _!2 = 6 cm :. Volume of che cuboid = Lengch x Breadth x Heighc
2
:. Volume of cylindrical ice-cream conrr.mer = 5.5 x 1 0 x 3.5 = 1 92.5 cm 3

= rc ,-. -h,
, 22
=- 1 1 880
x 6 x 6 x 1 5 = -- cm 3
Lee n be che number of coins for melting to make the
7 7 cuboid, chen
Now, che cone full of ice-cream having cop surface jusc like a n x Volume of one coin = Volume of the cuboid
hemisphere. nx
1 3.475
-- = 1 92 .5 => n = 1 92.5 x 28 = 400
Also, height of the cone, h2 = 1 2 cm
=>
28 1 3.475
Diamecer of chc cone = 6 cm Hence, 400 silver coins muse be melced to form a cuboid.
:. Radius of che cone, r2 = 3 cm = Radius of che hemisphere 7 A cylindrical bucket, 32 cm high and with
Volume of cone full of ice-cream radius of base 18 cm, is filled with sand. This
= Volume of cone + Volume of hemisphere bucket is emptied on the ground and a conical
(upper surface level)
= - rc1;-2 h-, + - rcr2 = rc(r,21. - + 2r, )
heap of sand is formed. If the height of the
1 2 J 1 ·'
- J, - conical heap is 24 cm, then find the radius and
3 3 3 -
slant height of the heap.
1 22 ,
= - x - [(3)- x 1 2 + 2 (3 )3 ] • Sol. Here, che sand 18 cm
3 7
22 ,
filled in
= -- x (3)- x (1 2 + 6) cylindrical
3x 7 bucket form a
= = -- cm J
22 x 3 x 1 8 1 1 88 conical heap of
7 7 sand on the
Lee be number of cones full of ice-cream, chen
n
ground. So,
Volume of ice-cream container volume of
= n x Volume of one cone full wich ice-cream cylindrical
buckec will be ..... _ _ _ _ _ _ .,,

;.' ""',-1>1

=>
1 1 880
--= n x 1 1 88 => 11 = 1 0
-- equal co the Cylinder Heap of sand ':.s.: �

uJ
7 7 volume of conical heap. Q
Hence, che required number of cones is 1 O. Lee the radius and slanc heighc of the heap of sand be �
NBto total n umber of cones will con tain all ice-cream of r and /, respectively. '�"'"'·
container. Thac means, total volume of cyl indrical · Given, the height of the heap of sand, h = 24 cm

ice-cream con tai ner fi ll number of cones full of


ice-cream .
:. Volume of the heap 'of sand = .!_ rcr·\ 24) 8 Ttr:. =

. 3
6 How man'y silver coins, 1 . 75 c m in Also, given the heighc and rad i us of cylind rical buckec are
32 cm and 1 8 cm, res pectively.
diameter and of thickness 2 mm, must be
Volume of cyli nd rical bucker
32
= rr(Radi us):. (He ighc) = rr (l sf
melted to form a cuboid of dimensions
x
x
5 . 5 cm x 1 O cm x 3 .5 cm?
3 48 Allin.One MATH EMATICS Class 1 0th Term 11

Now, accord ing to the question,


Volume of the heap of sand = Volume of the cyl indrical bucket
Hence, the required area for covering 8 cm or �
1 00
m height of
=> 8 rtr.! = rt x 1 8 x 1 8 x 32 => r! = 1 8 x 1 8 x 4 standing water
. . r = 36 cm [ raking positive square roocs] = 45ooo x 1 00 = 56250 0 m .? =
562500
hec
Now, slant height of the conical heap of sand, 8 1 0000
I = �h! + r! = �( 24 ).! + (36)2 = �576 + 1 296 = 56.25 hec [·: I hec = 1 0000 m 2]
= .J1 872 = .J1 44 x 1 3 = 1 2 .J13 cm
9 A farmer connects a pipe of internal diameter
20 cm from a canal into a cylindrical tank in his
H ence, the rad ius and slant height of heap are 36 cm and
field, which is 1 0 m in diameter and 2 m deep. If
1 2 .Jl3 cm, respectively. water flows through the pipe at the rate of 3 , km/h
8 Water in a canal, 6 m wide and 1 . 5 m deep is in how much time will the ta nk be filled?
flowing with a speed of 1 0 km/h. How much area Sol. Given, speed of flow of wacer
lOOOm]

[ 2; cm = l O m ]
will it irrigate in 3 0 min, if 8 cm of standing water = 3 km / h = 3 x 1 000 m / h [·: 1 km =
is needed? : . Length o f water in I h = 3000 m
Now, area of the pipe which is in the form of a ci rcle

[·: m]
Sol. Give n , speed o f flow of water = 1 0 km I h
= 1 0 x 1 000 m/h [·: I km = 1 000 m ] = (I O ) ' lt ·: area of ci rcle = nr' and r = c


=> Length of water i n I h (/) = 1 0 x 1 000 m
Area of rectangular canal = 6 x 1 . 5 9 m 2 = = 1 00 it cm 2 = 1 �0 m 2 1 cm =
---1 . 5 m --- 1 0

For cylindrical tank,

( )
Diameter = 1 0 m
= �2 m and height = 2 m , then
Canal
:. Radius

Volume of cylindrical cank = itx 1; ' x 2 [ ·: V = nr'h]


Volume o f water flowing in 1 h
= 5 0 7t m .\
Volume of cyl indrical cank
= Area of canal x Speed of flow of water . . Required time
=
9 x 1 0 x 1 000 m .\ Area of the pipe x Length of water

Volume of water flowi ng i n 1 /2 h


1t
50rt 50 x 60 x 1 00
3000
min = lOO min
1 0 x l OOO x 9 - x 3000
= 4 5000 m 3 1 00
2 Hence, in 1 00 min, the tank will be filled.

TO P I C EXERCISE
Very Short Answer Type Qu estions Short Answer Type I Question s
1 F i nd the number of cubes of side 2 cm which can be 4 Eight solid spheres of the same size are made by
cut of side 6 c m . CCE 2 0 1 1 melting a solid meta llic cylinder of base diameter

2 Th ree m eta l l ic solid cubes w hose edges are 12 cm,


6 cm and height 32 cm. Then, find the diameter of
CCE 20 1 2
16 a n d 20 c m r espectively, a re melted and
each sphere.
c rn

for m ed i n t o a si ngle cube. Find the edge of the cube 5· How many spherical lead shots each 4.2 c m in,
so forrnr..;cJ .
: di�meter can be obtai ned from a rectangu lar solid of

e 1 cm is d rawn into a wire of lead with dimensions 66 cm, 42 cm and 21 cm?


3 If a m eta l l ic cu b f · n f eclg
NCERT Exemplar
d ia m ete r 4 m m . t h e n f i n d the l engt h of the wire.
CCE 2 0 1 1
Surface Areas a n d Volumes f 349 I
Short Answer Type I I Q uestions Long Answer Type Questions
6 A hemispherical bowl of internal diameter 3 6 cm 9 Water flows through a circular pipe whose internal
contains liquid. This liquid is filled into cylindrical diameter is 2 cm at the rate of 0.7 mis into a cylindrical
shaped bottles of radius 3 cm and height 6 cm. tank, the radius of whose base is 40 cm. By how much
How many bottles are required to empty the will the level of water rise in the tank in half an hour?
bowl? CCE 2008

7 Two cones with sa m e base radius 8 cm and height


10 A sphere of diameter 12 cm is dropped into a right
15 cm are joi ned together along their bases. Find circular cylindrical vessel partly filled with water. If

%
the surface a rea of the shape so formed. the sphere is completely submerged in water, then the
NCERT Exemplar water level in the vessel rises by 3 cm. Find the
8 A wall 24 m long, 0.4 m thick and 6 m high is diameter of the cylindrical vessel. CCE 201 2
constructed with the bricks each of dimensions
25 cm x 16 cm x 10 cm. If mortar occupies _±_th of
11 A well whose diameter is 7 m, has been dug 22.S m
10 deep and the Earth dugout is used to form an
the volume of wall. then find the number of embankment around it. If the height of the
bricks u sed in constructing the wall. embankment is 1.5 m, then find the width of the
NCERT Exemplar embankment.

Answers
25
1t
1 . 27 2. 24 cm 3. - cm 4. 6 cm 5. 1500

6. 72 7. 855 cm 2 (approx.) 8. 12960 9. 78. 75 cm


1 0. 1 8 cm 1 1 . 1 0.5

TO PI C 8 Frustu m of a Cone
If a right circular cone is cut-off by a plane parallel to its Slant height of frustum The slant height of a frustum of a
base, then the portion of the cone between the cutting right circular cone is the length of the line segment which is
plane and the base of the cone is called a frustum of the obtained by joining the end points of two parallel radii, drawn
cone. In the given figure, ABC is a right circular cone in the same direction of the two circular bases.
which is cut by a plane parallel to its circular base with From the given figure, Slant height of the frustum ABB' A'
centre 0 and diameter AB.
= AA ' = CA - CA' or BB' = CB - CB'
Remove the portion which contains vertex C. The left out
= Difference between the slant heights of the cone
portion ABB' A' is called frustum of the cone ABC.

w
L n
CAB and CA' B'

c_
Formu lae Related to Frustum of a Cone
Let h be the height, l be the slant height and r, and r2 be the radii of
A' - --� a · - ·
_ _

0'
a· - · o 8'
·
0'
>
the circular ends (r, r2 ) of the frustum of a cone. Then,

- -------- -
1
--- --.---- - - --- --- - - -
The curved (lateral) surface area of the frustum of the cone
=
7t (r, + '2 ) l ,
A O
B A O B A
O
B where l = �h + (r, - r2 )2
2
Thus, a frustum of a right circular cone has two unequal
2 Total surface area of the fru stum of the c one
flat circular bases and a curved surface. Height of frustum = + r.,) + rr.r.2 nr;.
-
+
1
7t / (r, �

�h2
The height (or thickness) of a frustum is the
where l= + (r, 12 >'
perpendicular distance between its two circular bases.
From the above figure, Height = 00' = CO - CO' 3 Volume of the frustum of the cone
= Difference between the heights of the cone
= ..:!. rrh ('12 + '22 + r,r2)
CAB and CA' B' 3
350 Allinone MATHEMATICS Class 10th Term 11

Area of the M etal Sheet Used to Example 2. A bucket of heigh t 16 cm and made up of
M a ke a Bucket metal sheet is in the form of frustum of a right circular
cone with radii of its lower and upper ends as 3 cm and
Let h be the height or depth, l be the slant height, r. be the
15 c m , respectively. Calculate
radi us of top which is open and r2 be the radiu of the � (i) the height of the cone of which the bucket is a part.
bottom which is closed of a bucket which is in the shape of a
frustum of a right circular hollow cone, then (ii) the volume of water which can be filled in
the bucket.

Top
(iii) the slant height of the bucket.
(iv) the area of the m etal sheet required to make
the bucket.
Sol. (1) Le e ABCD be che bucker which is in che form of frustum
of righc circular cone wich
a

vercex 0 (as shown in che


figure below). Lee ON = cm x

Bottom / r2

Area of the metal sheet used fo r making the bucket


Given, MN = 16 cm

Then, OM = (1 6 + x) cm
= Outer (or i nner) curved surface area + Area of . . By AA similarity,
bottom llONB .. llOMC o
= rr./ ( r1 + r1 ) + 1tr 2 [·: LONB = LOMC = 90°, LOBN = LOO.I

[·: �; ��]
2 = corresponding angles]
Also, capacity of the b ucket is equal to its volume,
x
-- = - 3
. I
1.e. - rr.h (r1 - + r1 r.,- + r.,-- ).
., ., l6 + x 1 5 =
-- = -1
x
3
=>
Example 1. If the radii of the circ ular en ds of a 16 + x 5
frustum of h eight 6 cm are 1 5 cm a nd 7 crn => 5x = 1 6 + x
respectively, then find the volume and lateral surface => 4x = 1 6
area (curved s urface area) of the fru stum . => x=4
[Take, 7t = 3.1 4]
:. ON = 4 cm and OM = 4 + 1 6 = 20 cm
Sol. Given, radii of both circular ends, Hence, the height of the cone is 20 cm.
'i = 1 5 cm, r2 = 7 cm and height, h = 6 cm
(i1) Volume of water which can be filled in the bucket
c = Volume of frustum of cone ABCD
= -1 rth (1j + r22 + 1j r2 )
i

3
= .!.. 7t x 16 [( I 5 ) 2 + (3 ) 2 + 1 5 x 3 ]
1 3
Vo lume of rhe frus tum ( V) = 3 7th (1j 2 + ri2 + 1ir2) [·: h = l6 cm, 1j = 1 5 cm and r2 =3 cm]
= 31 1t x 6 (22 5 + 49 + 1 0 5) = .!..3 7[ x 16[225 + 9 + 45]
= 7t x 2 (379 ) = .!.. rtx 16 x 279
3
= 3. 1 4 x 758 [·: 1t = 3.14] = 7C x 16 x 93 = 14887C cm 3
= 2380. 1 2 cm 3
Hence, che volume of water which can be filled in the
Now, sla n r h e ighr (f) == �h 2 + (1j - r2 ) 2 = ,J36 + 64 = IO cm bucket is I 488rt cm 3•
[·: 'i - r2 == 1 5 - 7 = 8 cm ] (iii) Slane height of the bucker
LareraJ surface area (5) = nl (1j + r2 ) = rt x I O x (1 5 + 7 )
=Slant height of frustum of a cone ABCD
rr x I 0 x 22 = 3. 1 4 x 220
=
= Jhi + (1j - r2 ) 2 = �(16)2 + (1 5 -3) 2
[·: h = 1 6 cm, 1j = 1 5 cm and r2 =3cm]
== 69 0 . 8 cm 2
Surface Areas and Volu mes
/ 351 /
= J1s6 + o 2)1 = .J2s6 + 1 44 Example 4. Hanumappa and
his wife Gan gamma are busy
= .J4oO = 20 cm making jaggery out of sugarcane
Hence, rhc slanr hcighr of rhe bucker is 20 cm. juice. They have processed the
(iv) Area of rhc metal sheer required ro make rhc bucket
= re/( 1j + ,� ) + rer/ = re x 20 x (1 5 + 3) + re x (3) 2 sugarcane juice to make the
molasses, which is poured into moulds in the shape of
[ · : I = 20 cm, 'i = 1 5 cm and r2 = 3 cm] a frustum of a cone having the diameters of its two
= 360re + 9re = 369re cm 1
circular face as 30 cm, 35 cm and the vertical height
of the mould is 14 cm (see the figure) . If 1 cm 3 of

[
Example 3. The h eigh t of a cone is
molasses has mass about 1 . 2 g, then find the mass of
30 cm. A small

J
cone is cut-off at the top by a plane parallel to base. If
the molasses that can be poured into each mould.
its volume is _!_
7 of the volume of given cone, then at it = 2 2
2 Take, 7
what height above the base is the section made?
CCE 201 1 Sol. Given, mould is in the shape of a frustum of a cone, in which
Sol. LccVAB be a cone of height, VO =30 cm and base radiusrcm r2 = - 35 cm and h = l4 cm.
30 cm, lj = -
Suppose, it is cur-off by a plane parallel co the base at a height h 2 2

2
from the base of the cone. Then, we get a small cone VA' B'. Now, quantity (volume) of molasses char be poured into it
can

Let its radius = 'i and height = h1 =Volume of the frustum

: [( n ( J ( ; nJ
v = -re h (1j + r2 + 'i'2 )
2

3
+ 2 x 14 3 ' + 3; ' + 3 x 3
= 443 [306.25 + 225 + 262.5]
= 443 x 793.75
= 1 1641.7 cm 3
Clearly, � VOA .... � VO'A' Given, mass of 1 cm 3 of molasses = 1.2 g

[·: ]
[ ·: byAA similarity L VOA = LVO' A' =90°, LVAO = VA' O' :. Mass of the molasses chat can be poured into each mould
= Corresponding angles] = 1 1641 .7 x 1 . 2 = 13970.04 g
VO = OA 1
VO ' O'.A ' = 13. 97 kg = 14 kg (approx.) 1 g =-1000 kg
30 = -r
- [·: VO = 30] . . . (i)
h. 1j
=>

TRY YOU RSE LF

(�71)2
According co the question,

m· 2
Volume of cone VA'B' = -1 x Volume of cone VAB
27 Q. 1 A solid frustum is of height 8 cm . If the radii of its lower and

(!!J_)2
..!._ 1 ..!_ rer 2 x 30 => h1 = 19°
upper ends are 3 cm and 9 cm respectively, then find its
[Ans. 10 cm]
=>
3 h =- I
27 x 3
I slant height.

1O Q. 2 The diameters of the top and the bottom portions of a


30 h = I
9 [using Eq. (i)] bucket are 42 cm and 28 cm, respectively. If the height of
the bucket is 24 cm, then find the cost of paint ing i ts outer
h13 = 1000 => h1 = 10 cm surface at the rate of 50 paise per cm2 . [Ans. t J 683]
h = 30 - h1 [·: VO =30 = h + �] Q. 3 The radii of the ends of a bucket 30 cm hi gh are 21 cm and
= (30 - 10) = 20 cm 7 cm, respectively. Find its capaci ty (in litres) and the
Hence, the section is made at a height of 20 cm from the base of amount of sheet required to make this bucket.
[Ans. 20 . 0 2 L, 3067 .68 cm � l
cone.
#
n 22 .
Unless stated othervvise, take =
7

1 A drinking glass is in the shape of a frustum of a Sol. Lee the slant height of fez be I 4 cm
cone of h e i g ht 1 4 cm . The diameters of its two and the rad ius of u pper end
circular ends are 4 cm and 2 cm. Find the which is closed be 'i and che
capacity of the glass . �
ot er end which is open b 12 .
0
-2 cm -
Given, I = 1 5 cm, 1j = 4 cm
and 'i = 10 cm
Sol. Here, dri nki ng glass is in the shape
D _ _ _ e_ _ _ _ C

·. ·
of a frustum of a cone.
Hence, area of material used 1 5 cm

--r �
Given, height of the frustum, for making fez
h = 14 cm = Total curved surface area of

D ia meters of rw o circular ends the fez


are 4 cm and ·2 cm. 1 4 cm
= Lateral surface area of

__j- �:�- - frustum + Area of upper close end


Radius of two circular ends,

r2 =
4 2 = 7t (1j + r ) I + 7t1j 2 = rt[( 4 + 1 0) 1 5 + ( 4) ]
- cm and 1j = - cm 2
- - - - - - - - - - - - - -: B
_ _ _
2 2 A
::::::> r2 = 2 cm
and 1j = 1 cm p•
= 2 2 ( i I O + 1 6) = 2 2 x 226 = 4 9 72 = 7 1 0 3_ cm 2
-- 4 cm -- 7 7 7 7
:. Volume (capacity) of drinking

.!.
glass (frustum of a cone) 4 A container, opened from the top and made up
=3 1t h [ j 2 + r 2 + j
1 i 1 12 ] of a metal sheet, is in the form of a frustum of a
cone of height 1 6 cm with radii of its lower and
= x 22 x 1 4 ((1)2 + ( 2 ) 2 + (1 )( 2 )] upper ends as 8 cm and 20 cm, respectively.
3 7 Find the cost of the milk which can completely
44 08
= ( l + 4 + 2) = 44 x 7 = 3 = l 02 3_ cm 3 fill the container at the rate o f � 20 per L. Also,
3 3 3 3 find the cost of m etal sheet used to make the
container, if it costs � 8 per 1 00 cm 2
2 The slant height of � frustum of a cone is 4 cm

[Take, 7t = 3 . 1 4)
Given container is in the shape of frustum of a cone.
and the perimeters (circumference) of its
circular ends are 1 8 cm and 6 cm. Find the Sol.
Lee h be the height of rhe container, the radius of irs lower
curved surface area of the frustum.
end be 1j and upper end be r2 •
Given, h = 1 6 �m , 1j = 8 cm and r = 2 0 cm
Sol. Let the slam height of the frustum be l and radii of the both
ends of the frustum be 1j and r2 • 2
:. Volume of the container
G iven, l = 4 cm
=�x h (1j2 + ' 2 + 1j r2 )
Perimeter of one e n d of frustum which i s circular in shape, is 3 2
21t 1j = 1 8 :::::> 1j = 2. cm and perimeter of ocher end of = 3 .i 4 x 1 6 [(8)2 + (20)2 + 8 x 20 ]
1t
3

[·: I L]
frustum which is also circular in shape, is 2 nr = 6
2 = 3 .14 x 1 6 x (64 + 400 + 1 60) ----3 . 1 4 x 1 6 x 624 - cm
3

(; ;) x 4 = 1 2 x 4 = 48
::::::> r2 = - cm 3 3

7t
3 .1 4 x 1 6 x 624
Hence, required curved surface area of the frustum = L = l 0.45 L cm ' = I O
3 x 1 000 O
= " (r, + r, ) I = 1t +
2

2
cm · : Cose of I L milk = � 20
:. Cose of 1 0.45 L milk = 1 0.45 x 20 � � 209
3 A fez , th e ca p used by the Turks, is shaped like Now, slam height of the container ,
the fru stum of a cone (see the figure) . If its .. I= �hi + ( r2 - 'i )
radius o n th e open side is 1 0 cm, radius at the
upper base is 4 cm and its slant height is 15 cm, �(16)2 + (20 -8 )2 = ..j{t6)2 + (12 ) 2
=

then fin d the area of material used for = .J256 + 144


making it. = .J400 = 20 cm
Surface Areas and Volumes f353 '
In MOE,

[·: cot 60° = )3J


. . Arca of mcral shcce
OE - 1i

2
= Required surface area of fruscum of cone cot60 ° = -pB = -- = [·: 0 = 60° , given]
= Curved surface area of fruseum of cone AO I O
+ Arca of irs lower end (which is closed) J3
10 cm
1 = 1j => 1j = J3
=>
= n( 1i + r2 ) I + n 1i 10
= 3. 1 4 (8 + 20) x 20 + 3. 1 4 x (8) 2 and in MO' C,
= 3. 1 4 ( 28 x 20 + 64) coe60 ° = O'
--
C= 2
AO' 20
r.
-- [·: e = 60°' given]
= 3 . 1 4 x 624
= 1 959.36 cm 2 =>
1 = � => r = 20 cm
2
.J3

(
Cose of mceal sheer per 1 00 cm 2 = � 8 .J3 20
:. Volume of ehe firuscum = -7t h (1j 2 + r22 + 1j r2 )

J
. .Cose of metal sheer per 1959.36 cm 2 3
= 8 11959 x • 36
00 = 1 56.7
4 88 :::::: � 1 56.75
= �3 x I O 100 + 400 + 200 = 7000n cm 3
3 3 3 9
5 A metallic right circular cone 20 cm high and Now, this fruscum is converted into wire of cylindrical
a

whose vertical angle is 60°, is cut into two parts form.


at the middle of its height by a plane parallel to Lee ehe length of the wire be h.
its bas e . If the frustum so obtained be drawn Given, the diameter of wire = _!.._ cm
16

{ J
into a wire o f diameter __!__ cm, then find the
16 :. Radius of wire (r) = _!_ cm
32
and volume of wire (cylindrical shape)= 7tr 2h = � x h
length of the wire.
Sol. Lee 'i and r2 be rhe radii of
upper and lower ends, 3
respeceively of ehe fruscum According to the question,
cut by a plane. Volume of the wire =Volume of the frustum

[ J
Given, heighe of ehe cone =>
_!!!!.__ = 7000n => h = 7000 x 32 x 32 cm
= 20 cm 32 x 32 9 9
:. Heighe of ehe fruseum 7000 x 32 x 32 1_ m
= 1 0 cm => h= m ·: 1 cm = _100
9 x l00
For finding ehe radii of ehe · · h = 7964.44 m
ends of ehe fruscum, Hence, the length of the wire is 7964.44 m.

NCERT FO L D E R 9 . 5 (Opti ona l)


f! )it ' •· ' , • . \. �

[·: ]
I · . '
.· i� . .� ."..' , , .
J ; ,. ,,:
' - �· • • '
.
• .

I. " ' 1. ' , ·,

·. '
' .

1 A copper wire 3 mm in diameter is wound about :. Radius, r = lO2 = 5 cm r=1


a cylinder whose length is 12 cm and diameter
1 0 cm, so as to cover the curved surface of the Length of wire required to complete one round
cylinder. Find the length and mass of the wire, = Circumference of base of cylinder
assuming the density of copper to be = 2nr = 2n (5) = l O n cm
8.88 g per cm 3 •
Sol. Given diameter of the wire is 3 mm.
When a wire is one round wound about a cylinder, ehen it
covers a 3 mm of length of the cylinder.
Also .given, length of the cylinder 7 12 cm = 120 mm

. 120
. . N umbc:r of rounds to cover 120 mm = 3 = 40

Again given, diameecr of a cylinder, d = 10 cm


1 354 ;
' i
Allin. One MATH E MATICS Class 1 0th Term II

. . Length of the wire in covering the whole surface = 4 05 . 5 0 4 = 19_ 3 1


c m .1
= Length of the wire in completing 40 round 21
= 1 0 1t x 40 = 400 7t = 400 x 3. 14 = 1256 cm :.Required volume of double conL" = I 0.86 + 19.31

[": ] 2
N ow, d .ms o f copper wire
ra
. =- 3 mm = - 3 cm = 30. 17 cm 3
2 20 Now, surface area of cone A BD = rcr/
I mm = � cm

(; J
= 22 x 2. 4 x 3 = 1 SB . 4 = 22.63 cm

2
1 7 7
and surface area of cone A CD nrl =
. . Volume of wire = rr
0
( 400 rr) = 9rr' cm ·' = 22 x 2. 4 x 4 = 21 1 .2 = 30. 17 cm
7 7
:. Mass of the wire = Volume x Density :. Required surface area of double cone
= 9 7t2 x 8.88= 9 (3. 14)2 x 8.88 = 22.63 + 30. 17 = 5 2.8 cm �
= 88.74 x 8.88 = 788 g (approx.)
3 A cistern, measuring 150 cm
internally
2 A right triangle whose sides are 3 cm and 4 cm x 120 cm x 1 1 0 cm, has1 29 600 cm 3 of water in
(other than hypotenuse) is made to revolve it. Porous bricks are placed in the water until
about its hypotenuse. Find the volume and the cistern is full to the brim. Each brick
s urface area of the double cone so formed. absorbs one-seventeenth of its own volume of
[Choose the value of 7t as found appropriate] water. How many bricks can be put in without
Sol. Here, ABC is a right angled A overflowing the water, each brick being
triangle, right angled at A and BC 22.5 cm x 7 .5 cm x 6.5 cm ?
is the hypotenuse. Sol. Given, inrernally dimensions of cistern
Here, AB = 3 cm = 1 50 cm x 120 cm x 1 10 cm
and AC = 4 cm :. Volume of cisrcrn = 1 50 x 1 20 x 1 1 0 = 1 980000 cm 3
. . BC = �� 3 2_+_4_2 Given, volume of warer in cisrern = 129600 cm 3

Lee
[by Pythagoras theorem] D :. Remaining volume of cistern to be filled
= .J9 + 1 6 = .J2s = 5 cm = 1980000 - 1 29600 = 1 850400 cm 3
As MBC revolves about the hypotenuse BC. It forms two
cones ABD and ACD. required number of bricks be n.
By rule AA similarity criterion, MEB and fl. CAB are similar. :.Volume of each brick
= 22.5 x 7.5 x 6.5 = 1 096.875 cmJ
AE AB

( J
CA BC Water absorbed by 1 bnck . = 1 096.875 cm· \

LAEB L CAB = 90° and LABE = ABC


==
17
[· : in similar triangles, sides are proportional]
12 = 2.4 cm Then, water absorbed by n bnc. ks = 1 096.875
n cm J

- = 3 => AE = -
AE -
17
4 5 5

(
Now, Volume of remaining water + Water absorbed by

)
n

So, radius of the base of each cone = AE = 2.4 cm bricks


Now, in right angled MEB, =nx Volume of each brick
BE = �AB2 - AE2 [by Pythagoras theorem] => 1 850400 + n
1096.875 = n ( I 096.875)
17
=
�(3) 2 - q .4) 2 .J9-5.76 = ·b.24 = 1 .8 cm
.
==

So, height of che cone ABD = 1.8 cm => n x 1096.875 x 1 6 = 1 850400


:. Heighc of the cone ACD = CE = BC - BE 17
1 850400 x 1 7
n == 1096.875 x 1 6 = 1792.4 1 03 :::::: 1792 (approx)
= 5 - l .8 = 3.2 cm =>
Now, volum e of the cone ABD
Hence, 1792 bricks can be put in cistern.
= ..!_ 1tr 2 h = ..!_ x 22 x (2.4) 2 x 1 .8
...# 3 3 7 4 In one fortnight of a given month, there was a
L� 22 3
=- 2 1 x 1 0.368 = 10.86 cm
rainfall of 1.0 cm in a river va:lley. I f the area of
mm
.:.: If the valley is 97280 kin 2 , then show that the
and ol u e of rh e cone ACD
v m total rainfall was approximately equivalent to
1. -lf . 22
,, _

�I
'I
I �' = � nr 2h == 3..!_ x 7 x (2.4)2 x 3.2 the addition to the normal water of three
rivers each 1 07 2 km long, 7 5 m wide and 3 m
ti !'1 3
.:_,
�.
[ rad i us wi ll be same as AD is common] deep.
� Li
Surface Areas and Volumes 1 355 '
I

Given, area of the valley = 97280 km 2 = 22 [2 x 4 x 10 + (9 + 4) 13]

]
Sol.
7

[
and rate of rainfall = 10 cm = 100 x1 O1000 km
= 7 (80 + 1 69) = 227 x 249
22
· . · 1 cm =
1 ·1 km
100 m and 1 m = 1000 = 5478
7
= 782 i7 cm 2
:.Volume of rainfall = Arca of valley x Race of rainfall
6 Derive the formula for the curved surface area
= 97280 x 10 = 9.728 km J (i)
• • •
and total surface area of the frustum of a cone
100 x 1000 (using the symbols as explained) .
Also, given length of chc river = 1072 km Sol. Let h be the height, I be the slant height and, 1j and 72 be rhe
Breadth of the river = 75 m = _Z2._ radii of the bases (1j > 72 ) of the frustum of a cone. We
1000 km complete rhe conical part OCD.
and height (depth) of the river = 3 m = _ 3_ km The frustum of the right circular cone can be viewed as the
1000 difference of the two right circular cones OAB and OCD. Let
:.Volume of one cuboidal river = I x b x h slant height of the cone OAB be f. and its height be h1 •
= 1 072 x � x -3- = 0.2412 km 3 i.e. OB = OA = f. and OP = Ii.
1000 1000 Then, in2 right angled MEC,
2
So, volume of three rivers ( A C ) = ( AE ) + (C£) 2 [by Pythagoras theorem]
= 3x 0.24 1 2 = 0.7236 km ·' . . . (ii) 1 2 = (1j - r2 ) 2 + ( h) 2
From Eqs. (i) and (ii), it is clear that total rainfall is not
approximately equivalent to normal water of three rivers. I = �h 2 + (1j - 72 ) 2 [raking positive square root]
5 An oil funnel made of
Slant height of che cone, OCD = f. - /
·: llOQD -- llOPB [by AA similarity]
1 a cm
tin sheet consists of a

A� E
OD -- DQ
1 0 cm long cylindrical -=
OB PB
! - � = ;. t�h
portion attached to a
� -/ r, I
frustum of a cone. If the :::::) -�- = ;. =>
total height is 22 cm, 22 c m
diameter of the !_ = 1 - 2 => !_ = 'i - r2 c , '2 ,D 1 1
cylindrical portion is 1 o cm
=>
" 1j II 1j \ : /
\ I I
8 cm and the dia meter I1j \ I I

-
ll = -- ... (i) \1/
of the top of the funnel
8 cm
1j - r2 b
is 1 8 cm, then find the On subtracting (/) from both sides in Eq. (i), we get
area of the tin sheet required to make the
funnel. " - l = _l!j_ l = _l!j_ 'i - '2
. . . (ii)
'i - '2
Sol. Given, oil funnel is a combination of a cylinder and a
frustum of a cone. Now, curved surface area of the frustum of cone
For cylindrical portion,
= Curved surface area of the cone OAB
- Curved surface area of the cone OCD
Height, h = 10 cm and radius, r2 = �2 = 4 cm = 1t1j f. - 1tr2 (� - /)

- 1( )
Ir. -
Forfrustum, = 1t7j -- - 1t r2 ---
Ir,
[from Eqs. (i) and (ii))
Diameter of top of the funnel = 18 cm 'i '2 'i - '2

- re -- 'i - 722 - rc/( 1j - )('i + '2 )


1j - r2 - (1j - r2 ) = re/ ( + r, )
'2
2
:. Radius of top of the funnel, 1j = � = 9 cm r.I
-
2
and radius of bottom of the funnel :. Curved surface area of the fruscum of cone = re/ (7j + r1 )
= Radius of cylinder = r2 = 4 cm where, I = �// + (7j - 12)2
Height of the frustum = Total height of funnel
- Height of cylindrical portion = 22 -10 = 1 2 cm If A1 and A2 are the surface areas (A > A , ) of rwo circular
, , I -

:. Slant height of the frustum, ____ bases, then A1 = 1t1j- and A2 = rcr2- .
/ = �12 2 + (9 - 4) 2 = ��144 + 2 5 = .Jl69 = 13 cm So, the total surface area of the frustum o fcone
Hence, required area of the tin sheet = Curved surface area = re/ (1j + r2 ) + 1t'i :? + 'ftr.}
of cylindrical portion + Curved surface area of the frustum where, I = �// + (1j - r:f
= 2 rcr2 h + re (1j + r2 ) /
lI 355 Il All1�nOne MATH EMATICS Class 1 0th Term

7 Derive the formula for the volume of the Now, height of the cone J
hr.
frustum of a cone (using the symbols as OCD = h1 - h = -1- - h [from Eq. (i))
'i - r.!
explained) .
Sol. Let h be the height, I be the slant =
'i
hr.!
. ••(ii)

[, ]
height and, 'i and r.! be the radii of - r.!
the bases ( 1j > r.! ) of the frustum of a :. Volume of the frustum of cone
cone. We complete the conical part =Volume of the cone OAB - Volume of the cone OCD
OCD. h'i , hr,
= -I rr'i· h1 - - rrr.!· ( IJ1 - IJ) = - 'i . --
1, , rr •

( .I )
- r2 -·
-

The frustum of the right circular 3 3 3 1j - r2 1j - r2


cone can be viewed as the difference [from Eqs. (i) and (ii)]
of the two right circular cones OAB
and OCD. =
�I. 'i .\ - r.! =
7t h (1j - 1� ) (1j .! + r/ + 'ir:? )
Let the height of the cone OAB be h1 3 1j - 1�
and its slant height be /1 •
(1j - 1� )
[ · : 1 r1 - b ·' = ( 11 - b)(a i + b2 + ab)]
3

1. e. OP = h1 and OA = OB = 11 7t h
Then, height of the cone, OCD = h1 - h
l ,

= - ( r.1 • + r,· + r.1r., )


.
[by AA similarity] 3
Hence, volume of the frustum of cone 1s. -I rrI ( , + r;., + r.1r,. \,.
b.. OQD .... b.. OPB
• •


OQ
- =- QD J r.1

OP PB 3
h1 - h h
If A1 and Ai arc the surface areas ( A1 > A.! ) of che two
=> = 2 => 1 - = 2 => !!... = 1 - 2 = 'i - r2
circular bases, then A = 1tlj i and Ai = 7tr2.! .
hi 'i h. 'i hi 'i 'i
1

Then, volume of the frustum of the cone


... (i) = * [ 7t 1j 2 + 1tri2 + R ;;;[ 1 = * ( A. + A 2 + �A1 A2 )

TO P I C EXE RCISE
Very Short Answer Type Ql:Jestions Short Answer Type II Questions
1 If t h e rad i i of ci rcu lar ends of a frustum of a cone 5 A solid cone o f base radius 10 cm is c u t into two
a re 20 cm a n d 12 cm and its height is 6 cm, then parts through the mid - poi n t of its height by a
f i n d the slant height of frustum (in cm). CCE 201 1 plane pa rallel to its base. F i nd the ratio of the

2 The rad i i of t he circular ends of a frustu m are 6


volumes of the two pa rts of the cone. CCE 2013
cm a n d 14 c m . If its sla nt height is 10 cm, then 6 A right circular cone is divided by a plane parallel
f i n d its vertical height. to its base i nto small cone of volum e vl at the top
and a frustum of volu me V2 as second part at the
Short Answer Type I Questions bottom . If V1 : V2 = 1 : 3, then f i nd the ratio of the
3 A f r u s t u m of a c o n e of height 7 cm. The radius of height of the altitude of small cone a nd that of
its two circu lar ends a re 6 cm and 3 cm. Find the frustu m .
volume of frustum.
CCE 2013
Long Answer Type Questions
4 The height of a cone is 60 c m . A sma ll cone is
cut- off at the top by a pla ne parallel to the base
7 A n open container made u p o f a metal sheet i n the
form of frustum of a cone of height 8 cm with radii
a nd its volu m e is -2._t h of the volume of original of its lower and upper ends as 4 cm and 10 cm,
64
cone . .find the height f rom the base at which t he
·
respectively. Fi n d the cost of oil w hich can
sec tion is made. c mpletely fif led the container at the rate of � 50
b

An s wers
1 . 10 cm
2. 6 cm 3. 462 cm 5. 1 : 7
- 1)
4. 45
6. 1 : ( 41 / ;j 7. < 65 . .'35. � 245. 1 4
j,
�I Very Short A nswer Type Questi o ns [ 1 Mark each ]

1. What is the total surface area of a solid 12. What is the ratio of volume of a cone and a
hemisphere of radius 7 cm? cylinder of equal diameter and equal height?

2. Find the curved s urface area of a right 1 3. The radii of two cylinders are in the ratio of
circular cone of height 1 5 cm and base 2 : 3 and their heights are in the ratio of 5 : 3.
diameter 1 6 c m . CCE 20 1 2, 20 1 1 Then, find the ratio of their volumes.
CCE 201 1
3 . I f two cubes o f edge 5 cm, each are joined
end-to -end, then find the surface area of 14.· The radius of a sphere is 3 cm. It is melted
the resulting cuboid . and drawn into a wire of radius 2 mm. Find
the length of the wire.
4. A cube of side 6 cm is cut into cubes of 1 cm
side, then find the total surface area of all 1 5. A solid sphere of radius x is melted and cast
the small c ubes. into the shape of a solid cone of height x.
Find the radius of the base of cone.
5. A spherical ball of diameter 21 cm is
melted and recast into cubes, each of sides 1 6. If the radii of circular ends of frustum of a
1 cm. Find number of cubes so formed. cone are 20 cm and 12 cm and its height is
6 cm, then find the slant height of frustum
6. How many spherical balls each of radius (in cm) .
1 cm made from a solid sphere of lead of
radius 4 cm? 1 7. Volumes of two spheres are in the ratio
64 : 27. What will be the ratio of their surface
7. A metallic cube of edge 1 cm is melted into areas? NCERT Exemplar; CCE 2010
a wire of diameter 4 mm, then find the
length of the wire. 18. Find the volume of a largest sphere that can
be cut from cylindrical log o.f wood of base
8. Find the ratio of the volumes of a cube to radius 1 m and height 4 m.
that of a sphere, which will exactly fit -

1 9.
u.
inside the cub e . A cone is divided into two equal parts by
c::::
drawing a plane through the mid-point of its
9. The largest sphere i s cut out from a cube of axis, parallel to its base . Find the ratio of ...::::
side 7 cm. Find the volume of the sphere. volumes of two parts. CCE 201 1 ti
l'­
1 0. The base radii of two right circular cones of 20. A solid of iron in the form of a cuboid of u
the same height are in the ratio 3 : 5. What dimension 49 cm x 33 cm x 2 4 cm is moulded u
(./

is the ratio of their volumes? to form a solid sphere. Find the radius of
sphere. NCERT Exemplar
1 1 . If the radius of the base of a right circular ....
cylinder is halved, keeping the height 21. Find the volume of the largest right ::
circular cone that can be cut out trom a cube \�
same, then find the ratio of the volume of
1
the cylinder thus obtained to the volume of
of edge 4.2 cm. ri iIC

o riginal cylinder. CCE 2009 NCERT ExeJJlplar; CCE 20 1 2 , 1 1
I IC
358 Allinone MATH EMATICS C lass 1 0th Term II

�·
..- 1 Short A nswer Type Questi ons [ 2 Marks each]

22. Three cubes o f volume 2 7 cm 3 each are joined 3 1 . A solid cuboidal slab of iron of dimensions
end-to-end to form a solid. Find the surface 66 cm x 20 cm x 2 7 cm is used to cast an
area of the cuboid so formed . CCE 201 1 iron pipe. I f the outer diameter of the pipe
is 1 0 cm and thickness is l cm.
2 3 . Three metallic solid cubes whose edges are Then, calculate the length of the pipe.
3 cm, 4 cm and 5 cm are melted and formed
into a single cube. Find the edge of the single 32. How many shots each having diameter
cube so formed . NCERT Exemplar 3 cm can be made from a cuboidal lead
solid of dimensions 9 cm x 1 1 cm x 12 cm?
24. A cubical solid block of metal 49 cm x 44 cm
x 1 8 cm is melted and formed into a solid 33. The radius of a metallic solid sphere
sphere . Calculate the radius of the sphere. is 9 cm. It is melted and drawn into a wire
having diameter of cross-section as 0.2 cm.
2 5 . A circus tent is cylindrical upto a height of 3 m Find the length of the wire.
and conical above it. If the diameter of the
base is 1 05 m and the slant height of the 3 4 . The rain water from a roof 2 2 m x 20 m
conical part is 53 m, then find the total canvas drain into a conical vessel having
used in making the tent. diameter of base as 2 m and height 3.5 m.
If the vessel is just full, then find the
26. An ice-cream cone consisting of cone rainfall. N CERT Exemplar
surmounted by a hemisphere. The radius of
the hemisphere is 3 . .:> cm and height of the 3 5 . A solid of iron in the form of a cuboid of
ice-cream cone is 1 2 .5 cm. Calculate the dimensions 4 9 cm x 33 cm x 24 cm is
volume of the ice-cream in the cone. moulded to form a solid sphere. Find the
2 7. A solid is in the shape of a cone standing on a
radius of the sphere . NCERT Exemplar

[ ]
36. From a solid cube of side 7 cm, a conical
hemisphere with both their radii being equal
to 7 cm and height of the cone is equal of its
cavity of height 7 cm and radius 3 cm is
hollowed out. Fin d the volume of the
diameter. Find the volume of the solid.
take,
7t = 2 2
remaining solid . NCERT Exemplar
7

28. A soU d is in the shape of a cone surmounted 3 7. Water flows through a cylindrical pipe,
whose inner radius is 1 cm, at the rate of

[ ]
on a hemisphere, the radius of each of them
being 3 .5 cm and total height of solid is 80 cm/s in an empty cylindrical tank, the
9 . 5 cm. Find the volume of the solid. radius of whose base is 4 0 cm. What is the
n
rise of water level in tank in half an hour?
=
22
take, NCERT Exemplar
7

8. A metallic spherical shell of internal and


29. From a circular cylinder of diameter 1 0 cm external diameters 4 cm and 8 cm,
and height 1 2 cm, a conical cavity of the same respectively is melted and recast into the
base radius and of the same height is
J- 'Jllowed out. Find the volume of the
form a cone of base diameter 8 cm.
remaining solid . [take, 7t = 3 .1 4 )
Find the height of the cone.
NCERT Exemplar

30. 5 0 circular plates, each of radius 7 cm and 39. Three cubes of a metal whose edges are in
thi ck nes s 0 . 5 cm, are placed one above
anoth e r to form a solid right circular cylinder.
the ratio 3 : 4 : 5 are m elted and converted
Find the total s u rface area and the volume of
into a single cube whose diagonal is
1 2 .J3 cm. Find the edges of the three
the cyl inder s o form ed .
cubes. NCERT Exemplar
Surface Areas a n d Volu mes

40. An hour glass is made using identical


double glass cones of diamet er 42. [HOTS! A small terrace at a hockey ground
comprises of 10 steps each of which 20 m
1 0 c m each . If total height is 24 cm,
long and built of solid concrete. Each
t h e n fin d the s urface area of the
glass used in making it. step has a rise of .! m and a tread of .! m.
4 2
CCE 2 0 1 5, 1 4
Calculate the total volume of concrete
required to build the terrace.
41 . A wareho use is used a s a gra n nary. It i s i n the
shape of a cuboi d s u r m o u n ted by a half­
cylinder. The base of

r
i
t h e warehouse is 6 m
x 1 4 m and its height
is 8 m. Find the
surface are a of the
non-c ub oidal part of '--6 rn--i- 1 4 m -1
the warehouse .
CCE 20 1 5

Short A nswer Typ e Questi ons [3 Marks each]

4 3 . A solid metallic cylinder of radius 3.5 cm and 47. A circus tent i s made up using two
h e i g ht 1 4 cm is melted and recast into a different coloured cloth material . Red
number of s mall solid meta llic balls, each of coloured material is used to m ake
7 cylindrical part upto a height of 3 m and
ra d ius
. - cm. Find the number of balls so
12 green coloured material to m ake conical
formed. CCE 20 1 5 part above it. If the diameter o f the base
is 1 05 m and slant height o f the conical
part is 53 m, find the red coloured
44. The cost of painting of the total outside surface
material and green coloured material
of a closed cylindrical oil tank at 60 paise per
required. [Assuming no stitching
sq m is � 2 3 7 .60 and the height of the tank is
margins . ] CCE 20 1 5
6 times the radius of the base of the tank. Find
the radius and height of the tank.
[take, 7t = 22 I 7] CCE 20 1 5
48. A conical hole i s drilled i n a circular
cylinder of height 15 cm and radius 8 cm,
z
45 . From a solid cylinder of height 7 cm and base
which has same height and sam ba e UJ
radius . Find the total surface area after

[
diameter 1 2 cm, a conical cavity of same height
drilling of cone . [take , 7t = 3 .1 4] CCE 20 1 5

J
and same base diame ter is h ollowed out. Find l/)
the total surface area of the remaining solid.
49 . A well of diameter 2 m is d u g 1 4 m deep .
take, " =
22
The soil taken out of it, is spread evenly
UJ
7 all around it to a width of 5 m to f rm an
embankment. Find the h ight of th
46. A rocket is in the form of a right circular embankment .
cylinder closed at the lower end and
surmounted by a cone with the same radius as 50. A hollow cube of inter nal e d g e 22 cm i
of diam t r
that of the cylinder. The diameter and height of filled with s pherical marbl e
the cylinder are 6 cm and 12 cm, respectively.
0.5 cm a n d it is as urne d t h a t 1. p ace of
If the slant height of the conical portion is 5 cm, 8
find the total surface area and volume of the
rocket. [take, 7t = 3 . 1 4)
the cube remains unfi lled . Then , find the
NCERT Exemplar
numbe r of m arbl th at th cub an

accommodate . NC ERT Exemplar


360 �
l
Alli110ne MATH EMATICS Class 1 0th Term 11

5 1 . A bucket made up of a metal sheet is in 56. A conical vessel with base radius 5 cm and
the form of a frustum of a cone of height height 24 cm , is full of water. This water is

]
1 6 cm with radii of its lower and upper emptied into a cylindrical vessel of base

[
ends as 8 cm and 20 cm, respectively. radius 1 0 cm. Find the height to which the
Find the cost of the bucket, if the cost of water will ris e in the cylindrical vessel.
metal sheet used is � 1 5 per 1 00 cm 2 . take, n =
22
[take , 7t = 3 1 4] 7 CCE 2016

5 2 . Marbles of diameter 1 .4 cm are dropped 57. The barrel of a fountain pen, cylindrical in
into a cylindrical beaker of diameter 7 cm shape, is 7 cm long and 0 . 5 cm in diameter. A
containing some water. full barrel of ink in the pen can be used for
Find the number of marbles that should writing 275 words on an average. How many
be d ropped into the beaker, so that the words would be written using a bottle of ink
water level rises by 5 .6 cm. containing one-fourth of a litre? CCE 201 5, 14
NCERT Exemplar
58 . A trophy awarded to the best student in the
53. A container is in the form of the frustum class is in the form of a solid cylinder
of a cone. If its height is 1 6 cm and the mounted on a solid h emisphere ----
--
radii of its lower and upper ends are 8 cm with the same radius and is
and 20 cm respectively, then find the made from some metal. This
slant height of the container and also the trophy is mounted on a wooden
cost of milk that the container can hold, if cuboid as shown in the figure .
the cost of milk is � 30 per L. The diameter of the h emisphere
[take, 7t = 3.1 4] is 2 1 cm and the total height of
the trophy is 2 4 . 5 cm . Find the ,_.....;::i,,,,....c:;..___,
54. The radius of the base and the height of a

[ n
weight of the metal used in making the trophy,

]
3 of the metal is 1 .2 g.

[
right circular cylinder are in the ratio of if the weight of 1
3
2 : 3 and its volume is 1 6 1 7cm . Find the 2
7t =
curved surface area of the cylinder. CCE 2015
22
take, 7t =
7 5 9. How many cubic centimetres of iron is
5 5 . Two s olid right circular cones have the required to construct an open box whose
external dimensions are 3 6 cm, 25 cm and
same h eight. The radii of their bases are r1
16.5 cm provided the thickness of the iron is

�ri2 ; Tl .
and r2 . They are melted and recast into a
cylinder of same height. S how that the 1 .5 cm. If one cubic centimetre of iron weights
radius of the base of the cylinder is 7 .5 g, then find the weight of the box.
NCERT Exemplar

�I Long A nswer Type Questions [ 4 Marks each]

60. The inner diameter of a cylindrical 61 . A solid toy is in the form of a hemisphere
container is 7 cm and its top is of the surmounted by a right circular cone. The
shape of a hemisphere . height of the cone is 4 cm and the diameter of

[
I f the the base is 8 cm. Determine the volume of the

J
h eight of the container i s 16 cm,
th en fi n d the actual capacity of the toy. If a cube circumscribes the toy, then find
n =
22 the difference of the volumes of cube and the
cont a ine r. take,
7 CCE 201 5 toy. Also, find the total surface area of the toy.
NCERT Exemplar
Surface Areas a n d Volum es /361 j

62. A 65. A building is

r-
righ t a n gl e t ria n g l e , whose sides are in the form of a

1
1S cm a n d 20 c n 1 other than hypotenuse is cylinder surmounted by a

j
made to revo lve a b o u t its hypotenu s e . Find hemispherical dome ( see the - r-=::::=====::::i
figure) . The base diameter of H

1
the volu m e a n d the s urface area o f the
d o u b l e c o n e so for m e d . [ take, re = 3.1 4) the dome is equal to � of the
63. A cyl ind ri c a l p i p e h a s inner diameter of total height of the b ilding. - � -

[
7 c m and water flows through it at 192.S L Find the height of the
p e r min . Fin d the rate of flow in km/h . building, if it contains 67 _!.._ m 3 cof air.
21
II = 2 2
take ,
7 J 66. The interior of a building is in the
form of cylinder of diameter 4.3 m and 3 .8 m
6 4. Water is flowin g at the rate of 1S km/h
height, surmounted by a cone whose
thro u g h a p i p e of diameter 14 cm into a
vertical angle is a right angle . Find the area
cubo i d a l p o n d which is SO m long and 44 m
of the surface and the volume of building.
wide . I n w h a t tim e will the level of water in CCE 20 1 5

�1
p o n d ris e by 2 1 cm? NCERT Exemplar

Va l u e Based Questi ons (VBQs) [4 marks each]


(1 x1 x7
An
m m m) or in the form of cylinder o f
67. i c e- c re a m s eller has two types of
diameter l m a n d height 7 m. The rate to dig
ice -cre a m c ontainer in the form of
the well is � SO per m 3 •
cylin d rical shape and a cone with
(i) Find the cost to dig both wells .
hemisp h e rical b a s e . Both have same height
(ii) The farmer decides to dig the cylindrical
of 7 cm a n d s a m e diameter of 7 cm . The cost
well, by his decision which value is
of container a re same but the seller decide
depicted here?
to sell i c e - c re a m in cylindrical containers .

Calcu late the volume of both the


(i) 7 0. Due to heavy floods in a state , thousands
containers .
were rendered homeles s . SO schools
(ii) Which value depicted by the seller? collectively offered to the state government
to provide place and the canvas for lSOO
68 . Two type s o f water t a n kers are available in
tents to be fixed by the government and
a shop . One is in a cubic form of 1-
decided to share their whole expenditure
dimensions 1 m x 1 m x1 m a n d another is
equally. The lower �
part of each tent is �
i n the form of cylindrical form o f diameter cylindrical of base radius 2.8 m and height LI.Ji
a n d h e i g h t i s also 1 m.

lm
3. 5 m , with conical upper part o f same base ·-
(i) Calculate the volume of both the radius but of height 2 . 1 m. I f the canvas
tankers . 2
used to make the tents cost � 1 2 0 per m ,
(ii) The shopkeeper advise to purchase then find the amount s hared by each school

[ �]
cuboid tank. Which value is depicted by
to set up the tent s . What value is generated 't- · .
the shopkeeper?
by the above problem? "'4-

A farmer wants to dig a well either in the


4
take, II =
69. 2
CCE 20 1 6
form of cuboid of dimensions
So l u ti o n s
1. Given, radius o f a solid hemisphere = 7 cm TCr2 h = (Edge)J
Total surface area of a hemisphere = 37tr2 2 2
TC X - x - x h = l x l x l
10 10
3
= 7t x 7 x 7 = 147 7t cm 2 [1 1
� x h = I
2. Given, height of cone, h = 15 cm
25
and radius of cone = 1 6 = 8 cm
r
h=- 25
cm
2 TC
[112)
Let the slant height of a right circular cone be /. [·: length of wire = height of wire]
h2 r2
:. Slant height, l = � + = �1 52 + 82 8. Lee diameter of the sphere = Side of che cube = I
= .J12 5 + 64 = .J2s9 = 17 cm [1 /2] r
:. Radius of the sphere, = 1 1 2
Now, required ratio of volume of cube and volume of
[1/2)
:.Required curved surface area = 7trl = 7t x 8 x 17 sphere
= 1 367t cm 2
Hence, the curved surface area of a right circular cone is = Volume of che cube = (Side)3
Volume of che sphere �
1t
1 36 cm 2• [ 1 /2) 3 1tr

; 11 (H
3 . Do same as Q. 1 at page 57 . 3 [Ans.
250 cm 2]
= /3
---- �= 6 : TC [1/2)
TC
4. Volume of a cube of 6 cm side = 63 = 216 cm3
[·:
volume of cube = (sidc)3]
Volume of a cube of I cm side = i3 = 1 cm3 9. Given, diameter of che large sphere

]
[1 /2] = Side of the cube = 7 cm

[-:
:. Number of cubes of 1 cm side = 1 = 216216
r
:.Radius of the sphere, = ?... cm
radius = dia�erer
2
Now, total surface area of a small cube = 6 x 12
= 6 cm 2
:.Tocal surface area o f 216 cubes = 2 1 6 x 6
= 1296 cm 2 Volume of the sphere = 34 3 - 1tr
Hence, the total surface area of all the small cubes will
3 33x
= -4 x - 22 7 -7 x 7 539 cm
3 7 x -2 x 2 2
- == - (1 ]
be 1 296 cm 2• [1 /2]
5 . Given, the diameter of spherical ball = 21 cm 1 0. Lee che radii of two right circular cones be unics and
So, rad.1us = - 2 1 cm x
5 units.
2
Volume of spherical ball Volume of first cone = ! TC 3 (3x)2 h
X_

..::;
_ _

[·: volume of cone = � nr2h]


_

Number ofcub es = ------=- ---- Volume of second cone �- (5x )2 x


_

Volume of each cube


4x- 22 x --21 x - 21 x 21
[1 /2]
3 h7t

3 3 3
4
-1tr
= -__ =
( Side}3
-
7 2 2 2
1x1xI
= 485 1 [1 /2) = -2._
25
= 9 : 25
6. Do same as Q. 5.
Using formula, Hence, the ratio ofcheir volumes is 9 : 25. [1 )
Do same as Q. 10.
Volume of sphere ball = i. 3
3 1tr [Ans. 11.
64] Using formula, volume of cylinder= rcr2h [Ans. :
I 4)
7. Ler h be the length of wire and radius, = 2 mm r 1 2. Do same as Q. 10.
= -2 cm [·:1 mm = -1 cm ] Using formula,
10 10 Volume of a cone = 1
Since, a metallic cube is melted into a wire. 3 rcr 2h
-

[Ans. 1 : 3)
.. Volume ofwire =Volume ofcubc [1/2) and volume ofa cylinder = rcr 2h
Surface Areas a n d Volum es / 363 j
1 3. Do sameformula,
Using as Q. 10. = _i3 7t (1 )3 = _i3 rt m 3
[1 /2]
Volume of cylinder= rtr 2 h [Ans. 20: 27] Hence, che volume ofa largest sphere is i3 m 7t 3• [1 /2]
1 4. Given, radius of sphere = 3 cm2 1 9. Lee r andrespecci
R be che radii of upper and lower pares of
Radius ofwirc,r1 = 2 mm= 1 0 cm [·: I mm = /0 cm] fruscum
so, lee 2 h be chevelyhei. ght of
Ala cone.
·: Volume of wire = Volume of sphere A

Hence, 7t1j = -3 1tr 2


h
4 3 ( 1 /2)
AA
Since, by similaricy,
I
J
llABC .. llAB' C',
2 2 = -4 (3 cm [here, =
=> 7t x - x - x I 7t J
J
we gee 2h

10
h
3
/]
10 BC
-- = --
AB

Hence, che lengch of chc=wire900iscm900 cm.


I B' C' AB'

1 5. Do sameformula,
as Q. 14.
[1 /2)
=> -Rr = -2hh
Using
Volume of a sphere = -34 1tr 3 Now, R = 2r
=> (1 /2)

Volume of che upper part 3 7tr


.!_ 2 h
and volume of cone = .!_3 rtr 2
a h [Ans. r = 2x] Volume of che lower pare
x

.!_ 7th (4r 2 r 2 2r 2 )


+ +
1 6. Here, upper radius of fruscum of a cone, = 20 cm, R 3
1
Lower
and heiradius
g ht of offruscum
fruscumofofa cone, a cone, r= =6 cm.12 cm
h
=7
- [1 /2)

We know chac, slant1 2 height of fruscum2ofa cone [1 /2) Hence, racio of volume of two parts is 1: 7.
= + - r) Leeris gibevenradichat
us oflength
sphere.of cuboid = 49 cm,
hi (R 20.
=> 12 = 6 2 (20 - 12) 2+ lebreadth
=> 12 = 36 64 = 100 +
=

.".Volume of33sphere cm and=Vol heiguhtme= of24cuboi


cm d
I = lOcm [·: posicve squarecm.root]
Hence, che required slanc heicaking ghc of fruscum is 10 [1 /2)
=>

4
i 1tr3 = 49 33 24
223 r3 = 49 33 24
x x [1/2)

=> - - x x
1 7. Lee che radius of one sphere be and radius of ocher 'i 3 7 x x

sphereVolume
be of one sphere 64
r2 • =:) r3 = 49 334 2422 3 7
x x x x

Volume of other sphere = 27 =:) r = (49 3 3 3 7)1 13


x

x x
4 7t 1j3 r = 21
x x

[1 /2)
=>
3 = 64 21 . Since, the l a rgesc ri4.ght2 cm.circular cone can be cue ouc

--3 7t ,.2. 27 fromHeiaghtcubeofcheof edge
Radius of cone = 21 cm [-: di�eter = radiusJ
and diamecer ofcone= che coneEdge=Edge ofcheofcube=
cube =4.24.cm2 cm
. ·. [1 /2]
r.3 - 64
I -
=>
,.23. 27
=> -r1j2 = -34 [·: cakmg. cube root] [1/2)
=>

:. Rano of the1r. sur ace areas = --


. f:
4rt 1j22 = 1j2 = -- ( 4)22 :. Volume of cone= .!.3 7tr2h
2
4rt r2 r2 (3) 2
[·: surface area of sphere= 47tr ] = -31 -227 2 2 .1 4.2 = 19.404
x x . 1 x x cm 3
[1 /2)

= 169 = 16 9 : [1 /2) 22. Same as Q. l a t p age 33 7, N CERT fo lder 9. 1 .


[Ans. 126 cm 21
1 8. Here, the
c l
al a rgestof sphere of
of radi
base u
1 sm 1 m canhei_beghtcue4 m.from
and 2 3 . Do same at p age
as Q. 2 346, NCERT folder 9.3.
cylindri log wood4 Using formula,
:. Volume of sphere = -1tr· 3
Volume ofcube= (Edge)3 lAns. cm] 6
3
Allin.One MATH EMATICS Class 1 0th Term 11

24. Given, for cubical porcion, 27. Do sa me as Q. 26. [Ans. 1437.3 cm 3 ]


l = 49 cm, b = 44 cm and h = 1 8 cm
28. Do same as Q. 26. [Ans. 166.83 cm 2 ]
Ler r be rhe radius of sphere. 29. Given , diameter o f the cyl i nder = I 0 c m
According ro rhe question, 5 cm
Volume of cubical solid block = Volume of sphere :. Radius of the cy l i n<ler, r = 5 cm
Since, the base of the cone a n <l cyl i n<ler
=> l x b x h = _! 7tr3 1s same.
3 So, radius of cone, r = 5 cm
49 x 44 x 18 = -34 x - 22 x r3 [1 1 Height of cone = H eight of cyl i nder
= h = 1 2 cm
7
3 49 x 44 x 18 x 3 x 7
r = ------ Now, volume of ch c remai n i ng solid
4 x 22 =Volume of cyl i nder - Vol u me of cone (1 )
49 x 2 x 18 x 3 x 7 ., I , 2 ,
= ------
4 = 7t ,. -h - 3 ,. - h = -3 7t ,. - h
- 7t

= 49 x 7 x 9 x 3
r3 = (7 x 3)3 = -2 x 3 . 14 x (5 )-, x 1 2
3
=> r = 21 cm = 2 x 3.14 x 25 x 4 = 628 cm .\ (1]
Hence, rhe radius of sphere is 21 cm. [1 ] Hence, che vol u me o f the rem a i n i ng solid is 628 cm3•
25. Do same as Q. 7 at page 338, NCERT folder 9. 1 30. Given, radius of the cyl i nder formed, r = 7 cm
[Ans. 9735 m 2 ]
Thickness of one place = 0.5 cm
Height of che cyl i nder formc<l , h = Thickness of SO
places = 50 x 0.5 = 25 cm
Tocal surface area of chc cyl i nder formed
22
= 2 r (r + h ) = 2 x -
7t x 7 (7 + 25)
l
3m
7
= 2 x 22 x 32 = 1408 cm 2
l Volume of the cylinder formed = 7t r2 h

2
- 1 05 m-
26. Given, radius of hemisphere and cone, r 3.5 cm = = 22 x 7 x 7 x 25 = 3850 cm 3 (1 )
7
Height of conical portion, h = Height of ice-cream Hence, che total surface area and volume of che cylinder
cone - radius of hemisphere so formed are 1408 cm and 38 50 cm .1 , respectively. (1 ]
= 12.5 - 3.5 = 9 cm
3 1 . Here, outer radius of rhc pipe, R = .!..Q = 5 cm
2
Inner radius of the pipe, r = Outer radius - Thickness
= 5 - 1 = 4 cm
Lee h be che length of pipe.
:.Volume of the pipe = n( R 2 - r2 )h
= 7t (5 2 - 4 2 ) h = 227 x 9 h cm 3x
(1)
:. Volume of ice-cream in the cone Now, volume of the pipe = Volume of the cuboid
= Volume of hemisphere + Volume of cone �
22
- x 9 x h = 66 x 20 x 27

= -32 1t r3 + -1 1t r 2 h [1 1
7
[·: volume of cuboid = lbhJ
3
1 2 h =
66 x 20 x 27 x 7 = 1260 cm
= -3 7t r (2 r + h )
!�1
:1 I 22 (3.5 x 3.5) (2 x 3.5 + 9)
22 x 9
= - x -
Hence, the length of the pipe is 1 260 cm . [1 1
3
x
7 32. Given, dimensions of cuboidal lead
= I x 22 x 3.S x 3.5 x 1 6 = 205.33 cm 3 [1 1
::::it 9 cmx l I cmx l2 cm
3 7 :.Volume of cuboidal = 9 x 1 1 x 12 = 1 188 cm3
Hence, the volum e of the ice-cream in the cone is [·: volume of cuboid = l x b x hJ
205 .33 cm 3 .
Surface Areas and Volumes 13 65
i

·: Diameter of each shoe = 3 cm Radius of conical cavity, r = 3 cm


.. Radius of each shoe, r = �2 = 1.5 cm Now, volume of cube= (Side)3
= a3 = (Tf = 343 cm3
(1 1

Now, volume of each shoe= -7tr3 4 and volume of conical cavity= .!..3 7t h x r2 x

22 3297
= i3 7 (1.5)3 = 21 = 14.143 cm3
x x = -31 227 3 3 7 = 66 cin 3
x - x x x [1 1
:. Requi red number ofalshoes Accordi
Volumenofg=Vol otremaining
the questisolon,id
= Volume of each shotlead - --
Volume of cuboi d 1188 = 84 (approx.)
14.143 ume of cube -Volume of conical cavity
Do same as Example 3 at page 344. 972 m]
[1 ] = 343 -66 = 277cm3
33. [Ans. Hence, the required volume ofsolid is 277 cm3• [1 ]
34. Given, radius of conical vessel, r = �2 = Im 37. Given, radius of tank, = 40 cm 1i

and
Lee heigrainfall
che ht = 3.5bem cm. x
AlLetso,heigigvhtenofinternal
water radi
levelusinoftankcylinidrin halfcal pianpe,hourr2 ==1/Jicm
Now, volume of water on theofroofconical vessel Speed of water= 80 cm/s
=Volume In 1 s water flow = 80 cm
=> 22 20 = .!.3 7t r2h
. ·. In 30 (min) water flow = 80 60 x 30 x
x x x [1 ]
= 144000cm = 60] [1 ]
=> 22 20 = .!_3 227 (1)2 (3.S )
x x x x x x
According to the question,
[·: lmin
I Volume ofwater
:. x = -3 227 -3510 22I 201
x - x x - x
flowinfrom
2
cylinthedricicalrcular
tankpi=Vol
p e i n uhalmefanofwater
hour
=- 1 m = -1 100 cm 1 m= lOOcm] 1t7i "1 = 1t 7 2 hi
2
120 120 x [·:
40 40 x "1"' = 1144000
x x 1 144000
x

= �6 cm = 0.83cm = 40 40 = 90 cm
x
Hence, the rainfall is 0.8 3cm. [1 ] Hence,
in halfanthehour. level ofwater in cylindrical tank rises 90 cm
35. Gi24cm.
ven, dimensions of the cuboid arc 49cm, 33cm and (1 ]

:. Volume of the cuboid = I x 38. Given, internal diameter ofspherical shell = 4 cm


x b h and externalradidiaumeter of shelcallshel= 8 cm
= 49 33 24 = 38808 cm 3
x x
[1 ] :. Internal s of spheri l, di�cter]
According to theof thequestion,
Let the radius sphere be r. r = � cm= 2 cm [·: radius = 1-
2
Volume of the sphere =Volume of22the cuboid and external radius ofshell, R �2 = 4 cm = u.
=>
3 38808 ,!3 7 r3 = 38808
,! 7tr3 = 38808 => x x

1
73 = 4 22 3 r3 = 44 1 21x 7 x
=>
Now, volume of the spherical shell = i3 7t [ R3 - r3] (f
[·: volume of the spherical shell (/
x

u
x

r3 = 21 21
Hence, the radius of the21 sphere is 2 lcm.
=> x x r = 2 lcm
=>

[1 ]
= � 7t {(external radius)3 - (internal radius)3 } (/
l/
36. Given, side ofa solid cube, 7 cm a = id
Height of conical cavity, i.e. cone, = 7 cm h
= i3 1t (43 - 23 ) [1 ) I.Im
= -34 1t (64 - 8)= - 224 7t cm3
3
.. .
(i) ::=
-
t-
Let height of the cone = h cm <1
-=
Diameter of the base of cone = 8 cm c:
:. Radius of the base of cone =-i c
II:
-
=4cm -

"
366 1 AllinOne MATHEMATICS Class 1 0th Term 1 1

According to the question, = 2 2r rt h


= m· -
")

rtrh + rtr 2
Volume of cone= Vol224ume of spherical shell + 2 x
2
=> .!.. 7t( 4) 2 h =
3 3224 7t [from Eq. (i)] = = -227 3 (14 3)
rtr( h + r) x +

h = - = 14 cm Surface _area22of non-cuboid1al1 pare


Hence, the height ofa cone16 is 14 cm.
=>
2

[1 ]
= 7 3 17= 7 m x x
22
(1)

39. Let aleebethetheedges


and side ofofnew
three cube
cubes so(iformed
n cm) after3x,mel4xtand
be ing. 42. le is clear, from the figure, lengch = 20 m
5x respectively. and widch = .!_ m of each seep.
2
Therefore, and height of Isc seep which is in che boccom = .!_4 m
Volume of single new cube =Volume of three cubes
=> a3 = (3x)3 (4x)3 (5x)3
+ +
:. Heighc of second seep = .!_4 .!_2 2 x = m

=> a3 = 216x3
=>
We know chat,a = 6x [ · : caking cube root] ... (i )
[1 ] Height of third seep = 3 .!_4 �4 m x =

Diagonal of a cube= �a1 a1 a1 = aJ3 + +


Height of cench seep = 10 .!_4 = �4 m x
.. aJ3 = 12J3 Tocal vol1ume 1of the concrecc used
a = 12 ... (ii) I
From Eqs. (i) and (ii),6xwe= gee12 = 20 - -4 20 -2 -42 20 21 -34
x
2
x + x x + x - x + ...

=> x =3 2 2, 4 2, 5 2 + 20 .!_2 �4 [·: volume ofcuboid = I x h]


x x x b
Hence, the three edges,
i.e. 6, 8, 10 respectively. x x x
[1 ]
I
= 20 x -2 x -41 [1 2 3 10] + + + ... + [1 J

[. 1 2
40. Lee l be the sl10ant height of each cone given, radius of = 20 .!_2 .!_4 10 2 11
x x x
x
each cone = -2 = 5 cm
and height of each cone = 242 = 12 cm . .
1)] + + ... + n -
n (11 +
---

2
Now, we know chat, = 137.5 m3 [1 J
l = r 2 h2 l = �5 2 122
+ => + 43. Given, radius and height of cylinder, r = 3.5 cm and
=> l .Jl69
= => = 13 cm
I [1 ] h = 14cm.
. ·.Tocal surface area of one cone = 1trl 7tr 2 + Also, radius of sphere, R = !_12 cm
= 7t x x 7t x (5)2
5
= 657t 257t
13 +
Now, volume ofcylinder= 7tr2 h = -227 (3.5 )2 1

x x 4
- +
, = 907t cm 1 = 22 12.25 2 = 539 cm3
x x
(1 ]
) Hence, total surface area of two cone= 2 907t 2 x Now, volume of each solid mecallic ball = i3 1tR3
J
I
= 1807t cm [1 ]
1
r
41 . Lee r be the radius of cylinder and h be the height of
cylinder. = 34 7t ( 127 )3 = 34 722 12 7 12 49 12 x x
x
x

Itbreadth
is given chat= 6dimensi ohein ofghtcuboi d, length l = 14 m, 11 49
3 3 6 x 12 cm3
x

(b)
:.Radius of cylinder m and (h) = 8 m. ------

x x
(1 ]

(r)
= = Breadth of2 cuboid = �2 = 3 m :.Number ofballs VolumeVolofueach me of cylinder
solid metallic ball
539
and Height ofe cylinder
area of
= Length ofcuboid = 1
the non-cuboidal part
4 m [1]
= 3 x 11 xx 649x 12 = 539 x 1 1 3 49 6 12 = 3 x x x
Now, Surfac 3 x
648
= Curved surface area2 ofAreahalfofcylin
+ x onedersemi-circle. Hence, the required number ofballs so formed are 648.
(1 )
Surface Areas and Volumes /367
44. Lee che radius of closed cyli ndrical oil rank be r m. CSA of cylinder + CSA of cone + Base area of cylinder
Then heigh t of che c a n k , = 226.08 + 75.36 = 30 1 .44 cm 2
[1]
h = 6 r x
= 6r m . . . (i) and cocal volume o f th e rocket
Now, coral surface area of closed cylinder = Volume of the cylinder +Volume of the cone
= 2rc rh + 2itr.! [1 1 � 339.12 37.68 376.8
+ = cm 3

= 2 rcr ( h + r) = 2 22r x
(6r + r) [from Eq. (i)] 47. Given, height of cylinder = 3 m
7 105
=
44 x 7r-, = 44r-, m -
-
'
Diameter of cylinder =
. .
m
Diameter 1 05
. . . Radms of cylinder = (r) = = -m [1 1
7 2 2
According co chc question,
Cose of painting rhe coral outside surface of a closed
of
cylindrical oil rank ar 60 paise pe r m 2 = � 237.60
. . 44 r 2 x 0.60 = 237.60
, . -, 237.6 0 23760
=> = [1 1
44 44 x 60
x 0.60
----

r2 = 9 � r = 3 m
From Eq. (i), h = 6r = 6 x 3 = 1 8 m [1 ] So, red coloured material required
Hence, the ra diu s and height o f the rank arc 3 m and = Curved surface area of cylinder
1 8 m , respectively.
= 2rcrh = 2 x
22 x 1 05 m x3 2
45. Do same as Q. 8 ar p ge 339, NCERT folder 9. 1
a 7 2
[Ans. 55 1 cm 2 ] 2
= 66 x 1 5 m = 990 m 2 [1 ]
46. Given, roc ker is the combination Green coloured material required = rcrl
of a ri ght circular cylinder and a 22 x 105 x 53
cone. =
for cylindrical portion
7 2
[here, slant height of che conical portion, / = 53 cm]
Diameter o f ch c cylinder x 5
= 1 1 1 x 53 = 8745 m 2 [1]
= 6 cm
:. Radius of the cylinder 48. Given,
6 =3 Radius of cylinder, r = 8 cm = Radiw of conical hole
=
2 cn1-
Height of cylinder, h = cm 1....;._
5_

=
.{cFE) 2 + (ED) 2
_____

and height of ch c cylinder In right llFED , FD =

=
= 1 2 cm
[by Pythagoras theorem]
[1 /2]
:. Curved surface area of cyliner = 2rc rh
= 2 x 3. 1 4 x 3 x 1 2 = 2 2 6 . 0 8 cm 2 [1 /2] =�(15) 2 + (8 ) 2 J225 + 64
and volume of the cylinder = r 2 h rc = J2a9 17 cm � Slant height (/ ) [1 ]
3
= 3.1 4 x (3)2 x 1 2 = 339. 1 2 cm
For conical portion
In right angled M OC , the height of cone,
h= 2 �5 2 - 3
[·: slant height of cone, 12 = h2 + r 2]
.J2 5 - 9 = .J16 = 4 cm
=
:. Height of the cone, h = 4 cm
1' "
a cm
Radius of the cone, r = 3 cm . . Total surface area, after drilling of cone
Now, CSA o f cone + Base area of cylinder = rcr(l + r) = Curved surface area of cylinder (11
= 3. 1 4 3 (5 + x
= 75. 36 cm 3) 2 + Area of base of a cylin der
and volume of che cone = .!.. rc r2h + C urved su rface area of cone
3 = 2rcrh + nr2 + rt rl
= .!_ x 3. 1 4 x (3) 2 x 4 = 1 1 3 ·04 = 37.68 cm 3 [1 ] = 2x 3.1 4 8 x 1 5 + 31 4 x ( ) + 3 14
82 x 8 x 17
3 3 x
Hence, total surface area of the ro ket
c
= 753. 6 + 200 . 9 6 + 42 7.04 = 1 38 1 .6 cm 2 l1 ]
368 1 All{nOne MATHEMATICS Class 1 0th Term 11

49. Do same as Example 2 at page 344. [Ans. 0.40 m] 52. Given, diameter of a marble = 1 .4 cm
:. Radius of marble = � = 0. 7 cm
2

]
So, volume of one marble = i 7t (0. 7)3
[·: volume of sphere = � nr'
3

= i 7t x 0 . 34 3 t . 372 7t cm 3 (1 ]
==

SO. Given, edge of the cube = 22cm 3 3


Also, given diameter of beaker 7 cm ==

: . Volume of the cube = (Edge)3 = (22)3 = 10648cm3


:. Radius of beaker = ?_ = 3.5 cm

[
[1 /2) 2
Also, given that diameter of a marble = 0 . Scm Height of water level raised = 5. 6 cm
:. Radius of a marble = O.S = 0.25cm . . Volume of the raised water in beaker
2 ' . diameter ] = 7t (3.5)2 5.6 [·: volume of cylinder = nr2h]
·: rad ms

[·:
= ---
x

2 = 68.6 7t cm-� [1]


Volume of one spherical marble Now, required number of marbles
· = ,! 22 x (0.25 J3 volume of sphere = ,! 7tr3 ]
x = Volume of the raised water in beaker
3 7 3 Volume of one spherical marble
88 1 75
=
21
x 0.0 15625 = 3 = 0.0655cm3
21 [1 ] = 68.6 7t 3 = 1 50 marbles
x ( 1)
1 .372 1t
F illed space of the cube with marble 53. Do same as Q. 4 at page 355, NCERT folder 9.4
= Volume of the cube 1 Volume of the cube
-- x [Ans. 20 cm, � 313.80]
8
54. Lee radius of base of cylinder be 2x and ics height be3x.
= 10648 .!.. 1 0648 = 10648 - 1331 = 9317 cm3 Given, volume of a right circular cylinder= 1617 cmJ
8
- x
[1/2]
:. Required number of marbles => 1tr 2 h = 1617
= Total space filled by marbles in a cube
Volume of one marble
=> 7t(2x)2 (3x) = 1617
9317 => x3 = 16 1 22
7 7 x -I x

= = 142244 12
0.0655 7
=> x3 = - 147
Hence, the number of marbles chat the cube 24
x

= 7 x 4 9 = 343 = ( ?.... )
accommodate is 142244. [1 ] 3
5 1 . Here, radius of upper end of frustum
of a cone, R = 20 cm, radius oflower 8 8 2 [1 ]
J end of frustum of a cone, r = 8 cm =>
7
x = => x = 3.5 cm
-

-
and height of bucker (frustum), 2
) h = 1 6 cm. :. Radius, r = 2x = 2 3.5 = 7 cm x

) :. Slane height(/) = �h2 + (R - r)2


and height, h = 3x = 3 3.S = 1 0.5cmx (1 ]
J -
Now, curved surface area of che cylinder = 2 7trh
') =�162 + (20 - 8)2 = .J256 + .1 44 = 2 22 7 x I O.S = 462 cm 2
= .J4oo =20 cm
8 cm x
7
x
)
-
[1 /2)
Now, total surface area of the bucket Hence, curved surface area of the cylinder is 462cm 2 •
= Curved surface area of frustum + Area of base (1]
= rel (R + r ) + rcr2 = re [l (R + r) + r2 ] 55. It is given that 1j and r2 are radius of two cones
= 3.1 4 [20 (20 + 8) + (8)2] respectively.
=3.14 [ 20 x 28 + 64] Lee h be height of two cone and let be radius of R
cylinder.
= 3.1 4 x 624 = 1959.36 cm 2 [ 1%] and also ic is given chat height of cylinder =height cone.

: . Cose of m e caJ sheet used at the race of t15 per :.Height of cylinder = h (1)
1 00 cm 2 = 1 9 5 9. 36 11005 = ' 293.90 (approx.) [1 ]
x -
. ·.Volume of cylinder
= Sum of the volumes of two c nes o
Surface Areas a n d Volu m es /369
=>
...,
rrR - h = -
I 2
rrr.1 h + 31- rrr..,2 h [1 1 => .!_
4x 1 000 cm 3 = 275 x 1 6
22
x .!_
4
x 1000
3 -
=> [·.· he will use .!_ L ofink co write words]
4
Hence, che pen can write total number ofwords by .!_4 L
3 of ink.
R= � [1 1
16 x 1 000
275 x
v 22 x 4
� Hence proved. = 25 x 2 x 1000
56. Given, radius of cone, r = 5 cm, = 50000 words [1 1
height of cone, h = 24 cm 58. le is given that diameter of a hemisphere = 2 1
and radius of cylinder, 'i = 10 cm Le e radius of hemisphere be r.
Lee, height of cylinder = H cm [1 1 21
r = -cm
S cm 2

l l
and total height of trophy = 24.5 cm

T
:. Height of cylinder = Height of trophy
24 cm
-Radius of hemisphere

l
= 24.5 - E.
2
= 24.5 - 10.5 = 14 cm

( t)
[1 )
:.Volume of cylinder = Volume of cone [1 /2)
:.Volume of che metal used in che trophy
=Volume ofcylinder + Volume ofhemisphere
= 7tr2 h + � 7tr3 = 7tr2 h + �
1 1tr 2 h

x ( 14 + x )
2
=> 1t1j H = -
3
'i 2 H = - r 2 h
1
=> 22
3 = x E. E. � E.
7 2 2 2 3
=> (I 0 ) 2 H = .!_ x (5) 2 x 24 22 21
21
3 = - x x- - (14 + 7)
24
= 2 5x 1 00 x 7 2
2
H
3 =- 22 x 21
21 x 2 1
- x -
=> H = 2 cm [1%) 7 2
2
Hence, height co which che water rise in che cylindrical 1 1 3 x 21 x 21 cm 3
vessel is 2 cm. = x

2
[1 ]

57. Given, height of cylindrical pen = 7 cm


:. Weight of mecal = 1 1 x 3 x2 21 x 21 x 1.2 g �
. = Diameter = -
Rad 1us 0.5 cm U-J

: (o; r
2 2 = l l x 3 x 2l x 2l x 0 .6
:.Volume of barrel of a fountain pen = rcr2h = 8731.8 g = 8.7318kg [1 ] :�
-�
59. Lee the length(/), breadth (b) and height (h) be the �- i
·'

: ·-�
=2 7 external dimensions ofan open box and thickness be x. .....
. . ;"

t...l-.i
x x

Given, external length of an open box (/) = 36 cm


�.; .
= 227 External breadth of an open box (b) = 25 cm

- x 0.25 x 0.25 x 7
and external height of an open box (h) = 1 6.S cm
22
--
x 25 x 25 -- = 22
- cm 3 :. External volume of an open box :::: lbh
16
1 0000 [1 ] = 36 x 25 x 1 6 . 5 = 14850 cm3
It is given that, a pen can write 275 words by using che
. k-
m
22 cm 3 . Since, che thickness of the iron (x) = 1 . 5
So, internal length of an open box ( 12 ) I 2 x =
cm

-
16
Volume of ink = 275 words = 36 2 x 1 5 = 36 - 3 33 cm
- .
=
\
�'
.. : .: ·

22 cm 3 = 275 words
-.:..:.:.
.... ..._ .

Therefore, internal breadth of an open box


-
16 (b2 ) = b - 2x
2 7 5 16 words = 2 5 2= 1 . 5 - )(
=> 1 cm3 =
= 25 .3 22 cm
x
[1 ] _
22
370 1 �
AllinOne MATH EMATICS Class 1 0th Term 11

and incernal heighc o f an open box ( h2 ) = ( h - x) Volume of rhe coy = Volume of rhe hemisphere
+ Volume of che cone

(� )
= 16 . 5 - 1 . 5 = 1 5 cm
So, incernal volume of an open box =
2 l 1 l

= 1 x bi x h = 33 x 22 x 1 5
1tr- +
- 1tr-h -

2 2
3 3
= 1 0890 cm3 [1/2] ·
; = x
22 -' + I x 22 x 4-, x 4
j
x 4 - -

Therefore, required iron co conscrucc an open box 7 7


= Excernal volume of an open box 1 408 l
- lncernal volume of an open box = -- cm · [1]

7
= 14850 - 1 0890 = 3960 cm 3 A cube circumscribes chc given solid. Therefore, edge of
che cube should be 8 cm.

( �)
Hence, required iron co conscru�c an open box is
3960 cm3 . [1/2] Volume of rhc cube = 8-' cm·\ = 5 1 2 cm3
Again given char, 1 cm3 of iron weigh cs = 7 . 5 g Difference in che volumes of chc cube and che coy
14 8
= -2:2._ kg = 0.0075 kg = s12

[·: kg]
cm ·' = 3 I 0.86 cm3
-

1 000 [1]
:.3960 cm3 of iron weighcs = 3960 x 0.0075 ·
Tocal surface area of che coy = Curved surface area of
1 cone + Curved surface a rea of hemisphere
lg = _
1000 = 7tr/ + 2 7tr 2 where I = �// + r 2 ,

= 29.7 kg [1 ] = rcr(/ + 2r)


22
x 4 x [ �4 2 + 4 .! + 2 x 4 ]
60. Given, inner diamecer of cylindrical concainer = 7 cm =
7
:. Inner radius of cylindrical concainer = ?_ = 3.5 cm
2 22 r:::
x 4 x [ 4-v 2 + 8 ]
= -

= Radius of hemisphere 7
Heighc of cylindrical concainer = 1 6 cm 88 x 4 r::: ,
= -- [ v 2 + 2] = 1 7 1.69 cm •

Now, volume of cylinder = rtr 2 h 7 [2)


=
22 x
(3.5) 2 x 1 6 [1 ] 62. Do same as Q. 2 ac page 354, NCERT folder 9.5.
7 [Ans. 3768 cm3 , 1 3 1 8.8 cm 2)
= 2 2 x 1.75 x 1 6 = 6 1 6 cm 3 [1] p
Volume of hemisphere = � rtr3
3
= � x 22 x (3.5)3 =
44 x
6.1 25 Q "'""
\
-- ----- --- _, R
3 7 3 ,
,
',
2695 -''5'',, I
= = 89.83 cm 3 [1 ] ',
3 C"-?; ', , �
: . The accual capaci cy of che con rain er P'
= Volume of cylindrical concainer 63. Given, diamecer of cylindrical pipe = 7 cm
-

>
- Volume of hemisphere
Radius = ?_ cm
2
= 6 1 6 - 89.83
= 526. 17 cm3 [1 1 According co che quescion,
61 .

[-: � ]
Let r be che radius of che hemisphere and che cone and Volume of wacer flows through che cylindrical pipe per
h be the height of che cone. minuce = 1 92.5 L
Given, Radius of hemisp here [·: volume of cylinder = 192. SL)
8
= Radius of cone = r = - = 4 cm = 1 9 2.5 O O ( 1]
2 I
• • 11r2h x 1 0-3 I L = m3

192.5 1 0-3 1 92 .5 1 0-3 7


[1)

[-: � � m]
x x x
h = =
7t ,2
x
22 x __ 7 x 7
__

200 200
r = cm =

2 0
� 1
i '
Surface Areas and Volum es / 371
v
1 92.5 x 1 0 ..\ x 200 x 200 66. Given,

- =
22 x 7 Radius of base of cylinder,
= 50000 x 10 -.\ m = SO m [1] 1j = 2 4.3

- [·: ]
h 50
:. Race of flow = - = 60 mis
I
[t I min = 60s] = 2. 1 5 m
I mis = 1; km /h
18 :.Radius of base of a cone,
= 65 x - r2 = 2. 1 5 m E

:� :: � [-: r--4�...... _
5 co
Height of cylinder, h.. =3 . 8 m C")
= 3 km / h [1] ·
n � � sin 9 = �
[·: sin 45°= �J
Hence, che race of flow of waccr is 3 km/h.
m'-
3 ...._
64. Do same as Ex. 4. ac page 344. [Ans. 2 h]

J 1 4 cm
=>
1 = 2.1 5
F2 VA
Pipe => VA = F2 x 2.1 5 = 1.414 x 2.15 = 3.04 m [1/2]

]
Clearly, .1VOA in an isosceles triangle.
. . VO = OA = 215 m
. ·. Height of cone, h2 = VO = 2.15 m
21 cm Slant height of cone, 12 = VA = 3.04 m [1 /2]
50 m
Now, surface area of building
65. Le e r be che radius of chc hemispherical dome = Surface area of the cylindrical shape of building
+ Surface area of the conical shape of building
and co cal heighc of building be H m. = 2 7t1j h.. + rtrzl2
le is given char diameter of dome
= 27t1j fJ.. + 1t1j l2 [·: 1j = r
2]
= .3_3 x To cal height of the building = 1t1j (2 h.. + 12 )
D ame ce r 3 _
2- x H = 3.14x 2. 15 x (2x 3.8 + 3.04)
r = ______ = .=:.._
i [1] = 3.14 x 2.1 5 x 10.64= 71.83 m 2 [1 %]
2 2 :. Volume of building

- - -
=>
1
r = - H 1n = Volume of cylindrical shape of building
3 + Volume of conical shape of building

-
Let h m be the height of che cylinder.
h = H = 1t7i h.. + 1 7tr22hi = 1t1j2 '1..it + 1 1t1j2h2 [· . 'i = rz ]
2

)
r •

3 3
=> h = H - - = -2 H m
H
[1]

(
3 3 = 1t7i2 < h.. + 31 hi )
Volume of che air inside the building = Volume of air 2.15
inside the dome + Volume of air inside the cylinder = 3.14 x 2.15 x 2.15 3.8 + -3-

H�J n( �J( )
= �3 7tr3 + 7tr 2 h, where h =height of cylinder = 3. 14 x 2.15 x 2.15 x (3.8 + 0.7167)
=
2: = 3.14 x 2.15 x 2.15 x 4.5 167
+ [1] = 65.56 m 3 [1%]
= !._ There are two types ofice-cream container.

-
67.
8 1 7tH3 m3
Given, volume o f che air inside che building
3
= 67 1 m
21
8 1tH3 1408
-
? cm 7 cm
81 21 - ? cm -

=>
1408 x 81 x 7
21 x 8 x 22
=> H = 6m
Hence, height ofche building is 6 m. [1]
372 · Alllnone MATHEMATICS C lass 1 0th Term I

( 7 )2 7=
Volume of cylindrical shape container = nr 2 h So, cost of digging che wel l in the form
7 of cuboid at

2; ( H x 7 = ; �
(z)
the rare of � SO per m ·\ = � SO x = � 350

+
343
= 7t 2' x 4 7t cm3 [1] The volume of wel l in the form of cylinder [1]

+ - 1tr
Volume of cone with hemispherical base container = x
2 x x 7=
1; m l
= Volume of cone Volume of hemisphere
- :. Cose of digging chc well in chc form of cylinder at the

= HH m + �rr GJ
3
= 1tr 2 h
-
1 2
11
3 3 race o f ;,.' 50 per m ·\ = � 50 x
2
= � 275 [1]

= rr GJ G + �)
(ii)The decision caken by farmer shows e n m cal co o i
(1)
2.1
approach.
70. Given , height of cone = h1 = m
Height of cylinder = h2 = 3.5 m
and radius of cone = radius of cylinder = 2.8 m
cm 3
343
- 7t
Let I be slant height of cone.
=
8 [1 ]
Honesty
(iz) [1 ]
We know chat,
I = �h 12 r 2 +
......... , ........ .
I
I = �(2. 1 )2 )2
68. There arc rwo types of water tankers.
=:) + ( 2 .8
=:) I = 3.5 m (1 J

I +
:. Canvas required for one cent
1 m

+ -7
= Curved surface area of cylinder

-7
+ Curved sur face area of cone [1]
= 2 nrh2 nrl

-7
- 1 m --
22 22
= 2 x x 2 . 8 x 3.5 x 2.8 x 3.5
[1]

( �J
(z) Volume of cubic form tanker = (1 )3 = 1 m 3 [1 ] =3 x
22
x 2.8 x 3.5
Volume of cylindrical form tanker
; �
= 92.4 1.5 m 2
2
IT
= x I = x So, canvas required for 1 500 cents = 1 500 x 92 . 4 m 2
= 1 38600 m 2
= 0.7 9 m3 � o. 8 m 3 [1]
( it) The shopkeeper i s honest to their customers.

7
Therefore, cosc of the canvas = � 1 20 x 1 38600 (1)
[1 ] = � 1 6632000
69. Given, a farmer wants to dig a well either in che form of Hence, cost paid by each school
cuboid or in che form of cylinder. � 1 6632000 � 332640
The dimensions of cuboid are (I m x I m x m ) and che = =
diameter and heighc of che cylinder are 1 m and 7 m,

7 7
50
Values helping
��'.
,-.:. ->C-
respectively. Schools feel their responsibilities for
. .
r ( z) The volume of a well in the form of cuboid government in its efforts co face natural calamities like
' ,:,/ .:

t. .//� "\
= I x 1 x = m3 [1 ]
floods, etc. (1)
• •

Act1v1ty
Topic Covered Vol u m e of spherical body. Oral Questions
Objective To fin d rad i u s of a metallic sphere by 1. How will you find surface area of an object whkh is in
immersing it in water. the shape of a cone mounted on a hemisphere?
Skill Devel o pe d C reat ivity, analysis. 2. How will you find ·the curved surface area of an object
in the shape of cylinder having a conical depression?
Time Req u i red 15 m i n
3. How will you find the area of canvas needed to make a
Materia l s Req u i re d A small metallic sphere,
tent in shape of a cone mounted on a cylinder?
4. How will you find quantity of air contained in a shape
a graduated cyl inder (500 m l), 200 ml water.
Metho d
of a cylinder surmounted of a hemisphere?
1 . F i l l water i n g ra d uated cyli nder t o level o f 200 ml 5. A metallic sphere was melted and recasted into a
(200 cm 3 ). sphere with half the radius of the original sphere. How
many small spheres will be obtained?
500 6. What is the formula to find the volume of water flowing
400 out of pipe in given time?
300
200
1 00
True/False
1 . Curved surface area of figure = Curved surface area of
2. Now, put the meta l lic; ball in the cylinder. cylinder A + Total surface area of cylinder B

Note Choose the d i a m eter of ball such that it is


completely i mmersed i n water.
3. Now, note the l evel in the g raduated cylinder.
4. Also, d etermi n e the i n ner diameter of cylinder by
using a scale.

2. When one object is melted and casted into another


object, the volume of material of second object is
always less than that of first object.
3. Figure shows transparent plastic glass containing some
coloured water. This figure shows two frustums.

O bservati o n
Increase i n volume (volu me o f cylinder ABCD)
V = ... cm 3
Calcu latio n

4. In below figure, the plane through 8 and C (shown


Volume o f sphere = Volume o f cylinder ABCD
=> � 7tf3 = v => � 3.14 r3 = v
shaded) is p�rallel to the base. Then, !_ = � .!:....
x x

L H
3 3 =
v x 3 R
=> r3 = ---
A
4 x 3.14
v 3
H
x
r = 3• ---

4 x 3.14

[put the value of VJ


Allin.One MATHEMATICS Class 1 0th Term I I

G)
5 . I n below fig u re, s l a nt height of g iven fru stu m is 3 . Total surface a rea o f given fig u re = . . . ( . . . +...)
1 0 cm.
B ern ,

- � (. . . + . . . )
2
4. Volume of given figure =
1 8 cm - �-r---3
-

Fi ll i n the Blanks
1 . Surface a rea of g iven figure = Ttr ( . .. +...)

5. Volume of given figure = Ttr 2 ( . • . +...)

2. Cu rved su rface a rea of g iven figure = . . . (... + ... ) h

6. Volume of given figure = Ttr2 ( . . . + . . . )


h

-r-
---- h, ----

Answers

-
Ora l Questions
1 . Required area = Curved surface area of cone + Curved surface area of hemisphere
>
2. Required curved surface area = Curved surface area of cylinder + Curved surface area of cone
3. Required area = Curved surface area of cylinder + Curved surface area of cone
4. Required volume = Volume of cylinder + Volume of hemisphere
5. 8
6. V = Area of cross-section of pipe x Rate of flow x Time

True/Fa lse
1 . False 2. False 3. False 4. True
• 5. False

c Fill in the B l a n ks
+ 2h) 4. (2r + h)

• 1. l + 2 r 2. nr(l 3. 21tr(2r + h)

( "1 + ; )


..

� '
5. (� + ) h 6.
h
SU M MARY
• Whenever we h a ve to find the surface area of a complex solid, which is a combination of other solids, we add the
c u rved s u rface a re a of only visible portion of individual solids and for finding the volume of this solid, we add the
vol u me of i ndivi d u a l solid.
• If solid o r solids of one shape is converted i nto solid or solids of another shape, then
Tot a l vol u m e of the solids to be converted
= Tota l volume of the solids i nto which the given solids is (are) to be converted
• N um ber of s o l i d s of a g iven shape in which a given solid is to be converted
Total volume of the solid to be converted
Volume of one converted solid
• If a right c ircu l a r cone i s cut-off by a plane parallel to its base, then the portion of the cone between the cutting plane
a nd the b a se of the cone is called a frustum of the cone.
• The h e i g ht or t h ickness of a frustum is the perpendicular distance between its two circular bases.
• The s l a nt h e i g ht of a frustum of a right circular cone is the length of the line segment which is obtained by joining the
end p o i nts of two p a ra l lel radii, drawn in the same direction of the two circular bases.
• Let h be t h e height, I be the s lant height and 'i and r2 be the radii of the ends ('i > r2) of the frustum of a cone. Then,
(1) Volu m e of the frustum of the cone
= _! 1th (r,2 + Tf + 'iT2)
3
(ii) T h e c u rved s u rface a rea of the frustum of the cone
= 1t I (r, + r2 )
where, I = �h 2 + (r, r2) 2-

(iii) Tot a l s u rface a rea of the frustum of the cone


= 1t /( 'i +r2) + 1t'i2 + rtr{
where, I = �h 2 + (r, r2) 2 -

Let h be the height or depth, I be the slant height, 'i be the rad i us of to p which is open and r2 be the radius of the
b otto m which is c losed, of a bucket which is in the shape of a frustum of a right circular hello� cone.

Top (drin)

Area of the meta l sheet used for making the bucket


= O uter (or inner) curved surface area of frustum of a cone + Area of bottom
CHAPTE R EXE RCIS E
� I Very Short Answer Type Questions [ 1 Mark each]

1. Volume of cylinder with radius of base 7 cm is 6. If two solid hemisphere s of s a me base of radius r are
3 joined together along t heir bases, then find curved
1 02 cm • What will be the volume of a cone having
same radius of base and same height? surface area of this new solid .
7 . Find the number of cubes of side 2 cm which can be
2. If the volume of a cube is 2 1 6 cm 3 , then find its edge.
cut from a cube of side 6 cm. CBSE- 201 1
3. The radius of a wire is decreased to one-third . If the 8 . A spherical tank i s 2 l cm i n diameter. Find its surface
volume is same, then find the length of the wire. area .
CBSE 20 1 2
9 . Volumes of two spheres are in the ratio 64 : 27, find
4. Volume o f two spheres are i n the ratio 1 2 5 : 64. Find the ratio of their su rface area s . NCERT Exemplar
the ratio of their surface areas .
1 0 . Volume of cylinder is 1 078 m 3 and height of the
5 . During conversion of a solid from one shape to cylinder is equal to radius of cylinder. Find the height
another, find the volume of the new shape. and radius of cylinder.

�I Short Answer Type I Questions [ 2 Ma rks each]


1 1 . How many spherical bullets can be made out of a 1 9 . Three cubes each o f side 1 5 c m are joined end - to-end.
solid cube of lead whose edge measures 44 cm, each Find the total s urface area of the res ulting cuboid.
bullet being 4 cm in diameter? CBSE 201 1
1 2 . A metallic sheet is of the rectangular shape with 2 0 . The diagonal o f a cube is 2 7 .J3m. Find its surface area.
dimensions 48 cm x 36 cm . From each one of its
corners, a square of 8 cm i s cut-off. An open box is 2 1 . If the radius of a sphere is doubled . What is the ratio
made of the remaining sheet. Find the volume of of the volume of the first sphere to that of second?
the box .
2 2 . A copper sphere of radius 3 cm is bea ten and drawn
1 3 . The diameter o f a garden roller is 1 .4 m and i t i s 2 m into a wire of diameter 0 . 2 cm. Find the length of the
long. How much area will it cover in 5 revolutions? wire ( in metres ) . CCE 2013
1 4. If the radii of the ends of bucket 45 cm high are 28 cm
2 3 . Three cubes of a metal whose edges are in the
and 7 cm, then determine the whole surface area.
ratio 3 : 4 : 5 are melted and converted into a single
1 5 . Find the lateral surface area of the frustum of cone, if cube whose edge is 1 2 cm . Find the edges of the
the radii of the circular ends of height 12 cm are three cubes. CBSE 2012
30 cm and 14 cm. [ take, 7t = 314]
2 4 . The radius and height of a metallic cylinder are 24 cm
1 6 . How many planks each of which is 2 m long, 2.5 cm and 1 08 cm, respectively. It is melted and recast into a
broad and 4 cm thick can be cut-off from a wooden sphere. Find the radius of the sphere.
block 6 m long, 1 5 cm broad and 40 cm thick?
l 7. The ra dius and height of a right circular cone are in
2 5 . A solid metallic sphere of radius 1 0. 5 cm is melted

the ra tio of 5 : 1 2 and its volume is 25 1 2 cm • Find the


and recas t into a number of smaller cones, each of
3
s lan t h eigh t of the co ne.
radius 3 . 5 cm and height 3 cm. Find the number of
cone p 0 formed . NCERT Exemplar

1 8. If a marble of ra dius 2 . 1 cm is put into a cylindrical 26. A cone of radius 8 cm and height 1 2 cm is divided into
cup full of wa rer of ra d ius 5 cm and hei ght 6 cm, then
how much wat er fl o ws out of the cylindrical cup?
two parts by drawing a plane through the mid-point
of its axis and parallel to its base. Compare the
NCERT Exemplar volumes of the two part s . .NCERT Exemplar
Surface a reas a n d Volu mes : 377
,I


�l Short A nswer Typ e I I Questi ons [3 Marks each ]

27 . A t o y i s i n t h e s h a pe o f a r i g h t c i rc u l a r cyl inder
with a hem i s p h e re o n o n e e n d a n d a cone on the
34 . A wooden article as shown in the
figure wa s made from a cylinder I Lm
other. The h e i g h t a n d ra d i u s o f t he cyl indrical by scooping ou t a hemisphere (.)
0
l
-

I
part a rc 1 3 c m a ml 5 cm, re s pec t i vely. The radii from one end and a cone from the
o f t h e h e m i s p h e rica l a n d co n ic a l parts arc the ot her end. Find the total su rface
same as t h a t o f t h e cy l i n d rica l p a r t . Calculate area of the article. CBSE 20 1 2
1- 6 cm -1
the su rfa ce a rea o f the toy, i f height of the
con ica l pa rt i s 1 2 c m .
35. A spherica l copper ball of diameter 2 1 cm is melted
28. The d ia m e t e r o f t h e ro l l e r 1 2 0 cm l o n g is 8 4 c m . and recas t into cubes, each of side 1 .5 cm. Find the
I f i t t a ke s 5 00 co m p l e t e revo l u t io n s to level a nu mber of cubes so formed and the copper left .
playgro u n d , t h e n fi n d t h e cos t of levell ing at the CCE 20 13
rate of 3 0 p a i s c per m 1 .
36. A canal is 300 cm wide and 1 20 m deep. The water in
29 . Wa ter in a c a n a l , 3 0 m w i d e and 1 2 m deep is the ca nal is flowing with a speed of 20 km/h. How
flowing with a vel o c i t y o f l O km/h . How m uch much area wil l it irrigate in 20 min, if. 8 cm of
are a w i l l i t i r r i g a t e in 30 m i n , i f 8 c m o f s tand ing s tanding water is desired? NCERT Exemplar
water i s · req u i re d for irrig a t io n ? 37. A milk conta iner of height 1 6 cm is made of metal
30. A s o l i d i s i n t h e fo rm o f a cylinder with sheet in t he form of a frustum of cone with radii of i t s
h e mi s p h er i c a l e nd s . The t o t a l heig h t o f the solid lower a n d upper ends as 8 c m a n d 20 cm, respectively.
is 1 9 cm and the d ia me te r of cylinder i s 7 cm. Find the cost of milk at the rate of �22 per which L,
Find the vol u m e of s o l i d . the container can hold. NCERT Exemplar
31. I f t h e r a d i i o f t h e e n d s o f a b u cket 1 5 cm high 3 8. Two cones with same base radius 8 cm and height
are 28 cm a n d 2 1 cm re s pe c tively, t hen find the 1 5 cm are joined together along their bases . Find the
capacity of b u c ke t . surface area of the shape so formed.
3 2. I f a , b a n d c a rc r es pect ively length, breadth NCERT Exemplar
and height of a cuboid such that
39
=1 0 cm and a + b + c = 40 cm •
2 2 2 2 • A hollow cone is cut by a plane parallel to the base
a + b + c and u pper part is removed to make a Turkish cap. If
Find t h e s ur fa ce a re a o f t h e cuboid . CCE 20 13 24
the curved surface area of t he remainder is of the
33. A circ u s tent is in t h e s hape o f a cylinder 25
surmo u n t e d b y a c o n i c a l roof. I f the common
curved s urface area of the whole cone, then find the
d iameter is 56 m , t he h e i g h t of the cylindrical
ratio of the line segment s into which the cone's
portion is 6 m a n d the h e i g h t o f the roof from
height is d ivided by the plane from which the cut is
the grou n d is 30 m, t h e n fi n d the area of the made . CCE 20 15
canvas u s e d for the t en t . CCE 20 13

�I Long A nswer Typ e Questions [4 Marks each]

40. A n a g r i cu l t u re fie l d i s t h e for m o f a rectangle of 44. Half of a large cylindrical tank open at the top is filled ' ' .
16.1.�•
l e n g t h 20 m , w i d t h 14 m. A 10 m deep well o f w i t h water an identical heavy spherical balls are to be
d i a m e t e r 7 m i s d u g i n a c o r n e r o f the field and dropped into the tank wit hout spilling water out. If
. ·,�
""/
the s oil t a ke n o u t of t h e wel l i s s pread evenly the radius and the height of the tank are equal and
over t h e rem a i n i n g p a r t of the field . Find the each is four times the radius of a ball, what is the
maximum number of ball s that can be dropped? t '
rise in i t s l eve l . "�-=·

41. 45. A mason con s t ructs a wall of di m ension s �l }


u
Wat e r i s flowing a t t he r a t e o f 3 km/h t hro ugh a
circul a r p i p e o f 2 0 cm i n ternal d i a meter into a 270 cm x 300 cm x 3 5 0 cm with bricks each of size
circu l a r c i s t ern of d ia meter 1 0 m and depth 2 m. 22 . 5 cm x 1 1 . 2 5 cm x 8 . 75 cm a nd i t is as s um ed
1 .
I n how m u c h t ime, w i l l the cis tern be fil led? t l1at space is covered by t he mortar. Then , find th e
B
42.
wa l l .
A t a n k is 2 2 5 m long, 1 62 m b roa d . With wha t
number of bricks •u sed to construct the
velocity p e r s econd m u s t water flow into i t
through a n a pe r t u r e 60 cm b y 45 c m that the 46. The radius of the internal and external surfa ce s of a
level may b e r a i s e d by 20 cm in 5 h? hollow sph e r ic al shell are 3 c m a n d 5 cm, respect ively.
I f i t is melted and reca s t int o a s olid cyl in der of height

F:
43. The d i fference b e tween the inner and outer
..
s urfa c e s of a cyl inder 14 cm long is 88 cm • If t he
2
2 � cm. Then, find the dia mc c cr of t hl' cylinder. : :::..: :...

volume o f t h e cyl inder is 1 76 c m 3 , t hen fin d its 3


inner and o u t er rad i i .
. Allln one MATH EMATICS Class 1 0th Term 11
i
� 37 8 :�
r.--- - -;i
47. fHO� The height of a con� is 30 cm. A small cone is 48 . !t_QJ_�The length, brea d t h a nd heigh t of a rectangular
cut-off at the top by a plane parallel to the base. If its paralklopiped a rc in t h e ra t io 6 : 3 : I . If the surface
area of a cube is equal to the surface area of this
volume is -1- of the volume of the given cone, at what parallelopiped, t hen what is the ra tio of the volume of
27
height above the base is the section mode? the cube of the volume of t he pa rallelopiped?

�l Value Based Questions (VBQs) [ 4 M a rks each]

4 9 . In a flood hitted area, the volunteers of NSS erected a 5 1 . 50 students of class X planned a visit to an old age
conical tent made of tarpaulin. The vertical height of home and to spend the whole day with its inmates.
the conical tent is 4 m and the base diameter is 6 m. If Each one prepa red a cyl ind rica l flower vase using
the width of tarpaulin is 1 . 5 m, then cardboard to gift t he inma tes. The radius of cylinder is
( i ) fin d the length of the tarpaulin used, assuming 4.2 cm and the h eig h t is 1 1 .2 cm.
tha t 1 0% ex tra material is required for stitching (i) What is t he a m o u n t s p e n t for purchasing the
margins and wa s tage in cutting. cardboa rd a t t h e ra t l' o f � 2 0 per 1 00 2?
[ take, 7t = 3. 1 4 ) ( ii ) Which values a rc d epicted by the s t udents?
( ii ) which values are depicted by the volunteers?
5 2 . Due to sudden flood s, some welfare associations
5 0 . The patients in a hospital are given soup daily in a jointly requested the govern ment to get 1 00 tents
cylindrical bowl of diameter 7 cm. On a particular day, fixed immed iately and offered to contribute 50% of
the girls of kanya Mahavidyalaya decided to cook the
soup for the patients. the cost. I f the lower part of eac h ten t is of the form of
a cylinder at diameter 4.2 m and heigh t 4m with the
(i ) If they fil l the bowl with soup to a height of conical upper part of same d iameter but of height
5 cm , then how much soup is to be cooked for 2 . 8 m and the canvas to be used cost s � 1 00 per m2,
300 patien ts?
then find the amount, the associa tions will have to
( ii ) Which value is depicted by the girls? pay. What values arc shown by t hese association?
[ take, 7t =22 I 7 ]

Answers
1 . 3 4 cm3 2. 6 cm 3. 9 times 4. 25 : 1 6
5. Remain unaltered 6. 41tr 2 7. 27 8. 1 386 cm2
9. 1 6 : 9 1 0. 7, 7 1 1 . 254 1 bullets 1 2. 5 1 20 cma
1 3. 44 m2 1 4. 561 5.5 cm2 1 5. 2763.2 cm2 1 6. 1 80
1 7. 26 cm 1 8. 38 . 8 cm3 1 9. 3 1 50 cm2 20. 4374 cm2
21 . 1 : 8 22. 36 m 23. 6 cm, 8 cm and 10 cm 24. 36
25. 126 26. 1 : 7 27. 770 cm2
28. � 475.20 29. 2.25 km2 30. 64 1 .66 cm3 31 . 28.49 L

32. 60 cm2 33. 4301 .44 m2 3 4. 316.9 4 cm2 3 5 . 1 437, 1 89 cm3


36. 30 hec 37. f 230.12 38. 855 cm2 39. 1 : 4
40. 1 .594 m 41 . 1 h 40 min 42. 1.5 m/s 43. 1 . 5 cm, 2.5 cm
44. 24 45. 1 1200 46. 14 cm 47. 20 c m
48. 3:2 49. (i) 34.54 m (ii) Social value, helpfulness
50. (i) 57. 750 h (ii) Social collession
51 . (i) f 3 5 1 1 .20 (ii) Social value, Cooperation, Caring for old people.
52. f 3 79500
Val ues Ass oci ation feel their responsibilities for helping government in its effort� to face natural calamities like floods, etc.
CHALLE N G E RS*
1 . A shuttlecock, u s e d for p l ayi n g badmi nton , has the shape of a frustum of a cone mounted on a hemisphere. The external
d iameters of t h e fru s t u m are 5 cm and 2 cm and height of the entire shuttlecock is 7 cm. Find its external surface area.

2. A sphere of rad i u s 'a ' u n i t is i m m ersed completely in water contained in a right circular cone of semi-vertical angle 30 ° and
water is drain e d off from the cone till its surface touches the sphere. Find the volume of water remaining in the cone.

3. The base of a rig ht prism i s a right angled triangle. The measure of the base of the right angled triangle is 3 m and its height 4 m.
If the heig ht o f t h e p ri s m i s 7 m, then find
(1) the n u m ber of e d g e s of the prism.
{ii) t h e vol u m e of the pri s m .
(iii) the total s u rf ace a r e a of the prism.

4. A cone m a d e of paper has height 3 h and vertical angle 2 a . It contains two other cones of height 2 h and h and vertical angles
4 a and 6 a , respectively. F i n d the ratif) of the two volumes between the cones.

5. A golf ball has d i ameter e q ual to 4 . 1 cm. Its surface has 1 50 dimples each of radius 2 mm. Calculate total surface area which is
exposed to the s u rrou n d i n g s , assuming that the dimples are hemispherical.

6. A cyli ndrical tube i s s h ow n below.


l � 4 cm � I

When w e rotate a tube i n c lockwise direction , then find the spaced occupied by this figure.

Cr
7. Find the s pace accepted by the figure, when we rotate the figure about th e line EF.

E
� 5 cm --+-
8

1
6.S cm 7 cm

'i

* These q u es t i o n s m ay b e or may not be as ked in t h e exa m ina tion , have been g iven just fo r a dd ition al practice.
Note Solutions to t h es e q u estions a re ava ila ble a t p a g e n u m ber 397-398.
S O LUTI O N S TO
CHALLE NGE RS
Challe n ge rs
S O LUTI O N S TO

x + IO - Fw
Chapter 1 Quadratic Equations 4. Given equation is q
x2 + px + = 0.
1 . Given cq uation is .J
x + 10 = 5 ... (i) -2 -15. x 17, p = 17,
When we cake che coefficient of as i.e.
che roocs are and
then

=> x�G = 5 => x + 4 =5CF+J0l x2 + 17x + =0 -2


Thus, we can say
q
is a root of the equation

.. (-2)2 + 17 (-2) + =0
x q
=> q
4 -34 + =0
On squ;iring both sides, we gee
x2 + 16 + Bx = 25(x + 10) => q =30
=> x2 + Bx - 25x - 250 + 16 =0 Clearly, che new quadratic equation will be
=> x2 -17 x - 234 = 0 x2 + 13x + 30 = 0
=> x2 -26x + 9x -234 =0 x2 + IOx + 3x + 30 = 0
=>=> x(x - 26) + 9(x - 26) =0 x(x + 10)+3(x + 10) =O
=> x + x9(x== +026or9)(xorx --9-2626)= =00 x + I(x0 =+0IO)(x + 3) =O
or x + 3 = 0
=> x =-10
On puccing x = -9 in Eq. (i), we gee or x =-3
Given, y = x2 -3x + 2 and y = 0
.J-9 + I 0 - 6 = 5 => I - �1 = 5 -5 = 5
5. (z)
=>
.J-9 + 10 x2 -3x + 2 =0
which is not true. => x2 -2x-x+2 =0
Hence, extraneous root of given equation is -9. =>
Given equations of line and curve are
=>
x(x -( 2) -l(x -2)-1)
=0
=0
2.
y = kx -3 and = 2x2 + 7 y
x -2)(x x=2
=>
Now, fo r point of intersection consider, 2x2 + 7 = -3 kx
=> 2x2 - kx + I 0 = 0 or x-= I
Hence, roots are I and 2.
On comparing wich ax2 +bx+ c = 0, we gee (ii) When we rocace che axes ac x
a = 2, b = - k and c =I 0 ·

Since, the line is a tangent to che curve, so the discriminant


an angle of 90° in 2
anti-clockwise direction,
D =0. then the new graph is same Y Y'
i.e. b2 - 4ac =0 as shown alongside.
2 1
-1 -1 -2
=> ( -k )2 - 4 2 10 = 0 k2 = 80 => -2
X'
x x Here, we see that graph is
=> k = 4.JS ±
shown on negative of Y-axis.
y = x2 -3x x -y in the original
So,. we replace y by x and by
3. We +
have, x2 kx + 1 = 0 equation + 2.
O n comparing wich ax2 +bx+ c = 0 , we gee Now, we gee x = (-y)2 -3(-y) + 2
a
For linear faccors,
= b = k andD �c 0= 1
l,
=> x =/ 3 + .Y + 2
=> b2 - 4ac �O 6. Given equation is ax 2 +bx+ = O.
c

=> k 2 - 4 I 1 �0
x x a a
Since, sin and cos a re the roots of
the equation.
-b
:=::> (k2 - 22) � 0 : . Sum of che roo ts, sina+ =
cos a -
. . . (i)
==>
00
(k - 2)(k + 2) � o 00 .
a

-2 0 2
and product of the roots, sm a cos a ::: -
. . . (ii)
a
:.k � 2 and k � -2 Hence proved.
A lllnone MATH EMATICS Class 1 0th Term 11

(-:J
On squaring both sides of Eq. (i), we get x = I or 3
(sin a + cos a ) 2 = Also, another given equation is 2f2 - 7y + 3 =O
=> 2y2 - 6y - y + 3 = 0
. bl
==> sm l a. + cos l a. + 2 sm a. cos a. = 2
. => 2y(y - 3) - l(y - 3) = O
b2
a => (2y - l )(y - 3) = O
==> I + 2sin a cos a = 2 [·: sin l 9 + cosl 0 = 1]
=>
1
y = - or 3

(�)
a

:=:> 2sin a cos a = 2bl - 1 2 y


3 D
a Now, let us plot these roots on
the axes, which arc shown 2
=> 2x = l -l a l
b [from Eq. (ii)] alongside. The smallest triangle c1 A
a a formed by joining these points X' 0 •• •

B X
=> 2a = b2 - al
c
and origin is OA C. Y'
2 3

bl = a l + 2a c Hence proved.
:. Area of .6 0A C
7. Given equation of curve is y = x l + x - 12 = ..!._ x OC OA = ..!._ x ..!._ x 1 = .!_ sq unit
2 2 2 4
x

On comparing with y = ax 2 + bx + we get c , 9. Lee the total number of birds be x. Then, number of birds
a =l >0 moving about locus plants = .:_ and number of birds
So, it opens upwards. 4
To draw its graph, we need some different values of y moving on a hill = � + � + 7E.
corresponding to different values of x. 9 4

( )
x -4 -2 -1 0 1 2 3 Given, number of birds in vakula tree = 56
y 0 -1 0 -1 2 -1 2 -1 0 -6 0 According to the given condition,
.:. + .:. + � + 7fx + 56 = x
y 4 9 4
8
=> x -� - � - � - 7Fx - 56 = 0
6 4 9 4
4
-9x
36x - 4x -9x _ 7Fx _ 56 = O
36
B (-4, 0)
2 14x - 7Fx -56 = 0 => 7x - 7fx -56 = O
X'
-8 -6
x 36 18
.!... - Fx - 8 = 0 [dividing both sides by 7)
-1 0 4 6 8 1 0 12
18
=> x -1sf; -144 = O
Puc /; = y, then above equation becomes
y2 -18y -144 = 0
=> y2 - 24y + 6y -144 = 0
=> y(y - 24) + 6(y - 24) = 0
=> � - 24)(y + 6) = 0 => y = 24 or - 6
But y :;t; -6 .Jx = y
as

We know that, abscissa of point of intersection of curve .. y = 24 => ,J; = 24 => x = 576
and X-axis, is the root of equation y = 0. Here, we see that Hence, total number of birds is 576.
graph intersects the X-axis at two points A and C.
:. The roots of the given curve are 3 and -4.
Now, length of A C = 4 + 3 = 7 and length of OB = 12 Chapter 2 Arithmetic Progression
. ·. Area of M.BC = ..!._2 x A C x OB = ..!._2 x 7x 12 1 . We know chat, the condition for common difference of
= 42 sq unirs two terms of an AP is constant.
8. Given equarion is x 2 - 4x + 3 = 0 Lee linear expression be an = an + b
where a and b are constants.
=> x 2 - 3x - x + 3 = 0 [by factorisation] Now, an - an _ = ( an + b) - [ a(n - 1) + b]
1

=> x(x - 3) - I(x - 3) = O = an + b - a n + a - b = a


=> ( x - l ) ( x - 3) = 0 which is constant.
Solutions to Challengers /385 /

2. We assume chac, b + c, c + a and a + b are in AP. :.Area of circle = 1tr 2 = 3.1 4 x (2) 2 = 314 x 4
:.(c + a ) - (b + c ) = (a + b ) - (c + a ) = 1 2.56 sq units
=> a - b = b - c :::::::> 2b = a + c 6. The sum of given sequence is
=> a, b and c arc in AP. 1 · 1
-- + -- + -- + . . . .
1
Hence, our assum ption is rruc. lx2 2x3 3x 4
3. Herc, we sec char rhc inrcgcr values of x lying on rhe line I
from A to B , arc - 2, - 1, ... , 7, 8. Here, n ch term is an 1 => a = - - --
1
n n n+1
n(n + 1)
Herc, 11 = - 2 , ti = - I - (-2 ) = 1 and I = 8
I = a11 = a + (11 - I )d
On purring n = 1, 2, 3, . . . , n respectively, we ger
1 1
8 = - 2 + (11 - 1 )1 at = - - -
=> 8 = - 2 + 11 - l :::::::> n = i l 1 2
1 I
51 1 = - [ 2 x ( - 2) + (1 1 - l) x l ]
11
Now, z= -
'1 1 3
2
'1J = -
11 11 I I
= - [ - 4 + 1 0] = - x 6 = 33 --
2 2 3 4
4. Lee S = ( 4 5 2 2 2
- 43 ) + (44 - 42 2 ) + (43 2 - 4 1 2 )
+ (42 2 - 40 2 ) + ... upco 1 5 cerms an = -1 - 1 -­
n n+1

(HJ (� i) (; � )
= (45 + 43 ) ( 4 5 - 43 ) + (44 + 42) (44 - 42) On adding all terms, we gee
+ ( 43 + 4 1 ) ( 43 - 4 1) + (42 + 40) (42 - 40 ) + = a1 + a2 + . . . + an
S
. . . upto 1 5 terms
[·: 2
a - b2 = (a - b) (a + b)] = + - + . . .+ -
n 1
= (45 + 43)2 + ( 44 + 42)2 + (43 + 4 1)2
1 1 n + l - 1 = -- n

[;
+ ( 42 + 40)2 + ... upto l S terms = - - --
= 2 [{ 45 + 44 + 43 + ... upto 1 5 terms} 1 n+l n+l n+l
+ { 43 + 42 + 4 1 + . .. upco 1 5 terms} ] 7. We know that,

i { 2 x 43 (1 5 - 1) (- l )JJ
= 2 x 1 { 2 x 45 + (1 5 - 1) (- 1)) Area of circle = 1t x (Radius) 2
Let area of che larger circle be

[·: S,, i { 2a + (n - l)d)J


I 22 1 98 [·: R = 3] . . . ( i)
+ + A1 + A2 = 1tR 2 = x 3 2 =
7 7
Since, A1 and A2 , Ai + A2 are in AP.

[i ; m1J
=
.. 2A2 = Ai + (A1 + Ai )
A2 = 2A1
Ai + Ai = Ai + 2A1 [adding both side
1 1 =>
=z { 76 1 + s Ai ]
=>
=> A1 + Ai =3A1
= 1 5 ( 76 + 72) = 1 5 x 1 48 = 2220
=> 1 98
= 3A1 [from Eq. (i)]
Hence, che sum of the given series is 2220.
7
5. Given point is A (3, 4 ). The x-coordinate of A is 3. 2 66
=>
66
Since, che com mon difference is 2. Ai = - => rtr = - 7
7

( 1 , 4)
:.x-coordinates of che point before A is (3 - 2), i.e. 1 and 22 66
=:>
2
- x r =-
after A is (3 + 2 ), i.e. 5. 7
=>
7

� D�B(S,4)
D(l, 4) and B(5, 4).
=> r = 3 => r = J3
2 [·: radius cannot be negative]
The y-coordinate of A is 4. y


c (3,6) 8. Lee the first term and common difference of an AP be a
Since, che com mon 6
difference is 2. and d, respectively.
Given, q + b5 + q 0 + q 5 � b20 + b24 = 300

4 5
:. y-coordinaces of che point
:.a + (a + 4d) + (a + 9d) + (a + 1 4d)
E
above A is (4 + 2) , i.e. 6 and 2
+ (a + 1 9d) + (a + 23d) = 30 0[·: bn == a + ( n - l) ]
below A is ( 4 - 2) i.e. 2. d
' -'--+--+--+--1.;,.._+- X
(3 , 2)
=> C(3, 6) and £(3, 2 ). X!+- => 6a + 69d = 300 � 2a + 23 d = l 00
A circle shown in che figure
1 2 3
[dividing by 3 1 . . . (i)
y,
Now, S.,4 d] = 1 2 [ 2a + 23 d 1
having cencre A(3,4) and 24
passing through che poinrs A. B, C and D. - = - [ 2a + ( 24 - l }
[from Eq. (i)}
2
So, radius of circle is, AB = 5 - 3 = 2 units = 1 2 x I oo == 1 2 00
\ 3 86 ' Allinone MATH EMATICS Class 1 0th Term II

9. p
n
Q R
I I I I I I I I I I I I I I I I I I I I I I I
n
2. Given,

LA TC = 36° and L.ACT = 48°.
Jo i n OC .

36"
3 21 1 23
Let there are ( 2n + 1) stones . Middle stone is at Q. Then,
there are n scones on one side of the middle stone and 11 a o> ,
scones on other side of che middle stone.
Lee the man stares from P. T
He picks the stone and travels to Q.
. · . Distance covered = 1 0 m
We know c h a t , rad i us is pcrpcn<l icular co che cangem.
He comes back to pick the next stone and goes back co P.
.. L.OCT = 90°
Then, distance covered Now, L.Olil = L.OCT - L.ACT = 90° - 48° = 42°
0 x (n - 1) + 1 0 x (n - 1) = 2 x 10 (11 - 1) m . and OA = OB = OC [ radii ofrhe same cirde]
=> LOAC = L OCA = 42°
=I
Similarly, distance covered to pick and place that stone
at Q = 2 x 1 0 (n - 2) m, and so on. [angles opposite ro che equal sides are equal]
In MCT,
Exterior LCAB = L.A CT + LCTA = 48° + 36° =84°
Now, total distance covered,
51 = 1 0 x n + 2 x lO(n - 1) + 2 x IO(n - 2) + . . . Now, LOAB = LCAB - LOAC= B4° - 42° = 42°
upto n'terms
=> LOBA = LOAB = 42°
51 = [2 x 1 0n + 2 x IO(n - 1) + 2 x lO(n - 2) + ...

[% J
upro n rerms] - 10 n In isosceles MOB,
= 20[n + (n - 1) + (n - 2) + ...+ upto n rerms]- I On LOBA + LOAB + LAOB = 1 80° ,

[i (n + ]
[by angle sum property of a triangle]
= 20 x { 2 x n + (n - I ) (- l)) - 1on => 42° + 42° + LAOB = 1 B0°
::::> LAOB = 1 B0° - B4 ° = 96°
= 20 x I ) - I On Hence, che angle subtended by AB a t centre i s 96°.
3. Given, LBAC = 66° and I is B
Now, to pick the srone at R, he will walk from Q to R and che incenrre of MBC.
then will come back ro Q. In order ro pick all the scones, Join BD and BI.
rota! distance covered will be 51 + Distance covered from
Q ro R = 51 + 1 On Al is rhe bisector of L.BAC
But it is given, total distance covered = 3 km :. LDAC = 33°

[ J
.. 51 -+' 51 + IOn =3000 Now, LDBC = LDAC
=:> 251 + 1 On = 3000 • [·: angle i n che same segmenr
c
of a circle are equal]
� 2 20 x
n(n + I) - I On + IOn = 3000 ::::> L DBC = 33°
2 Again, LBAC = 66 ° and LACB = B0° [given]
:::::> 2
20n + 20n - 20n + l0n =3000 Now, in MBC, LABC + LACB + LBAC = 1 80°
=:> 20n 2 + 10n -3000 = 0 [by angle sum property of a triangle]
=:> 2n 2 + n - 300 = 0 [dividing by 1 O] L.ABC = 180° - (66° + 80°) = 1 80° - 146° = 34°
·: BI is che bisector of LABC.
=:> 2n2 + 25n - 24n + 300 =0 .. LIBC = 1 7°
=:> n(2n + 25) -12(2n + 25) = 0 Now, LDBI = LDBC + L/BC = 33° + 17° = 50°
=:> (2n + 25) (n - 12) =0 Again, LADB = LACB
- 25 [·: angles in same segment of a circle are equal]
=:> n = 1 2,
2 =::> n = 12 [since, n cannot be negative] Bue
..
LACB = 80° [given)
:.Total number of stones = 2n + 1 = 2 x 12 + 1 = 25 LADB = B0°
Now, i n fl. IBD, LBID = 1 B0° - ( LDBI + LIDB)
Ch apter 3 Circles = 1 80° - (50° + LADB)
= 1 B0° - (50° + B0°) = 50°
1. Let radii of rhe circles with centres P, Q and R are p, q and Hence, LDBC = 33°, LIBC = 17°, L.BID = 50°
r, respecrively. 4. Given, two tangenrs are drawn from an external point A
Then, PQ = p - q = lO [given] . . . (i) co rhe circle wirh cenrre 0.
PR = p - r = 8 [given] . . . (ii)
an d QR = q + r = 12 [given] . . . (iii)
On ad di ng Eq s . (ii) and (iii), we ger
p + q == 20 . . . {iv)
On adding Eqs. (i) a n d (iv) , we gee 2 p == 30
Hence, di ameter of the Iargesr circle == 2p = 30 cm
Solutions to Challengers J 387 /
Tangent PA =PB [·: tangents codrawn from anequal
external point
equals co BC
Scm.OPis drawnAP=>ac aLOPA
point R and radius of circle
OP =QP a ci r cl e are in l
[common sides]e ngth]
..l
circle=is90°theperpendi
[tangent ac any poincradiusof athrough poi n t cofulacontact]
r co the .Then,
. llOLAPO :: llOBP
AP = LBPO [ b y SSS congruence
[ b y CP rulCT]e]
Now, in right angl2 ed MPO, => OPlieissonchethebisector bis of of the angle between / and / •
ector LAPE.

OA = OP22 + PA2 [by Pythagoras theorem] => 0


Sibismector
ilarly, ofO' ltheies onanglthee 1 2
(13) 2 = 5 + PA 2 [QA =13 cm, given]
PA 2 =169 -25 between
Hence, andlocus/2 • of centre
/1
the
=> PA 2 =144 =122 => PA =12 cm oftwociinrcltersecti
es whingchlinescouches
Now, perimeter of �BC =(AB = AB ++BCBR)++CA(RC + CA)
/2 is the. bisector line of/1 and che angle between /1 and'/2•
[·.· BR = BP, RC==ABCQ;poi+ tangents
BP + CQfrom+ CAinternal
n t to a ci r cl e are equal ]
7. Joint AO, OC, O'D and O'B. c

= AP + AQ= 2AP = 12) = 24 cm


·: AP =2(A Q, tangentscofroma cirinclternal
[ poin]t
Hence, e are equal
5. In fl.OAPtheandperimeter
ll.OCP,of MBC is 24 cm. Now, in llEO'D andO'MO' D B, B [radii ofthe circle]
=O'
AP = PC [·: P is an external point of a circle]
c O'EDE =EB
=O'E [common side]
[since, tangentsto drawn the ci r from
cl e are anequal
external
i n le point
ngth]
OP
A
== OP [common sirdcles]e] => O'E illEO'
· ·

=>
LO' EDD =llEO'
=:
LO' BB [by SSS congruenceCPCT]
E
s the angle bisector of LDEB. [by
rule]
. . . (i)
OA OC
fl. OAP = fl.LCPO
=:
[ s ame radi
OCP [by SSS congruent rule]u s of a ci Simi l a rl y , OE i s the angl e bi secto r of LAEC.
=>But LAPC LAPO [byCPCT] Now, in quadrilateral LO' DEBO'
E = LO', B. E = 90°
=90° [given] D
[since.CE Dis ithes a tangent and
coe.theO'DcircleCED]
90°
leSinimce,plABies chat,
LAP0 = -2 =45° O' D= 180°
radi u s, i. ..l

In right angled fl.OAP,APAlso, LO' PD = 45° => LO'


LDEB DE++LDO' LO'BBE=180° ... (ii)
coc45° = OP 1 = QAAP => DEBO' i
is a straight+linLDEBs a cycl
e. =180° i c quadri la teral.
=> AP = OA = 2 [·: radius = � = 2, given] .. LAED +LAED
=> 180° - LDO' B ==180°LDO'B [from Eq.. ..(i(iiii)])
: Diameterof theof alalargerrgercicircl
·:.Radius rcleeOA= 4=cm2 cm diameter
=>
Siminla,rlfrom y, Eq. (ii) , LALDEB LAEDED == 180° LAOC- LDO'B ... (iv)
[. . radius=--- ] Agai
2 => _!_ LDEB 90° - _!_ LDO'B
=
Similarly, iPD n ll.O' PD, 2 2
coc 45 = O ' D = -1PD- [ ·: radius, D =I cm, given]
0
0'
=> LDEO' = 90° .!_ LDO'B _
[dividing both sides 2] by

.. . (v)
=>Now, PD
AD
=1 cm
= AP + PD = 2 + 1 =3 cm [since, O'E is the angle 2
bisecror of LDEB i . e.
Since, length of cransverse axis are equal. _!_ LDEB = LDEO' 1
2
.. AD = BC =3cm
6. Let and be the two intersecting lines, which intersect
/1 12 Similarly, LAEC == 1 80° - LA OC

at pointl1 andP and 0 be the centre of the circle, which couches


both /2 •
=>
2
.!_ LAEC = 900 _!_2 LAOC
-

In fl.OAP [dividing both sides by 2]


QA =andOBll.OBP, we have [radii of the same circle]
\ 388 Allinone MATHEMATICS C lass 1 0th Term II

=> LAEO = 90° - .!_2 LAOC . . . (vi) 50 A


y

45 F
[since, 0£ is the angle bisecror 40
35

( � )( � )
of LAEC i.e. .!_ LAEC = LAEO] 30 0
2 25
Now, LAED + LDEO' + LAEO 20
15
= LAED + 90° - LDO'B + 90° - LAOC 10
5
X' 05 1 0 1 5 20 25 30 35 40 45 50 x
.!_2 ( LDO'B + LAOC ) 5
1 0 al \Jc
= LAED + 1 80° _

= LAED + 1 80° .!_ ( LAED + LAED)


15
_
20
Y'
2
[from Eqs. (iii) and (iv)]
AB
= LAED + 1 80° .!_ ( 2 x LAED)
_
(1) In right angled MBC, tan 60° =
2 BC
= LAED + 1 80° - LAED = 1 80° .J3 = �
:. LAEO + LAED + LDEO' = 1 80° BC
So, OEO' is a straight line. BC = 60 x
.J3
J3
=>
Hence, 0, E and O' are collinear. Hence proved. .J3
= 20 J3 m
6o J3
BC =
3
C h apter 5 Some Applications Hence, horizontal distance between BA and CD is
20.J3m.
of Trigonometry (ii) In right angled MED, can 30° = AE
1 . Let OC = h m be the height of
ED
I AE
J3 20 J3
the rower at the centre of the =
circular park.
Given, AB = a => AE = 20 m
and LAOB = 60°. Now, CD = BE = AB - AE = 60 - 20 = 40 m
Also , OA = OB Hence, height o f the lamppost i s 4 0 m.
[same radius of a circle] (iii) We know that, if two tangents of a circle are parallel,
.. LOAB = LOBA . . . (i) then distance between two tangents is equal to the
diameter of a circle.
In the given figure, distance between Y-axis and AB is
We know that, sum of all angles in a triangle is 1 80°.
.. LAOB + LOAB + LOBA = 1 80° 1 0 m, which is equal to the diameter of a circle.
=> 60° + LOAB + LOAB = 1 80° [from Eq.(i)] 10
=> 60° + 2 L OAB = 1 80° :. Radius of required circle = = S m
=> 2 L OAB = 1 20° 2
L OAB = 60° 3. We plot all the points A (2, 4), B (6, 4), C (6, 2) and


=>
.. L OAB = LOBA = 60° [from Eq.(i)) D(2, 2) on a graph paper and join them, a rectangle
Hence, l:l.OAB is an equilateral triangle. ABCD is formed.
� OA = AB = OB = a y
E
In right angled llCOB, tan 30° = OC
� (2,
4 A
OB h
I -;; F
3
h 8 (6, 4)
=> = 2m
.fj 2 (2,D2) C
(6, 2)
.=> h = .!!.._ m
1
X'--+-+-if--+-+- -+----X
.fj 123456
Hence, h eigh t of t h e tow er is a 1 .J3 m.
Y'

2. In the given figure, d raw ED perpendicular co the line AB. (1)


Let a pole of height BE = h m is standing at point B.
From the fi gure , Now, AB = �(6 - 2) 2 + (4 - 4) 2 = �4 2 + 0 2 = 4 m
LFAD = LEDA = 30° [alternate angles]
and LFA C = LA CB = 60° [alternate angles] [· : distance = �(x2 - x1 ) 2 + (y2 - y1 ) 2 ]
Solutions to Challengers

and BC = J<6 - 6 ) 2 + ( 2 - 4) 2 = J(-2)2 = .J4 = 2m Now, in right angled dTAW (formed in air),
!!... TW
In right angled 6. ABE, ran 30 ° = BE AB 4 = can a = --AW
h => AW = -- 1W = --h
1
= =:> h = 4 m can a can a . ... (i)
.fj "4 J3 or AW = h coca
Again, in right angled 6.EBF, tan 45° = _!!. FB .._
In right angled dTWB (formed in air),
can J3 = -1W
4
1 = / Jj
BW
FB => BW = -- 1W = -.- h ... (ii)
FB = - 4 canJ3 canJ3
.fj or BW = hcotJ3
(ii) Now, area of rectangle ABCD = AB x BC By using Pythagoras theorem in right angled M ti 3
(formed on ground),
=4 x 2 =8 m2
AB 2 =AW 2 + BW 2
and area of MBE = -1 x AB x BE d2 = h2 cot 2 a + h2 coc 2 J3
2
= -1 x 4 x - 4 =- 8 m2 d2 = h2 (coc2 a + coc2 J3)
2 .J3 .J3 h= 2 d 2 Hence proved.
: . Toral area of rhc figure = Area of rectangle ABCD �coc a + coc J3
.J3+ Area of ll.ABE
- 8 + _- .J3+ 1 ) m 2
_ s s c
J3
4. Let 7W represents the tower of height h i.e. 1W = h with
T as top and Was base of the tower (on ground) A and B
are two stations due South and due Ease <o cower
respectively. Then, angle of elevations are
L TA W = a and LTBW = J3
Observing fig. we sec there are three right angled triangles
two in air (6.TWB and 6.TWA) and one on the ground
(MWB).
T (Observer's eye) Projection of bird path

Let 0 be the observer and R be mid-poi nt of PQ. Lee


arc

OA = d and CA =CB = r (radii). From the figure, i r is dear


that PA =RM =QB = h, OC = d + r,

OB = d + r + r = d + 2r
Let M be projection of R on ground, chen in right angl d e

dOCM, (formed on ground) 2


OM 2 = OC 2 + CM 2
2
OM 2 = ( a + r) + r
'2
OM = fcd + r) 2 +
A
Alllnone MATHEMATICS C la ss 1 0th Term 11

In right angled D.OAP,


PA = !!_ :. P( E, ) = Number of favourable outcomes
tan 60° = Total number of o uc comes
OA d
... (i)
2 1
In right angled 6.0BQ, 36 18
QB
tan 30 o = - = --- h
Let £2 = Event of gening sum 3
OB d + 2r
... (ii) (ii)
= { ( 1 , 2 ) . ( 1 , 2 ) . ( 2. 1 ) , ( 2, l )}
In right angled D.OCR, So, number of favourable outcomes to £2 = 4
RM
tan e = -- = --;======
h
P( E2 ) -4 - 1
OM �(d + r )2 + r 2
... (iii ) ·
- -
•• 36 9
- -

(iii) Let £3 = Event of geccing sum 4


From Eq. (i) , .J3 = !!_d = { ( 2. 2 ) , ( 2, 2), (3, 1 ) , (3, 1), (1 , 3), (1, 3)}
h = .J3d So, number of favourable ourcomcs to E3 = 6
6 1
From Eq. (ii) , 1
= d +h ir : . P ( E,,. ) = - =-
.J3 36 6
.J3d (iv) Let E4 = Eve nt of getting sum 5
=
1
.J3 d + 2r = {(2, 3 ) , ( 2, 3 ), (4, 1), (4, 1 ), (3, 2), (3, 2)}
So, number of favourable outcomes to £4 = 6


=> 3d = d + 2r
2 d = 2r
: . p ( E., ) = 36 = 6
=> 6 1
=> d =r
From Eq. (iii), tan 9 = .J3d �3 r = 3
.J3 (v) Let Es = Evenr of gccring sum 6
--;:::=
: == =
�4 r 2 + r 2 5r 5 = {(3, 3), (3, 3 ), (4, 2 ) , (4, 2), (5, 1 ) , (5, l)}
So, number of favourable outcomes to Es = 6
tan 2 e = -
3 6 1
: . p (£5 ) = - = -
5 36 6
(vt) Let £6 = Evcnr of gecring sum 7
C h ap ter 6 Pro �ab ility = { ( 4, 3 ), ( 4, 3 ), (5, 2), (5, 2), (6, 1 ), (6, l)}
So, number of favourable outcomes to E6 = 6
(t) Here, total number of outcomes = 2 + 3 = 5
:. p ( 6 ) = -
1.
Given, odd in favour = 2 : 3
E 6 = -1
36 6
It means that out of 5 outcomes, 2 are in favour of the (vit) Let £7 = Event of geccing sum 8
event. = {( 5, 3)' ( 5 ' 3), (6, 2), ( 6, 2 )}
:. Probability of occurrence of the event = � So, number of favourable outcomes to £7 = 4
5
:. p
4 1
Hence, probability of non-occurrence of the event (E7 ) = - =-
36 9
= l - 3. :::: � (viii) Lee E8 = Event of getting sum 9 = {(6, 3), (6, 3)}
5 5
(ii) Here, the total number of outcomes = 3 4 = 7 +
So, number of favourable outcomes to £8 = 2
: . p ( ER )
2 1
Given, odd against an event = 3 : 4 = =
It means that out of7 outcomes, 3 are not in favour of 36 Is
the event and 4 outcomes are in favour of the event. 4. Given, at a fece, cards bearing numbers 1 to 1 000, one
number on one card, are puc in a box. Each player selects
:. Probability of occurrence of the event = _± one card at random and chat card is not replaced, so the
7
coral number of outcomes = 1 000.
2. Total number of integer points in rectangle ABCD
If the selected card has a perfect square number greater
= Number of integer points on rectangle than 500, then player wins a prize.
+ Number of integer points inside the rectangle (1) Lee £1 = Eve nc char che first player wins a prize
= 1 4 + 6 = 20 = Player selects a card which is a perfect square greater
:. Required probability = -1- than 500
20 = {529, 576, 625, 676, 729, 784, 84 1 , 900, 961}
3. Total number of ourcomes = 36 = { (23 ) 2 , (24 ) 2 , (25 ) 2 , (26 ) 2 , ( 27 ) 2 , ( 28) 2 , (29) 2 ,
( t) Lee £1 = Evenr of ge rri ng sum 2 = {(l, 1 ), (1, 1 ) } (30) 2 , (31 )2 }
So, number of favo u rable outcomes co £1 = 2 So, number of favourable ouccomes to E1 = 9
Solutions to C h a llengers / 391 /
:. Required probabi l i ty Clearly, chere are coral number of outcomes = 36
elementary events out of which, we have 6 elementary
= -------
N umber o f favourable.: outcomes events (a, b) in the diagonal, for which a = b and 1 5
Total n u mber of outcomes elementary events {a, b ) below the diagonal,
9
= -- = 0 .009 for which a b. >

1 000 : . Number of favourable outcomes co an ·event, E = 1 5


(ii) First has won , i .e. one: card is al ready sclecced, greacer Hence, required probability
than 500, has a pc.:rfc.:cc square. Si nce, repecicion is not Number of favourable outcomes
allowed. So, one: card is removed ouc of 1 000 cards. So, =
number of rema i n i ng cards is 999. Total number of outcomes
15 5
:.To ca! number of remai n i ng oucco!JleS = 999 =-=-
Lee £2 = Evc.:nc thac the second player wins a prize, 36 12
if the: first has won
= Remai n i ng cards has a perfecc square greater
than 500. C h apter 7 Coordinate Geometry
:. N umber of favourable ouccomes co £2 = 9 - 1 = 8
1. Let A(x1 , y1 ), B(x2 , Ji ) and C(x3 , y3 ) be the vertices of a
So, requ i red probability = � MBC, when X; , Yi • i = 1 , 2, 3 are integers. Then, the area
999 ofMBC
5. Lee the n umber of blue marbles be b, number of green 1
� = - lx1 (J2 - y3 ) + x2 (J3 - y1 ) + x3 (Y1 - Yi )j
marbles b e g and n u mber o f whice marbles be w. 2
Then, b + g + w = 54 . . . (i) => Area of MBC =A rational number
. . P (selecting a blue marble) = J_ [·: X; , J; are integ��-1
54 If possible, let the MBC be an equilateral triangle, then ' '
and P (selecting a green marble) = ..!_ area is given by
54
Area of MBC = .J3 (side) = .J3 =( AB) 2
Given, che p ro bab i li ty of seleccing blue marble = .!_
a 4 4
b 1
3 [·:AB = p:; C4J
-=-
54 3 => Area of MBC = .J3 (a positive integer)
54 4
b= = 18 [·: vertices are integers :. AB2 is a posicive inceger]
3
=> Area of MBC = An irrational number
=> b = 18
This is a contradiction co the face chat the area is a racional
And P (selecting a green marble) = i [given] number.
9 Hence, the triangle cannoc be equilateral.
g =-
- 4 2. Let the coordinate of top of the rower be P(x,y ).
Now, SR = �(6 - 3)2 + (2 - 2) 2 = j32
54 9

g = -- = 24
4 x 54
=> p (x, y)
9
On subsrituring che values o f b and g in Eq. (i), we get
1 8 + 24 + w = 54
=> UJ
= 54 - 42 = 1 2
Hence, the box contains 1 2 white marbles .
6. Here, the arrow can scop in any of che six sectors. So, and a

b both can assume values from 1 to 6 . Thus, che ordered


pair (a, b) can assume che following values. Let che coordinate of foot of cower Q be (x, 2) and p be
(x, y).
( 1 , 3) (1 , 4) (1 , 5) ( 1 , 6) Then, QR = (x -6) and PQ = (y 2) _

(2, 5) (2, 6) In right � RQP, can60 ° � =

(x - 6)
(2, 4)
1 5 order (3, 5) (3, 6) 1 5 order
pairs (a, b) , pairs (a, b), => J3 ::::: .Y - 2 . . . (i)
where a > b (4, 1 ) (4, 2) (4, 6) where a < b
x -6
(5, 1 ) (5, 2 ) (5, 3)
(6, 1 ) (6, 2) (6, 3)
And in right angled !lSQP, ran 4 5 ° =
_Y_
x -3
\ 392 \ Alllnone MATHEMATICS C lass 1 0th Term II

= ( 2x1 -;2 - X3 ) + ( 2J1 - �2 - J3 r


=> l =- y­ '

= .!_ 4x1 ., xi ., xl ., x1x2 x2x3 4 3


[ ]
x -3
=> y = x -3
From Eq. (i), .J3 = x x- 3 - i
2 + + - 4 + 2 - x1x
-6 9 + 4 yj + Yi + yj - 4 Y1 Y + 2yi.J3 - 4J1 1J

- 4x2x3
=>

:; � [
.J3 (x - 6) =x -5 2
+
[·: (a + b + c) 2 = 112 + b 2 + c 2 + 2ab + 2bc 2t4]
]
=> x <.J3 -1) = - 5 + 6.J3 Si r ,
: � x� + 4xi + x_i - 4 .,·1x2 + 2x1x3
[ l
=> x = - 5 + 6 .J3
./3 -l
9+ y� + 4 Ji + J32 - 4y1 Y2 + 2J1 J3 - 4y
.z]J
and y = - 5 + 6 .J3 3 = -2 + 3./3
_

.J3 - 1 ./3 - 1 x12 + xi + 4 xJ + 2x1x2 - 4x1x3


1

y, )
3. When we rotate the line PQ in anti-clockwise direction at and GC 2 = '9 - 4x2x3 + y12 + Yi2 + 4y32
an angle of 2 70°, then the new coordinates of point Q will

J+ k -
+ 2Y1 Y2 - 4Y2Y3 - 4J1 Y3

be at R, which touches the X-axis at (2, O) . '
y
Also, GP2 = h - ,
( x + � + x3 ( Yi + +

J
4

= h2 + ( XI + X; + X3 l h (XI + � + X3 )
+ k ' + ( y, + y; + Y3 J - 2 { Yi + � JJ )
3
_
2

------1---+=--'+---+-----1,__..___X +

-
1 R 2 3 4

Hence, the coordinates of R arc (2, 0). => 3GP2 = 3h2 + .!.. c x1 + x + x3 ) 2 - 2h( x1 + x + x3 )
Now, PQ = �( 4 - 2)2 + (2 2 )2 = � = 2 units 3 2 2
[·: distance = �(x2 -x1 ) 2 + (y2 - y1 ) 2 ] + 3k 2 + .!_3 ( Yi + Y2 + Y3 ) 2 - 2k(yi + Yi + Y3 )
� �
Area of the figure = 7tr2 = x 314 x 4 = 9.42. sq units Now, PA2 + PB 2 + PC 2
4 4 = 3(x12 + xi + xJ ) + 3( y12 + y; + yJ ) - 2h(x1 + x2 + x3 )
4. Let the vertices of a triangle be + 3h2 + 3k 2 - 2k(yl + Yi + JJ )
A(xpy1 ), B(x2 , y2 ) and C(x3 , y3 ).
Since, G is the centroid of a MBC . S imilarly, GA2 + GB2 + GC2 + 3GP2
Coordinates of G are
. ·.
=3 (x1 + x2 + X3 ) +3 (y12 + Yi + yJ )
( x1 + x; + X3 , y, + Y; + Y3 } + 3h2 + 3k 2 - 2 k(yl + Yi + J3 )
Let P(h, k) be a point in the plane. :.PA2 + PB2 + PC 2 = GA2 + GB2 + GC2 + 3 GP2
Hence proved.
Then, PA 2 = (x1 - h) 2 + (y1 - k) 2
5. When we rotate the given graph at an angle of 180°, then
[·: distance = �(x2 -x1 ) 2 + (y - y1 ) 2 ] the new graph is shown below
2
= x� + h2 - 2x1 h + y{ + k2 - 2y1 k
PB2 = (x2 - h)2 + (y2 - k)2
= xi + h2 - 2x2 h + Yi + k2 - 2y2 k

:. GA = ( x, - ,
x + x; + X3 r + ( - y, + �2 + Y3
and PC2 = (x3 - h)2 + (y3 - k)2
= xJ + h2 - 2x3 h + yJ + k2 - 2y3k
2
Y1
J Thus, the new coordinates will be remain same.
i.e. A(l, I ), B(3, I ), C (3, 3) and D (l, 3).
Solutions to Challe ngers 1393 1

(3 ; I I � 3 ) = ( H) =
y
We know char in a square, the diagonals bisect each ocher. 7. Lee OABC be a rhombus, such
:.M id-poi n t of BD char OA is along X-axis. Lee BL
and CM be perpendiculars
= •
( 2, 2 ) from B and C respectively on
X-axis. Clearly, MBL and
6. Lee che third vertex of an equilateral triangle be (x, y). !lOCM are congruent.
Then, vertices of tria ngles are A (- 4, 3), B( 4, 3) and :. OM = AL and CM = BL
C (x, y). Lee the coordinates of A and C
be (x1 , O) and (x2 , Y ), respectively.
We know char, i n eq ui lateral triangle, the angle between
OM i = X and OA = x
Then, 1
two adjacen t sides is 60° and all three sides arc equal. i
.. OL = OA + AL
.. AB = BC = CA => OL = OA + OM [·: OM = AL]
AB 2 = BC 2 = CA 2 . . . (i) => OL = x1 + x
Now, caking first cwo terms, A B i = BC i i
and BL = CM = Y
i
=> ( 4 + 4 ) 2 + (3 - 3 ) i = ( x - 4 ) 2 + (y - 3 ) i So, che coordinates of B are (x1 + X , Y ).


i is a rhombus]
[·:i OABC
.-_ ___---___ Now, OA = OC
[ · : dista n ce = (x_ __-
2 x1 ) i + (yi y1 ) i ] => OA i = 0C 2
=> 64 + 0 = x i + 1 6 - Bx + / + 9 - 6y => Xii = xJ + yJ . . . (i)
=> xi + yi - Bx - 6y = 39 . .. (ii) In order co prove that the diagonals OB and AC are
mutually perpendicular, ic is sufficient to show that
Now, taking fi rst and third terms, AB 2 = CA i LODA = 90°.

f ; ;)
=> ( 4 + 4 ) i + (3 - 3 ) i = ( - 4 - x ) i + (3 - y ) i Si nce, che diagonals of a rhombus bisect each
ocher. Therefore,
=> 64 + 0 = 1 6 + x 2 + Bx + 9 + y i - 6y Coordinates of D = Mid-point of OB

( : J (; J
=> x i + y i + Bx - 6y = 39 . .. (iii) x y
=> D I 2,

--
On subtracting Eq. ( i i) from Eq. (iii), we gee
x i + y 2 + Bx - 6y = 39
-0
x, xz
Now, OD 2 = -0 + Y
x i + y i - Bx - 6y = 39
J
+ + ..-- --
..- - -:-
1 6x = 0
[·: distance = cx2 - x1 ) 2 + (yi - Yi ) i l
x =0
Now, puccing the value o f x i n Eq. (ii), we gee
0 + Y i - 0 - 6y = 39 => Y i - 6y - 39 = 0

�:�
6 ± �( - 6) i - 4 (1) ( - 39)

]
y-

[
2 x 1

( ; r ( ;)
-b ± - 4ac
by quadratic formula, x= ., ., 2
and OA 2 = (x1 - Ot + (O - o)- = xi

( ; -�1 J ( � J'
'
6 ± .J36 + 1 56 6 ± .J192 = 6 ± 2 .J48 ± .J48
OD2 + AD2 =
x, x,
+ Y
= = =3 Now,

, 4 -, ,
2 2 2
= 3 ± 4.fj = 3 + 4J3 or 3 - 4J3
+ -�z + 2
So, the poin ts of third vertex are (0, 3 + 4..J3) or
."<1- + 2x1x + x:;
., ., .,
+ x1-
.,
y;
(O, 3 - 4.fj ). y:; -
x :; - 2x.,x1
y ----...
-- ...= + .:......::.

\ , -
--=-
- + -=- +
4
-4, 3) --+-___, 8 (4, 3)
But given chat, the origin l ies 4 4
i n the i n terior of the tiAB C 1
A( -
- - ( 2x12 + 2x.,-2 + 2y,-2 )
and the x-coordinate of third
4

--
vertex is zero.
( y + x -:; + .y:;., )
.._..,...,-::-:- -
,- :::.3t-t-t- --=--- x 1 .,
Then , y-coord in ate of thi rd X' = -
.

vertex should be negative. 2


Y' 2
( xi + x12 ) lus ing E.q. (i)]
Hence, the requi red - 1
coord i nates of third vertex are
C(O, 3 - 4.fj ). [·: C � (O, 3 + 4.J3}] 2
�394 �
' Allinone MATH EMATICS C lass 1 0th Term II

= x11 = OA1 [ ·: OA = �x12 + 02 ] 9. Given, coordinates of pole: be: P( I, 3) and Q (3, 3) and
: . fl.ODA is a right angled triangle, such that A ( 1 , 1 ) be chc position of man.
LODA = 90°. (1) Now, AP = Jo - 1 ) 2 + (3 - 1 ) 2 = Jo 2 + 22 = 2 units
Hence, the diagonals of a rhombus are perpendicular to

..,.f
each other. Hence proved.
8. (i) Given, points are A (2, 4), B(6, 4), C(6, 6) and
D (2, 6) plotting on a graph paper, is shown below
y
6 (2""'."'.'"
.-:
oI -=- . 6- ) (6.6)
5
___
4
3
A (2. 4)
2 B (6,4 )
and PQ = )C3 -l)1 + (3 -3)2 = .J2 + 0 2 = 2 untis 1

X ---.-4-....+- -+--+--+----+�--x
Now, in llAPQ , can 8 = PQ
0
...
1 2 3 4 5 6 AP
Y'
2
can 8 = = I -
2

-4) SD
When we rotate the axes at an angle of 90° in => 8 = 45 °
anti-clockwise direction, che new axes are shown below. (it) When we shift the origin at ( I , I ) , chcn the angle wiU
x remain same, i.e. e = 45°.

D
6 (6, c (6, -6) 1 0. (1) y
.5
5 (3 3 ) (5 3 )
4
4
3
3
2 (2. -4) A D (2, -6) 2
(3, 1 ) A
. .
B (5, 1 )
Y --=t-1---+---+--+---+----+--- y• x
-1 -2 -3 -4 -5
0 -6 0 1 2 3 4 5 6
X'

( )
Here, we see that, in first quadrant, y-coordinaces will be We know char, rhe incerscccion points of rwo diagonals is
negative. the centre of rhe inscribed circle. Now, mid-point of A
:.The new coordinates of A, B, C and D are respectively
A(2, - 4), B(6, - 4) C (6, -6)and D(2, -6). and C is, £ 3 + 5 , 1 + 3 i .e. £(4, 2)
2 . 2

1 � T)
(it) Now, AB = �(6 - 2)2 + (-4 + 4)2 AJso, we know that inscribed circle couch the square in the
mid-point of che line segment.
= �42 + 02 =4 units
Now, mid-point of A and B is , i.e. F(4, ii
BC = �(6 - 6)2 + (-6 + 4)2 = ��(0_)_2+ ( 2 ) 2 = 2 units
---
---

CD = �(2 6 ) 2 + (-6 + 6)2 = �(-4)2 + 02 = 4 units


- Now, radius of a circle, EF = �( 4 - 4)2 + ( 2 -1)2
and DA = �(2 - 2)2 + (-6 + 4)2 = .JO+l = I
= �02 + (-2)2 = 2 units (i1) Now, side of a square, AB = �(5 -3)2 + (I - 1)2
. . AB CD and BC = DA
= = .J2i = 2
Now, diagonals, AC = �(6 - 2) 2 + (-6 + 4)2 Now, area of square, A1 = ( 2 ) 2 = 4 sq units
= �4 2 + (-2)2 = .J16+4 = ..fiO= 2.J5 uni rs and area of circle, A2 = 7tr2
a

and BD = �(2 - 6)2 + (-6 + 4)2 = �(-4)2 + (-2)2


= 3.14 x l [·: r EF = I] =

= 314 sq units
.J16+4 = ..fiO = 2./5 units
=
:.Area between the square and inscribed circle
A C = BD = A1 - A2 = 4 - 3.1 4
Hence, adjacent poi n ts A, B, C and D form a rectangle . = 0 86 sq uni rs
.
Solutions to Challengers
/ 3 95 /
Now, in tl.AOB, we draw a
C h apter 8 Areas Related to Circles perpendicular line OD, which meets
at D on AB and divides chord AB
1. Given, PQR is an isosceles triangle in which right angle ac into two equal parts.
Q , i.e. L Q = 90 °
So, AD = DB = AB 2_
and PQ = RQ = x (say) 2 = 2 cm
[since, the perpendicular drawn .
from che cencre co che chord of a Minor segment
circle divides the chord into two equal parts]

( }i ) ( )
In tl.ADO, by Pythagoras theorem,
(OA) 2 = OD 2 + AD 2 => OD 2 = OA 2 - AD 2
= ' - % ' = 2; _ 2; = 50 � 25 = �

In righc angled., !!:,.PQR, =>


OD =-cm5
., ., [by Pythagoras theorem] 2
PQ - + RQ - = PR -
., ., r::: ., ., 2 )2
.\. - + x - = (7v 2 > - ::::. 2x - = (7-vr:::
:. Area of an isosceles tl.AOB = ..!..2 x Base x Heighc
O n taking square roots both sides, we gee = ..!_ x AB x OD = ..!_ x 5 x 2_ = 25 cm 2
J2 x = 7 J2 ::::> x = 7 cm 2 2 2 4
1tr 2

(*r
Now, semi-perimeter of an isosceles ll.PQR Now, area of sector AOBA = -- x 0
7 + 7 + 7.J2 14 + 7.J2 cn1 360°
----- ==

2 2 "x
Also, we know that radius of inscribed circle, x 900 = " x 25 = 25rr cm 2
Arca of !!:,.PQR 360° 2x 4 8

( )
r == ------

. . Area of minor segment = Area of sector AOB


Semi- pcrimcccr of an isosceles ll.PQR ·

I - Area of an isosceles llAOB

( )
x 7x 7 25 2 5
= 2 = 7 x 7 = 7 cm = 7t _ cm 2
8 4
... (i)
14 + 7.J2 7(2 + ./2 ) 2 + J2
2 Now, area of the circle = rrr '= }i ' = 2� " cm 2
x

. . Area of major segment = Area -Area


Area of circle inscribed in !!:,.PQR = 7tr 2
-227 ( 2 +7J2) (2 +7.J2.)
( )
==
of circle
= 252 7t - 258 7t - 4 = 8 (4 - 1) + 4
x x ---,--
of minor segment

=(7�" )
25 25 7t 25
= 1 54 cm 2

(2 + ..fi.>2
Hence, the area of the circle inscribed in an isosceles ll.PQR + � cm 2 (ii)
•••

( �)-(2�" )
. 1 54
r::: cm 2 .
(2 + v 2 ) 2
ts
:. Difference of ch.e areas of cwo segments of the circle
Note In an inscribed circle, if !!:,. , and r be the area,

f
=Area of major segment -Area of minor segment
= 7�7t + - 2;
s

H )
semi-perimeter and radius of a circle respectively, then
�" _ 2�" 2; + 2:
!!:,.
r = -.

( 75rr ; 25rr + 5:) = ( 5�rr + 5� )


s

2. Let the radius of the circle be r.


OA = OB = r cm

=(2!" + �}m2
..
Given that, length of chord of the circle, AB = 5 cm =
and central angle of the sector A OBA, 9 = 90°
Now, in MOB,

of a . . (4
(AB ) 2 = (QA )2 + ( OB)2 [by Pythagoras cheorem]

+ 2 5 ) .,
(5 ) 2 = r 2 + 7 2 of areas of
Hence, the requi red d ifference che two segments
2 2 25r == 25n l
c1rc le is cm -.

r .J2 5 cm
==
Allinone MATHEMATICS Class 10th Term II

3. Suppose, R and r be rhe radii of Then, LI = L2 = 90°


bigger and smaller circles, [since, rangenr makes 90° angle wi th the
respectively. radius ac rhe poinc of cont20]
AB - AC = CB Through O', draw O' M I I A /J meeting OA ar M.
2R - 2r = 9 A Also, Ice AB = O' �I = x.

:::::::) R -r = -92 = 4.5 ... (i) Thus, ABO' M is a rectangle with AJ\1 = 0' B = 4 an.

F
In !!:.OMO', OM = OA - AM = 12 - 4 cm =8 cm

Now, join AD and CD. and 00' = OP + O' P = 1 2 + 4 = 16 cm


MOD ... !!:.DOC Also, 00' 2 = OM 2 + (0' M )2 [by Pyrhagoras theorem]
:::::::) OD = -
- OC => I62 = 82 + (0' M ) 2
OA OD
:::::::> OD2 = OA x OC => x 2 = I 62 - 8 2 = (1 6 - 8)(1 6 + 8)
:::::::> (R -5)2 = R x (R -9) => x2 = 8 x 24 = 8 x 8 x 3
[·: DE = 5 cm and CB =9 cm] => ·"' = .Js x 8 3 = sJJ cm
x

:::::::> R2 + 25 -10R = R2 -9R


:::::::>
Area of ABO' 0 = Area o f rectangle ABO' M
R = 25 cm + Arca of llOMO'
From Eq. (i), we get R - r = 4.5 => 25 -r = 4.5
:::::::> r = 25 - 4.5 = 20.5 cm = AM x O' M + _!_ x OM x O' M
Now, area of the shaded portion = 1CR 2 - 1Cr 2 . 2
= 1t (R2 -r 2 ) = (R + r) (R - r )
1C = 4 x 8J3 + _!_ x s x s.Jj
2

--
=3.1 4 (25 + 20.5) (25 -20.5) = 32 .fj + 32.Jj = 64.[3 cm
=3.1 4 x 45.5 x 4.5 = 642.9 1 5 cm 2 Now, !!:.OMO' is a right angled triangle wirh LM =90°.
Hence, the required area of shaded portion is 642.9 15 cm 2 • s.fj = -.fj
4. Given, AB = 1 8 cm, DC = 32 cm, height (h) = 14 cm . . cos (L 3) = O'M 00' = -- 16 2
and arc of radii = 7 cm => L3 = 30°
Since, AB ll DC => LMO' O = 30°
LA + LD = 1 80° OM = -8 = -I
and LB + LC = 180° and cos(L4) = -- 00' 1 6 2
:.Area of sector with angles A and D = _e_ x 7tr 2 L4 = 60°
360°
80 0 22 2 => LAOP = 60° = Angle of sector AOP
= 360° x 7 x ( 7) = I l x 7 = 77 cm 2
1
Now, LBO' P = 90° + LMO' O = 90 ° + 30°
Similarly, area of sector with angles B and C = 77 cm 2 = 120° = Angle of secror BOP
:. Area of the portion enclosed between rhe circles and che
Now, area of trapezium = .!_2 (AB + DC ) x h tangents

(
=Arca of rhe trapezium ofABO' 0

--
= .!_2 (1 8 + 32) x 14 = 502 x 14 =350cm2 - (Area of sectors AOP +Area of sector BO'P)

J
. . Area of shaded region
=64.J3 - (Area of sector AOP + Area of sector BO'P )
= Area of trapezium - (Area of sector points A and D = 64.J3 - -- 60° x 7t x 1 2 2 + 1 20° x x 4 2 .
1C

+ Area of sector points B and C) 360° 360°


= 350 - ( 77 + 77) = 196 cm 2 =64..f3 - �6 (144 + 2 x 16)
Hence, the required area of shaded region is 196 cm 2 •
S . Let tangent touches the circles with centres 0 and O' at = 64.J3 - � x 16(9 + 2)

= (64 x I.73. - 8 x 7
the points A and B, respectively. 6
A =(64v'3 + 7t X 1 1)
22 x·1 1 )
3
= (1 1 0.72 -92.1 904)
= 18. 5296 cm 2
Solutions to Challengers f 397 ,
6. From P, draw PQ ..L AB. :. Area of shaded portion in one circle having centre A
. Join PA and PB. = --9nr2
= 240° 3.14 x (4)2 = -2 x 31 4 x 1 6 = 33.5 cm 2
x

:. AP = AB = 6 cm 360° 360° 3
[radii of same circles] 6 cm 6 cm
. . Toral area of the shaded region
Similarly, = 6 �ea of one shaded portion
x

BP = BA = 6 cm =6 33.50 = 201 cm 2
x

[radii of semi-circles] 8. Firstly, draw a circle having centre A(4, 5) and radius Ji.
Now, in right angled 6 cm
Then, from point P(2, 3), draw a pair of tangents which
MQP and !:l.BQP, meet the circle at points B and C .
AP = BP Also, draw perpendicular lines from centre to the tangents
PQ = PQ [common sides] and join AP.
L.AQP = LBQP = 90° y
=> M QP = !:l.BQP [by RHS congruence rule] 6
:. AQ = BQ = -2 AB => AQ = BQ = -62 = 3 cm
1 5
4
Let LPBQ = 9
:. In !:l.BQP, cos 8 = - QB 3
p (2, 3)
PB 2

=> cos e = -36 = -21 1

=> cos e = cos 60° => e = 60° o 2 3 4 s s x

and sin 60°= PQ PB => PQ = PB sin 60° Now, AP = �(4 - 2)2 + (5 -3)2 = �2 2 + 2 2 = 2J2.
In right angled MCP,
=> PQ = 6 .J3 2 = 3 1.732 [·: .J3 = 1.732]
x x

PQ = 5.196 cm ... (i) sm. 0 = -AC Ji


= --
=>
60° 22 AP 2.[i
Now, area of sector BPAB = -- x x6x6
-

sm. e = -I => a =30°


360° 7 =>
2
= .!_ x
22 6 6 =1 8.857 cm 2
x x · · LBPC = 20 = 2 x 30° = 60°
6 7 Also, we know that,
Arca of !:l.BPQ = 21 QB x PQ = -1 3 5.196
- x x x L CAB = 180° - 29 =180° - 60° =120°
2 LCAB 120° 0
= 7.794 cm [using Eq. (i)] N ow, LEAC = -- = - = 60
2
:. Area of porcion APB 2
In right angled MCP,
= 2(Area of sector BPAB -Area of llBPQ) [using Pythagoras theorem]
= 2 (1 8.857 - 7.794) = 22.126 cm PC = �AP 2 - AC 2

( )
:. Area of the shaded portion = �( 2.Ji.)2 - (./i.)2 = �8 - 2 = JG
= 2 (Area of quadrant ABC -Area of the portion APB)

( ;
= 2 � 7tr 2 - Area of the portion APB
)
Now, area of MCP = 21 x PC x AC
x
-

x x

= 2 � 2 6 6 - 22.1 26
= !2 .J6 J2 = !2 2 x J3 = .J3 = 1.73 sq units
x x x

97tr 2
x x x x

= 2 (28.286 - 22.1 26) = 12.32 cm 2


x
Now, area of sector LCAE = -- 360 0
Hence, the required area of shaded portion is 12.32 cm 2 • 60° x 314 x (Ji.)2 314 x 2 = - 3.14- = 1.05 sq un1r. s
7. Given figure is a regular polygon having six sides. 3600 6 3
:. Interior angle o f regular po1ygon, LFAB = n ..,... 2 18 o o
--
. . Area of curve PEC = Area of MCP

2
x
n - Area of sector LCA.E
= 6 - 2 1 80 ° = 4 x 30 ° = 120 °
x
= 1.73 - 1.05 = 0.68 sq unirs
6 . . Required area between the pair of rangents and circle
. . Exter ior angle of L FAB = 360° - 120° = 240° = (Area of curvePEC) = 2 0.68 = 1 36 sq units
x
Alllnone MATH EMATICS Class 1 0th Term 11

C h apter 9 Surface Areas = -I 7t ( v3r::; 11 ) ., x 311 =3 mr cu umcs


-
1 .

3
and Volumes and volume of sphere = 4 mr cu unlCs -
1 •

3
1. Given, Radius of the lower :. Volume of water remaining in chc cone
end of the frustum, 'i = 1 cm =Volume of cone BCD Volume of sphere -

= 3 na3
Radius of the upper end of 4 ,Sn \

the frustum, r2 = 2.5 cm, --


3 mr = 3 ,r cu -

Height of the frustum, D


h = 6 cm unics
Now, slant height of the 3. It is given char
frustum, AD = 7m, BA = 4 m,
l = �,...
h_. 2_+_(_r2---r.-,)-2 AC = 3m
(i) le is clear from figure,
l = �36 + (2.5 -1) 2 = .J38.25 =6.1 8 cm Toral edges are BC. A B. A C,
c
External surface area of shuttlecock DF, DE, EF, EC. FB, DA .
= Curved surface area of the frustum . · . T oral edges = 9
+ Surface area of hemisphere (ii) Volume of prism = (Arca of base) x Height
= 7t( 'i + r2 )l + 21t1i2 = 7t(l + 2.5) x 618 + 2 x 7t x 1 2 = (Area of right triangle) x Height
=722 x 3.5 x 6.1 8 + 2 x 7 22 =67.98 +6.28 = 74.26 cm 2 = -1 x 4 x 3 x 7 = 42 m 3
2
(iii) Total surface area = Arca of two ends + Area of
2. Let radius of sphere be a, i.e. OK = OA = a. remaining surface
Then, centre 0 of sphere will be centroid of the llBCD . = 2 x Arca of triangle + Arca of rectangle EDA C+
OA = -1 AB => A B = 3 (OA) Area of rectangle A DFB + Arca of rectangle FECB
3 = 2 x -21 x 4 x 3 + 3 x 7 + 4 x 7 + 5 x 7
= 12 + 7(3 + 4 + 5) = I I + 7 x 12
= 1 1 + 84 = 95 m 2
4. Let Vi , V2 and ltJ be the volumes of cones CAB, C1 AB
and, C AB, respectively.
2

For cone CAB, we have CO = 3h ·


In �COA, tan a = - OA = - OA
OC 3h
=> OA = 3h ran a
B
:. Volume, Vi = 1 7t (3h ran a ) 2 x 3h = --
-
27 7t h3 tan 2 a
In right angled � OKB, 3 3
• o
sm30 = - a => 1 = - a => OB = 2a
-
For cone C1 OA, C1 0 = 2h
OB 2 OB c
:.AB = OA + 0B = a + 2 a = 3 a
Now, in �BAC, AC = tan 30 °
AB
AC = 1
AB .j3
AC = AB
.J3
AC = �
.J3
A C = .J3 a units ·

tan 2 a = -- OA
OA = -
Now , vol u me of cone BCD = .!..3 7t ( AC) 2 ABx
OC1 2h
Solutions to Cha llen gers / 399/
=> OA = 2h can 2 2a whereas the surface area of hemisphere, i.e. 2nr 2 is
:. Volume, \/,- = 3 -- 7t 2h la n 2a) 8 J
x 2h = rrh ran 2 2a
-
exposed co the surroundings.
3 . . Total surface area exposed to che surroundings
C, n

(i� )
For cone C , A B.
- -
=h
=Surface of the ball -150 1tr 2 + 150 2 7tr2
area
x x

I n � C20A , tan 3 o. =
OA OA
----:- = 16.8 ht + l 507tr 2
OC 2 h
=-
=> OA = h tan 3 a = J6.8 !7t + 1 507tX '= J6.8 17t + 67t
., I 3 .,
:.Volume, \�i = 3 rr ( h can 3 at h = -3 rr lr ran 3 a
1
- x -

= 22.8 l 7t = 22.81 22 = 71.68 cm 2


x

27 rr , 8n , 3 ., 7
Now, V.1 \/,- = ----
- If can - - h can - 2a - a.
3 3 6. When we rorate a cylindrical cube in clockwise direction, a
it// l another cylindrical cube is obtained, where radius is
=

3
(27 tan- a - 8 can - 2 a )
--
.,

10 cm and height 4 cm.


8 � l 7t 1 l :.Required volume = 1tr2h
and V.,- -
. = rr/x ran- 2 a - Ix ran - 3 a
Vi - -
= 3. 14 (10) 2 x 4
x
3 3 [·:r = lOcm, h = 4 cm]
//' (
7t , x
=3. 14 400
- -- 8 c a n 2 a r:rn - 3 a )
- -
'l

3
=
=1256 cm 3
:. R=eq u i red racio = (\1i V? ) : (V2 - V3 )
-
When we rotate a figure along the line EF, a frustum of a
( 2 7 can 2 - 8 t a n 2 2 a) : (8 ran 2 2 a - can 2 3 a)
a
7.
cone
5. Given , d i amc ccr of ball, ti = 4.1 cm
a
= 31 7th (7j2 + 7i r2 + '22 )

Surface area of hall = 4 m· 2 = 47tX ( �I r = 16.8 !7tcm 2


-
:.Radius of base, = :!__ = � cm
a r

= .!. 3. 14 6.5 [ 2 2 + 2 x 2.5 + (2.5) 2 ]


2 2 x x

3
[·: 1j = 2 cm, r = 2.5 cm and h = 6. 5 cm]
the

In each d i m ple,
2
fa c e area u a l ro m· 2 (,- is rhe radius of
s ur eq = 6.80 (4 + 5 + 6.25]
each d i m p l e ) is removed base surface of the
from the =6.80[15.25] 103.7 cm 3
=
SAM P LE
QUESTION PAPERS
A Colle ction of 1 0 S ample Question Papers,
Strictly following the Pattern & Syllabus of CB S E
Term-I I Exalll i nation
SAM P L E QU ESTIO N PAP E R 1
A Sam p le Qu esti o n Pa p e r fo r CBS E Class X Su m mative Exam i nation II

MATH EMATICS
Gene ra l I nstru cti o n s
1. All q u estio n s a re co m p u lsory.
2. The q u esti o n paper consists of 31 q uestions divided i nto 4 Secti ons A, B, C and D.
Sectio n A co m p rises of 4 q u e sti o n s of 1 m a rk each, Section B co m p rises of 6
questio n s of 2 ma rks each, Section C comprises of 1 0 q u esti o n s of 3 m a rks each a n d
Sectio n D co m p rises of 1 1 q uestions of 4 marks each.
3. There is no ove rall choice.
4. Use of calc u lator is not permitted .
....
_ ...__ ____ ___ ----- -- - - - - - - -- --------1

Time : 3 hours Max. Marks : 90

Section A
1. In the given figure , ABCD is a. cyclic 2 . A sphere and a cube have equal surface areas .
quadrilateral, 0 is the centre of the circle. If Find the ratio of the volu m e of the sphere to
LBOD = 1 6 0 °, then find the measure of LBPD. that of cube.
3 . If the first term of an AP is -7 and the common
A

difference is 5, then find its 1 8th term.


4 . A die is thrown. Find the probability of getting
a prime number.

Section B

B�
5 . If the pth term of an AP is q and the qth term is
p, then p rove that n th term is (p + q - n) .
6. What is the angle subtended at the cen tre of a
circle o f radius 1 0 cm by an arc of length
5 7t cm'? p A Q
7. A solid cube o f side 1 2 cm is cut into eight 9. If a square is inscribed in a circle, then what is
cub e s of equal volume. What Will be the side of the ratio of the areas of the circle and the
the new cube? square?
8 . In the given figure , 0 is the centre of the 1 O. What is the ratio of the volume of a cube to that
circle, PO is a tangent to the cirde at A. If of a sphere which will fit insi de it?
LPAB = 5 8 °, then find LABO and LAQB.
\ 404 � Alllnone MATH EMATICS Class 1 0th Term 11

Section C
1 1 . A well with 1 0 m inside diameter is dug 14 m 1 6. If the s u m o f a n u m b e r a n d its p ositive square
d e e p . E a rth taken out of it is spread all around
root is � , then fi n d t h e n u mber.
to a width of 5 m to form an embankment. Find 25

Three unbiased c o i n s a re tossed together. Find


the height of e mbankment.
1 7.
1 2. Find the sum of n terms of the sequence an , the probability o f g e tting
where an = 5 - 6n and n e N. (i) atleast two tail .

1 3. Construct a triangle similar to a given triangle (ii) atmost two h ea d .

as per the given scale factor 3.. . Also, justify it. 1 8. The p oints A ( 2 , 9) , B (a , 5 ) a n d C ( 5 , 5) are the
3 vertices of a M B C rig h t a n gled at B. Then, find
the valu e of a a n d h e n c e the a rea of MBC.
1 4. In the a dj oining figure, two
tangents PO and PR are 2x + 3 x -3
1 9. Solve for x , 2 ( ) - 2 s( ) = 5.
d rawn to a circle with x -3 2x +3
centre 0 from a n external P
point P, prove that 20. In a circle of rad i u s 2 1 c m , an arc subtends an

LQPR = 2 LO QR. angle of 60° at the c e n tre . Fin d


(i) the length of th e arc.
1 5. The hypotenuse of a right angled triangle is
(ii) area of the sector formed by the arc.
1 m less than twice the shortest side. If the
thfrd side is 1 m more than the shortest side, (iii) the area of the segment made by this arc.
fin d the sides of the triangle .

Section D

2 1 . A m a n on the top of a vertical tower observes a 26. In the given figure, from a n external point P, a
c a r m ovin g at a uniform speed coming directly tangent PT and a lin e s e g m e n t PAB, is drawn
towards it. If it takes 1 2 min for the angle of to a circle with centre 0 . ON is perpendicular
d epression to change from 30° to 45°, then on the chord AB. Prove that
h ow soon after this , will the car reach the T

aH
tower? Give you r answer to the nearest
second . 0 p

22. A s e q u e n c e i s d e fined b y
an = n3 6 n2 + 1 ln - 6 . Show that the first three
-
Q
terms of the sequence are zero and all other
(i) PA · PB = PN2 - AN2
terms are positive .
(ii) PN2 - AN2 = O P2 - OT 2
23. T h e area o f a n isosceles triangle i s 60 cm 2 and (iii) PA · PB = PT2
the length of each one of its equal sides is 27 . A b ag contains 1 8 balls o u t o f which x balls are
13 c m . Find its base. red.
24. A vessel is in the form of a hemispherical bowl (i) If one ball is drawn at random from the bag ,
then what is the probability that it is a red
mounted by a hollow cylinder. The diameter of
ball?
the hemisphere is 14 cm and the total height of
the vessel is 1 3 cm . Fin d its capacity. (ii) If 2 more red balls are put in the bag, then
[take , 7t
= 22 / 7 )
the probability of drawing a red ball will be
9 /8 times that of probability of red ball
25. Water is fl owing a t t h e rate o f 5 km/h through a coming in part (i) . Find the value o f x.

p i pe o f diam eter 1 4 cm into a rectangular tank, 28. In a school, the X class students were awarded
which is 5 0 m l ong and 44 m wide. Determine full marks for the following question in the
the tim e in whi ch the level of the water in the paper of Mathematics even when none of the
tank will ris e by 7 cm . students attempted it.
Sample Question Paper 1
f 405 f
'Co n struct a c i rc l e o f ra d i u s 6 cm a n d d raw the B
CT 1
CT2 fro m a p o i n t o utsid e the

~
tan g e n ts and
circle s u c h t h a t CT 1 = 9 c m a n d CT2 = 1 0 cm . '
(i) I s th e g iven c o n struction i s correct?
F2 F1
(ii) If the p o i n t C i s a t the d i stance of 10 units 1 0 km
from t h e c e n t r e , t h e n d raw the tangents
from C and m e a s u re their l ength s . (iii) Which mathematical concept is involved in
the above problem?
(iii) By d o i n g s o , which va l u e is displayed by the
school a d m i n i s tra t i o n ? (iv) By sending its team, which value is
depicted by the fire station f1 ?
29. A fire a t a b u il d i n g B is rep o rted o n telephone
to two fire s t a ti o n s R and Hi , 1 0 km apart from 3 0. Point A i s o n X-axis, point B i s o n Y-axis and
each o th e r o n a s t ra i g h t road . 11 observes that point P lies on line segment AB, such that
the fire i s at a n a n gl e of 60° to the road and � P= (4, 5) and AP : PB = 5 : 3 . Find the
coordinates of point A and B.
observes t h a t it i s a t a n gl e o f 4 5° from it. The
statio n 11 s e n d s i t s t e a m . · 31 . If mid-points of the sides of MBC are (1, 2), (0, 1)
(i) W h y t h e tea m of station F1 was sent?
and (1, 0), then find the coordinates of the
(ii) How m u c h d istance the station f1 team will vertices of MBC.
have to travel?

So luti ons
1 . Consider the arc BCD of the circle. This arc makes Hence, the ratio of volume of sphere to the volume of cube
LBOD = 1 60° at the centre of rhe circle and LBAD at a
is JG :.fie. (1 /2)
point A on the circum ference. 3. We have, a = First term = - 7
.. LEA D = .!._ LBOD =
1 600
= 80° and d = Common difference = 5
2 2 :. � 8th term of an AP , a18 = a + (1 8 - l)d
[ ·: n th term of an AP, an = a + (n - l )d]
[·: ch,. angle subtended by an arc ac the centre
(1 )
is double che angle subtended by ic on
the remaining pare of che circle] = a + 17 d = - 7 + 17 x 5 = 78
Now, ABPD is a cyclic quadrilateral. 4. Possible outcomes of the experiment are 1 , 2, 3, 4, 5 and 6.
.. LEAD + LBPD = 1 80°
: . Total number of outcomes = 6
[·: opposite angles of a cyclic quadrilateral] Lee E be the even t of getting a prime number on a die, i.e. 2 ,
80° + LBPD = 1 80° 3 and 5.
LBPD = 1 00° (1 1 So, the number of outcomes favourable co E = 3
2. Let r be the radius of sphere and fl be the side of cube. : . Probability of getting a prime number
According co the question, Number of favourable outcomes 3
=- =-
Surface area of sphere = Surface area of cube
=
Tocal number of outcomes 6 2
(1 ]
=>
2
= 6a 2 fl 2
4 1tr 2 = - nr 2
=-
2
4 7t r => Note Prime numbers are those numbers which have no
other factor apart from 1 and itself.
6 3
a = J2ic r 5. Let a be the first term and d be the co mmon difference of the
.J3 given AP. Then,
[taking positive square root both sides] pch term = q => a + (p - l)d = q . . . (i)
r .J3 qth term = p => a + (q - I )d = p . . . (ii)
-;, = J2ic [·: nth term of an AP is an = a + (n - l)d] (1]
. . . (i)
(1 /2]

3
i_7t(!:_) 3
4 On subtracting Eq. (ii) from Eq. (i) , we get
Vol ume of sphere 3 7tr = (p - q)d = ( q - p)
Now, = d == - 1
Volume o f cube a3 3 a =>
On putting d = - 1 in Eq. (i), we get a = (p + q - 1 )
[from Eq. (i)] : . nth term, a,. = a + (n - l) d
= (p + q - 1) + (n - 1 ) x ( - 1)
= ( p + q 11)
_
Hence proved. l1 1
i
C lass 1 0th Term II
1 40 6 1 Allinone MAT H E MATI CS

6. Given, radius of a circle = 1 0 cm A C: -' = A H -' + BC -'


( 2 r ) -' = , , -' + ,,
-'

a
Length of the arc of a circle = 57t cm
-'
We know that, length of the arc, I = -- x 2 nr ( 2 r ) -' = 2 ,1
2r ./i ,,

--
360°
a
(1 /2) =

sq uare root both sides]


2r
57t = X27t X 1 0 ( o n c a k i n g pos i c i ve

a
-- a
360° (1 12)
,,
==
/i
=>
57t 1
-= => - = --
2r
.J 2
·-

of rh e sq u a re = -1�. r (1)
207t 360° 4 360°
a = 360 0 = 900
S ide =

,2
·-

4 Arca of c h e c i r c l e m
Hence, the angle subtended by an arc at the centre of a circle Arca of che Sl) l l:He
nr .:?
is 90°. (1 )
7t

r .! 2
2
7. Given, side of cube = 1 2 cm
Volume of cube = (1 2)3 cm3 [·: volume of cube = (side) 3] Hence, rhe rario of c h e a n:as of" rh e circle and the square is
Let the edge of new cube be x cm. 7t : 2. (1)

c:r
:.Volume of new cube = x3 cm3
be 11. Th en ,
(1 ]
1 0. Lee che edge of rhc cube diameter of rhe sphere
According to the question, 8x3 = (1 2)3 char will fit i n s i d e rhc given cubl.· = 11.
=:> x' = (!!)' =:> x' = :. Radius of rhc s p h e re = 11 12
[WI

-
Now, volum e of rhc cub e , /' ==

4
=> x3 = ( 6)3 => x = 6 [on taking cube root both sides] and vo lum e of rhc s p here m· =

3
· \
( !!_) .\
Hence, the side of the new cube is 6 cm.
i 7t ,,.1 1t113
(1 1
- in = =
3 2
8. Given, LPAB = 58° (112)
- 3 8 6


Vol ume of rhc cube
Volume of rhe sphere
,,.� x 6 6
=
p A Q � ;
Hence, che rario of che vol ume of a cube ro char of a sphere
Join OA. LOAP = LOAQ = 90° ... (i) which will fir i nside it, is 6 : 7t. (1)
[·: a line drawn from centre to the 11. We have , volume of the Earth d ug out = 7t 2 h
tangent is perpendicular] 22 x 5 x 5 x 1 4 = I I 00 m .l
=

�m
]
Now, L OAB = LOAP - LPAB 7 (1)
= 90° - 58°
[
[given] d i a mete r
· · radius r =

LJ sm
10
= 32 ° ... (ii) = - = Sm
. . • 2 2
LABO = L OAB
[·: OB = OA = radius of same circle and opposite angles Area of the embankment (shaded region)
= 7t ( R2 _ ,. 2 ) = n ( I 02 - 52 )

.
are equal if opposite sides are equal] (1 )
[·: = 5 + 5 = I 0 m]
R

-
LABO = 32 ° = LABQ . . (iii)
Also, i n MBQ,
22
= - (1 00 - 25) =
22 x 75 m -

LABQ + LBAQ + LAQB = 1 80° 7 7 [1]


[·.· sum of all angles of a triangle = 1 80°) :. Height of che embankment
Volume of the Earth dug our
=> LABQ + ( L OAB + LOAQ) + LAQB = 1 80°
Area of che embankment
[·: LBA Q = LOAB + LOAQ]
1 1 00 7 x I 1 00
=> 32°+ 32° :+- 90°+LAQB = 1 80° 22 x
= 4. 67 m [1)
-- . 22 75
x 75
[from Eqs. (i), {ii) and (iii)] 7
LAQB = 1 80°- 1 54 ° = 26° (1 1 Hence, che height of embankmen t is 4 . 67 m.
1 2. Given, a11 = 5 - 6n
-
9. Let th e radius of the circle be r units.
Then, diagonal of the square = 2r
Le t the side of a sq uare be a.
-
=> aII + I = 5 6(n + I) - I 611 =

Now aII + I - aII (- 1 - 611) - (5 - 6n) = - 6>


=

I n right angled MB C, by Pythagoras


I

· which is independent o f n. (1)


:. { a,, } is a n AP.
theore m
Sample Questio n Paper 1 / 407/
Now, 111 = 5 - 6
11
x

+
I
,
, 11
= - I
II
- + 5 - 611 )
(1 J
=> LOQR = � (1 80° - LQD_R) = � LQPR [from Eq. (i)]
2
s = - ( (/I )= (- I
II •
2 LQPR = 2 LOQR c1 1
= � (4 - II ( 2 - 3 11 ) (1 I Hence proved.
2
611 ) =
Alternate Mechod
1 3. Steps ofco11str11ctio11 LQPR = x
A
Proof Lee
I . Draw a l:iARC. PQ = PR
2. Draw a rar BX , so chac ic
makes an acucl' angle wirh BC
[·: cangents drawn from an external
on chc oppos i t l' si(k of chc B
poinc co the circle are equal]
C :.!lPQR is an isosceles triangle. (1)

( chc grcacer �) LPQR = LP�Q


A. Locarc c h rec
=>
vertex
. . . (i)

of 2 a n <l .3 i n In !lPQR, LPQR + LPRQ + LQPR = 1 80°


[by angle sum propercy of a triangle]
points on IJX as /J1 , IJ2 an<l BJ ,
such chac B/J1 = B1 /J2 = 112 11., . (1 )
=> LPQR + LPQR + L QPR = 1 80° [from Eq. {i)]
3. Join B.1 C and draw a line through B2 parallel co B3C 2LPQR + x = 1 80°
which intersect /JC ar C1 • LPQR = -1 (1 80°-x) = 90°--x . . . (ii)
4. Draw a line C1 A1 p ara l l e l co C./L 2 2
LOQP = 90°
(1 1
Thus, A 1 BC1 is chc required rrianglc each of whose sides is -
2 and
3 [·: radius of a circle is perpendicular to
of the corresponding sides of L1ABC. chc cangenc ac point of contact]
I I CA. LOQR = LOQP - LPQR
( - i) i
c, A I
justificatio11 Si nce, Now,
So, M1 BC1 - L1ABC = 90° - 90° = 90°- 90°+
A, B BC1 = A1C1 �
such that =
AB BC AC 3
= LOQR = lx
[1 )
(1 1
1 4. Givm PQ and PR arc two tangents drawn to a circle with LOQR = !._ LQPR LQPR = 2LOQR
2
=>
centre 0 from an cxrcrnal poi nt P.
Hence proved. [1 J
A
1 5. Let shortest side be AB = x rn .
p Now, by given condicion,
Hypotenuse = (2x - 1 ) m +
xm

and third side = (x + 1 ) m . [11

Then, in given right angled MBC, (x + 1)


C m
B
To prove L QPR = 2 L OQR
Proof Since, PQ and PR arc cwo tangents of the same circle AC2 = BC2 + AB 2
[by Pythagoras theorem]
= (x + I t + x 2
.,
ac points Q and R, respectively.
2
(1 )
.. OQ .L PQ and OR .L PR (2x - I )
=> LOQP = 9 0 ° and L ORP = 90°
'
4 x 2 - 4 x + 1 = x- + 2x + I + x 2
Now, L OQP + L QPR + L ORP + L QOR = 360° 2x 2 - 6x = 0 ::::> x 2 - 3x = O [1 )
[·: chc sum of all angles of a quadrilaceral is 360°] x(x - 3 ) = 0 :::::::> .'( = O or x - 3 = 0
=> 9 0 °+ L QPR + 90°+L QOR = 360° .\' = O or x = 3
=>
=> L QPR + L QOR = 360°- 1 80° Since, x = 0 is noc possible, hence .\· = 3.
=> L QPR + L QOR = 1 80° So, che sides of the trian gle arc 3 , 3 + 1 and 2 x 3 - l,
=> = 1 80° - LQOR . . . (i) c1 1
L QPR i.e. 3 m, 4 m and 5 m. [1 }
1 6. Let the number bey.
In l:!J.OQR, OQ = OR [radii of the same circle]
=> LOQR = LORQ . . . (ii)

According co che ouescion, y + 'V = �


"\J .Y
[·: opposice angles of equal sides are equal] •

=>
25
25y + 25[.V = 6
L OQR + L ORQ + L QOR = 1 80°
=>
[angle sum propercy of triangle]
=> L OQR + L OQR + L QOR = 1 80° => 2sy + 2s f.Y -6=o [1 }
[ from Eq. (ii)] z . . . (i)
2 L OQR = 1 80° - LQOR
Puc
.JY =

,
.Y = z- (on squaring)
Allt�none MATHEMATICS C lass 1 0th Term 11
2
25z2 + 25z - 6 = 0 Now, AB = �(2 - 2) + (5 - 9)2 = .J42 = 4 units
=> 25z2 + 30z -5z - 6 = 0 [by factorisation method] and BC = �(5 - 2)2 + (5 - 5)2 = .J3i = 3 units
=>

. . Area of !:>.ABC = - x AB x BC = - x 4 x 3 = 6 sq units


5z(5z + 6) -1(5z + 6) =0
=>
I 1
(5z + 6)(5z -l) = 0
. "(2x + 3) ( x -3 ) Ill
=> 5z + 6 = 0 or 5z - 1 = 0 2 2
z = - -65 or -5I -- - 25 -- = 5
. .
=>
1 9. G 1ven equation 1s _

(1 )
x -3 2x + 3
On substituting the value of z in Eq. (i), we get Let 2x
-- = y
+ 3 (i) •••
.\" -3
=> fY = !5 => y = ..!...
25 x -3 = -1
-- 11n1
-6
and ..JY = , which is not possible because ..[Y is positive . 2.\" + 3 y
5 Therefore, the given equation reduces to
Hence, the required number is ..!... . 2 · y -25 · -y1 = 5 => 2y 2 -25 = 5y
25 (1 )
1 7. If three unbiased coins are tossed together, then all possible => 2y2 - 5y -25 = 0
outcomes are HHH, HHT, HTH, THH, HIT, THT, =>
TTH and TIT. 2
2 y -lOy + 5y -25 = O
:. Total number of possible ourcomes = 8 (1 )
[by factorisation method)
=> 2 y(y - 5) + S(y - 5) = O
(t) Let £1 = the event of getting atleast two tail. Then,
favourable outcomes are HIT, THT, ITH and ITT. =>

4
(y - 5)(2y + 5) = 0
:. Number of outcomes favourable to £1 = y - 5 = 0 or 2 y + 5 = 0
Hence, required probability = P(getting acleast two tail) => y = 5 or y = -5/2 (11
2x + 3 =�
= 4-8 = -2I Now, put the value of y = 5 in Eq. (i), we get
x -3 I
(1 )
=>

5x - 2x = 3 + 15
{it)Let E = the event of getting atmost two head. Then, 5x - 1 5 = 2x + 3
favourable2 outcomes are HHT, HTH, THH, HIT, THT, =>
TTH and TIT.
3 11n1
=> 3x = 18 => x = - 18 = 6
:. Number of outcomes favourable to £ = 7
2
Hence, required probability = P (getting atmost two head) Again, put the value of y = - S Eq. (i), we get
7 2 in
=- 2x + 3 = -
-- -5
(1 )

-5x + 15 = 4x + 6
8
x -3 2
1 8. In right angled AABC, c (5,5) =>

(2,9) A�: -9x = -9 => x = 1


AB2 + BC 2 = AC2 => -5x -4x = 6 - 1 5
[by Pythagoras theorem] =>

=> [(a - 2) 2 + (5 -9)2 ] Hence, the values of x arc I and 6. (11

+ ((5 - a)2 + (5 - 5) 2 ]
(a ,5)
( 11
:'\ Length of the arc = --
2 1tr e

(iJ
20.
[1)
3600
= [ ( 5 - 2 ) 2 + (5 - 9)2 ] [·.· by disrance formula] 22 60°
= 2 x - x21 x -- 360° = 22 cm [1 )
(a -2)2 + 16 + ( a - 5)2 = 9 + 16 7
=> A
+ 4 - 4a + 1 6 + a2 c
a2 {it) Area of the sector, OA CB
+ 25 - lOa = 25 7tr 20 22 60°
[·: (A - B) 2 = A 2 + 8 2 - 2AB] = -- = - x (21)2 x --
3600 7 360°
2a2 - 14a + 20 = 0 22 x21 x21 x -- 60°
=- . .. (i)
1
2
2
a - 7a + 10 = O [dividing by 2] 360° = 23 l cm (11
=> a 2 - Sa - 2a + 10
=0
(iii) Arca of the segment ACB = Area of the sector OACB
[by factorisation method]
=> a ( a - 5) -2 (a
- 5) = 0 - Area of an equilateral tlOAB . .. (i i)
=> (a - S) ( a - 2) O
=> a - 5 == 0 or a - 2 0
=

=
4
Now, area of an equilateral !!J .OAB = .J3 ( Side)2
=> a == S or a == 2 = .J3 x21 x 2 1 = 1.73 4x441
= 7624· 93 = 190.73 cm 2
Bur a = 5 is nor possible because if we take a = 5, then area of
.
4
mangle becomes zero.
.. a == 2 [1 )
. .. (iii)
Sample Questio n Paper 1 f409/
On subsci rucc chc va l ues from Eqs. (i) and (iii) co = 1 - 6 + 1 1 - 6= 12 - 12 = 0 (1)
Eq. (ii) , we gee a2 = 23- 6 x 2 2 + I I x 2 - 6
Area of chc segme n t A CB = 23 1 - 1 90.73 = 40. 27 c m 2 [1) = 8 - 24 + 22 - 6 = 30 - 30 = 0 (1/2)
2 1 . Lee be c h c cower of heighc h m and
and a = 33 - 6 x 3 2 + 1 1 x 3 - 6
AB be c h e initial C 3
posieion of h c e car and Ice
afcer 1 2 m i n ehe car be ac D. is le = 27 - 54 + 33 - 6 = 6Q - 60 = 0 (1 /2)
given chat the angles of depressio n ac and D are 30° and C Thus, we have a1 = lli = a = 0 [1/2)
3
45°, respectively. We observe thac, an = n 3 - 6n2 + I ln - 6 is a cubic
Lee the speed o f che car be v m/ m i n . Then, polynomial in n and it vanishes for n = 1 , 2 and 3.
CD = Distance eravcl lcd by chc car in 12 min Therefore, by the factor theorem (n - 1), (n - 2)
=> CD = 12 v m [·: d istance = speed x rime] [1 ] and (n - 3) are factors of an . [1 /2)
Suppose, ehe car cakes t m i n co reach the cower AB from D. Thus, we have
vt m an = (n - 1 ) (n - 2 ) (n - 3)

I
Then, DA =
B I n chis expression, i f we subscicuce any value o f n which is
greater than 3, then each factor on the RHS is positive.
Therefore, an > 0, for all n > 3
Hence, first chree cerms of the sequence are zero and all other

�� f
terms are positive. Hence proved. (1)
23. Let ABC be the given isosceles A

- v t- A -
triangle in which
C 12 v_p (1 )
AB = AC = 1 3 cm
In right angled ll.BAD, we have Let BC = 2 x cm
AB Then, BD = DC = x cm _
ran 4 5 ° = --
AD [·: AD is a median] a .___ ___,
O.__ ___. C
h
=> 1 = [·: can 45° = l ]
In right angled ll.ADB ,we have
vt AB2 = AD 2 + BD 2
=> h vt = . . . (i)
[by Pythagoras theorem] [11
AB
In right a n gled ll.BA C, we h ave can 30 ° = => 132 = AD 2 + x2
AC

[ AD]
=>
1
=
h => AD = �1 3 2 - x 2 = �1 69 - x 2
.J3 vt + 1 2 v

,5 an
[on caking positive square root] [11
Area of triangle = 60 cm 2 [given]
·.· can 3 0° =
d A C = CD +
·1
- (BC x AD) = 60
]
=>
.J3 h = vt + 1 2 v ... ( ii ) c 1 1
[-: �
=> 2
area of rriangle = x base x height
On substituting the value of h from Eq. (i) in Eq.1(ii), we get
.J3 Vt = Vt + 1 2 V
=> .J3 = t + 1 2
t [o n dividing by v both sides] => .!. (2x x J169 - x 2 ) = 60
2
=> t ( .f3 - 1 ) = 1 2 => x J1 69 - x2 = 60
1 2( ../3 + 1 ) x2 (1 69 - x2 ) = 3600
= J3 .J3
12
t = _ [rationalising]
.J3
=> _

1 ( 1) ( + I) [on squaring both sides] [11


1 2 ( .[3 + I ) x4 - l 69x 2 + 3600 :;::: 0
2 => x 4 - 1 44x2 - 2 5x 2 + 360 0 = 0
t = 6 c .J3 + 1 ) = 6( 1. 73 + 1 ) = 1 6. 38 m i n [by factorisation method)
t min 23 s
o
=> = 16 => x 2 (x 2 - 1 44) -25(x 2 - 1 44 ) = O
[·: 0.38 min = 0. 38 x 60 s = 22.8 ::::: 23 s] � (x 2 - 1 44 ) (x2 - 25) =
Thus, the car will reach the tower from D in 1 6 min and 23 s.
(1) => o r x2 = 25
x2 = 1 44
x = 1 2 o r ,,. 5 =
We have, an = n3 - 6n2 + 1 I n - 6
=>
22.
[on taking positive square root)
O n putting n = 1 , 2 , 3 , we gee Hence, base = 2x = 2 xl2 = 24 or 2 x S = 1 0 cm. l11
cm
a1 = ( 1 )3 - 6 x 12 + 1 1 x 1 - 6
! 410 '
i
Allinone MATH EMATI CS C lass 1 0th Term 11

24. Let r be the radius of the hemispherical bowl and h be the = Of' � - ( ON :. + A N 2 ) = OP2 - OA 2
height of the cylinder. [·: i n � ONA by Pythagoras theorem,
7 cm ? cm OA 2 = ON 2 + NA1J
= 01'2 - O T 2 . . . (ii)
[· : OA = O T. rad ii of same circlc]l1J
(iii) PA · PB = PN :. - AN 2 [from Eq. (i)]
= Of':. - O T 2 [using Eq. (ii)]
= P T !.
[ · : i n � O TP, O T ..1. l' T b y Pythagoras
2
theorem
01'2 = O T + PT 2 ] 111
Then,
14
r = - = 7 cm and h = 13 - 7 = G em (1 ) 27. (1) Toca I n u m ber o f balls in rhe bag = 1 8
2
Total n u m be r o f red balls i n the bag = x
Total capacity of the bowl = Volume of the cylinder
. . P (getting a red ball) = �
( � ,,, , ) ( H [. ]
+ Volume of the hemisphere [1]
18
= "' ' h + = "'' h+ . p _ favourable outcomes

rob -

2 ( � )
11 1 co ral n u mber of outcomes (1 )
22 32
= x 7 x 6+ x 7 = 22 x 7 x Now, nu mber of red balls added co the bag = 2
:.Total n u m ber of balls in che bag = 1 8 + 2 = 20
7 3 3
4928
= = 1 642. 67 cm3 Total nu mber of red balls in the bag = x + 2
3 [1] x +2
20
:. P (gecri ng a red ball) =
25. Suppose, t h e level of che water in che rank will rise b y 7 cm "�
in x h. Since, che water is flowing ac che race of
5
km/h. Accord i ng t o c h e q uest i o n ,
Therefore, length of the water flow in x h 2 x+2 x
= x km = 00"·: m
5 5
[·: l km = l OOOm] � �(;�)
.\"2
= => 20 16
=> 20x - l 6x = 32

; � [
Clearly, the wacer flow from a cylinder whose radius, 1 6x + 3 2 = 20x

� ]
1 32
r = cm = m ·: ! cm = m 4x = 32 => x = - =8
1 0 1 0 11 /2] 4
Length = h = 000x m
5
Hence, the val ue of x is 8 .

( J2
:. Vol um e of the water flowing through the cylindrical pipe 28. (1) The given conscruccion is wrong, since from an
22 exterior poi n t two tange n ts of differe n t length cannot
in x h = 1t r2 h = - - 000 x be drawn co a c i rcle. The rwo tangents char can be
7
5
x x
1 00
drawn from an exterior poi n t to a circle are always of
7
22 _]_ x __?__ x 000 x = 77 x m 3 the same length . p y,J
5
x

1 00 1 00 (ir) Given, a ci rcle of radi us 6 cm whose cencre is 0 and


=
7
Also, volume of the wacer char falls into the tank in x h
(1 )
a point P, 1 0 cm away from i ts centre.
Steps of Construction
= 0 x 44 x _J_ = 1 54
5 1 00
m3
[1 /2 ) 1 . Draw a circle wich 0 as cen tre and radius is equal to
B u e volume of che wacer flowing through che cylindrical 6 cm .
pipe in x h 2. Take a poin t C s u c h c h a r OC = 1 0 c m a n d bisect i t . Let
= Volume of the wacer chat falls in che rank in x h l1J M be the mid-po i n t of OC.
77 x = 1 54 => x = - = 2
3. Taking M as centre and MO as radi us, draw a dotted
=>
1 54
77 circle. Lee this circle cuts the given circle at A and B.
4. Join CA and CB.
Thus, CA and CB are rhe required tangents. By
Hence, the level of the water in the rank will rise by 7cm in
2 h. [1 ]
measurement (using scale) , CA = CB = 8 cm. P 'hl
26. From the figure, ON .l AB => AN = BN
[·: perpendicular drawn from centre co the chord
of circle bisects che chord] [1 J
(i) PA · PB = ( PN - AN) ( PN + BN)
= ( PN - AN) ( PN + AN) [ ·: AN = BN]
= PN 2 - AN 2 . . . (')
l [1 ]

(iz) PN 2 - AN 2 = ( OP2 - ON 2 ) - AN2


[·: i n l!J. ONP. ON .l PN, by Pythagoras theorem (jir) Justice. (1)
OP2 = ON2 + PN2 ]
Sample Question Paper 1 'I 411 .i
29. Lee BL be che p erpendicular from B on
B
Fi F2 • Now, P(4 , 5) = p(3: , 5:)
On comparing boch sides, we get
=>
x 3 =4
=> x =
x8 4 32
=
(1 )

--- --
F2 1 0 km- · F,
and
5-y = 5 y -
8
=>
53 = 3
=
x8
8

(�2 , )
8 5 .
(1) ·: BL J.. gF1F2c a g LLF1 B = 60°, LLF,B
From ri h
and
n le d M·; I.fl a nd !::,. F,
=45°
LB, �vc have : . Coordinates of point A and B are O and (0, 8),

BL [ ·: sin p ]
P
. respeccively. (1)
0=
BL
s i n 60 0=- Fi B an sin45°
d = --
F2B H 31 . Lee (1 ,2), Q (0 , 1 ) and R ( 1 ,0) be che coordinates of the

F.B = sin-- BL BL mid-poincs of sides AB, BC and CA, respectively of/). ABC.
60° F,B- = sin45°-- and
A (x1 .Y1 )
·: s i n 0 i n c re as 0 i ncreases from 0° co 9 0°.
ases
:. sin 4 5° < sin 60°
BL BL
- .-- > - F,, B
.-- or F,- B >
s 1 11 4 5 ° s111 60°
:. D iscance of B fro m F2 is more
So, che scacion /-� senr ics co chan char F,, . of from

60° !=i !J [·: sO .!!...]


rea m B. (1)

( it) I n righc a gl n e d !::,./� LB, cos = Fil co =


H
Lee checoordinates of vertices of MBC be A
che ( x1 ,y1 ),
Let F.B be x k m . B (x2 , Y z ) and C(x 3 ,y3 ). [1 ]
. 60o
BL
x cos 6 0 ° = F.L =- BFi sm . . . (i) . . . (i)

[ x, : J
[ ·: can0= -P ]
=> BL = => BLBL=
BF.. si n 60° x s i n 60° . . . (ii)
x -coordinate of mid -point =
x
,

ln nghc!::,.F2LB,can45 F2L 0 == --

B Y1 + Yi .. .
---2 = 2 => Y 1 + Yi =4
!J_.; J
(ii)

F2L =
. . . (iii)
[ y -coordinate of mid -poin t =
Yi

= IO => [ & + �] =
From Eqs. ( i i ) a n d ( i i i ) , we gee , x sin 60°
F.L F2L = IO X2 + X3
=>Now,x o + + I O
. . . (iii)
= 0 => x 2 + x3 = 0
c s 60° x sin 60° = 2

[· �]
Yi + Y 3 = I =::> y,, + 3 = 2 . . . (iv)
=> x [ o + c s 60° s i n 60° ] x 10
2 J �

&
=> x[ I +/3 J = I O => [ I + �732 ] IO
X 3 + x1 I =::> X3 x = 2 . . . (v}
= + 1
: co s 60° = and sin 60° = 2
. . . (vi)
J3 + Y1 = 0 =::> Y3 + Y1 == 0
and (1)
2
x =
On adding Eqs . (i), (iii) and (v}, we gee

=> x2.732 = 20 => = -- 2 => x = 7.32 0 2(x1 + X2 + X 3 ) =4


x x km x 1 + X2 + X 3 =2 . . . (vii) (1 ]
2.732 (1 )
=::>
,
team
:. Station Fi cravclled 7.32 km. On adding Eqs. (ii), (iv) and (vi) we gee
Trigonometry [Heighcs and
(iiz) D is cances] (1 ]
2(J1 Yi Y3 = 6 + +
)
(viii)
P ro p tne .
(iv) m ss [1J => Y 1 + Yi + Y 3 = 3 ...

30. Coo r na e of any po


di t s o is (x , O). y).
inc A n X-axis
Now, subtracting Eqs. (i) , (iii) and (v) from Eq. (vii) ,
we get
And coordinates of any point B o n Y-axis is (0,
X1 = 2, X 2 = 0 and X 3 == O
Point P lies on the line segment AB such AP: PB = 5 : 3 as
5 3
[l ) On suberaccing Eqs. (ii), (iv), (vi) fro m Eq. (viii),
A (x,O) P(4, 5) 8 (0,y) [1 1
we get y1 = l
Y2 = 3 and y3 = - 1
n formula, we have che P l11
By sec ti o coordinaces of point is
(5 xO + 3 Sy + 3 xO)
x (3x , Sy ) Hence, the coordinates of the vertices of ehe MBC
5+3 S+3 , i.e.
8 8 are A (2 , l ) , B (O, 3 ) and C (O , - 1).
SAM PLE QU ESTI O N PAP E R 2
A Sa m p le Qu esti on Paper for CBSE Class X Su m m ative Exa m i n ati o n II

MATH EMATICS
General I nstructions
1. All q uestions are compulsory.
2. The q uestion paper consists of 31 questions divided i nto 4 Sections A, B, C a n d D.
Section A comprises of 4 questions of 1 mark each, Section B com p rises of 6
q uestions of 2 marks each, Section C comprises of 1 0 questions of 3 ma rks each a n d
Section D comprises of 1 1 questions of 4 marks each.
3. There is no overall choice and use of calculator is not permitted.

Time : 3 hours Max. Marks : 90

Section A 7. Two tangents PA a n d PB a re d rawn to the


circle with centre 0, such that LAPB = 1 20°.
1 . An unbiased die is thrown. Find the probability
Prove that OP = 2 AP.
of getting a number between 3 and 6 .
8 . How m any cubes of s i d e 3 m c a n be cut from a
2 . I n a n AP , if a = - 1 0, n = 6 and an = 1 0, then find cuboid measuring 1 8 m x 1 2 m x 9 m ?
the valu e of d .
3. If the perimeter and the area of a circle are 9 . Find t h e area of a quadrant of a circle, where
numerically equal, then find the radius of the the circumference o f the circle is 44 cm.
circle . (take, 7t = 22 I 7)

4. I n the given figure, if LDBC = 70° and 1 0 . Two cubes, each of side 4 c m are j oined end to
end . Find the surfac e a re a of the resulting
LBAC = 40 °, then find the value of LBCD.
cuboid.

Section C
1 1 . In the given figure, AB a n d
PQ are p erpendicular
diam eters of the circle
whos e centre is 0 and radius Q
@B
Section · e
P
OA = 7 cm. Find the a rea of
the sh aded region.
5. I f the radius and height o f a cylinder are i n the (take, 7t = 22/7) A
rat io 5 : 7 and its volume is 550 cm3 , then find
its radi u s. 1 2. How many terms of an AP, - 6, --, -5, . are
11
. .

6 . Whi h term of the sequence 2


c
�, 1 1 7 3 . . . is the first neg ative needed to give the sum - 2 5 ? Explain the
ZO, l 9 18 , 4, double answer.
Z
te rm?
Sample Question Pap er 2 I 41 3 f

1 3. S olve the fol lowi ng quadratic equation by 24. A solid iron rectangular block of dimensions
factorisation metho d . 4 . 4 m, 2.6 m and 1 m is cast into a hollow
+1 34
-- + -1
cylindrical pipe of internal radius 30 cm and
; � �
x x
x+1 15
-- = - x 0, x thickness 5 cm. Find the length of the pipe.
x

1 4. A right circular c o n e i s 3 . 6 c m high and radius 25. A lead pencil consists of a cylinder of wood
of its base is 1 . 6 c m . It is m elted and recast into with a solid cylinder of graphite filled into it.
a right circula r con e with radius of its base as The diameter of the pencil is 7 mm, the
1 .2 c m . Fin d its height. diameter of the graphite is 1 mm and the length
of the pencil is 10 cm. Calculate the weight of
1 5. A s hopkeeper buys a n u mber of books for the whole pencil, if the specific gravity of the
\' 800. I f h e h a d bought fou r more books for the wood is 0.7 g/cm3 and that of the graphite is
same amou nt, each book would h ave cost him 2 . 1 g/cm3 •
' 1 0 less. H o w many books did he buy?
26. 1\vo stations due South of a leaning tower
1 6. In the given fig u re, PT is a tangent and PAB is which leans towards the North are at distances
a secant. I f PT = 6 c m a n d AB = 5 cm, then find a and b from its foot. If a and J3 are the
the length of PA . elevations of the top of the tower from these
stations, then prove that its inclination e to the
horizontal is given by
p 9 b cot a - a cot J3
cot = .
b-a
B 27 . Draw a circle of radius 4 cm, take a point P
1 7. Given a line s e g m e n t A B . Divide it in the ratio outside the circle. Construct the pair of
m : n, whe re both m and n a re p ositive integers tangents from this point to the circle. Also,
a n d let m = 4 a n d n = 3 . justify it.

1 8 . I f the point (x, y) is equidistant from the points 28. The lower window of a house is at a height of
(a + b, b a) a n d ( a - b, a + b), then p rove that
-
2 m above the ground and its upper window is
bx = ay. 4 m vertically above the lower window. At
certain instant, the angles of elevation of a
1 9 . A b a g c ontains 3 red and 2 blue marbles. A balloon from these windows are obs erved to be
marble is d rawn at random. What is the 60° and 30°, respectively. Find the height of the
probability o f d ra wi n g a blue marble? balloon above the ground.

n(-! �) F(I,IJ
20 . Determine the AP, w h o s e fourth term i s 1 8 and 29. Find the third vertex of an equilateral triangle
the d iffe re n c e of the ninth term from the whose two vertices are (6, 0) and (3, 0).
fifteenth term is 30.
30. If E (7, 3) and are the
• 2 '2 . 2 2
Section D mid-points of sides of MBC, then find the area
of M BC.
2 1 . A takes 6 d ays l e s s than the time taken by B to
finish a piece o f work. I f b oth A a n d B together 3 1 . A carton consist of 200 shirts of which 176 are
can finis h it in 4 d ays , then find the time taken good, 1 6 have minor defects and 8 have major
by B to finis h t h e work. defects. A trader Jyoti will only accept two
shirts which are good while another trader
22. I f there are (2 n + 1) terms in an AP , then prove Riya will only reject the shirt which have major
that t h e ratio o f t h e sum of odd terms and the defects. One shirt is drawn at random from the
sum o f even terms is (n + 1) : n. carton.
23. ABCD is a cyclic quadrilateral and PQ is a (i) What is the probability that it is acceptable to
tangent to the circle at C. I f BD is a diameter, (a) Jyoti?
LDCQ = 40 ° a n d LABD = 60 °, then find the (b) Riya?
measure of (ii) What value is indicated from the above
(i) LDBC (ii) LBCP question?
(iii) LBDC (iv) LADB
So lutio n s
1. In a single throw of a die, we can gee any one of che six
numbers l , 2, 3, 4, 5, 6 marked on ics six faces. Therefore, =:> x = 1 cm [on caki ng cube root both sides]
total number of elementary events associated with the Hence, radius of chc cylinder = 5x = (5 I ) = 5 cm [ti
x
random experiment of ch rowing a die is 6.
The event of getting a number between 3 and 6 occurs, if 6. Given sequence is an A P in which firsc cerm, a = 20 and
we obtain either 4 or 5 as an outcome. .
common d1ffcrcncc, d = 1 9 - 20 =
1
- -
-3
77 - 20 = -
:. Favourable number of outcomes = 2 4 4 4

Hence, required probability = 3. = .!_ Let mh ccrm of given AP be chc firsc negative term.
6 3 (1) Then, a,, < 0
2. Given, a = -10, n = 6 and a,, = 10 => a + (11 - 1 ) d < 0
We know char, a,, = a + (n - l)d 20 + (11 -1) (-3 I 4) < 0 =:> 834 - 3114 < 0 (1)
x

=:> 10 = -10 + (6 -l)d => 83 -311 < 0


=:> 20 = 5d
20 = 4 => 311 > 83 [dividing by 4)
d=- 83 · 2
5 => 11 > - =:> 11 > 27 3
_;

Hence, the common difference (d) is 4. [1)


3
3. Lee r be the radius of che circle.
Since, 28 is the nacural number just greater than 27�.3
• • 11 = 28
According to the question, Hence, 28ch term of che given sequence is che first
Perimeter of a circle = Area of a circle negative term. (1)
21tr = m· 2 7. Given A circle C ( 0, r). PA and PB are cwo tangencs co
=:> r2 - 2r = 0 che circle from P outside the circle such chat
=:> r (r - 2) = 0 LAPB = 1 20°. OP is joined.
=> r = 0 or r = 2 A
But r = 0 is nor possible.
r = 2 units
Hence, che radius of che circle is 2 uni cs. [1)

4 . Consider che chord CD. We find chat LCBD and LCAD


are angles in che same segment of the circle. Toprove OP = 2 AP (1 12)
.. L CBD = LCAD = 70° Construction Join OA and OB.
Now, LBAD = LBA C + L CAD Proof In tl. OAP and 6. 0BP, we have PA = PB
=:> LBAD = 40° + 70° [·: tangents co a circle from a point outside
= 1 10° ... (i) che circle are equal]
Since, ABCD is a cyclic quadrilateral. OP = OP [common sides]
Therefore, LBAD + LBCD = 180° L OAP = L OBP = 90°
[·: sum of each pair of angles in cyclic [·: radius is perpendicular co the tangent of che circle]
quadrilateral is 180°] tl. OAP ::: 6. 0BP [by RHS rule] l 1 121
=> 1 1 0° + LBCD = 180° [using Eq. (i)] LOPA = L OPB [ by CPCT]
=> LBCD = 70° (1)
Then, LOPA = L OPB = _!_ LAPB = _!_ 120° = 60°

base ]
x
5. Lee che radius of che base and height of che cylinder be 2 2

['.' cos9 = hypotenuse


5x cm and 7x cm, respectively. AP
In right angled l:lOAP, cos 60° = OP
Then, volume of cylinder =550 cm3 [given]

[-: cos60° = �]
=:> 7tr 2 h = 550 [·: volume of cylinder = nr2 h]
2 2 (5x) 2 7x = 550
x x
==> [1 )
7 AP
[using r = 5x and h = 7x] => -=-
2 OP
22
x 25x 2 x 7x = 550 => 22 x 25x3 = 550 => OP = 2 AP Hence proved. [l)
7
Sample Questio n Paper 2

8. We have, edge of each cube = 3 m 1 2. Here, a = -6 and d = --1 1 -(-6) = -1


:.Volume of each cube = (Edgc)3 = (3)3 = 27 m 3 [1 12) 2 2
Lee -25 be che sum of n terms of chis AP, where n e N.

-25 = !!..2 [2(-6) + (n -1) (..!.2 .)]


Volume of che cuboid = 1 8 x 12 x 9m3 [l /2)
Using, Sn = !!.. [2a + (n -I)d]
[·.· volume of che cuboid = l x b x h]

_50 =n ( -24 ; n-1 )


2-
Now, num ber o f cu b cs = Volume of che cuboid �)
Volume of each cube

-5 0 =n ( n �25 )
x x
= 1 8 2712 9 = 18 x
72 4 =

Hence, chc number of cubes arc 72. (1 ]

9. Given, circumference of che circle = 4 4 cm


� 27tr = 44 [·: r = radius of a circle] -100 =n2 -25n
22 44 7 x
� 2 x -7 r = 44 =:> r = ---
x
2 x 22 = 7cm [ 1 ) �
n2 - 25n + 100 = 0
n 2 -20n -5n +100 =0
:. Area of a quadrant of circle = _!_ 7tr2
a � n(n -20) -5 (n -20) =0
4
� (n -5) (n -20) = O
= -41 x -22 x 7 7 = -
7
77 = 38.5 cm 2
x
2 n =5 or 20

[2 (-6) + (20 -1) x 21 ]


Boch che values of n are natural numbers and therefore
Hence, che area of a quadrant of a circle is 38.5 cm2• [1) admissible. (1 )
1 O. Given, edge of a cube = 4 cm Expklnatiori ofDouble Answer,

[": Sn = % [2a + (n -l)dJ


·: Two cubes arc joined end co end, chen a cuboid is Now, S20 = 2
20
formed.

=10 [-12 + 2
19 ] =10 [ -24 +19 ]
So, length of a cuboid (! ) = 4 + 4 = 8 cm
Breadth of a cuboid ( b) = 4 cm
Height of a cuboid (h) = 4 cm (1 )
:.Surface area of a cuboid = 2(/b + bh + hi)

H2 (-6) + (5 -1) iJ=U-2�+ 4]


= S x (-5) = - 2 5
x x
= 2(8 4 + 4 4 + 4 x 8) 2
= 2(32 + 16 + 32) = 2 x 80 = 160 cm 2
Hence, the surface area of a resulting cuboid is 160 cm 2 . [1 J and S5 =
11. From the given figure, diameter of che smaller shaded
circle = Radius of larger circle (1j ) = 7 cm = 5 x (-20) = 5 x (-5) = -25
(1)
4
:.Radius of smaller circle (r) ?._ = 3.5 cm
= So, here double answer is possible.
2 x +l 34
Then, area of che smaller circle = 7tr2 1 3. We have, x x+ l + x = -
-- 15
22 x ?.... x ?.._ = 77 =38.5 cm 2 (i)

-
= •••

7 2 2 2 (1 ) + (x-+-
x 2 _...;. 1)2 = -
34
(1 12)
_

Area of the semi-circle APBOA _!_ 7t 1j2 == x (x +I) 15


=
1 x- 22 x 7 x 7 = 772 cm 2 ... (ii) x2 + x2 + 2x + 1 34
2 7 x2 +x 15
Area of the MPB [·:(a + b)2 =a2 + b2 + 2ab)
= _!_ AB OP = _!_ x 14 x 7 = 4 9 cm 2
x � 34x2 +34x =15x2 + 15x2 + 30x + 15 (1 /2)
2 2
[·: PO ..l AB] ... (iii) ] 11 1 � 4 x2 + 4x -15 =0
Area of the shaded region APB => 2
4x + IOx -6x -15 = 0 [by factorisation] r1121
= Area of the semi-circle -Area of che 8APB => 2x ( 2x + 5) -3 ( 2x + 5) = 0
= 77 - 49 = 28 cm2 => (2x -3) (2x + 5) = O [1 /2]
Hence, total area of che shaded region =38.5 + 28 => 2x - 3 = 0 or 2x + 5 = 0
= 66.5 cm 2 (1 ) => x = 3 1 2 or x = - 5 1 2 lll
Allin.One MATH EMATICS C lass 1 0th Term 11

1 4. We have, ( OP ) 2 = 6 2 + r 2

First cone Second cone ( 0P ) 2 = 36 + r 2 . . . (i) [tn)

Radius r, = 1.6 cm r2 = 1 .2 cm In right angled � OC�l .


Height h1 = 3.6 cm h2 = ? ( 0C ) 2 + ( A C ) 2 = ( 0A ) 2

(%)
]
v,

[
Volume V2 '
(1 1 � (OC J ' + = r'

and OC .L ,-JB � A C = BC = �
Here, second cone will form by melting the first cone. So,
volume of both cones will be in equal. ·: OA = r
1 2 1
V. = Vi =>
3 1t1j 11 = 3 7tr22 h2 (1 )
=>
- r2
{ Oc ) 2 - - -
25
()
. . . II
"
(tnl
=>
1j2 l1 _
4
h2 -- In right angled t:,. OC/'.
'22
1 .6 x 1 .6 x 3.6 = 16 x 16 x 36 = 6.4 cm OP 2 = ( 0C )+ (PC) 2 2
=> h2 =
1 .2 x l .2 12x 1 2 x 10 (11
=>
., 2 5
36 + ,. - = r- - 4 + ( PC t
., .,
1 5. Let the number of books purchased for ' 800 be x.
. 800 [by using Eqs. (i) and (ii}]
Then, cost of each book = , _ . . . (i) (1 12) -2 5
36 = 4 + (PC ) .,
x => - -

If he buys four more books, then number of books


36 = - 25 +(PA + AC) 2

( 2
= (x + 4 ) =>
4

J
and cost of each book = ' � ... (ii) (1 121
(x + 4 ) [·: PC = PA + AC]

According to the given condition, -25 y + 5


36 = --
=>
800 - 800 = I O 4 2
- --
x (x + 4) + y ., + 4 + 5y
-25 25
800x -800x
(1 12) 36 = 4 -

=>
+ 3200 = 10
x (x + 4) [·: (a + b) 2 = a 2 + b 2 + 2ab]
lOx (x + 4) = 3200 => y 2 + 5y - 36 = 0
2
10x + 40x - 3200 = O => y 2 + 9y - 4y -36 = 0
=> x 2 + 4x -320 = 0 [dividing by 10] => y (y + 9) - 4 (y + 9) = 0
=> x 2 + 20x - I 6x - 320 = 0 [by factorisation] => (y + 9) (y 4) = 0 -

=> x(x + 20) -16 (x + 20 ) =O => y = 4 or -9 (11

=> (x + 20) (x -16) =O Since, length cannot be negative.


=> x + 20 = O or x -16 = 0 (1 J .. y = 4 cm
x = -20 or 1 6 Hence, PA =4 cm (1'21

Bue number of books cannot be negative 1 7. Steps ofconstruction


So, on neglecting x = -20, we get x = 16 1 . Draw a line segment AB and then draw any ray AX
Hence, he purchased 1 6 books. (1 /21 making an acute angle with AB.
A
A 7
1 6. Given, X
As 6
P T = 6 cm , AB = 5cm A4
A3
Join OT, OA and OP. A2
A,
Draw OC ..L AB . B
.Q"'--___._____. B
Let radius of the circle = r cm [·: OT = OA = r em ]
and PA = ycm 2. Locate 7 (m + = 4 + 3 = 7) points A 1 , A 2 , ... , A7
n on

Here, OT ..L PT AX: such that AA 1 = A 1 A 2 = ... = A 6A 7 •


[·. · rangent at any point of a circle is perpendicular to 3. Join A7B. (11
the radius through che point of contact] 4. Through the poinc A4(m = 4), draw a line parallel to
:. In right angled D. O TP , [by Pythagoras theorem] A7B by making an angle equal to L AA7B at A4
( OP)2 PT 2 + (OT)
=
2 intersecting AB at a point C. (1)
(1 121
Then, AC: BC= 4 : 3 ltl
Sample Questio n Pap e r 2
/ 417 /
1 8. Lee T (x, y ) , Q ( 11 + b, b - 11 ) and R (a - b, a + b) be che 21 . Suppose, B alone rakes x days ro finish the work. Then,
given points and T is cquidisranc from che points A alone can finish ic in (x - 6) days.
Q and R. (A's one day's work) + (B's one day's work) = -1 - + .!.
Then, TQ = T R [given] x -6 x
=> �[x - ( a + b ) ] 2 + [ y - (b - a ) ] 2 ·. ( A + B )'s one day's work = .!_4 [given]

·

(1 )
= )! x - ( 11 - b ) ] 2 + [y - ( a + b ) ] 2
�����-----

l 1
- + -- = -
1

[ by discance formula] ( 1 )
x x -6 4
On squaring b o c h sides, we gee ---
x -6 + x = l
[ x - ( a + b ) ] 2 + [ y - (h - 11 ) ] 2
x (x -6) 4
[ x - ( 11 - b ) ] 1 + [y - ( a + b ) ] 2
=
2x -6 = I
x2 -6x 4
=> x2 -2x ( a + b ) + ( a + h ) 2 + y 2 - 2y (b - a ) + (b - a )2 Bx -24 =x2 -6x [1]
= x 2 + ( a - b ) 2 - 2 x ( a - h ) + y 2 - 2y ( a + b) + ( a + b ) 2
(1 J
=> x 2 -14x + 24 =0
=
=> -2x ( a + b ) - 2y (b - a ) -2x ( a - b) - 2 y ( a + b ) => x 2 -12x - 2x + 24 = 0
=> x(a + b ) + y ( b - 11 ) = x( a - b ) + y ( a + b) [by faccorisarion merhod]
[dividing by (- 2) b o ch sides] x(x - 1 2) - 2 (x - 1 2 ) = 0
=> � + � + � -� = � -� + � + � => (x - 1 2) (x - 2) = 0
=> 2 bx = 2 ay � bx == ay (1) . => x (x - 1 2) - 2 (x - 1 2) = 0
Hence proved. x = l 2 or x = 2 [1]

1 9. . There are coca! 5 marbles in che bag. Our of chese


Bue x cannot be less than 6.
5 marbles, one can be chosen in 5 ways. .. x =12
:. Toca) number of clcmcncary ev nt = 5 e s
(1)
Hence, B alone can finish rhe work in 12 days. [1 1

S i nce, the bag contains 2 blue marbles. Therefore, one 22. Ler a and d be the first term and common difference
blue marble can be drawn in 2 ways. (1 )
respecrively of rhe given AP. Again, lee ak denotes rhe krh
:. Favourable number of clcmencary events = 2 term of che given AP.
Then, It = 11 + ( k - I ) d
a
Hence, required probabilicy P (getting a blue marble) = � Ler 51 = Sum of an odd terms
5
[1 ) = a1 + 113 + ll5 + . . .+ a2 + l
[1]
n
20. Lee a b e the first term and d be che common difference n+I
= -- { a 1 llin+ . 1
+
of an AP. 2
Given, fourch cerm, a4 = 1 8 [·: num ber of an odd cerms = n + I]
n+1 {
=> a + 3 d = 1 8 [·: a11 = 11 + (n - I ) d] . . . (i) = a + 11 + ( 2n + 1 - l) d}
and difference of chc ninch re rm from fifreench cerm = 30 2
[·: a2n+ l = a + (2n + 1 - I ) d and a1 = a ]
[1)
=> a1 5 - a9 = 30 = (n + 1 ) ( 11 + nd) [1 )
=> a + I 4d - ( a + B d) = 30 an d 5 = S um of even rerms
a + I 4d - a - B d = 30
2 n
=>
= 112 + 114 + a6 + . . . + = 2 [ a z + '1211 1
lli n
=> 6 d = 30
[·: number of an even terms = 11]
30
=> d = =5 n
6 (11 = - [(11 + d) + { a + ( 2n - l) d}]
On puccing the value of d in Eq . (i), we gee 2
[·: 112,, = 11 + ( 2n - l ) d and a 2 = a + d]
a +3 5 = 18
= n (a + nd) c11
x

=> a = l8 - 1 5 = 3
Now, 51 : 52 = (11 + 1 ) ( a + nd) : 11 (a + nd) = (11 + 1 ) : 11 C1 J
Hence, the req uired AP is 3 , (3 + 5 ) , ( 8 + 5 ) , . . . i.e. 3,
8, 1 3, 1 8, . .. . [1 ) Hence proved.
418 l �
All/non e MATH EMATICS Class 1 0th Term 11

23. (i) Here, LDCQ and LDBC are angles in rhe alrernatc . . Voluml.' of iron in chl.· pipl.'
segmenrs. = Ex te rn a l vol u m l.' - Inrerna.I volume
= rrR 2 h - nr2h = rr( R 2 - r :i. )h
= rr ( R + r ) ( R - r ) h
= T[ X (35 + 30 ) X ( 3 5 - _10 ) X /J
= rt x 6 5 x 5 x h cm \ (11
p
Now,
Voluml.' of iron in the pipe = Volume of iron in the block
Join OC. => rt x 65 x 5 x h = 4 4 0 x 260 x I 00 (11

Given, LDBA = 60°, LDCQ = 40° => - x 65 x 5 x h = 440 x 260 x I 00


22
Now, from figure, since BD is diamerer. 7
7 I I
LBAD = LBCD = 90° h = 4 4 0 x 260 x I OO x x -x--

2 2 65 5
· ·

[·: angle in a semi-circle] h = I I 200 cm = I I 2 m


Now, in MDB, we ger Hence, the lengrh of rhl.' pipe is I 1 2 m. 111

[-: cm]
LBAD + LDBA + LADB = 1 80°
[angle sum properry of rriangle] a 25. We have, diamccer o f chc graphicc cylinder = Imm
I mm = �
=> 90°+ 60° + LADB = 1 80°
= IQ cm
=> L ADB = 1 80 - 1 50° 1
... (i) 1
=> L ADB = 30° Radius = -20- crn

Now, since OC .L PQ
[·: radius is perpendicular co tangenr] Length of rhc graphite cylinder = I 0 cm
LOCD + LDCQ = 90° Volume of the graphite cylinder
=> LOCD + 40° = 90° 22 x _I_ x -1- x I 0 cm3
=
=> LOCD = 90°- 40° 7 20 20
=> L OCD = 50° . . . (ii) [·: volume of cylinder = 7tr2h] 11n1
Now, OC = OD [radius ofcircle] Weight of graphite = Volume x Specific gravity
LOCD = LODC 22 I I
= - x - x - x 1 0 x 2. I
[angle opposite to equal sides are eq ual] 7 20 20
L ODC = 50° [by Eqs.(ii)] 22 I I 21
= - x - x - x I O x - = 0. 1 65 g 111
LBDC = 50° . . . (iii) 7 20 20 10
In 6DCB, Also, given diameter of pencil = 7 mm
LBCD + LBDC + LDBC = 1 80° = -7 cm
[angle sum property ofa triangle] IO
=> 90°+ 50°+ LDBC = 1 80° [by Eq. (iii)] :. Radius of pencil = .!_ cm

( J
=> LDBC = 40° . . . (iv} 20
and LDCQ + LDCB + LBCP = 1 80° [linear pair] and length of pencil = 1 0 cm
40°+90° + LBCP = 1 80° :. Volume of pencil = 22 x .!_ x ..!_ x 1 0 cm3
LBCP = 50° . . . (v} 7 20 20 (1121
Hence, Now, volume of wood = Volume of pencil - Volum of e
(t) LDBC = 40° [by Eq . (iv)] 11 1 the graphitic cylinder
(iz) LBCP = 50° [by Eq. (v)] c1 1 =-22 x 7 7 22 1 1
- x - x 10 -- x - x -x to
(iiz) LBDC = 50° [by Eq. (iii)] c1 1 7 20 20 7 20 20
(iv) LADB = 30° [by Eq . (i)] 11 1 =-22 1
x-x - I x IO ( 7 x 7 - 1 ) = -
1 1 I 48
x -x
7 20 20
Ler the length of the pipe be / cm. Then, volum e of iron
7 20
24.
in rhe p ipe is equal to the volume of iron of the block. = 3. 77 cm3 111

We ha ve, vo lume of the block =(4.4 x 2.6 x 1 ) m3 :. Weight of wood = Volume x Specific gravity
= 3.77 x 0.7 = 2.64 g
= ( 440 260 x 100 ) cm3
x [·: 1 m = 1 00 cm] Hence, weight of the whole pencil = 0. 1 65 + 2.64
Now, = I n ce n aJ radi us of the pipe =30 cm
r r . = 2.805 g 111

an d R = External rad ius of the pipe = (30 + 5) = 35 cm 11 J


Sample Question Paper 2 1 41 9 /
26. Lee ABbebecwoche lean i ng cower and 3. iDraw perpendi cularatbisectors AR
of is theandcencre which
MT
C, D given stations ac ncirtersect each ocher 0. Then, 0 of the
fooc A of pccc i c from
distances a and b rc s v ly
cle. (1 )
che chc cower. 4. Take
it. a point P outside che circle. Join OP and bisect
Lee AE = x and BE = h
In we 5 . Lee Q be cheng mid-poicentre
nt of

[ . can = perpendicul
!l.AEB, have
as
BE
can 0
OP. Taki Q
a -
base J
=-
AE
and OQ asciradi
docced r cl eus, draw
whi ch p,...,,... �-+-r----t

0
. ar Sintersect
and N.the given circle at

= h co [·: _I_0 = e] ... (i)


can f)= -xh 6. Join
tangentsPS andfromPN.P. Thus, PS and PN are the required (1 )
justification
X t 0 CO C Join
BE
can [1 J semi-OScircle.andSo,ON.LOSP Then,=90°.LOSP is the angle lies in the
In � CEB, we have can a=- CE
=>
Since,
OS..1-SP
OSaistangent
the raditoustheof thecirclcierclwietwih centre
th centre SimSo,ilarlSPy,
0.

=> a = --
can
h
+x 11
has to be
PN is also a tangent to the circle with centre at 0.
0. (1 )
=> 11 + x = hcoc a 28. Let AG ofbe chethewiheindows. ght of che balloon and be che C, D
=> x = hcota - 11 ... (ii) [ 1 12) position
AcHere,points and= manglande of ele=vati4 mons are
In we�DEB, have
BE
BC 2 CD
f3 = C D,
DE
can -­

LECG = 60° and = 30°


LFDG c1 12 1
canf3 = +h x -­

b G
t

4 if-x
=> b + x = hcocf3 h m
=> x = hcocf3-b . . (iii) (1 12)
t
From Eqs. (i) (ii),and we get
hcocO = hcoca - 11
=> h(coc - cot = a
a 0)

h = ---11 _
__ . . (iv) E r+======:;(==trr+C(Lower window)
coca - cocO 2m ! fm
From Eqs. (i) and (iii), we get- A B
0
hear = hcocf3 b l i n e and FD on AG.
Also, l e t
[. . tan perpendi
Draw a perpend i c ul a r EC

base J
=> h (cocf3- coc0) = b CE = DF =xm and GF =hm. r1 1
b In right anglGFed tl GFD ,

[·: tan30° = )3]


h 0
=> h= ... (v)
cot f3 -cote (1) ran30° =- = ­x =
cular
DF
On equating the values ofh from Eqs. (iv) and (v), we get -;; h
---- 11 - ---- b - => = I
cot - cot 0 cotf3-cot 0
a
= J3
=> (corf3-cot 0) = b (cot -cot 0)
a a => x= J3h ... (i) [1 ]
=> (b - cot = b co c a. - cot f3
a) 0 a In right angled ll GEC,

cot 0 = -bcota. -acorf3


----..:..... tan60 °= CE => J3 = � [·: tan 60° J3J
EG
x
=
b -a £1 1
Hence proved . J3 = � [from Eq. (i) ]
27. Given A the .J3h (1 12]

circle. circle of radius cm and point P outside


4 a
=> 3h = h + 4
2h = 4 � h = 2 m
Steps ofconstruction
=>
of
:. Required height the = 2 + 4 balloon,
1 . Draw a circle of radius cm.
FG + 2 ll l
4
AG = AE
+ EF + =8 m
2.Draw two chords and MT. AR (1J
42 0 All{110n e MATH EMATICS C lass 1 0th Term 11

29. Let B(x,y) be the third vertex of equilateral MBC . J'J+ Yi 3 =

FG, � )
2 2

L
=> + Xi = 1 4
.\.·_\ . . . (iii)
and J'_, + Y1 = 6 ••• (iv)
Also, is che mid-poinc of AB. (1)
B (x, y) C (3, 0)
x1 + Xi = ?._
AB = BC = AC 2 2
=> AB2 = BC2 = AC2 Yi + Yi 7
--- = -
No� and
(1 /2)
AB2 = BC2 2 2
=> (x - 6) 2 + (y - 0) 2 = (x - 3) 2 + ( y - 0) 2 => X1 + Xi 7 = . . . (v)
and +
y1 Yi = 7 . .. (vi}
[ by distance formula, d = �(x1 - x ) 2 + (y1 - Y ) 2 ] On adding Eqs . (i), (iii) and (v), we get
2 i
=> x i - l 2x + 36 + y 2 = x i - 6x + 9 + y2 2(XI + Xi + X3 ) = 20
+
[·: (A - B)2 =A2 - 2AB + B2] => Xi X 2 + X3 = I O .. . (vii)
=> - l 2x + 36 = -6x + 9 => - l 2x + 6x = 9 - 36
On subtracting Eqs. (i), (iii) and (v) from Eq. (vii)
respectively, we get
=> -6x = - 27 => x =
27 = 2.
... (i) x, = 1 1, X2 = - 4, X3 = 3
6 2 [1 ] On adding Eqs. (ii), (iv) and (vi),wc get
and BC2 = AC2 2( y, + Yi + Y.i ) = 1 8
=> (x - 3) 2 + (y - 0) 2 = (6 - 3)2 + (0 -0)2 . => Y1 + Yi + YJ = 9 .. .(viii)
(11
=> x i - 6x + 9 + y 2 = 3 i On subtracting Eqs. (ii), (iv) and (vi) from Eq. (viii)

%
x-") + y 2 - 6x = O

(% )'
=> . . . (ii) l1 J respectively, we get
On putting the value of x from Eq. (i) in Eq. (ii), we get y1 = 4, Yi = 3 and y3 = 2
So, the vertices of MBC are A (I 1, 4 ), B( -4, 3) and
+ y2 - 6 x = 0 C(3 , 2 ).
:.Area of MBC
4 + y 2 - 27 = 0
81
=> = -1 [x1 ( Yz - Y3 ) + Xi (y3 - Y1 ) + X3 (y1 - J2)l
[1 /2) 2
=> Y = 27 - � => y2 =
i 1 08 - 81 = _!_ [1 1(3 - 2) + (-4)(2 - 4) + 3(4 - 3)]
4 4 2
=> y i = - = .!_ [1 1 x 1 + ( -4)(-2) + 3(1)]
27
4 2

(% �) ( }
=> Y = ± '14{27 = ±3 ./3 [on squaring both sides] = .!_ [1 1 + 8 + 3] = 22 = 1 1 sq units
2 2
2
Hence, the area of MBC is 1 1 sq units. [11
Hence, the third vertex is • 3 and 2. , -3 .[3 31 . (i) (a) Total number of shirrs
2 2 [1 )

( � · %}
= Total number of outcomes = 200

FG , f l
30. Let A = (x1 , y1 ), B = (x2 , Yi ) and C = (x3 , y3 ) Shirts acceptable to Jyoti
= Number outcomes favourable to Jyoti = 176
Given, n -

( �· % )
E (7, 3) and are the :.Probability that the shirt is acceptable to Jyoci
mid-points of the sides BC, CA and AB, respectively. = 176 = 0.88
200 (1)
Since, n - is rhe mid-point of BC. (b) Shi rrs acceptable to Riya = Number of outcomes
favourable to Riya = 1 76 + 16 = 1 92
Xz + X3 :. Probabili ty that shirt is acceptable to Riya
= _ .!_ and Y2 + J3 = 2_
2 2 2 2 = 1 92 = 0.96
[1 )
=> X2 + X3 = -1 . . . (i) 200
and Yi + Y3 5 = . . . (ii)
[1 )
(ii) Removal of barriers, women can take any profession
without any bias in a diverse society of our country.
As £(7, 3) is the m id-poi nt of CA . some people have tolerance for minor defects. (2)
SAM P L E QU ESTI O N PAPER 3
A Sa m p le Q u esti o n Pa p e r fo r C BS E C lass X S u m m ative Exa m ination II

MATH EMATI CS
.----- ---- - --- -- - - - - - -- - - -- ------- ------ --------.

Genera l I n stru cti o n s


1. All q u estio n s a re co m p u lso ry.
2. The q u estio n p a p e r consists of 3 1 q u estio ns d ivided into 4 Sections A, B, C and D.
Section A co m p rises of 4 q u estio n s of 1 mark each, Section B comprises of 6
q uestions of 2 m a rks each, Sectio n C co mprises of 1 0 q u estions of 3 marks each and
Sectio n D co m p rises of 1 1 q uestio n s of 4 marks each.
3. There is no ove ra ll c h o i ce a n d u se of ca lcu lator is not permitted.

Time : 3 hours Max. Marks : 90

Section A
1 . It is given that i n a group o f 3 students, the 3. If AOBC is a rectangle whose three vertices
probability of 2 students not having the are A (0, 3), 0 (O, 0) and B (5, O), then find the
same birthday is 0 . 9 9 2 . Find the p robability length of its diagonal.
of 2 stu dents having the s a m e birthday. 4. The diameter of a sphere is 6 cm . Ii is melted
2. Find the a re a o f a circle that can be inscribed and draw into wire ·of a diameter 4 mm. Find
in a square of sid e 6 cm. the length of wire .

Section B
5. Two cubes e a c h with 1 2 c m edge are joined 8. Prove that the points (a, b + c), (b, c + a) and
end-to -end . Find the s urfa c e area o f the ( c, a + b) are collinear.
resulting cuboi d .
9 . The circumferences o f two concentric circles
6. If Sn the s u m of the first n terms of an AP is forming a ring are 88 cm and 66 cm,
given by Sn = 5n 2 + 3 n , then fin d its nth t e r m . respectively. Find the width of the ring.


1 0. Solve the quadratic equation 2x2 + x -4 =0
7. Find the value o f 6 + �6 + �. by the method of completing square.

Section C
1 1 . The sum of the square of two consecutive o d d 1 6. C o nstruct a M B C i n w h ic h BC = 6 .5 cm,
numbers is 3 9 4 . Find the numbers . AB = 4 . 5 c m a n d LA B C = 60 °. onstruct I

3
1 2. Find the s um of all three-digit n umbers , which i'd e s are -

§
4
a triangle si m ilar to M B C, wh
leaves the remainder 3 when divided by 7 .
of the c orre s p o n d i n g sid e s o f BC.
1 3 . Suppose a die is dropped
1 7. Fin d 2 7 th term o f a n AP who th term is
1 m
J
2m
at random on the
- 1 0 a nd c o m m o n d i fference i
5
-

l 7
rectangular region as
shown in figure, what is
the probability that it will 3m 1 8. The length o f th e minute h n d of a clock
is 5 c m , fin d the a r e a swept by t h e mi nute
- -

land inside the circular region of diameter


1 m. hand d u ri n g ti m e p e riod 7 : 10 am to
7 : 45 am.
1 4. In the given figure, 1 9. Q uad rilat e ral ABCD in whi h LD = 9 0 °. A
find the perimeter of circle C' (0, r) i n scribed i n i t tou hes the
shaded region, where sides AB, B C, CD a n d DA at P, Q, R a n d S.
ADC, AEB and BFC respectively. I f BC = 3 8 c m , CD = 25 cm and
are semi-circles on BP = 27 cm, t h e n fin d the radius f t h e circle.
D R C
diameters AC, AB and B C, respectively.
1 5. A wooden articles was made by
scooping out a hemisphere s
from each end of a solid
cylinder as shown in the figure . A p B

20. Prove that the line segment JOmmg the


I f height of cylinder is 1 0 cm
a n d its base radius i s 3 . 5 cm,
t h e n find the total surface area
points of contact o f two parallel tangents
passes through the c en tre .
o f a rticle .

Section D
2 1 . A bucket made up of a metal sheet is in the 25. A motor boat whose speed is 1 8 km/h in still
form of a frustum of a cone of height 1 6 " cm water takes 1 h more to g o 2 4 km upstream ,
with r a d i i of its lower and upper ends as 8 cm than to return downstream to the same spot.
and 2 0 cm, respectively. Find the cost of Find the speed of the stream .
the bucket, if the cost of metal sheet used is
� 1 5 per 1 0 0 cm 2 . [take, 7t = 3 .1 4]
26. Water flowing o u t thro u gh a circular pipe
whose internal diameter is 2 cm at the rate of
0 . 7 mis into a cylindrical tank, the radius of
22. What least number of terms of the sequence whose base is 4 0 c m, by h ow m uch will the
17, 15
4 3
5
- , 14 - , . . . should be taken, so that the level of water rise in an hour?
coordinates of the vertices of a
5
sum is negative? 2 7 . The
rectangular ground ABCD are A (0, O),
2 3 . Prove that the tangent at any point of a circle B (20, O), C (20, 1 0) and D (O, 1 0).
is perpend icular to the radius through the
{i) Five trees are to be planted at the
poin t of conta ct . mid-points of the sides and at the centre
24. D raw a pair o f tangent to a circle of radius of the ground . Decide the positions of the
5 cm wh i c h a re incline d to each other at an trees .
an gle of 6 0 °, give s t ep s of construction . (ii) What value is depicted from the above?
Sample Question Pa per 3 4 23

28. A pe rs on s t a n d i n g o n the ba n k o f a rive r card is d rawn a t random fro m a

.
30. A
observes t h a t t h e a n 9 l c o f e l eva tion o f t h e top well-shuffled deck of pl a y i n g card . Find the
of a buil d i n g of <J n o rg d n i s a t i o n wo rki n g for

.
probability that the card drawn is
conservat i o n of wi l d l i fe , sta n d i n g on the (i) a spade or an a c e
opposite ba n k is 6 0 °. W h e n he moves 40 m
(ii) a r e d kin g .
away from t h e b a n k , h e fi n d t h e a n gle o f
(iii) ne ith e r a king n o r a q ue e n
(iv) ei th e r a kin g or a que n .
elevation to be 3 0 °.
(i) Find t he h e i g h t of a b u i l d i ng a nd width o f
t h e ri v e r. 3 1 . I f t h e a n g l e o f elevation o f a cloud from a
c o n s e rve w i l d l i fe ? point h m above a lake is u a n d the a n gl e o f

(- - �9_)
(ii) Why do we need lo
depress io n o f i t s reflection i n t h e lake is p.
29. If P and Q

-1-
a rc t w o p o i n t s w h o s e coord inates Prove that the h eight of the clou d is
are (at 2 , 2 at) and C!_ ,
h (tan p + tan a)
t
res pectively,
t2 (tan f3 - tan a)

S is a p oi n t (a, 0).
!__
S h ow that _ + is
SP SQ
independe n t o f I .

,
Answe rs
1 . 0.008 2 . 9 7t cm
-
3. J34 units 4. 900 cm 5. 1 440 c m 2

-
1 ± J33
6. 7. j 1 1.
-

an = l On - 2 9. 712 cm 1 0. or
4
1 3, 1 5 - 1 5, - 1 3

1 2. 7082 1
11 20
1 3. 1 4. 1 3. 2 cm 1 5. 374 cm 2 1 7.
84 7
18. 45.83 cm 2 1 9. 1 4 c m 21. � 294 22. 30th term 25. 6 km/h

26. 1 57.S cm
27. (z) (1 0, O), ( 20, 5 ) , ( 1 0, 1 0 ) , (0, 5 ) , ( I O, S ) (iz) Removing air pollution, preservatio n o f env i ronmen r.

28. (z) 20 m and 34.64 m

- (iz) Wildlife is a pare of our enviro n ment and conservation of each of its element is important for ecological balance.

30. (z) _!_ (i it) .!..]_ ( iv) 2


1
(iz) -
13 26 13 13
SAM PLE QU ESTI O PA P E R 4
A Sa m ple Qu estio n Pa per fo r CBSE Class X Su m mative Exa m i n ati o n II

AT CS
Genera l I nstru ctions
1. All q u estions are com p u lsory.
2. Th e q uestio n paper consists of 31 questions d ivided into 4 Sections A, B , C a n d D.
Sectio n A com p rises of 4 q u estions of 1 mark each, Section B com p rises of 6
q u estio ns of 2 marks each, Section C comprises of 1 0 questions of 3 m a rks each a n d
Sectio n D com p rises of 1 1 q uestions of 4 marks each.
3. The re is n o overall choice and use of calcu lator is not permitted .

Time : 3 hours M ax. M a rks : 90

Section A Section B
1. A bag contains 5 red balls and some blue 5. Find the nature of t h e roots of t h e equation
x + l Ox + 3 9
2 = 0.
balls . I f the probability of drawing a blue ball
6. S how that the sequence a 11 =
2
from the bag is four times that of a red ball, 2n + 1 is not
then fin d the n umber of blue balls in the bag. an AP.

2. Fin d t h e a r e a o f largest triangle that can be 7. Construct a line segment AB of length 7 cm,
inscribed in a semi-circle of radius r units . using ruler and comp a s s . Fin d a point P on

3. A B is the diameter of a circle whose centre


AB such that P divides A B into 3 : 2.
is 0 (2, - 3) and coordinates of point B is (1, 4). 8. A cone o f height 20 cm a n d radius o f base
Fin d the c o o rdinates of point A. 5 cm is made up of clay. A child reshapes it

4.
in the form of a sphere . Fin d the diameter of
In th'e given figure , AB, A C and AD are
the sphere .
tangents . If the length of AB = 7 cm, then
find the length of AD. 9. In an AP, if d = - 2, n = 5 and an = 0, then find
the value of a .

1 O.

<Cl)
Find the area o f the shaded
region ·as shown in the given
figure , if the diameter of a
circle with centre O is 2 8 cm
1
A and AP = - AB.
4 B

Sectio n C Section D
1 1. If the equation 21 . n and
I f in a n A P, t h e s u m o f its m te r m s i s e q u a l to
(1 + m2 ) x 2 + 2 m cx + ( c
2 - a2 ) = 0 has sum o f its n term i s e q u a l to m , t h e n p rove that t h e
equal roots, then prove t h a t s u m o f i t s ( m + n) t h term is e q u a l to - ( m + n).
c2 = a2 ( 1 + m2 ) .

22.
1 2. In an AP, sum of firs t ter m s , n Wh i l e b o a rd i n g an a e ro p l a n e , a p a s s e n g e r got

i.e. Sn = 3n2 Sn. h u rt . The pilot s h ow i n g p ro m p t n e s s and c o n c e r n ,

(:} ( :}
+ Find i t s 2 5 th term .
m a d e a r ra n g e m e n t to h o s p ita l i s e the inj u re d a n d
1 3. Solve the quad rat ic e q u a t i o n s o t h e p l a ne s ta rted l a t e b y 30
m i n To rea c h t h e
.

0
b
d e s t i n a tion 1 5 0 0 k m a w a y i n t i m e , the p i l o t
x2 + 1= 0 + by u s i n g i n c re a s e d t h e s p e e d by 1 00 km/h . Find t h e o rigi nal
b s p e e d o f t he pla n e .
23.
the factorisation method .
Prove tha t i n t e rc e p t s o f a ta n g e n t b e tween two
1 4. Construct a c i rcle whose rad i u s is p a ra l l e l ta n ge n t s to a c i r cl e s u b t e n d s a rig h t a n g le
equal t o 4 cm . P is a p o i n t whose at the c e n t re o f a c i rcl e .
distance fro m its cen tre is 6 cm .
24. Prove th a t t h e l e n g t h o f t h e t a n g e nts d rawn fro m
Construct tangents fro m point P.
a n exte r n a l p o i n t to a ci rcl e a re e q u a l . U s i n g
1 5 . An isosceles MBC i n which AB = AC res u l t , f i n d t h e d i s ta nce o f exteri o r p o i n t from the
c e n t re , fro m which t a n g e n ts a re d rawn , i f l e n g t h o f
is circumscribed abo u t a c i rcle . S how
t a n g e n t i s 8 c m a n d ra d i u s o f circle is 6 c m .
that BC is bisected at a point of
contact. 25. An a e ro pl a n e fly i n g h o ri z o n ta l l y 1 . 2 km above t h e
1 6. Find the coordinates of the
gro u n d is o b s e rved a t a n elevation a n g l e o f 60°.
After 1 5 s e c , i t s elevation a n gle is observed t o b e
circumcentre of the tri a n gle
6) , (8, - 2)
whose
30°. F i n d t h e s p e e d o f t h e a e roplane i n km/h .
[ ta ke , J3 = 1 . 73)
vertices are (8, and (2 , - 2).
Also, find its circumradius .
26.
17 A vessel is in the form o f a h ollow
Fo r a S ci e n c e E xh ibitio n , A
·

Ra hul pres e n t e d a d i a g ra m m a tic


hemisphere mounted by a h o llow
repre s e ntati o n of ra i n water
cylinder, the radius o f h emisphere is
h a rvesting a s a p roj ec t . A B a n d AC,
7 cm and total h e ig h t o f ves s e l is
20 cm. Find the inner surfa c e a re a of
t h e pipes o f 5 m l o n g a re bringing
water fro m the terra c e of a bu i l d i ng
(as s hown in the fig u re ) . The
the vessel.

1 8. Find the area of a triangle, whose t riangular s p a c e is d eveloped as a


vertices are (-8, 4 ), (-6, 6) a n d (-3 , 9 ). gard e n .

1 9. 1\vo dice are thrown at the same time


( i ) What is the perimeter of the triangular gard e n ?
and the product of numbers appeari n g ( ii ) If the radius of circle is 1 2 cm, t h e n fin d the
on the m i s noted . F i n d the probability leng th of OA .
that the product is less than 9 . (iii) What qualities do you think is encouraged by
20. Ravi and Rani are busy making
such exhibitions ?

jaggery out of sugarcane j ui c e . They 27 . MBC is an equilateral A


have processed the sugarcane j uice to triangle of sides 1 4 c m . D, E
making molasses which is poured into and F are mid -points of sides
mould in the shape of a frustum of a AB, BC and AC, respectively.

[ J
cone having the diameters of its two Find the area of s haded part in
circular fall 30 cm and 35 cm and the given figure .
vertic al height of the m ould 1 4 c m . 22
Find the volum e of molasses . take , it = and .J3 = 1 . 7 3
7
Section C
1 1 . The sum of the square of two consecutive odd 1 6. Construct a MBC i n which B C = 6.5 cm,


numbers is 394 . Find the numbers . AB = 4 . 5 cm and LAB C = 60 °. Also, construct


1 2 . Find the sum of all three-digit numbers , which a triangle similar to M B C, whose sides are
leaves the remainder 3 when divided by 7 . 4
of the corresponding s i d es of MBC.
1 3. S uppose a die is dropped
at random on the J 1 7. Find 27th term of an AP whose 9th term is
rectangu 1 ar
.

�.
m 2m

1
reg10n as - 1 0 and common d i fference is
shown in figure, what is 7
the probability that it will 3m- -­ 1 8. The length of the mi nute hand of a clock
land inside the circular region of diameter is 5 cm, find the a rea swept by the minute
1 m. hand during time p e riod 7 : 1 0 am to
7 : 45 am.
1 4. In t h e given figure,
1 9. Quadrilateral ABCD in which LD = 90 °. A
find the perimeter of circle C' (0, r) inscribed in it touches the
shaded region , where sides AB, BC, CD and DA at P, Q, R and S.
ADC, ABB and BFC respectively. I f BC = 3 8 cm, CD = 25 cm and
are semi-circles on BP = 27 cm, then find the rad ius of the circle.
diameters AC, AB and B C, respectively. D R C

1 5 . A wooden articles was made by


sco_oping out a hemisphere s
from each end of a solid
cylinder as shown in the figure . A B
p
If height of cylinder is 1 0 cm
2 0. Prove that the line segment joining the
and its base radius is 3 .5 cm,
points of contact of two parallel tangents
the n find the total surface area
passes through the centre .
of a rticle .

Sectio n D
2 1 . A bucket made up of a metal sheet is in the 25. A motor boat whose speed is 1 8 km/h in still
form of a frustum of a cone of height 1 6 cm water takes 1 h more to go 2 4 km upstream,
with radii of its lower and upper ends as 8 cm than to return downstream to the same spot.
and 2 0 cm , respectively. Find the cost of Find the speed of the stream .
the bucket, if the cost of metal sheet used is
2 6. Water flowing out through a circular pipe
� 1 5 per 1 00 cm 2 • [take , 7t = 3 .1 4]
whose internal diameter is 2 cm at the rate of
0. 7 mis into a cylindrical tank, the radius of
2 2 . What least number of terms of the sequence
whose base is· 40 cm, by how much will the
1 7 , 1 5 � , 1 4 � , . . . should be taken, so that the level of water rise in an hour?
5 5
sum is negative? 27. The coordinates of the vertices of a
rectangular ground ABCD are A (O, 0),
2 3 . Prove that the tangent at any point of a circle B (20, 0), C (20, 10) and D (O, 1 0).
is p erpen dicu lar to the radius through the
(i) Five trees are to be planted at the
point of con tact.
mid-points of the sides and at the centre
24. Draw a pair of ta ngent to a circle of radius of the ground . Decide the positions of the
5 cm which are in clined to each other at an trees.
a ngle of 6 0 °, give st eps of construction . (ii) What value is depicted from the above?
SAM PLE QU ESTI O N PAP E R 5
A Sa m p le Q u esti o n Pa p e r fo r C BS E Class X S u m mative Exa m i n ation II

MATH EMATI CS
.----- - - - - - - - - - -- ---- - - - - - - -- - - - - ---

G e n e ra l I nstru cti o n s
1. All q u estio n s a re com p u lso ry.
2. The q uestio n paper con sists of 31 q u estions d ivided i nto 4 Sections A, B, C and D.
Sectio n A co m p rises of 4 q uestions of 1 mark each, Section B comprises of 6
q u esti o n s of 2 m a rks each, Sectio n C com p rises of 1 0 q uestions of 3 marks each and
Sectio n D · co m p rises of 11 q uestions of 4 marks each.
3. The re is no overa ll c ho ice a n d u se of calcu lator is not perm itted .

..._
_ _ ____ - - - - - -- - - - - - - - ----- ---

Time : 3 h ours Max. Marks : 90

Section A Section B
1 . I n the giv e n figu re , 0 is the c e n tre of a circle. 5. In an AP, the sum of first n terms is n2 + 2n.
PA and PB is the pair o f tangents d rawn to the Find its 1 8th term.
circle fro m point P o u tside the circle . I f 6 . In the adjoining figure,
LA O B = 1 1 7 °, the n fin d t h e v a l u e o f 8.
p
PA and PB is a pair of
A tangents drawn to a
circle havi ng its centre
at 0. If LAPB = 5 2 °, the;n B
p
fin d L.PAB and LPBA.

7 . Find the curved surface area of a right circular


B cone whose radius is 5 cm and vertical height
2 . Fin d the area o f a c i rcle who s e radius is 1 0 c m . · is 1 5 c m .
[take , 7t = 3 . 1 4] 8. Find the perimeter of the given A D
3. I f P(E) = 0 . 0 6 , then what is the p robability of

tj'
figure, where AED is a
' not E'? semi-circle and AB CD is a
rectangle.
4. Find the roots o f the equation 2 x 2 - 6 x = O.
8 •- 1 4 cm -· c
11

428 l Allin.One MATHEMATICS C l a s s 1 0th Term 1 1

9 . I f t h e radius o f a circle is 9 . 8 c m , then find the 20. Cards marked with numbers 3 , 4 , 5 , . . . , 50 a re.

area of a quad ra n t of the circle. placed in a box and mixed thoroughly. One
1 O . The radii o f two circles are 8 cm and 6 cm, card is drawn at random from th box. Find the
respectively. probability that n u mber on the d r w n c a rd is
(i) divisible by 7 .
Find the radius of the circle having area equal
to the s u m of the areas of the two circles . (ii) a number which i s a perfect sq u a r

Section C Sectio n D
2 1 . A train travels at a certain avera s p e d for a
1 1 . S olve the quadratic equation distance of 63 km and then travel a d i s ta nce of
cru2 + 2 abcd u - ( 1 + 2c)(b2 d2 ) = O 72 km at an average speed o f 6 km/h more t h a n
its original speed . I f it ta kes 3 h t c o m plete the
by completing perfect square . total journey, then find its o ri g i n a l a verage
Prove that the line joining the centre 0 o f a
speed.
1 2.
circle to an exterior point P bisects the angle 22. A room is 8 m long, 4.5 m wid e
between the tangents drawn to the circle from and 2 . 8 m high . It has o n e door
the point P. which is 1 . 4 m wide and 2 m high
with a semi-circular ventilator
1 3. A student of Class X saves � 4 on day 1 , � 7 on as shown in figure and two
day 2 , � 1 0 on day 3 , . . . and so on. How much rectangular windows 2 m by 1 . 5 m

[ ]
amount will the student save in the month of each. Find the cost of
February 2 0 1 1 ? white-washing the inner four - 1 . 4 m -
walls of the room at the rate
1 4 . Find the s u m of all the m ultiples of 9 lying 1t
of � 10 per sq m. take, =
22
between 300 and 7 00 . 7
15 · A circle is inscribed in a MBC having sides 23. In the adjoining figure, A

/GA
8 c m , 10 cm and 1 2 cm as shown in figure. ABC is a right triangle,
Fin d AD , BE and CF. right angled at A . Find

zCT;:
c the area of shaded
AB = 6 cm, C
B
region, if
BC = 10 cm and 0 is the centre of the incircle of
MBC. [take, 7t = 3 . 1 4 )
A D
B
24. The tangent a t a point C o f a circle and a
1 2 cm diameter AB, when extended intersect at P. If
1 6. The speed of a boat in still water is 1 1 km/h. LPCA = 70 °, then find LCBA.
It can go 12 km upstream and return 25 . At a point on level ground, the angle of
d ownstream to the original point in elevation of a vertical tower is found to be such
2 h 45 min . Fin d the speed of the ·Stream.
�.
On walking 1 92 m

]
that its tangent is
1 7 . A toy is in the form of a cone mounted on a

[
12
hemisphere of diameter 7 cm. The total towards the tower, the tangent o f the angle of
height of the toy is 1 4 . 5 cm. Find the total elevation is � . Find the height of the tower.
1t 4
volume of the toy. take, =
22
7 26. Deepanshi donates money to a trust, working
1 8 . Find a point on X-axis which is equidistant for education of children and women in some
from the points (7 , 6) and (-3 , 4 ) . villages. If she donates � 5000 in the first year
and his donation increased by � 2 5 0 every year.
1 9. AB C i s an isosceles-triangle such that (i) Find amount donated by her in 8th yr and
A B == A C . D is the mid-point of AC. A circle is total money donated in 8 yr.
drawn taki n g B D as diameter which interest
1
(ii) Which mathematical concept is being used
AB at the poi nt E. Prove that AE = 4' AC. here?
{iii ) Write any two values Deepanshi possess.
Sample Question Pa p e r 5 1 429 1
27. I f A (-2 , -1 ) , B ( a , O ) , C ( 4 , b ) a n d D ( 1 , 2 ) a re t h e (i) Find the position of red and yellow flowers.
vertices o f a p a ra l l e l o g ra m , t h e n fi n d t h e (ii) Find the distance between these flowers .
valu e s o f a a n d b. (iii) Who makes the flower closer to the origin?
28. I n sports d a y a c t i v i t i e s o f M ee r u t P u b l i c (iv) Write the values associated with sports
Scho o l , t h e l i n e s were d rawn wi th c h a l k activities in day- to-day life .
p owd e r i n recta n g u l a r s h a pe O B CD . E a c h 29. A b oy standing on a horizontal plane finds a
lin e is a t 1 I 2 m d i s ta n ce fro m e a c h o t h e r. bird , flyin g a t a distance of 1 00 m from him at
60 flower p o t s h a v e been p l a ce d a t a dista n c e a n elevation of 3 0° . A girl standing on the roof
of 1 I 2 m fro m e a c h o t h e r a l o n g OD . Ya m i n i of 20 m high building , find the angle of
r u n s 1 I 4 th o f the d i s ta n c e OD o n the 3 rd l i n e elevation o f the s a m e bird to be 45°, if both the
and m a ke s a r e d fl ower. K a m l a r u n s 1 I 5 th o f b oy a n d the girl a re o n opposite sides of the
t h e distance OD o n t h e 7 th l i n e a n d m a ke s a bird . Find the distance of the bird from the girl.
yellow flower.
y 30. O n e c a rd i s d rawn from a well-shuffled deck of
5 2 card s . Find the p robability of drawing
(i) an ace (ii) 2 of spades
(iii) 1 0 of a black suit.
Flower
pots 31 . Co nstruct a circle of radius 3 . 5 cm . From a
point P o u tside the circle at a distance of 6 cm

0 1 2 3 4 5 6 7 8 9
from t h e c e ntre of the circle, draw two
101 1 B X
tangents to t h e circle .

A n swe rs
1 . 63° 2. 3 1 4 cm 2 3. 0.94 4. x = 0 o r 3 5. 37
6. LPAB = LPBA = 64 ° 7. 248.46 c m 2 8. 76 cm 9. 75.46 cm 2
bd hd 1 4. 2 1 978
1 0. 1 0 cm 1 1. - , -- ( I + 2c ) 1 3. � 1 246
a a

1 5. AD = 7 cm, BE = 5 c m , CF = 3 cm 1 6. 5 km/h 1 7. 23 l cm 3 1 8. (3,0)

20. (t) !_ (it) .!_ 2 1 . 4 2 km/h 22. � 604.30 23. 1 1 .44 cm


2 24. 70°
48 8
25. 1 80 m
26. (t) � 6750, � 47000 (it) AP (iit) Social awareness and responsible citizen.
27. a = l , b = 3
28. (t) A (l.5, 1 5 ) H (3.5, 1 2 ) {it) .Jlj un its
(iit) Kamla makes flower closer co the o rigin .
(iv) Values associated with spores activities i n day-co-day l i fe are, they
give b reak from regular life.
(a)
i ncrease physical strength o f i ndividual.
(b)

26
relax m i n d and body.
(c)
(d)
help in increas i ng concentratio n in o ther works.

29. 30.fi. m
1
30. (t) 1 / 1 3 (ii) 1 15 2 (iii) -
SAM P L E QU ESTION PAP E R 6
A Sam p le Qu estion Pa per for CBSE Class X Su m mative Exa m i nati o n II

AT C
General I nstructio ns
1. All questions a re com pulsory.
2. The ·q u estio n paper consists of 31 questions divided into 4 Sections A, B, C a n d D.
Section A com prises of 4 questions of 1 mark each, Section B comprises of 6
q u estions of 2 marks each, Section C comprises of 10 questions of 3 marks each a n d
Sectio n D com p rises of 1 1 questions of 4 marks each.
3. There is no overall choice a n d use of calcu lator is not permitted.

Time : 3 hours Max. Marhs : 90

Section A 7 · . Find the roots of the quadratic equation


1. Fin d the perpendicular distance of the point 2 1x 2 - 2x + _!__ = 0 by factorisation method.
A(5, 3) from Y-axis . 21

2. 8 . A card is drawn a t random from a


Fin d the value o f x for which (Bx + 4), (6x - 2)
well-shuffled pack of 5 2 cards. Find the
and (2x + 7) are in a n· AP.
probability of gettin g
3. Fin d t h e area of a square inscribed i n a circle (i) a red king. (ii) a queen o r jack.
of radius 8 cm.
9. From a rectangular sheet of paper ABCD
4. From a p oint Q, 1 3 cm away from the centre with AB = 40 cm and AD = 2 8 cm, a

[ J
of a circle, the length of tangent PQ to the semi-circular portion with B C as diameter is
circle is 1 2 cm, find the radius of circle. cut- off. Find the area of remaining paper.
7t 22
• =

Sectio n B
take,
7
1 0. Prove that tangents drawn at the ends of a
5. Two dice a re thrown at the same time. diameter of a circle are parallel.
Determine the . p robability that the

Section
difference of the n umbers on the two dice
is 2 . C
6 .. A solid sphere o f radius 1 0 . 5 c m is melted 1 1 . How many three -digit numbers are divisible
and recast into s maller solid cones, ea ch of by 1 2 ?

[ 2:]
radius 3 .5 cm and height 3 cm . Find the
1 2. Solve the quadratic e quation for x
n u mber of co nes so form ed.
x2 - 4 ax - b 2 + 4 a2 = 0.
take, rr =
Sample Questi o n Paper 6 1 431 1
1 3. A hemispherical bowl of internal radius 23. P P and 00' are two direc t common tangents
9 cm is full of water. Its content are emptied to two circles intersecting at points A and B .
in a cylindrical vessel of internal radius of The common chord on produced intersects
6 cm. Find the length of water in the PP on R and 00' in S. Prove that
cylindrical v ess e l . (i) RA = SB.
1 4. In the adj o i n i n g (ii) RS 2 = P' p2 + AW .
figure , A B is d i amete r
24. Madhu Sadan house has an overhead tank
of a circle with centre A i-::::;....i.��--q..l.--�
in the shape of cylinder. This is fill ed by the
0 a n d Q C is a tangent
pu m p i n g water coming for a sump
to the circle at C. If
(as underground tank) , which is in the
LCAB = 3 0 °, then fin d
shape of a cuboid . The sump has dimensions
(1) L CQA (ii) L CBA as 2 m x 1 . 5 m x 1 m. The overhead tank
1 5. Find the coordinates of po i n t P, which lies o n h a s radius 30 cm and height 40 cm .
the line segment j oi n i n g t he po int s A ( 2 - 2) - , [ take, n = 3 .1 4).
a n d B (2 , - 4) such that AP =
3
- AB. (i) Find the height of water left in the sump
7 a fter the overhead tank has been
1 6. If - 4 is a root of the quadratic equation
completely filled with water from the
2
+ kx - 4 = 0 and the quadratic equation
sump .
x
2
+ px + k = O has equal roots , then find the
(ii) Compare the capacity of the tank with that
x
values of p a n d k .
of the sump .
(iii) Which mathematical concept is used to
1 7. A chord P O of a circle i s parallel to the solve the question?
tangent drawn at a point R of the circle . {iv) Which values are associated with saving
Prove that R bisects the arc PRQ. the water?
25. The a ngles of elevation and dep re ss ion of
A
1 8. In the adj oining figure , 0 is the top and bottom of a light house from the
the centre of circle with top o f a 60 m high building are 30° and 60 °
AC = 24 cm, AB = 7 cm and respectively, find
LB OD = 9 0 ° . Find the area of (i) the difference between the heights of the
the shaded region . light house and the building.
[take, 7t = 3 . 1 4] (ii) the distance be tween the light house and
1 9. Find the value of p, if the points A (1 , 2), the building.

B (-3, p) and C (S , - 4) are collinear. 2 6. A s p h e ri ca l balloo n of radius R subtends an


20. a n gl e 0 at the eye of an observer. If the angle
Split 2 0 7 into three parts such that these are
of elevati on of its centre is <!>, then prove that
in an AP and the product of the two smaller
the height of the centre of the balloon is
parts is 4 62 3 .
given by R sin cf>cosec �.

Section D
2

2 7. In the given figure, ABC is a right angled


triangle, right angled at A, semi-circles are
2 1 . An AP consist of 3 7 terms . The sum of th e
drawn on AB, AC and BC as diameter. Find
three middle most terms is 2 2 5 a n d t h e sum
of the last three is 4 2 9 . Find AP.
the area of shaded region .

22. Construct a triangle with sides 5 cm, 6 cm


and 7 cm, then construct another triangle
whose sides are � times the corresponding
.
3
sides of the first triangle .
\ 43 2 \ Allln one MATHEMATICS Class 1 0th Term 11

28. If Sn denotes the sum of first n terms of an AP, of each of the conical d epressions is 0 .5 cm
2 1 c m . The edge of the
Si2 = 3 ( Sa - S4 ).
and the depth is
3 c m . Find the volu m e
then prove that
cubical depressio n is
29 . If the coordinates of mid-point of the sides of of the wood in the e n tire s ta n d .
a triangle are (l , 1), (2, - 3 ) and (3 ,
'
4).
3 1 . Miss Deepanshi earns � 30000 in a month.
Then, find
its centroid .
She spends � 25000 on h e r needs .
30. A pen stand made of wood is in the shape of
(i) What is the probability o f her savings?
(ii) Which mathematical concept will be us e d
a cuboid with four conical depressions and a
cubical depression to hold the pens and
to solve the above problem?
pin s , respectively. The dimensions of the
cuboid are 1 0 cm, 5 cm and 4 cm . The radius (iii) What value is depicted by Deepanshi from
it?

Answe rs
3.
1 . 5 units 2.
15
x=- 128 cm 2 4. S cm 5.
2
2 9
1 1 1
(i) - (ii) _3_

)
6. 1 26 8. 9. 812 cm 2 1 1. 75
7. 21' 2i

P(-3_
26 13
1 2. x = 2a ± b 1 3. 13.S cm 1 4. (i) LCQA =30° (ii) L CBA = 60°
1 5.
-20 1 6. p = ±2.J3, k = 3 1 8. 226.65 cm 2 1 9. p =8 20. 67, 69, 7 1
7' 7
21. 3, 7, 1 1, 1 5 , ...
24. (i) 0.9 6 m (ii) 0.038

( 2, �)
(i!t) Volume of solid figure.
(iv) Values associated with saving the water are environment concern, social issue concern, etc.

25. ( z) 20 m {it) 20./3 m 27. 6 sq cm 29.

30. 170.8 cm3


31 .
1
(t) - (ii) Probability (iit) Habit of savings
6
SAM P L E QU ESTI O
.
PAP E R 7
A Sa m p le Qu esti o n Pa p e r fo r C BS E C lass X S u m m a ti ve Exa m i n ati o n II

E AT - - - - - - - - - -- - -- --
s
----- ------ - - - - - -

Gene ral I nstructi o n s


1. All q u estio ns a re co m p u lsory.
2. The q u estio n pa per con sists of 31 q uestio ns d ivided i nto 4 Secti o n s A, B, C a n d D.
Section A com p rises of 4 q u esti o n s of 1 m a rk each, Sectio n B com p rises of 6
questions of 2 m a rks each, Sectio n C co m p rises of 1 0 q u esti o n s of 3 m a rks each a n d
Sectio n D com p rises o f 1 1 q u esti o n s o f 4 ma rks each.
3. There is no overa ll c h oice and u se of calcu lato r is n ot p e rm itted .

Time : 3 hours Max. Marks : 90

Section A 6. The roots a and 13 of the quadratic equatior:


1 . A largest sphere is inscribed in a cube of side
x2 -5 x + 3 (k - 1) = 0, are such that ex - 13 = 11
7 cm . Find the volume of the sphere . Find the value o f k.
2 . In the given figure , find the length of B C. 7. Three unbiased coins are tossed . What is the
probability of getting
(i) atleast two head s ?
(ii) o n e head o r two heads?
8. In
adjoining
figure, POL
p
and PRM are
3. If the area of a circle is 3 0 1 . 84 cm 2 , then find tangents to
its circumference . the circle M
4. A letter is chosen at random from the word with centre 0 at the points Q and R,
'PROBABI LITY. ' Find the probability that it respectively. If S is a point on the circle, such
is a vowel . that L S QL = 50° and LSRM = 6 0° , then find
the value of LQSR.

Section B 9. A toy is in the form o f a cone mounted on a

[ J
hemisphere of radius 3 . 5 cm . The total
height of the toy is 1 5 . 5 cm . Find the total
5. A bag contains 8 red balls and some yellow 7t 272
surface area of the toy. take , =
balls . If the probability of drawing a yellow
ball is double that of a red ball, then find the
1 0. In the given figure , the shape of a solid
number of yellow balls in the bag.
copper piece is shown (made o f two pi eces) .
:434 ; Allin.one MATH EMATICS C lass 1 0th Te rm 11

The face ABCDEFA is u niform 1 8. In th g iv n


cross-section. Assume that the a n g le s A, B, fig u r , tw
C, D, E and F are right angles. Calculate ircl s with
the volume of the piece. ce ntr s A and B
tou h ea h o t h r
externally.
PM = 15 m is t a n g nt t o ir l with centre A
a n d ON = 1 3 m is t a n g n t to c i r l
' th ntre
B from xtern a l p i n ts P n d Q . I f PA = 1 7 cm
nd BO = 1 2 m th n fi n d t h d i ta nce
between th entr A nd B f ir l .

1 9 . PO is a h o rd of l n g t h 8 c m o f a c i rcl of
E ra d i u s 5 m . Th ta n g n ts a t P a n d Q i n t e rsect
a t T . Fin d the l n g t h o f TP.

Section C 20 . The n u merator of a fra t i o n i s


denominator. If thr is add d t
n less than its
a h of the
num rator and d e n o m i n t o r, t h n the frac tion
1 1 . What is the probability of having is i n c reased by 3 /2 8 . F i n d th fra t i o n .
53 Sunday in a non-leap year?
1 2 . Determine the sum of the first 20 terms o f
an AP whose 3 rd term is 10 and when 6th Section D
21 . An express train takes 1 h less than a
term is subtracted from 9th term, we get
passenger train to travel 1 3 2 km between
24.
1 3. PORS is a diameter of a circle of radius Mysore and Bengaluru (without taking into
6 cm. The length PO, QR and RS are equal. consideration the time they stop at
S emi-circles a re d rawn o n PO and OS as intermediate station) . If the average speed of
d ia meters . Find the perimeter of the the express train is 1 1 km/h more than that of
shaded region . the passenger train, then find the average
speed of the two trains .
22. Construct a MBC with sides BC = 6 cm,
A B = 5 cm and LABC = 60°. Then, construct a

triangle whose sides are � times, the


4
corresponding sides of MBC.
1 4. In a circle of radius 1 0 cm, an arc subtends
an angle of 90° at the centre. Find the area 23. A vertical tower stands on a horizontal plane
of major sector. and is surmounted by a vertical flagstaff.
At a point on the plane 7 0 m away from the
1 5. S olve the equation
tower, an observer notices that the angles of
9x 2 - 9(a + b)x + 2a2 + 5ab + 2b2 = 0. elevation of the top and the bottom of the
1 6. Find the value of p for which the points flagstaff are respectively 60° and 45°. Find the
A (- 1 , 3) , B (2 , p) and C(5, - 1) are collinear.
height of the flagstaff and that of the tower.
Also, find the ratio in which B divides AC. 24. A canal is 300 cm wide and 120 cm deep . The
water in the canal is flowing with a speed of
1 7. Find the area of the shaded region .
20 km/h. How much area will it irrigate in
[take, n = 3. 1 4)
20 min, if 8 cm of standing water is desired?
25. A MBC is drawn to circumscribe a circle of
radius 4 cm such that the segme n ts BD and DC
in which BC is divided by the point of contact
D are of length 8 cm and 6 cm, respectively.
Find the sides AB and AC.
Sample Questio n Paper 7 j 435 f
26. A solid right c ircula r 60 cm 28.

Blfc
In the given figure, AD is
cone of height 1 2 0 cm a median of the MB C
and ra dius 60 cm is and E is a point on AD
AE = _! J26
placed in a right
such that
circular cylinder fu l l 4
of water of height units . Find the (2, 5) D (4, 1 )

{Right circular cone)


1 80 cm, s u c h that it coordinates of the

.
touches the botto m . point E.
Find the volume o f water left i n the cyli nder,
2 9. Find the point on Y-axis which is
if the rad i u s of the cyli nder is equal to the
equidistant from the points (5, - 2) and ( -3, 2)
radius to the c on e .
60 cm 3 0 . The sum of ages of two friends Mohan and
Sohan is 30 yr, wh ere Mohan's age is less
than S ohan's a g e . Three years ago, the
product of their ages was 63 yr.
(i) Determine the present age of Mohan.
(ii) Which mathematical concept is used to
Water solve the above question?
(iii) Is it possible that anyone has age more
than 30 yr?
{Right circular cyclinder)
(Cylinder containing cone) (iv) Discuss the values associated with true
friendship?
2 7. A contra c tor employed 1 50 workers to finish
3 1 . The angle of elevation of a jet fighter plane
a piece of work in a c e rtain fixed n umber of
from a point A on the ground is 60°. After a
days . On the firs t d ay, all 1 5 0 workers
flight of 1 5 s , the angle of elevation changes
30°. If the jet is flying at a speed of
worked . He d ropped four workers on the
to
720 km/h , then find the constant hei ght at
second d ay, four more workers were dropped
on the third d ay a n d so o n . I n this way, eight
extra d ays were required to finish the work.
which the jet is flying. [take, .J3
= 1 . 732]
Find the number of d ays in which the work
was to be completed originally.

Answers
1 . 1 79.67 cu cm 2. 10 cm 3. 6 1 .6 cm 4. i 5. 16
11
6. k = -7 7 . (t) 1 /2 (ii) 314 8. 1 1 0° 9. 2 1 4.5 cm 2 1 0. 880 cm3
2a + b a + 2b
11. 1
-
· 1 2. 1 3. cm 1 4. 235.7 1 cm 2 1 5. -- --

2 1 . Average speed of passenger tra1. fl 33


1 400 6n 3
,

3
7
1 6. p 20.
20 3
= 1, 1 : 1 72. 6 7 cm 2 1 7.AB = 13 cm - cm
3 1 8. 1 9.
4

(- �4 ' 2_4) (0, -2)


= km I
h and average speed ofexpress train = 44 km/h
23 . 70( .[3 l 70 m
- )m , 30 hec AB = 1 5 cm and AC =
24. 25. 1 3 cm

2 6 1 .584 m 3
• 1 7 days 27 • 28. E = 29.

30. ( 6 yr (ii) Quadratic equation (iii) No (iv) Values associated with true friendshi p
t) ·

Sharing of thoughts and emotions.


(a) True and honest company. (b)
31 . 2.598 km
SAM PLE QU ESTION PAPE R 8
A Sa m p le Qu esti o n Pa per fo r CBSE Class X Sum mative Exa m i n atio n II

MATH E AT CS
General I nstructions
1. A l l q u esti o n s a re co m p u lso ry.

2. T h e q u esti o n p a p e r co nsists of 31 q u esti o n s d ivided i nto 4 Secti o n s A, B, C a n d D.


S e cti o n A co m p rises of 4 q u esti o n s of 1 m a rk each, Section B co m p rises of 6
q u esti o n s of 2 m a rks each, Secti o n c co m p rises of 1 0 q u estions of 3 m a rks each a n d
Secti o n D co m p rises o f 1 1 q u esti o n s of 4 ma rks each.

3. T h e re i s no ove ra ll c h o i ce a n d use of ca lc u lato r is not perm itted .

Time : 3 hours Max. Marhs : 90

Section A 6. In the adjoining figure,


a right angled MBC
1. Find the common root of the equations
circumscribes a circle of
x 2 - 5x + 6 = O and x 2 - 1 0x + 2 1 = 0 . radius r. If AB and B C
2 . In the adjoining figure, AT are o f lengths 8 c m and
is a tangent to the circle 6 cm respectively, then
with centre 0 such that find the value of r.
OT = 4 cm and
7 •
The diameter of a cycle wheel is 2 1 cm.
LOTA = 30°. Find the
length of AT. How many revolutions will it make to
3 . Find the probability of getting a number
travel 1 .98 km?
8. A solid metallic cylinder of base radius
between 1 and 1 00, which is divisible by 1 and
itself only. 3 cm and height 5 cm is melted to form
cones, each of height 1 cm and base
4. Find the volume of the largest right circular cone radius 1 mm. Find the number of cones.
that can be cut-out from a cube of edge 4 cm. 9 . How many spherical lead shots of

Section
diameter 4 cm can be made out of a solid
B cube of lead whose edge measures
44 cm?
5. Fin d the roots o f the quadratic equation given 1 0 . Two cubes each with 6 cm edge are
below by using the quadratic formula joined end to end. Find the surface area
.!. x 2 - .Jllx + 1 = 0 of the resulting cuboid.
2
Section C

�r
1 1 . Solve for x : 1, x - 1
x - 1 5
"*- "*- 1 6. In the given figure, AB is
x + l
-- -- -

\J:LT
- = , x
x - 1 x + l 6 a diameter of the
1 2 . The sum of terms of an AP whose first term
n
circle. The length of
AB = 5 cm. If O is the
is 5 and common difference is 3 6 , is equal to
centre of the circle and A
the sum of 2 n terms of another AP, whose first
the length of tangent
CT.
term is 3 6 and common difference is 5 . Fi n d
AT = 12 cm, then determine
the value of n .
1 7 . The 8th term of an AP is half of its second
1 3 . The centre o f a circle is (2 a, a - 7). Fi nd the
term and the 1 1th term exceeds one-third
value of a, if t he circle passes through the of its 4th term by 1 . Find the 15th term.
point (1 1, - 9) and has diameter 1 o .J2 units .
1 8. Construct a triangle similar to a given
1 4. A box contains 5 red balls , 4 green balls and MBC such that each of its sides is (6 / 7)th
7 white balls . A ball is drawn at random from of the corresponding sides of MBC. It is
the box . Find the probability that the ball given that AB = 5 cm, AC = 6 cm and
drawn is BC = 7 cm.
(i) wh it e (ii) red (iii) green 1 9. A solid cylinder has a total surface area of
1 5 . Find the area of the shaded field shown in 462 sq cm. Its curved surface area is
figure . one-third of the total surface area. Find the
--- 8 m- volume of the cylinder.
20. Which term of the progression 19, 18 .!,
5
17 � .. . . is the first negative term?
.
5

Section D
2 1 . A rectangular container, whose base is a 24. Two circles with radii .a and b touch each
square of side 5 cm, stands on a horizontal other externally. Let c be the radius of a
table and holds water upto 1 cm from the top . circle which touches these two· circles as
When a cube is placed in the water, it is well as a common tangent to the two
completely submerged. The water rises to the circles. Then, prove that
1 1 1
=
top and 2 cu cm of water overflows . Calculate
the volume of the cube and also the length of .JC
+
.jQ Jb'
its edge.
25 . Thro boats approaching a light house in
2 2 . 2 00 logs are stacked such that 20 logs are in mid sea from opposite directions. The
the bottom row, 1 9 in the next row, 1 8 in the angles of elevation of the top of the light
row next to it and so on. house from two boats are 30° and 45°,
(i) I n how many rows are the 200 logs placed respectively. If the distance between two
and how many logs a re in the t op row? boats is 100 m, then find the height of the
(ii) Wh i ch values are depicted in the pattern of light house.
logs? 26. A contractor was awarded to construct a
� 3 . Construct a MBC in which AB = 6 cm,
vertical pillar at a horizontal distance of
100 m from a fixed point. It was decided
AC = 5 . 5 cm and LB = 60°. Draw the
·
that angle of elevation of the top of the
circumcircle of the MB C. Also, write the complete p illar from that point was to
steps of construction . be 60°.
AllinOne MATH EMATICS Class 1 0th Term 11

Contractor finished the job by making a pillar 29. In a game, the entry fee is � 5. The game
such that the angle of elevation of its top was consists of a tossing a coin 3 times. If one
45°. or two heads show, S hweta g e ts her entry ·

(i) Find the height of the pillar to be increased as fee back.


per the terms of contract. If she throws 3 heads, s he receives
(ii) Contractor demands full payment for this double the entry fees . Otherwise, she
work. will lose. For tossing a coin three times,
(a) Is he justified? find the probability that she
(b) Which 'value ' is he lacking? (i) loses the entry fee .
27. In the given figure, AB and CD are two (ii) gets double entry fee .
diameters of a circle perpendicular to each other (iii) j ust gets her entry fee .
and OD is the diameter of the smaller circle. If
OA = 7 cm, then find the area of the shaded
30. If (-4, 3) and (4, 3) a re two vertices of an
region. equilateral t ri ang l e, then find the
coordina tes of the third vertex, given
that the origin lies in the interior of the
triangle.
3 1 . At t min pa s t 2 pm, the time needed by
. the minute hand of a clock to show 3 pm
0 2
was found to be 3 min less than !____ min.
28. Determine the ratio in which the line 4
2x + y 4 = O divides the line segment joining
- Find the value of t.
the points A(2, - 2) and B (3, 7).

Answe rs
1. 3 2. 2 .J3 cm 3.
25
-
4. 16.76 cm3 5. .Jl1 + 3, .Jl1 3
-

98
2
6. r = 2 cm 7. 3000 8. 13500 9. 2541 1 0. 360 cm

(iz)
1 7 '" 1
11. 5, - - 1 2. n = 10 1 3. a = 5, 3 1 4. (1) - 2_ ( I l l ) -
5 16 16 4

1 5. (32 + 2n) m 2 1 6. 1 44 cm 1 7. 3 1 9. 539 cm3 20. 25rh rerm


13
2 1 . 27 cm3 and 3 cm
22. (z) 25 or 1 6
(iz) Values depicred i n che pacrern of logs are space saving, creativicy, reasoning and balancing.
25. 50 (.Jj - 1 ) m
26. (z) 1 00 ( .J3 - 1) m
r ;,)(a) No, he has nocfinished his work according to terms of contract.
(b) Honescy.
27. 1 1 5.5 cm 2 28 . 2 : 9 29. (1) � (ii) � (iii) �
8 8 4
30. ( 3 - ./3)
0, 4 31 . 1 4 min
SAM P L E QU ESTI O N PAPER 9
A S a m p le Q u esti o n Pa p e r fo r C BS E C lass X Su m mative Examination II

MATH EMATICS
G e n e ra l I nstru ctio n s
1 . All q u estio n s a re co m p u lso ry.
2. The q u estio n p a p e r co n sists of 3 1 q uestions d ivided into 4 Sections A, B, C and D.
Sectio n A co m p rises of 4 q u estions of 1 mark each, Section B comprises of 6
q u esti o n s of 2 m a rks each, Sectio n c comprises of 1 0 questions of 3 marks each and
Sectio n D com p rises of 1 1 q u estions of 4 marks each.
3. The re is no overa ll c h o ice a n d use of calcu lator is not permitted.

Time : 3 hours Max. Mcirhs : 90

Section A Section B
1 . What is the common difference of an AP in 5 . The areas of two concentric circles forming a
which T18 - T14 = 32? ring are 154 sq cm and 616 sq cm. Find the

2 . I f two solid hemispheres o f same base radius bread th of the ring.

are joined together along their bases, then 6. The sum of the squares of three consecutive
find the curved surface area of this new solid . positive inte gers is 50. Find the integers.

3 . In the given figure, i f TP and TO are the two 7. The volume of a cuboid is 440 cm 3 and the
area of its base is 88 cm 2 , then find its height.
tangents to a circle with centre 0, so that
LPOQ = 1 40°, then find the value of LPTQ. 8. In the adjoining figure, B
tangents PQ and PR are
T
drawn to a circle such that
LRPQ = 3 0°. A chord RS is
drawn parallel to the
tangent PQ. Find the value Q
of LRQS.

9. Find the volume of a right circular cylinder

4. When we toss a coin, there are two possible which has a height of 21 cm and the base

outcomes head or tail. Is it true th at radius 5 cm.

probability of each outcome is ..!... Justify your 1 0. Find the area of the minor segment of a circle
2 of radius 14 cm, when the angle of the
answer. corresponding sector is 60°.
Section C Section D
1 1 . The first term and last term of an AP are 21 . Solve for x,
1 7 and 350, respectively. If the common
difference is 9 , then how many terms are
�+ --
1
+
3x + 9
= O.
x -3 2x + 3 (x - 3) (2x + 3 )
there in the AP and what is their sum? _
1 2 a bx 2 - (9 a2 - 8 b2 ) x - 6 a b = O .
22. A solid is hemispherical at the bottom and
1 2. Solve for x:
conical above. I f the surface ar as of two
1 3. The line joining the points (2 , 1 ) and is (5, - 8) parts are equal, then find the ratio of the
trisected at the points P and Q. If point P lies radius and the height of the conical part.
on the line 2x - y + k = 0, then find the value
23. A right circular cone is 4 . 1 cm high and the
of k. radius of its base is 2 . 1 cm. Another right
1 4. Find the area of the shaded region in the circular cone is 4 . 3 cm high and th radius of
figure given below, where arcs drawn from the base is 2 . 1 cm. Both the cones are melted
centres A, B, C and D intersect in pairs at and recast into a sphere. Find the dia meter of
mid-points P, Q, R and S of the sides the sphere.
AB, BC, CD and DA, respectively of a square 24. In the given figure, the diameters o f two

:�:J2 cm
ABCD. [take, 1t = 3 .14] wheels have measured 2 cm and 4 cm.
Determine the lengths of the belts AD and
BC that pass around the wheels, if it is given
that belts cross each other at right angles.
B

D R Cl
1 5. Construct an isosceles triangle whose base
is 8 cm and altitude 4 cm and then another
25. A die is thrown once. Find the probability
triangle whose sides are 1 ]: time$ the that you will get
2
(i) a prime number.
(ii) a number between 3 and 6 .
corresponding sides of the isosceles
triangle.
(iii) even number. .
1 6. 1 000 tickets of a lottery were sold and there (iv) a number which is multiple of both
are 5 prizes on these tickets. If John has 2 and 3 .
purchased one lottery ticket, then what is the 26. I n a cyclic quadrilateral ABCD, the diagonal
probability of winning a prize? AC bisects LC. Prove that the diagonal BD is
1 7 . A circle touches the side BC of a MBC at a parallel to the tangent at A to the circle
point P and touches AB and AC, when passing through the points A, B , C and D.
produced at Q and R, respectively. Show that 27. Shyam gets pocket money from his father
AQ = ]: (Perimeter of MBC). everyday. Out of the pocket money, he saves
2 -� 2 . 7 5 on first day and on each succeeding
1 8. A classroom is 7 m long, 6 . 5 m wide and 4 m day, he increases his saving by 25 paise.
high. It has one door of measures 3 m x 1 .4m (i) (a) Find the amount saved by Shyam on
and three windows each measuring 2 m x lm. 14th day.
The interior walls are to be white-washed. (b) Find the amount saved by Shyam on
The contractor charges � 5 .2 5 per m 2 . Find 25th day.
(c) Find the total amount saved by
Shyam in 30 days .
the cost of white-washing.
1 9 . If Riya had walked 1 km/h faster, she would
have taken half an hour less to walk 6 km .
(ii) At the end of every month, Shyam
purchases some books, pens and
Find the spged at which she was walking. notebooks from the amount that he save d
and distribute these items to the needy
20. The sum of first 8 terms of an AP is 140 and
students in his class. What values are
sum of first 24 terms is 996. Find the AP. depicted by this act of Sh yarn?
Sample Question Paper 9 f 441 1
28. S uppose there are two wi ndows i n a house . (iv) If the balloon is moving towards the
A window of the h ouse is at a height of building, then will the angles of elevation
1 . 5 m above the g round and the other remains same?
window is 3 m vertically above the lower 2 9 . I f {a ,b) is th e m id - p oint of the line segment
j o in i n g th e p oints A (1 0, - 6), B (k, 4) and

-
window. Anil and Sanj eev are sitting inside
the two windows . At an instant, the angles a - 2 b = 1 8, then find the value of k and the
of elevation of a balloon from these d is ta n c e AB .
windows are obs e rved as 4 5° and 3 0°,
30 . If (-3, 2), (1, 2) and (5, 6) are the mid-points of
respectively.
the side of a triangle, then find the coordinates
(i) Find the h e ig h t o f the balloon from the
ground . of th e vertices of the triangle.
(ii) Among Anil a n d S a nj eev, who i s more 3 1 . The angle of elevation of the top of a
clos e r to the balloon? unfinished tower at a point of distance 120 m
(iii) Which valu e s a re added by windows in from its base is 45°. How much height must the
any construction commercial or tower be raised, so that the angle of elevation
res i dential? becomes 60°?

Ans we rs

(� J
1. 8 2. 4nr 2 3. 40° 4. Yes 5. 7 cm
6. 3, 4, 5 7. S cm 8. 30° 9. 1650 cm3 1 O. 3 8 - 49Jj cm '

11. 38,6973 2b 3 a
x = - -, - = 1 4. 30. 96 cm2 0.005

-31
1 2.
3 a 4b
1 3. k -8 1 6.

1 8. � 5 13 . 45 1 9. 3km/h 7, 1 0, 1 3, 16, .
20. . x = -1
. 21 . .J3 22. 1 :

2 5 . (t) - ( Ill)
23. 4 .2 cm 24. 3 cm each
1 (ii) (iv) .!_
"

-
I
2 6 2
27. (1) (a) ' 6 (b) ' 8.75 (c) ' 1 9 1 .2 5 (ii)
Concerns for others, helping and caring nature.

28. (1) 4.0 98 m (i1) Sanjeev


(iii) Windows are the most important pare of any u il d i n g . They add different values
b ro i r,
like they are useful for the proper
ventilation, which is very much required as natural air keeps the building fresh and suffocarion free.
(iv) No, when the balloon is moving cowards the b u i ldin g then the angle of
, elevarion will
automatically increase.
29. k = 22, AB = 2.J61 units 30. (9, 2) , ( I , I O) , (-7, -6) 87.84 m 31 .
SAM P LE QU ESTION PAPER 1 0
A S a m p le Questio n Paper for CBSE Class X Su m rilative Exa m i nati o n II

MATH E AT CS
General I nstructions
1. All questions are compulsory.
2. The q uestion paper consists of 31 questions divided into 4 Sections A, B, C and D.
Section A comprises of 4 questions of 1 mark each, Section B comprises of 6
questions of 2 marks each, Section C comprises of 10 questions of 3 marks each and
Section D comprises of 1 1 questions of 4 marks each.
3. There is no overall choice and use of calculator is not permitted.

Time : 3 hours Max. Marks : 90

Section A 6. Find the coordinates of a point A, where AB is


1. Find the perpendicular distance of the the diameter of a circle whose centre is (2, -3)
point A (4 , 2) from the X-axis. and the point B is ( 1 , 4 ) .
2. In a throw o f a pair o f dice, what i s the 1. The 8th term of an AP is 37 and its 1 2th term
probability of getting a doublet? is 57 . Find the AP.
3 . PAO is a tangent to the p
c 8. The sum of two numbers is 9 and the sum of
circle with centre 0 at a their reciprocals is ! . Find the numbers.
point A as shown in figure. A �-..... 2
If LOBA = 35°, then find 9 . Flowers are to be planted in the shaded
the values of LBAQ and 0 portion which is shown by sectors of two
LACB. B concentric circles of radii 7 m and 3 .5 m.
Find the area of the shaded region.
4 . Write the perimeter of a quadrant of a circle [take, 7t = 22 / 7]
of radius r.

Section B

rT\
I n the adjoining figure, 0 is the


5.
centre of the circle with radius 1 0 . Which term of the AP 92 : 88 , 84, 80, 76, . . .
5 cm, AB 1 1 CD and AB = 6 cm.
is O?
Find OP.
Section C
1 1. Wa t e r flows o u t t h ro u g h a circular pipe wh ose 1 7. · In the given figu re, the boundary of
interna l rad i u s is 1 c m , at the ra te of 80 emfs shaded region consist of four
·

into an e m pty cyl i n d rica l ta n k , the ra dius of s emi-circular arcs, two smallest arcs being

[
whose b a s e is 4 0 c m . By how much will the equal. I f diameter of the largest arc is

J
level of water rise in the la n k i n half a n ho u r? 1 4 cm and that of the smallest arc is 3.5
1 2. Which term of the A P : 1 2 1 , 1 1 7 , 1 1 3 , . . . is its first
cm, then calculate the area of the shaded
ir 22
negative term? region. take, =
7
1 3. If PAB is a seca n t to a c i rcle intersecting the
circle at A, B a nd PT is a ta n g e n t , then p rove
that PA x PB = PT 2 •

1 4. Cards m a rked w i t h numbers 5 to 5 0 , are


placed in a b ox a n d m ixed thoroug hly. A card
is d rawn from t h e box a t ra ndom. Find the
p r o b a b i l ity that t h e number on the taken out
1 8. In right angled MBC, if LB = 90°,
card is
AB = 1 2 cm and BC = 16 cm, · then find the
(i) a prime number less tha n 1 0 .
radius of t he circle inscribed in the MBC.
(ii) a n u mber wh ich i s a perfect square .
S o l v e fo r x , 9 x + 2 - 6 · 3 x + t + 1 = 0.
1 9. Find the sum of all two-digit odd positive
1 5.
numbers .
1 6. I n the adj oining fi g u r e , a rcs A
20. A t en t is in the shape of a right circular
a re d raw n by taking vertices
cylinder upto a height of 3 m and co nical
A, B and C of an equilateral above it. The total height of the tent is
triangle of s i d e 1 0 cm to 1 3 .5 m and radius of base is 1 4 m. Find the
intersect the sides B C, CA and cost of cloth required to make the tent at the
AB at the i r res p ec t i ve 8 0 c rate of � 80 per sq m.
mid -points D, E and F. Find
the a rea of the s haded region. [ t a ke , 7t = 3 . 1 4)

Sect ion D
21 . I n the following figure, OP is equal to diameter 24. The 6th term of an AP is zero, prove that its
of the circle. Prove that MBP is an equilateral 2 1 st term is triple its 1 1th term.
triangle . 25 . In what ratio does the line x -y -2 =0
divide the line segment joining the points
(3, - 1) and (8, 9) ?

26 . Three consecutive vertices of a


parallelogram are (-2 , - 1) , (1 , 0) and (4 , 3) .
Find the coordinates of the fourth vertex.
22 . I f p times the pth term of an AP is eq ual to
(2 p + 1) times the (2 p + l)th term, then show that 27 . A conical tank is full of water. Its base
its (3 p + 1 ) t h term is zero . radius is 1 . 75 m and height 2 .25 m. It is
connected with a pipe which empties it at
23. Construct a circle of radius 2 cm. Take two
the rate of 7 L I s. How much time will it
points A and B on one of its extended diameter
take to empty the tank c om pletely ?
each at a distance of 5 cm from its centre.
Construct tangent to the circle from these two
points A and B .
AllinOne MATHEMATICS C l a ss 1 0th Te rm 1 1

28. An aeroplane when flying at a height of 30. A d e c o rative block which i s made of two s olids, a

]
cube an d a hemisphere. The base of the b l o c k is a

[
3 125 m from the ground passes
vertically below another plane at an cube with edge 5 cm and the hemisphere, fixed
instant when the angles of elevation of on the top, has a diameter of 4 . 2 c m . Find th total
rr
=
the two planes from the same point on 22
surface area of the block. take,
the ground are 3 0 ° and 60°, 7
respectively. Find the distance
between the two planes at that instant. 3 1 . Venkatesh Prashad was a referee of cricket test
match between India and Pakistan whi h was
29. A straight highway leads to the foot of a held on 20 August, 2 0 1 0 . He was having two
tower. A watchman standing at the top
Q
types of coins, which are shown below
of the tower observes a car at an angle
of d epression of 3 0 ° which is (a) A fair coin
approaching the foot of the tower with Type A

a uniform speed. Two minutes later, the


angle of depression was found to be (b) Q An unfair coin, having head on both
60°. The watchman suspects that some Type a the sides
terrorist are a pp roaching the tower. It
needs half a minute for the watchman He decid � d to toss with type B coin.
to inform the security staff to be on the (i) Find the chance of getting a head on type A
alert. coin .
(i) How much time the car will take to (ii) Which type of coin had more chance of getting
reach the foot of the tower? head?
(ii) Will the security staff be able to save (iii) As he decided to use the coin of type B, which
the tower from terrorists? value is shown by referee?

Answers
4. !..... (7t + 4)
1
1 . 2 units 2. - 3. 55°, 55° 5. 4 cm
6 2
6. (3, - I )
O 7. 2, 7, 1 2, 1 7, . . . 8 . 3, 6 9. 9.625 m 2 1 0. 24

1 4. (i) __!__ x =
5
1 1 . 90 cm 1 2. 3 2 n d term ( ll' " ) - 1 5. -2 1 6. 39.25 cm 2
23 46
1 7 . 72.2 cm 2 1 8. 4 cm 1 9. 2475 20. � 82720 2 5. 2 : 3

26. ( 1 , 2) 27. 1 7 1_ min 28. 6250 m 29. (i) I min (ii) Yes
16
30. 1 63.86 cm 2 31 . (i) _!_ (ii) Type B (iii) Dishonesty
2
ii� g
l'-
't'"""

>,°c arihant
� 295.00 Arihant Pra kashan, Meerut

You might also like